Anda di halaman 1dari 640

11

YEAR

MATHS
Quest

MATHS C
FOR QUEENSLAND
D EDITION
SECON

TEACHER EDITION
11
YEAR

MATHS
Quest

MATHS C
FOR QUEENSLAND
D E
D T INO N
T II O
DEI D
N N
S ESCEOC O

TEAC HE R EDI TI ON
Nick Simpson
Catherine Smith
Peter Posetti
Sue Campbell

CONTRIBUTING AUTHOR
Robert Rowlan
Second edition published 2009 by
John Wiley & Sons Australia, Ltd
42 McDougall Street, Milton, Qld 4064

First edition published 2001

Typeset in 10.5/12.5pt Times

© John Wiley & Sons Australia, Ltd 2001, 2009

The moral rights of the authors have been asserted.

National Library of Australia


Cataloguing-in-Publication data

Author: Simpson, N. P. (Nicholas Patrick), 1957–


Title: Maths Quest: Maths C Year 11 for Maths Quest: Maths C Year 11 for Queensland,
Queensland/Simpson, Smith and Posetti. Teacher edition/Simpson, Smith and Posetti.
Edition: 2nd ed.
ISBN: 978 0 7314 0814 6 (pbk.) 978 0 7314 0831 3 (pbk.)
978 0 7314 0868 9 (web.) 978 0 7314 0866 5 (pdf)
Notes: Includes index.
Target Audience: For secondary school age.
Subjects: Mathematics — Textbooks.
Other Authors/
Contributors: Smith, Catherine, 1969–
Posetti, Peter.
Dewey Number: 510

Reproduction and communication for educational purposes


The Australian Copyright Act 1968 (the Act) allows a maximum
of one chapter or 10% of the pages of this work, whichever is the
greater, to be reproduced and/or communicated by any educational
institution for its educational purposes provided that the educational
institution (or the body that administers it) has given a remuneration
notice to Copyright Agency Limited (CAL).

Reproduction and communication for other purposes


Except as permitted under the Act (for example, a fair dealing
for the purposes of study, research, criticism or review), no part
of this book may be reproduced, stored in a retrieval system,
communicated or transmitted in any form or by any means without
prior written permission. All inquiries should be made to the
publisher.

Illustrated by Paul Lennon, Liz Sawyer and the Wiley Art Studio

Cover photograph and internal design images:


© Digital Vision

Printed in Singapore by
Craft Print International Ltd
10 9 8 7 6 5 4 3 2 1
Contents
Introduction ix Investigation — Approximations for p 63
Investigation — Real numbers — application
About eBookPLUS xi
and modelling 64
Acknowledgements xii Summary 66
Chapter review 69
CHAPTER 1
 Number systems: the Real CHAPTER 2
Number System 1  Number systems: complex
Introduction 2 numbers 75
The Real Number System 3 Introduction to complex numbers 76
Classification of numbers: rational and Exercise 2A 79
irrational 3 Investigation — Complex numbers in
Exercise 1A 9 quadratic equations 80
Recurring decimals 11 Basic operations using complex numbers 80
Exercise 1B 14 Investigation — Plotting complex
Investigation — Real number numbers 84
investigations 15 Exercise 2B 86
Investigation — Other number systems 16 Conjugates and division of complex
Surds 17 numbers 87
Exercise 1C 20 Exercise 2C 91
Simplifying surds 21 Radians and coterminal angles 95
Exercise 1D 22 Exercise 2D 96
Addition and subtraction of surds 24 Complex numbers in polar form 96
Exercise 1E 25 History of mathematics — Abraham de
Moivre 108
Multiplication of surds 27
Exercise 2E 109
Exercise 1F 30
Basic operations on complex numbers in
The Distributive Law 32
polar form 111
Exercise 1G 35
Investigation — Multiplication in polar
Division of surds 36 form 111
Exercise 1H 38 History of mathematics — William Rowan
Rationalising denominators 40 Hamilton 118
Exercise 1I 42 Exercise 2F 119
Rationalising denominators using conjugate Investigation — Complex numbers:
surds 44 applications 120
Exercise 1J 48 Summary 122
Further properties of real numbers — Chapter review 123
modulus 49
Exercise 1K 50
Solving equations using absolute
CHAPTER 3
values 51  Matrices 127
Exercise 1L 54 Introduction to matrices 128
Solving inequations 55 Operations with matrices 130
Exercise 1M 62 Exercise 3A 135
vi
Multiplying matrices 137 Investigation — Some applications of group
Exercise 3B 140 theory 197
History of mathematics — Olga Taussky- History of mathematics —
Todd 142 Cryptography 199
Powers of a matrix 143 Summary 201
Investigation — Matrix powers 143 Chapter review 202
Exercise 3C 144
Investigation — Applications of matrices 145
Multiplicative inverse and solving matrix CHAPTER 5
equations 146  Matrices and their
Exercise 3D 152 applications 205
The transpose of a matrix 154 Inverse matrices and systems of linear
Exercise 3E 154 equations 206
Applications of matrices 155 Exercise 5A 208
Exercise 3F 160 Gaussian elimination 209
Investigation — Matrix multiplication using a Exercise 5B 215
graphics calculator 162 History of mathematics — Carl Friedrich
Dominance matrices 164 Gauss 216
Investigation — Dominance matrices — Investigation — Performing Gaussian
another application of matrices 165 elimination using a graphics
Exercise 3G 169 calculator 217
Summary 170 Introducing determinants 222
Chapter review 172 Exercise 5C 224
Properties of determinants 224
Exercise 5D 227
CHAPTER 4 Inverse of a 3 ¥ 3 matrix 228
 An introduction to Exercise 5E 232
groups 177 Cramer’s Rule for solving linear
Introduction 178 equations 234
Investigation — Algebraic structures 178 Exercise 5F 237
Modulo arithmetic 179 Investigation — Solving simultaneous
equations 242
Exercise 4A 180
Investigation — Applications of
The terminology of groups 180
determinants 243
History of mathematics — Niels Henrik
Summary 244
Abel 183
Chapter review 246
Exercise 4B 184
Properties of groups 184
Exercise 4C 188
CHAPTER 6
Cyclic groups and subgroups 189
Exercise 4D 191
 Transformations using
Investigation — Application of groups — matrices 249
permutations 191 Geometric transformations and matrix
Further examples of groups — algebra 250
transformations 192 Exercise 6A 258
History of mathematics — Arthur Cayley 194 Linear transformations 259
Exercise 4E 195 Exercise 6B 262
vii
Linear transformations and group History of mathematics — Sir Isaac
theory 263 Newton 363
Exercise 6C 269 Newton’s First Law of Motion 364
Rotations 270 Exercise 8B 371
Exercise 6D 275 Momentum 374
Reflections 276 Investigation — Conservation of momentum
Exercise 6E 283 using i and j notation 378
Dilations 284 ˜ 8C ˜ 379
Exercise
History of mathematics — Maurits Cornelius Investigation — Collision momentum 381
Escher 290 Relative velocity 382
Exercise 6F 291
Exercise 8D 384
Shears 291
Using vectors in geometry 385
Exercise 6G 295
Investigation — Three-dimensional non-zero
Investigation — Transformations 295
vectors 387
Summary 296
Investigation — Vector geometry 388
Chapter review 297
Exercise 8E 388
Summary 390
CHAPTER 7 Chapter review 391
 Introduction to vectors 299
Vectors and scalars 300
CHAPTER 9
Exercise 7A 305
Position vectors in two and three
 Sequences and series 395
dimensions 308 Introduction 396
Exercise 7B 320 Arithmetic sequences 396
Multiplying two vectors — the dot Exercise 9A 404
product 324 Geometric sequences 406
Exercise 7C 329 Exercise 9B 414
History of mathematics — Charles Lutwidge Applications of geometric sequences 418
Dodgson 331
Exercise 9C 424
Resolving vectors — scalar and vector
resolutes 332 Finding the sum of an infinite geometric
sequence 427
Exercise 7D 337
Investigation — Vectors and matrices 337 Exercise 9D 431
Time-varying vectors 339 Contrasting arithmetic and geometric
Exercise 7E 344 sequences through graphs 432
Summary 346 Exercise 9E 438
Chapter review 348 Investigation — Reward time 440
Investigation — Changing shape 441
Fibonacci Sequence 442
CHAPTER 8 Investigation — Fibonacci numbers 445
 Vector applications 353 The Mandelbrot Set 446
Introduction 354 Investigation — Draw the Mandelbrot
Force diagrams and the triangle of Set 449
forces 354 Summary 450
Exercise 8A 361 Chapter review 453
viii
CHAPTER 10 History of mathematics — Blaise
Pascal 506
 Permutations and Summary 507
combinations 459 Chapter review 509
Introduction 460
The addition and multiplication
principles 460 CHAPTER 11
Exercise 10A 465  Dynamics 513
Factorials and permutations 467 Displacement, velocity and
Exercise 10B 474 acceleration 514
Arrangements involving restrictions and Exercise 11A 521
like objects 476 Projectile motion 524
Exercise 10C 480 Exercise 11B 536
Combinations 482 Motion under constant acceleration 540
Exercise 10D 488 Exercise 11C 544
Applications of permutations and Summary 547
combinations 490 Chapter review 548
Exercise 10E 495
Pascal’s triangle, the binomial theorem and Appendix 553
the pigeonhole principle 497
Investigation — Counting paths 498 Answers 591
Exercise 10F 504 Index 621
Introduction
Maths Quest Maths C Year 11 for Queensland 2nd edition is one of the
exciting Maths Quest resources specifically designed for the Queensland
senior Mathematics syllabuses beginning in 2009. It has been written and com-
piled by practising Queensland Maths C teachers. It breaks new ground in
Mathematics textbook publishing.
This resource contains:
• a student textbook with accompanying student website (eBookPLUS)
• a teacher edition with accompanying teacher website (eGuidePLUS)
• a solutions manual containing fully worked solutions to all questions con-
tained in the student textbook.

Student textbook
Full colour is used throughout to produce clearer graphs and headings, to pro-
vide bright, stimulating photos and to make navigation through the text easier.
Clear, concise theory sections contain worked examples, graphics calculator
tips and highlighted important text and remember boxes.
Worked examples in a Think/Write format provide clear explanation of key
steps and suggest how solutions can be presented.
Exercises contain many carefully graded skills and application problems,
including multiple-choice questions. Cross-references to relevant worked
examples appear beside the first ‘matching’ question throughout the exercises.
Investigations, often suggesting the use of technology, provide further dis-
covery learning opportunities.
Each chapter concludes with a summary and chapter review exercise con-
taining questions that help consolidate students’ learning of new concepts.
As part of the chapter review, there is also a Modelling and problem solving
section. This provides students with further opportunities to practise their
skills.
Technology is fully integrated within the resource. To support the use of
graphics calculators, instructions for two models of calculator are presented in
worked examples and graphics calculator tips throughout the text. The two
models of graphics calculator featured are the Casio fx-9860G AU and the
TI-Nspire CAS. (Note that the screen shots shown in this text for the TI-Nspire
CAS calculator were produced using OS1.4. Screen displays may vary
depending on the operating system in use.)
For those students using the TI-89 model of graphics calculator, an appendix
containing matching instructions has been included at the back of the book.
The Maths Quest for Queensland series also features the use of spreadsheets
with supporting Excel files supplied on the student website. Demonstration
versions of several graphing packages and geometry software can also be
downloaded via the student website.
x
Student website — eBookPLUS
The accompanying student website contains an electronic version of the entire
student textbook plus the following additional learning resources:
WorkSHEETs — editable Word 97 documents that may be completed on
screen, or printed and completed later.
SkillSHEETs — printable pages that contain additional examples and problems
designed to help students revise required concepts.
Test yourself activities — multiple-choice quizzes for students to test their
skills after completing each chapter.

Programs included
Graphmatica: an excellent graphing utility
Equation grapher and regression analyser: like a graphics calculator for PCs
GrafEq: graphs any relation, including complicated inequalities
Poly: for visualising 3D polyhedra and their nets

Teacher edition
The teacher edition textbook contains everything in the student textbook and
more. To support teachers assisting students in the class, answers appear in red
next to most questions in the exercises and investigations. Each chapter is
annotated with relevant syllabus information.

Teacher website — eGuidePLUS


The accompanying teacher website contains everything in the student website
plus the following resources:
• two tests per chapter (with fully worked solutions)
• fully worked solutions to WorkSHEETs
• a syllabus planning document
• assessment tasks (and answers)
• fully worked solutions to all questions in the student textbook.

Solutions manual
Maths Quest Maths C Year 11 for Queensland Solutions Manual contains the
fully worked solutions to every question and investigation in the Maths Quest
Maths C Year 11 for Queensland 2nd edition student textbook.
Fully worked solutions are available for all titles in the Maths Quest for
Queensland senior series.
Maths Quest is a rich collection of teaching and learning resources within one
package.
xi

Next generation teaching and learning

About eBookPLUS
Maths Quest Maths C Year 11 for Queensland 2nd edition features eBookPLUS: an electronic version of the
entire textbook and supporting multimedia resources. It is available for you online at the JacarandaPLUS
website (www.jacplus.com.au).

Using the JacarandaPLUS website Step 1. Create a user account


To access your eBookPLUS resources, simply log on The first time you use the JacarandaPLUS system,
to www.jacplus.com.au. There are three easy steps for you will need to create a user account. Go to the
using the JacarandaPLUS system. JacarandaPLUS home page (www.jacplus.com.au)
and follow the instructions on screen.

Step 2. Enter your registration code


Once you have created a new
account and logged in, you will
be prompted to enter your unique
registration code for this book,
which is printed on the inside
front cover of your textbook.

LOGIN
Once you have created your account,
you can use the same email address and
password in the future to register any
JacarandaPLUS books.

Step 3. View or download eBookPLUS resources


Your eBook and supporting resources are provided
in a chapter-by-chapter format. Simply select the
desired chapter from the drop-down list and navigate
through the tabs to locate the appropriate resource.

Minimum requirements Troubleshooting


• Internet Explorer 7, Mozilla Firefox 1.5 • Go to the JacarandaPLUS help page at
or Safari 1.3 www.jacplus.com.au
• Adobe Flash Player 9 • Contact John Wiley & Sons Australia, Ltd.
• Javascript must be enabled (most browsers Email: support@jacplus.com.au
are enabled by default). Phone: 1800 JAC PLUS (1800 522 7587)
Acknowledgements
The authors and publisher would like to thank the following copyright holders,
organisations and individuals for their assistance and for permission to
reproduce copyright material in this book.

Illustrative material
Screenshots:
TI–Nspire CAS and TI89 screenshots reproduced with permission of Texas Instruments;
• Casio fx-9860G AU screenshots reproduced with permission of Casio.

Images:
• © Author’s Image, p. 540; • © Corbis, pp. 2 (bottom)/Matthias Kulka/zefa, 118/
Hulton-Deutsch Collection, 183/Bettmann, 194/Bettmann; • © Corbis Corporation,
pp. 169, 299, 320, 456, 495 (flag), 550 (middle); • © Digital Stock/ Corbis Corporation,
pp. 206, 307, 406, 425, 498, 544; • © Digital Vision, pp. 75/Jaroslav, 177/Jaroslav, 200
(top left), 202/Jaroslav, 205, 276, 352, 482; • © Emerald City Images p. 161/John
Carnemolla; • © Flat Earth, p. 192; • © Getty Images, p. 130/Photonica/Loungepark,
300/Allsport/Nick Wilson; • © imageaddict.com.au, p. 383; • © Image Disk Photog-
Dot points and Suggested Learning raphy, pp. 444 (left), 455 (bottom); • © Creative Cohesions, p. 446; • © John Wiley &
Experiences (SLEs) appearing Sons Australia, pp. 259/Kari-Ann Tapp, 368/Jennifer Wright, 455 (top); • © Photodisc,
throughout the textbook (overprinted in p. 1, 2 (top), 16, 17, 74, 127, 141, 145 (bottom, top), 146, 164, 174 (bottom), 175, 178,
red) have been taken from the 200 (middle right, top right), 249, 250 (top), 270, 284, 322, 337, 350, 353, 354, 355
Mathematics C Senior Syllabus (2008), (right, left), 365, 366, 372, 380, 405 (jelly beans), 444 (right), 457, 459, 460, 466, 481,
reproduced with permission from the 495 (roulette wheel), 496, 503, 509, 510, 512, 513, 546, 550 (bottom), 552;
Queensland Studies Authority, • © Photolibrary, p. 89/SPL/Gregory Sams, 216/Photo Researchers/Photo Inc, 323/Bay
www.qsa.qld.edu.au.
Hippisley, 331/Science Photo Library, 363/Science Photo Library, 393/Vince Cavataio,
395/Alfred Pasieka, 405 (batsman)/Tommy Hindley, 506/Sydney/ SPL; • © Purestock,
p. 174 (top); • © Stockbyte, p. 416 (eight images), 463.

Software
The authors and publisher would like to thank the following software providers
for their assistance and for permission to use their materials. However, the use
of such material does not imply that the providers endorse this product in any
way.
Third party software — registered full version ordering information
Full versions of third party software may be obtained by contacting the companies
listed below.

GraphEq and Poly


Evaluation copies of GraphEq and Poly have been included with permission from
Pedagoguery Software, Inc.
email: peda@peda.com
Web: www.peda.com

Graphmatica
Reproduced with permission of kSoft, Inc.
345 Montecillo Dr., Walnut Creek, CA 94595-2654.
email: ksoft@graphmatica.com
Web: www.graphmatica.com
xiii
Software included is for evaluation purposes only. The user is expected to register
share-ware if use exceeds 30 days. Order forms are available at www.graphmatica.com/
register.txt

Equation Grapher with Regression Analyser


Reproduced with permission of MFSoft International.
email: info@mfsoft.com
Web: www.mfsoft.com

Microsoft Excel
Screen shots reproduced by permission of Microsoft Corporation.
Note: Microsoft Software has been used only in screen dumps.
Microsoft Excel is a registered trademark of the Microsoft Corporation in the United
States and/or other countries.

Every effort has been made to trace the ownership of copyright material. Infor-
mation that will enable the publisher to trace the copyright holders or to rectify
any error or omission in subsequent reprints will be welcome. In such cases,
please contact the Permission Section of John Wiley & Sons Australia, who
will arrange for the payment of the usual fee.
5_61_08144_MQ11C2E_Prelim Page xiv Monday, November 10, 2008 1:51 PM
Number
systems: the
Real Number
System 1
syllabus reference
Core topic:
Real and complex number
systems

In this chapter
1A Classification of numbers
1B Recurring decimals
1C Surds
1D Simplifying surds
1E Addition and subtraction of
surds
1F Multiplication of surds
1G The Distributive Law
1H Division of surds
1I Rationalising denominators
1J Rationalising denominators
using conjugate surds
1K Further properties of real
numbers — modulus
1L Solving equations using
absolute values
1M Solving inequations
2 M a t h s Q u e s t M a t h s C Ye a r 1 1 f o r Q u e e n s l a n d
• structure of the real number system including rational numbers and irrational numbers
• simple manipulation of surds

Introduction
The number systems used today evolved from a basic and practical need of primitive
people to count and measure magnitudes and quantities such as livestock, people,
possessions, time and so on.
Early cultures and societies used their body parts, such as
fingers and toes, as a basis for their numeration systems. As
the need for larger numbers grew, symbols were developed to
represent them. Ancient Egyptians, for example, used the
symbol of the lotus flower to represent the number 1000, and
Romans used the letter M to represent 1000. Roman numerals
can be seen today on some clock and watch faces. At the end
of movie credits Roman numerals are often used to indicate
the year in which the movie was made. For example,
MCMXCIX represents the year 1999 and MMIX represents
the year 2009.
As societies grew and architecture and engineering
developed, number systems became more sophisti-
cated. Number use developed from solely whole
numbers to fractions, decimals and irrational
numbers.
We shall explore these different types of
numbers and classify them into their specific
groups.
Consider solutions to equations such as:
2x = 10, 3x = 15, 20x = 100
What do they have in common? Each of
the statements is true for a whole-number
value of x.
This type of equation represents many
real-life situations; for example, how
many people will I need to collect $2
from to cover the cost of hiring a $10
game?
The first types of numbers to
evolve were the whole numbers. As
you work through this chapter on
the Real Number System and
Chapter 2 you will be introduced
to types of numbers that
evolved to fill other, more
sophisticated needs.
Chapter 1 Number systems: the Real Number System 3
The Real Number System
The Real Number System contains the set of rational and irrational numbers. It is denoted
by the symbol R.
Real numbers R

Irrational numbers I Rational numbers Q


(surds, non-terminating
and non-recurring
decimals, π ,e) Integers Non-integer rationals
Z (terminating and
recurring decimals)

Negative Zero Positive


Z– (neither positive Z+
nor negative) (Natural
numbers N)

The set of real numbers contains a number of subsets which can be classified as shown
in the chart above.

Classification of numbers: rational


and irrational
Rational numbers (Q)
A rational number (ratio-nal) is a number which can be expressed as a ratio of two
a
integers in the form --- where b ≠ 0 and a and b have no common factors.
b
Rational numbers are given the symbol Q. Examples are:
--1- , --2- ,
3-
----- , --9- , 7
5 7 10 4
Rational numbers may be expressed as terminating decimals. Examples are:
7-
----- = 0.7, 1
--- = 0.25, 5
--- = 0.625, 9
--- = 1.8
10 4 8 5
These decimal numbers terminate after a specific number of digits.
Rational numbers may be expressed as recurring decimals (non-terminating or
periodic decimals). For example:
1
.
--- = 0.333 333 . . . or 0.3
3
9
..
------ = 0.818 181 . . . or 0.8 1
11
.
--5- = 0.833 333 . . . or 0.83
6
3
. .
------ = 0.230 769 230 769 . . . or 0.2 30769
13

These decimals do not terminate, and the specific –3.743 3 1–


–2 –4 2 1.63 3.6
digit (or number of digits) is repeated in a pattern.
Recurring decimals are represented by placing a dot
or overscore above the repeating digit or pattern. –4 –3 –2 –1 0 1 2 3 4Q
4 M a t h s Q u e s t M a t h s C Ye a r 1 1 f o r Q u e e n s l a n d

Rational numbers are defined in set notation as: Q = set of rational numbers
a
{ }
Q = --- , a, b ∈ Z, b ≠ 0, g.c.d (a, b) = 1 where ∈ means ‘an element of’ and
b
g.c.d. (a, b) = 1 means greatest common divisor of (a, b) = 1.
Rational numbers may be represented on the number line (as illustrated on page 3)
and include whole numbers, fractions, and terminating and recurring decimals.
Whole numbers form a set of integers (which is a subset of the set of rational
numbers).

Integers (Z)
The set of integers consists of positive and negative whole numbers, and 0 (which is
neither positive nor negative). They are denoted by the letter Z and can be further
divided into subsets. That is:
Z = {. . . −3, −2, −1, 0, 1, 2, 3, . . .}
Z + = {1, 2, 3, 4, 5, 6, . . .}
Z − = {−1, −2, −3, −4, −5, −6 . . .}
Positive integers are also known as natural numbers (or counting numbers) and are
denoted by the letter N. That is:
N = {1, 2, 3, 4, 5, 6, . . .}
Integers may be represented on the number line as illustrated below.

–3 –2 –1 0 1 2 3 Z 1 2 3 4 5 6 N Z – –6 –5 –4 –3 –2 –1
The set of integers The set of positive integers The set of negative integers
or natural numbers
Note: Integers on the number line are marked with a solid dot to indicate that they are
the only points in which we are interested.

Irrational numbers (I)


Numbers that cannot be expressed as a ratio between two integers are called irrational
numbers. Irrational numbers are denoted by the letter I. Numbers such as surds (for
example 7 , 10 ), decimals that neither terminate nor recur, and π and e are
examples of irrational numbers. The numbers π and e are examples of transcendental
numbers; these will be discussed briefly later in this chapter.
Irrational numbers may also be represented on the number line with the aid of a ruler
and compass.
An irrational number (ir-ratio-nal) is a number which cannot be expressed as a
ratio of two integers in the form --a- where b ≠ 0.
b
Irrational numbers are given the symbol I. Examples are:
7
13 , 5 21 , ------- , π, e
7,
9
Irrational numbers may be expressed as decimals. For example:
5 = 2.236 067 977 5 . . . 0.03 = 0.173 205 080 757 . . .
18 = 4.242 640 687 12 . . . 2 7 = 5.291 502 622 13 . . .
π = 3.141 592 653 59 . . . e = 2.718 281 828 46 . . .
These decimals do not terminate, and the digits do not repeat themselves in any
particular pattern or order (that is, they are non-terminating and non-recurring).
Chapter 1 Number systems: the Real Number System 5
Once the decimal approximation for an irrational number is obtained, it can be
shown on a number line. For example, 18 ≈ 4.24. This approximation is between 4
and 5, but closer to 4.
–2 7 – 5 0.03 e 18

–6 –5 –4 –3 –2 –1 0 1 2 3 4 5
Irrational numbers in surd form can also be represented on the number line exactly,
as follows.
Consider an isosceles right-angled triangle of side length 1 unit.
By Pythagoras’ Theorem, (OB)2 = (OA)2 + (AB)2;
therefore the length of the hypotenuse is 2 units. B
By using a compass, we can transfer the length of the
2 units
hypotenuse OB to the number line (labelled C). This distance 1 unit
can now be measured using a ruler. Although this distance
O A C
will be inaccurate due to the equipment used, there is an exact 0 1 unit 1 2 2 R
point on the number line for each irrational number.
This geometric model can be extended to any irrational number in surd form.
π (pi)
The symbol π (pi) is used for a particular number; that is, the circumference of a circle
whose diameter length is 1 unit. It can be approximated as a decimal which is non-
terminating and non-recurring. Therefore, π is classified as an irrational number. (It is
also called a transcendental number and cannot be expressed as a surd.)
In decimal form, π = 3.141 592 653 589 793 23 . . . It has been calculated to trillions
of decimal places with the aid of a supercomputer.
Rational (Q) and irrational (I) numbers belong to the set of real numbers denoted by
the symbol R. They can be positive, negative or 0. The real numbers can be represented
on a number line as shown (irrational numbers above the line and rational numbers
below the line).
2
–π
–1– 3e
3 —
– 12 2 4 9 2

–4 –3 –2 –1 0 1 2 3 4

–3.62 – 3– 0.2 1.75


3
3–
2 5

Relationship between subsets ε =R


The relationship which exists between the sub-
Q (Rational numbers)
sets of the Real Number System can be illus-
trated in a Venn diagram as shown on the right.
We can say N ⊂ Z, Z ⊂ Q, and so on, where Z (Integers) I
⊂ means ‘is a subset of’. N (Irrational
(Natural numbers)
To classify a number as either rational or numbers)
irrational:
1. Determine whether it can be expressed as an
integer, a fraction, or a terminating or
recurring decimal.
2. If the answer is yes, the number is rational; if the answer is no, the number is irrational.
6 M a t h s Q u e s t M a t h s C Ye a r 1 1 f o r Q u e e n s l a n d

WORKED Example 1
Specify whether the following numbers are rational or irrational.
3 3
a 1
---
4
b 16 c 11 d 2π e 0.28 f 64 g 22 h 3 1
---
8

THINK WRITE
1--- 1
a is already in rational form. a --- is rational.
4 4

b 1 Evaluate 16 . b 16 = 4

2 The answer is an integer, so classify 16 is rational.


16 .

c 1 Evaluate 11 . c 11 = 3.316 624 790 36 . . .

2 The answer is a non-terminating and 11 is irrational.


non-recurring decimal; classify 11 .

d 1 Use your calculator to find the value d 2π = 6.283 185 307 18 . . .


of 2π.
2 The answer is a non-terminating and 2π is irrational.
non-recurring decimal; classify 2π.

e 0.28 is a terminating decimal; classify it e 0.28 is rational.


accordingly.

3 3
f 1 Evaluate 64 . f 64 = 4
3
2 The answer is a whole number, so 64 is rational.
3
classify 64 .

3 3
g 1 Evaluate 22 . g 22 = 2.802 039 330 66 . . .

2 The result is a non-terminating and 3


22 is irrational.
3
non-recurring decimal; classify 22 .

3 1 3 1
h 1 Evaluate --- . h --- = 1
---
8 8 2

2 The result is a number in a rational 3 1


form.
--- is rational.
8
Chapter 1 Number systems: the Real Number System 7
Graphics Calculator tip! Square, cube and nth roots
A graphics calculator can be used to find the square root, cube root or higher root of a
number.

For the Casio fx-9860G AU


1. Press MENU and select RUN-MAT (highlight RUN-MAT and press EXE ).
2. To calculate the square root of a number (for example, 8 ), press SHIFT [ ]
followed by the number (8 in this case) and press EXE .
3. To calculate the cube root of a number (for example,
3
8 ), press SHIFT [ 3 ] followed by the number
(8 in this case) and press EXE .
4. To calculate a higher root of a number (for example,
4
81), first enter the type of root (4 in this example),
then press SHIFT [ x 8] followed by the number
(81 in this case) and press EXE .

For the TI-Nspire CAS


1. From the Home screen (press c), highlight 1: Calculator and press ·.
Alternatively, open a new Calculator document. Press / N and follow the
prompts as to whether you wish to save the previous document. Then press 1 to
select 1: Add Calculator.
2. To calculate the square root of a number (for example, 8 ), press / and q,
followed by the number (8 in this case) and press ·. For the decimal
approximation to this answer, press / and ·.
3. To calculate the cube root of a number (for example, 3 8 ), press / and l.
Complete the gaps in the expression on the screen. First enter 3 for the type of root
(n) and then enter the number (8 in this case for x). Use the tab key to move to the
appropriate place in the expression. Press ·.
4. To calculate a higher root of a number (for example,
4
81), repeat the steps used for the cube root. In this
example, enter 4 for the type of root (n).
(Note that the square and nth root functions can also be
accessed from the Catalog menu. Press k to access
the catalog. Select Option 5 (by pressing 5) then
highlight the required symbol and press ·.)

Summary of set notation


The following symbols are used to describe relationships in sets.
Consider a group of numbers from 1 to 9 (i.e. 1, 2, 3, 4, 5, 6, 7, 8, 9). These numbers
can be referred to as a set and denoted by A such that A = {1, 2, 3, 4, 5, 6, 7, 8, 9}. We
can say that 2 is an element of set A and write this as 2 ∈ A.
Similarly 0 is not an element of set A and this is written as 0 ∉ A.
The elements 2 and 4 both belong to set A and this can be written as {2, 4} ⊂ A,
where 2 and 4 are a subset of A.
8 M a t h s Q u e s t M a t h s C Ye a r 1 1 f o r Q u e e n s l a n d

WORKED Example 2
12 p
13
{
Classify each of the following elements of the set 5, ------ , −3.9, − ------ , 23 , --- into the
2 2 5 }
smallest subset in which it belongs, using Q, I, Z, Z + and Z −.
THINK WRITE
1 The number 5 is a positive whole number; 5 ∈ Z+
classify it accordingly.
13
2 (a) Change ------ into a decimal.
13
------ = 6.5
2 2
(b) The fraction 13
can be expressed as a
------ 13
------ ∈Q
2 2
terminating decimal; therefore it can be
classified as a rational number. The fraction
a
is in the form --- , b ≠ 0, so it is rational.
b
3 The number −3.9 is a terminating decimal, −3.9 ∈ Q
so classify it accordingly.
4 (a) Simplify − 12------ . − 12
------ = −6
2 2

(b) The result is a negative whole number, − 12
------ ∈ Z
2
so classify − 12
------ accordingly.
2
5 (a) Use your calculator to find the value of 23 . 23 = 4.795 831 523 31 . . .
(b) The result is a non-terminating and 23 ∈ I
non-recurring decimal, so 23 can
be classified as an irrational number.
6 (a) Change --π- into a decimal. --π- = 0.628 318 530 718 . . .
5 5
(b) The resulting decimal is neither π
--- ∈ I
terminating nor recurring, 5
so π
--- is an irrational number.
5

remember
remember
1. The real number system (R) contains the set of rational numbers (Q) and the set
of irrational numbers (I).
2. Rational numbers are those that can be written as a ratio of two whole numbers
a
in the form --- where b ≠ 0. Rational numbers include whole numbers,
b
fractions, and terminating and recurring decimals.
3. The set of rational numbers includes the set of integers (Z).
4. The set of integers consists of positive whole numbers (Z +), negative whole
numbers (Z −) and 0. Positive whole numbers (Z +) are also called natural
numbers (N).
5. Irrational numbers cannot be expressed as a ratio of two whole numbers in the
a
form --- where b ≠ 0. Irrational numbers include surds, non-terminating and
b
non-recurring decimals, and numbers such as π and e.
Chapter 1 Number systems: the Real Number System 9

1A Classification of numbers
WORKED 1 Specify whether the following numbers are rational (Q) or irrational (I).
Example
4 7
1 a 4 Q b --- Q c --- Q d 2 I e 7 I
5 9
f 0.04 Q g 2 1--- Q h 5 I i 9
--- Q j 0.15 Q
2 4
k −2.4 Q l 100 Q m 14.4 I n 1.44 Q o π I
p 25
------ Q q 7.32 Q r – 21 I s 1000 I t 7.216 349 157 . .I .
9
3 3
u – 81 Q v 3π I w 62 I x 1
------ Q y 0.0001 I
16

2 Specify whether the following numbers are rational (Q), irrational (I) or neither.
a 1
--- Q b 625 Q c 11
------ Q d 0
--- Q e −6 1--- Q
8 4 8 7
3
f 81 I g – 11 I h 1.44
---------- Q i π I j 8
---
0
Undefined
4
3 π 3 1
k 21 I l --- I m ( –5 )2 I n − -----
3-
Q o --------- Q
7 11 100
64 3
------6-
2- 1-
p ------ Q q ----- I r I s 27 Q t ------ Q
16 25 2 4

( 2)
4
22 π 3
u --------- I v – 1.728 Q w 6 4 Q x Q y 4 6 I
7

3 multiple choice
Which of the following best represents a rational number?
3
A π B 4
--- C 9-
----- D 3 E none of these
9 12

4 multiple choice
Which of the following best represents an irrational number?
A − 81 B 6--- C 3 343 D 0.0676 E 22
5

5 multiple choice
Which of the following statements regarding the numbers −0.69, 7 , π--- , 49 is correct?
3
A π--- is the only rational number.
3
B 7 and 49 are both irrational numbers.
C −0.69 and 49 are the only rational numbers.
D 7 , --π- and 49 are all irrational numbers.
3
E −0.69 is the only rational number.

6 multiple choice
3
Which of the following statements regarding the numbers 2 --1- , − -----
11-
, 624 , 99 is
2 3
correct?
A − 11
------ and 624 are both irrational numbers.
3
3
B 624 is an irrational number and 99 is a rational number.
C 2 1--- is the only rational number.
2
D 624 and 3 99 are both irrational numbers.
E 2 --1- is a rational number and − -----
11-
is an irrational number.
2 3
10 M a t h s Q u e s t M a t h s C Ye a r 1 1 f o r Q u e e n s l a n d

WORKED 7 Classify each of the following into the smallest subset in which it belongs, using Q,
Example
Z + and Z −.
2 .
a 5 Z+ b 0.621 Q c 1-
----- Q d 0.26 Q e 3 + 16 Z +
81
Q
f 0.515 151 . . . g 3
8 Z +
h 42
------ Z + i 7 4 Z+ j − 27
------ Z−
6 3
2 2
Z− k 9 – 144 l 0
--- Q m –( – 4 ) Z − n 9
--- Z+ o −  3--- Q
4 3 2
. . 2
p 0.4 21 Q q ------8- Z+ r 5
– 32 Z− s (– 6) Z+ t − 6--- Q
2 5
6
Z− u – 8× 2 v 4
--- Q w ( 2) Z+ x 100-
------------ Z + y 3
– 343 Z −
3 2

8 Classify each of the following into the smallest subset in which it belongs using Q, I,
Z + and Z −.
a 6 Z+ b 0.3415 . .I . c 7 I 9- Q
d ----- e – 2 25 Z −
16
− Q
f 6× 2 I g – 49 Z h 21 × 5 I i 0.612 612 . . . j 0.25 Q

k 144
--------- Z+ l 3
– 64 Z − m

11
------ Q n 9-
-------- Q o 50
------ Z+
9 Z 19 144 2
3
p 5π I q 16 × 3 – 27 r ( 3) I s 7× 5 I t – 6 × 3 16 Z −

u − 16
------ I v 8 × 12.5 w − 1--- Q x (π)2 I y – 3 – 125 Z+
8 5
Z+
9 multiple choice
The smallest subset in which 7 + 2  3 1--- belongs is:
8
A Q B I C Z+ D Z E Z−

10 multiple choice
The smallest subset in which 144
--------- × 3 512
--------- belongs is:
9 8
+
A Q B I C Z D Z E Z−

11 multiple choice
Which of the following statements regarding numbers 16, −3 2 , 0, π, { 8
---
2 } is correct?
A 16 and 0 are the only rational numbers.
B 16, 0 and 8 may be expressed as rational numbers.
---
2
C 16 and 0 are positive integers.
D −3 2 is the only irrational number.
E π is the only irrational number.

12 multiple choice
Which of the following statements regarding the given set of numbers
{ 2 + 9 , 11 , 16 2 , 32 , 81 } is correct?
A All of the above numbers in the set are irrational.
B 2 + 9 and 11 are the only irrational numbers of the set.
C 2 + 9 is a rational number of the set.
D 81 is the only rational number of the set.
E 11 and 32 are the only irrational numbers of the set.
Chapter 1 Number systems: the Real Number System 11
Recurring decimals
A rational number may be converted to a decimal by dividing the numerator by the
denominator. The resulting decimal may be a terminating decimal containing a specific
number of digits, that is:
7
--- = 1.4 or 1
--- = 0.125
5 8

or it may be a recurring decimal containing a repeating digit or pattern, that is:


2
--- = 0.222 . . . or 7-
----- = 0.538 461 538 461 . . .
9 13

For convenience, recurring decimals are represented by placing a dot over the
repeating digit, for example:
.
1. 0.777 777 7 . . . can be written as 0.7.
.
2. 0.26666 . . . can be written as 0.26.

If two or more digits repeat the same pattern, then dots or the overscore ( ) are
used as shown:
..
1. 0.454 545 . . . can be written as 0.4 5 or alternatively 0.45 .
. .
2. 0.752 137 521 3 . . . can be written as 0.7 5213 or alternatively 0.75213 .

Note: When using the overscore, place it over the whole pattern. The dots, however, are
placed over the first and the last digits only of the repeating pattern.

WORKED Example 3
State which of the following rational numbers can be expressed as recurring decimals.
2 5
a ------
27
b ---
8

THINK WRITE
2-
a 1 To convert ----- to a decimal, divide 2 a 2-
----- = 0.074 074 074 . . .
27 27
by 27.

2 Use the overscore to indicate the


= 0.074
repeating pattern.
2-
3 Write your conclusion. can be written as a recurring
-----
27
decimal.

5---
b 1 Convert to a decimal (divide 5 b 5
--- = 0.625
8 8
by 8).
5
2 The resulting decimal terminates, so cannot be written as a recurring
---
8
state your conclusion. decimal.
12 M a t h s Q u e s t M a t h s C Ye a r 1 1 f o r Q u e e n s l a n d

Whole numbers and terminating decimals such as 3, 0.25 and 6.731 can easily be
expressed as rational numbers. For example:
3
1. we may write 3 as ---
1
2. we may write 0.25 as 25
--------- = 1
---
100 4
731- 6731
3. we may write 6.731 as 6 ----------- or ------------ .
1000 1000
a
In each of these cases, the whole number and decimals are expressed in the form --- .
b
Recurring decimals are rational numbers. Therefore they can be converted to the
a
form --- .
b

WORKED Example 4
Express the following recurring decimals as rational numbers in their simplest form.
. ..
a 0.4 b 0.2 1 c 1.285

THINK WRITE
a 1 Let x represent the recurring a x = 0.444 444 . . . [1]
decimal. This is equation [1].
2 We need to multiply both sides of the 10x = 4.444 444 . . . [2]
equation by a power of 10. The
number of zeros in the power of 10
should be equal to the number of
repeated digits. Since 1 digit is
repeated, multiply both sides of
equation [1] by 10. Label the new
equation [2].
3 Subtract equation [1] from equation [2] − [1]:
[2]. This removes all the repeating 10x − x = 4.444 444 . . . − 0.444 444 . . .
digits after the decimal point.
9x = 4
9x 4
4 Divide both sides of the equation ------ = ---
by 9. 9 9
4
x = ---
9
5 Verify the answer using a calculator
and you will
. obtain the original
value, 0.4.

b 1 Let x represent the recurring b x = 0.212 121 21 . . . [1]


decimal. This is equation [1].
2 Since 2 digits are repeated, multiply 100x = 21.212 121 21 . . . [2]
both sides of equation [1] by 100
and label the new equation [2].
Chapter 1 Number systems: the Real Number System 13
THINK WRITE
3 Subtract equation [1] from equation [2]. [2] − [1]:
This removes all the repeating digits after 100x − x = 21.212 121 21 . . .
the decimal point. − 0.212 121 21 . . .
99x = 21
99x 21
4 Divide both sides of the equation by 99. --------- = ------
99 99
21
x = ------
99
7
5 Cancel to the simplest form; that is, x = ------
divide through by 3. 33

6 Verify the answer using a calculator.

c 1 Let x represent the recurring decimal. c x = 1.285 285 285 . . . [1]


This is equation [1].
2 Since 3 digits are repeated, multiply both 1000x = 1285.285 285 285 . . . [2]
sides of equation [1] by 1000 and label
the new equation [2].
3 Subtract equation [1] from equation [2]. [2] − [1]:
This removes all the repeating digits after 1000x − x = 1285.285 285 285 . . .
the decimal point. − 1.285 285 285 . . .
999x = 1284
999x
------------ = 1284
------------
4 Divide both sides of the equation by
999. 999 999
1284
x = ------------
999
428
5 Cancel to the simplest form; that is, x = ---------
divide through by 3. 333

6 Verify the answer using a calculator.

remember
remember
1. Rational numbers can be converted to decimals by dividing the numerator by
the denominator. The resulting decimal can be either terminating or recurring.
2. Terminating decimals contain a specific number of digits.
3. Recurring decimals contain a repeating digit or a repeating pattern of digits.
4. Recurring decimals are represented by placing dots over the first and the last
digits of the repeating pattern. Alternatively, an overscore can be placed over
the whole pattern that repeats.
5. Recurring decimals are rational numbers and may be expressed as a ratio of
two integers.
14 M a t h s Q u e s t M a t h s C Ye a r 1 1 f o r Q u e e n s l a n d

1B Recurring decimals
WORKED 1 State which of the following rational numbers can be expressed as recurring decimals.
Example
3 a --1- b --1- c --1- 6-
d ----- 1-
e -----
8 2 3 19 17
4- 5 7- 9- 5
f ----- g --- h ----- i ----- j ---
1 c f g j k l n 11 9 16 25 7
2 1 3 3- 5-
oq r t w x y k --- l --- m --- n ----- o -----
3 6 4 13 21
2- 2 41- 5 17
p ----- q --- r -------- s --- t ------
31 9 333 8 18
8- 7- 7- 3- 7-
u ----- v ----- w ----- x ----- y -----
17 23 15 22 33

WORKED 2 Express the following recurring decimals as rational numbers in their simplest form.
Example . 2 . . 8 . 5 .
4 a 0.2 --9- b 0.7 --79- c 0.8 --9- d 0.5 --9- e 0.4 --49-
. . 17 . 19 . 31 . 32
f 0.16 1--6- g 0.37 ----- -
45 h 0.42 -----
45
- i 0.68 ------
45 j 0.71 ----- -
45
. 28 . . 53 .. 4 . . 34 . . 367-
k 2.62 2 ----- 45
-
l 0.53 ----- -
99 m 0.1 2 -----
33
- n 1.3 4 1 -----
99
-
o 3.74 1 3 --------
495
. . 361 . . 427 . . 868 . . 323 . . 152
p 0.3 61 999 --------
- q 0.4 27 999 ---------
r 0.52 13 1665 s 0.3 23 999
------------ --------
- t 3.4 56 333 3 ---------
. . 157 . . 1237 . 5611 . . 2
u 0.72 13 ------
18 v 0.523 ---------
300 w 0.624 7 ------------ x
1980 0.6234 ------------ y
9000 0.1 53846 ----- -
13

3 multiple choice
..
The recurring decimal 0.7 8 can be expressed as:
A 71
------ B 78
------ C 77
------ D 71
------ E 78
------
90 90 99 99 99

4 multiple choice
..
The recurring decimal 0.53 2 can be expressed as:
266- 479- 532- 527- 532-
A -------- B -------- C -------- D -------- E --------
495 900 999 990 990

5 multiple choice
Which statement regarding the fractions 1--- , 3--- , 11 1 4
------ , --- , --- is correct?
2 7 13 3 5
1
--- , 3--- and 11 ------ are the only fractions which represent terminating decimals.
A
2 7 13
3
--- and 1 --- are the only fractions which represent terminating decimals.
B
7 3
3
--- , 11
------ and 1 --- are fractions which represent recurring decimals.
C
7 13 3
11
------ and 1 --- are the only fractions which represent recurring decimals.
D
13 3
1 4
--- and --- are fractions which represent recurring decimals.
E
2 5

6 multiple choice
. .
The recurring decimal 0.369 can be expressed in its simplest form as the following fraction:
407
A 369
--------- B 3663
------------ C 123
--------- D -----------
- 41-
E --------
999 9900 1100
333 111

7 Irene and Bella are arguing about the correct way of writing the recurring decimal
0.020 20 . . . . Irene says it should be written as 0.020, while Bella thinks it is 0.020.
Which of the girls is right? Irene. It can also be written as 0.02.
Chapter 1 Number systems: the Real Number System 15
Real number investigations
A real number can be defined as a number that can be plotted on a number line.
Even if the position of the number on the line is only an approximate value, as long
as a number can be represented by one point on a line it can be regarded as real.
This is not so with the numbers you will deal with in Chapter 2.
The following steps will enable you to plot irrational numbers such as surds (for
example 2 and 5 ) on a number line.
Materials needed: ruler, a set of compasses, set square.
Step 1 Draw a number line approximately 10 cm long, with unit
divisions of 2 cm. How can we draw a line segment
√2 1
exactly 2 units long? Using Pythagoras’ Theorem we
can obtain the triangle shown at right which shows us
that 2 = 1 + 1 . 1

Step 2 Use a set square to construct a right-angled triangle as shown below:

√2 1

–1 0 1 2 3

Step 3 Use a set of compasses to transfer the length of the hypotenuse to the
number line.

√2
1

–1 0 1 √2 2 3

Step 4 If a second right-angled triangle (of height 1 cm) were constructed on this
hypotenuse, what would be the length of its hypotenuse?

√3

√2
1

–1 0 1 √2 √3 2 3

Step 5 Continue constructing in this way to plot 7 on the original number line.
Use your number line to give an approximate value for 7 .
16 M a t h s Q u e s t M a t h s C Ye a r 1 1 f o r Q u e e n s l a n d

Other number systems


Introduction
Throughout early civilisations, numbers have been represented and recorded in a
variety of ways.
Our numeration system uses the 10 digits
0, 1, 2, 3, 4, 5, 6, 7, 8, 9 and combinations of
these. It is called the decimal or base 10 system
(possibly influenced by the fact that we have
10 fingers).
Past civilisations have used base 5 and base 20
systems (again influenced by the fingers on
one hand and the total number of digits).
Mesopotamians used a base 60 system, which is
still used today for units of time (60 seconds in
1 minute and 60 minutes in 1 hour). Numeration
systems that are used today include a binary or
base 2 system and a modular or base 12 system.
Place value
The place value system was introduced as a means of recording numbers.
Look at the number 285. In our numeration system — the base 10 (decimal)
system — we interpret the number 285 (base 10) or 28510 as:
plus 2 lots of 100 or 2 × 102
plus 8 lots of 10 or 8 × 101
plus 5 lots of 1 or 5 × 100
Using the base 5 system Using the base 2 system
Numbers in the base 5 system use the Numbers in the base 2 system use the
digits 0, 1, 2, 3 and 4 only. The number digits 0 and 1 only. The number 285
285 (base 10) can be written as a base (base 10) can be written as a base 2
5 number in the following way: number in the following way:
2 lots of 125 or 2 × 53 1 lot of 256 or 1 × 28
plus 1 lot of 25 or 1 × 52 plus 0 lots of 128 or 0 × 27
plus 2 lots of 5 or 2 × 51 plus 0 lots of 64 or 0 × 26
plus 0 lots of 1 or 0 × 50 plus 0 lots of 32 or 0 × 25
So 28510 = 21205 plus 1 lot of 16 or 1 × 24
plus 1 lot of 8 or 1 × 23
plus 1 lot of 4 or 1 × 22
plus 0 lots of 2 or 0 × 21
plus 1 lot of 1 or 1 × 20
So 28510 = 100 011 1012
Notice how we need to use zeros to hold each place value.
See Solutions Manual. 1 Investigate the following points relating to non-base 10 numbers, giving
examples in each case:
a How could numbers of different bases be compared to each other?
b How are numbers of the same (non-base 10) system added and subtracted?
c How are numbers of the same (non-base 10) system multiplied and divided?
d How are fractions and decimals of a non-base 10 system represented?
Chapter 1 Number systems: the Real Number System 17
Binary systems
As the name suggests, this
numeration system is based on 2.
In this system, 0 and 1 are the only
two digits used. The binary system
is used in computers.
2 Investigate how the binary
system is used in computers,
circuits or compact discs.
Devise a situation which calls
for the use of a binary system.

Modular arithmetic
Modular arithmetic involves
‘clock’ arithmetic where, instead of
saying that the time is 14 o’clock,
we say it is 2 o’clock. This is called
modular (mod 12) arithmetic. Any
integer can be converted to modular
(mod 12) arithmetic by subtracting
12 or any multiple of 12 from the
integer. The remainder is called the
residue.
For example:
32 = 2 × 12 + 8 68 = 5 × 13 + 3 29 = 4 × 6 + 5
≡ 8 (mod 12) ≡ 3 (mod 13) ≡ 5 (mod 6)

The remainders or residues in this case are 8, 3 and 5 respectively.


3 Investigate the purpose, usefulness and limitations of modular arithmetic.
Include illustrations of how numbers of modular arithmetic are represented via a
clock pattern.

Surds
A surd is an irrational number which can only be represented exactly using a root
3 4
sign or radical, for example: , ,

3 4
Examples of surds include: 7, 5, 11 , 15

Examples that are not surds include:


3 4
9, 16 , 125 , 81

Numbers that are not surds can be simplified to rational numbers, that is:
3 4
9 = 3, 16 = 4 , 125 = 5 , 81 = 3
18 M a t h s Q u e s t M a t h s C Ye a r 1 1 f o r Q u e e n s l a n d

WORKED Example 5
Which of the following numbers are surds?
1 3 4 3
a 25 b 10 c --- d 11 e 59 f 343
4

THINK WRITE
a 1 Evaluate 25 . a 25 = 5
2 The answer is rational (since it is a 25 is not a surd.
whole number), so state your
conclusion.

b 1 Evaluate 10 . b 10 = 3.162 277 660 17 . . .


2 The answer is irrational (since it is a 10 is a surd.
non-recurring and non-terminating
decimal), so state your conclusion.

c 1 Evaluate 1
--- .
4
c 1
--- = 1
---
4 2

The answer is rational (a fraction); 1 is not a surd.


2 ---
4
state your conclusion.

d 1 Evaluate 3 11 . d 3
11 = 2.223 980 090 57 . ..
3
2 The answer is irrational (a non- 11 is a surd.
terminating and non-recurring
decimal), so state your conclusion.

e 1 Evaluate 4 59 . e 4
59 = 2.771 488 002 48 . . .
4
2 The answer is irrational, so classify 59 is a surd.
4
59 accordingly.

f 1 Evaluate 3 343 . f 3
343 = 7
3
2 The answer is rational; state your 343 is not a surd.
conclusion.
So b, d and e are surds.

Proof that a number is irrational


As part of your Mathematics C course you are required to study a variety of types of
proofs. One such method is called ‘Proof by contradiction’.
This method is so named because the logical argument of the proof is based on
an assumption that leads to contradiction within the proof. Therefore the original
assumption must be false.
a
An irrational number is one that cannot be expressed in the form --- (where a and b
b
are integers). The next worked example sets out to prove that 2 is irrational.
Chapter 1 Number systems: the Real Number System 19
SLE 8: Use a proof by contradiction to show that 2 is irrational.

WORKED Example 6
Prove that 2 is irrational.
THINK WRITE
a
1 Assume that 2 is rational; that is, it 2 = --- where b ≠ 0
b
a
can be written as --- in simplest form.
b
We need to show that a and b have no
common factors.
2
a
2 Square both sides of the equation. 2 = -----
2
b
3 Rearrange the equation to make a2 the a2 = 2b2 [1]
subject of the formula.
4 If x is an even number then x = 2n. ∴ a2 is an even number and a must also
be even; that is, a has a factor of 2.
5 Since a is even it can be written as ∴ a = 2r
a = 2r.
6 Square both sides. a2 = 4r2 [2]
But a2 = 2b2 from [1]
7 Equating [1] and [2] ∴ 2b2 = 4r2
2
4r
b2 = --------
2
= 2r2
∴ b is an even number and b must also be
2

even; that is, b has a factor of 2.


8 State your conclusion. Both a and b have a common factor of 2.
This contradicts the original assumption that
a
2 = --- where a and b have no common factor.
b
∴ 2 is not rational.
∴ It must be irrational.

The ‘dialogue’ included in the worked example should be present in all proofs and is an
essential part of the communication that is needed in all your solutions.
Note: An irrational number written in surd form gives an exact value of the number;
whereas the same number written in decimal form (for example, to 4 decimal places)
gives an approximate value.

remember
remember
A number is a surd if:
1. it is an irrational number (equals a non-terminating, non-recurring decimal)
2. it can be written exactly only by using a radical (or root sign).
20 M a t h s Q u e s t M a t h s C Ye a r 1 1 f o r Q u e e n s l a n d

1C Surds
WORKED 1 Which of the numbers below are surds?
Example
5 a 81 b 48 c 16 d 1.6 e 0.16
3 3 3
1 b d f g h i l f 11 g --- h ------ i 1000 j 1.44
4 27
m o q s t w 3 3
k 4 100 l 2 + 10 m 32 n 361 o 100
3 3
p 125 q 6+ 6 r 2π s 169 t 7
---
8
4 2 3 5
u 16 v ( 7) w 33 x 0.0001 y 32

2 multiple choice
The correct statement regarding the set of numbers
3
{ 6
--- ,
9
20 , 54 , 3
27 , }
9 is:
A 27 and 9 are the only rational numbers of the set.
6
B --- is the only surd of the set.
9
6
C --- and 20 are the only surds of the set.
9
D 20 and 54 are the only surds of the set.
E All of the numbers of the set are surds.
WORKED 3 Prove that the following numbers are irrational, using a proof by contradiction:
Example
6 a 3 b 5 c 7

4 multiple choice
Which of the numbers of the set { 1 1-
--- , 3 -----
4 27
, 1
--- ,
8
21 , 3
}
8 are surds?
3 1 1 1 3 1
A ------ B 21 only C --- only D --- and 8 E --- and 21 only
27 8 8 8

5 multiple choice
Which statement regarding the set of numbers π, { 1-
-----
49
, 12 , 16 , 3 + 1 is not true?}
A 12 is a surd. B 12 and 16 are surds.
C π is irrational but not a surd. D 12 and 3 + 1 are not rational.
E 1-
----- when simplified is a rational number.
49

6 multiple choice
Which statement regarding the set of numbers 6 7 , { 144
--------- ,
16
7 6, 9 2, 18 , }
25 is
not true?
144 144
A --------- when simplified is an integer. B --------- and 25 are not surds.
16 16
C 7 6 is smaller than 9 2 . D 9 2 is smaller than 6 7 .
E 6 7 , 7 6 , 9 2 and 18 are surds.
Chapter 1 Number systems: the Real Number System 21
6
7 If a is a multiple of 4, find the smallest, non-zero rational value of a . 2 (when a = 64)
3
8 Find the smallest value of m, where m is a positive integer, so that 16m is not a surd. m = 4

Simplifying surds
To simplify a surd means to make a number (or an expression) under the radical ( )
as small as possible. To simplify a surd (if it is possible), it should be rewritten as a
product of two factors, one of which is a perfect square, that is, 4, 9, 16, 25, 36, 49, 64,
81, 100 and so on.
We must always aim to obtain the largest perfect square when simplifying surds so
that there are fewer steps involved in obtaining the answer. For example, 32 could be
written as 4 × 8 = 2 8 ; however, 8 can be further simplified to 2 2 , so
32 = 2 × 2 2 ; that is 32 = 4 2 . If, however, the largest perfect square had been
selected and 32 had been written as 16 × 2 = 16 × 2 = 4 2 , the same answer
would be obtained in fewer steps.

WORKED Example 7
Simplify the following surds. Assume that x and y are positive real numbers.
a 384 b 3 405 c − 1--- 175 d 5 180 x 3 y 5
8

THINK WRITE

a 1 Express 384 as a product of two a 384 = 64 × 6


factors where one factor is the
largest possible perfect square.
2 Express 64 × 6 as the product of = 64 × 6
two surds.
3 Simplify the square root from the =8 6
perfect square (that is, 64 = 8).

b 1 Express 405 as a product of two b 3 405 = 3 81 × 5


factors, one of which is the largest
possible perfect square.
2 Express 81 × 5 as a product of two = 3 81 × 5
surds.
3 Simplify 81 . = 3×9 5
4 Multiply together the whole numbers = 27 5
outside the root (3 and 9).
Continued over page
22 M a t h s Q u e s t M a t h s C Ye a r 1 1 f o r Q u e e n s l a n d

THINK WRITE

c 1 Express 175 as a product of two factors c − 1--- 175 = − 1--- 25 × 7


8 8
where one factor is the largest possible
perfect square.
2 Express 25 × 7 as a product of 2 = − 1--- × 25 × 7
8
surds.
3 Simplify 25 . = − 1--- × 5 7
8

4 Multiply together the numbers outside = − 5--- 7


8
the square root sign.
d 1 Express each of 180, x3 and y5 as a d 5 180x 3 y 5 = 5 36 × 5 × x 2 × x × y 4 × y
product of two factors where one factor
is the largest possible perfect square.
2 Separate all perfect squares into one = 5 × 36x 2 y 4 × 5xy
surd and all other factors into the other
surd.
3 Simplify 36x 2 y 4 . = 5 × 6 × x × y 2 × 5xy
4 Multiply together numbers and the = 30xy 2 5xy
pronumerals outside the square root
sign.

remember
remember
1. To simplify a surd means to make a number (or an expression) under the radical
as small as possible. For example, 2 5 is equal to, but simpler than, 20 .
2. To simplify a surd, write it as a product of two factors, one of which is the
largest possible perfect square.

1D Simplifying surds
WORKED 1 Simplify the following surds.
Example
7a a 12 2 3 b 18 3 2 c 24 2 6 d 56 2 14 e 27 3 3

eBook plus
f 75 5 3 g 125 5 5 h 99 3 11 i 54 3 6 j 60 2 15

Digital docs: k 112 4 7 l 98 7 2 m 68 2 17 n 150 5 6 o 180 6 5


SkillSHEET 1.1
Simplifying surds p 338 13 2 q 88 2 22 r 135 3 15 s 162 9 2 t 200 10 2
EXCEL Spreadsheet
Simplifying surds u 245 7 5 v 320 8 5 w 448 8 7 x 735 7 15 y 405 9 5
Chapter 1 Number systems: the Real Number System 23
WORKED 2 Simplify the following surds. – 30 3 24 7
Example
7b, c a 2 8 4 2 b 3 50 15 2 c 8 90 24 10 d 6 112 e 9 80 36 5

10 17 f 5 68 g 7 54 21 6 h 10 32 40 2 i – 6 75 j 3 252 18 7
1 1
– 28 5 k – 7 80 l 9 120 m 16 48 64 3 n --- 90 10 o --- 392 2 2
3 7
18 30 1 1 2 1 1
p --- 625 5 q --- 162 2 r --- 54 2 6 s --- 192 2 3 t --- 288 2 2
5 9 3 4 6
--1-
3
15 u 1
--- 135 v 5
--- 320 w 3
------ 175 --3- 7 x 7
--- 176 --7- 11 y − 4--- 108 −8 3
9 2 10 2 8 2 3
20 5
WORKED 3 Simplify the following surds. Assume that a, b, c, d, e, f, x and y are positive real numbers.
Example
7d a 16a 2 4a b 81a 2 b 2 9ab c 72a 2 6a 2
d 54a 2 b 2 3ab 6 e 90a 2 b 3a 10b f 48a 3 b 4a 3ab
g 338a 4 13a 2 2 h 150a 4 b 2 5a 2 b 6 i 338a 3 b 3 13ab 2ab
3 5
j 12a 5 b 7 2a 2 b 3 3ab k 68a b 2ab 2 17ab l 80x 6 y 4x 3 5y

m 125x 6 y 4 5x 3 y 2 5 n 3 64x 2 y 24x y o 5 80x 3 y 2 20xy 5x


p 2 343x 3 y 3 14xy 7xy q 6 162c 7 d 5 54c 3 d 2 2cd r 3 126c 4 d 5 9c 2 d 2 14d
1
18c 3 d 4 5cd s 2 405c 7 d 9 t 4 294c 10 d 10 28c 5 d 5 6 u ---
2
88ef 22ef

1 1 3
v --- 120e 4 f 6 2
--- 2
3
e f 3 30 w --- 392e 11 f 11 7e 5 f 5 2ef x ------ 175e 12 f 5 3--- 6
e f2 7f
4
3 2 20

1 1
xy 4 6xy 1
y ------ 54x 3 y 9 ---
9 z ------ 108x 10 y 12 1--- 5 6
x y
3
3
27 18

4 multiple choice
When expressed in its simplest form, 45 is equal to:

A 3 15 B 5 9 C 5 3 D 9 5 E 3 5

5 multiple choice
When expressed in its simplest form, 3 128 is equal to:

A 6 32 B 12 8 C 24 2 D 16 2 E 32 3

6 multiple choice
1
When expressed in its simplest form, --- 539 is equal to:
7

A 49 11 B 7 11 C 77 D 11 E 11

7 multiple choice
1
- 325x 4 y 3 when expressed in its
Assuming that x and y are positive real numbers, – -----
15
simplest form is equal to:
A – 1--- xy 13x 2 y 1 2
B – -----
- x y 13y C – 1--- x 2 y 13y
3 15 3

D – 3x 2 y 13y E – 3xy 13x 2 y


24 M a t h s Q u e s t M a t h s C Ye a r 1 1 f o r Q u e e n s l a n d

Addition and subtraction of surds


Surds may be added or subtracted only if they are alike.
Examples of like surds include 7 , 3 7 and – 5 7 . Examples of unlike surds
include 11 , 5 , 2 13 and – 2 3 .
In some cases surds will need to be simplified before you decide whether they are
like or unlike, and then addition and subtraction can take place. The concept of adding
and subtracting surds is similar to adding and subtracting like terms in algebra.

WORKED Example 8
Simplify each of the following expressions containing surds. Assume that a and b are
positive real numbers.
a 3 6 + 17 6 – 2 6
b 5 3 + 2 12 – 5 2 + 3 8
1
c ---
2
100a 3 b 2 + ab 36a – 5 4a 2 b

THINK WRITE
a All 3 terms are alike, since they a 3 6 + 17 6 – 2 6 = ( 3 + 17 – 2 ) 6
contain the same surd ( 6 ) , so
group like terms together and = 18 6
simplify.

b 1 Simplify surds where b 5 3 + 2 12 – 5 2 + 3 8


possible.
= 5 3+2 4×3–5 2+3 4×2

= 5 3+2×2 3–5 2+3×2 2

= 5 3+4 3–5 2+6 2

2 Add like terms to obtain the = 9 3+ 2


simplified answer.

1
c 1 Simplify surds where c --- 100a 3 b 2 + ab 36a – 5 4a 2 b
2
possible.
1
= --- × 10 a 2 × a × b 2 + ab × 6 a – 5 × 2 × a b
2
1
= --- × 10 × a × b a + ab × 6 a – 5 × 2 × a b
2

= 5ab a + 6ab a – 10a b

2 Add like terms to obtain the = 11ab a – 10a b


simplified answer.
Chapter 1 Number systems: the Real Number System 25
WORKED Example 9
Determine the perimeter of a rectangle whose length is ( 17 – 2 50 ) m and width is
( 5 + 32 ) m.

THINK WRITE
1 Write down the rule for the P = 2l + 2w
perimeter of a rectangle where l is
the length and w is the width.

2 Substitute the values of l and w P = 2 ( 17 – 2 50 ) + 2 ( 5 + 32 )


into the rule.

3 Expand and simplify where = 34 – 4 50 + 10 + 2 32


possible.

4 Simplify surds where possible. = 34 – 4 25 × 2 + 10 + 2 16 × 2


= 34 – 4 × 5 2 + 10 + 2 × 4 2
= 34 – 20 2 + 10 + 8 2

5 Collect like terms. = 44 – 12 2

6 State the answer, including the P = ( 44 – 12 2 ) m


appropriate unit.

remember
remember
1. Only like surds may be added and subtracted.
Examples of like surds: 7 , 3 7 and −5 7 .
Examples of unlike surds: 3 , 5 and 2 13 .
2. Surds may need to be simplified before adding and subtracting.

1E Addition and subtraction


of surds
WORKED 1 Simplify the following expressions containing surds. Assume that x and y are positive
Example
8a
real numbers.

a 3 5+4 5 7 5 b 6 2 + 11 2 17 2

c 2 3+5 3+ 3 8 3 d 6 7 + 8 7 + 5 7 19 7

e 8 5 + 3 3 + 7 5 + 2 3 15 5 + 5 3 f 2 6 + 9 2 + 6 2 + 5 6 15 2 + 7 6
26 M a t h s Q u e s t M a t h s C Ye a r 1 1 f o r Q u e e n s l a n d

g 6 11 – 2 11 4 11 h 12 13 – 5 13 – 2 13 5 13

i 7 2 + 9 2 – 3 2 13 2 j 3 7 – 2 5 + 7 7 – 9 5 10 7 – 11 5

k 9 6 + 12 6 – 17 6 – 7 6 –3 6 l 5 2 – 12 2 – 3 6 + 8 6 –7 2 + 5 6

17 3 – 18 7 m 12 3 – 8 7 + 5 3 – 10 7 n xy + 7 xy – 3 xy 5 xy
x – 5 y + 7 xy
o 2 x+5 y+6 x–2 y 8 x+3 y p 3 x + 4 y + 7 xy – 2 x – 9 y
WORKED 2 Simplify the following expressions containing surds. Assume that a and b are positive
Example
8b real numbers.

a 200 – 300 10 ( 2 – 3 ) b 18 + 50 – 72 2 2

c 125 – 150 + 600 5 ( 5 + 6 ) d 96 – 5 24 + 12 – 6 6 + 2 3

e 27 – 3 + 75 7 3 f 8 + 18 + 50 10 2

g 2 20 – 3 5 + 45 4 5 h 45 + 20 5 5

14 3 + 3 2 i 6 12 + 3 27 – 7 3 + 18 j 44 – 99 + 121 – 3 11 11 – 4 11

3 6+6 3 k 150 + 24 – 96 + 108 l 98 – 2 50 + 5 32 17 2

15 10 − 10 15 + 10 m 3 90 – 5 60 + 3 40 + 100 n 2 99 – 44 – 176 0

– 8 11 + 22 o 5 11 + 7 44 – 9 99 + 2 121 p 5 3 + 8 27 – 4 3 + 2 147 39 3

12 30 – 16 15 q 2 30 + 5 120 + 60 – 6 135 r 20 – 50 – 80 – 120 + 60


1
–2 5 – 5 2 – 2 30 + 2 15
12 ab + 7 3ab s 6 ab – 12ab + 2 9ab + 3 27ab t --- 50 + 2--- 98 – 3--- 32
5 7 4
0

u 1
--- 98 + 1
--- 48 + 1
--- 12
7
---
2
2+2 3 v 1
------ 512 – 5
--- 128 + 1
--- 72 –3 2
2 3 3 16 8 6

1
w --- 32 – 7--- 18 + 3 72 15 2 x 1
---
7
27 + ----- 5
- 12 – ------ 48 --5-
8
3
8 6 8 16 32

WORKED 3 Simplify the following expressions containing surds. Assume that a and b are positive
Example
8c real numbers. 52 a – 29 3a
34 a – 6 2a a 7 a – 8a + 9 9a – 32a b 10 a – 15 27a + 8 12a + 14 9a

c 150ab + 96ab – 54ab 6 6ab d 16 4a 2 – 24a + 4 8a 2 + 96a

f 1
36a + 1--- 128a – 1--- 144a a + 2 2a
e 8a 3 + 72a 3 – 98a 3 a 2a ---
2 4 6

g 9a 3 + 3a 5 3a a + a 2 3a h 6 a5b + a3b – 5 a5b ( a 2 + a ) ab

4ab ab + 3a 2 b b i ab ab + 3ab a 2 b + 9a 3 b 3 j a 3 b + 5 ab – 2 ab + 5 a 3 b 3 ab ( 2a + 1 )

– 6ab 2a + 4a 2 b 3 3a k 32a 3 b 2 – 5ab 8a + 48a 5 b 6 l 4a 2 b + 5 a 2 b – 3 9a 2 b – 2a b

32a + 2 6a + 8a 2
Chapter 1 Number systems: the Real Number System 27
4 multiple choice
When expressed in its simplest surd form, 112 – 63 is equal to:
A 5 7 B 5 C 1 D 7 E none of these

5 multiple choice
When expressed in its simplest surd form, 2 40a – 6 72ab 2 is equal to:
A – 32b 12a B – 32b 8a C 4 10a – 12b 18a
D – 4b 32a E 4 10a – 36b 2a

6 multiple choice
7
When expressed in its simplest surd form, ------ 100a 2 – 2--- 25a 2 + 1--- 72b 2 is equal to:
10 5 6
A 5a + b 2 B 5 + 2b C 5a D 9a + 2b E 9a a + 2 b

7 multiple choice
When expressed in its simplest surd form, 243a 3 b 6 – 27a is equal to:
A 6ab 3 3a B 3ab 3 3a C 6ab 3
D 3ab 3 E 3 3a ( 3ab 3 – 1 )

8 multiple choice
When expressed in its simplest surd form, 150c 2 d 2 – cd 96 – c 54d 2 is equal to:
A 6cd 6 B – 2cd 6 C 4cd 6 D – 2cd E – 6cd 6
WORKED 9 Find the perimeter of the following shapes, giving answers in the simplest surd form.
Example
9
Specify the units.
9
a b 6 cm c 5 + 2 cm a 12 2 cm
eBook plus b ( 6 6 + 8 3 ) cm
18 cm 48 cm
c ( 18 – 2 3 + 2 5 ) cm
Digital docs:
SkillSHEET 1.2 27 + 54 cm d 3π 5 m
Substitution using 7 – 3 cm
surds 1 e ( 18 2 + 2 5 ) m
24 + 3 cm
WorkSHEET 1.1 f 21 11 m
d e 5 2– 5m f 3 44 – 99 m
2 44 m
45 m

5+2 2m 4 44 + 2 99 m

Multiplication of surds
To multiply surds, multiply together the expressions under the radicals. For example,
a × b = ab , where a and b are positive real numbers.
When multiplying surds it is best to first simplify them (if possible). Once this has
been done and a mixed surd has been obtained, the coefficients are multiplied with each
other and then the surds are multiplied together. For example,
m a ¥ n b = mn ab
28 M a t h s Q u e s t M a t h s C Ye a r 1 1 f o r Q u e e n s l a n d

WORKED Example 10
Multiply the following surds, expressing answers in the simplest form. Assume that x and
y are positive real numbers.
a 11 ¥ 7 b 5 3¥8 5 c 5 ¥ 10
3 1
d 6 12 ¥ 2 6 e ---
5
70 ¥ --4- 10 f 15 x 5 y 2 ¥ 12 x 2 y

THINK WRITE
a Multiply surds together, using a 11 × 7 = 11 × 7
a × b = ab (that is, multiply = 77
expressions under the roots).
Note: This expression cannot be simplified
any further.

b Multiply the coefficients and then multiply b 5 3×8 5 = 5×8× 3× 5


the surds. = 40 × 3 × 5
= 40 15

c 1 Multiply the surds. c 5 × 10 = 5 × 10


= 50
2 Simplify the product surd if possible. = 25 × 2
=5 2

d 1 Simplify 12 . d 6 12 × 2 6 = 6 4 × 3 × 2 6
= 6×2 3×2 6
= 12 3 × 2 6
2 Multiply the coefficients and multiply = 24 18
the surds.
3 Simplify the product surd. = 24 9 × 2
= 24 × 3 2
= 72 2

3
e 1 Multiply the coefficients and multiply e --- 70 × 1--- 10 = 3
--- × 1--- × 70 × 10
5 4 5 4
the surds.
3
= ------ 700
20
3
2 Simplify the product surd. = ------ 100 × 7
20
3
= ------ × 10 7
20

3 3 7
3 Simplify by dividing both 10 and 20 by = --- 7 or ----------
2 2
10 (cross-cancel).
Chapter 1 Number systems: the Real Number System 29
THINK WRITE

f 1 Simplify each of the surds. f 15x 5 y 2 × 12x 2 y


4 2 2
= 15 × x × x × y × 4 × 3 × x × y
= x2 × y × 15 × x × 2 × x × 3 × y
= x2y 15x × 2x 3y

2 Multiply the coefficients and multiply = x2y × 2x 15x × 3y


the surds. = 2x3y 45xy

3 Simplify the product surd. = 2x3y 9 × 5xy


= 2x3y × 3 5xy
= 6x3y 5xy

When working with surds, we sometimes need to multiply surds by themselves; that is,
square them. Consider the following examples:
( 2 )2 = 2× 2 ( 5 )2 = 5× 5
= 4 = 25
=2 =5
We observe that squaring a surd produces the number under the radical. This is not
surprising, since squaring and taking the square root are inverse operations and, when
applied together, leave the original unchanged.

When a surd is squared, the result is the number (or expression) under the
radical; that is, ( a )2 = a , where a is a positive real number.

WORKED Example 11
Evaluate the area of a square of length ( 1--2- 28 xy ) m, expressing the answer in the
simplest form.
THINK WRITE
1 Write the rule for the area of a square. A = l2

( 1--2- 28xy )
2
2 Substitute the value for l into the rule. A=

( a )2 ( 1--2- ) × ( 28xy)
2 2
3 Simplify, using = a. =
= 1
--- × 28xy
4
= 7xy
4 Write the answer, including an A = 7xy m2
appropriate unit.
30 M a t h s Q u e s t M a t h s C Ye a r 1 1 f o r Q u e e n s l a n d

remember
remember
1. When multiplying surds, simplify the surd if possible, then apply the following
rules:
(a) a× b = ab
(b) m a × n b = mn ab , where a and b are positive real numbers.
2. When a surd is squared, the result is a number (or an expression) under the
radical: ( a )2 = a , where a is a positive real number.

1F Multiplication of surds
WORKED 1 Multiply the following surds, expressing answers in the simplest form.
Example
10a–e
a 2× 7 14 b 5 × 11 55 c 6× 7 42

d 2 × 12 2 6 e 8× 6 4 3 f 12 × 6 6 2

g 10 × 10 10 h 5 × 75 5 15 i 21 × 3 3 7

j 2 8× 5 4 10 k 27 × 3 3 27 l 45 × 60 30 3

m 5 3 × 2 11 10 33 n 6 2 × 4 48 96 6 o 10 15 × 6 3 180 5

p 9 2 × 7 2 126 q 4 20 × 3 5 120 r 6 18 × 2 8 144


1
s 10 6 × 3 8 120 3 t 9 20 × 4 15 360 3 u --- 48 × 2 2 2 6
4

1
v --- 72 × 1--- 3 6 w 1
--- 48 × 2 3 2 2--3- x 2
--- 4 × 1--- 125 4---
3
5
2 3 9 3 5

1
y ------ 60 × 1--- 40 2
---
5
6 z 3
--- 30 × 2--- 10 3 3
10 5 4 5

WORKED 2 Simplify the following expressions with surds. Assume that a, b, x and y are positive
Example
10f
real numbers.

a xy × x 3 y 2 x2 y y b x3 y4 × x2 y2 x2 y3 x

c 3a 4 b 2 × 6a 5 b 3 3a 4 b 2 2ab d 5a 2 b 3 × 10ac 5 5abc 2 2abc

e 12a 7 b × 6a 3 b 4 6a 5 b 2 2b f 18a 4 b 3 × 2a 2 b 5 6a 3 b 4

g 15x 3 y 2 × 6x 2 y 3 3x 2 y 2 10xy h 3 10x 7 y × 5x 5 y 3 15x 6 y 2 2

1 1
i --- 15a 3 b 3 × 3 3a 2 b 6 9
---
2
a 2 b 4 5ab j --- 12a 4 b 2 × 1--- 6a 3 b 3 --1-
2
a 3 b 2 2ab
2 3 4
Chapter 1 Number systems: the Real Number System 31
WORKED 3 Find the area of the following shapes. Answers must be expressed in the simplest surd
Example
form and the appropriate units specified.
11
a b c
eBook plus 2 4m
7 2 cm 5 3 cm
Digital doc:
SkillSHEET 1.3 5 11 m
Substitution using
surds 2

d e f
3 a 98 cm2
5 10 m
b 75π cm2 6 5m
3 3m 8 8m
c 20 11 m2
3 6m
d 6 6 m2 2 10 m
e ( 45 π + 96 10 ) m2 2 8m 3 6m

f 72 15 m2

4 multiple choice
The product of 3 30 × 5 6 expressed in its simplest form is:

A 15 36 B 90 C 15 180 D 45 20 E 90 5

5 multiple choice
The product of 8x 5 y 2 × 5x 6 y 3 expressed in its simplest form is:

A 2x 2 y 5 10xy B 40x 11 y 5 C 2x 5 y 2 10xy D 13x 11 y 5 E x 5 y 2 13xy

6 multiple choice
3
The product of --- x 7 y 2 × 1--- x 4 y 3 expressed in its simplest form is:
8 2
3 3 3 3 3
A ------ x5 y7 B ------ x7 y5 C ------ x 3 y 2 xy D ------ x 5 y 2 xy E ------ x 2 y 3 xy
16 16 64 16 16

7 multiple choice
The area of the triangle expressed in its simplest form is:

5 3m

4 6m

A 30 2 m2 B 60 2 m2 C 24 12 m2 D 48 3 m2 E 20 18 m2

8 The height of a square-based pyramid is 20 8 units and the length of the side of its
base is 12 8 units. Find the volume of the pyramid, expressing the answer in the
simplest surd form. (Volume = --1- area of base × height) 15 360 2
3
32 M a t h s Q u e s t M a t h s C Ye a r 1 1 f o r Q u e e n s l a n d

The Distributive Law


The Distributive Law states that a(b + c) = ab + ac.
When multiplication of surds involving brackets is required, the Distributive Law is
applied as in the case with algebraic terms. That is:

a ( b + c) = ab + ac
If there is a negative number outside the bracket, then every term inside the bracket
will undergo a sign change since it has been multiplied by the negative number.

WORKED Example 12
Expand and simplify the following where possible.
a 5 ( 6 + 11) b 7 ( 18 – 3) c – 2 3 ( 10 – 5 3)

THINK WRITE

a 1 Write the expression. a 5 ( 6 + 11)


2 Apply the Distributive Law: = 5 × 6 + 5 × 11
Multiply the term outside the bracket
with the first term inside the bracket,
then multiply the term outside the
bracket with the second term inside
the bracket.
3 Simplify. = 30 + 11 5

b 1 Write the expression. b 7 ( 18 – 3 )


2 Simplify 18 . = 7 (3 2 – 3 )
3 Apply the Distributive Law to = 7 × 3 2 + 7 × –3
remove the brackets.

4 Simplify. = 3 14 – 3 7

c 1 Write the expression. c – 2 3 ( 10 – 5 3)


2 (a) Expand the brackets, using the = – 2 3 × 10 – 2 3 × – 5 3
Distributive Law. = – 2 30 + 10 9
(b) Be sure to multiply through with
the negative.
3 Simplify. = – 2 30 + 10 × 3
= – 2 30 + 30

When expanding two binomial brackets the FOIL method is applied; that is, 4 pairs of
terms must be multiplied in the order First, Outer, Inner and Last.
Chapter 1 Number systems: the Real Number System 33
WORKED Example 13
Expand ( 5 + 3 6 )( 2 3 – 2 ) . Write your answer in its simplest form.
THINK WRITE
1 Write the expression. F L
( 5 + 3 6 ) (2 3 – 2 )
I
O
2 Apply FOIL.
Multiply the first terms of each bracket. = 5×2 3+ 5×– 2+3 6×2 3
Multiply the outer terms of each bracket.
+ 3 6×– 2
Multiply the inner terms of each bracket.
Multiply the last terms of each bracket.
3 Simplify. = 2 15 – 10 + 6 18 – 3 12
= 2 15 – 10 + 6 × 3 2 – 3 × 2 3
= 2 15 – 10 + 18 2 – 6 3

Recall the perfect square identities:


(a + b)2 = a2 + 2ab + b2
(a − b)2 = a2 − 2ab + b2

The perfect square identities can be applied to surds as follows:


( a + b) = ( a)2 + 2 a b + ( b)
2 2

= a + 2 ab + b
( b) = ( a)2 – 2 a b + ( b)
2 2
a–
= a – 2 ab + b

WORKED Example 14
Expand ( 19 – 6 ) . Write your answer in its simplest form.
2

THINK WRITE
( 19 – 6)
2
1 Write the expression.

2 Apply the perfect square identity. = ( 19) – 2 19 × 6 +


2
( 6) 2
3 Simplify. = 19 – 2 114 + 6
= 25 – 2 114

Note that the expansion of ( 19 – 6 ) in the previous example could also be done by
2

writing it as a product of two repeated factors, ( 19 – 6 )( 19 – 6 ) , and applying


FOIL. Naturally, the result would be the same, but the solution would take longer.
34 M a t h s Q u e s t M a t h s C Ye a r 1 1 f o r Q u e e n s l a n d

Recall the difference of two squares (DOTS) identity:


(a − b)(a + b) = a2 − b2
The DOTS identity can be applied to surds as follows:
( a – b )( a + b ) = ( a ) – ( b )
2 2

=a-b

WORKED Example 15
Expand ( 5 y – 3 2 x )( 5 y + 3 2 x ) .
THINK WRITE
1 Write the expression. ( 5y – 3 2x )( 5y + 3 2x )
2 Use DOTS identity for expansion. = ( 5y ) – ( 3 2x )
2 2

3 Simplify. = 5y – 9 × 2x
= 5y – 18x

In the above example the binomial factors which were multiplied together are a conju-
gate pair (that is, one bracket contains a sum and the other a difference of the same
terms). Although the terms of the factors are irrational, the answer is not a surd, but an
expression with rational terms.
The product of a conjugate pair of surds (irrational numbers) yields a rational
number.
Note that to find the product of a conjugate pair (as in Worked example 15), FOIL
could be used as an alternative to the DOTS identity. The latter, however, leads to the
answer much more quickly.

remember
remember
1. When expanding brackets, the Distributive Law is applied:
a ( b + c ) = ab + ac
2. When expanding binomial brackets, FOIL is applied:
( a + b )( c + d ) = ac + ad + bc + bd
3. Perfect square identities:
( a + b)2 = ( a) + 2 a b + ( b)
2 2

= a + 2 ab + b
( a – b) = ( a) – 2 a b + ( b)
2 2 2

= a – 2 ab + b
4. DOTS identity:
( a – b )( a + b ) = ( a ) – ( b )
2 2

=a−b
5. The product of a conjugate pair of surds is rational.
Chapter 1 Number systems: the Real Number System 35

1G The Distributive Law


WORKED 1 Expand and simplify the following, where possible. 2 5 – 10
Example 3 10 – 7 5
12
a 3( 7 + 6) b 5 ( 18 – 7 ) c 5(2 – 2) eBook plus
21 + 6 3
d 2( 3 + 5) e 7 ( 3 72 – 12 ) f 6 ( 5 14 – 4 ) Digital doc:
6 + 10 SkillSHEET 1.4

g 2 2 ( 6 18 + 7 15 ) h – 5 12 ( 3 5 – 4 8 ) –2 3 ( 4 6 – 2 3 )
Algebraic
72 + 14 30 i expansion

– 30 15 + 80 6 126 2 – 14 3 – 24 2 + 12
WORKED 2 Expand and simplify where possible. 24 3 – 18 30 – 8 10 + 60
Example 10 21 – 4 6
13 a ( 18 – 5 ) ( 5 + 3 ) b ( 7 + 5 )( 2 5 – 3 7 ) – 35 – 11

3 10 + 9 2 – 5 5 – 15 c ( 4 8 + 2 6 )( 8 – 3 6 ) – 4 – 40 3 d ( 3 6 – 2 5 )( 4 2 – 3 20 )
e ( 7 8 + 6 3 )( 4 2 – 5 6 ) f ( 11 – 2 3 )( 2 5 – 8 ) 2 55 – 2 22 – 4 15 + 4 6

g (2 7 – 3 2(5 5 + 7 2) h ( 5 18 – 3 3 )( 2 18 – 6 )180 – 30 3 – 18 6 + 9 2

15x + 26 xy + 8y i ( 5 x + 2 y )( 3 x + 4 y ) j ( 8x – 10y )( 2x + 10y ) 4x + 2 5xy – 10y


10 35 + 14 14 – 15 10 – 42 112 – 140 3 + 24 6 – 90 2
WORKED 3 Expand and simplify where possible.
Example
14 a ( 2 + 5 ) 2 27 + 10 2 b ( 6 + 10 ) 16 + 4 15 c
2
( 3 + 15 ) 2 18 + 6 5
53 + 10 6 d ( 3 + 5 2)2 e ( 8 + 3 3)
2
35 + 12 6 f ( 2 2 + 3 5 ) 2 53 + 12 10

104 + 60 3 g (3 6 + 5 2) 2
h ( 5 – 3)
2
14 – 6 5 i ( 7 – 3 ) 2 10 – 2 21
j ( 2 8 – 5 ) 37 – 8 10
2

WORKED 4 Expand and simplify the following where possible.


Example
15 a ( 3 + 7 )( 3 – 7 ) −46 b ( 19 + 1 )( 19 – 1 ) 18
c ( 2 5 + 3 )( 2 5 – 3 ) 11 d ( 3 11 + 7 )( 3 11 – 7 ) 50
e ( 8 + 2 )( 8 – 2 ) 6 f ( 10 + 12 )( 10 – 12 ) −2
g ( 13 – 3 )( 13 + 3 ) 10 h ( 7 – 12 )( 7 + 12 ) −5
i ( 2 3 – 5 )( 2 3 + 5 ) 7 j ( 3 7 + 12 )( 3 7 – 12 ) 51
k ( 2 10 + 14 )( 2 10 – 14 ) 26 l ( 18 – 19 )( 18 + 19 ) −1
m ( 13 – 6 )( 13 + 6 ) 7 n ( 3 5 + 2 7 )( 3 5 – 2 7 ) 17
o ( 6 3 – 3 5 )( 6 3 + 3 5 ) 63 p ( 5 2 – 6 )( 5 2 + 6 ) 44
q ( 7 2 – 3 5 )( 7 2 + 3 5 ) 53 r ( 11 3 + 2 5 )( 11 3 – 2 5 ) 343
s ( 6 3 + 2 8 )( 6 3 – 2 8 ) 76 t ( 7 2 – 3 9 )( 7 2 + 3 9 ) 17
u ( x – y )( x + y ) x − y v ( 2x – 3y )( 2x + 3y ) 2x − 3y
w ( 3 x – 4 y )( 3 x + 4 y ) 9x − 16y x ( 2x x + 5 y )( 2x x – 5 y ) 4x − 25y 3

xy(49x − 9y) y ( 7x y – 3y x )( 7x y + 3y x ) z ( 9 x 2 y – 5 xy 2 )( 9 x 2 y + 5 xy 2 ) xy(81x − 25y)


36 M a t h s Q u e s t M a t h s C Ye a r 1 1 f o r Q u e e n s l a n d

5 multiple choice
When expressed in its simplest form, 15 ( 5 – 3 ) is equal to:
A 5 3–3 5 B 5 5–3 3 C 75 – 45
D 30 E 2 2

6 multiple choice
When expressed in its simplest form, ( 5 8 + 2 7 )( 6 5 – 3 3 ) is equal to:
A 30 40 – 15 24 + 12 35 – 6 21
B 60 10 – 15 24 + 12 35 – 6 21
C 60 10 – 30 6 + 12 35 – 6 21
D 30 40 – 30 6 + 12 35 – 6 21
E 60 10 – 30 6 + 12 21

7 multiple choice
When expressed in its simplest form, ( 7 5 – 2 3 ) is equal to:
2

A 49 25 – 4 9 B 245 C 257 + 28 15
D 269 E 257 – 28 15

8 multiple choice
When expressed in its simplest form, ( 15 x 2 y + 4 xy )( 15 x 2 y – 4 xy ) is equal to:
A 225x 2 y – 120 xy + 16xy
B 15x 2 y – 4xy
C 225 x 4 y 2 – 16 x 2 y 2
D 225x 2 y – 16xy
E 225x 2 y – 120xy x + 16xy

9 Given that x = 3 5 – 2 3, find:


a x 2 57 – 12 15
b x 2 + 3x + 2 59 – 12 15 + 9 5 – 6 3

Division of surds
To divide surds, divide the expressions under the radicals;
a a
that is, ------- = --- , where a and b are positive real numbers.
b b
When dividing surds it is best to simplify them (if possible) first. Once this has been
done, the coefficients are divided next and then the surds are divided.
Chapter 1 Number systems: the Real Number System 37
WORKED Example 16
Divide the following surds, expressing answers in the simplest form. Assume that x and y
are positive real numbers.

55 48 9 88 36 xy
a ---------- b ---------- c ------------- d ------------------------
5 3 6 99 25 x 9 y 11
THINK WRITE
a a 55 55
a 1 Rewrite the fraction, using ------- = --- . a ---------- = ------
b b 5 5
2 Divide numerator by the denominator = 11
(that is, 55 by 5).
3 Check if the surd can be simplified any
further.

a a 48 48
b 1 Rewrite the fraction, using ------- = --- . b ---------- = ------
b b 3 3
2 Divide 48 by 3. = 16
3 Evaluate 16 . =4

a a 9 88 9 88
c 1 Rewrite the surds, using ------- = --- . c ------------- = --- ------
b b 6 99 6 99
9 8
2 Simplify the fraction under the radical = --- ---
by dividing both numerator and 6 9
denominator by 11.
9×2 2
3 Simplify the surds. = -------------------
6×3
18 2
4 Multiply the whole numbers in the = -------------
numerator together and those in the 18
denominator together.
5 Cancel the common factor of 18. = 2

36xy 6 xy
d 1 Simplify each surd. d ----------------------- = --------------------------------------------
25x 9 y 11 5 x 8 × x × y 10 × y
6 xy
= ------------------------
5x 4 y 5 xy
6
2 Cancel any common factors — in this = --------------
case xy . 5x 4 y 5
38 M a t h s Q u e s t M a t h s C Ye a r 1 1 f o r Q u e e n s l a n d

WORKED Example 17
Find the perpendicular height of a triangle, given that its area is 27 15 cm2 and its base
length is 6 3 cm. The answer must be expressed in the simplest surd form and the
appropriate unit specified.

THINK WRITE

1 Write the rule for the area of a triangle. A = 1--- bh


2
2 Substitute the values for A and b into 1
the rule. 27 15 = --- ×6 3×h
2

3 Cancel the 2 and the 6. 27 15 = 3 3 × h

4 Transpose the equation to make h the 27 15


h = ----------------
subject.
3 3

5 Divide numerator and denominator by 9 5× 3


3 (cancel down). = ------------------------
3
6 Simplify and write the answer, using
the appropriate unit. h = 9 5 cm

remember
remember
When dividing surds, simplify the surd if possible, then apply the following rule:
a a
a ÷
b = ------- = ---
b b
where a and b are positive real numbers.

1H Division of surds
WORKED 1 Simplify the following surds, expressing answers in the simplest form. Assume that x
Example
xample
16
and y are positive real numbers.
15 14 8 72
a ---------- 5 b ---------- 7 c ------- 2 d ---------- 2 3
3 2 2 6
60 90 128 45 3---
e ---------- 6 f ---------- 15 g ------------- 4 h ------------- 5
10 6 8 125
18 2 24 4---------2- 65 5 5 72
i ---------- ------3- j ------------- k ------------- ------- l ------------- 5 6
4 3 2
4 6 3 3 2 13 12
Chapter 1 Number systems: the Real Number System 39
96 2 63 1 7 44 336
m ---------- 2 3 n ------------- 1 --5- o ---------------- 1 p ------------- 2 6
8 5 7 14 11 14
9 63 4 540 2040 12 99 2
q ------------- 1 --5- r ------------- 3 3 s ---------------- 2 17 t ---------------- 2 --5-
15 7 20 30 15 11
x4 y3 x x 6 y 11 1 16xy 2 72x 4 y 3
u --------------- -- v ------------------- ---------- w ------------------ ---------- x --------------------- 6x xy
y x 3 y2 x3 y4
x2 y5 x 12 y 15 8x 7 y 9 2xy 2

2 Simplify the following. Assume that all pronumerals are positive real numbers.

xy 12x 8 y 12 6x 2 y 3 3x 7 y 2 2 2a 2 b 4 10a 9 b 3 4 a
2xy 3y a --------------- × ----------------------- b --------------------- × ------------------ c ---------------------- × --------------------- ----------
3
x5 y7 x2 y3 27x 4 y 4 3xy 3 5a 3 b 6 3 a7b

3b 2 2b 3ab 5 6a 7 b 3 2mn 3 3m 4 n 6 5 3m 3 n 2m 5 n 8 15
------------------
- d ------------------ × -------------- e -------------------- ÷ -------------------- f ----------------------- ÷ -------------------- ---------------
2a a 2a 5b 2
2m 2 n 2
2a b 6 2 2x 6m 5 n 2 8mn 3 2 6m 3 n 2 6 mn 5
------------
3y 2 2
----------------------
3m 3 n m
3 multiple choice
75
Expressed in its simplest form, ---------- is:
5
5 3
A 70 B 15 C 13 D ---------- E 17
5

4 multiple choice
9 18
Expressed in its simplest form, ------------- is:
21 3
9 6 54 9 2 9 6 3 6
A ---------- B ---------- C ---------- D ---------- E ----------
21 7 7 3 7 7

5 multiple choice
10x 5 y 8
Expressed in its simplest form, --------------------- is:
20x 3 y 2
xy 3 2 xy 3 10 x3 y 2 x 2 y 4 10 x2 y6
A ---------------- B ------------------- C ---------------- D --------------------- E --------------
-
2 20 2 xy 20x 2

6 multiple choice
2x 4 y 6x 7 y 3
Expressed in its simplest form, ---------------- × ------------------ is:
9xy 4x 3 y 5
y3 y x 12 x x 3 3x x7 2x 3 3x
A ------------ B -------------- C ---------------- D ---------- E -------------------
x 3 y 3 3y y 3 6y
40 M a t h s Q u e s t M a t h s C Ye a r 1 1 f o r Q u e e n s l a n d

WORKED 7 Find the length of the unknown side in each of the following. Answers must be
Example
expressed in the simplest surd form and the appropriate units specified.
17

a b c
7 a 4 13 m A = 28 39 m2 A = 12 30 cm2 A = 21 55 m2

b 4 6 cm
w 3 5 cm h
c 7 11 m
d 3 7m
7 3m b
e 5 13 cm
6 5m
15
f ------
2
5 cm
d e f
V = 90 21 m3 V = 315π 13 cm3 V = 60π 75 cm3

h
h
h
3 6m 3 7 cm
5 2m Area of base = 24π 15 cm2

2E
8 Velocity v of the object can be found using the formula v = ------- , where E is the kinetic
m
energy of the object and m is the mass of the object. Express v as the simplest surd, if:
a E = 80 J, m = 2 kg 4 5
5-
------ b E = 250 J, m = 60 kg
3
c E = 480 J, m = 120 kg 2 2

2
9 A rectangular fish tank has a base 20 3 cm by 30 6 cm and the height h. When --- of
3
the tank is filled, the volume of water is 84 L. Find:
a the height of the tank (give the answer as the simplest surd) 35 2 cm
b the full capacity of the tank in litres. 126 L
(Remember that 1 cubic centimetre holds 1 mL of water.)

Rationalising denominators
If the denominator of a fraction is a surd, it can be changed into a rational number.
In other words, it can be rationalised.
As we discussed earlier in this chapter, squaring a surd (that is, multiplying it by
itself) results in a rational number. This fact can be used to rationalise denominators as
follows.
a b ab b
------- ¥ ------- = ---------- , (where ------- = 1)
b b b b
If both numerator and denominator of a fraction are multiplied by the surd contained
in the denominator, the denominator becomes a rational number. The fraction takes on
a different appearance, but its numerical value is unchanged, because multiplying the
numerator and denominator by the same number is equivalent to multiplying by 1.
Chapter 1 Number systems: the Real Number System 41
WORKED Example 18
Express the following in their simplest form with a rational denominator.
6 2 12 17 – 3 14
a ---------- b ------------- c -----------------------------
13 3 54 7
THINK WRITE
6
a 1 Write the fraction. a ----------
13
2 Multiply both the numerator and denominator by
6 13
the surd contained in the denominator (in this = ---------- × ----------
case 13 ). This has the same effect as multiplying 13 13
13 78
the fraction by 1, since ---------- = 1 . = ----------
13 13
2 12
b 1 Write the fraction. b -------------
3 54

Simplify the surds. (This avoids dealing with large 2 12 2 4 × 3


2 ------------- = -------------------
numbers.) 3 54 3 9 × 6
2×2 3
= -------------------
3×3 6
4 3
= ----------
9 6
4 3 6
Multiply both the numerator and denominator by 6. = ---------- × -------
3
9 6 6
4 18
(This has the same effect as multiplying the fraction by = -------------
9×6
6
1 since ------- = 1.)
6
Note: We need to multiply only by the surd part of the
denominator (that is, by 6 , rather than by 9 6 ).
4 9×2
4 Simplify 18 . = -------------------
9×6
4×3 2
= -------------------
54
12 2
= -------------
54
2 2
5 Divide numerator and denominator by 6 = ----------
(cancel down). 9
Continued over page
42 M a t h s Q u e s t M a t h s C Ye a r 1 1 f o r Q u e e n s l a n d

THINK WRITE

17 – 3 14
c 1 Write the fraction. c -----------------------------
7
( 17 – 3 14 ) 7
2 Multiply both the numerator and denominator by 7 . = ------------------------------------ × -------
Use brackets so you realise the whole numerator must 7 7
be multiplied by 7 .
17 × 7 – 3 14 × 7
3 Apply the Distributive Law in the numerator. = ----------------------------------------------------------
a(b + c) = ab + ac 7× 7
119 – 3 98
= --------------------------------
49
119 – 3 49 × 2
4 Simplify 98 . = ------------------------------------------
7
119 – 3 × 7 2
= ---------------------------------------
7
119 – 21 2
= --------------------------------
7

remember
remember
To rationalise the surd denominator, multiply the numerator and denominator by
the surd contained in the denominator. This has the effect of multiplying the
fraction by 1 and thus the numerical value of the fraction remains unchanged,
while the denominator becomes rational:
a a b ab
------- = ------- × ------- = ----------
b b b b

1I Rationalising denominators
WORKED 1 Express the following in their simplest form with a rational denominator.
Example
18a, b 5 5 2 7 7 3 4 4 11 8 4 6 12 2 21
a ------- ---------- b ------- ---------
3
- c ---------- ------------
11 d -------
- ---------- e ---------
3 - -------------
2 7
2 3 11 6 7
15 10 2 3 2 15 3 7 3 35 5 2 5 6 4 3 4 15
-------------
f ---------- ----------
2
g ---------- ------------- h ---------- ------------- i ---------- ---------- j
6
---------- 15
5 5
6 5 5 2 3 3 5
5 14 5 7 16 3 8 3 8 21 8 60 2 35 10
k ------------- ---------- l ------------- m ---------- ------------- n
49 ------------- o ------------- ----------
14 3
7 8 6 5 7 7 28 3 14
8 15 8 105
------------- ----------------
15 7
Chapter 1 Number systems: the Real Number System 43
WORKED 2 Express the following in their simplest form with a rational denominator.
Example
18c 6 + 12 15 – 22 6 2 – 15 2 18 + 3 2 9 10
a ----------------------- b -------------------------- c -------------------------- d ------------------------------ -------------
5
2+2 3 6 10 12 5–5 6 5
------------------------------
10
3 10 + 6 14 3 5+6 7 4 2 + 3 8 5---------6- 3 11 – 4 5 2 7–2 5 21 – 15
--------------------------------- e --------------------------- f --------------------------- 3 g ------------------------------ h --------------------------- --------------------------
4 3
8 2 3 18 12
14 – 5 2 7 12 – 5 6 6 2– 5 6 3–5 5 6 15 – 25
----------------------
6 i ------------------------------ j ----------------------- k --------------------------- -------------------------
3 22 – 4 10
70 ---------------------------------
6 3 4 8 12 – 10 7 20 6
3 10 – 2 33 ----------------------
---------------------------------
6 16
3 multiple choice
12
When expressed in its simplest form, ------- is equal to:
3
4 3 C 48 12 3
A ---------- B 4 3 D ------------- E 6 3
3 3

4 multiple choice
8 5
When expressed in its simplest form, ------------- is equal to:
9 12
40 16 15 4 5 4 15 320
A --------- B ---------------- C ---------- D ------------- E ---------
108 108 9 27 972

5 multiple choice
7 5–6 7
When expressed in its simplest form, --------------------------- is equal to:
12
6 7 15 – 6 21
A ------- B 7 15 – 21 C ---------------------------------
6 6
7 60 – 6 84
D --------------------------------- E none of these
12

6 multiple choice
5 5–3 3
When expressed in its simplest form, --------------------------- is equal to:
8 8
5 10 – 3 6 80 10 – 48 6
A ------------------------------ B 10 10 – 6 6 C ------------------------------------
32 64
40 40 – 24 24 10 10 – 6 6
D --------------------------------------- E ---------------------------------
64 8

7 Solve for x, giving the answers as the simplest surds with rational denominators:
21 15 2 6
a x2 = 3
--- ± ---------- b 3x2 = 5 ± ---------- c 6x2 − 4 = 12 ± ----------
7 7 3 3
44 M a t h s Q u e s t M a t h s C Ye a r 1 1 f o r Q u e e n s l a n d

Rationalising denominators using


conjugate surds
As shown earlier in the chapter, the product of pairs of conjugate surds results in a
rational number. (Examples of pairs of conjugate surds include 6 + 11 and 6 – 11 ,

a + b and a – b , 2 5 – 7 and 2 5 + 7 .)
This fact is used to rationalise denominators containing a sum or a difference of surds.

To rationalise the denominator which contains a sum or a difference of surds, we


multiply both numerator and denominator by the conjugate of the denominator.

Two examples are given below:


1 a– b
1. To rationalise the denominator of the fraction -------------------- , multiply it by -------------------- .
a+ b a– b

1 a+ b
2. To rationalise the denominator of the fraction -------------------- , multiply it by -------------------- .
a– b a+ b
A quick way to simplify the denominator is to use the DOTS identity:

( a – b )( a + b ) = ( a ) – ( b )
2 2

=a−b

WORKED Example 19
Rationalise the denominator and simplify the following.
1 6+3 2
a ---------------- b ------------------------
4– 3 3+ 3
THINK WRITE
1
a 1 Write the fraction. a ----------------
4– 3
1 (4 + 3)
2 Multiply the numerator and denominator = --------------------- × ---------------------
by the conjugate of the denominator. (4 – 3) (4 + 3)
 4 + 3
(Note that  ---------------- = 1 .)
 4 + 3
4+ 3
3 Apply the Distributive Law in the numerator = ------------------------------
2
and the DOTS identity in the denominator. ( 4 )2 – ( 3 )
4+ 3
4 Simplify. = ----------------
16 – 3
4+ 3
= ----------------
13
Chapter 1 Number systems: the Real Number System 45
THINK WRITE

6+3 2
b 1 Write the fraction. b ------------------------
3+ 3
( 6 + 3 2) (3 – 3)
2 Multiply the numerator and = ----------------------------- × ---------------------
denominator by the conjugate of (3 + 3) (3 – 3)
the denominator.
 3 – 3
(Note that  ---------------- = 1 .)
 3 – 3
6×3+ 6×– 3+3 2×3+3 2×– 3
3 Apply FOIL in the numerator and = -----------------------------------------------------------------------------------------------------------
2
DOTS in the denominator. ( 3 )2 – ( 3 )

3 6 – 18 + 9 2 – 3 6
4 Simplify. = ------------------------------------------------------------
9–3
– 18 + 9 2
= -------------------------------
6
– 9×2+9 2
= -------------------------------------
6
–3 2+9 2
= -------------------------------
6
6 2
= ----------
6
= 2

You might wish to use a calculator to check if the final answer is correct. To do that,
evaluate the original fraction and the final one (the one with the rational denominator)
and check whether they both equal the same number.

WORKED Example 20
1 1
Rationalise the denominators and simplify the following. ----------------------- + ---------------------------
2 6– 3 3 6+2 3
THINK WRITE
1 We will rationalise the denominator of 1
each term and then add them. -----------------------
Write the first fraction. 2 6– 3

1 (2 6 + 3)
2 Multiply the numerator and denominator by = ---------------------------- × -----------------------------
the conjugate of the denominator. (2 6 – 3) (2 6 + 3)
Continued over page
46 M a t h s Q u e s t M a t h s C Ye a r 1 1 f o r Q u e e n s l a n d

THINK WRITE
2 6+ 3
3 Apply the Distributive Law in the numerator = ----------------------------
and DOTS in the denominator. (Note that ( 2 )2 × 6 – 3
when squaring 2 6 , we need to square both 2
and 6 .)
2 6+ 3
4 Simplify the denominator. = ------------------------
21
1
5 Write the second fraction. ---------------------------
3 6+2 3
1 (3 6 – 2 3)
6 Multiply the numerator and denominator = -------------------------------- × -------------------------------
by the conjugate of the denominator. (3 6 + 2 3) (3 6 – 2 3)
3 6–2 3
7 Apply the Distributive Law in the = -----------------------------------
numerator and DOTS in the denominator. 3 × 6 – 22 × 3
2

3 6–2 3
8 Simplify the denominator. = ---------------------------
42
2 6+ 3 3 6–2 3
9 Add the two fractions together. ------------------------ + ---------------------------
Bring them to the lowest common 21 42
denominator first. 2 6+ 3 2 3 6–2 3
= ------------------------ ×  --- + ---------------------------
21  2 42

4 6+2 3 3 6–2 3
= --------------------------- + ---------------------------
42 42
7 6
10 Add the numerators. = ----------
42
6
11 Simplify where appropriate. = -------
6

The following worked example demonstrates the rationalisation of the denominator


when it is a trinomial (has three terms).

WORKED Example 21
1
Simplify ----------------------------- .
2+ 2– 3
THINK WRITE
1
1 Use a set of brackets to group the ----------------------------------
trinomial into a binomial. (2 + 2) – 3
Chapter 1 Number systems: the Real Number System 47
THINK WRITE

1 [(2 + 2) + 3]
2 Multiply the numerator and denominator = ---------------------------------- × ---------------------------------------
by the conjugate of the denominator; that (2 + 2) – 3 (2 + 2) + 3
is, (2 + 2 ) + 3 . Use brackets around
both factors so that you will recognise
that all terms need to be multiplied.
(2 + 2) + 3
3 Use FOIL to expand the denominator. = -----------------------------------
2
(2 + 2) – 3
(2 + 2) + 3
4 Expand the squared terms of the = --------------------------------------
denominator. 4+4 2+2–3
2+ 2+ 3
5 Group and simplify the denominator. = ------------------------------
3+4 2
(2 + 2 + 3) (3 – 4 2)
6 Rationalise the denominator as shown = ----------------------------------- × ------------------------
previously. Use brackets as in step 2. (3 + 4 2) (3 – 4 2)
6–8 2+3 2–4 4+3 3–4 6
7 Expand the numerator, making sure that = --------------------------------------------------------------------------------------
every term in the first set of brackets is 9 – 16 × 2
multiplied by every term in the second set.
6–8–8 2+3 2+3 3–4 6
8 Group like terms and simplify. = -------------------------------------------------------------------------------
9 – 32
–2 – 5 2 + 3 3 – 4 6
= --------------------------------------------------------
–23
2+5 2–3 3+4 6
9 Multiply numerator and denominator = -----------------------------------------------------
by –1 to eliminate the negative 23
denominator.

remember
remember
1. To rationalise the denominator containing a sum or a difference of surds,
multiply both the numerator and denominator of the fraction by the conjugate
of the denominator. This eliminates the middle terms and leaves a rational
number.
2. To simplify the denominator quickly, use the DOTS identity:
( a – b )( a + b ) = ( a)2 – ( b)2
= a–b
1 a– b
3. To rationalise the denominator of the fraction -------------------- , multiply it by -------------------- .
a+ b a– b
1 a+ b
4. To rationalise the denominator of the fraction -------------------- , multiply it by -------------------- .
a– b a+ b
5 14 + 2 10 – 25 7 – 10 5
48 M a t h s Q u e s t M a t h s C Ye a r 1 1 f o r Q u e e n s l a n d
1 i ------------------------------------------------------------------------
155
– ( 10 3 + 15 6 + 9 2 + 27 )
-
1 p ---------------------------------------------------------------------

– 20 2 + 9 10 + 4 30 – 9 6
m --------------------------------------------------------------------------
2
Rationalising denominators
1J
42

using conjugate surds


2 2+ 5
WORKED 1 Rationalise the denominator and simplify. ------------------------
Example 3
19 1 1 3+ 6 1 1 2 6+ 7
a ---------------- 5–2 b ---------------- ---------------- c -------------------- d ----------------------- ------------------------
3 17
5+2 3– 6 8– 5 2 6– 7
eBook plus
4 7 5 3 9 3 9 11 + 9
Digital docs: e ----------------------------- f ------------------------------ g --------------------------- h ----------------------------- ----------------------
20
SkillSHEET 1.5 2 11 – 13 2 12 + 2 5 3 5+4 2 2 33 – 12
Conjugate pairs
SkillSHEET 1.6
12 2 – 17
2–5 8–3 12 – 7 11 + 7 9 2 + 154
Applying DOTS to
expressions
i -------------------------- j ---------------- k ----------------------- l -------------------------- ------------------------------
4
with surds 5 7 – 20 8+3 12 + 7 22 – 14
3
-------
1 12
8 11 + 4 13 5– 3 2 8–3 2 3 6 + 2 12 5 2+3 3
e ---------------------------------
31 m --------------------------------- n ------------------------------ o ------------------------------ p ------------------------------
2 21 – 35 4 10 + 3 18 3 24 – 2 6 4 18 + 3 8 2 6 – 3 12
f -----------------------------
14
15 15 – 20 6 4 5 + 10 4 15 – 2 3 2 7+5 3 2 11 – 3 3 71 – 12 33
g ------------------------------------
13 q ------------------------------ r ------------------------------ s --------------------------- t ------------------------------ ----------------------------
17
19 – 4 21 6 15 + 20 2 30 – 5 2 5 7–3 3 2 11 + 3 3
k -------------------------
5
3 3+2 6
o --------------------------- 4 12 – 3 8 3 8+6 3 3 11 – 2 7 4 15 + 2 5 7 3+9
18 u ------------------------------ v --------------------------- w --------------------------------- x ------------------------------ -------------------
3
115 + 31 21 3 6–5 2 7 2– 3 3 14 + 4 11 3 5 – 15
s -------------------------------
148 – 9 154 + 132 + 42 2 – 8 77
---------------------------------------------------------------------------
u 18 2 + 10 6 – 9 3 – 15 3 7–5 2 3 6 – 15 – 6 + 6 2 + 10 – 2 5 50
y --------------------------- z -------------------------- ---------------------------------------------------------
2
102 + 48 6 35 + 2 2 6+2 3 12 3 – 4 + 3 6 – 2
v ---------------------------- 1 q ---------------------------------------------------
52
-
95 21 5 – 6 14 – 5 70 – 20
------------------------------------------------------------------
27 r 60 2 + 10 30 – 6 10 – 5 6
WORKED 2 Rationalise the denominator and simplify. ------------------------------------------------------------------------
35
Example
20 1 1 9 2+8 1 1 9 7 – 13 3
a ---------------- + ------------------- ------------------- b --------------------------- – ------------------------ ------------------------------
14 120
8–2 2 8–2 2 7+2 3 3 7+ 3

16 210 – 12 14 3 7 4 8 2 3 4 6+ 3
------------------------------------------ c ----------------------- × ------------------------ d ----------------------- × --------------------------- 6–7 2
77
3 5– 3 5+3 3 6–2 3 2 6+3 3

3 5 7–2 2 2 2+ 3 2 2+ 3 66 + 24 6
e -------------------- ÷ ----------------------- f ------------------------ ÷ ------------------------------ -------------------------
5
7+ 2 5+ 2 2 2 – 3 12 2 + 6 3
– ( 45 + 15 14 + 9 10 + 6 35 )
----------------------------------------------------------------------------
5
7+ 8 2 8–2 7 3 7+2 5 7–2
g --------------------------- + --------------------------- 5 – 4 14 h -------------------------- + ---------------------------
3 7–3 8 3 8+3 7 2 7 – 11 7 + 2 11

3 + 7 65 – 16 11 13 + 5 11 + 2 5+ 6 2 6–2 5 – ( 41 + 6 30 )
--------------------------------------------- i ----------------------- – ------------------- j --------------------------- – --------------------------- ---------------------------------
28 12
13 – 5 11 – 2 4 5–4 6 3 6–3 5
959 + 281 77 + 182 7 + 6 11
-----------------------------------------------------------------------------
629
Chapter 1 Number systems: the Real Number System 49
WORKED 3 Rationalise the denominator and simplify.
Example
21 2+ 3– 5 5– 3– 2 –6 + 2 15 + 3 10 – 5 6
a ------------------------------------ b ---------------------------------- --------------------------------------------------------------
6
3+2 3+2 5 5+ 3+ 2
230 + 257 3 – 137 5 – 80 15
4 multiple choice ----------------------------------------------------------------------------
431
-

1
If x = 7 + 11 , then x + --- when simplified with a rational denominator is equal to:
x
3 7 + 5 11 5 7 + 3 11 5 7 + 3 11
A ------------------------------ B ------------------------------ C ------------------------------
4 4 –4
3 7 + 5 11 3 7 + 3 11
D ------------------------------ E ------------------------------
–4 4
7–3 5
5 Given that x = ------------------------ find each of the following, giving the answer in surd form
7+3 5
with a rational denominator:
1 1 2 1
a x + --- −2 -----
14-
19
b x – --- 6------------
35
- c x – ----- – 312 35
-------------------
x x 19 x2 361

6 Given that x = 5 2 – 3 find each of the following, giving the answer in surd form
with a rational denominator:
1 1 1
a x + --- 210 2 – 120
------------------------------- b x – --- 200 2 – 126
------------------------------- c x 2 + ----- 99 238 – 50 400 2
---------------------------------------------
x 41 x 41 x2 1681

99 120 – 50 460 2 1
--------------------------------------------- d x 2 – ----- e x 2 + 6x + 3 44 f x 2 – 12x + 8 103 – 90 2
1681 x2
x 2 + 3x x 2 – 3x 295 2 – 382
g ----------------- 7 2 + 4 h ----------------- -------------------------------
x–2 x+2 49

7 Is x = 5 + 3 2 a solution for the equation x 2 – 10x + 7 = 0 ? Show all working. Yes

8 Solve for x giving answers in surd form with rational denominators.


11 + 5 42 5 + 28 7
----------------------- a 11x + 7 = 8 + 5x b 3 5x – 10 = 2 7x + 4 ---------------------------------
6 17

Further properties of real


numbers — modulus
The modulus or absolute value of a number is the magnitude of that number. It rep-
resents the distance of the number from the origin (that is, 0 on a number line). The
modulus of x is denoted by |x| and is always positive.
Note: Do not confuse the modulus of a number with modular arithmetic (see page 17).
For example,
|−2| = 2
|2| = 2
|0| = 0
50 M a t h s Q u e s t M a t h s C Ye a r 1 1 f o r Q u e e n s l a n d

WORKED Example 22
Evaluate the following where a, b, c, d ≥ 0.
– 4cd ¥ – – 6cd
a – 50 b –6 ¥ – 3 c – 6a 2 b ¥ – 2a 3 b d ----------------------------------------
– 12
THINK WRITE
a 1 Write the expression. a – 50
2 The modulus sign indicates that we want only the = 50
magnitude of a number and not the sign of it. So the
negative in front of the number should be omitted.
b 1 Write the expression. b –6 × – 3
2 Evaluate each modulus separately and then simplify. = 6 × −3
= −18
c 1 Write the expression. c 6a 2 b × – 2a 3 b
2 Evaluate each modulus separately, then simplify. = 6a 2 b × 2a 3 b
= 12a 5 b 2
– 4cd × – – 6cd
d 1 Write the expression. d ----------------------------------------
– 12
4cd × – 6cd
2 Evaluate each modulus separately, then simplify. = ----------------------------
– 12
– 24c 2 d 2
= --------------------
– 12
= 2c 2 d 2

remember
remember
1. The modulus (or absolute value) of a number is the magnitude of that number.
It tells us how far the number is from zero, and is always positive.
2. The modulus of x is denoted by |x|.

Further properties of real


1K numbers — modulus
WORKED 1 Evaluate the following where a, b, c, d ≥ 0:
Example
22 a 19 19 b 1--- 1--4- c 0.75 0.75
4
d – 15 15 e –8 8 f – 2a 2a
1
g 3 – 4 12 h – --- 1--2- i – 3.21 3.21
2
2
j 0 0 k – – --- − 2--3- l –7 – –3 4
3
Chapter 1 Number systems: the Real Number System 51
m –2 + –8 10 n – 2 – 8 10 o –a 2 b 2 a2b2
p – 21 – – 37 −16 q –3 × –9 27 r 5 × – 3 15
1
s 12 × – – 6 −72 t – – 18 × – 3 −54 u 7 ÷ – 14 ---
2

v – 24 ÷ – 3 −8 w – 6 – 3 – 20 −11 x 9 × 10 ÷ – 3 30
−3a y – 2a × – 6b ÷ – 4 – b z 4cd × 3cd ÷ – 2 cd −6cd

2 multiple choice
When simplified, – 2 × 3 becomes:
A –6 B −6 C 6 D –1 E −5

3 multiple choice
When simplified, – ab 2 × a 3 b ÷ – a 2 b 4 becomes:
a2 a4b3 a a a2
A ----- B – ----------- C – ----- D ----- E – -----
b 2
a b 4 b 2 b2 b

4 multiple choice
When simplified, – – 8 × – 2 + 5 – 3 becomes:
A −8 B 14 C −18 D −14 E 8

5 multiple choice
When simplified, – 6 2 × 2 – 5 ÷ – 8 becomes:
A 15
------ B 9
--- C − 15
------ D − 9--- E − 9---
8 2 8 2 8

6 a Fill in the table below for the function y = 2x − 4.

x –2 –1 0 1 2 3 4 5 6
y –8 –6 –4 –2 0 2 4 6 8

|y| 8 6 4 2 0 2 4 6 8

b Use the table to plot (on the same set of axes) the graph of y = 2x − 4 and
b y = |2x – 4|
y y = 2x – 4 y = 2x – 4 .
4 c State the range of each of the two functions. R and y ≥ 0
d Compare the ranges of the two functions and their graphs. Explain the difference.
x
2
–4

Solving equations using absolute values


If | x | = 3 then, by definition of absolute values, there are 2 values of x that satisfy this
equation that states that x is three units from 0.
That is, x = 3 or x = −3
3 units 3 units

–4 –3 –2 –1 0 1 2 3 4
52 M a t h s Q u e s t M a t h s C Ye a r 1 1 f o r Q u e e n s l a n d

Therefore, two separate cases need to be considered when solving equations


involving absolute values.
In general:
Case 1: | x | = x when x > 0 (that is, when x is positive)
Case 2: | x | = −x when x < 0 (that is, when x is negative)
(Note: There is also a trivial case of | x | = 0 when x = 0.)
So for | x | = 3, the two cases to consider are x = 3 and −x = 3 giving a solution of x = 3
or x = −3.

WORKED Example 23
Solve: a | 4x | = 16 b | 4 - 3x | = 3.
THINK WRITE
a 1 Write the equation. a | 4x | = 16
2 Remove the absolute value symbols Case 1: Case 2:
and write the positive (+ve) and 4x = 16 or −4x = 16
negative (−ve) cases to be −4x = −16
considered.
3 Work the two cases side by side. x=4 x = −4
4 Verify your solution by substituting Check:
into the original expression. Start Using x = 4 Using x = −4
with the left-hand side (LHS) and LHS = | 4 × 4| LHS = | 4 × −4|
ensure that it equals the right-hand = | 16| = | −16|
side (RHS). = 16 = 16
= RHS = RHS
Solutions are correct for both cases.
5 State the solution. x can equal either The solution is x = ±4.
4 or −4, written ±4.

b 1 Write the equation. b | 4 − 3x | = 3


2 Remove the absolute value symbols Case 1: Case 2:
and write the +ve and –ve cases. 4 − 3x = 3 or −(4 − 3x) = 3
3 Solve for x in both cases. 4 − 3 = 3x −4 + 3x = 3
1 = 3x 3x = 7
x = 1--- x = 2 1---
3 3
4 Verify your solution by substituting Check:
into the original expression. Start Using x = 1--- Using x = 2 1---
3 3
with the LHS and ensure that it LHS = | 4 − 3 × 1--- | LHS = | 4 − 3 × 7
--- |
3 3
equals the RHS. = |4 − 1| = |4 − 7|
=3 = |−3|
= RHS =3
= RHS
Solutions are correct for both cases.
5 State the solution. The solution is x = 1--- or x = 2 1--- .
3 3
Chapter 1 Number systems: the Real Number System 53
WORKED Example 24
Solve | x - 3 | = 3x + 8.
THINK WRITE
1 Write the equation. | x – 3 | = 3x + 8
2 Remove the absolute value Case 1: Case 2:
symbols and write the +ve and x – 3 = 3x + 8 or –(x – 3) = 3x + 8
–ve cases. Use brackets for the
LHS of the –ve case.
3 Solve for x. −3 – 8 = 3x – x −x + 3 = 3x + 8
−11 = 2x 3 − 8 = 3x + x
x = −5 1--- −5 = 4x
2
x = −1 1---
4
4 Verify your solution for both Check:
cases by substituting into the Using x = −5 1---
2
original equation. As the RHS – 11
LHS = | −5 1--- − 3 | RHS = 3 × --------- + 8
should always be +ve, the 2 2
– 33
solution x = −5 1--- is not suitable = | −8 1--- | = --------- + 8
2 2 2
and should be ignored.
Notice how important this = 8 1--- = −16 1--- + 8
2 2
verification step is. We have = −8 1---
2
followed all the correct steps but (Not the correct solution since LHS ≠ RHS)
logically arrived at an answer that Using x = −1 1---
is not possible. 4
–5
LHS = | −1 1--- − 3 | RHS = 3 × ------ + 8
Verify all results for these 4 4
questions. – 15
= | −4 --- |
1
= --------- + 8
4 4
= 4 1--- = −3 3--- + 8
4 4
= 4 1---
4
(Correct solution since LHS = RHS)
5 State the solution. The solution is x = −1 1--- .
4

WORKED Example 25
Solve | x – 1 | = | 2x + 3 |.
THINK WRITE
1 Write the equation. | x – 1 | = | 2x + 3 |
2 Remove all absolute value symbols Case 1: Case 2:
and write the +ve and –ve cases. x − 1 = 2x + 3 or x − 1 = −(2x + 3)
Reassure yourself that there are only
two possible cases.
−(x – 1) = −(2x + 3) is the same as
(x – 1) = (2x + 3) and
(x – 1) = −(2x + 3) is the same as
−(x – 1) = (2x + 3) Continued over page
54 M a t h s Q u e s t M a t h s C Ye a r 1 1 f o r Q u e e n s l a n d

THINK WRITE
3 Solve for x for both cases. −1 − 3 = 2x − x x − 1 = −2x − 3
−4 = x x + 2x = −3 + 1
x = −4 3x = −2
2
x = − ---
4 Verify the solutions with respect to Check: 3
the original equation. Using x = −4
LHS = | −4 −1 | RHS = | 2(−4) + 3 |
= | −5 | = | −8 + 3 |
=5 = | −5 |
=5
(Correct solution since LHS = RHS)
2
Using x = − ---
3
2 2
LHS = – --- – 1 RHS = 2 × – --- + 3
3 3
= | −1 --- |
2
= | −1 --- + 3 |
1
3 3
= 1 2--- = | 1 2--- |
3 3
= 1 2---
3
(Correct solution since LHS = RHS)
2
5 State the solution. Therefore x = −4 and x = − --- are both suitable
solutions. 3

remember
remember
To solve equations with absolute values:
1. remove the absolute value symbols and state the equation as positive and
negative cases
2. verify your solutions by substituting your answer into the original equation.

Solving equations using


1L absolute values
x = 4 or x = −6 x = 1 1--2- or x = − 1--2-
WORKED 1 Solve for x. eBook plus
Example
xample
23
a | 2x | = 10 x = ±5 b |x + 1| = 5 c | 2x − 1 | = 2
x Digital doc:
d | 3x + 2 | = 4 e | 1 + 2x | = 0 x = − 1--2- f --- = 3 x = ±9 SkillSHEET 1.7

x= 2
--- or x = −2
3 Solving equations
WORKED 2 Solve for x. 3
Example
xample
24
a | x + 1 | = 2x − 1 x = 2 b | 3x + 5 | = x − 3 No solutions
c | 2x + 3 | = x − 5 No solutions d | x − 2 | = 2x − 7 x = 5
WORKED 3 Solve the following for x.
Example
xample
a | 2x − 5 | = | x + 1 | x = 6 or x = 1 --3- b | 3x − 6 | = | 2x + 4 | x = 10 or x = 2--5-
1
25
c | 3x − 1 | = | 2x + 2 | x = 3 or x = − --15- d | x − 5 | = | 3x − 8 | x = 1 --12- or x = 3 --14-
Chapter 1 Number systems: the Real Number System 55
Solving inequations
You have graphed inequations on a number line in your junior mathematics studies.
These examples require more care and you will notice that the verification step is
essential to test the values you obtain.

WORKED Example 26
Solve and graph (x - 1)(x + 2) > 0.
THINK WRITE/DRAW
1 Write the inequation. (x − 1)(x + 2) > 0
2 If a × b > 0 then either a and b Case 1:
are both positive (+ve) or a If a and b > 0
and b are both negative (−ve). (x − 1) > 0 and (x + 2) > 0
This gives rise to 2 cases.
Rewrite the terms of the
inequation.
Note: > 0 means +ve, and
Note: < 0 means −ve x>1 x > −2
Solve each inequation.
3 Graph both these inequations
and decide which part of the –3 –2 –1 0 1 2 3 4
graph satisfies both
inequations.
Note that the region graphed in
the last graph (x > 1) satisfies –3 –2 –1 0 1 2 3 4
both parts of case 1.
4 Repeat steps 2 and 3 for case Case 2:
2. If a and b < 0
Graph both these inequations (x − 1) < 0 and (x + 2) < 0
and decide which part of the x<1 x < −2
graph satisfies both
inequations. –3 –2 –1 0 1 2 3 4
Note that the region graphed
in the last graph (x < −2) –3 –2 –1 0 1 2 3 4
satisfies both parts of case 2.
5 Either case 1 is true or case 2
is true at the one time but not –3 –2 –1 0 1 2 3
both, as they are contradictory.
Combine both cases on one Either x > 1 or x < −2
number line so that either x > 1
(from case 1) or x < −2 (from
case 2). –3 –2 –1 0 1 2 3
Graphed Not graphed Graphed

Continued over page


56 M a t h s Q u e s t M a t h s C Ye a r 1 1 f o r Q u e e n s l a n d

THINK WRITE
Use a tabular form to verify
this solution. x < −2 −2 < x < 1 x>1
Note how the number line is (let x = −3) (let x = 0) (let x = 2)
divided into 3 regions. x−1 − − +
When completing this table x+2 − + +
choose a number that falls in (x − 1)(x + 2) + − +
each region and work out the graphed not graphed
sign only of each expression. OK graphed OK
Because the original product OK
was greater than 0 (or positive)
the table has verified the results
The solution is either x > 1 or x < −2.
on the graph. We do not want to
include those values between −2
and 1.

WORKED Example 27
Solve and graph:
3 x–2
a --- < 6 where x ≠ 0 b ------------ < 0 where x ≠ −1.
x x+1

THINK WRITE/DRAW
3
a 1 Write the inequation. a --- < 6
x
2 x can be either +ve or −ve. When x is Case 1: Case 2:
−ve and multiplied across the 3 3
inequality sign, the sign must be If x > 0, --- < 6 If x < 0, --- < 6
x x
reversed. 3 < 6x 3 > 6x
1 1
--- <x --- >x
2 2
1 1
or x > --- or x < ---
2 2

3 Draw separate graphs for both


these inequalities, but remember 0 1–
2 1 2 –1 0 1
that in case 2, x < 0 so the only part
that should be graphed is where
x < 0.

4 Combine these graphs on the one


–3 –2 –1 0 1– 1 2 3
number line and state your answer. 2
Remember that an initial condition
of the problem was that x ≠ 0 so that x < 0 and x > 1
---
2
has been satisfied also.
Chapter 1 Number systems: the Real Number System 57

THINK WRITE/DRAW
x–2
b 1 Write the inequation. b ------------ < 0
x+1
a
2 Remember that < 0 means −ve. If --- < 0 Case 1:
b If x − 2 < 0 and x+1>0
then either a < 0 or b < 0, but not both at
x<2 x > −1
the one time.
3 Graph both these inequalities.
0 1 2 –2 –1 0

4 Since both results from case 1 occur


at the one time combine the two –3 –2 –1 0 1 2 3
different graphs that satisfy both parts.
5 Repeat steps 2 to 4 for case 2. Case 2:
If x − 2 > 0 and x+1<0
x>2 x < −1
6 Since both results of case 2 occur at
the one time think about how to 1 2 3 –2 –1 0
combine the two different graphs Case 1 Case 2
that satisfy both results.
Note that x can’t be greater than 2
and less than −1 at the same time.
Therefore this solution is impossible.
Reject this solution.
7 Graph and state the final solution
(that was obtained in step 4 above). –3 –2 –1 0 1 2 3
The solution is −1 < x < 2.

WORKED Example 28
Solve and graph | 2x − 3 | < 2.

THINK WRITE/DRAW

1 Write the inequality. | 2x − 3 | < 2


2 Consider the positive and negative Case 1: Case 2:
cases of the absolute value. 2x − 3 < 2 or −(2x − 3) < 2
3 Solve for x. In case 2, note that 2x − 3 < 2 or 2x − 3 > −2
multiplying both sides of an inequality 2x < 5 or 2x > 1
by a negative number reverses the x < 5--- or x > 1---
2 2
inequality sign.
x < 2 1---
2
4 Graph both solutions on separate
1
number lines. 1 2 2 –2 3 0 1–
2 1 2
Case 1 Case 2
Continued over page
58 M a t h s Q u e s t M a t h s C Ye a r 1 1 f o r Q u e e n s l a n d

THINK WRITE/DRAW
5 Graph this combined solution and state
1– 1
the solution. –2 –1 0 2 1 2 2 –2 3

The solution is 1
--- < x < 2 1--- .
2 2
6 Verify the 3 regions of this solution. Check:
For x < 1--- (let x = 0)
2
| 2 × 0 − 3 | < 2 (Not valid and not graphed)
For 1--- < x < 2 1--- (let x = 2)
2 2
|2 × 2 − 3| < 2
| 1 | < 2 (Valid and graphed)
For x > 2 1--- (let x = 3)
2
|2 × 3 − 3| < 2
| 3 | < 2 (Not valid and not graphed)

WORKED Example 29
1
Solve and graph ----------------- < 2.
x–3
THINK WRITE/DRAW
1
1 Write the inequality. ----------------- < 2
x–3
2 As with Worked example 28, state the Case 1: Case 2:
two cases that are possible. 1 1
Solve for case 1. -------------- < 2 or ------------------- < 2
x–3 –( x – 3 )
For case 1
if x − 3 > 0 (x > 3)
1 < 2(x − 3)
1 < 2x − 6
7 < 2x
--- < x
7
2
x > 3 1---
2
Since x > 3 (initial condition) x > 3 1---
2
satisfies this condition.
1
Draw the graph for this solution. 0 1 2 3 3 –2 4 5

3 Solve for case 2. Case 2


Reverse the inequality sign when you 1
multiply by a negative. -------------- > −2
x–3
If x − 3 < 0 (x < 3)
1 < −2(x − 3)
1 < −2x + 6
−5 < −2x
Chapter 1 Number systems: the Real Number System 59
THINK WRITE/DRAW
Reverse the inequality sign when you 2 1--- > x
2
divide by a negative.
Remember to always check with the x < 3 from the initial condition; therefore
initial condition. x < 2 1--- satisfies this condition.
2
Draw the graph for this solution. 1
0 1 2 2 –2 3 4

4 Draw the combined graph for these two


1 1
solutions and state the answer. –1 0 1 2 2 –2 3 3 –2 4
The solution is x < 2 1--- or x > 3 1--- .
2 2
5 Verify the results for the three regions Check:
on the graph. For x < 2 1--- (let x = 0)
2
1
----------------- < 2
0–3
1
------------ < 2 ( Valid and graphed)
–3
For 2 1--- < x < 3 1---
2 2
(let x = 3 1--- ; remember x ≠ 3)
4
1
-------------------- < 2
1
3 --4- – 3
1
------- < 2
1
---
4
| 4 | < 2 (Not valid and not graphed)
For x > 3 1--- (let x = 4)
2
1
----------------- < 2
4–3
1
-------- < 2 (Valid and graphed)
All solutions have been verified. Even 1
though this verification is a fairly
lengthy step it gives you confidence
that your solutions are more than likely
going to be correct.

WORKED Example 30
x–2
Solve and graph ------------ < 2 where x ≠ −3.
x+3
THINK WRITE/DRAW
x–2
1 Write the inequality. ------------ < 2
x+3 Continued over page
60 M a t h s Q u e s t M a t h s C Ye a r 1 1 f o r Q u e e n s l a n d

THINK WRITE/DRAW
2 Remember x has the same value in the Case 1: Case 2:
numerator and denominator at any one x–2 x–2
time. Separate the solution into the ------------ < 2 or –  ------------ < 2
x+3  x + 3
+ve and −ve cases for the absolute
x–2
value. ------------ > −2
x+3

3 If the denominator is –ve, then the sign Case 1 (+ve): If x + 3 > 0 (that is, x > −3)
must be reversed when it is multiplied x − 2 < 2(x + 3)
across. Therefore, we will have to make
x − 2 < 2x + 6
2 sub-cases for each of case 1 and
case 2. −8 < x
x > −8
But the initial condition is that x > −3, therefore
x > −3 is a valid solution.

Graph this first solution.


–4 –3 –2 –1 0 1

Reverse the sign due to the –ve Case 1 (−ve): If x + 3 < 0 (that is, x < −3)
denominator. x–2
------------ < 2
x+3
x − 2 > 2(x + 3)
x − 2 > 2x + 6
−8 > x
x < −8
But the initial condition is that x < −3, therefore
x < −8 is a valid solution.

Graph this solution.


–9 –8 –7 –6 –5

x–2
4 Determine case 2 as for case 1. ------------ > −2
x+3
Case 2 (+ve): If x + 3 > 0 (that is, x > −3)
x − 2 > −2(x + 3)
x − 2 > −2x − 6
3x > −6 + 2
–4
x > ------
3
But x > −3 from initial condition so x > −1 1--- .
3

Graph this solution. –2 –1 0 1 2


Chapter 1 Number systems: the Real Number System 61
THINK WRITE/DRAW
Determine case 2 for the –ve Case 2 (−ve): If x + 3 < 0 (that is, x < −3)
denominator. x–2
Remember to reverse the sign when ------------ > −2
x+3
you multiply by the –ve denominator.
x − 2 < −2(x + 3)
x − 2 < −2x − 6
3x < −6 + 2
4
x < − ---
3
But x < −3 from the initial condition, so
x < −3.
Graph this solution.
–5 –4 –3 –2 –1 0

5 Since the denominator must be either Combining case 1 (+ve) and case 2 (+ve) gives:
positive or negative at any one time it is
case 1 (+ve) and case 2 (+ve) that we –4 –3 –2 –1 0 1
need to combine as well as case 1 (−ve) Combining case 1 (−ve) and case 2 (−ve) gives:
and case 2 (−ve) to produce the final
graph. –10 –9 –8 –7 –6 –5

6 Combine the two solutions.


–10 –9 –8 –7 –6 –5 –4 –3 –2 –1 0 1 2
The solution is x < −8 or x > −1 1--- .
3

7 Verify the solutions by testing within Check:


the 3 regions, using the original For x < −8 (let x = −9)
inequality. –9–2
---------------- < 2
–9+3
– 11
--------- < 2 (Valid and graphed)
–6
For −8 < x < −1 1--- (let x = −2)
3
–2–2
---------------- < 2
–2+3
–4
------ < 2 (Not valid and not graphed)
1
For x > −1 1--- (let x = 0)
3
0–2
------------ < 2
0+3
–2
------ < 2 (Valid and graphed)
3

This type of problem demonstrates higher level reasoning for this study of numbers. All
possibilities need to be carefully considered and examined in a thoughtful, methodical
manner.
62 M a t h s Q u e s t M a t h s C Ye a r 1 1 f o r Q u e e n s l a n d

remember
remember
1. When solving inequations reverse the inequality sign when you multiply or
divide by a negative expression.

2. In any equation if the product of a and b (that is a × b) is positive then a and b


are either both positive or both negative.

3. If a product a × b is negative then either a or b is negative.

4. If x − a is positive then x − a > 0 and x > a.

5. If x − a is negative, then x − a < 0 and x < a. 1


a 1<x<2

0 1 2 3 4
b x > 2 or x < −2

1M
–3 –2 –1 0 1 2 3
Solving inequations c 1 --12- < x < 2

1
–1 0 1 1–2 2 3 4
WORKED 1 Solve and graph the following inequations. d 2<x<3
Example
xample
26 a (x − 1)(x − 2) < 0 b (x − 2)(x + 2) > 0 0 1 2 3 4

c (2x − 3)(x − 2) < 0 d 3x − 10x + 4 < 2x − 5x − 2


2 2

2 a x < 0 or x > 2
d x > 2 2--1- or x < 2

c x < 3 or x > 4 2--1-

b x < 1 or x >
WORKED 2 Solve and graph the following inequations.
Example
xample
27 4 1
a --- < 2 (where x ≠ 0)) b ----------- < 3 (where x ≠ 1)
x x–1

1 3--1-
SLE 2: Solve simple inequality
statements such as |z - a | > b in 3 x–1
the real system, and be able to c ----------- < 2 (where x ≠ 3) d ----------- < 3 (where x ≠ 2)

–1
give a verbal description of the x–3 x–2 1
0

0
meaning of the mathematical
−5 < x < −3

0
symbolism.
2
1

3 Solve and graph the following.


1
WORKED –6 –5 –4 –3 –2 –1 0
1 13–
Example
xample
−4 < x < 4
3
2 22– 3

28 a |x| < 4 b |x + 4| < 1 2


1

2
4 42– 5

3
–5 –4 –3 –2 –1 0 1 2 3 4 5
x < −1 1--3- or x > − 2--3-
1

x–1
| 2x − 5 | < 1 2 < x < 3 d ----------------- < 2
4

c
4

–3 –2 –1 0 1 2 5
–11–3 – 2–3 0 1 2 3 4 5 −9 < x < 11
WORKED 4 Solve and graph the following. –10 –5 0 5 10
x < −3 or x > − 3--1-

Example
xample
–4 –3 –2 –1 _ 1– 0
x < 1 4--1- or x > 1 4--3-

29, 30
0

1 1
a ----------------- < 3 (where x ≠ −1) b -------------------- < 2 (where x ≠ 1 1--- )
x+1 2x – 3 2
eBook plus
1 1–1
4

1 x–1
c ----------------- < 1
(where x ≠ 6) d ------------ < 2 (where x ≠ −1)
3
14–3 2

Digital doc: ---


x–6 3 x+1
WorkSHEET 1.2
1

x+1 x
2

e ------------ > 2 (where x ≠ 3) f ----------- > 4 (where x ≠ 4)


3

x–3 x–4
x < 3 or x > 9 3 1--5- < x < 5 1--3- , x ≠ 4
1 2--3- < x < 7, x ≠ 3 3 31–5 4 5 51–3 6 7
0 112– 2 3 4 5 6 7 8
3
2 3 4 5 6 7 8 9 10
Chapter 1 Number systems: the Real Number System 63
Approximations for p
SLE 9: Investigate some of Research the following historical approximations for π and present your findings in
the approximations to p
which have been used. concise form.
1 3000 BC Egypt: The pyramids are built. The sides and heights of the pyramids
of Cheops and Sneferu at Giza are constructed in the ratio of 11:7. Hence the
ratio of one perimeter to 2 heights is 22:7. The value of π is approximately -----
22-
.
7

2 2000 BC Egypt: The Rhind Papyrus, the oldest mathematical text in existence,
gives the following rule for constructing a square having the same area as a
given circle:
Cut one-ninth off the circle’s diameter and construct a square on the
remainder. Using this method, π is found to equal ( 16 2
------ ) .
9

3 240 BC Greece: Archimedes, engineer, architect, physicist and mathematician,


------ < π < 3 1
constructs polygons of 96 sides to show that 3 10 --- .
71 7

4 20 BC Italy: Vituvius, architect and engineer, measures distances using a


wheel and determines that π is equal to 3 1--- .
8

5 AD 125 Greece: Ptolemy writes his famous work on astronomy, Syntaxis


Mathematica. He finds that π is equal to 3 + -----
8- 30
+ --------
2
.
60 60
6 AD 480 China: Tsu Ch’ung-chih, expert in mechanics and interested in
machinery, gives the value of π as 355
--------- .
113

7 AD 1150 India: Bhaskara writes on astronomy and mathematics and gives


several values of π, the most accurate is 3927
------------ .
1250

8 AD 1579 France: Vieta finds π correct to nine decimal places by considering


polygons of 6.516 = 393 216 sides. He also discovers that

2 2 2+ 2 2+ 2+ 2
--- = ------- × -------------------- × ---------------------------------- .
π 2 2 2
In this and the next two series, examine how the approximation improves as the
number of terms is increased.
9 AD 1650 England: John Wallis uses a complicated and difficult method to
4 3 × 3 × 5 × 5 × 7 × 7 × 9 × 9…
obtain π from --- = --------------------------------------------------------------------------
π 2 × 4 × 4 × 6 × 6 × 8 × 8…

10 AD 1699 England: Sharp calculates π to 72 decimal places by evaluating the


1- 3
 ------ 1- 5
 ------ 1- 7
 ------
π 1  3  3  3
series --- = ------- − --------------- + --------------- − --------------- …
6 3 5 7
3

11 AD 1913 India: The mathematician Ramanujan presents the following as an


2 2
approximation for π : π = 4 9 + 19
--------- .
22
64 M a t h s Q u e s t M a t h s C Ye a r 1 1 f o r Q u e e n s l a n d

Real numbers — application


and modelling
1 Solve the following inequality and graph the solution on a number line
2x + 1
---------------- < 1. −1 − 2 < x < 2
–3 –2 –1 0 1 2
x– 2 –2.4 √2

2
a– b (a – b) a – b
2 Rationalise the denominator for ---------------- . ----------------------------------------
2
a+ b a –b

2x – 5
3 Solve the inequation --------------- > 0. x ∈ R, x ≠ −3 1--2- , x ≠ 2 1--2-
2x + 7

4 By noting the expansion of ( a + b )2 and the fact that 7 + 5, 3 + 2 2

12 + 2 35 = 7 + 5 + 2 7 × 5 , determine 12 + 2 35 .

Hence, determine 17 + 6 8 .
--1-
4
, m = 3, k = 4 5 If the integer points, n, and the points midway between them, n + --1- , are
2
mapped on a number line, how far away from the nearest of these points can
any point on the number line be?
Find an integer m such that | 2 – 1--- m | < 1--- and an integer k such that
2 4
| 5 – 1--- k | < 1--- .
2 4
Explain the significance of these results with respect to the topic of
approximation of irrationals to rationals. (Your response should not rely on
calculator computations.)
6 The most common way students learn to find the greatest common divisor of
1
7 + ----------------------------------------- g.c.d. = 225 two integers is to factorise both numbers into their prime factors and take the
1
1 + ------------------------------- - common prime factors. For example, to find the greatest common divisor of
1 -
3 + --------------------- 45 024 and 5712 we can write:
1
1 + ------------
2 + ----- 1
- 45 024 = 25 × 3 × 7 × 67
5712 = 24 × 3 × 7 × 17
13

So the greatest common divisor is 24 × 3 × 7 = 336.


However, finding the prime factors is not always that straightforward and
Euclid developed an algorithm that produces the greatest common divisor. To
apply it we divide the smaller integer into the larger integer.
Consider the integers 45 024 and 5712 again. When 45 024 is divided by
5712 the result is 7 with a reminder of 5040. Thus
45 024 = 7 × 5712 + 5040
Now divide the remainder (5040) into the divisor (5712)
5712 = 1 × 5040 + 672
and the new remainder into the previous remainder, and so on:
5040 = 7 × 672 + 336
672 = 2 × 336 + 0
The last non-zero remainder is the greatest common divisor, because it divides
both 45 024 and 5712 and every divisor of both 45 024 and 5712 must also
divide it.
Chapter 1 Number systems: the Real Number System 65
We can rewrite the previous equations as
45 024 1 5712 1 5040 1 672
---------------- = 7 + --------------- ------------ = 1 + --------------- ------------ = 7 + --------------- --------- = 2
5712 5712 5040 5040 672 672 336
------------ ------------ ---------
5040 672 336
45 024 1
which can be combined as a continued fraction as ---------------- = 7 + ---------------------- and can
5712 1
1 + ------------
1
7 + ---
2
also be written as 7, 1, 7, 2 (whole numbers obtained at each division step).
Express the quotient 327 600 ÷ 42 075 as a continued fraction and hence state the
greatest common divisor of these two integers.

7 Show that 2+ 2 + 2– 2 <2 2 (Do not use your calculator.)


The next 2 terms would be
1
1 2 = 1 + -------------------------------------------- -
8 Show that 2 = 1 + ---------------- and hence deduce that 1
2 + -----------------------------------
1+ 2 1
2 + --------------------------
1
1 1 2 + ----------------
2 = 1 + -------------------------- = 1 + ----------------------------------- 1+ 2
1 1 and
2 + ---------------- 2 + --------------------------
1 1
1+ 2 2 + ---------------- 2 = 1+ -----------------------------------------------------
-
1
1+ 2 2 + --------------------------------------------
-
1
2 + -----------------------------------
Now write the next two terms in such a sequence. 1
2 + --------------------------
1
2 + ----------------
9 Diophantus of Alexandria, born between AD 75 and AD 250, was the author of 1+ 2
Arithmetica and wrote many papers investigating only whole numbers. Diophantine
equations are involved in many real-life situations, due to their whole number
solutions. The solutions are often found by trial and error. The following problem,
which you might like to try, is based on Diophantine equations. (The answer is given
at the end of the problem.)
Five men and a monkey were shipwrecked on an island and they set out to collect
as many coconuts as they could on their first day so that they would have provisions
until they were rescued. After many hours collecting coconuts they piled them all
together and went to sleep.
Later that night, while the others slept one man woke up, and started to worry
about whether the coconuts would be divided fairly. He decided to take his one-fifth
share of the nuts and gave the one remaining coconut to the monkey.
Throughout the night all of the men did likewise; they took their fifth share of
what was left of the nuts and gave the one remaining nut to the monkey. In the
morning they met to divide the nuts into five equal shares. Each man knew that
there were some nuts missing but none admitted he had some extras, of his own,
hidden away.

How many nuts were there to start with? (Solution)


The Diophantine equations that lead to the solution of this problem simplify to
1024x – 8404 = 15 625y where x is the original amount collected and y is the
number each is given in the morning.
The smallest possible value for x is 3121 with other values at intervals of 15 625.
66 M a t h s Q u e s t M a t h s C Ye a r 1 1 f o r Q u e e n s l a n d

summary
The Real Number System
• The set of real numbers (R) is divided into two main sets: rational and irrational
numbers. These sets may be further divided into smaller subsets as illustrated on
the chart and Venn diagram below.
Real numbers R

Irrational numbers I Rational numbers Q


(surds, non-terminating
and non-recurring
decimals, π ,e) Integers Non-integer rationals
Z (terminating and
recurring decimals)

Negative Zero Positive


Z– (neither positive Z+
nor negative) (Natural
numbers N)
• Rational numbers (Q) can be expressed in ε =R
a
the form --- , where a and b are integers Q (Rational numbers)
b
with no common divisors and b ≠ 0. They
may also be expressed as whole numbers, Z (Integers) I
terminating decimals and recurring N (Irrational
(Natural numbers)
decimals. numbers)
• Irrational numbers (I) cannot be expressed
in the form a--- . They can be expressed only
b
as non-terminating and non-recurring
decimals and include surds and numbers such as π and e.
• Recurring decimals are rational numbers which may be expressed as a ratio of two
integers.
• Surds are irrational numbers which can only be represented exactly using a root
symbol (or radical), that is:
, 3 , 4 and so on.
• π and e are examples of irrational numbers which may be converted to non-
terminating and non-recurring decimals; however, they are not surds.

Set notation
• Set notation is used when defining the Real Number System.
• The following symbols are useful when working with sets:
{ }‘set’
∈ ‘is an element of’
∉ ‘is not an element of’
⊂ ‘is a subset of’
Chapter 1 Number systems: the Real Number System 67
Working with surds
• To simplify a surd, it should be written as a product of two factors, one of which is
the largest perfect square.
• Like surds may be added and subtracted; surds may need to be simplified before
adding and subtracting.
• Surds may be multiplied according to the rules:
a× b = ab
m a × n b = mn ab
• When a surd is multiplied by itself (squared), the result is the number under the
radical:
( a)2 = a
• Multiplication involving brackets:
1. The Distributive Law: a( b + c) = ab + ac

2. FOIL: ( a + b )( c + d ) = ac + ad + bc + bd

3. Perfect squares: ( a + b)2 = a + 2 ab + b


( a – b)2 = a – 2 ab + b
4. Difference of two squares DOTS: ( a + b )( a – b) = a – b
• The product of a conjugate pair of surds is rational.
• Surds may be divided according to the rule:

a
a ÷ b = -------
b
a
= ---
b
• Rationalising denominators:
1. If the denominator contains a surd, multiply both numerator and denominator by
the surd part of the denominator:

a a b
------- = ------- × -------
b b b
ab
= ----------
b
2. If the denominator is a sum or difference of surds, multiply both the numerator
and the denominator by the conjugate of the denominator:

1 1 a+ b
-------------------- = -------------------- × --------------------
a– b a– b a+ b
a+ b
= --------------------
a–b
68 M a t h s Q u e s t M a t h s C Ye a r 1 1 f o r Q u e e n s l a n d

Modulus
• The modulus (or absolute value) of a number is the magnitude of that number and
is always positive.
• The modulus of x is denoted by | x |.
• | x | = −x if x < 0
= 0 if x = 0
= x if x > 0

Solving equations using absolute values


• First remove the absolute value symbols and state the equation as positive and
negative alternative cases.
• Verify your solutions for all these questions by substituting your answer into the
original equation.

Solving inequations
• Remember that if x > 0 then x is positive, and if x < 0 then x is negative.
• If a product of two factors is greater than 0 then both factors must be either positive
or negative.
• Likewise, if a product of two factors is less than 0 then only one of the factors must
be positive and the other must be negative,
• Organise your solution into two cases that will develop arguments for all possible
values.
• The two values that result for each case are values that should occur at the one time.
The graph you draw must be a combination of these two solutions for each case.
• Verify your solutions by choosing values that fall in each of the regions of your
graph.
• When you multiply or divide by a negative factor across an inequality sign,
remember to reverse the sign.
Chapter 1 Number systems: the Real Number System 69
2 a Irrational, since equal to non-recurring
and non-terminating decimal

CHAPTER b Rational, since can be expressed as a


whole number

review c Rational, since given in a rational form


d Rational, since it is a recurring decimal
e Irrational, since equal to non-recurring
and non-terminating decimal
1 multiple choice
Which of the given numbers, 6- ,
-----
. π
0.81 , 5, −3.26, 0.5 , --- , 3-
----- are rational?
1A
12 5 12
. 6 π
A 0.81 , 5, −3.26, 0.5 and 3-
----- B ------ and ---
12
12 5
C 6-
----- , 0.81 and 3-
----- D 5, −3.26 and 6-
-----
12 12 12

E 5
2 For each of the following, state whether the number is rational or irrational and give the
reason for your answer: 1A
.
a 12 b 121 c 2--- d 0.6 e 3 0.08
9

3 multiple choice
Which of the following statements is not correct?
1A
A 9-
----- ∉Q B 8
--- ∉Q C 0
--- ∉ Z+ D 3
– 125 ∈ Z − E (−5)2 ∈ Z +
81 4 4

4 multiple choice
Which. of the following statements regarding the given set of numbers,
1A
{5, 0.7, 64 , 21, 8 , 20 }, is correct?
A 5, 64 , 20 , ∈ Z +
B 8,
20 cannot be expressed as rational numbers.
.
C 5, 0.7 and 21 are the only rational numbers of the set.
D 64 , 8 and 20 cannot be expressed as rational numbers.
E None of the above.
5 Classify each of the following into the smallest subset in which they belong using Q, I, Z,
Z + and Z −. (Simplify first where possible.) 1A
3
1 4 15 15 8 2
a 4× – --------- – 0.2 Z − b ------------------ Z+ c – ------ × ------ Q d --- + ------ I
125 2 0.01 8 2 9 25

6 multiple choice
Which of the following fractions, 1- -----
----- , 3- , -----
5- 1
, --- , 2--- , cannot be expressed as recurring
1B
17 13 12 5 3
decimals?
1- 1--- 2--- 3- -----
A ----- , , B ----- , 5- , 2--- C 3- -----
----- , 5- D 1
--- E 1- 1
----- 3-
, --- , -----
17 5 3 13 12 3 13 12 5 17 5 13

7 multiple choice
..
The recurring decimal 0.7 2 can be expressed as a fraction in its simplest form as:
1B
18 73 73 4 8
A ------ B --------- C ------ D --- E ------
25 100 99 5 11
70 M a t h s Q u e s t M a t h s C Ye a r 1 1 f o r Q u e e n s l a n d

8 Express the following recurring decimals as fractions in the simplest form.


1B .. 62
a 0.6 2 ------
. 99337
b 0.374 --------
900
-
. . 157
c 0.95 1 --------
165
-

9 multiple choice
1C Which of the following numbers of the given set,
{3 2 , 5 7 , 9 4 , 6 10 , 7 12 , 12 64 }, are surds?
A All of these
B 9 4 , 12 64
C 3 2 and 7 12 only
D 3 2 , 5 7 and 6 10 only
E 3 2 , 5 7 , 6 10 and 7 12

m 20 3 3
10 Which of 2m , 25m , ------ , ------ , m, 8m are surds
1C 16 m 10 a 2m ,
20
------ ,
m
3 m, 3 8m
a if m = 4?
m 20
b if m = 8? b 25m , ------ , ------
16 m
11 multiple choice
1D
The expression 250 may be simplified to:
A 25 10 B 5 10 C 10 5 D 10 25 E 5 50

12 multiple choice
1D
The expression 392x 8 y 7 may be simplified to:

A 196x 4 y 3 2y B 2x 4 y 3 14y C 14x 4 y 3 2y D 14x 4 y 3 2 E 14x 4 y 3 2xy

13 Simplify the following surds. Give the answers in the simplest form.
1D
a 4 648x 7 y 9 72x 3 y 4 2xy

2 25 – 1--4- x 2 y 5 xy
b – --- ------ x 5 y 11
5 64

14 multiple choice
1E
When expressed in its simplest form, 2 98 – 3 72 is equal to:
A –4 2 B –4 C –2 4 D 4 2 E None of these

15 Simplify the following, giving answers in the simplest form.


1E
a 7 12 + 8 147 – 15 27 25 3

1 1
b --- 64a 3 b 3 – 3--- ab 16ab + --------- 100a 5 b 5 3ab ab
2 4 5ab
Chapter 1 Number systems: the Real Number System 71
16 Determine the length of the unknown side, giving the answer in the simplest form and
specifying the appropriate unit. 1E
a b 16 a 5 m 1F
x b ( 17 – 4 6 ) cm
l c ( 26 – 4 2 ) m
10 cm d 22 cm
P = 44 – 8 6 cm
P = 80 m

c 6+2 3m d
c
y 11 3 cm

P = 64 + 4 3 – 8 2 m 11 cm

17 multiple choice
1F
When expressed in its simplest form, 9 12 × 3 5 is equal to:
A 27 60 B 15 54 C 18 5 D 54 15 E 6 15

18 Simplify the following, giving answers in the simplest form.


1
1F
a --- 675 × 27 27
5

b 10 24 × 6 12 720 2

19 multiple choice
1G
When expanded and expressed in its simplest form, 12 ( 8 – 6 ) is equal to:
A 4 6–6 2 B 96 – 72 C 4 3–6 D 24 E 2 6–3 2

20 multiple choice
1G
When expanded and expressed in its simplest form, ( 9 x 2 y – 7x ) ( 9 x 2 y + 7x ) is equal to:
A 9x 2 y – 7x 2 B 32y C 32x 2 y – 7x 2 D 81x 2 y – 49x 2 E 2x 2 y

21 multiple choice
1G
When expanded and expressed in its simplest form, ( 2 8 – 7 ) is equal to:
2

A 39 – 16 14 B 39 – 8 14 C 25 – 8 14 D 39 E 25 – 16 14

22 multiple choice
8x 3 1H
When expressed in its simplest form, -------- is equal to:
32
x x x3 x3 x x x3
A ---------- B --------- C --------- D ---------- E -----
2 4 2 4 4
72 M a t h s Q u e s t M a t h s C Ye a r 1 1 f o r Q u e e n s l a n d

23 multiple choice
1H When expressed in its simplest form,
6x 2 y 3 8xy
----------------- × -------------- is equal to:
12x y 4 5 10x 2 y
10x 10 x x 10 x 10x
A -------------- B ------------- C ------------- D --------- E -------------
25x 2 y 5x 2 y 5x 2 y 5xy 5x 2 y

24 Determine the length of the unknown side of a rectangle, given its area is 7 18 – 2 3 cm2
1H
and its width is 2 + 3 cm. ( 23 6 – 48) cm

25 multiple choice
1I When expressed in its simplest form with a rational denominator, 18
------ is equal to:
63
2 2 14 7
A ------- B --- C ---------- D ------- E 2
7 7 14
7

26 Express the following with a rational denominator, giving the answer in the simplest form.
1I
7 56 – 3 126 5 7
------------------------------------ ----------
4
32

27 Express the following with a rational denominator, giving answers in the simplest form.
1I Assume that a, b, x and y are positive real numbers.
5x 3 y x 5y 20x 5 y 3 9a 2 b
a ---------------- ------------- b --------------------- x2 y 2 c ---------------- 3a
2
2 x 10xy b

28 multiple choice
1J 1
When expressed in its simplest form with a rational denominator, ------------------------ is equal to:
3 8+ 5
3 8– 5 6 2– 5 3 8– 5 3 3 6 2– 5
A ----------------------- B ----------------------- C ----------------------- D ---------- E -----------------------
77 77 67 67 67

29 multiple choice
1J
If x = 11 – 3 then x 2 + 8x + 5 is equal to:
A 1 – 6 11 B 2 11 + 22 C 2 11 + 1 D 22 – 5 11 E 23
2323 – 594 14
30 Given that x = 2 7 – 3 2 find: 30 a -------------------------------------
1J 1 1 2277 – 606 14
50

a x 2 + ----- b x 2 – ----- c x 2 – 9x + 5 b -------------------------------------


50
x 2 x2
c 51 – 12 14 – 18 7 + 27 2
31 Express the following with a rational denominator, giving the answer in the simplest form.
1J 1 1
--------------------------- – ----------------------- 3-----------------------
7– 3
40
2 7–2 3 3 3– 7
Chapter 1 Number systems: the Real Number System 73
32 Determine the area of the triangle at right, expressing the
answer with a rational denominator in the simplest surd form. 1J
Measurements are in metres. —1
2+ 5
3
------- m2
2
6 + 15
33 multiple choice
–2 3 × –7 × –3 1K
When expressed in its simplest form with a rational denominator, -------------------------------------------- is
equal to: 3 6
14 7 18
A –7 2 B 2 7 C ------- D 7 2 E -------------
2 3

34 Simplify the following.


–3 × – 5 × 4
1K
a – 7 – – 4 −11 b ------------------------------------ – – 8 −3
–6 + 6

35 multiple choice
1L
Which of the given values of x solve the equation | 3 − 2x| = 2?
A x= 1---
B x= 5
--- C x = 1--- , 5
--- D x = 2, 2
--- E x=2
2 2 2 2 5

36 multiple choice
1L
Which of the given values of x solve the equation | x + 3 | = 2x + 7?
A x = −4, −3 1--- B x = −4, 1 1--- C x = −3 1--- D x = −1, −4 E x = −2
3 3 3

37 multiple choice
1L
Which of the following values of x solve the equation | x − 3 | = | 2x − 1 |?
A x = −2, 2---
B x = −2, −1 C x = −2, 1 1--- D x = 2--- , 1 1--- E x= 1
---
3 3 3 3 3

38 multiple choice
1M
Which values of x are a solution for the equation (x + 1)(x − 2) < 0?
A −1 < x < 2 B x < −1 or x > 2 C −1 < x < 0 D −1 < x < 1 E x>2

39 multiple choice
2 1M
Which values of x are a solution for --- < 4?
x
A 0 < x < 1--- B 0 < x < 1--- C x < 0 or x > 3 D x < 0 or x > 1
--- E x>1
4 2 2

40 multiple choice
x+1
1M
Which of the following values of x are a solution for ------------ < 3?
x–1
A --1- <x<2 B x< --1- or x > 2 C 1 < x < 2 --1- D x < 1, x > 2 --1- E x>1
2 2 2 2
74 M a t h s Q u e s t M a t h s C Ye a r 1 1 f o r Q u e e n s l a n d

Modelling and problem solving


1 The electrical current I in amps, delivering electrical power P, through resistor R, is given by
P
the rule I = --- . Express the current as a surd in the simplest form when:
R
5 10
a P = 500, R = 18 ------------- amps
3 5 102
---------------- amps
b P = 425, R = 6 6

c P = 729, R = 0.38 135 38


------------------- amps
5 34 19
d P = 1700, R = 8 ------------- amps
2

2 An ice-cream cone with measurements as shown is completely filled with ice-cream, and has
a hemisphere of ice-cream on top.

27 cm

175 cm

a Determine the height of the ice-cream cone in simplest surd form. 2 37 cm


b Determine the volume of the ice-cream in the cone. 18 π 37 cm3
Determine the volume of the ice-cream in the hemisphere. 54 π 3 cm
3
c
d Hence, find the total volume of ice-cream. 18 π ( 37 + 3 3 ) cm3

3 A gold bar with dimensions of 5 20 , 3 12 and 2 6 cm is to be melted down into a


cylinder of height 4 10 cm.
a Find the volume of the gold, expressing the answer in the simplest surd form and
specifying the appropriate unit. 360 10 cm3
eBook plus
b Find the radius of the cylinder, expressing the answer in the simplest surd form and
specifying the appropriate unit. 3 10 π
Digital doc: ----------------- cm
Test Yourself π
Chapter 1 c If the height of the cylinder was 3 40 cm, what would be the new radius? Express your
answer in the simplest surd form. 2 15 π
----------------- cm
π
Number
systems:
complex
numbers
2
syllabus reference
Core topic:
Real and complex number
systems

In this chapter
2A Introduction to complex
numbers
2B Basic operations using
complex numbers
2C Conjugates and division
of complex numbers
2D Radians and coterminal
angles
2E Complex numbers in polar
form
2F Basic operations on complex
numbers in polar form
76 M a t h s Q u e s t M a t h s C Ye a r 1 1 f o r Q u e e n s l a n d

Introduction to complex numbers


• definition of complex numbers In 1545, the Italian mathematician Girolamo Cardano proposed (what was then) a start-
including standard and
trigonometrical (modulus- ling mathematical expression:
argument) form
• algebraic representation of 40 = (5 + – 15 )( 5 – – 15 )
complex numbers in
Cartesian, trigonometric and This was a valid expression, yet it included the square root of a negative number,
polar form which seemed ‘impossible’.
• geometric representation of
complex numbers — Argand What is a number such as – 1 or – 15 and how does it relate to a real number, and
diagrams
• operations with complex what does it signify in mathematics?
numbers including addition, Back in Chapter 1, you may recall that our definition of real numbers included
subtraction, scalar
multiplication, multiplication whole numbers, fractions, irrational and rational numbers as subsets of the real
of complex numbers, number set. Whenever the square root of a negative number was encountered was
conjugation
• simple, purely mathematical this classified as real? Where did we sometimes encounter such numbers in calcu-
applications of complex lations? Solution of quadratic equations sometimes brought these numbers to the
numbers
foreground. What was the difference between these two situations: x2 + 3x − 6 = 0
and x2 + 3x + 1 = 0? How did the solutions to these equations relate to properties
of the associated parabolas?
In terms of the mathematics that you have studied so far, these square roots of nega-
tive numbers have some significance.
But why did the square roots of negative numbers become central to the study of a
new set of numbers called the complex numbers? It was partly curiosity and partly
because mathematicians such as Diophantus (the Greek mathematician) and Leibniz
(the German mathematician) found that real numbers could not solve all equations.
Eventually scientists and engineers discovered their uses. Complex numbers are now
used extensively in the fields of physics and engineering in areas such as electric cir-
cuits and electromagnetic waves. Combined with calculus theory, complex numbers
form an important part of the study of mathematics known as complex analysis.

Square root of a negative number


The quadratic equation x2 + 1 = 0 has no solutions for x in the Real Number System R
because the equation yields x = ± – 1 and there is no real number which, when
squared, gives −1 as the result. If, however, we define an ‘imaginary number’ denoted
2
by i such that i 2 = −1, then x = ± – 1 becomes x = ± i = ±i. For the general case
x2 + a2 = 0, with a ∈ R, we can write:
2
x = ± –a
2
= ± –1 × a
2 2
=± i ×a
= ±ai
Powers of i will produce ±i or ±1. We have i 2 = −1, i 3 = i 2 × i = −1 × i = −i,
i 4 = i 2 × i2 = −1 × −1 = 1, i 6 = (i 2)3 = (−1)3 = −1 and so on. The pattern is quite obviously
that even powers of i result in 1 or −1 and odd powers of i result in i or −i.

Definition of a complex number


A complex number (generally denoted by the letter z) is defined as a quantity
consisting of a real number added to a multiple of the imaginary unit i. For real
numbers x and y, x + yi is a complex number. This is referred to as the standard or
Cartesian form.
Chapter 2 Number systems: complex numbers 77
C = {z: z = x + yi where x, y ∈ R} defines the set of complex numbers.
The real part of z is x and is written as Re (z). That is, Re (z) = x.
The imaginary part of z is y and is written as Im (z). That is, Im (z) = y.
Note: Every real number x can be written as x + 0i and so the set of real numbers is a
subset of the set of complex numbers. That is, R ⊂ C.

WORKED Example 1
Using the imaginary number i, write a simplified expression for:
a – 16 b –5 .

THINK WRITE
a 1 Express the square root of −16 as the product of a – 16 = 16 × – 1
the square root of 16 and the square root of −1.
2
2 Substitute i 2 for −1. = 16 × i
3 Take the square root of 16 and i2. = 4i
b 1 Express the square root of −5 as the product of b –5 = 5 × –1
the square root of 5 and the square root of −1.
2
2 Substitute i2 for −1. = 5 × i
3 Simplify. =i 5

WORKED Example 2
Write down the real and imaginary parts of the following complex numbers, z.
a z = −3 + 2i b z = − 1--2- i

THINK WRITE

a 1 The real part is the ‘non-i’ term. a Re (z) = −3


2 The imaginary part is the coefficient of Im (z) = 2
the i term.
b 1 The real part is the ‘non-i’ term. b Re (z) = 0

2 The imaginary part is the coefficient of Im (z) = − 1---


2
the i term.

WORKED Example 3
Write i 8 + i 5 in the form x + yi where x and y are real numbers.
THINK WRITE

1 Simplify both i 8 and i 5 using the lowest i 8 = (i 2)4 = (−1)4 = 1


possible power of i. i 5 = i 4 × i = (i 2)2 × i = (−1)2 × i = 1 × i = i
2 Add the two answers. i8 + i5 = 1 + i
78 M a t h s Q u e s t M a t h s C Ye a r 1 1 f o r Q u e e n s l a n d

WORKED Example 4
Simplify z = i 4 − 2i 2 + 1 and w = i 6 − 3i 4 + 3i 2 − 1 and show that z + w = −4.

THINK WRITE
1 Replace terms with the lowest possible i 4 − 2i 2 + 1 = (i 2)2 − 2 × −1 + 1
powers of i (remember i 2 = −1). = (−1)2 + 2 + 1
=4
i − 3i + 3i 2 − 1 = (i 2)3 − 3(i 2)2 + 3 × −1 − 1
6 4

= (−1)3 − 3(−1)2 − 3 − 1
= −1 − 3 − 3 − 1
= −8
2 Add the two answers. z + w = i 4 − 2i 2 + 1 + i 6 − 3i 4 + 3i 2 − 1
z+w=4−8
z + w = −4

WORKED Example 5
Evaluate each of the following.

d Im  ----------------------------------
1 – 3i – i 2 – i 3
a Re (7 + 6i) b Im (10) c Re (2 + i − 3i 3)
 2 

THINK WRITE
a The real part of the complex number a Re (7 + 6i) = 7
7 + 6i is 7.

b The number 10 can be expressed in b Im (10) = Im (10 + 0i)


complex form as 10 + 0i and so the =0
imaginary part is 0.

c 1 Simplify 2 + i − 3i 3. c Re (2 + i − 3i 3) = Re (2 + i − 3i × i 2)
= Re (2 + i + 3i)
= Re (2 + 4i)
2 The real part is 2. =2

1 – 3i + 1 + i
d Im  ---------------------------------- = Im  -------------------------------
d Simplify the numerator of 1 – 3i – i 2 – i 3
1
 2   2 
 1---------------------------------
– 3i – i 2 – i 3-
.
 2  = Im  --------------
2 – 2i
 2 
2 Simplify by dividing the numerator by 2. 2(1 – i)
= Im ------------------
2
= Im (1 − i)
3 The imaginary part is −1. = −1
Chapter 2 Number systems: complex numbers 79
remember
remember
1. The ‘imaginary number’ i has the property that i 2 = −1.
2. A complex number z is of the form z = x + yi where x, y ∈R.
3. The real part of z is x and is written as Re (z).
4. The imaginary part of z is y and is written as Im (z).

Introduction to
2A complex numbers
WORKED 1 Using the imaginary number i, write down expressions for:
Example
1 a – 9 3i b – 25 5i c – 49 7i d –3 3i

e – 11 11i f – 7 7i g – 4--- 2--3- i h – 36


------
6
---
5
i
9 25
WORKED 2 Write down the real and imaginary parts, respectively, of the following complex
Example
2
numbers, z.
a 9 + 5i 9, 5 b 5 − 4i 5, −4 c −3 − 8i −3, −8 d 11i − 6 −6, 11
e 27 27, 0 f 2i 0, 2 g –5 + i –5, 1 h –17i 0, –17
WORKED 3 Write each of the following in the form x + yi, where x and y are real numbers.
Example
3 a i 9 + i 10 −1 + i b i 9 − i 10 1 + i c i 12 + i 15 1 − i d i 7 − i 11 0 + 0i
−1 + 2i
e i 5 + i 6 − i7 f i(i 13 + i 16) −1 + i g 2i − i 2 + 2i 3 h 3i + i 4 – 5i 5 1 − 2i
1 + 0i
WORKED 4 Simplify z = i + 3i − 2i − 3 and w = 4i 8 − 3i 11 + 3 and show that z + w = 5.
6 7 10
Example
4 z = −2 − 3i, w = 7 + 3i

WORKED
5 Evaluate each of the following.
Example a Re (−5 + 4i) −5 b Re (15 − 8i) 15 c Re (12i) 0
5 d Im (1 − 6i) −6 e Im (3 + 2i) 2 f Im (8) 0

g Re (i 5 − 3i 4 + 6i 6) −9 h Im  4i -
9 – 5i 14 – 2i 7
----------------------------------- 2
 3 
i3 – i + 2
- in the form x + yi, where x and y are real numbers. 4 − i
6 Write 3 – --------------------
i2 – i4

7 multiple choice
a The value of Re (i + i 3 + i 5) is:
A 2 B −1 C 3 D 1 E 0
b The value of Im [i(2i 4 − 3i 2 + 5i)] is:
A 0 B −5 C 5 D 10 E 4
c The expression i + i − i + i − i + i simplifies to:
2 3 4 5 6

A i B 0 C i−1 D i+2 E −i
1+i+ +…+ i2 i 11
d If f ( i ) = ---------------------------------------------- which one of the statements below is true?
4
A f(i) = 2 + i B Re [f(i)] = 5 C Im [f(i)] = − 1---
4
D f(i) = 1 − i E f(i) = 0
n
---
8 If n is an even natural number show that ( – 1 ) 2 = i n . Check with your teacher.
80 M a t h s Q u e s t M a t h s C Ye a r 1 1 f o r Q u e e n s l a n d

Complex numbers in
SLE 1: Solve quadratic
equations whose discriminant
is negative.
quadratic equations
In your junior mathematics studies you graphed quadratics and found the real roots
Equation A: y = x2 − 2x −3 of the expressions using the formula for the solution of a general quadratic
1 Roots are −1 and 3, hence
equation of the form ax2 + bx + c. Sometimes the values for a, b and c meant that
there are two x-intercepts
at x = −1 and x = 3. The
the value under the radical sign was negative; that is, it had a negative discriminant
discriminant is positive (for example, – 16 ). You might have been told that this meant there were ‘no real
(b2 − 4ac = 16). roots’ for this quadratic. That was correct, but only half the answer. Follow the
2 (1, −4) steps below and you will hopefully develop a better understanding of the results
3 y you obtain.
2
y = x − 2x − 3
The following formulas are included for your assistance:
2
–b ± b – 4ac –b
−1 0 1 3
x = ------------------------------------- , turning point occurs where x = ------
2a 2a
−3 Step 1 Use the formula for the solution of a quadratic equation to find the roots
(1, −4)
of:
y = x2 − 2x − 3 ....................(A)
Equation B: y = x2 − 2x + 1 Interpret this result.
1 Root is 1, hence there is
one x-intercept at x = 1. Step 2 Use the formula for the x-coordinate of the turning point and substitute
Discriminant is 0 this into the original quadratic to find the y-coordinate of this turning
(b2 − 4ac = 0). point.
2 (1, 0)
3
Step 3 Graph this quadratic equation using the information from steps 1 and 2.
y y = x2 − 2x + 1
Repeat steps 1–3 with the following quadratic equations. Note the effect of the
negative discriminant in equation (C).
y = x2 − 2x + 1 ..................................(B)
1 y = x2 − 2x + 2 ..................................(C)
0 1 x Graphically, we can see that there are no real values of x that satisfy the equation
x2 − 2x + 2 = 0.

Basic operations using complex


numbers
Equation C: y = x2 − 2x + 2
1 No real roots, hence there Complex numbers can be added, subtracted, multiplied and divided. In general, the
are no x-intercepts. solutions obtained when performing these operations are presented in the standard form
Discriminant is negative z = x + yi.
(b2 − 4ac = −4).
2 (1, 1) Argand diagrams
3 y y = x − 2x + 2
2
We know that an ordered pair of real numbers (x, y) can be represented on the Cartesian
plane. Similarly, if we regard the complex number x + yi as consisting of the ordered
pair of real numbers (x, y), then the complex number z = x + yi can be plotted as a point
2
1
(x, y) on the complex number plane.
(1, 1)
0 1 x This is also referred to as the Argand plane or an Argand diagram in recognition of
the work done in this area by the Swiss mathematician Jean-Robert Argand.
In the complex system,
x2 − 2x + 2 = 0 has roots
1 + i and 1 − i.
Chapter 2 Number systems: complex numbers 81
The horizontal axis is referred to as the Real axis
and the vertical axis is referred to as the Imaginary Im (z) (Imaginary axis)
axis. 5
C 4
The points A, B, C, D and E shown on the 3
Argand diagram at right represent the complex 2 B
numbers 3 + 0i, 0 + 2i, −4 + 5i, −3 − 4i and 2 − 2i 1 A
respectively. –4 –3 –2 –1 1 2 3 4 Re (z)
–1 (Real axis)
This method of representation is a useful way of –2
E
illustrating the properties of complex numbers –3
D –4
under the operations of addition, subtraction and
multiplication.

SLE 4: Use geometry to


demonstrate the effect of
Addition of complex numbers
addition, subtraction and Addition is performed by adding the real and imaginary parts separately.
multiplication of complex
numbers.
If z = a + bi and w = c + di then z + w = (a + c) + (b + d)i
where Re (z + w) = Re (z) + Re (w) and Im (z + w) = Im (z) + Im (w).

Geometric representation
If z1 = x1 + y1i and z2 = x2 + y2i then z2 + z1 = (x2 + x1) + (y2 + y1)i. If a directed line
segment connects the origin (0 + 0i) to each of the points z1, z2 and z1 + z2, then the
addition of two complex numbers can be associated with standard methods of addition
of the directed line segments. Im (z)
The figure at right illustrates the situation for z2 + z1,
with, say, positive values for x1, x2, y1, y2 and x1 < x2 and
(y1 + y2) z1 + z 2
y1 < y2.
y2 z1
y1 z2
Note: The origin, z1, z2 and z2 + z1 form a parallelogram.
You will use this concept later in this course when you Re (z)
study vector addition. 0 x1 x2 (x1 + x2)

Subtraction of complex numbers


If we write z − w as z + −w we can use the rule for addition of complex numbers to
obtain z + −w = (a + bi) + − (c + di)
= a + bi − c − di
= (a − c) + (b − d)i

If z = a + bi and w = c + di then z − w = (a − c) + (b − d)i.

Geometric representation
Im (z)
If z1 = x1 + y1i and z2 = x2 + y2i then z2 − z1 = (x2 − x1)
+ (y2 − y1)i. If a directed line segment connects the
origin (0 + 0i) to each of the points z1, z2 and z2 − z1 y2 z2
then the subtraction of two complex numbers can also
be associated with standard methods of the addition of (y2 – y1) z1 z2 – z 1
directed line segments. The figure at right illustrates y1
the situation for z2 − z1, again with positive values for Re (z)
x1, x2, y1, y2 and x1 < x2 and y1 < y2. 0 x1 (x2 – x1) x2
82 M a t h s Q u e s t M a t h s C Ye a r 1 1 f o r Q u e e n s l a n d

WORKED Example 6
For z = 8 + 7i, w = −12 + 5i and u = 1 + 2i, calculate:
a z+w b w−z c u − w + z.
THINK WRITE
a Use the addition rule for complex numbers. a z + w = (8 + 7i) + (−12 + 5i)
= (8 − 12) + (7 + 5)i
= −4 + 12i
b Use the subtraction rule for complex b w − z = (−12 + 5i) − (8 + 7i)
numbers. = (−12 − 8) + (5 − 7)i
= −20 − 2i
c Use both the addition rule and the c u − w + z = (1 + 2i) − (−12 + 5i) + (8 + 7i)
subtraction rule. = (1 + 12 + 8) + (2 − 5 + 7)i
= 21 + 4i

Multiplication by a constant (or scalar)


If z = x + yi and k∈R Im (z)
then kz = k(x + yi) ky kz
= kx + kyi
y z
For k > 1, the product kz can be illustrated as shown at
right. The ratio of corresponding sides of the two triangles
is k:1.
A similar situation exists for k < 1. 0 x kx Re (z)
So when a complex number is multiplied by a constant, this produces a directed line
segment in the same direction (or at 180 degrees if k < 0) which is larger in length if
k > 1 or smaller if 0 < k < 1. Geometrically this is called a transformation or dilation,
which means magnifying or decreasing by a constant factor.

WORKED Example 7
If z = 3 + 5i, w = 4 − 2i and v = 6 + 10i, evaluate:
a 3z + w b 2z − v c 4z − 3w + 2v.
THINK WRITE
a 1 Calculate 3z + w by substituting values a 3z + w = 3(3 + 5i) + (4 − 2i)
for z and w. = (9 + 15i) + (4 − 2i)
2 Use the rule for adding complex numbers. = (9 + 4) + (15 − 2)i
= 13 + 13i
b 1 Calculate 2z − v by substituting values for b 2z − v = 2(3 + 5i) − (6 + 10i)
z and v.
2 Use the rule for subtraction of complex numbers. = 6 + 10i − 6 − 10i
= 0 + 0i
=0
c 1 Calculate 4z − 3w + 2v by substituting c 4z − 3w + 2v
values for z, w and v. = 4(3 + 5i) − 3(4 − 2i) + 2(6 + 10i)
2 Use the addition rule and the subtraction = 12 + 20i − 12 + 6i + 12 + 20i
rule to simplify. = 12 + 46i
Chapter 2 Number systems: complex numbers 83
Multiplication of two complex numbers
So far we have shown that complex numbers can be plotted on an Argand diagram;
adding and subtracting them is geometrically equivalent to adding and subtracting
directed line segments and multiplication by a positive constant is equivalent to
extending or shrinking the directed line segment without altering the direction.
What geometrical interpretation, if any, can be given to multiplication of two (or
more) complex numbers?
The multiplication of two complex numbers also results in a complex number.
If z = a + bi and w = c + di
then z × w = (a + bi)(c + di)
= ac + adi + bci + bdi 2
= (ac − bd) + (ad + bc)i (since i 2 = −1)

If z = a + bi and w = c + di then z × w = (ac − bd) + (ad + bc)i.

WORKED Example 8
If z = 6 − 2i and w = 3 + 4i express zw in standard form.
THINK WRITE
1 Expand the brackets. zw = (6 − 2i)(3 + 4i)
= 18 + 24i − 6i − 8i 2
2 Express in the form x + yi by substituting −1 for i 2 = 18 + 24i − 6i + 8
and simplifying the expression using the addition = 26 + 18i
and subtraction rules.

WORKED Example 9
Simplify (2 − 3i)(2 + 3i).
THINK WRITE
1 Expand the brackets. (2 − 3i)(2 + 3i) = 4 + 6i − 6i − 9i 2
2 Substitute −1 for i and simplify the expression. = 4 − 9 × −1
2

= 13

WORKED Example 10
Determine Re (z2w) + Im (zw2) for z = 4 + i and w = 3 − i.
THINK WRITE
1 Express z w in the form x + yi. z2w = (4 + i)2(3 − i)
2

= (16 + 8i + i 2)(3 − i)
= (16 + 8i − 1)(3 − i)
= (15 + 8i)(3 − i)
= 45 − 15i + 24i − 8i 2
= 53 + 9i
2 The real part, Re (z2w) is 53. Re (z2w) = 53
Continued over page
84 M a t h s Q u e s t M a t h s C Ye a r 1 1 f o r Q u e e n s l a n d

THINK WRITE
3 Express zw in the form x + yi.
2
zw2 = (4 + i)(3 − i)2
= (4 + i)(9 − 6i + i 2)
= (4 + i)(8 − 6i)
= 32 − 24i + 8i − 6i 2
= 38 − 16i
4 The imaginary part, Im (zw2), is −16. Im (zw 2) = − 16
5 Calculate the value of Re (z2w) + Im (zw 2) = 53 − 16
Re (z2w) + Im (zw2). = 37

Plotting complex numbers


You will need 1 cm square grid paper, a ruler and a protractor.
2 zw = −17 + 7i (see figure at left)
For z = −3 + 2i and w = 5 + i 3 Length of z = 13 , length of w = 26 ,
1
length of zw = 338
Im (z) 1 Accurately plot z and w on an Argand diagram. γ = 11.31°, β = 146.31°, α = 157.62°
zw = –17 + 7i Length of zw = length of z × length of w;
β 2 Find zw and plot this on the same diagram. α=γ+β
z = –3 + 2i
w=5+i
3 Measure each length and angle from the positive end on the Real axis. Do you
0 Re (z)
α γ notice any pattern between the numbers you started with and your result?

4 Try this with some other complex numbers. Plot your original complex
numbers accurately and plot the product. Test your original hypothesis.

Equality of two complex numbers


If z = a + bi and w = c + di then z = w if and only if a = c and b = d.
The condition ‘if and only if’ (sometimes written in short form as iff ) means that both
of the following situations must apply.
1. If z = w then a = c and b = d.
2. If a = c and b = d then z = w.

WORKED Example 11
Find the values of x and y that satisfy (3 + 4i)(x + yi) = 29 + 22i.

THINK WRITE
1 Write the left-hand side of the equation. LHS = (3 + 4i)(x + yi)
2 Expand the left-hand side of the equation. LHS = 3x + 3yi + 4xi + 4yi 2
3 Express the left-hand side in the form a + bi. LHS = (3x − 4y) + (4x + 3y)i
Chapter 2 Number systems: complex numbers 85
THINK WRITE
4 Equate the real parts and imaginary parts to 3x − 4y = 29 [1]
create a pair of simultaneous equations. 4x + 3y = 22 [2]

5 Simultaneously solve [1] and [2] for x and y. 9x − 12y = 87 [3]

Multiply equation [1] by 3 and equation [2] 16x + 12y = 88 [4]


by 4 so that y can be eliminated.

6 Add the two new equations and solve for x. Adding equations [3] and [4]:
25x = 175
x=7

7 Substitute x = 7 into equation [1] and solve Substituting x = 7 into equation [1]:
for y. 3(7) − 4y = 29
21 − 4y = 29
−4y = 8
y = −2

8 State the solution. Therefore x = 7 and y = −2.

9 Check the solution by substituting these values Check: 4 × 7 + 3 × −2 = 22.


into equation [2].

Multiplication by i
Let us examine the effect on z = x + yi after Im (z)
multiplication by i, i 2, i 3 and i 4.
z = x + yi iz x
iz = i(x + yi) = −y + xi z or i 4z
i 2z = −1z = −x − yi = −z
i 3z = i(i 2z) = y − xi = −iz –x –y –y x Re (z)
i 4z = i(i 3z) = x + yi = z
i 2z
The five points are shown on the complex plane
at right. i 3z
It is observed that multiplying z by i n, n ∈ N
produces an anticlockwise rotation of 90n degrees.

remember
remember
If z = a + bi and w = c + di for a, b, c, d∈R then:
1. z + w = (a + c) + (b + d)i
2. z − w = (a − c) + (b − d)i
3. kz = ka + kbi, for k∈R
4. z × w = (ac − bd) + (ad + bc)i
5. z = w if and only if a = c and b = d.
(Note: ‘If and only if’ can be written as iff.)
86 M a t h s Q u e s t M a t h s C Ye a r 1 1 f o r Q u e e n s l a n d

f Im (z)

Basic operations using


2B
–8 + i 3 2
1
complex numbers –8 0 Re (z)
a Im (z)
1 Represent each of the following complex numbers on an Argand diagram.
1 3+i a 3+i b 4 − 5i c −2 − 6i
0
d 3i + 7 e 5 – 2i f –8+i 3
1 2 3 Re (z)

WORKED 2 For z = 5 + 3i, w = −1 − 4i, u = 6 − 11i and v = 2i − 3 calculate:


Example
xample eBook plus
a z + w 4−i b u − z 1 − 14i c w + v −4 − 2i
6
d u − v 9 − 13i e w − z − u −12 + 4i f v+w−z Digital doc:
−9 − 5i SkillSHEET 2.1
WORKED 3 If z = −3 + 2i, w = −4 + i and u = −8 − 5i, evaluate: Operations with
complex numbers
Example
xample
7
a 3w −12 + 3i b 2u + z −19 − 8i c 4z − 3u 12 + 23i
d 3z + u + 2w −25 + 3i e 2z − 7w + 9u −50 − 48i f 3(z + 2u) − 4w −41 − 28i

WORKED 4 Using z, w, u and v from question 2 express each of the following in the form x + yi.
Example
xample
a zw 7 − 23i b uv 4 + 45i c wu −50 − 13i
8
d zu 63 − 37i e u 2 −85 − 132i f u(wv) 176 − 61i
WORKED 5 Simplify the following. 111 + 33i
Example
xample
a (10 + 7i)(9 − 3i) b (3 − 4i)(5 + 4i) 31 − 8i c (8 − 2i)(4 − 5i) 22 − 48i
9
d (5 + 6i)(5 − 6i) 61 e (2i − 7)(2i + 7) −53 f (9 − 7i)2 32 − 126i
6 For z = −1 − 3i and w = 2 − 5i calculate z 2w. 14 + 52i
WORKED 7 Determine Re (z 2) − Im (zw) for z = 1 + i and w = 4 − i. −3
Example
xample
10 8 For z = 3 + 5i, w = 2 − 3i and u = 1 − 4i determine:
a Im (u 2) −8 b Re (w 2) −5
c Re (uw) + Im (zw) −9 d Re (zu) − Im (w 2) 35
e Re (z 2) − Re (zw) − Im (uz) −30 f Re (u2w) + Im (zw 2) −115

WORKED 9 Find the values of x and y that satisfy each of the following. 21 ,
Example
xample
- y = – 16
a (2 + 3i)(x + yi) = 16 + 11i x = 5, y = −2 b (5 − 4i)(x + yi) = 1 − 4i x = ----- ------
11 41 41
c (3i − 8)(x + yi) = −23 − 37i x = 1, y = 5 d (7 + 6i)(x + yi) = 4 − 33i
x = −2, y = −3
10 multiple choice
If z = 8 − 7i and w = 3 + 4i, then:
a Re (zw) is equal to:
a Im (z) A −4 B 4 C 5 D 11 E 52
4 3 + 4i
b Im (w ) + Re (z ) is equal to:
2 2

A 76 B 39 C 105 D 56 E −32
0 3 Re (z) c 3z − 2w is equal to:
A 30 − 13i B 30 − 29i C 18 − 29i D 24 − 13i E 18
11 If z = 2 + i and w = 4 − 3i then represent each of the following on an Argand diagram.
a z2 b zw c z+w d w−z
e Im (z) e 3z + w f 2w − 4z g (z + w) 2
h (w − z)3 Im (z)
3 z, i4z, –i2z
12 If z = 3 + 2i, represent each of the following on the same Argand diagram. iz, i5z
2
0 10 Re (z) z, iz, i 2z, i 3z, i 4z, i 5z, −iz, −i 2z 1

–3 –2 –1–10 1 2 3 Re (z)
–2 i3z, –iz
i2z –3
Chapter 2 Number systems: complex numbers 87
Conjugates and division of
complex numbers
The conjugate of a complex number
The conjugate of a complex number is obtained by changing the sign of the imaginary
component.

If z = x + yi, the conjugate z of z is defined as z = x − yi.

Conjugates are useful because the multiplication (or addition) of a complex number
and its conjugate results in a real number.
Multiplication: zz– = (x + yi)(x − yi)
= x2 + y2
where x, y ∈ R, and x − yi and x + yi are conjugates. You will use this result when
dividing complex numbers.
Note: Compare this expression with the formula for the difference of two squares
(a − b)(a + b) = a 2 − b 2
Addition: z + z– = x + yi + x − yi
= 2x

Graphing complex conjugates


As seen earlier, z and its conjugate can be written as
z = x + yi and z– = x − yi
The geometrical representation of z and z– is Im (z)
shown at right.
z = x + yi
Notice that the conjugate z– appears as
a reflection of z in the Real axis. y
x Re (z)
Other properties of conjugates include:

1. z = z y
2. z1 ± z2 = z–1 ± z–2 z = x – yi
3. z1z2 = z–1z–2
z1  z1
4.  ------- = -------
- where z2 ≠ 0.
 z2  z2

WORKED Example 12
Write the conjugate of each of the following complex numbers.
a 8 + 5i b −2 − 3i c 4+i 5

THINK WRITE
a Change the sign of the imaginary component. a 8 − 5i
b Change the sign of the imaginary component. b −2 + 3i
c Change the sign of the imaginary component. c 4–i 5
88 M a t h s Q u e s t M a t h s C Ye a r 1 1 f o r Q u e e n s l a n d

WORKED Example 13
If z = 5 − 2i and w = 7 − i, show that z + w = z + w .

THINK WRITE

1 Add the conjugates z– and w . z– + w = ( 5 + 2i ) + ( 7 + i ) = 12 + 3i


2 Add z to w. z + w = ( 5 – 2i ) + ( 7 – i ) = 12 – 3i
3 Write down the conjugate of z + w. z + w = 12 + 3i

4 The conjugate of z + w equals zz + w . z + w = z– + w

Division of complex numbers


The application of conjugates to division of complex numbers will now be investigated.
z
Consider the complex numbers z = a + bi and w = c + di. To find ---- in the form x + yi
w
we must multiply both the numerator and denominator by the conjugate of w to make
the denominator a real number only.
(You might need to review rationalisation of the denominator which was discussed in
Chapter 1.)
a + bi
z- -------------
--- = -
w c + di
a + bi c – di
= -------------- × ------------- Multiply by the conjugate of c + di.
c + di c – di
( ac + bd ) + ( bc – ad )i
= ------------------------------------------------------
- Simplify the expressions in the numerator
c2 + d 2 and in the denominator.
( bc – ad )i-
ac + bd ------------------------
= ------------------ + 2 Express in the form x + yi.
c +d
2 2 c + d2

Thus we can state:


z
If z and w are complex numbers in the form x + yi, then ---- can also be expressed
w
in the form x + yi by simplifying:
z- w
--- ¥ ----
w w

WORKED Example 14
2+i
Express ----------- in standard form.
2–i
THINK WRITE
+ i (2
2---------- + i) (2 + i)
1 Multiply both the numerator and = --------------- × ---------------
2–i (2 – i) (2 + i)
denominator by the conjugate of 2 − i
to make the denominator real.
Chapter 2 Number systems: complex numbers 89

THINK WRITE
4 + 4i + i 2
2 Expand the expressions obtained in the = -----------------------
-
numerator and denominator. 4 – i2
4 + 4i – 1
3 Substitute −1 for i 2 and simplify the = -----------------------
4+1
expression.
3 + 4i
= --------------
5
3 4i
= --- + -----
5 5

Multiplicative inverse of a complex number


Given a non-zero complex number z, there exists a complex number w such that zw = 1,
1
with w being the multiplicative inverse of z denoted by w = z –1 = --- .
z

WORKED Example 15
If z = 3 + 4i, determine z−1.
THINK WRITE
1 1 1
1 Write z–1 as a rational expression: z –1 = --- z−1 = --- = --------------
z z 3 + 4i
1 (3 – 4i)
2 Multiply both the numerator and denominator = ------------------- × ------------------
(3 + 4i) (3 – 4i)
by the conjugate of 3 + 4i.
3 – 4i
= --------------
25
Write the expression in the form x + yi. 3 4i
3 = ------ – ------
25 25

a – bi
This example shows that if z = a + bi then z –1 = ----------------
-.
a2 + b2
Complex numbers can be used to generate fractal patterns such as the ‘Julia Set’ shown.
90 M a t h s Q u e s t M a t h s C Ye a r 1 1 f o r Q u e e n s l a n d

WORKED Example 16
2
If z = 3 + i and w = ---------- , determine Im (4z − w).
4–i
THINK WRITE
2
1 Substitute for z and w in 4z − w. 4z − w = 4 ( 3 + i ) – ----------
4–i
2 Express 4z − w with a common
4(3 + i)(4 – i) – 2
denominator. = --------------------------------------------
4–i
4 ( 13 + i ) – 2
= -------------------------------
4–i
50 + 4i
= -----------------
4–i
3 Remove i from the denominator by (50 + 4i) (4 + i)
= ---------------------- × ---------------
multiplying the numerator and (4 – i) (4 + i)
denominator by the conjugate of 4 − i.
196 + 66i
= -----------------------
17
196 66i
4 Simplify the expression so that it is in = --------- + --------
the form x + yi. 17 17
66
5 State the imaginary component of Im (4z − w) = ------
17
4z − w.

WORKED Example 17
Prove that z1z2 = z–1z–2.

THINK WRITE
1 When asked to ‘Prove’ you should not Let z1 = a + bi
use actual values for the pronumerals. z–1 = a − bi
State the general values of z1, z2, Let z2 = c + di and z–2 = c − di
z–1 and z–2.

2 Generally, in a proof do not work both LHS = (a + bi) × (c + di)


sides of the equation at once. Calculate LHS = ac + adi + bci + bdi2
the LHS first.
LHS = (ac − bd) + (ad + bc)i
LHS = (ac − bd) − (ad + bc)i

3 Calculate the RHS and show that it RHS = (a − bi)(c − di)


equals the LHS. RHS = ac − adi − bci + bdi2
RHS = (ac − bd) − (ad + bc)i
RHS = LHS
Hence z1z2 = z–1z–2
Chapter 2 Number systems: complex numbers 91
remember
remember
If z = a + bi and w = c + di, for a, b, c, d ∈ R, then:
1. The conjugate of z is z– = a − bi.
z ac + bd (------------------------
bc – ad )i
2. ---- = ------------------ + 2 -.
w c2 + d 2 c + d2 2
1 a b a Im (z)
3. The multiplicative inverse of z is z–1 = --- = ----------------- – ----------------
2
-i .
2
z 2 2 z=3+i
a +b a +b
Re (z)
z= 3 – i
b Im (z)
z = –1 + 3i

Conjugates and division


2C of complex numbers Re (z)

z = –1 – 3i
WORKED 1 Write down the conjugate of each of the following complex numbers.
Example
12
a 7 + 10i 7 − 10i b 5 − 9i 5 + 9i c 3 + 12i 3 − 12i
d 7 – 3i 7 + 3i e 2i + 5 5 − 2i f −6 − i 11 – 6 + 11 i

2 Graph the following complex numbers and their conjugates on an Argand diagram.
a 3−i b −1 + 3i c −4 − 5i z = – 4 + 5i Im (z)

WORKED 3 If z = 6 + 3i and w = 3 − 4i, show that z – w = z– – w .


Example Re (z)
13 Check with your teacher.
2+i
WORKED 4 Express ---------- in the form x + yi. 1--2- + 1--2- i z = – 4 –5i
Example 3–i
14 5 Express each of the following in the form x + yi.
1–i 3 – 2i + 5i- − -----7- +
2------------- 26
a ---------- 0 − i b -------------- 0 − i c ------ i
1+i 2 + 3i 4 – 3i 25 25

4 – 3i −
4-------------
– 5i- 43
+ 18 2 + i 3- 2 5 – 6 2--------------------------
2 + 15
d -------------- 14
------ 23
------ i e ------ ------ i f --------------------- ----------------------- + -i
5 + 2i 29 29
2 – 7i 53 53
5–i 2 7 7

WORKED 6 Determine z−1 if z is equal to: 2+i 3–i


Example 6 a ----------- b ----------
15
a 2−i b 3+i c 4 − 3i 5 10
5 – 4i – 3 – 2i
d 5 + 4i -------------
41
- e 2i − 3 ------------------ f 3–i 2 4 + 3i
13 c --------------
−1 25
7 If 676z = 10 − 24i, express z in the form x + yi. 10 + 24i 3 + 2i
----------------------
5
1
WORKED 8 If z = 2 − i and w = ---------- determine each of the following:
Example 3+i
16 a Re (z + w) 23 ------ b Im (w − z) -----
9-
c Re (z−1 + w−1) 17
------
10 10 5
d Im (3z + 2w) − -----5-16
e Re (4w − 2z) − -----5-
14

2 + i 9 – 2i 7 + i
9 Write ---------- + -------------- + ---------- in the form x + yi. 17
------ + 9--2- i
1+i 2–i 1–i 2

( 2 + 5i ) 2 ( 5i – 2 )
10 Simplify --------------------------------------------------- . –29
3 ( 4 + 7i ) – 2 ( 5 + 8i )
11 Determine the conjugate of (5 − 6i)(3 − 8i). −33 + 58i
92 M a t h s Q u e s t M a t h s C Ye a r 1 1 f o r Q u e e n s l a n d

12 multiple choice
If z = 5 − 12i, w = −9 − i and u = 15 − 6i, then:
a Re (z−1) is:
12 5 12i
A 5 B 12 C --------- D --------- E ---------
169 169 169
b Im ( zw ) is equal to:
A −33 B 103 C 113 D 70 E 0
c The expression 2z – w + 3u is equal to:
A 26 − 7i B 64 + 41i C 46 − 29i D 34 − 41i E −64 − 19i

13 If z = 6 + 8i and w = 10 − 3i: Check with your teacher.


a show that zw = z × w
b generalise the result by letting z = a + bi and w = c + di.

14 Use the result zw = z × w to prove that z n = ( z )n. Check with your teacher.
eBook plus
15 If z = 4 + i and w = 1 + 3i Check with your teacher.
Digital doc:
SkillSHEET 2.2
a show that  ---- = ----
Complex numbers and
z z
their conjugates  w w
b generalise the result by letting z = a + bi and w = c + di.

16 If z = −5 − 4i and w = 2i, calculate Re (z w + z w). −16

17 If z1 = 2 + 3i, z2 = −4 − i and z3 = 5 − i find:


a 2z1 − z2 − 4z3 −12 + 11i b z1 z 2 + z2 z 3 −30 − 19i c z1 z2 z3 − z1 z2 z3 0

18 If z1 = a + bi and z2 = c + di show that (z1z2)−1 = z1−1z2−1. Check with your teacher.

19 a If z = 1 + i, find z4, z8 and z12. –4, 16, –64


b Deduce from your results in a that z4n = (2i)2n, n ∈ N.

z–1
20 If z = x + yi, find the values of x and y such that ----------- = z + 2 . x = −1, y = ± 2
z+1

z+i
21 Find values for a and b so that z = a + bi satisfies ----------- = i . a = − 1--2- , b = 1
---
z+2 2

22 If z = x + yi, determine the values of x and y such that z = 3 + 4i . x = 2, y = 1; x = −2, y = −1

WORKED 23 If z = 2 − 3i and w = 1 − 2i
Example
xample
17 a find i zz– 13 ii ww– 5
b show that iii z + w = z + w
iii zw = z– × w

iii  ---
z- ---
z
= -
eBook plus  w w

i  --- ii  ----
Digital doc: 1 1 – 4 + 7i
c find d z2 + w2 −8 + 16i e z + zw −2 + 10i f z–1w–1 ------------------
WorkSHEET 2.1  z  w 65
2-
-----
13
− 3-
-----
13
i --1- − --25- i
5
Chapter 2 Number systems: complex numbers 93
Graphics Calculator tip! Simple algebra of
complex numbers
Operations with complex numbers, finding the real and imaginary parts of a complex
number and finding the complex conjugate can be achieved with a graphics calculator.
You may not need to use a graphics calculator with simple complex numbers but it can
be useful in more complicated questions.

For the Casio fx-9860G AU


Operations with complex numbers
1. Press MENU to display the MAIN MENU. Use the
cursor keys to highlight RUN-MAT. Select it by
pressing EXE .
To perform simple algebra on complex numbers, use
the standard keys to enter the expression. To enter i,
press SHIFT [i ]. (The i is located above the zero
key.) Press EXE to obtain the answer. For example:
(a) Input (2 − 2i)(1 + 3i) and then press EXE .
(b) Input (2 − 2i) ÷ (1 + 3i) and then press EXE .
(c) Input (2 − 2i)^3 and then press EXE .
Notice that we include a multiplication symbol when entering 2i and 3i in these
examples. The calculator will generally assume that 2i means 2 × i. However, with
some terms, the calculator may not read it as you intend so it is good practice to
press the multiplication key each time the multiplication operation is needed. Try
2
calculating ( 1 – 2i ) with and without including a multiplication sign for 2i .
2. A complex number can be stored and then retrieved
if a number of operations need to be performed on it.
Input 2 − 2i and then press Æ followed by ALPHA
[Z] to store this expression as the variable z. Press
EXE .
Input 1 + 3i and then press Æ followed by ALPHA
[W] to store this expression as the variable w. Press
EXE .
3. We can now calculate expressions involving z and/or
w; for example, z × w, z ÷ w and z^3. Press EXE to
obtain each answer.

Features of a complex number


1. To find the complex conjugate, the real part or the
imaginary part of a complex number or expression,
first press OPTN then F3 (CPLX). You will see a
row of complex number options. There is also a
s

second row of options available. Press F6 ( ) to


move between the two rows.
94 M a t h s Q u e s t M a t h s C Ye a r 1 1 f o r Q u e e n s l a n d

2. You may then select F4 (Conj) for the complex

s
conjugate or F6 ( ) followed by F1 (ReP) for the

s
real part or F6 ( ) followed by F2 (ImP) for the
imaginary part as required. Type the complex
number within brackets and press EXE .
3. Alternatively, you can enter an expression, for
example (2 − 2i)(1 + 3i), and then press Æ
followed by ALPHA [A] to store the output as the
variable A. Press EXE .
4. To access the complex number options, first press
OPTN then F3 (CPLX). Remember that you can
move between the two rows of complex number

s
options by pressing F6 ( ).
(a) To find ( 2 – 2i ) ( 1 + 3i ) , press F4 (Conj). Key
in the variable assigned to the stored data (A in
this case) and press EXE .
(b) To find Re [ ( 2 – 2i ) ( 1 + 3i ) ], press F6 ( ) to access the second row of options

s
and then press F1 (ReP). Key in the variable assigned to the stored data and
press EXE .
(c) To find Im [ ( 2 – 2i ) ( 1 + 3i ) ], press F6 ( ) to access the second row of options
s
and then press F2 (ImP). Key in the variable assigned to the stored data and
press EXE .

For the TI-Nspire CAS


Operations with complex numbers
1. Open a new Calculator document.
To perform simple algebra on complex numbers, use
the standard keys to enter the expression. Use the j
button located on the left of the calculator. Press ·
to obtain the answer. For example:
(a) input ( 2 – 2i ) ( 1 + 3i ) and then press ·.
(b) input ( 2 – 2i ) ÷ ( 1 + 3i ) and then press ·.
(c) input (2 − 2i)^3 and then press ·.
Notice that a multiplication symbol (a dot) appears on the screen for any
multiplication operations in these examples. The calculator has assumed that 2i
means 2 × i. However, if there were two or more variables in a term, the calculator
may not read it as you intend so it is good practice to press the multiplication key
each time the multiplication operation is needed.
2. A complex number can be stored and then retrieved
if a number of operations need to be performed on it.
Input 2 − 2i and then press /h and Z to store
this expression as the variable z. Press ·.
Input 1 + 3i and then press /h and W to store
this expression as the variable w. Press ·.
We can now calculate expressions involving z and/or
w; for example, z × w, z ÷ w and z^3. Press · to
obtain each answer.
Chapter 2 Number systems: complex numbers 95
Features of a complex number
1. To find the complex conjugate, the real part or the
imaginary part of a complex number or expression,
press b and select 2: Number, then 8: Complex
Number Tools. A list of complex number options
will be displayed.

2. You may then select 1: Complex Conjugate or


2: Real Part or 3: Imaginary Part as required.
Type in the complex number, close the brackets
and press ·.

3. Alternatively, you can enter an expression, for


example, (2 − 2i)(1 + 3i), and then press /h and
A to store the output as the variable a. Press ·.
(a) To find ( 2 – 2i ) ( 1 + 3i ) , press b and select
2: Number, then 8: Complex Number Tools and
1: Complex Conjugate. Key in the variable
assigned to the stored data (A in this case) and
close the brackets. Press ·.
(b) To find Re [ ( 2 – 2i ) ( 1 + 3i ) ], press b and select 2: Number, then 8: Complex
Number Tools and 2: Real Part. Key in the variable assigned to the stored data
and close the brackets. Press ·.
(c) To find Im [ ( 2 – 2i ) ( 1 + 3i ) ], press b and select 2: Number, then 8: Complex
Number Tools and 3: Imaginary Part. Key in the variable assigned to the stored
data and close the brackets. Press ·.

Radians and coterminal angles


When a complex number is expressed in a geometrical representation, we use a
directed line segment which has length (modulus) and which lies in a certain direction
with respect to the positive Real axis. This angle formed with the positive Real axis is
called the argument. The argument of a complex number z is written as arg (z) and arg
(z) = θ.
Before we look at complex numbers in polar form (in the next section), a new unit of
measuring angles is needed, the radian.

Radian measure
A radian is the angle subtended by an arc the length of the radius of
a circle, as shown in the diagram on the right. r r
Because the circumference of a circle is given by c = 2π r, there are
2π radians in one complete circle. Taken in an anticlockwise θ
rotation from the positive end of the x-axis as shown, the common 1 radian r
angles have radian equivalents.
96 M a t h s Q u e s t M a t h s C Ye a r 1 1 f o r Q u e e n s l a n d

360° π
Therefore, if 2π radians = 360°, 1 radian = ----------- g 57.3°. –
2

So an arc of 2π r subtends an angle of 2π radians.
π
Coterminal angles π –
4 0

Consider the angle 420°. This angle is made up of a full
revolution, 360° plus 60°. When using degrees as our unit
of angle measure, 420° and 60° are said to be coterminal
angles; that is, angles which differ by a multiple of 360°. π
3—
2

2D Radians and coterminal angles


Im (z) 1 Draw a circle with a set of axes through its centre. Mark the following on the circum-
π
3— π
4
π


4 ference of the circle.
2
π π π 3π π 3π 5π 7π π 5π 7π
a 0, --- , π, ------ , 2π
5—
π 6

6
0
b --- , ------ , ------ , ------ c --- , ------ , ------
2π Re (z)
2 2 4 4 4 4 6 6 6
7—π
6 2 Convert the following common angle measures to radians.
π
5— π
7— π π 3π 3 π- 5π
4 π
3— 4 a 45° --4- b 60° --- c 135° ----- - d 270° -----
2
e 150° ------
2
3 4 6
3 Convert the following radian measures to degrees.
7π 5π 4π 5π
Im (z) a ------ 210° b ------ 225° c ------ 240° d ------ 300°
6 4 3 3
(c)
(b)
(a) (d)
4 Draw the following sets of coterminal angles:
a 30°, 390° b 60°, 420° c 135°, 495°
π 13 π 11 π 23 π 5 π- 13 π
Re (z) d --- , --------- e --------- , --------- f ----- , ---------
(e) 6 6 6 6 4 4
(f)

Complex numbers in polar form


The modulus of z Im (z)
The magnitude (or modulus or absolute value) of the P(x, y)
complex number z = x + yi is the length of the line segment y
z = 2
x +y 2 z = x + yi
joining the origin to the point z. It is denoted by
z, x + yi or mod z. y
The modulus of z is calculated using Pythagoras’ theorem.
z = x 2 + y 2 so that we have zz = z 2 . θ Re (z)
0 x
x

WORKED Example 18
Find the modulus of the complex number z = 8 − 6i.
THINK WRITE
Calculate the modulus by rule. z = 8 2 + ( –6 )2
Because it forms the hypotenuse of a right-
= 100
angled triangle, the modulus is always
greater than or equal to Re (z) or Im (z). = 10
Chapter 2 Number systems: complex numbers 97
WORKED Example 19
If z = 4 + 2i and w = 7 + 6i, represent the position of w − z on an Argand diagram and
calculate w − z.
THINK WRITE/DRAW
Im (z)
1 Calculate w − z. w − z = 7 + 6i − (4 + 2i) 6 w
= 3 + 4i 5 w–z
2 Represent it on an Argand 4 P
3
diagram as a directed line 2 z
segment OP. 1
O Re (z)
0 1234567
3 Use Pythagoras’ theorem to OP 2 = 3 2 + 4 2 = 25
determine the length of OP. OP = 5
So w – z = 5

WORKED Example 20
Represent z1 = 2 + 3i, z2 = 5 − 2i and z3 = −4 − 2i on the complex number plane and
calculate the area of the shape formed when the three points are connected by straight
line segments.
THINK WRITE/DRAW
1 Show the connected points on the complex Im (z)
number plane. 4
3 z1
2
1
Re (z)
–4–3–2 –1 1 2 3 4 5
z3 –2 z2
–3

2 Calculate the area of the triangle obtained. Area of triangle = 1--- × 9 × 5


2
The length of the base and height can be Area of triangle = 22.5 square units.
found by inspection (base = 9, height = 5).

The argument of z
The argument of z, arg (z), is the angle measure- Im (z)
ment anticlockwise of the positive Real axis.
In the figure at right, arg (z) = θ, where P(x, y)
y z = x + yi
y x y
sin θ = -------- and cos θ = -------- or tan θ = --
z z x
z 
As seen in the previous section, for non-zero z
y
an infinite number of arguments of z exist since,
for a given z {θ :θ = ±2nπ, n ∈ N} also represents
θ
the position of point P in the figure at right x
0 Re (z)
because a clockwise or anticlockwise rotation x
consisting of multiples of 2π radians (or 360°)
merely moves P to its original position.
98 M a t h s Q u e s t M a t h s C Ye a r 1 1 f o r Q u e e n s l a n d

To ensure that there is only one value of θ corresponding to


z we refer to the principal value of θ and denote it by Arg (z).
Note the capital A.
π
–π
Arg (z) is the angle θ in the range −π < θ ≤ π or (−π, π].

Exact values
π 1 π 1 π
cos --- = ------- , sin --- = ------- , tan --- = 1
4 2 4 2 4
π 1 π 3 π
cos --- = --- , sin --- = ------- , tan --- = 3
3 2 3 2 3
π 3 π 1 π 1
cos --- = ------- , sin --- = --- , tan --- = -------
6 2 6 2 6 3

Some useful triangles


It will be easier if you remember these 2 triangles only — not the ratios shown above.
Draw a quick sketch and work out each trigonometric ratio when you need to.


6

4
√2 1 2 √3

— —
4 3
1 1

WORKED Example 21
Find the Argument of z for each of the following in the interval (−π, π ].
a z = 4 + 4i b z=1− 3i

THINK WRITE/DRAW
a 1 Plot z. a Im (z)
2 Sketch the triangle that has sides in this
1:1 ratio. 4
π Re (z)

4 θ
4
√2 1
π
From the diagram

4 π
1 θ = ---
4
π
∴ Arg (z) = ---
4
3 This result can be verified using an Check:
y 4
inverse trigonometric ratio, θ = tan–1 -- . θ = tan–1 ---
x 4
π
θ = ---
4
Chapter 2 Number systems: complex numbers 99

THINK WRITE/DRAW
b 1 Plot z. b Im (z)
2 Sketch the triangle that has sides in this
ratio. 1 Re (z)
θ
π

6
√3
2
√3

π

3 From the diagram
1 π
θ = ---
3
π
∴ Arg (z) = − ---
3
3 This result can be verified using an Check:
y – 3
inverse trigonometric ratio, θ = tan–1 -- . θ = tan–1 ----------
x 1
π
θ = − ---
3

WORKED Example 22
7π 5π
Convert each of the following into Arguments. a ------ b – ------
4 2
THINK WRITE/DRAW
a 1 Sketch the angle. a Im (z)

π
7— Re (z)
4 – –4π


2 Since the given angle is positive, Arg (z) = ------ – 2 π
subtract multiples of 2π until it lies in 4
π
the range (−π, π ]. = – ---
4
b 1 Sketch the angle. b Im (z)

Re (z)
– –2π

– 5—2π


2 Since the given angle is negative, add Arg (z) = – ------ + 2 π
multiples of 2π until it lies in the range 2
π
(−π, π ]. = – ---
2
100 M a t h s Q u e s t M a t h s C Ye a r 1 1 f o r Q u e e n s l a n d

WORKED Example 23
Find the modulus and Argument for each of the following complex numbers.
a 3+i b – 1 – 2i

THINK WRITE/DRAW
a 1 Plot z. a Im (z)
2 This triangle has sides 1
π
in the same ratio as –
6 θ Re (z)
2 √3
√3

π
– z = 2
π
3
1 Arg (z) = ---
6
3 These results can be verified by Check:
2 2
calculating the modulus and Argument z = ( 3 ) + 1
by rule. = 4
=2
1 π
θ = tan–1 ------- = ---
3 6
b 1 Plot z. b Im (z)

1 Re (z)
α θ
√2

2 2
2 Find the modulus. z = ( –1 ) + ( – 2 )
z = 1+2
z = 3
y
3 The triangle in the third quadrant will α = tan–1 --
x
be used to find α but the answer will be
finally expressed as θ and Arg (z). – 2
= tan–1 ----------
–1
= 0.955
θ = −π + 0.955
4 Remember Arg (z) Arg (z) = −2.187
can be thought of as
the quickest way to
get to z. Arg (z)

z
Chapter 2 Number systems: complex numbers 101
Graphics Calculator tip! Modulus
Argument
and

Your graphics calculator is also able to calculate the modulus and Argument of a
complex number. Consider 3 + i and – 1 – 2i from Worked example 23.

For the Casio fx-9860G AU


Modulus (or absolute value) of a complex number
1. Press MENU to display the MAIN MENU and select
RUN-MAT. Press OPTN then F3 (CPLX) to display
the complex number options. To select the modulus
or absolute value option, press F2 (Abs).
2. (a) Enter ( 3 + i ) and press EXE . (Press SHIFT
[ ] and type in 3 for 3 .)
(b) The modulus of –1 – 2i is also shown in the
screen at right.
(c) The modulus of a stored complex number can
also be calculated. For example, using the
expression from the previous graphics calculator
tip where we assigned the variable A to
( 2 – 2i ) ( 1 + 3i ) , we can obtain the value as
shown in the screen at right.
Argument of a complex number
1. Decide whether you want the angle to be shown in radians or degrees. See the
instructions below for changing the system settings for Angle.
2. As before, press OPTN then F3 (CPLX) to display the complex number options. To
select the Argument option, press F3 (Arg).
3. Enter the required complex number 3 + i within
brackets and press EXE to obtain the answer. If the
calculator is set to radians, the answer will be shown
π
as the decimal equivalent to --- . If the calculator is set
6
to degrees, the answer will be shown as 30 for 30°.
4. (a) The Argument of –1 – 2i is shown in degrees
as a decimal in the screen at right.
(b) To convert this angle to degrees, minutes and
seconds, we can first press SHIFT [Ans] and
EXE to display the decimal answer again. (This
step is optional and used to show both answers on
s

the screen.) Press OPTN and then F6 ( ) for


more options. Press F5 (ANGL) followed by
F5 ( ∞¢¢¢ ) to obtain the answer in degrees,
minutes and seconds.
(c) The Argument of a stored complex number can also be calculated. See the screen
above for the Argument of the variable A where A is assigned to
( 2 – 2i ) ( 1 + 3i ) . The calculator is set to degrees for this example.
102 M a t h s Q u e s t M a t h s C Ye a r 1 1 f o r Q u e e n s l a n d

Changing the system settings for Angle


To change the settings for Angle, press SHIFT [SET
UP] and use the down arrow to highlight Angle. Press
F1 (Deg) to set degrees or press F2 (Rad) to set
radians.

For the TI-Nspire CAS


Modulus (or magnitude) of a complex number
1. Press b and select 2: Number, then 8: Complex
Number Tools and 5: Magnitude. A pair of modulus
signs appears with a box to enter the required
complex number.
2. (a) To enter 3 + i , press / then q and type in 3
for 3 . Use the right arrow of the NavPad to move
the cursor out from under the square root sign.
Press + and j to complete the expression.
Again use the right arrow of the NavPad to move
the cursor to the right of the second modulus sign.
Press · to obtain the answer.
(b) The modulus of –1 – 2i is also shown in the
screen at right.
(c) The modulus of a stored complex number can also be calculated. For example,
using the expression from the previous graphics calculator tip where we assigned
the variable a to ( 2 – 2i ) ( 1 + 3i ) , we can obtain the magnitude as shown in the
screen above.
Argument of a complex number
1. Decide whether you want the angle to be shown in radians or degrees. See the
instructions below for changing the system settings for Angle.
2. Press b and select 2: Number, then 8: Complex
Number Tools and 4: Polar Angle.
3. Enter the required complex number 3 + i , close the
brackets and press · to obtain the answer. If the
calculator is set to radians, the answer will be shown
π
as --- . If the calculator is set to degrees, the answer
6
will be shown as 30 for 30°.
4. (a) The Argument of –1 – 2i is shown as an exact
answer in radians in the screen at right.
(b) For an approximate answer, press / then ·.
The answer is now shown as a decimal in radians.
(c) To convert this angle in radians to degrees,
minutes and seconds, first press / then v and
select DMS in the catalog (press k, select 1,

scroll down to DMS and press ·).


Now that you are back in the calculator screen, press · to obtain the answer.
Note: You can convert degrees to radians by selecting Rad in the catalog.

Chapter 2 Number systems: complex numbers 103
5. (a) The Argument of a stored complex number can
also be calculated. See the screen at right for the
Argument of the variable a where a is assigned to
( 2 – 2i ) ( 1 + 3i ) . The exact answer is shown.
(b) For the approximate answer, press / then ·.
The calculator is set to degrees for this example.

Changing the system settings for Angle


1. To change the settings for Angle, press the home key
(c) and select 8: System information and then
2: System Settings. Use the tab key (e) to move
down to Angle and then use the down arrow to select
Degree or Radian as required.

2. Press · to accept your selection. Tab to OK and


press ·. A message will appear asking whether to
apply these system settings to current document
settings. Select Yes and press ·.
Note that you can also change the settings for
Angle by accessing the setup menu (press / [#])
and selecting 1: File and then 6: Document
Settings.

Expressing complex numbers in polar form


Suppose z = x + yi is represented by the point P(x, y)
on the complex plane using Cartesian coordinates. Im (z)
P(x, y)
Using the trigonometric properties of a right- y
angled triangle, z can also be expressed in polar
r = z 
coordinates as follows. We have: y
x
cos θ = -- or x = r cos θ Re (z)
r θ
0 x
y
sin θ = -- or y = r sin θ
r
where z = r = x 2 + y 2 and θ = Arg (z).

The point P(x, y) in polar form is shown at right. Im (z)


Now z = x + yi in Cartesian form becomes y P(rcosθ , r sinθ )

z = r cos θ + r sin θ i (after substitution of x = r cos θ,


y = r sin θ ) r r sin θ
= r (cos θ + i sin θ)
O θ Re (z)
= r cis θ, where cis θ is the abbreviated form of 0 r cosθ
cos θ + i sin θ.
(Note: The acronym ‘cis’ is pronounced ‘sis’.)
104 M a t h s Q u e s t M a t h s C Ye a r 1 1 f o r Q u e e n s l a n d

WORKED Example 24
Express each of the following in polar form, r cis q, where q = Arg (z).
a z=1+i b z=1− 3i

THINK WRITE/DRAW
a 1 Plot z. a Im (z)

1
Re (z)
1

2 The ratio of the sides of π


From the diagram

this triangle matches the 4
z = 2
√2
following special 1 π
triangle: θ = ---
π

4
4
1

3 These results can be verified by rule: Check:


(a) Find the value of r using r = 12 + 12
r = z = x2 + y2 . = 2
y 1
(b) Determine tan θ from tan θ = -- and tan θ = ---
hence find θ. x 1
=1

π
4 The angle θ is in the range (−π, π ], which is θ = ---
required. 4

π π
5 Substitute the values of r and θ in z= 2 cos --- + 2 sin --- i
z = r cos θ + r sin θ i = r cis θ. 4 4
π
z= 2 cis ---
4
b 1 Sketch z. b Im (z)

1 Re (z)
θ

√3

2 The ratio of the sides of From the diagram


this triangle is the same π

6
r=2
as that in the following π
special triangle:
2 θ = ---
√3 3
π
π
– Arg (z) = − ---
3 3
1
Chapter 2 Number systems: complex numbers 105

THINK WRITE/DRAW
3 These results can be verified by rule: Check:
(a) Calculate the value of r. r= 1 + ( 3)2
r=2
3
(b) Determine the appropriate value of θ. tan θ = – -------
1
=– 3

θ = – --π-
3
π π
z = 2 cos  – --- + 2 sin  – --- i
4 Substitute for r and θ in  3  3
z = r cos θ + r sin θ i and write in the form
r cis θ. π
= 2 cis  – ---
 3

Graphics Calculator tip! Expressing complex


numbers in polar form
Complex numbers in Cartesian form (also known as rectangular form) can be written in
polar form if we know the modulus and the Argument. Consider 1 + i and 1 – 3i
from Worked example 24.

For the Casio fx-9860G AU


1. Press MENU to display the MAIN MENU and
select RUN-MAT. Press OPTN then F3 (CPLX)
to display the complex number options. To access
the second row of complex number options, press
s

F6 ( ).

2. Enter the complex number in Cartesian form. To


convert the number to polar form, press F3
( r– q ). Press EXE to show the modulus and

Argument.
For 1 + i, we can see that the modulus is
1.41421… (which is the decimal equivalent to
2 ) and the Argument is 45° (as the calculator is
set to degrees). For 1 – 3i , the modulus is 2 and
the Argument is −60°.

3. If the calculator is set to radians, the Argument


will be displayed in radians as a decimal.
106 M a t h s Q u e s t M a t h s C Ye a r 1 1 f o r Q u e e n s l a n d

For the TI-Nspire CAS


1. From the home screen, select 1: Calculator. Enter
the complex number in Cartesian form. Press b
and select 2: Number, then 8: Complex Number
Tools followed by 6: Convert to Polar. Press · to
show the modulus and Argument. For 1 + i, we can
see that the modulus is 2 and the Argument is 45°
(as the calculator is set to degrees). For 1 – 3i , the
modulus is 2 and the Argument is −60°.
2. If the calculator is set to radians, the answer is
shown in a different format but we can still see
the modulus and Argument. Another way of iπ
π
expressing 2 cis --- (the polar form of 1 + i) is
-----
2e 4 .
4
(This is beyond the scope of this course.)

Expressing complex numbers in Cartesian form


WORKED Example 25
π
Express 3 cis --- in Cartesian (or standard a + ib) form.
4
THINK WRITE
1 Sketch z. Im (z)

3
π– Re (z)
4

π π π π
2 Express 3 cis --- in Cartesian form. 3 cis --- = 3 cos --- + 3 sin --- i
4 4 4 4
3 Simplify using exact values from the 1 1
following triangle: = 3 × ------- + 3 × ------- i
2 2
π

= ------- +  ------- i
4 3 3
√2 1 2  2
π 3

4 = ------- ( 1 + i )
1 2

Graphics Calculator tip! Expressing complex numbers in


Cartesian form
Complex numbers in polar form can be written in Cartesian form by entering both the
π
modulus and the Argument into the calculator. Consider 3 cis --- from Worked
4
example 25.
π
The modulus is 3 and the Argument is --- or 45°.
4
Chapter 2 Number systems: complex numbers 107
For the Casio fx-9860G AU
1. Press MENU to display the MAIN MENU and select
RUN-MAT. Press OPTN then F3 (CPLX) to display
the complex number options. Access the second row

s
of complex number options by pressing F6 ( ).

2. Enter the modulus (3 in this case) and then press


π
SHIFT [–] and enter the Argument in radians ( --- in
this case) or degrees (45°) as appropriate to the 4
calculator setting. The screen at right shows the
angle in radians. To convert to Cartesian form
(a + bi form), press F4 ( a + bi). Press EXE to


show the complex number.
3. If the calculator is set to degrees with the Argument
entered as 45, the same result is obtained.

For the TI-Nspire CAS


1. Open a new Calculator document. Enter the
modulus (3 in this case) then access the symbol
palette (press / then k) and highlight the angle
symbol ∠.

2. Press ·. Complete the entry line by entering the


π
angle in radians ( --- in this case) or degrees (45°) as
4
appropriate to the calculator setting. The screen at
right shows the angle in radians. Press ) to close
the set of brackets.

3. Press b and select 2: Number, then 8: Complex


Number Tools followed by 7: Convert to
Rectangular. Press ·.
3 3 2
The fraction ------- is equivalent to ----------
2 2
(shown with a rational denominator).

4. If the calculator is set to degrees with the Argument


entered as 45, the same result is obtained.
108 M a t h s Q u e s t M a t h s C Ye a r 1 1 f o r Q u e e n s l a n d

History of mathematics
ABRAHAM DE MOIVRE
(26 May 1667 – 27 November 1754)

During his lifetime. . . had been the first to discover


Christopher Wren finishes calculus. He was appointed by the
St Paul’s Cathedral. Royal Society to the commission
Blackbeard the pirate is set up to investigate the rival
killed. claims.
De Moivre always had
The first successful
difficulty earning money, but
appendectomy is performed.
was able to eke out a living by
People are put to death, as working as a private tutor and by
witches, in Salem. writing books. Unlike many other
mathematicians of the time, he could
Abraham De Moivre was born in the not find a rich patron to support him
French town of Vitry but from the age of because he was a foreigner. Even though he
eighteen he lived in England. The son of a was made a fellow of the Royal Society in
doctor, he was educated at the Protestant 1697 and had famous friends such as Newton
Academy at Sedan and then attended college and Halley, he was always poor and
in Paris. In 1685 his family emigrated to eventually died in poverty.
England to escape the growing tensions Apparently, De Moivre predicted the time
between Catholics and Protestants in of his own death. Near the end of his life he
France. noticed that he needed to sleep for an extra
De Moivre contributed to the development 15 minutes each night. He calculated the date
of analytic geometry and to the theory of when the cumulative result of this would
probability. One of his most famous books, mean that he was asleep for 24 hours. He died
The Doctrine of Chances, was published in in his sleep on that day.
1718 and contained major advances in
probability theory. In 1725, after investigating
Questions
mortality statistics, he published Annuities on 1. What was the subject of De Moivre’s
Lives. Insurance companies of the day used book The Doctrine of Chances? Probability
his work to calculate the probabilities of 2. Why couldn’t De Moivre find a
various events. He is best known to students patron? He was a foreigner.
for his formula (r cis θ)n = r n cis nθ which
3. How did De Moivre make a living? Tutoring students
can be used to work out the powers of and writing books
4. Which famous mathematician played a
complex numbers.
major role in his life? Newton
It is said that De Moivre was inspired to
further research by reading Isaac Newton’s 5. What was unusual about the date he
died? De Moivre predicted it.
book Principia. He had little spare time so
he tore out pages and carried them around Research
with him, studying them in any free moment. Investigate how insurance companies use
Later in life he became involved in the probability to work out how much each
controversy about whether Newton or Leibniz insurance policy costs you.
3
a i
Im (z)
Chapter 2 Number systems: complex numbers 109
z–w 4

–1 0 Re (z)
remember
remember
b i 1. The magnitude (or modulus or absolute value) of z = x + yi is the length of the
Im (z)
6 u+z
line segment from (0, 0) to z and is denoted by z, x + yi or mod z.
2. z = x 2 + y 2 and z z = z 2.

y y
0 1 Re (z) 3. arg (z) = θ where tan θ = -- . ∴ θ = tan–1 -- .
x x
c i
Im (z) 4. z × i , n ∈ N produces an anticlockwise rotation of 90n degrees.
n

5. z = r cos θ + r sin θ i = r cis θ in polar form.


0 6 Re (z)
6. Arg (z) is the angle θ in the range −π < θ ≤ π.
1 a Im (z)
w–u z = 4 + 8i
–8 8

Complex numbers in polar


2E form
0 4 Re (z)
3 d i
Im (z)
In the following questions give arg (z) or Arg (z) correct to 3 decimal places where the
angle cannot be easily expressed as a common multiple of π.
0
–2
7 Re (z) 1 a Represent z = 4 + 8i on an Argand diagram.
w+z
b Calculate the exact distance of z from the origin. (Do not use your calculator.) z = 4 5
3 e i
WORKED 2 Find the modulus of each of the following. 65 Im (z)
Example
18 a z = 5 + 12i 13 b z = 5 – 2i 3 c z = −4 + 7i 0 9 Re (z)

d z = −3 − 6i 3 5 e z= 3 + 2i 5 f z = (2 + i)
2
5
–7 z+w–u
WORKED 3 If z = 3 + i, w = 4 − 3i and u = −2 + 5i then: f i
Example
19 i represent each of the following on an Argand diagram Im (z)
z2
ii calculate the magnitude in each case. 6

a z − w ii 17 b u + z ii 37 c w − u ii 10
d w + z ii 53 e z + w − u ii 130 f z2 ii 10 0 8 Re (z)

WORKED 4 a Show the points z1 = −3 + 0i, z2 = 2 + 5i, z3 = 7 + 5i and z4 = 9 + 0i on the complex


Example
20
number plane.
4 a
b Calculate the area of the shape formed when the four points are connected by
Im (z)
straight line segments in the order z1 to z2 to z3 to z4. 42.5 square units
6 z2 z3
4 5 a Show the points z = −1 + 3i, u = 3 and w = 3 + 12i on the complex number plane.
z1 2 z4 b Calculate the area of the triangle produced by joining the three points with straight
–4 –2 2 4 6 8 10 Re (z) line segments. 24 square units

WORKED 6 Find the Argument of z for each of the following in the interval (−π, π ]. (Give exact
Example
21
answers where possible.)
π π
5 a Im (z) a z = 3 + 2i 0.588 b z = 3 + i --6- c z = 5 − 5i – --4- d z = −4 + 8i 2.034
12 w
π
10
e z = −2 − 2 3i f z = 6 − 10i g z = 3i --2- h z= – 7 π
8
π
6 i z = −6i – --- j z = 55 0
z
4 2
2
u π
– 2------
–4 –2 0 2 4 6 8 Re (z) 3 −1.030
110 M a t h s Q u e s t M a t h s C Ye a r 1 1 f o r Q u e e n s l a n d

WORKED 7 Convert each of the following into Arguments.


Example
xample
3π π 11 π π 15 π π 5 π 3 π-
22 a ------ – --2- b − --------- --6- c --------- – --8- d − ------ -----
4
2 6 8 4
19 π 5 π 20 π 6 π 18 π 2 π 13 π 11 π
e --------- – ------ f --------- ------ g − --------- ------ h − --------- ---------
6 6 7 7 5 5 12 12

WORKED 8 Find the modulus and Argument of each of the following complex numbers.
Example
xample
23 a 3 − 3i 3 2, – π--- b −5 + 5i 5 2, 3-----π- c −1 − 3i 2, – 2-----π- d 4 3 + 4i 8, π---
4 4 3 6
e −7 − 10i f 6i − 2 g ( 3 + i ) 2 4, π---
3
149, – 2.182 2 10, 1.893 π
2 5 cis  – ---
WORKED 9 Express each of the following in the polar form, z = r cis θ, where θ = Arg (z).  3
Example
xample
24 a z = −1 + i b z = 6 + 2i c z = – 5 – 5i d z = 5 – 15i

a 2 cis ------ 3 1 1
4 e z = − 1--- – ------- i f z = − --- + --- i
π π
2 π 2 2 4 4 2 3 π- c 10 cis  – 3------
e cis  – ----- ------- cis ----- b 2 2 cis ---  4
- 6
 3 4 4
WORKED 10 Express each of the following complex numbers in Cartesian (or standard a + ib) form.
Example
xample
2π π 5π π
25 a 2 cis ------ b 3 cis --- c 5 cis ------ d 4 cis  − ---
3 4 6  3
10
7π π a −1 + 3i
e 7 cis  − ------ f 8 cis --- g 3 cis π
 4 2 3 2
b ---------- (1 + i )
2
11 multiple choice 5
c ------- ( − 3 + i )
If z = 3 − 50i and w = 5 + 65i, the value of z + w is: 2
A 64 B 15 C 17 D 225 E 289 d 2–2 3 i
14
12 multiple choice e ---------- (1 + i )
2
The perimeter of the triangle formed by the line segments connecting the points f 8i
2 − 4i, 14 − 4i and 2 + i is: g – 3
A 13 B 30 C 10 D 17 E 25

13 multiple choice
The Argument of 4 3 – 4i is:
π π 5π π π
A --- B --- C ------ D – --- E – ---
6 3 6 6 3

14 multiple choice
In polar form, 5i is:
π 5π π
A cis --- B cis 5π C cis ------ D 5 cis 5π E 5 cis ---
2 2 2

15 multiple choice

The Cartesian form of 3 cis  – ------ is:
 6
1 3 1 3 3 1 3 3 3 1
A --- + ------- i B − --- + ------- i C − ------- + --- i D − --- + ------- i E − ------- – --- i
2 2 2 2 2 2 2 2 2 2
Chapter 2 Number systems: complex numbers 111
Basic operations on complex numbers
in polar form
Addition and subtraction
In general there is no simple way to add or subtract complex numbers given in the polar
form r cis θ. For addition or subtraction, the complex numbers need to be expressed in
Cartesian form first.

Multiplication in polar form


SLE 3: Use polar forms to In earlier sections we performed multiplication and division on complex numbers
demonstrate multiplication
and division of complex in standard form. This is quite a lengthy process for both these operations.
numbers. However, as is the case in many aspects of mathematics, patterns exist that make
the job so much easier. Work through the following investigation that will form the
basis of future work.
1 Given that z = 1 + 3 i and w = 2 + 2i:
a find zw in standard form 2 − 2 3 + 2( 3 + 1)i

b express the product from part a in polar form 4 2 cis ------
12
c verify that | zw | = | z || w |
d verify that arg (zw) = arg (z) + arg (w).
2 Given that z = 1 – i and w = 2 – 2 3 i:
a find zw in standard form 2 − 2 3 − 2( 3 + 1)i

b express the product from part a in polar form 4 2 cis  – ------
12
c verify that | zw | = | z || w |
d verify that arg (zw) = arg (z) + arg (w)
3 Given that z = a + bi and w = c + di
a find zw in standard form
b verify that | zw | = | z || w |
c For z = r1 cis θ = r1 (cos θ + i sin θ) and w = r2 cis φ = r2 (cos φ + i sin φ),
show that zw = r1 r2[(cos θ cos φ – sin θ sin φ) + i(cos θ sin φ + sin θ cos φ)]
4 Using the trigonometric identities:
cos (A + B) = cos A cos B – sin A sin B
sin (A + B) = cos A sin B + sin A cos B
verify that zw = r1 r2[cos(θ + φ) + i sin (θ + φ)]
= r1 r2 cis (θ + φ)
This investigation illustrates the following useful facts concerning multiplication
of complex numbers in polar form:
If z and w are two complex numbers, then
| zw | = | z || w | and arg (zw) = arg (z) + arg (w)
Similarly, for division of complex numbers:

= -------- and arg  ---- = arg (z) – arg (w)


z- z z
---
w w  w
The proofs required to establish these rules are outside the Mathematics C
syllabus and will not be included in this chapter on complex numbers.
112 M a t h s Q u e s t M a t h s C Ye a r 1 1 f o r Q u e e n s l a n d

WORKED Example 26
π 5π
Express 5 cis --- ¥ 2 cis ------ in the form r cis θ where θ ∈ (−π, π].
4 6
THINK WRITE/DRAW
π 5π π 5π
1 Simplify using the multiplication rule 5 cis --- × 2 cis ------ = ( 5 × 2 ) cis  --- + ------
4 6  4 6
zw = r1r2 cis (θ + φ) (see part 4 on
13 π
page 111). = 10 cis  ---------
 12 
2 Sketch this number. Im (z)

—π
13
12 Re (z)

π
–11—
12

13 π 11 π
Subtract 2π from θ to express the 10 cis  --------- = 10 cis  – ---------
3  12   12 
answer in the required form.

WORKED Example 27
5π π
Express z1z2 in Cartesian form if z 1 = 2 cis ------ and z 2 = 6 cis  – --- .
6  3
THINK WRITE
5π π
1 Use z1z2 = r1r2 cis (θ1 + θ2). z1z2 = 2 cis ------ × 6 cis  – ---
6  3

= ( π π
2 × 6 ) cis  5------ – ---
 6 3
π
= 2 3 cis ---
2
π π
2 Write the result in standard form. = 2 3 cos --- + 2 3 sin --- i
2 2
= 2 3 × 0 + 2 3 × 1i
= 2 3i

WORKED Example 28
If z = 5 3 + 5i and w = 3 + 3 3i , express the product zw in polar form.
THINK WRITE/DRAW
1 Sketch z. Im (z)

5
θ Re (z)
5 √3
Chapter 2 Number systems: complex numbers 113

THINK WRITE/DRAW
2 Write z in polar form. Use the special Let z = r 1 cis θ1.
triangle below: r1 = 5 × 2 = 10
π
π
θ1 = ---

6 6
2
√3

π

3
1
The ratio of sides in z is 5 times that of
the sides in this triangle.
3 Verify this by rule if you wish. Alternatively:
r1 = (5 3)2 + 52 = 10
5 π
θ = tan–1 ---------- , so θ 1 = ---
5 3 6
π
Therefore z = 10 cis ---
6
4 Sketch w. Im (z)
The ratio of sides in w is 3 times that of
the sides in the special triangle shown
in step 2.
3 √3

θ Re (z)
3

5 Write w in polar form. Let w = r2 cis θ2


r2 = 3 × 2 = 6
π
θ2 = ---
3

6 Verify this by rule if you wish. Alternatively:


r2 = 32 + (3 3)2 = 6
3 3 π
tan θ 2 = ---------- =
3 , so θ 2 = ---
3 3
π
Therefore w = 6 cis ---
3
π π
7 Determine zw using zw = 10 cis --- × 6 cis ---
z1z2 = r1r2 cis (θ1 + θ2). 6 3
 π π 
= 60 cis --- + ---
 6 3
= 60 cis --π-
2
114 M a t h s Q u e s t M a t h s C Ye a r 1 1 f o r Q u e e n s l a n d

WORKED Example 29
π 5π
Express 10 cis  – --- ÷ 5 cis ------ in the form r cis θ where θ ∈ (−π, π ].
 3 6
THINK WRITE/DRAW
π 5π π π
1 Simplify using the division rule. (See 10 cis  – --- ÷ 5 cis ------ = 2 cis  – --- – 5------
 3 6  3 6
part 4 on page 111.)

= 2 cis – ------ 
 6
2 Sketch this number. Im (z)
2
π
–π
5—
6 6 Re (z)

– 7—6π


3 State θ, the principal Argument. Arg (z) = ------
6

4 State the result in polar form. Arg ()z = 2 cis ------
6

Powers of complex numbers


Whole powers of z
As with real numbers, powers of complex numbers can be written as:
zn = z × z × z × z × … × z to n factors.
Since z = a + bi is a binomial (containing two terms) we can express zn using Pascal’s
Triangle to generate the coefficients of each term.
1
1 1
1 2 1
1 3 3 1
1 4 6 4 1
5th row → 1 5 10 10 5 1 and so on.
(a + bi)5 can therefore be expanded using the elements of the fifth row of Pascal’s
Triangle:
(a + bi)5 = 1a5 + 5a4(bi)1 + 10a3(bi)2 + 10a2(bi)3 + 5a(bi)4 + (bi)5
(a + bi)5 = 1a5 + 5a4bi + 10a3b2i2 + 10a2b3i3 + 5ab4i4 + b5i5
(a + bi)5 = 1a5 + 5a4bi − 10a3b2 − 10a2b3i + 5ab4 + b5i
(a + bi)5 = 1a5 − 10a3b2 + 5ab4 + 5a4bi − 10a2b3i + b5i
(a + bi)5 = 1a5 − 10a3b2 + 5ab4 + (5a4b − 10a2b3 + b5)i grouped into standard form.
Re [(a + bi)5] = 1a5 − 10a3b2 + 5ab4
Im [(a + bi)5] = 5a4b − 10a2b3 + b5
Chapter 2 Number systems: complex numbers 115
Graphics Calculator tip! Pascal’s Triangle coefficients
The coefficients of each term of the expansion of (a + bi)n can be found using your
graphics calculator. For example, the coefficients of the expansion of (a + bi)5 can also
be written as:
5
C0 a5 + 5C1 a4(bi)1 + 5C2 a3(bi)2 . . . 5C5(bi)5
5 5 5
where C0, C1, . . ., C5 represent the coefficients.
The following steps show how to calculate 5C3 using a graphics calculator.

For the Casio fx-9860G AU


1. Press MENU and select RUN-MAT. Press OPTN and
s

then F6 ( ) for more options.

2. Press F3 (PROB) and you will see the function nCr.


Enter 5 (for n), then press F3 (nCr) and enter 3
(for r). Press EXE to display the value.

For the TI-Nspire CAS


1. Press k to access the catalog and press 1 for the
list of functions. Scroll down to select nCr(. You can
do this more quickly by first pressing N.

2. Press · and then complete the entry line to obtain


nCr(5,3). Press · to display the value.

Note that with this calculator, we can obtain the actual


expansion of (a + bi)5.

1. Open a new Calculator document.


2. Press b and select 1: Actions and then 4: Clear
a-z. This sets the variables a–z to their default values
and makes them ready for use. Accept OK by
pressing ·.
3. Enter (a + bi)^5 and press ·. You will see a
small arrow at the end of the answer line
indicating that there are more terms. Use the
arrows on the NavPad to see more of the
expansion. The full expansion is shown as
a(a4 − 10a2b2 + 5b4) + (5a4 − 10a2b2 + b4)bi.
116 M a t h s Q u e s t M a t h s C Ye a r 1 1 f o r Q u e e n s l a n d

WORKED Example 30
Use Pascal’s Triangle to expand (2 - 3i)3.

THINK WRITE
1 Use the third row of Pascal’s Triangle (2 − 3i)3
to expand (1 3 3 1). = 1(23) + 3(2)2(−3i) + 3(2)(−3i)2 + (−3i)3
Use brackets to keep the negative sign of
the second term.
2 Simplify the expression. = 8 − 36i + 54i2 − 27i3
= 8 − 36i − 54 + 27i
= −46 − 9i

Negative powers of z
1 1
Your earlier studies have shown that z–1 = --- . Similarly, z–3 = ---3- .
z z

WORKED Example 31
Evaluate (3 - i)-4.

THINK WRITE
1 First find the expansion with a positive (3 − i)4
power. Use the fourth row of Pascal’s = 34 + 4(3)3(−i) + 6(3)2(−i)2 + 4(3)(−i)3 + (−i)4
Triangle to expand (1 4 6 4 1).
(Use brackets to keep the negative sign = 81 − 108i − 54 + 12i + 1
with the second term.)

2 Simplify to obtain the standard form. = 28 − 96i


1 ( 28 + 96i )
3 Express this as the denominator then (3 − i)–4 = ------------------------ × -------------------------
multiply by the complex conjugate. ( 28 – 96i ) ( 28 + 96i )
28 + 96i
(3 − i) = ---------------------------
–4
784 + 9216
28 + 96i
(3 − i) –4 = --------------------
10 000
7 6
4 Write the final expression in standard (3 − i)–4 = ------------ + --------- i
form. 2500 625

Fractional powers of z
Fractional powers of complex numbers generally follow the same rules as with real
numbers.
p
--- q p
zq = z 1---
2
Our discussion here will deal only with the square root of z, where z = z .
Chapter 2 Number systems: complex numbers 117
WORKED Example 32
Express 3 + 4i in standard form.

THINK WRITE
1 Let 3 + 4i be a complex number such Let 3 + 4i = a + bi
as (a + bi), where a, b ∈ R.

2 All dialogue given in the ‘write’ Square both sides:


column should appear as 3 + 4i = (a + bi)2
‘communication’ in your working. 3 + 4i = a2 + 2abi − b2
Equating real and imaginary terms:
3 = a2 − b2 [1]
4 = 2ab [2]
4 2
a = ------ = --- from [2] [3]
2b b
Substitute for a into [1]
2 2
3 =  --- − b2
 b
4
3 = ----2- − b2
b
3b2 = 4 − b4
b4 + 3b2 − 4 = 0
(b2 − 1)(b2 + 4) = 0
Therefore, b2 = 1, b = ±1
or b2 = −4, b = ±2i
Since a and b are real numbers discard b = ±2i.
Substitute for b = ±1 into [3]
2
When b = 1, a = --- = 2
1
2
When b = −1, a = ------ = −2
–1

3 State the final result in standard form. Therefore 3 + 4i = 2 + i or −2 − i


= ±(2 + i)
4 Verify this result. Check:
[±(2 + i)]2 = 4 + 4i − 1 = 3 + 4i

5 Alternatively, you can use a graphics


calculator (such as a TI-Nspire CAS
calculator) to verify this result.
118 M a t h s Q u e s t M a t h s C Ye a r 1 1 f o r Q u e e n s l a n d

History of mathematics
W I L L I A M R O WA N H A M I LT O N ( 1 8 0 5 – 1 8 6 5 )
During his life . . . In 1833 Hamilton further
Charles Darwin developed his theory developed his work on complex
of evolution. numbers and in 1843 he released
Charles Babbage developed the what he considered to be his
first automatic digital computer. greatest discovery — the algebra
of quaternions. With these
Gregor Mendel laid the
ordered sets of four numbers,
mathematical foundation for the
magnitude and direction in
science of genetics.
3-dimensional space could be
Sometimes considered the second determined. The fact that
greatest mathematician of the English- multiplication of quaternions is
speaking world, after Sir Isaac Newton, not commutative led to the
William Hamilton was born in Dublin, Ireland, development of the first ‘ring’ in which
on 3 August 1805. Even the fact that Hamilton the commutative property does not hold.
did not attend school before he entered college This inspiration came to him while he was
did not deter his thirst for knowledge. By the crossing the Brougham Bridge in Dublin and
age of three he was skilled at reading and he left the inscription i2 = j2 = k2 = ijk = −1
arithmetic, by the age of five he read and in a stone in the bridge. A stamp featuring
translated Latin, Hebrew and Greek, and by the these quaternions was issued in Ireland in
age of 14 he could speak 14 languages. 1983.
By the age of 21 he published a paper entitled His work also led to the development of the
‘A Theory of Systems of Rays’, introducing and concepts of vectors, scalars and tensors, which
developing concepts that created the field of you will encounter later in your studies.
mathematical optics. Propelled by the success of Plagued throughout his life with alcoholism,
this work, at 22 he was unanimously voted into he died in 1865.
the chair of the professor of astronomy at Trinity
College (Dublin), even though he was still an Research
undergraduate and had not applied for the 1. Find out more about quaternions.
position. 2. Research the notion of ‘rings’.

remember
remember
1. If z1 = r1 cis θ1 and z2 = r2 cis θ2, then:
z1 × z2 = r1r2 cis (θ1 + θ2)
z r
---1- = ----1 cis (θ1 − θ2).
z2 r2
2. A complex number zn = (a + bi)n can be expanded using Pascal’s Triangle to
generate the coefficients of each term.
1
3. Negative powers of z: z –n = ---n-
z p
---
q q p
4. Fractional powers of complex numbers: z = z
Chapter 2 Number systems: complex numbers 119
Basic operations on complex
2F numbers in polar form
WORKED 1 Express each of the following in the form r cis θ where θ ∈ (−π, π ].
Example
26 π π 3π
a 2 cis --- × 3 cis --- 6 cis ------
4 2 4

2π π π
b 5 cis ------ × 4 cis  – --- 20 cis ---
3  3 3

3π π
6 5 cis  – ---
c 6 cis ------ × 5 cis π  4
4
5π π 2π
d 3 cis  – ------ × 2 cis  – --- 6 cis ------
 6  2 3

7 π × 2 5π π
e 7 cis  – -----
- cis ------ 2 7 cis  – ---
 6
 12  12
WORKED
Example 2 Express the resultant complex numbers in question 1 in standard form.
2 a –3 2+ 3 2 i
27
b 10 + 10 3 i
WORKED 3 Express the following products in polar form. c 3 10 – 3 10 i
Example

28 a (2 + 2i)( 3 + i) 4 2 cis ------ 3 2
d – ------6- + ---------- i
12 2 2
b ( 3 − 3i)(2 3 − 2i) 8 3 cis  – π--- e 21 – 7 i
2
π-
c (−4 + 4 3 i)(−1 − i) 8 2 cis  – -----
12
WORKED 4 Express each of the following in the form r cis θ where θ ∈ (−π, π ].
Example
5π π
29 a 12 cis ------ ÷ 4 cis --- 3 cis π---
6 3 2

3π π 11 π
b 36 cis ------ ÷ 9 cis  – --- 4 cis ---------
4  6 12

3 π
c 20 cis  – π π
--- ÷ 5 cis  – --- 2 cis  – -----
 10 
-
 2  5
π 3 6 3 6
5 a i 3 3 cis --- ii ---------- + ---------- i
4π 11 π 3 π-
2 2 cis  – -----
d 4 3 cis ------ ÷ 6 cis ---------  14 
4 2 2
7 14 b i 16 cis π ii −16
c i 9 cis π ii −9
7 π 5π 7 π-
3 5 cis  – -----
3 2
e - ÷ 2 10 cis ------ ---------- cis ----- 3π
 12  6 4 12 d i 32 cis ------ ii – 16 2 + 16 2i
4
3π π
WORKED 5 If z = 3 cis ------ and w = 2 cis  – --- then express each of the following in:
Example 4  4
30 i polar form ii standard form.
3
a z b w4 c z4 d w5
WORKED 6 If z = 1 − i and w = – 3 + i , write the following in standard form.
Example
a z−4 − --4- b w−3 − --8- i z−3 − --4- + --4- i
1 1 1 1
31 c
3 1
d w−5 ------- – ------ i z3 f z2w3 16
64 64
e -----4- 0.171 – 0.046i
w
120 M a t h s Q u e s t M a t h s C Ye a r 1 1 f o r Q u e e n s l a n d

7 Determine ( 2 + 2i ) 2 ( 1 – 3i ) 4 in standard form. – 64 3 – 64i

( 3 – i )6
8 Write ----------------------------3 in the form x + yi. 1
( 2 – 2 3i )

9 multiple choice
π π
a 5 cis  – --- × 8 cis  – --- is equal to:
 3  6
A 6 2i B – 2 10i C –6 3 D −6i E 6 6

b If z = ( 6 + 2 ) + ( 6 – 2 ) i then 2z –3 is:
A 1+i B 2i C 1-
----- (1 − i) D 2+i E –1 – 2i
64
w4
c If z = –1 – 3i and w = 2 + 2i then -----3- is equal to:
z
A −4 + 4i B 2 3 C 2 D −4i E −8

3π π z6 8 π
---, – ---
10 If z = 2 cis ------ and w = 3 cis --- , find the modulus and the argument of -----4- . 9 6
4 6 w
11 If z = 4 + i and w = −3 − 2i, determine (z + w)9. 16 − 16i

12 Find z6 + w4, if z = 2 – 2i and w = 2 − 2i. −64 + 64i

π 3π π
13 If z 1 = 5 cis – 2------  , z 2 = 2 cis ------ and z 3 = 10 cis ------ , find the modulus and
 5 8 12
z 12 × z 23 2---, – --------
π
the argument of -------------------
4
. 5 120-
z3
eBook plus
WORKED 14 Express each of the following in standard form:
Example
xample
32 a 5 + 12i ±(3 + 2i) b 5 – 12i ±(3 − 2i) Digital doc:
WorkSHEET 2.2

c 2 + 2i ±( 1 + 2 + – 1 + 2 i ) d 3 – 4i ±(2 − i)

Complex numbers: applications


1 Choose a complex number that falls in the first quadrant of the complex plane.
Calculate the first 8 powers of this number and investigate any pattern that
exists between the modulus of each of the powers. Plot each power on an
Argand diagram. What do you notice?

2 Let z = r cis θ, a complex number. Find, in terms of r and θ:


2 a iii z2 = r2 cis 2θ
a iii z2 = z.z ii z3 = z.z2 iii z4 = z.z3
iii z3 = r3 cis 3θ iv z5 = z.z4 v z6 = z.z5 vi z7 = z.z6
iii z4 = r4 cis 4θ GP, a = r r = r
iv z5 = r5 cis 5θ b Write the moduli of the powers of z as a sequence. Tn = ar n – 1
iv z6 = r6 cis 6θ c What do you notice about the sequence given in part b? Geometric progression
vi z7 = r7 cis 7θ
3
Im (z)
z2
Chapter 2 Number systems: complex numbers 121
π
2—
3

0 z1
–2
—– π
Re (z)
3 As mentioned at the beginning of this chapter the equation z2 = 1 has two
solutions, z = ±1, whereas the equation z = 1 has only one solution, z = 1. The
3
z3
z1 = 1 = cis 0 2π 2π
equation z3 = 1 has 3 solutions, z = 1, cis ------ and cis  − ------ .
2π 3  3
z2 = −1 + 3 i = cis ------
3
Graph these solutions on an Argand diagram. Express all solutions in both

z3 = −1 − 3 i = cis  – ------ rectangular and mod–arg form.
 3
2 2
4 Let z = x + yi. Therefore | z | = x + y , and | z |2 = x2 + y2, where this is the
general equation of a circle, of radius | z |, about the origin. Graph this circle and
fully label the path of the rotating z as it moves about the origin. Therefore,
what is the meaning of the statement | z | < x + yi? Sketch | z | < 4 and | z | > 1.

5 Research the life of William Rowan Hamilton and his contribution to the study
of complex numbers.

6 Research the area of mathematics called fractals. You will investigate this
fascinating area in more detail later in your studies.

7 In Chapter 1, you were introduced to the term ‘transcendental numbers’ —


irrational numbers that are not algebraic, that is, cannot be produced by the
algebraic operations of addition, subtraction, multiplication and division, and
by taking roots. Pi (π) is one such transcendental number and e is another,
1 2 3 4
1 1 1 1
where e = e1 = 10 + ----- + ----- + ----- + ----- . . .
1! 2! 3! 4!
(and 3! = 3 × 2 × 1, and so on. The symbol 3! is referred to as factorial 3.)
The function ex is referred to as the exponential function.
1 2 3 4
x x x x
ex = x0 + ----- + ----- + ----- + ----- . . .
1! 2! 3! 4!
The graph of the function ex is especially interesting because the slope of the
curve at any point equals the value of the curve, at that point. That is, the slope
of a tangent to the curve at x = e2 is e2.
Euler discovered a special relationship between e and i,
1 2 3 4
i i i i
where ei = i 0 + ----- + ----- + ----- + ----- . . .
1! 2! 3! 4!
Write four expressions for ei, with increasing numbers of terms and simplify
them where possible. The alternating positive and negative signs suggest that
the expression is approaching a particular value as the number of terms in the
series increases. You might find it more methodical to list the results as each
new term is added as you ‘creep’ closer to the value. Can you suggest what that
value might be? What is the modulus of this number? Use a graphics calculator
to evaluate ei.
122 M a t h s Q u e s t M a t h s C Ye a r 1 1 f o r Q u e e n s l a n d

summary
Introduction to complex numbers
• We define the ‘imaginary number’ i as having the property that i 2 = −1.
• A complex number z = x + yi with x, y ∈ R and C = {z: z = x + yi, x, y ∈ R} defines
the set of complex numbers.
• The real part of z is x and is written as Re (z).
• The imaginary part of z is y and is written as Im (z).
Basic operations using complex numbers
• If z and w are two complex numbers such that z = a + bi and w = c + di for a, b, c,
d ∈ R then:
1. z = w if and only if (i.e. iff) a = c and b = d
2. z + w = (a + c) + (b + d)i
3. z − w = (a − c) + (b − d)i
4. kz = ka + kbi, for k ∈ R
5. z × w = (ac − bd) + (ad + bc)i.
Conjugates and division of complex numbers
• If z = a + bi and w = c + di for a, b, c, d ∈ R then:
1. The conjugate z of z is z = a − bi.
2. z. z = a2 + b2.
z ac + bd- (-----------------------
bc – ad )i-
3. ---- = -----------------
2 2
+ 2 2
w c +d c +d
Radians and coterminal angles
• A radian is the angle subtended by an arc of the radius of a circle. That is, an arc of
2πr subtends an angle of 2π radians.
• 2π radians = 360°. 1 radian ≈ 57.3°.
• Coterminal angles differ by a multiple of 360°.
The polar form of complex numbers
• The magnitude (modulus or absolute value) of z = x + yi is the length of the line
segment from (0, 0) to z. It is denoted by z, x + yi or mod z.
• z = x 2 + y 2 and z z = z 2 .
• The argument of z, arg (z), is the angle measurement anticlockwise of the positive
y
Real axis and arg (z) = θ where θ = tan–1 --x .
• z = x + yi can be expressed in polar form as z = r cos θ + r sin θi = r cis θ.
• Arg (z) is the angle θ in the range −π < θ ≤ π and is called the principal argument.
Basic operations on complex numbers in polar form
• If z1 = r1 cis θ1 and z2 = r2 cis θ2, then:
1. z1 × z2 = r1r2 cis(θ1 + θ2)
z r
2. ---1- = ----1 cis(θ1 − θ2)
z2 r2
• A complex number zn = (a + bi)n can be expanded using Pascal’s Triangle to
generate the coefficient of each term.
1
• z−n = ---n-
z
Chapter 2 Number systems: complex numbers 123

CHAPTER
review
Questions 1 and 2 refer to the complex number z = 2 5 – 4i .

1 multiple choice
2A
The real and imaginary parts of z respectively are:
A 2 5 and 4 B 2 5 and −4 C 4 and 2 5 D −4 and 2 5 E 2 5 and −4i

2 multiple choice
2A
The Argand diagram which correctly represents z is:
A Im (z) B Im (z) C Re (z)
4 z

0 Re (z) 0 Re (z) 0 Im (z)


2 5 2 5 2 5

–4 z –4 z
D Re (z) E Im (z)
4 z z
2 5

0 0
2 5 Im (z) –4 Re (z)

3 Simplify i 6 − i 3 (i 2 − 1). −1 − 2i
2A
Questions 4 and 5 refer to the complex numbers u = 5 − i and v = 4 + 3i.

4 multiple choice
2B
The expression 2u − v is equal to:
A 1 − 4i B −3 − 7i C 6 − 5i D 5 + 8i E 14 + i

5 multiple choice
2B
The expression uv is equal to:
A 9 + 2i B 20 − 3i C 20 + 3i D 15 − 4i E 23 + 11i

6 multiple choice
2B
If z = 5 − 12i, decide which statement is true concerning −iz.
A −iz = 13
B −iz = 12 − 5i
C The point z is rotated 90° clockwise.
D Re (−iz) = 0
E Im (−iz) = −i
124 M a t h s Q u e s t M a t h s C Ye a r 1 1 f o r Q u e e n s l a n d

7 If z = 3 − 8i, then find:


2B a Im (z2) −48 b a and b if z3 = a + bi. a = −549, b = 296
8 If z = 2 − 5i, u = −3 + i and w = 1 + 2i evaluate:
2B,C a z − 2u + 3w 11 − i b z 29 c uz + w 6 5

9 multiple choice
2C
Im  -------------- is equal to:
1 + 2i
 1–i
A 2 B − 1--- C 3
--- D 2
--- E −2
2 2 3

10 multiple choice
2C
If z = 3i and w = 4 − i then z w is equal to:
A 12 + 3i B 12 − i C 7 + 3i D 12 − 3i E z = 4 − 2i

11 multiple choice
2C
2i 3
The expresion ----------- – ---------- simplifies to:
1+i 2–i
1 2--- 1 3
A – --- + i B 3 + 7i C --- – --- i D 4 − 2i E 3−i
5 5 4 4

z 1
- ( 12 – 14i )
12 If z = 6 − 2i and w = 5 + 3i, express ---- in the form a + bi, a, b ∈ R. -----
2C w 17

13 Convert the following common angle measures to radians.


2D π
a 30° --6- b 180° π

14 Convert the following radian measures to degrees.


2D 3π 7π
a ------ 135° b ------ 210°
4 6

15 multiple choice
2D Of the following pairs of angles, the pair that is coterminal is:
π 9π 3π 5π
A 40°, 220° B --- , ------ C 135°, 435° D ------ , ------ E 180°, 360°
4 4 2 2
16 multiple choice
2E Arg (2 − 2i) is equal to:
π 3π π
A π B --- C ------ D – --- E 2π
4 4 4

17 multiple choice
2E The polar form of −3 + 3i is:
π 3π 3π π 5π
A 3 2 cis --- B 3 2 cis ------ C 3 cis ------ D 3 cis – ---  E 3 2 cis ------
4 4 4  4 4
3 π
18 If z = −7 − 7i, express z in polar form. 7 2 cis  – -----
-
2E  4
Chapter 2 Number systems: complex numbers 125
19 multiple choice
2E
How many degrees apart are two consecutive roots of z8 = 1 on the unit circle?
A 180 B 90 C 135 D 225 E 45

20 multiple choice
π π
2F
If z1 = 10 cis --- and z2 = 5 cis  – --- then z1z2 in polar form is:
4  6
π 5π π π- E 2 cis  – -----
D 15 cis  – -----
5 π
A 50 cis ------ B 15 cis ------ C 2 cis ------  12  12 
-
12 12 12

21 multiple choice
3π π 2F
In standard form, 12 2 cis ------ ÷ 3 cis  – --- is equal to:
4  2
A 4 + 4i B −4 − 4i C 4 − 4i D −4 + 4i E 36 − 36i

22 If z = −3 − 4i, write the following in standard form.


117 44 i 2F
a z4 –527–336i b z−3 ---------------
15 625 15 625
- + ---------------
- c z ±(1 − 2i)

Modelling and problem solving


π π
1 Let z = 2 cis --- and w = 2 cis --- .
3 4
z π π
a Express ---- in the form r cos θ + r sin θ i. cos ------ + sin ------ i
w 12 12
b Express z and w in Cartesian form. z = 1 + 3 i, w = 2 + 2 i
z 2 + 6 + ( 6 – 2 )i
c Express ---- in Cartesian form. -----------------------------------------------------
-
w 4
d Using the results of parts a and c, find the exact values for:
π 6+ 2
i cos ------ --------------------
4
12
π 6– 2
ii sin ------ --------------------
4
12
π
iii tan ------ . 2 – 3
12
π π z
e By letting z = 2 cis --- and w = 2 cis --- and following parts a to c for zw instead of ---- ,
4 6 w

deduce that tan ------ = 2 + 3 . Check with your teacher.
12

2 Let u = 1 – i.
a i Find u u . 2
ii Find Arg u + Arg(2 u ). 0
b Let z = x + yi, x, y ∈ R, and |z – u | = |z – 2u|.
Find the value of x when y = 0. x = 3
126 M a t h s Q u e s t M a t h s C Ye a r 1 1 f o r Q u e e n s l a n d

3 Consider the complex number z such that z = 3 + 2i.


a Find the value for iz, i2z, i3z and i4z. (Give answers in standard x + yi form.) iz = −2 + 3i, i2z = −3 − 2i,
b Comment on the value of i4z. i4z = z i3z = 2 − 3i, i4z = 3 + 2i
c Plot each number from part a on the same Argand diagram.
d Use a pair of compasses to draw a circle whose centre is at the origin and which passes
through each point on the diagram.
e Find the radius of the circle, giving your answer in exact (surd) form. 13
f Carefully study the five points on your diagram. What transformation is required to
transform:
i point z into point iz? i, ii and iii One-quarter turn
(rotation by 90°) in an
ii point iz into point i 2z? anticlockwise direction.
iii point i 2z into point i 3z?
eBook plus
g On the Argand diagram, what transformation takes place when a complex number is
Digital doc: multiplied by i? One-quarter turn in an anticlockwise direction
Test Yourself h For a complex number z such that z = x + yi, describe the curve that all points representing
Chapter 2
numbers of the form zi n (that is, z, zi, zi2, zi3, and so on) would lie on an Argand diagram.
3 c Im z d Im z Circle with centre at the origin and radius
4 4 2 2
iz 3 iz 3 r = x +y
4 4
2 i z 2 i z
Im z
1 1
0 0 z = x + yi
–4 –3 –2 –1 Re z
–1 1 2 3 4 Re z
–4 –3 –2 –1 –1 1 2 3 4 zi
–2 2 –2 r
i 2z –3 i 3z i z –3 y
i 3z
–4 –4 x Re z
3
zi
zi 2
Matrices

3
syllabus reference
Core topic:
Matrices and applications

In this chapter
3A Operations with matrices
3B Multiplying matrices
3C Powers of a matrix
3D Multiplicative inverse and
solving matrix equations
3E The transpose of a matrix
3F Applications of matrices
3G Dominance matrices
128 M a t h s Q u e s t M a t h s C Ye a r 1 1 f o r Q u e e n s l a n d

Introduction to matrices
• definition of a matrix as Four towns are connected by roads as shown in the figure. There is one road connecting
data storage and as a
mathematical tool A and B, two roads connecting A and C and so on. This information may be rep-
• dimension of a matrix resented as shown in the table.
• matrix operations — addition
and subtraction, transpose, To
multiplication by a scalar,
multiplication by a matrix
• inverse of a matrix B A B C D
• solution of simple matrix
equations
• definition and properties of A A 0 1 2 0
the identity matrix
• singular and non-singular D B 1 0 0 1
matrices From
• applications of matrices in
both life-related and purely C
C 2 0 0 3
mathematical situations
D 0 1 3 0

If the headings at the top and side of this display are removed, an array of numbers
only is left:

0 1 2 0
1 0 0 1
2 0 0 3
0 1 3 0

This array of numbers is called a matrix (plural, matrices).


The arrangement of numbers in matrices is an extension of our number system and,
as we will see, the rules that govern matrix calculations have many similarities with the
arithmetic of numbers. Matrices are particularly useful in solving complex problems in
linear programming.

A matrix is a rectangular array of numbers arranged in rows and columns.


The numbers in the matrix are called the elements of the matrix.

The matrix above is a 4 × 4 matrix as it has 4 rows and 4 columns. We say the order
of the matrix is 4 by 4.

2 0
The matrix 1 – 4 is a 3 × 2 matrix because it has 3 rows and 2 columns. Note the
–1 2
square brackets used to enclose the array.

A matrix with m rows and n columns is called an m × n matrix. We say the order
of the matrix is m × n. The dimensions of a matrix are always given as the number
of rows multiplied by the number of columns.

The elements of the matrix are referred to by the row and then by the column pos-
ition. In the 3 × 2 matrix above, the row 1, column 1 element is 2, the row 3, column 1
element is −1 and the row 1, column 2 element is 0.
Chapter 3 Matrices 129
We often use capital letters as symbols for matrices. Thus we may write

2 0
A= 1 –4
–1 2

In general, the elements of a matrix A are referred to as ai j where i refers to the


row position and j refers to the column position.

a 11 a 12 a 13 a 14 … a 1n
a 21 a 22 a 23 a 24 … a 2n
That is, A = a 31 a 32 a 33 a 34 … a 3n , depending on the order of the matrix
. . . . ... .
. . . . ... .
. . . . ... .
a m1 a m2 a m3 a m4 … a mn
where A is an m × n matrix.
The row 1, column 1 element is a11.
The row 3, column 1 element is a31 and so on.

WORKED Example 1
For each of the following give the order of the matrix, if it exists, and where possible write
down the elements in row 2, column 1 and row 1, column 3.
2 5 –1
A= 3 6 B= 1 2 3 C = –2 D= 5 0 2
–1 –2 –3 6
4 7 –3

THINK WRITE
1 A has 3 rows of numbers and 2 columns A is a 3 × 2 matrix.
of numbers.
2 B has 2 rows and 3 columns. B is a 2 × 3 matrix.
3 C has 3 rows and 1 column. C is a 3 × 1 matrix.
4 D is not a rectangular array of numbers D is not a matrix.
as it does not have all positions filled.
5 The row 2, column 1 element is the
number in the second row and the first Row 2, Row 1,
column. column 1 column 3
Matrix element element
6 The row 1, column 3 element is the
number in the first row and the third A 3 —
column.
7 In A and C there is no row 1, column 3 B –1 3
element since there is no third column
C –2 —
in either matrix.
130 M a t h s Q u e s t M a t h s C Ye a r 1 1 f o r Q u e e n s l a n d

Operations with matrices


Addition
The sports coordinator at Mathglen State High School kept records of the number of
first, second and third ribbons awarded to competitors in each house at the swimming
and athletics carnivals and sports events.
The results were:

Swimming Athletics and sports


House 1st 2nd 3rd House 1st 2nd 3rd
Hamilton 25 28 24 Hamilton 35 35 27
Leslie 38 30 35 Leslie 33 34 39
Barnes 34 36 35 Barnes 30 33 36
Cunningham 35 38 38 Cunningham 34 34 30

To find the total number of first, second and third places for each house, the swimming,
athletics and sports results may be added. The elements in corresponding positions are
added to give the total number of first, second and third places for each house:

House 1st 2nd 3rd


Hamilton 25 + 35 = 60 28 + 35 = 63 24 + 27 = 51
Leslie 38 + 33 = 71 30 + 34 = 64 35 + 39 = 74
Barnes 34 + 30 = 64 36 + 33 = 69 35 + 36 = 71
Cunningham 35 + 34 = 69 38 + 34 = 72 38 + 30 = 68

60 63 51
Adding the elements for each event results in the following matrix: 71 64 74
64 69 71
69 72 68
Addition of matrices is performed by adding corresponding elements.
Chapter 3 Matrices 131
Subtraction
The subtraction of matrices is also performed by the usual rules of arithmetic on
corresponding elements of the matrices. It follows that:

1. Subtraction of matrices is performed by subtracting corresponding elements.


2. Addition and subtraction of matrices can be performed only if the matrices are
of the same order; that is, they have the same number of rows and columns.
Furthermore, addition of matrices is commutative. That is, for two matrices A and B of
the same order:
A+B=B+A

WORKED Example 2
If A= 1 2 B= 1 4 C= 2 2 0
34 23 220
find, if possible:
a A+B b A−B c B − C.
THINK WRITE

a Add the numbers in the corresponding a A+B= 1 2 + 1 4


positions of each matrix. 34 23

= 26
57

b Subtract the numbers in the corresponding b A−B= 1 2 – 1 4


positions of each matrix. 34 23

= 0 –2
1 1

c Subtraction cannot be performed since the order c B − C cannot be calculated because B


of B is 2 × 2 and the order of C is 2 × 3. and C are of different orders.

Multiplication by a scalar
Consider the matrix B = 1 4
23
To find 3B we could use repeated addition: 3B = B + B + B

= 14 + 14 + 14
23 23 23

= 3 12
6 9
3B could have been calculated more efficiently by multiplying each element of B by 3.
132 M a t h s Q u e s t M a t h s C Ye a r 1 1 f o r Q u e e n s l a n d

Thus 3B = 3 1 4 = 3 × 1 3 × 4 = 3 12
23 3×2 3×3 6 9
The number 3 in the term 3B is called a scalar because it is a real number. Terms
such as 3B refer to scalar multiplication of matrices.
When a matrix is multiplied by a scalar, each element of the matrix is multiplied
by the scalar.

WORKED Example 3
If A = 2 – 3 and B = 3 3 find:
4 1 –3 –2
a 2A b 5B c 2A + 5B d 4(A + B) e 2(B − A).
THINK WRITE

a Multiply each element of A by 2. a 2Α = 2 2 – 3


4 1

= 4 –6
8 2

b Multiply each element of B by 5. b 5Β = 5 3 3


–3 –2

= 15 15
– 15 – 10

c Add the two matrices found in parts a c 2Α + 5Β = 4 – 6 + 15 15


and b. 8 2 – 15 – 10

= 19 9
–7 –8
 
d 1 Add A and B inside the brackets. d 4(Α + Β) = 4  2 – 3 + 3 3 
 4 1 –3 –2 

=4 5 0
1 –1

2 Multiply each element of the resulting = 20 0


matrix by 4. 4 –4
 
e 1 Subtract A from B (inside the brackets). e 2(Β − Α) = 2  3 3 – 2 – 3 
 –3 –2 4 1

=2 1 6
–7 –3

2 Multiply each element of the resulting = 2 12


matrix by 2. – 14 – 6
Chapter 3 Matrices 133
There are some obvious but important features of scalar multiplication. If A and B
are matrices of the same order and a, b are real numbers, then:
1. aA + bA = (a + b)A
2. aA + aB = a(A + B)
3. (ab)A = a(bA)
Operations 1 and 2 are similar to the Distributive Law of Multiplication over Addition.
Operation 3 is similar to the Associative Law of Multiplication.
If aA = 0, then a = 0 or A is a zero matrix. A zero matrix is a matrix which has all
elements equal to zero.

Equality of matrices
This leads to an important principle about the equality of matrices.
Two matrices are equal if they are of the same order and all corresponding

elements are equal; that is, if A = a b and B = a b then A = B.


c d c d

Simple matrix equations


We know that to solve an algebraic equation such as 2x + 5 = 11, we:
1. subtract 5 from both sides to obtain 2x = 11 − 5 which gives 2x = 6
2. then, divide both sides by 2 (or multiply by 1--- ) to obtain x = 6 × 1--- or x = 3.
2 2
Simple matrix equations which require the addition or subtraction of a matrix or
multiplication of a scalar can be solved in similar ways to those employed with
algebraic equations.

WORKED Example 4
Solve the following matrix equations.

a 5A = 50 35 b P+ 3 2 = 6 9 c 2B – 1 2 – 3 = 3 4 7
– 15 20 15 –2 4 20 1 –2 6 –5

THINK WRITE

a 1 To get A by itself multiply both a 5Α = 50 35


sides by 1--- . – 15 20
5

Α= 1
---
50 35
5
– 15 20

2 Simplify the matrix A. Α = 10 7 ⇒


–3 4
Continued over page
134 M a t h s Q u e s t M a t h s C Ye a r 1 1 f o r Q u e e n s l a n d

THINK WRITE

b 1 To get P by itself subtract 3 2 from b P+ 3 2 = 6 9


15 15 –2 4
both sides.
P= 69 – 32
–2 4 15

2 Simplify the matrix P. ⇒P = 3 7


–3 –1

c 1 First get 2B by itself by adding c


2B – 1 2 – 3 = 3 4 7
1 2 – 3 to both sides. 20 1 –2 6 –5
20 1
2B = 3 4 7 + 1 2 –3
–2 6 –5 20 1

2 Simplify the right-hand side. = 46 4


0 6 –4

Multiply both sides by --1- to get B by itself. Β= 1


---
46 4
3 2
2
0 6 –4

4 Simplify the matrix B. Β= 2 3 2


0 3 –2

Matrix operations can also be performed using a graphics calculator. However, tips on
how to do this appear later in the chapter and in Chapter 5, as you should have first
practised performing these operations manually.

remember
remember
1. A matrix is a rectangular array of numbers arranged in rows and columns.
2. An m × n matrix has m rows and n columns.
3. The numbers in the matrix are called the elements of the matrix.
4. Elements are referred to by the row and column position. For example, ai j
refers to the ‘ith’ row and the ‘jth’ column of matrix A.
5. Addition of matrices is performed by adding corresponding elements.
6. Subtraction of matrices is performed by subtracting corresponding elements.
7. Addition and subtraction of matrices can be performed only if the matrices are
of the same order.
8. When a matrix is multiplied by a scalar, each element of the matrix is
multiplied by that scalar.
9. Two matrices are equal if they are of the same order and all corresponding
elements are equal.
Chapter 3 Matrices 135

3A Operations with matrices


WORKED 1 Using a table format, give the order of each of the following matrices and where
Example
possible write down the row 2, column 1 and row 1, column 3 elements of each.

1
1

E
D
C
B
A
Matrix
6 5 02
A = – 5 2 , B = 5 , C = 1 8 10 20 , D = 4 4 4 , E = 1 1 8
84 444

Order element element


3×3
2×3
1×4
3×1
2×2
7 0 –5 3

3 –6
2 If A = – 2 3 , B = 5 0 and C = 1 – 3 , calculate:

2, 1
WORKED


1
4

5
8
Example
47 –4 2 2 6 –2 –1
33 2 –7 3 6 –3
09 a A+B b A−B c B+C d C−A
85 –2 8

1, 3
10


2
4
WORKED 3 Using the matrices A, B and C from question 2 find:
Example
–4 6 3 a 2A b 2A − B c 2A + 3B d 3(A + B) e 2A + 3B − 4C
8 14 –9 6 11 6 9 9 7 18
4 multiple choice
12 12 – 4 20 0 27 – 12 – 4
eBook plus
1 23
Digital doc: Use A = –2 –1 4 , B = 5 –4 , C = –4 3 , D = 2 34 1 and
SkillSHEET 3.1 1 3 27 0 –2 7 5
Operations 6 30
with matrices

0 5 –1
E = 2 3 9 to answer questions a to e.
6 4 –2
a The order of D is:
A 3×2 B 4×2 C 2×4 D 3×3 E 2×2
5 a
2 0 14
b Which one of the following cannot be calculated?
40 0 A A+E B B+C C 4D D A+B E 6(E − A)
6 0 18 c 2A + 3E is equal to:
b 3 16 7 17 2 4 15 – 2 2 16 3 2 19 3
0 8 0
0 10 16 A – 2 3 30 B 0 2 13 C 6 3 8 D – 2 7 35 E 2 7 35
0 12 0 30 17 – 6 12 7 – 2 9 7 2 30 18 – 6 30 18 – 6
c
2 8 14 d 3C − 4B is equal to:
4 10 16
A – 32 25 B 8 –7 C – 31 0 D 10 E – 32 – 7
6 12 18
2 9 10 33 5 19 01 5 9
d
3 4 21 e The element e3 2 is equal to:
65 8
A 9 B 4 C 3 D 7 E −1
9 6 27
e
4 0 28 147 1 –4 7
80 0 5 If C = 2 5 8 and D = 2 – 5 – 8 , calculate:
12 0 36
369 3 –6 9
f
–1 4 –7 a C+D b C−D c 2C d 2C + D e 2(C + D) f −D
–2 5 8
–3 6 –9
136 M a t h s Q u e s t M a t h s C Ye a r 1 1 f o r Q u e e n s l a n d

WORKED 6 Solve the following matrix equations.


Example
4
a 3P = 6 0 2 0 b Q+ 4 0 = 2 0 –2 0
9 –3 3 –1 14 –5 6 –6 2

2 –2 0 4 2 –6
c 3M – – 2 0 3 = – 1 0 0 –1 0 1 d
6 –6 – N = 7 6 – 1 – 12
4 6 –1 2 3 –2 2 3 –1
3 –3 5 –2 –2 –1

7 Explain why the following matrix equation has no solution. Different orders

2A + 48 = 513
–4 0 –2 5 2

8 Write down the matrix representing the following maps in the form:
Number of
routes to

Number
of routes
from

Use alphabetical order for the sequencing of rows and columns.


a A B 0131
b J 0 0 1 1
1022 0 0 1 2
D 3201 1 1 0 3
K
1210 1 2 3 0
C
L

82 54 9 A mathematically inclined student has decided to keep a record of her test results in
75 68 matrix form. Her results so far are Maths B tests: 82%, 75% and 91%; Maths C tests:
91 82 54%, 68% and 82%.
Write these results in a 3 × 2 matrix.

15 14 104 10 Place the following sporting results in a suitable matrix format.


7 10 52 a Brisbane Lions 15 goals 14 behinds 104 points defeated Geelong 7 goals 10
behinds 52 points.
13 7 5 1 31 18 26 b Adelaide Crows have played 13 games for 7 wins, 5 draws and 1 loss; they have
12 4 4 4 17 15 16 scored 31 goals for and 18 against; their points score is 26. Fremantle have played
12 games for 4–4–4; their goals are 17–15 and their points score is 16.
14 8 5 1 35 19 29
11 Adelaide Crows defeat Fremantle 4 goals to 1. Update the matrix in question 10b
13 4 4 5 18 19 16
(note that 3 points are awarded for a win and 0 for a loss).

12 Write down any 2 × 2 matrices called A, B and C. Check if the following are true.
a A + B = B + A True
b (A + B) + C = A + (B + C) True
c A − B = B − A False
d 2A + 2C = 2(A + C) True
Chapter 3 Matrices 137
Multiplying matrices
The sports results at Mathglen State High School were:

Position Position Points


House 1st 2nd 3rd 1st 5
2nd 3
Hamilton 60 63 51 3rd 1
Leslie 71 64 74
Barnes 64 69 71
Cunningham 69 72 68

5
To calculate the total points for each house, this matrix is multiplied by 3 since 5
points are awarded for first, 3 for second and 1 for third.
1

The result can be obtained using the following operations.


Hamilton: 60 × 5 + 63 × 3 + 51 × 1 = 540
Leslie: 71 × 5 + 64 × 3 + 74 × 1 = 621
Barnes: 64 × 5 + 69 × 3 + 71 × 1 = 598
Cunningham: 69 × 5 + 72 × 3 + 68 × 1 = 629

60 63 51 540
5
We can also write A × B = C, where A = 71 64 74 , B =
3 and C =
621
64 69 71 598
1
69 72 68 629

The order of A is 4 × 3, B is 3 × 1 and C is 4 × 1.


Therefore, a 4 × 3 matrix multiplied by a 3 × 1 matrix gives a 4 × 1 matrix. Two
matrices can be multiplied only if the number of columns of the first matrix equals the
number of rows of the second matrix.

In general, if A is of order m × n and B is of order n × p then A × B exists and its


order is m × p. Such a matrix is said to be conformable where m × n multiplied by
n × p results in a matrix of order m × p.

The order of AB should be established before multiplying.


The procedure for multiplying two 3 × 3 matrices is outlined below.

a 11 a 12 a 13 b 11 b 12 b 13
If A = a 21 a 22 a 23 and B = b 21 b 22 b 23
a 31 a 32 a 33 b 31 b 32 b 33

then
138 M a t h s Q u e s t M a t h s C Ye a r 1 1 f o r Q u e e n s l a n d

a 11 × b 11 + a 12 × b 21 + a 13 × b 31 a 11 × b 12 + a 12 × b 22 + a 13 × b 32 a 11 × b 13 + a 12 × b 23 + a 13 × b 33
AB = a 21 × b 11 + a 22 × b 21 + a 23 × b 31 a 21 × b 12 + a 22 × b 22 + a 23 × b 32 a 21 × b 13 + a 22 × b 23 + a 23 × b 33
a 31 × b 11 + a 32 × b 21 + a 33 × b 31 a 31 × b 12 + a 32 × b 22 + a 33 × b 32 a 31 × b 13 + a 32 × b 23 + a 33 × b 33

The rows of the first matrix are multiplied by the columns of the second matrix.
The sum of the products of the elements of row 1 multiplied by column 1 results in the
row 1, column 1 element.
The sum of the products of the elements of row 3 multiplied by column 2 results in the
row 3, column 2 element.

WORKED Example 5
2 –1
A = 123 , B = 0 4
456
5 3
a Write down the order of the two matrices.
b Which of these products exist? i AB ii BA
c Write down the order for the products which exist.
d Calculate the products which exist.
THINK WRITE
a 1Matrix A has 2 rows and 3 a A is a 2 × 3 matrix.
columns.
2 Matrix B has 3 rows and 2 B is a 3 × 2 matrix.
columns.
b i AB is the product of a 2 × 3 b i AB exists since a 2 × 3 matrix multiplied by a
and a 3 × 2 matrix so it exists. 3 × 2 matrix results in a 2 × 2 matrix.
A and B are conformable.
ii BA is the product of a 3 × 2 ii BA exists since a 3 × 2 matrix multiplied by a
and a 2 × 3 matrix so it also 2 × 3 matrix results in a 3 × 3 matrix.
exists.
c i The product of AB is a 2 × 2 c i AB is a 2 × 2 matrix.
matrix.
ii The product of BA is a 3 × 3 ii BA is a 3 × 3 matrix.
matrix.
2 –1
d i 1 Multiply the rows of matrix d i AB = 1 2 3 0 4
A by the columns of matrix 456
5 3
B.
1×2 + 2×0 + 3×5 1 × –1 + 2 × 4 + 3 × 3
AB =
4×2 + 5×0 + 6×5 4 × –1 + 5 × 4 + 6 × 3

2 Simplify AB. AB = 17 16
38 34
Chapter 3 Matrices 139
THINK WRITE
2 –1
ii 1 Multiply the rows of B by the ii BA = 0 4 1 2 3
columns of A. 456
5 3
2 × 1 + –1 × 4 2 × 2 + – 1 × 5 2 × 3 + – 1 × 6
ii BA = 0×1+4×4 0×2+4×5 0×3+4×6
5×1+3×4 5×2+3×5 5×3+3×6

–2 –1 0
2 Simplify BA. ii BA = 16 20 24
17 25 33

Note: In Worked example 5, AB is a 2 × 2 matrix but BA is a 3 × 3 matrix. In general,


matrix multiplication is not commutative.
That is, for two matrices A and B, AB ≠ BA.
For the product AB we say that A is post-multiplied by B and B is pre-multiplied by A.

The identity matrix


There is one circumstance in which matrix multiplication is commutative. Look at the
following example.

WORKED Example 6
If A = 2 – 3 and I = 1 0 , calculate AI and IA.
–5 4 01
THINK WRITE
1 A and I are both 2 × 2 matrices so both A 2 × 2 matrix multiplied by a 2 × 2 matrix
the products AI and IA exist and are of results in a 2 × 2 matrix.
order 2 × 2.

Find AI using the procedure for AI = 2 –3 1 0


2
multiplying matrices. –5 4 0 1

= 2 –3
–5 4

3 Find IA using the procedure for IA = 1 0 2 – 3


multiplying matrices. 0 1 –5 4

= 2 –3
–5 4

This example demonstrates the only case in which matrix multiplication is always com-
mutative — that is, when AI = IA = A. Here, I is called the multiplicative identity matrix.
140 M a t h s Q u e s t M a t h s C Ye a r 1 1 f o r Q u e e n s l a n d

The multiplicative identity matrix, I, acts in a similar fashion to the number 1 when
numbers are multiplied, where I is the multiplicative identity matrix.
An identity matrix can be defined only for square matrices; that is, for matrices of
order 1 × 1, 2 × 2, 3 × 3. The other feature of an identity matrix is that it has the
number 1 for all elements on the leading diagonal and 0 for all other elements.
1 0 0 0 0
0 1 0 0 0
Leading diagonal
0 0 1 0 0
0 0 0 1 0
0 0 0 0 1
AI = IA = A where A is a square matrix and I is the multiplicative identity matrix.
If A is not square (say it is 3 × 2), then A × I = A means I would have to be a 2 × 2
matrix because a 3 × 2 matrix multiplied by a 2 × 2 matrix results in a 3 × 2 matrix.
But I × A = A means that I would be a 3 × 3 matrix because a 3 × 3 matrix multiplied
by a 3 × 2 matrix results in a 3 × 2 matrix. However, I cannot be a 2 × 2 and a 3 × 3
matrix at the same time. Therefore I can be defined only for square matrices.

remember
remember
1. In general, if A is of order m × n and B is of order n × p then A × B exists and
its order is m × p; that is, A and B are conformable.
2. In general, for two matrices A and B, AB ≠ BA.
3. AI = IA = A where A is a square matrix and I is the multiplicative identity matrix.

3B Multiplying matrices
24
WORKED 1 A = 2 –3 , B = 1 1 , C = 6 8 , D = –2 4 , E = –2 3 –1 , I = 1 0
Example
4 5 10 0 –4 2 01
5 01
a Write down the order of the six matrices. A (2 × 2), B (2 × 2), C (3 × 2), D (1 × 2), E (2 × 3), I (2 × 2)
– 4 18 – 8 2 – 3
–8 –8 6 4 5

2 – 3 1 1 – 8 – 21 14 15
4 5 1 0 28 13 – 24 – 30

b Which of the following products exist? CA, DB, AE, AI, IA, IB, A2, EC
i AC ii CA iii DB iv BD v AE vi AI
2 2
vii IA viii IB ix EB x E xi A xii EC
c Write down the order of the products which exist. (3 × 2), (1 × 2), (2 × 3), (2 × 2),
(2 × 2), (2 × 2), (2 × 2), (2 × 2)
d Calculate those products which exist.
44 22 2 – 2

2 a If M = 2 4 and N = 5 2 , calculate MN and NM. 10 20 8 26


20 14

4 5

– 5 10 – 4 12
–1 3 04
b Is matrix multiplication commutative? That is, does MN = NM? No
d

WORKED 3 A = 2 – 1 , B = 2 0 , C = 5 – 2 , D = 3 2 , I = 1 0 and O = 0 0
Example
0 3 0 –3 8 3 –8 5 01 00
6
Calculate the following products.
a AB b AC c DO d DI e IB f BC g CD h CA i OI j ID
4 3 2 –7 00 32 2 0 10 – 4 31 0 10 – 11 00 32
0 –9 24 9 00 –8 5 0 –3 – 24 – 9 0 31 16 1 00 –8 5
Chapter 3 Matrices 141
4 a Calculate the following products.

i 4 3 4 –3 10 ii –2 –3 –8 3 10 iii –1 –2 –5 2 10
5 4 –5 4 01 –5 –8 5 –2 01 –2 –5 2 –1 01
b What do you notice about all of the answers? All are I
c What term could be given to these matrices? Multiplicative inverses

5 multiple choice
Use the matrices below to answer questions a to d.
2 5 12 3
A = 3 2 , B = 2 –2 4 , C = 1 –3 , D = –2 0 2 , E = 5 2 , F = 3 –2 4
01 1 36 –1 –3
0 4 4 1 –3
a Which one of the following products does not exist?
A AD B AB C BC D FC E CE
b The order of the matrix BD is:
A 2×2 B 3×3 C 2×3 D 5×3 E 4×3
c Which one of the following products gives a matrix of order 2 × 2?
A BF B AB C DC D BC E FD
d Which one of the following represents the matrix CE:
20 – 17 5 – 11
A –1 8 B C Does not D 5 8 –4 E 20 – 1 8
8 11
exist. – 11 11 – 12 – 17 8 – 12
8 – 12 – 4 – 12
6 The matrix below shows the number of wins, draws and losses for two soccer teams,
the Sharks and the Dolphins.
10 2 5
8 72

Thus the Sharks have 10 wins, 2 draws and 5 losses. If 3 points are awarded for a win,
3 1 for a draw and 0 for a loss:
1 a write down a 3 × 1 matrix for the points awarded Sharks have a total of
32
0 b use matrix multiplication to find the total points for the two teams. 32 points. Dolphins have
31 a total of 31 points.
7 In Australian Rules Football, 6 points are awarded for a goal and 1 point for a behind.
The scores in two games were:
Southport 18–12 defeated Broadbeach 14–15 and Lions 10–14 defeated Eagles 9–16.
142 M a t h s Q u e s t M a t h s C Ye a r 1 1 f o r Q u e e n s l a n d

18 12
The first number is for goals scored and the second is for behinds.
14 15
a Write the results in a 4 × 2 matrix. 6 Southport 120, Broadbeach 99,
10 14
b Write down the 2 × 1 matrix for the points. 1 Lions 74, Eagles 70
9 16
c Use matrix multiplication to find the total number of points scored by each team.
8 Two shops, A and B, are supplied with boxes of different brands of chocolates —
Yummy, Scrummy and Creamy — as shown in this table:
Yummy Scrummy Creamy
Shop A 20 20 10
Shop B 10 5 10
eBook plus
The cost of the boxes are Yummy $10, Scrummy $25 and Creamy $12. 10
Digital doc:
a Write down the costs in a 3 × 1 matrix. 25
WorkSHEET 3.1
b Use matrix multiplication to find the total cost for each shop. 12
Shop A = $820, Shop B = $345

History of mathematics
O L G A TA U S S K Y- T O D D
( 3 0 Au g u s t 1 9 0 6 – 7 O c t o b e r 1 9 9 5 )
During her life … teaching position. In London she met and
Mt Everest is finally climbed. later married Jack Todd.
The Richter scale for measuring the strength of After the Second World War, the couple
earthquakes is devised. moved to America where Olga began work on
Morse code is used by the Titanic when it sinks. the design of computers. In 1943, she moved
Gandhi struggles to free India from British Rule. to the Ministry of Aircraft where she
conducted research into stability in matrices.
Olga Taussky-Todd worked in the fields of This work encouraged her to look in more
matrix theory and number theory. detail at matrix theory.
She was born in Olmütz, now part of the Olga was awarded the Austrian Cross of
Czech Republic, but when she was three the Honour, which is Austria’s highest award; in
family moved to Vienna and later to Linz. 1964, she was named woman of the year by
Her father died early so it became difficult for the Los Angeles Times. In 1970 she was
her to continue her studies. Her father, an awarded the Ford Prize for her publication on
industrial chemist, had encouraged her ‘The Sums of Squares’. In 1971 she was
studies in mathematics. named Professor Emeritus at CalTech.
Olga went to the University of Vienna
where she studied mathematics and chemistry. Questions
She completed a doctorate in 1930 with 1. What field of mathematics was Olga’s
research into algebraic number fields. After speciality? Matrix theory and number theory
completing her studies she was employed at 2. What did Olga work on when she
moved to America? Computer development
the university of Göttingen as an assistant and
3. What award did Olga receive from the
worked with Helmut Ulm by editing his book
Austrian Government? Cross of Honour
on number theory. By 1932 Olga had been
4. Where was Olga a professor? Caltech
promoted to the position of tutor.
In 1935 Olga moved to Cambridge where Research
she undertook a research fellowship before Find out about the uses of matrices,
moving to London in 1937 to take up a especially in dynamic programming.
Chapter 3 Matrices 143
Powers of a matrix
A logical extension of matrix multiplication is using the power of a matrix, where
A1 = A
A2 = AA
A3 = A2A, and so on.
In general form, An = An –1A, where n is a positive integer.
But what dimension can matrix A have?

Matrix powers
x y Investigate powers of matrices by completing the following steps. (Remember to
1 a A= use pronumerals for the elements of A, not constant values.)
z w
p q 1 a Let matrix A be any 3 × 2 matrix.
x y x y b Find A2.
b A = z w
2
z w c What do you notice? Cannot multiply A × A if A is a 3 × 2 matrix
p q p q 2 a Let matrix A be any 2 × 2 matrix. 3 a A= x y z
2 w p q
b Find A .
Not conformable
3 a Let matrix A be any 2 × 3 matrix. x y z x y z
b A2 =
b Find A2. w p q w p q
2 multiply A × A if A is a
a A= x y c What do you notice? 2Cannot × 3 matrix Not conformable
z w 4 a What general conclusion can you make concerning the order of a matrix
that is to be raised to a power? If a matrix is to be raised to a power it must be a square matrix.
b A2 = x y x y
b Justify your conclusion by referring to the dimensions of matrices involved
z w z w
in a product.
A2 = x + yz xy + yw From the above investigations we can conclude that A × A must be conformable;
2

zx + zw zy + w that is, the number of columns of the first factor in the product should be the same
2

as the number of rows in the second factor.


That is, A must be a square matrix where n × n is multiplied by n × n to get A2.
Hence powers of matrices are only defined for square matrices.

WORKED Example 7
If A = 1 3 , find: a A2 b A3
–1 2

THINK WRITE
a Write the power as a product. a A2 = A × A

A2 = 1 3 1 3
–1 2 –1 2

A2 = – 2 9
–3 1
Continued over page
144 M a t h s Q u e s t M a t h s C Ye a r 1 1 f o r Q u e e n s l a n d

THINK WRITE
b Write the power as the product of lesser b A3 = A2A
powers.
A2 = – 2 9 1 3
–3 1 –1 2

A2 = – 11 12
–4 –7

remember
remember
1. The power, n, of matrix A, in general form, is An = An – 1A, where n is a positive
integer.
2. Powers of matrices are only defined for square matrices; that is, A has to be a
square matrix to obtain An.

3C Powers of a matrix

WORKED 1 If A = 2 – 1 , find:
Example
xample
0 0
7 8 –4 16 – 8
2
a A b A3 c A4
4 –2 0 0 0 0
0 0
1 0 0
2 If A = 0 1 0 , find:

1 0 0 0 0 1 1 0 0 1 0 0
2 3 n
0 1 0 a A b A 0 1 0 c A 0 1 0
0 0 1 0 0 1 0 0 1
1 0 0
3 If A = 2 3 0 , find:

1 0 0 0 1 1 1 00
2
8 9 0 a A b A3 26 27 0
2 4 1 10 13 1
1 1 0
4 If A = – 2 – 2 – 1 :
–1 –1 –1 1 1 0
1 1 2 a find A2 b confirm that A2A = AA2
–1 –1 –1
Chapter 3 Matrices 145
Applications of matrices
1 A garden supplier provides live plants for displays
22
15

12
10
18
8

in 5 penthouse patios, 7 office foyers, 3 banks and


4 hotels. The plants in each different type of display
Hanging basket

Indoor plant
Geranium

are listed below.


Camellia

Palm
Fern

• The patio displays consist of 2 ferns, 1 camellia,


Cost: C =

1 geranium and 2 hanging baskets.


• The office foyer displays have 1 palm, 1 geranium,
3 hanging baskets and 2 indoor plants.
iii

• The bank displays have 1 palm, 3 camellias and


5 indoor plants.
• The hotel displays have 2 palms, 3 ferns,
Palm

1
1
2
0

2 camellias, 2 hanging baskets and 5 indoor plants.


The wholesale cost of each plant bought by the supplier is:
Indoor
plant

2
5
5
0

ferns $22, palms $18, geraniums $8, camellias $15,


Hanging

hanging baskets $12 and a variety of indoor plants that


Camellia Geranium basket

3
0
2
2

cost $10 on average.


The supplier needs to be able to use this information to calculate costs of
1
0
0
1

displays, number of plants required and profits, but in this form, the information is
Type and number:

difficult to handle.
1
0
3
2

a Develop matrices to display the following information (labels outside the


Fern

2
0
0
3

matrices will help clarify the meaning of the elements): Patio Office Bank Hotel
iii the number of displays supplied to each type of venue Venue: V = [ 5 7 3 4 ]
Office

Hotel
Bank
Patio

iii the number and variety of plants used in each display


T=

iii the cost of each type of plant. Quantity:


b Use matrix operations to determine the following: Hanging Indoor
ii

Fern Camellia Geramium basket plant Palm


iii the quantities of each plant needed to fill the orders Q = [ 22 22 12 39 49 18 ]
iii
Patio 163.80 iii the supplier’s total outlay to provide the displays TC = $2192
Office 147.60 iii the charge for each type of display if the supplier adds 80% profit to the
CD =
Bank 203.40 (iii) cost.
Hotel 370.80
2 A home builder advertised three
designs of ‘Ownit Homes’ to entice
people to buy rather than rent their
home — the ‘Taps’ for $155 per week,
the ‘Avalon’ for $203 per week and the
‘Torana’ for $238 per week. The weekly
payments were based on finance
available from a public finance company.
Ownit Homes received orders for 10 ‘Taps’ homes, 8 ‘Avalon’ homes and
12 ‘Torana’ homes. The materials (given in units as stated in their building guide)
required for each home are listed below:
• The ‘Taps’ home requires 9 units of steel, 11 of timber, 6 of glass, 7 of paint and
20 of labour.
• The ‘Avalon’ home requires 12 units of steel, 14 of timber, 15 of glass, 12 of
paint and 25 of labour.
• The ‘Torana’ home requires 14 units of steel, 12 of timber, 12 of glass, 16 of
paint and 24 of labour.
146 M a t h s Q u e s t M a t h s C Ye a r 1 1 f o r Q u e e n s l a n d

35]
Buns

To reduce costs all materials are purchased from one supplier. The prices per unit
Pastries
65

are steel $660, timber $1140, glass $1020, paint $660 and labour is priced at $1128
per unit.
Cakes
150 45

Use matrix methods to obtain the following information:


Bread

a the amount of money the bank would receive per week from the repayments on
these homes $5018
Sugar rolls

b the total cost of raw materials for all the constructions. $1 661 420
[15

3 A small bakery sells 5 main items:


N=

sugar rolls, bread, cakes, pastry and buns.


The major ingredients (given in applicable
1.20

1.20
1.50
1.20
S2

units) required to make one of each item


1

are listed below.


0.80

1.20
1.20
S1

• Sugar rolls (1 dozen) require 1 egg,


1

4 units of flour, 0.25 of sugar, 0.25 of


Shortening

shortening and 1 of milk.


Sugar
Flour
Eggs

Milk

• Bread (1 loaf) requires 3 units of flour,


1 Q=

0.25 of shortening.
Milk

1
1
0

• Cake (1) requires 4 eggs, 3 units of


Shortening

flour, 2 of sugar, 1 of shortening and 1 of milk.


0.25
0.25

0.33

• Pastry (1) requires 1 egg, 1 unit of flour, 0.33 of shortening.


1

• Buns ( 1 dozen) require 2 units of flour, 3 of sugar, 1 of shortening and 1 of milk.


0.25
Sugar

Two suppliers (Supplier 1 and Supplier 2) provide quotes for the ingredients, given

2

3

as ordered pairs with prices given in dollars:


Flour

4
3
3
1
2

eggs (1, 1.20), flour (0.8, 1), sugar (1, 1.20), shortening (1.20, 1.50) and milk
Eggs

(1.20, 1.20).
1
0
4
1
0
Bread loaves

For one office function the following orders were received:


Sugar rolls

Pastries
Cakes

15 dozen sugar rolls, 150 loaves of bread, 45 cakes, 65 pastries and 35 dozen buns.
Buns

a Represent all the above information in matrix form taking into account
A=

ingredients, orders, suppliers’ quotes.


260 eggs, 780 units of b Use these matrices to provide a list of the amounts of the ingredients required
flour, 198.75 units of sugar,
142.7 units of shortening to fill the orders for the function.
and 95 units of milk c Which supplier provides the cheapest total quote? What savings are made by
Sugar rolls $11.30, using this supplier? Supplier 1 is cheaper by $290.56.
Bread loaves $5.15, d Provide a list of selling prices (to the nearest 5 cents) if a 90% markup on the
Cakes $20.50, cost prices is used to fix the price.
Pastries $4.15, e Calculate the total takings based on this information from part d. $2599.75
Buns $13.30

Multiplicative inverse and solving


matrix equations
In question 4 of exercise 3B, you should have found that the product of the matrices
was I. This means that one matrix is the multiplicative inverse of the other. We use the
symbol A−1 for the multiplicative inverse of A.
If AA-1 = A-1A = I, then A−1 is called the multiplicative inverse of A.
In working with numbers, a similar result would be 7 × 1--- = 1 or 4--- × 5--- = 1. Numbers
7 5 4
such as these are called reciprocals or multiplicative inverses of each other.
Chapter 3 Matrices 147
WORKED Example 8
If A = 4 1 and B = 3 – 1 , find AB and hence write down the multiplicative inverse
63 –6 4
of A.
THINK WRITE

1 AB will be a 2 × 2 matrix since A and B are both AB = 4 1 3 – 1


2 × 2 matrices. 6 3 –6 4

= 60
06

6 0 = 6 1 0 = 6I since 6 is a common factor AB = 6I


2
06 01
of each element.
3 To produce I we need to multiply both sides by 1--- . A ( 1--6- B ) = I
6

So A−1 = 1--- B
6

3
--- – 1--6-
= 6
4
4 Since A ( 1--6- B ) = I , the inverse of A is 1--- B. – 6--6- ---
6
6

1
--- – 1--6-
= 2
2
–1 ---
3

Inverse of a matrix
Consider matrix A, a 2 × 2 matrix, such that A = a b . If a multiplicative inverse of A
exists, then A × A–1 = I. c d

If A–1 exists, let A–1 = x y


u v
That is,
AA–1 = I

LHS = a b x y
c d u v

LHS = ax + bu ay + bv
cx + du cy + dv
LHS = RHS

LHS = 1 0
0 1
148 M a t h s Q u e s t M a t h s C Ye a r 1 1 f o r Q u e e n s l a n d

Equating terms of the two matrices:


ax + bu = 1 [1]
cx + du = 0 [2]
ay + bv = 0 [3]
cy + dv = 1 [4]
Solving this system of simultaneous equations:
Use [1] and [2] to eliminate x by multiplying equation [1] by c and equation [2] by a.
acx + bcu = c [5]
acx + adu = 0 [6]
Equation [6] minus equation [5] gives:
adu − bcu = −c
u(ad − bc) = −c
–c
u = ------------------ This will replace u in A–1.
ad – bc
Continue in a similar fashion to arrive at:
–b d a
y = ------------------ x = ------------------ and v = ------------------
ad – bc ad – bc ad – bc

d -
----------------- –b -
-----------------
Therefore A = ad –1 – bc ad – bc
–c -
----------------- a -
-----------------
ad – bc ad – bc
1
= ------------------ d – b where ad − bc ≠ 0
ad – bc – c a
If ad – bc = 0 then this scalar is undefined, therefore A–1 does not exist. That is, there is
no matrix that, when multiplied by A will yield I, the identity matrix. If A has no
inverse then it is said to be singular.
There is a relationship between A and A−1 which is outlined below. If A is the matrix
a b , proceed as follows.
c d

1. Swap the elements on the main diagonal of A and multiply the elements

on the other diagonal by −1 . This gives the matrix d –b .


–c a
2. Evaluate ad – bc.
1
3. Divide each element by (ad − bc) (or multiply by ----------------------- ).
( ad – bc )
These steps demonstrate a clear method for finding the multiplicative inverse of a
matrix.

1
The inverse of A = a b is A –1 = ------------------ d – b .
c d ad – bc – c a
The number (ad − bc) is called the determinant of the matrix A and is written as
det A or |A|.
Chapter 3 Matrices 149
Note: Only square matrices have inverses.
We will be concerned only with the inverse of 2 × 2 matrices at this stage in this course.

WORKED Example 9
If C = 2 – 3 find C −1.
1 5
THINK WRITE
1
1 Write the general form of C and the C= a b C –1 = ------------------ d – b
general form of its inverse. c d ad – bc – c a

1
2 Swap the elements on the main C –1 = ------------------------------------------- 5 3
( 2 × 5 ) – ( –3 × 1 ) –1 2
diagonal of C.
1
5 = ----------------------- 5 3
10 – ( – 3 ) – 1 2
2
Multiply the elements on the other
diagonal of C by −1.
3
–1

1
3 Write down the inverse of C. C –1 = ------ 5 3
13 – 1 2

We can check that CC –1 = I and C –1 C = I .


C–1 is the multiplicative inverse of C if C × C –1 = C –1 × C = I.

CC –1 = 1-
----- 2 –3 5 3 C –1 C = 1-
----- 5 3 2 –3
13 13
1 5 –1 2 –1 2 1 5

= 1-
----- 13 0 = 1-
----- 13 0
13 13
0 13 0 13
= I = I

Singular matrices
1
Matrices for which the determinant equals 0 do not have an inverse, since --- is
0
undefined. Such matrices are called singular matrices.
If det A = 0 then A is singular and an inverse does not exist.
There are two special types of singular matrices: nilpotent and idempotent.
A square matrix A is nilpotent if A2 = O where O is the zero matrix. The zero
matrix is a square matrix with all elements equal to zero. For example, the 2 × 2
00
zero matrix is .
00
150 M a t h s Q u e s t M a t h s C Ye a r 1 1 f o r Q u e e n s l a n d

A square matrix A is idempotent if A2 = A. The only non-singular idempotent matrix is


the identity matrix.

WORKED Example 10
Show that

a 6 – 3 is nilpotent
12 – 6

b 5 2 is idempotent.
– 10 – 4
THINK WRITE

a 1 Nilpotent means that A2 = O. a A2 = 6 –3 6 –3


Find A2. 12 – 6 12 – 6

= 00
00
2 State your conclusion. A2 = 0; therefore A is nilpotent.

b 1 Idempotent means that A2 = A. b A2 = 5 2 5 2


Find A2. – 10 – 4 – 10 – 4

= 5 2
– 10 – 4
2 State your conclusion. A2 = A; therefore A is idempotent.

Further matrix equations


Matrix equations of the type AX = B may be solved by using the properties of
multiplicative inverses.
A matrix equation AX = B is similar to the equation 3x = 7. To solve this we would
divide both sides of the equation by 3 (or multiply by 1--- ). To solve the matrix equation
3
we multiply both sides by A−1. Since the order of multiplying matrices is important we
must be careful in which position we multiply by the inverse.
1. For AX = B
Pre-multiply by A−1: A−1AX = A−1B
or IX = A−1B since A−1A = I
X = A−1B since IX = X
2. For XA = B
Post-multiply by A−1: XAA−1 = BA−1
or XI = BA−1 since AA−1 = I
X = BA−1 since XI = X

1. If AX = B, then X = A−1B.
2. If XA = B, then X = BA−1.
Chapter 3 Matrices 151
Note: A–1 cannot be ‘inserted’ between 2 matrices. It can either pre- or post-multiply A
on one side of a matrix equation.

WORKED Example 11
A = 1 2 and B = 2 5
03 –2 1
Find X if:
a AX = B
b XA = B.

THINK WRITE

a 1 We require A−1 so first calculate det A. a A= 1 2


03
∴ det A = 3 − 0 = 3

2 Write down A−1. A−1 = --1- 3 –2


3
0 1

3 Write the equation. AX = B

4 Pre-multiply both sides of the equation A−1AX = A−1B


by A−1.

5 Remember A−1A = I and IX = X.


Calculate the product of A−1 and B. X= 1
--- 3 –2 2 5
6 3
0 1 –2 1

= 1
--- 10 13
3
–2 1

b 1 Write the equation. b XA = B

2 Post-multiply both sides of the equation X = BA−1


by A−1.
Calculate the product of B and A−1 = 1
--- 2 5 3 –2
3 3
using A−1 which was found in part a. –2 1 0 1

= 1
--- 61
3
–6 5

In part a of Worked example 11 both sides of the equation were pre-multiplied by A−1;
in part b both sides were post-multiplied by A−1. Remember that the matrix and its
inverse must be next to each other so that AA−1 = I.
Fractional scalars should be left outside the matrix unless they give whole numbers
when multiplied by each element.
152 M a t h s Q u e s t M a t h s C Ye a r 1 1 f o r Q u e e n s l a n d

remember
remember
1. If AA−1 = A−1A = I, then A−1 is called the multiplicative inverse of A.
1
2. The inverse of A = a b is A−1 = ------------------ d – b
c d ad – bc – c a
The number (ad − bc) is called the determinant of the matrix A and is written
as det A or | A |.
3. If det A = 0 then A is singular and an inverse does not exist.
4. (a) If AX = B, then X = A−1B.
(b) If XA = B, then X = BA−1.

Multiplicative inverse and


3D solving matrix equations 1 AB = 6 1 0
01
1
4 1 and B = 1 – 1 , find AB and hence write down: a A –1 = --- B
WORKED 1 If A = 6
Example
–2 1 2 4 1
8 b B –1 = --- A
a the inverse of A b the inverse of B. 6

eBook plus
2 If M = 2 6 and N = – 1 – 6 , find MN. Hence write down M −1 and N −1.
Digital doc: 0 –1 0 2
SkillSHEET 3.2
Inverse of a
3 Calculate the determinants of the following matrices. 2 MN = – 2 1 0 ,
2 × 2 matrix
01

a A = 2 3 5 b B = –2 –3 12 c C = 2 6 –1 1
−2 M = – --- N ,
5 10 4 0 0 –1 2
–1 1
N = – --- M
d D = 43 −8 e E = –2 –1 7 f F = 2 –1 14 2
41 –3 –5 6 4
1
WORKED 4 Write down the inverses of each matrix in question 3. 4 a --- 10 – 3
Example 5 –5 2
9
5 multiple choice 1
b ------ 0 3
12 – 4 – 2
Using the matrices below, select the correct answer in questions a to d.
1
c --- 1 6
P = 4 3 , Q = 2 – 3 and R = 8 6 2 0 –2
2 –1 –1 0 –4 2
1
a Det P is equal to: d --- – 1 3
8 4 –4
A 10 B 2 C −10 D −2 E 8
−1
b R is equal to: 1
e --- – 5 1
7 3 –2
A − 1--- 2 –6 B 1
---
8 6 C 1
------
86
8 8 40
4 8 –4 2 –4 2 1
f ------ 4 1
14 – 6 2
D − -----
1- 2 –6 E 1-
-----
2 –6
40 40
4 8 4 8
Chapter 3 Matrices 153
c Det (PQ) is equal to:
A −30 B 10 C −10 D 30 E −20
d If QX = R, then X is equal to:

A − 1--- – 12 6 B 1
---
12 6 C − 1--- 12 6
3 3 3
0 10 0 – 10 0 – 10

1
7 a --- 1 6 D 1
---
6 28 E − 1--- 6 28
2 0 –2 3 3
2 8 2 8
1 12
b --4- 6 Write down a 2 × 2 matrix which is singular. Answers will vary.
–2 0

c 12 2
7 C = 2 6 , D = 0 –2
–2 –1 0 –1 2 1
1
d --- 1 2 Find:
8 – 2 – 12 a C −1 b D−1 c CD d (CD)−1 e C −1D−1 f D−1C −1
1
e --- – 11 2 8 Explain why these matrices do not have an inverse.
8 40
1 14
f --- 1 2
8 – 2 – 12 a D = 21 D − det = 0 b E = –2 –4 c F = 25 F − Not a square matrix
42 5 10
E − det = 0 36 9
matrices where the matrix, A, is
A 2 = O, (O is the zero matrix).
nilpotent if it has the property

0 8 1
a b --- – 2 – 8
SLE 13: Research nilpotent

9 If A = 4 0 and B = 0 2 , find: –1 –2 8 1 0
–1 1 –1 0
−1 12
a AB b (AB)
1
Check with a --- – 31 – 22
WORKED 10 Show that the following matrices are nilpotent. your teacher. 2 24 18
Example
10a 4 2 – 10 20 6 –9
a b c 1
b --- – 5 5
–8 –4 – 5 10 4 –6 2 14 – 8
matrices where the matrix, A, is
idempotent if it has the property
SLE 14: Research idempotent

Check with
WORKED 11 Show that the following matrices are idempotent. your teacher.
1
c --- – 6 2
Example 6 –6 4
10b 6 –3 –4 4 10
a b c
1
10 – 5 –5 5 50 d --- 18 23
2 – 12 – 16

12 Let A = 2 3 , B = 5 5 , C = 0 – 1 . Find X if: 1


A2 = A .

WORKED
Example e ------ 78 103
45 –2 1 6 6 30 – 24 – 34
11
a AX = B b XA = B c XC = A d AX = C
10
e ABX = C f CX = C g XB = I h A−1BX = C f
01

13 A = 3 4 , B = 6 1 and X = x . 1
g ------ 1 – 5
–1 2 21 y 15 2 5

Solve these matrix equations. 1


h ------ – 132 – 114
2 15 186 162
a AX = b BX = 15 a 2 b 2
–4 7 –1 3 or
1 – 44 – 38
---
5 62 54
154 M a t h s Q u e s t M a t h s C Ye a r 1 1 f o r Q u e e n s l a n d

14 Find the value of x and y by solving these matrix equations.

a 34 x = –2 x = −2, y = 1 b 2 3 x = 8 x = 1, y = 2
–1 5 y 7 4 –1 y 2

c –4 2 x = 14 x = −2, y = 3 d –1 3 x = 5 x = 7, y = 4
3 –2 y – 12 2 –3 y 2

The transpose of a matrix


The transpose of matrix A is A′, where A = a b and A′ = a c . The transpose of
c d b d
a matrix is an interchange of rows and columns (row 1 becomes column 1 and so on).
Consider the following laws that apply to the transpose of matrices A and B:
1. (A¢)¢ = A
2. (A + B)¢ = A¢ + B¢
3. (kA)¢ = kA¢
4. (AB)¢ = B¢A¢
5. AA¢ is a symmetric matrix [that is, (AA¢)¢ = AA¢].
The proofs of these laws are given as problems in exercise 3E.

remember
remember
1. When required to prove a statement is true:
(a) do not assume it is true and use the statement in your proof
(b) work only one side of the statement at a time, not both together
(c) do not use actual constant values for the elements, use pronumerals only.
2. If you are asked to show a statement is true, you are expected to use actual
values as given.

3E The transpose of a matrix


Check with your teacher.

1 Prove that for any 2 × 2 matrix A, (A′)′ = A.

2 Show that for A = 1 3 and B = 0 – 1 , (A + B)′ = A′ + B′.


0 2 1 2

3 Show that for k = −2 and A = – 3 1 , (kA)′ = kA′.


0 1

4 Show that for A = 3 – 4 and B = 0 1 , (AB)′ = B′A′.


1 2 1 0
5 Show that for any 2 × 2 matrix, AA′ is symmetrical.
Chapter 3 Matrices 155
Applications of matrices
Application 1: Simultaneous equations
As we saw in questions 13 and 14 from exercise 3D, matrices may be used to solve
linear simultaneous equations. The pair of equations may be written in the form AX = B

where A is the matrix of the coefficients of x and y in the equations, X =x and B is


y
the matrix of the numbers on the right-hand side of the simultaneous equations.
A is called the coefficient matrix.
For example, the simultaneous equations:
ax + by = u
cx + dy = v
can be expressed as the matrix equation:

a b x = u
c d y v

which is of the form AX = B. Here a b is called the coefficient matrix, x the


c d y

variable matrix and u the constant matrix.


v
As we have seen, this equation can be solved by using:
A−1AX = A−1B
X = A−1B

WORKED Example 12
Solve 3x − y = 16 and 2x + 5y = 5 by matrix methods.

THINK WRITE

1 Write the simultaneous equations under 3x − y = 16


each other making sure the variables 2x + 5y = 5
are in corresponding positions.
2 Write the matrix equation. AX = B
3 –1 x = 16
2 5 y 5

3 Rearrange the equation in general form A–1AX = A–1B


so that X is the subject. IX = A–1B
X = A–1B
Continued over page
156 M a t h s Q u e s t M a t h s C Ye a r 1 1 f o r Q u e e n s l a n d

THINK WRITE

1 5 1
4 Calculate A−1. A–1 = ------
17 –2 3

1 5 1 16
5 Multiply A–1 by B. X = ------
17 –2 3 5
1 85
= ------
17 – 17

x = 5
y –1

6 Write the answers in the form x = . . . ∴ x = 5 and y = −1


and y = . . .
Note: The solution should be verified
by substituting x = 5 and y = −1 into the
original equations.

Graphics Calculator tip! Solving matrix equations


Most graphics calculators provide a facility for calculating inverses of matrices. To
solve the equations in Worked example 12, follow these steps.

For the Casio fx-9860G AU


For operations on matrices, press MENU to display the MAIN MENU. Use the arrow
keys to highlight RUN-MAT. Select it by pressing EXE .

1. Set up the dimensions for matrix A.


(a) Press F1 (

MAT) to enter the matrix editing


screen.
(b) Highlight Mat A and press EXE or F3 (DIM).
(c) Specify the number of rows, 2 in this case, and
then press EXE .
(d) Specify the number of columns, 2 in this case,
and then press EXE .

2. Press EXE again to display the 2 × 2 array for


matrix A.

3. Enter the values for the elements of matrix A,


pressing EXE after each number.
Chapter 3 Matrices 157
4. Exit the Matrix input screen by pressing EXIT .

5. Repeat steps 1 to 4 to create matrix B.

6. Press EXIT again to return to the MAT screen.

7. Pre-multiply matrix B by the inverse of matrix A.


(a) Press OPTN then F2 (MAT) to bring up the
matrix menu.
(b) Press F1 (Mat) then ALPHA [A] and then
SHIFT [x–1] to specify the matrix A–1.
Press F1 (Mat) then ALPHA [B] to specify
matrix B.

8. Press EXE to obtain the answer screen. (Press


EXIT to leave.)

For the TI-Nspire CAS


1. Open a new Calculator document (press / N
and select 1: Add Calculator). Press k to access
the catalog. Select Option 5 (by pressing 5) then
highlight the 2-by-2 matrix symbol.

2. Press ·. Use the arrow keys to move from one


element to the next to fill in the 2 × 2 matrix.

3. Use the arrow keys to move the cursor outside the


matrix, to the right. Press the power key (l) and
type in the index (–1).
158 M a t h s Q u e s t M a t h s C Ye a r 1 1 f o r Q u e e n s l a n d

4. Press the right arrow to bring the cursor to the base


then press the multiplication key (r). The
multiplication symbol appears as a dot on the
screen.

5. Press k to access the catalog. Select Option 5


and highlight the 2-by-1 matrix symbol.

6. Press ·. Fill in the values for the 2 × 1 matrix.

7. Move the cursor outside the matrix, then press ·


to obtain the answer.

Application 2: Summarising information


We have already seen how matrices may be used to summarise information such as
town–road connections. Information which can be summarised in tabular form may
also be presented as a matrix.

WORKED Example 13
In a large country town, there are three major supermarkets. Customers switch from one
to another due to advertising, better service, prices and for other reasons. A survey of 1000
customers has revealed the following information for the past month.
Best Buys started with 40% of the market; 90% of its customers remained loyal to Best
Buys but 5% changed to Great Groceries and 5% to Super Store.
Great Groceries started with a 36% market share; 85% remained loyal, 10%
transferred to Best Buys and 5% to Super Store.
Super Store started with 24% of the customers; it lost 15% to Best Buys and 5% to
great Groceries, but 80% remained.
Summarise the information in matrix form and calculate the new market shares.
Chapter 3 Matrices 159
THINK WRITE
1 The information may be
summarised in a 3 × 3 matrix with Retention rates and losses
the rows representing retention (%)
rates and gains and the columns
representing retention rates and Best Great Super
losses. This may be called a Buys Groceries Store
transition matrix.

Retention rates and gains


Row 1 shows that Best Buys Best 90 10 15
retains 90% of its customers, Buys
gains 10% of Great Groceries’

(%)
customers and gains 15% of
Super Store’s customers. Great 5 85 5
Column 1 indicates that Best Groceries
Buys retains 90% of its
customers, loses 5% to Great Super 5 5 80
Groceries and loses 5% to Super Store
Store. Note that each column
totals 100%.
2 Write the initial market shares as
0.40
a 3 × 1 matrix. This information is
The initial market share matrix is 0.36
found as the market share at the
beginning of the month. 0.24
Note: The values total 1.
3 The new market share will be the
transition matrix, converted to 0.90 0.10 0.15 0.40 0.432
decimal numbers, multiplied by 0.05 0.85 0.05 0.36 = 0.338
the market share matrix. 0.05 0.05 0.80 0.24 0.230

4 Express the new market shares as The new market shares are Best Buys 43.2%, Great
percentages. Check the values add Groceries 33.8% and Super Store 23.0%.
up to 100%.

remember
remember
1. Matrices may be used to solve simultaneous equations:
ax + by = u
cx + dy = v
The pair of equations may be written in the form AX = B,

where A = a b , X = x and B = u .
c d y v
2. Matrices can also be used to summarise information which is in table form
and solve related problems; however, care must be taken in setting up the
matrices.
160 M a t h s Q u e s t M a t h s C Ye a r 1 1 f o r Q u e e n s l a n d

3F Applications of matrices

In the following exercise solve all problems manually then use a graphics calculator wher-
ever appropriate to check your solutions.
WORKED 1 Solve these simultaneous equations by matrix methods.
Example
12 a 2x − 3y = 13 and x + 2y = 3 (5, −1) b 3x + y = 9 and −2x + 5y = −6 (3, 0)
c −x + 4y = −2 and x − 5y = 0 (10, 2) d 6x + 7y = 0 and 4x − 3y = 0 (0, 0)
e 4x + y = 20 and x − y = 0 (4, 4) f 3x − 2y = 0 and x − y = 1 (−2, −3)
2 Consider these two pairs of simultaneous equations:
eBook plus
i 3x − 2y = 4 ii 3x − 2y = 6
Digital doc: 6x − 4y = 12 6x − 4y = 12
a and b Answers will vary.
SkillSHEET 3.3
Using matrices to a Show by algebraic means that the simultaneous equations in i have no solution.
solve linear equations
b Show that the simultaneous equations in ii have an infinite number of solutions.
d i y
c Write the equations in matrix form and explain how these facts are related to the
determinant of the matrix of the coefficients. det = 0
0 4– x d Draw, on two sets of axes, graphs of the two lines in each of i and ii.
3
2 3
–1 e Explain how the graphs are related to parts a and b. In i there are parallel lines;
in ii there is only one line.
–2
–3 3 multiple choice
Consider the simultaneous equations: 3x − 2y = 5
ii y Both lines y + 2x = 8
0 2 x
a The coefficient matrix is:

A 3 –2 B 5 C 32 D 31 E 3 –2
–3
1 2 8 –2 1 –2 2 2 1

b The solution to the simultaneous equations is:


A x = 2, y = 3 B x = 3, y = 2 C x= 13
------ ,y= 19
------
4 8
D x = −2, y = 2 E x = 4--- , y = 7---
3 3

4 multiple choice
In an alternative Australian Rules Football game, a team gains x points for a goal and y
points for a behind. In one game Cairns obtained 66 points by scoring 10 goals and 8
behinds and Townsville obtained 70 points from 12 goals and 5 behinds.
a This information is represented by which of the following matrix equations?

A 8 10 x = 66 B 10 12 x = 66 C 10 8 x = 66
5 12 y 70 8 5 y 70 12 5 y 70

D 8 5 x = 66 E 12 10 x = 70
10 12 y 70 5 8 y 66
b The value of x − y is:
A 5 B 4 C 6 D 3 E 2
Chapter 3 Matrices 161
5 The sum of two numbers is 20 and their difference is 12. Find the numbers by setting
up simultaneous equations and solving by matrix methods. 16, 4
6 In a factory, two types of components are processed on two separate machines. The
respective processing times on the first machine are 18 minutes and 21 minutes, while
for the second machine the times are 4 minutes and 42 minutes. How many of each
type of component, per machine, should be processed in an 8-hour shift so that both
machines are fully occupied and the output of each machine is the same? 15, 10
WORKED 7 In a swimming competition, 5 points are awarded for first place, 3 for second, 2 for
Example
13
third and 1 point for an unplaced result. The top competitors’ results were:

Number of races First Second Third


Name competed in placings placings placings

Rania 6 2 — 2

Patricia 4 4 — —

Anh 5 3 2 —

Mayssa 6 1 3 2

Rachel 6 2 3 —

Place the results and points in suitable matrices and use matrix multiplication to find
the highest points scorer. Anh
8 Cyril’s circus arrived in town last week and
during the week the number of adults, children
and pensioners attending the circus was
recorded for the first five shows (see table
below).

Adults Children Pensioners

Monday 400 200 20

Tuesday 450 350 50

Wednesday 370 410 45

Thursday 290 380 70

Friday 420 530 65

The entry cost is $20 for adults, $6 for children


eBook plus
and $5 for pensioners.
Digital doc: Set up the information in suitable matrices to
WorkSHEET 3.2
find the total takings for the first five shows. $51 070
162 M a t h s Q u e s t M a t h s C Ye a r 1 1 f o r Q u e e n s l a n d

Matrix multiplication
using a graphics calculator
Worked example 13 may be solved using a graphics calculator as follows.

For the Casio fx-9860G AU


1. Enter the 3 × 3 transition matrix as matrix A.
(For more details, see the previous graphics
calculator tip on page 156.)
(a) Press F1 ( MAT) to enter the matrix


editing screen.
(b) Set the dimensions of A to 3 × 3 and press
EXE .
(c) Enter the values of A.
2. Press EXIT .
3. Enter the 3 × 1 market share matrix as matrix B.
(a) Scroll down to Mat B and press EXE .
(b) Set the dimensions to 3 × 1 and press
EXE .
(c) Enter the values of B.
4. Exit the Matrix input screen by pressing EXIT .
Press EXIT again to return to the MAT screen.
5. Multiply the matrices A and B (and store as
matrix C).
(a) Press OPTN then F2 (MAT) to bring up
the matrix menu.
(b) Press F1 (Mat) then ALPHA [A] to specify
matrix A. Press F1 (Mat) then ALPHA [B]
to specify matrix B. Press EXE to obtain the
answer screen. Alternatively, press SHIFT
[{], then Æ then F1 (Mat) and ALPHA [C]
SHIFT [}] to store as matrix C.
6. Press EXE to obtain the answer screen. (Press
EXIT to leave.)

For the TI-Nspire CAS


1. Open a new Calculator document (press / N
and select 1: Add Calculator). Press k to
access the catalog. Select Option 5 then
highlight the m-by-n matrix symbol.
Chapter 3 Matrices 163
2. Press ·. Create the matrix with Number of
rows: 3 and Number of columns: 3, pressing
e to move from one box to the next. Press e
until OK is hightlighted.

3. Press · to display the matrix template. Fill in


the values in the matrix using the arrow keys or
the tab key to move from one element to the
next.

4. Move the cursor to the right of the matrix and


press the multiplication (r) key. Press k to
access the catalog and highlight the m-by-n
matrix symbol.

5. Press · then select Number of rows: 3 and


Number of columns: 1.

6. Highlight OK and press ·. Fill in the values in


the 3 × 1 matrix.

7. Move the cursor outside the matrix and press


· to obtain the answer.
164 M a t h s Q u e s t M a t h s C Ye a r 1 1 f o r Q u e e n s l a n d

Questions
Use a graphics calculator to find A × B for each of the following:
133 9 30
1 A= 5 0 2, B= 2 AB = 55
628 5 98

2.5 6.1 40.36 36.99 36.92


2 A = 9.2 0.3 , B = 3.7 0.4 9.4 AB = 35.57 5.45 87.14
6.6 0.7 5.1 5.9 2.2 27.99 6.77 63.58
3.7 4.6 37.15 28.62 44.9

2 3 0 –2 7 –3 8 74 67
0 1 –1 4 –7 5 11 – 66 –4
3 A= 5 –3 9 –2 6, B= 4 –7 AB = 62 – 40
–5 1 1 9 6 –1 –2 69 – 42
56 48
4 4 4 4 4 9 2

SLE 10: Investigate the use of


matrices in dominance problems Dominance
such as in predicting the next
round results (rankings) for the
national netball competition.
matrices
Have you ever wondered how
tennis players are seeded or
ranked? It obviously has
something to do with their
performance against past
opponents. In a knock-out
competition, one loss and you
are out of the competition.
Only the winners continue to
play. Dominance matrices are
often used to determine player
rankings.
The following investigation
will explain how matrices are
used to establish the seedings
or rankings of players in
round-robin situations where
each player plays every other
player, thereby creating a
more just system of ranking.
Chapter 3 Matrices 165

Dominance matrices — another


application of matrices
Consider 4 players Alan, Brian, Carlo and Denis (A, B, C, D), who on past
performances have shown that A defeats D and B, D defeats B, C defeats A and D,
and B defeats C.
This situation can be represented on a digraph — a network diagram that has
arrows on the edges, where A → B indicates that A defeats B.
A

B C

The information from the digraph can be converted into matrix form (a dominance
matrix) as below:
defeats A B C D
A 0 1 0 1
M= B 0 0 1 0
C 1 0 0 1
D 0 1 0 0
where 1s are used to indicate ‘defeats’ and 0s to indicate otherwise. Obviously A
can’t defeat A so a ‘0’ is used along the leading diagonal.
Notice also that:
(1) there are as many 1s as there are paths
(2) corresponding elements occur on either side of the leading diagonal. That is, if
A defeats B (1), then a 0 will be stored in the B defeats A element on the
opposite side of the leading diagonal.
When the elements of each of the rows are added they yield a dominance vector,
showing how many players each has defeated.

A 2
V= B 1
C 2
D 1
This result can be readily checked from the original digraph by counting the
number of arrows out of each node.
Note, from now on the row/column labels will be omitted.
It can be seen from this information that A and C are ranked equally, and B and D
are ranked equally; this can be written as

A   B 
   
 C  D 
166 M a t h s Q u e s t M a t h s C Ye a r 1 1 f o r Q u e e n s l a n d

So we still need to distinguish between A and C, and B and D to establish the


ranking.
We assume in most ranking situations that if A defeats B and B defeats C then A
will defeat C. This relationship is described as being transitive, where if variable
a < b and b < c, then a < c.
In our example, A defeats B who defeats C, and A defeats D who defeats B. This
means that A has second-order influence over C and B but not D. (A doesn’t defeat
anyone who defeats D). The matrix M2 can be used to investigate second-order
influence.

0 1 0 1 0 1 0 1
M2 = 0 0 1 0 0 0 1 0
1 0 0 1 1 0 0 1
0 1 0 0 0 1 0 0

0 1 1 0
M2 = 1 0 0 1
0 2 0 1
0 0 1 0

Notice that the leading diagonal is still 0. It is impossible for a player to have
second-order influence over themselves. Row 1 represents the second-order
influence of player A over the other players. The element 2 in row 3 occurs because
C defeats 2 players (A and D) who defeat B.
2
We can find the second-order dominance vector, V2 = 2 , but how much
importance should it be given? 3
1
If we assign equal importance, we calculate

0 1 0 1 01 1 0
M + M2 = 0 0 1 0 + 10 0 1
1 0 0 1 02 0 1
0 1 0 0 00 1 0

0 2 1 1
= 1 0 1 1
1 2 0 2
0 1 1 0

4
This gives a dominance vector 3 = V1 + V2 and allows us to rank the competitors
in the order C, A, B, D. 5
2
Chapter 3 Matrices 167
If we wanted to investigate third-order influence, we could calculate M 3 (= M 2M).
0 1 1 0 0 1 0 1 10 1 1 3
M M= 1
2 0 0 1 0 0 1 0 = 02 0 1 giving V3 = 3
0 2 0 1 1 0 0 1 01 2 0 3
0 0 1 0 0 1 0 0 10 0 1 2
At this stage, notice that the leading diagonal is no longer 0. If there were
more players in the tournament, we could continue finding powers of M, but with
4 players, we stop at M 3. In general, if there are m players, we stop at M m – 1.
In most scenarios, it is probably unfair to assign equal importance to first-,
second- and third-order influence. We can allocate arbitrary constants to weight the
influence; that is, M + xM 2 + yM 3. The resulting dominance vector can be found by
calculating V1 + xV 2 + yV3.
If we choose x = 0.5 and y = 0.25, the dominance vector would be
2 2 3 3.75
V1 + 0.5V2 + 0.25V3 = 1 + 0.5 2 + 0.25 3 = 2.75
2 3 3 4.25
1 1 2 2

This would rank the four players as C, A, B, D.


When you compare the final seeding with the initial information, we can see that
A and C both won 2 games, but the wins by A were against the lower ranked B and
D. Players B and D both won 1 game but B managed to defeat the higher placed C.
This justifies the seeding as produced.

Alternative method for adding


Graphics Calculator tip! the elements in each row in a
dominance matrix
Multiplying a square matrix by a column vector with the same number of rows and all
entries shown as 1 has the effect of adding the elements in each row of the matrix. In
the example above, the dominance vector V1 could have been obtained using the
following steps. (The main advantage is if the dominance matrix is 5 × 5 or larger. You
don’t need to arrow across the screen to see the elements when you are adding them.)

For the Casio fx-9860G AU


1. Enter the 4 × 4 dominance matrix M. (Refer to the
graphics calculator tip on page 156 if you are unsure
how to do this.)

2. Enter the 4 × 1 vector N with 1 shown for every


element.
168 M a t h s Q u e s t M a t h s C Ye a r 1 1 f o r Q u e e n s l a n d

3. Evaluate M × N. Notice that this gives V1.

For the TI-Nspire CAS


1. Enter the 4 × 4 dominance matrix M. (Refer to the
graphics calculator tip on page 157 if you are unsure
how to do this.)

2. Move the cursor to the right of the matrix. Press the


multiplication (r) key.

3. Enter the 4 × 1 vector N with 1 shown for every


element.

4. Move the cursor outside the matrix and press · to


obtain the answer to M × N. Notice that this gives V1.

remember
remember
1. Dominance matrices are often used to determine player rankings in round-robin
situations.
2. Information from a digraph that indicates the win–loss outcome of matches
played in a tournament (for example, A defeats B, D defeats C, and so on) can
be converted into matrix form. This matrix is called a dominance matrix.
3. A dominance vector shows how many players each has defeated. It is obtained
by adding the elements of each of the rows of the dominance matrix. This
allows you to rank the players.
4. For a dominance matrix, M, we can calculate the second-order influence of
players by calculating M 2 (third-order influence by calculating M 3 and so on)
and finding the resulting dominance vector, V2 (V3 and so on).
5. Arbitrary constants can also be allocated to weight the influence; for example,
when considering four players, we calculate M + xM 2 + yM 3 where x and y
are constants. The resulting dominance vector can be found by calculating
V 1 + xV 2 + yV 3. This refines the ranking process.
Chapter 3 Matrices 169

3G Dominance matrices
1 We want to seed 4 chess players, Breanna, Kayley, Teagan and Cameron. In past
matches, Cameron defeated Breanna and Teagan, both Breanna and Teagan defeated
Kayley, Kayley defeated Cameron, and Breanna defeated Teagan.

C
T K a Draw a digraph to represent this information.
B b By giving equal importance to first- and second-order influence, use dominance
matrices to rank the players. Cameron, Breanna, Kayley, Teagan
2 Three friends have noticed that when they played chess, Mair defeated Ann and Janine,
and Ann defeated Janine. Use dominance matrices to rank these players. Mair, Ann, Janine
3 A round-robin netball match was arranged for house competitions where Barnes lost to
all but Cunningham, Cunningham lost to Leslie but defeated Hamilton. No teams went
undefeated.
a If it is decided to give equal importance to first- and second-order influence, use
dominance matrices to rank the students’ houses. Hamilton, Leslie, Cunningham, Barnes
b If house points are allocated as 20, 15, 10, 5 for the overall ranking, how many
points did each house receive? 20 points to Hamilton, 15 to Leslie, 10 to Cunningham, 5 to Barnes
4 Five schools are debating in a round-robin tournament — the following table shows the
results.

Clifton Warwick Goondiwindi Stanthorpe Ipswich


Clifton — L W W L
Warwick — W W L
Goondiwindi — L W
Stanthorpe — W

0 0 1 1 0 Clifton lose against Warwick and Ipswich and so on.


1 0 1 1 0 a Construct a dominance matrix of this information.
M= 0 0 0 0 1 b A total of 15 points is divided in the ratio 5:4:3:2:1 and awarded according to the
0 0 1 0 1 ranking of the schools at the end of the tournament. If this division of points is
1 1 0 0 0 allocated according to the figures produced by the dominance matrix sum
M + 0.8M 2 + 0.5M 3, list the number of points each school wins.
5 points to Warwick, 4 points to Ipswich, 3 points to Stanthorpe,
2 points to Clifton, 1 point to Goondiwindi
170 M a t h s Q u e s t M a t h s C Ye a r 1 1 f o r Q u e e n s l a n d

summary
Operations with matrices
• A matrix (plural: matrices) is a collection of numbers arranged in rows and
columns.
• An m × n matrix has m rows and n columns.
• The numbers in the matrix are called the elements of the matrix. Elements are
referred to by the row and column position.
• Addition and subtraction of matrices is performed by adding or subtracting
elements in corresponding positions. These operations can be performed only if the
matrices have the same order.
• Scalar multiplication of a matrix is performed by multiplying each element of the
matrix by a number. Thus kA means each element in matrix A is multiplied by the
number k.

Multiplying matrices
• Matrices are multiplied in the following way:

a 11 a 12 a 13 b 11 b 12 b 13
If A = a 21 a 22 a 23 and B = b 21 b 22 b 23
a 31 a 32 a 33 b 31 b 32 b 33

then
a 11 × b 11 + a 12 × b 21 + a 13 × b 31 a 11 × b 12 + a 12 × b 22 + a 13 × b 32 a 11 × b 13 + a 12 × b 23 + a 13 × b 33
AB = a 21 × b 11 + a 22 × b 21 + a 23 × b 31 a 21 × b 12 + a 22 × b 22 + a 23 × b 32 a 21 × b 13 + a 22 × b 23 + a 23 × b 33
a 31 × b 11 + a 32 × b 21 + a 33 × b 31 a 31 × b 12 + a 32 × b 22 + a 33 × b 32 a 31 × b 13 + a 32 × b 23 + a 33 × b 33

The orders are related as follows:


(m × n) (n × p) = (m × p).
• Matrix multiplication is usually not commutative. That is, AB ≠ BA.

Powers of a matrix
• The power, n, of matrix A, in general form, is An = An – 1A, where n is a positive
integer.
• Powers of matrices are only defined for square matrices; that is, A has to be a
square matrix to obtain An.
• A matrix A is nilpotent if A2 = O where O is the zero matrix. The zero matrix is a
square matrix with all elements equal to zero. For example, the 2 × 2 zero matrix is
00 .
00
• A matrix A is idempotent if A2 = A.
Chapter 3 Matrices 171
Multiplicative inverse and solving matrix equations
• An identity matrix, I, is defined for square matrices such that AI = IA.
• The multiplicative inverse of matrix A is A–1 such that AA−1 = A−1A = I.

1
If A = a b , then A –1 = ------------------ d – b .
c d ad – bc – c a

The number ad − bc is called the determinant of A and has the symbols det A or |A|.
If det A = 0, then A−1 does not exist (A does not have an inverse) and A is said to be
singular.

The transpose of a matrix


• The transpose of a matrix is an interchange of rows and columns.

• The transpose of a matrix A is A′ where A = a b and A′ = a c .


c d b d

Applications of matrices: solving simultaneous equations and


summarising information
• Matrices may be used to solve simultaneous equations:
ax + by = u
cx + dy = v.

The pair of equations may be written in the form AX = B, where A = a b ,


c d
X= x and B = u .
y v
• Matrices can also be used to summarise information which is in table form and
solve related problems but care must be taken in setting up the matrices.
• Dominance matrices can be used to determine player rankings in sports
competitions.
172 M a t h s Q u e s t M a t h s C Ye a r 1 1 f o r Q u e e n s l a n d

CHAPTER
review
1 multiple choice
3A
The solution to 2 0 – 2 A = 4 2 is given by A equals:
–2 0 02

31 B 12 4 –4 0 –1 –1 11
A C D E
–1 1 –4 4 –4 –8 –1 –1 11

2 A is a 3 × 2 matrix, B is 2 × 2 and C is 3 × 2. Which of the following may be


3A calculated?
a A+B b A+C c B+C

3 multiple choice
3B
If A is a 3 × 2 matrix and B is 2 × 1, then the order of AB is:
A 2×2 B 3×2 C 3×1 D 1×3 E 2×1

4 multiple choice
3B
1
The product of 1 2 3 0 is:
4 5 6
1

1 0 3
A B 4 C 1 0 3 D E
4 10 14 0 0 0
10 4 0 6
4 0 6

5 Using the same matrices as in question 2, which of the following may be calculated?
3B a AB b AC c BA
d BC e CA f CB
g A + CB h A + BC i AB−1

6 If A = 1 2 , find
3C –2 0
–3 2 –7 –6 5 – 14
a A2 b A3 c A4
–2 –4 6 –4 14 12

1 0 0
7 If A = 0 2 1 , find
3C
1 0 0
a A2 100 b A3 100 c A4 1 00
142 384 7 16 8
100 100 1 00
Chapter 3 Matrices 173
8 multiple choice
3D
Consider the following matrices.

A= 10 B= 6 3 C= 3 2
01 – 12 – 6 3 –2
a Which of the following are idempotent?
A A B Β C Α and Β D C E A and C
b Which of the following are nilpotent?
A Α B Β C Α and Β D C E A and C

9 multiple choice
3D
The determinant of – 2 0 is:
1 5
A 0 B −11 C −2 D −10 E 10

10 multiple choice
3D
If AB = 4I then B−1 is:
1 1
A 4A B A C --- A D --- B E 4B
4 4

11 multiple choice
3D
Which of the following matrices is singular?

1 0 B 1 0 C 4 –2 D 4 2 E 10 2
A
0 1 0 –1 –6 3 –2 1 5 0

12 multiple choice
3D
If AX = B then X is given by:
B
A A−1B B BA−1 C --- D AB−1 E IA−1
A

13 Find matrix A if A 0 1 = 6 6 . 93
2 –1 0 –6 –6 0 3D
14 multiple choice
3E
Using matrices, the solution to: 4x − y = 7
y−x=2 is:
A (2, 3) B (3, 5) C (−3, 4) D (1, 1) E (5, 3)

15 a Write down the inverse of 4 – 2 1


------ 1 2
3 1 10 – 3 4 3D,E
b Hence solve 3x + y = 14 and 4x − 2y = 22. (5, −1)
174 M a t h s Q u e s t M a t h s C Ye a r 1 1 f o r Q u e e n s l a n d

16 In a township, 25% of households own no pets,


3F 40% of households own one pet, 20% have two
pets and 15% own more than two pets.
a Set up a 1 × 4 matrix to represent the
percentage ownership of pets. [0.25 0.40 0.20 0.15]
b Write an equation that will enable you to
calculate the number of households for each
category, given that there are 800 households
in the town. A = 800B
c Evaluate the number of households for each
category as a 1 × 4 matrix. [200 320 160 120]

17 The matrix below represents the prices (in dollars)


3F of some mobile phone options. The first column
displays the costs of two types of pre-paid mobile
phones and the second column represents two types
of 12-month-plan mobile phones.
249 29
680 49
The company wants to increase the price of
the pre-paid mobile phones by 12% and
decrease the cost of the 12-month-plan
mobile phones by 5%.
a Show the matrix used to represent the 1.12 ˙ 0
price changes (a 2 × 2 matrix). 0 0.95
b Use matrix multiplication to calculate
the new prices. 278.88 27.55
761.60 46.55
18 In a backgammon competition, four players —
3G Glen, James, Cameron and William —
competed with the following results:
Glen’s only win was against James.
James defeated both Cameron and
William. Cameron also defeated
William. James, Cameron, Glen, William
Using dominance matrices and
assigning a weighting of 1
to first-order influence
and 0.5 to second-order
influence, rank the
players.
Chapter 3 Matrices 175
Modelling and problem solving
1 A company has two plants manufacturing components for different models of car. The time
spent in hours per car is given in the following matrix.
Assembly Packaging Despatch
Standard model 25 1 0.5
Deluxe model 30 1.5 1
4-wheel drive 35 1.5 0.5

The wage rates ($ per hour) at the two sites are given by:
Plant 1 Plant 2
Assembly 16.50 16.00
14: Packaging at Plant 1
Packaging 14.00 14.00 has a wage rate of
1: Despatch for Deluxe
model takes 1 hour. Despatch 13.50 13.00 $14 per hour.
a In the first matrix, write down the 2, 3 element and explain what it refers to.
b In the second matrix, write down the 2, 1 element and explain what it refers to.
c Write down the order of each matrix and the order of the matrix found by multiplying the
first matrix by the second matrix. 3 × 3, 3 × 2, 3 × 2 433.25 420.50
d Find the product of the two matrices. 529.50 514.00
e Explain what the first row of the product matrix represents. 605.25 587.50
f Explain what the first column of the product matrix represents. The assembly costs for each
model at Plant 1
g Write down the cost of producing the Deluxe model at:
ii Plant 1 $529.50
ii Plant 2. $514.00

The total costs for the Standard


model at Plants 1 and 2
176 M a t h s Q u e s t M a t h s C Ye a r 1 1 f o r Q u e e n s l a n d

2 Tickets for a one-way trip on a Brisbane-to-Sydney passenger train can be purchased as either
Adult, Child (under 15 years old) or Pensioner. The table below shows the number of
passengers and the total takings for three trips.

Number of
Number of adult Number of child pensioner
passengers passengers passengers Total takings ($)
145 103 121 20 260
130 110 90 18 400
142 115 80 19 200

a Let x = the cost of an adult’s ticket. 145x + 103y + 121z = 20 260 145 103 121 x 20 260
Let y = the cost of a child’s ticket. 130x + 110y + 90z = 18 400 130 110 90 y = 18 400
Let z = the cost of a pensioner’s ticket. 142x + 115y + 80z = 19 200 142 115 80 z 19 200
Construct three equations in terms of x, y and z.
0.025 544 –0.093 523 0.066 579
b Using matrices, express the equations in the form AX = B.
A−1 = – 0.039 222 0.091 991 –0.044 166
c Use your graphics calculator to find A–1. 0.011 042 0.033 767 – 0.042 189
d Use your graphics calculator to determine the costs of a train ticket for an adult, a child
and a pensioner. The cost of an adult’s ticket is $75, a child’s ticket is $50 and a pensioner’s ticket is $35.
a b a b
3 Use a and b to complete A where A = so that it is nilpotent (that is, A2 = O). A = a
2
– ----- – a
b

a b a b
4 Use a and b to complete A where A = so that it is idempotent. A = a – a 2
-------------- 1 – a
b

5 Prove that if a square idempotent matrix A is non-singular, then A must be the identity matrix.

6 Prove that if A is idempotent


eBook plus a I – A is idempotent
b A(I – A) = O.
Digital doc:
Test Yourself
Chapter 3
4
An introduction
to groups

syllabus reference
Core topic:
Introduction to groups

In this chapter
4A Modulo arithmetic
4B The terminology of groups
4C Properties of groups
4D Cyclic groups and
subgroups
4E Further examples of groups
— transformations
178 M a t h s Q u e s t M a t h s C Ye a r 1 1 f o r Q u e e n s l a n d

Introduction
Concepts of:
• closure Through your study of mathematics, you have developed an understanding of the rules
• associativity
• identity that apply to numbers. You know that if you add two integers, the result is also an
• inverse
• definition of a group integer. However, if you divide two integers, you don’t always get an integer as the
answer. You know that addition is associative; for example, 7 + (6 + 2) = (7 + 6) + 2;
but subtraction is not; for example, 7 − (6 − 2) ≠ (7 − 6) − 2.
Towards the end of the 19th century, mathematicians began to talk about the concept
of groups. Essentially, a group is a set of elements, such as integers or matrices, that
can be combined using an operation, like addition or multiplication, and which satisfy
certain conditions. For example, integers form a group under addition but not under
division (because dividing integers does not always result in an integer).
Historically, group theory came from the study of number theory and the theory of
algebraic equations at the end of the 18th century and the study of geometry at the
beginning of the 19th century. Today, group theory is applied to many areas of science
such as genetics, quantum theory, molecular orbits, crystallography and the theory of
relativity.

Algebraic structures
In algebra, symbols that can be manipulated are elements of some set and the
manipulation is done by performing certain operations on elements of that set. The
set involved is referred to as an algebraic structure.
Research the topic of algebraic structures examining early algebraic systems that
developed in ancient civilisations such as the Indian, Arabic, Babylonian, Egyptian
and Greek. Highlight differences and similarities among the various forms.

But first a new tool to help you deal with some notions used in groups.
Chapter 4 An introduction to groups 179
Modulo arithmetic
Not to be confused with the modulus of a number (see Chapter 1 on real numbers, R,
where the modulus of −4, written | −4 | = 4), modulo arithmetic uses a finite number
system with a finite number of elements. This is sometimes referred to as ‘clock
arithmetic’ because of the similarities with reading the time on an analog clock.
Consider reading the time shown on the clock face to the right.
Whether it is 2 am or 2 pm we would say it is 2 o’clock,
but in 24-hour time the 2 pm would be 1400 hours. In effect 11 12 1
we have subtracted 12 hours from the 1400 (14 hours) to give 10 2
an answer of 2. In this case we say that 2 is the residue, or 9 3
what is left over when 12 hours is subtracted from the 14. 8 4
In modulo 12 arithmetic the same principle is used except 7 6 5
that the 12 is replaced by a 0.
5 + 6 ≡ 11
5+7≡0
5 + 8 ≡ 1 and so on. 11 0 1
In our normal decimal system 5 + 8 = 13, but in modulo 12 10 2
arithmetic the residue of 1 differs from 13 by 12 (or a mul- 9 3
tiple of 12) and 1 and 13 are said to be congruent. That is, in 8 4
modulo 5 arithmetic, the numbers 3, 8 and 13 are congruent 7 6 5
and in modulo 12 arithmetic, 2, 14, and 26 are congruent
numbers. The symbol for congruency, ≡, is used.
Using more precise terminology, addition modulo 10 is written
3 + 9 ≡ 2 mod 10, 5 + 5 ≡ 0 mod 10, and so on.
(Note the abbreviation of modulo to mod.)
In mod 12, the numbers 0 to 11 are referred to as residues, as with 0 to 5 in mod 6.
This information can be stored in a table, known as a Cayley Table.

WORKED Example 1
Draw up a Cayley Table that shows the residues using addition modulo 4.
THINK WRITE/DRAW

1 Draw an empty table with 0, 1, 2, 3 in + 0 1 2 3


the first row and column and put a +
sign in the top corner. 0
1
2
3

2 Start working across the first row. + 0 1 2 3


0 + 0 = 0 etc. and do likewise with the
first column. 0 0 1 2 3
1 1
2 2
3 3
Continued over page
180 M a t h s Q u e s t M a t h s C Ye a r 1 1 f o r Q u e e n s l a n d

THINK WRITE/DRAW

3 The residues are the numbers left over + 0 1 2 3


when 4 is taken from the answer (if the
answer is 4 or greater). As you 0 0 1 2 3
complete the table note that the answers 1 1 2 3 0
are less than 4. So, for 2 + 2 the residue 2 2 3 0 1
is 0. 3 3 0 1 2

remember
remember
1. Modulo arithmetic is like clock arithmetic where 5 + 9 ≡ 4 in mod 10.
2. The residues of modulo x are all the whole numbers less than x.
3. Congruent numbers in mod x all differ by multiples of x.

4A Modulo arithmetic
1 List 4 numbers congruent to:
eBook plus a 4 in mod 8 4, 12, 20, 28, 36 …
b 4 in mod 6. 4, 10, 16, 22
Digital doc:
SkillSHEET 4.1 2 List the residues in:
Modulo arithmetic
a mod 3 0, 1, 2 b mod 9 0, 1, 2, … 8 c mod 11. 0, 1, … 10
WORKED 3 Draw up a Cayley Table that shows the residues for each of the following:
Example
xample
1
a addition mod 6
b multiplication mod 4
c multiplication mod 5.

The terminology of groups


4
0
4
3
2
1

In Chapter 1 you dealt with different sets of numbers within the Real Number System.
3
0
3
1
4
2

Throughout your student life you have used the operations of addition, multiplication,
2
0
2
4
1
3

subtraction and division, finding a square root, reciprocals, and so on. These are
1
0
1
2
3
4

examples of operations performed on numbers that are part of a certain set.


0
0
0
0
0
0

Operations (such as addition) that involve 2 input values, for example 2 + 3, are called
×
0
1
2
3
4
c

binary operations. Those that involve only one input value, such as finding the square
3
0
3
2
1

root of a number (for example 8 ) are called unary operations. Others that involve 3
2
0
2
0
2

input values are called ternary; for example, the principal, interest and term of a loan
1
0
1
2
3

are the 3 input values involved in calculating the amount of interest due on a loan.
0
0
0
0
0

(Strictly speaking the multiplication involved is still carried out on pairs of values.)
×
0
1
2
3
b
5
5
0
1
2
3
4

Definition of terms
4
4
5
0
1
2
3

Groups that we will deal with consist of a system that involves a set of elements (often
3
3
4
5
0
1
2

numbers) and a binary operation. Lower case letters, a, b, c …, are used to refer to
2
2
3
4
5
0
1

elements of the set and the symbol ‘°’ denotes whatever operation is involved.
1
1
2
3
4
5
0
+ 0
0 0
1 1
2 2
3 3
4 4
5 5
3 a
Chapter 4 An introduction to groups 181
For a non-empty set of elements S = {a, b, c, …} involved in the binary operation ‘°’
to be a group, G = [S, °], the following properties must hold.
1 Closure
An operation is closed if the result of that operation is an element of the same set
as the two inputs. That is, a ° b must be in S.
For example, consider 2 + 3 = 5 where S = {Real numbers} (or R) and ° is the oper-
ation of addition. The operation is closed because 5 ∈ R.
But consider 2 − 3 = −1 where S = {Natural numbers} (or N) and ° is the operation
of subtraction. Because the result (−1) is not a member of the set of natural numbers
this operation is not closed. That is, the answer is not part of the initial set of natural
numbers.
2 Associativity
If an operation is associative, the order in which operations are performed does
not affect the answer. That is, (a ° b) ° c = a ° (b ° c).
Often brackets are employed to determine the order of operations.
For example, consider (2 × 3) × 4 and 2 × (3 × 4):
(2 × 3) × 4 = 6 × 4 2 × (3 × 4) = 2 × 12
= 24 = 24
In this case, both answers are the same. Note that only the position of the brackets
changes and the order of the numbers remains the same.
But consider the operation of division:
(20 ÷ 2) ÷ 4 and 20 ÷ (2 ÷ 4)
= 10 ÷ 4 = 20 ÷ 0.5
= 2.5 = 40
Here the answers are not the same.
Division, like subtraction, is not associative. You would have realised this in your
earlier junior mathematics studies.
3 Identity
For all elements of a set, if a unique element exists in the set such that
a ° u = u ° a = a then u is the identity element (IE) for that operation.
That means that there is only one element that leaves every element unchanged when
the operation ‘°’ has been applied.
For example, 3 + 0 = 3, so 0 is the identity element for addition (IE+) for real
numbers.
However, 3 × 0 = 0 so 0 is not the identity element for real numbers under the oper-
ation of multiplication.
Note: The one identity element must work for all elements of the set so 5 + 0 = 5
and −8 + 0 = −8. It must also work from both the left and right sides of the oper-
ation. For example, 6 + 0 = 6 and 0 + 6 = 6. That is, 6 + 0 = 0 + 6 = 6.
4 Inverse
For each element of a set there is a unique element a–1 such that
a ° a–1 = a–1 ° a = u where u is the identity element for that operation.
Unique means that every element has only one inverse.
2× 1
--- = 1
--- × 2 = 1 where 1 is the identity element for multiplication (IE×)
2 2
1
Therefore --- is the multiplicative inverse of 2.
2
182 M a t h s Q u e s t M a t h s C Ye a r 1 1 f o r Q u e e n s l a n d

Now consider 2 + −2 = −2 + 2 = 0 where 0 is IE+; in this case −2 is the additive


inverse of 2. However, note that the set involved here would have to be integers (that is,
both positive and negative) not just whole numbers because −2 ∉ {Whole numbers}.
We can now restate the definition of a group.
If the following 4 properties hold for a set of elements under a certain operation ‘°’:
1. closure
2. associativity
3. existence of an identity element
4. existence of an inverse
then the system under investigation [S, °] is a group. If a fifth property,
commutativity, also holds, then the group is an Abelian group.
Commutativity
If the order of the elements involved has no effect on the outcome, then the operation is
commutative. That is, a ° b = b ° a.
For example, 2 × 5 = 10 and 5 × 2 = 10.
Hence multiplication with real numbers is commutative. Note the stated condition,
‘with real numbers’, because you have already worked with matrices where multi-
plication is not commutative.
However, consider 10 ÷ 2 = 5 and 2 ÷ 10 = 0.2.
So division is not commutative. You would be familiar with other operations as well
that are not commutative.

WORKED Example 2
Find a the identity element and b the inverse for the operation defined as a ° b = a + b + 2.
THINK WRITE
a 1 An identity element (IE) is an a Let a ° b = a (where b = IE)
element that, when involved in an therefore a + b + 2 = a
operation with another element, does b = −2
not change the value of that element. so IE = −2
2 State the identity element. The identity element is −2.

b 1 An inverse is an element that, when b Let a ° b = −2 (where b is the inverse of a


involved in an operation with and −2 = IE from part a)
another element, results in the IE for therefore a + b + 2 = −2
that operation. a + b = −4
b = −4 − a
2 The inverse must work from the left Check b ° a where b = −4 – a
as well as from the right of the b°a=b+a+2
operation. = −4 – a + a + 2
= −2
= IE
Therefore, a ° a = a−1 ° a = −2
−1

when a−1 = −4 − a.
3 State the inverse. The inverse is −4 − a.
Chapter 4 An introduction to groups 183
WORKED Example 3
2 2
Find the identity element for the operation defined as a ˚ b = a + b where a and b are
non-negative real numbers.

THINK WRITE
1 An identity element (IE) is an element Let a ˚ b = a (where b = IE)
that, when involved in an operation Therefore
2
a +b =a
2
with another element, does not change
the value of that element. Squaring both sides:
a2 + b2 = a2
b2 = 0
b=0

2 Check that the identity element works Check b ˚ a where b = 0


from both sides of the operation. 0˚a= 0 +a
2 2

=a
Therefore, 0 ˚ a = a ˚ 0 = a. Thus, IE = 0.

3 State the identity element. The identity element is 0.

History of mathematics
NIELS HENRIK ABEL (1802–1829)
During his life . . . Lagrange. As the sole supporting male of his
Lord Byron, the family, at 18 he tutored private pupils while
English poet, writes continuing his own mathematical research. By
Don Juan. the age of 19 he had proved that there was no
Napoleon finite formula for the solution of the general
Bonaparte fifth degree polynomial.
becomes emperor He died of tuberculosis on 6 April 1829,
of France. two days before the announcement of his
Jean-Baptiste posting as professor to the Berlin university.
Lamarck, the His life in poverty stands in contrast to the
French biologist, regard with which he is held in his field; the
proposes that term Abelian group is used in honour of Abel.
acquired traits are His studies on group theory were central to
inherited by individuals the development of abstract algebra.
in a population.
Niels Abel was one of the most productive Questions:
mathematicians of the 19th century. Born in 1. How did Abel financially support his
Norway on 5 August 1802, by the age of 16 he family? He tutored students.
had started his private study of the 2. Which property do groups bearing his
mathematics of Newton, Euler, Gauss and name exhibit? Abelian groups are those that have the property
of commutativity.
184 M a t h s Q u e s t M a t h s C Ye a r 1 1 f o r Q u e e n s l a n d

remember
remember
A set S forms a group under the operation ‘°’ if and only if (iff) all of the
following are true:
1. it is closed under ‘°’; that is, the result is an element of S
2. the order in which operations are performed has no effect on the results; that is,
it is associative
3. there is only one identity element (IE), u, such that a ° u = u ° a = a
4. there is a unique inverse a–1 for every element such that a ° a–1 = a–1 ° a = u,
where u = IE.
5. If the property of commutativity also holds, then it is an Abelian group.

SLE 2: Determine the identity element and inverses in a group table.

4B The terminology of groups


a+b
1 Show that a ° b = ------------ is not closed with respect to whole numbers. (Remember you
2
only have to find one example where the operation is not closed to disprove a state-
ment.) 3-----------
+2
- = 2 1--- and 2 1--- is not an element of the set of whole numbers.
2 2 2
2 2
2 If an operation a ° b is defined as a + b determine whether this is closed if a and b
are whole numbers. 1 ° 3 = 1 + 9 = 10 Not a whole number, ∴ not closed.
WORKED 3 Find a the identity element and b the inverse for the operation ° on real numbers
Example
xample
2
where a ° b = a + b − 1. a IE = 1
b b=2−a
WORKED 4 What is the identity element of the operation a ° b = a + b − ab if a and b are real
Example
xample
3
numbers? IE = 0 a ° 0 = a + 0 − a × 0
Assuming this operation
5 The operation a ° b = 4ab2 is defined for positive real numbers a and b. Does the
has an identity then
identity element for this operation exist? No identity. 4a × ( 1--2- )2 = a but 1--2- ° a ≠ a
let a-----------
+b =a
-
ab a+b
a + b = a2b
6 Develop a proof to show that a ° b = ------------ has no identity.
ab
a = a2b − b
But a ≠ a2b − b
7 An operation is defined with respect to an ordered pair of integers as
therefore the operation (a, b) ° (c, d) = (ad + bc, bd). Show that (0, 1) is the identity element for the operation.
has no identity.
8 Show that a ° b = (a + b)2 has no identity for real numbers.
Let (0, 1) = (a, b) = IE.
Therefore, (0, 1) ° (c, d) = (0 × d + 1 × c, 0 × c + 1 × d) = (c, d)
Properties of groups and (a, b) ° (0, 1) = (a × 1 + b × 0, a × 0 + b × 1) = (a, b).

Let (a + b)2 = a
where b = IE
In the previous section, we looked at the conditions under which a set forms a group.
Take the square root of To check whether a set S forms a group under the operation ‘˚’, that is, [S, ˚], there are
both sides: a + b = ± a four properties to be tested.
1. Closure: the result of the operation is an element of S; that is, a ˚ b ∈ S.
If a is negative then a ∉ R.
Since an identity must be
2. Associativity: the order in which the operation is performed has no effect on the
applicable to all elements of result; that is, (a ˚ b) c = a ˚ (b ˚ c).
the set, there is no IE for a ° b.
Chapter 4 An introduction to groups 185
3. Existence of an identity element: there is only one identity element (IE), u, such that
a ˚ u = u ˚ a = a.
4. Existence of an inverse: there is a unique inverse for every element such that
a ˚ a−1 = a−1 ˚ a = u where u = IE.

Abelian groups
If a set forms a group and the property of commutativity also holds, then it is an
Abelian group. An operation is commutative if the order of the elements involved has
no effect on the result. That is, a ˚ b = b ˚ a.

WORKED Example 4
a Verify that the set of integers forms a group under addition.
b Is this group Abelian?

THINK WRITE
a 1 What numbers are involved? All a Let Z = {a, b, c, …} be the set of integers;
positive and negative integers and 0 are the operation is addition.
involved so state the set and operation.
While you can think of actual values for
the integers (−1, 0 4 …) your answer
should use only variables, with
constants used as examples only.
2 Test each of the 4 properties in the
same order each time to help you
remember the 4 tests.
iii The sum of any 2 integers is an iii The operation is closed:
integer. a + b = c where a, b and c ∈ Z
iii The order in which the operation is iii The operation is associative:
performed has no effect on the (a + b) + c = a + (b + c)
result.
iii Since 0 ∈ Z, IE+ exists. iii The identity element exists:
a+0=0+a=a
iv Since Z contains all positive and iv The inverse exists:
negative whole numbers, the a + −a = −a + a = 0
inverse is −a.
3 State that the system forms a group Thus the set of integers forms a group
under the conditions stated. under addition.

b If the group is Abelian we need to show b Commutativity


that this operation is commutative. a+b=b+a
Therefore the group is Abelian.

Note that the test for commutativity is performed last because the first 4 properties are
necessary to state that it is a group in the first place, before it is shown to be Abelian.
This group, G = [Z, +], is an infinite group, having an unlimited set of elements. You
will also deal with finite groups which have a countable number of elements.
186 M a t h s Q u e s t M a t h s C Ye a r 1 1 f o r Q u e e n s l a n d

WORKED Example 5
Verify that the set of odd integers does not form a group under addition.
THINK WRITE
1 What numbers are involved? S = {a, b, c, …} is the set of odd integers.
The set of odd integers includes The operation is addition.
−5, −3, −1, 1, 3, 5 …
State the set and operation.
2 Test the 4 properties as shown in Closure: a + b ∈ S
Worked example 4. Let a = 3 and b = 5
3 + 5 = 8 and 8 ∉ S
Therefore G ≠ [S, +]
3 There is no need to proceed any further The set of odd integers does not form a group
with tests to verify the system is a under addition.
group as it is not closed.

WORKED Example 6
Construct a Cayley Table for [{1, i, −1, −i}, ×] and determine whether this constitutes a
group.
THINK WRITE
1 Set up the empty table. × 1 i −1 −i
1
i
−1
−i

2 Complete the table. Remember from × 1 i −1 −i


Chapter 2 on complex numbers that
i = – 1 and i × i = −1. 1 1 i −1 −i
i i −1 −i 1
−1 −1 −i 1 i
−i −i 1 i −1

3 Test the 4 group properties — closed 1. All the results are members of the original
set, associative, identity element and set {1, i , −1, −i}. This is a closed set.
multiplicative inverse. The answers can 2. The set is associative
be obtained from the table. e.g. (1 × i) × −i = i × −i = 1
(Multiplication by 1 leaves all elements and 1 × (i × −i) = 1 × 1 = 1
unchanged.) 3. The identity element, IE× = 1
4. Multiplicative inverse: there is a 1 (IE×) in
every row of the table so each element has a
unique inverse.
4 State your conclusion. Therefore, the system is a group.
Chapter 4 An introduction to groups 187
Note that the Cayley Table is symmetrical about the
× 1 i −1 −i
leading diagonal. The table could be flipped over on
the leading diagonal and remain unchanged. This 1 1 i −1 −i
means that the order of operations will not affect the
results; that is, that the operation is commutative. i i −1 −i 1
Therefore this group is also Abelian. −1 −1 −i 1 i
−i −i 1 i −1

Leading diagonal

WORKED Example 7
Construct a Cayley Table for [{mod 5}, +] and determine whether it is an Abelian group.

THINK WRITE
1 Decide what numbers are present in
+ 0 1 2 3 4
mod 5 and complete a Cayley Table of
residues. 0 0 1 2 3 4
1 1 2 3 4 0
2 2 3 4 0 1
3 3 4 0 1 2
4 4 0 1 2 3

2 Test for the 4 group properties. 1. All results are members of the original set.
So, the set is closed.
2. Addition with whole numbers is associative.
3. The identity element, IE+ = 0 exists.
4. There is a 0 entry in each row because each
element has a corresponding element that,
when added, results in 0 (IE+). So, there is
an additive inverse.
Therefore the system forms a group.

3 Test for commutativity. Addition mod 5 is commutative as shown by


the symmetry about the leading diagonal.
+ 0 1 2 3 4 For example: 4 + 0 ≡ 4 and
0 0 1 2 3 4 0+4≡4
1 1 2 3 4 0 and 4 + 2 ≡ 1 and
2+4≡1
2 2 3 4 0 1
Therefore the group is Abelian.
3 3 4 0 1 2
4 4 0 1 2 3

Note: There are 9 axioms that relate to operations and whole numbers that require no
proof: they are assumed to be true. The associativity statement in the example above
relied on one of these axioms and you can state that these axioms have been used.
6 a + 4 16 64 256 …
188 M a t h s Q u e s t M a t h s C Ye a r 1 1 f o r Q u e e n s l a n d 4
16 20
8 20
32
68
80
260
272


64 68 80 128 320 …
They are given here with no explanation. 256 260 272 320 512 …
. . . . .
1. Closure Law of Addition . . . . .
. . . . .
2. Commutative Law of Addition
3. Associative Law of Addition
4. Identity Law of Addition
5. Closure Law of Multiplication
6. Commutative Law of Multiplication
7. Associative Law of Multiplication
8. Identity Law of Multiplication
9. Distributive Law of Multiplication over addition, where a(b + c) = ab + ac

5 ×
1
1
1
2
2
3
3
4
4
remember
remember
2 2 4 1 3 1. To determine whether a set forms a group under an operation (°) test each of
3 3 1 4 2 the four properties; that is, test whether it is closed and associative, whether
4 4 3 2 1 there is an identity element and a unique inverse.
Closed, associative, 2. To determine whether the group is Abelian, show that the operation is
IE× = 1 and there is
an inverse; therefore
commutative (e.g. a ° b = b ° a).
it is a group.
SLE 1: Determine whether the elements of a set form a group under a binary operation.
SLE 2: Determine the identity element and inverses in a group table.
SLE 3: Use a small Cayley table to determine whether a set of elements under a binary
operation forms a group.

4C Properties of groups
SLE 4: Investigate when the integers modulo n form groups under addition or multiplication.
SLE 9: Investigate commutativity and abelian groups.
WORKED 1 a Verify that the set of real numbers, [R, +], forms a group under addition.
Example
xample [R, +] It is closed, associative, IE+ = 0, inverse = −a, therefore it is a group.
4 b Is this group Abelian?
It is Abelian. Closed, associative, no IE since
0 ∉ {even numbers}, there is an
2 a Consider the set of even numbers (2n) where n ∈ ±Z. inverse; therefore not a group.
eBook plus
b Does this form a group under addition? (Note: 0 ∉ {even numbers})
Digital doc: c Does it form a group under multiplication? Closed, associative, no IE since 1 ∉ {even
SkillSHEET 4.2 numbers}, no inverse; therefore not a group.
Properties of groups
3 Does the set of powers of 1 form a group under:
a addition? 12 + 13 is not closed; not a group.
b multiplication? 12 × 13 is closed, and associative, IE× = 1, there is an inverse; so it is a group.
Check with
WORKED 4 Verify that the set of even integers does not form a group under division. your teacher.
Example
xample
5

WORKED 5 Construct a Cayley Table for [{mod 5 excluding 0}, ×] and determine whether this
Example
xample
6
constitutes a group.

6 a Draw up a Cayley Table for the set of even powers of 2 under addition.
Under addition: not closed, associative, no IE+ since
b Does this form a group under addition? 0 ∉ 22n, no inverse (always +ve); not a group.
c Does this form a group under multiplication?
Under multiplication: closed, associative, IE× = 1 is not present as no 20 (0 ∉ {even numbers}), inverse is 2–2n; not a group.
WORKED
Example
xample 7 Construct a Cayley Table for [{mod 3}, ×] and determine whether it is an Abelian
7 group. × 0 1 2 It is closed and associative, IE× = 1; inverse does not exist since there are no 1s in
0 0 0 0 the first row or column. This is not a group; therefore, it is not Abelian, even
1 0 1 2 though the commutative law does apply.
2 0 2 1
Chapter 4 An introduction to groups 189
8 Determine whether each of the tables below forms a group.
a a b c Yes b a b c
No, not closed
° °
10
a c a b a a b c
° N L R A
b a b c b b a c
N N L R A
L L A N R c b c a c c d a
R R N A L
A A R L N c a b c
No, no inverse d
a b c
Yes
° for b °
Closed, associative,
IE° = N, there is an
a a b c a b c a
inverse, N appears in b b b c b c a b
every row and column. c c b a c a b c
° 5 10 20
5 5 10 20
9 a Construct a Cayley Table for the set 10 10 10 20
a ° b = [{5, 10, 20}, lowest common multiple of a, b] 20 20 20 20
b Does this set form a group? It is closed, associative, IE° = 5, no inverse; so not a group.
10 The movements of a robot are restricted to no change (N), turn left (L), turn right (R),
turn about (A): {N, L, R, A}. Construct a Cayley Table and show that this set of
movements under the operation ˚, meaning ‘followed by’, forms a group.

Cyclic groups and subgroups


By completing the exercises so far, you would be aware that both real numbers and
integers form a group under addition. However, the set of integers is contained within
the set of real numbers (this is called a subset). Because of this, we can say that integers
under addition is a subgroup of real numbers under addition. When identifying sub-
groups, it is not necessary to verify associativity, but the existence of closure, the
identity element and inverses needs to be confirmed.

WORKED Example 8
Does the set of numbers {0, 2, 4} form a subgroup of addition modulo 6?
THINK WRITE
1 To check for closure, construct a
+ 0 2 4
Cayley Table.
0 0 2 4
2 2 4 0
4 4 0 2

This is a closed set.


2 Is the identity included? IE+ = 0. Therefore the identity is included.
3 Do inverses exist? There is a 0 entry in each row, so the inverses
are included.
4 State your conclusion. The set of numbers {0, 2, 4} forms a subgroup
of addition modulo 6.
190 M a t h s Q u e s t M a t h s C Ye a r 1 1 f o r Q u e e n s l a n d

In modulo 6 addition, 2 + 2 = 2 × 2 ≡ 4 mod 6, 2 + 2 + 2 = 3 × 2 ≡ 0 mod 6,


2 + 2 + 2 + 2 = 4 × 2 ≡ 2 mod 6, 2 + 2 + 2 + 2 + 2 = 5 × 2 ≡ 4 mod 6. If we continue,
we get 6 × 2 ≡ 0 mod 6, 7 × 2 ≡ 2 mod 6 and so on. This means that all members of
the subgroup {0, 2, 4} can be a result of repeatedly adding the number 2. This sub-
group is said to be cyclic (because the answers cycle through the set) and 2 is the
generator of the cyclic subgroup. We can write this group as G = [{0, 2, 4}, + mod
6] = <2>, where <2> means that G is cyclic and 2 is the generator.
From previous sections, we know that integer multiplication modulo 5 forms a group
if zero is not included. We can write this set of numbers as Z5*. If we calculate powers of
3, we find that 30 ≡ 1 mod 5, 31 ≡ 3 mod 5, 32 ≡ 4 mod 5, 33 ≡ 2 mod 5, 34 ≡ 1 mod 5,
35 ≡ 3 mod 5 and so on. This means that all elements in Z5* can be found by repeatedly
multiplying the number 3. The group is cyclic and 3 is a generator. We can write this
group as G = [Z5*, ×] = <3>.
Writing this more mathematically, let’s consider a group G = [S, ˚] and let a be one
element of S. If ˚ represents addition, that is, G = [S, +], and all elements of S can be
written as na, where n is an integer, then the group is cyclic and a is the generator.
Similarly, if ˚ represents multiplication, that is, G = [S, ×], and all elements of S can
be written as an, then the group is also cyclic and a is the generator.
In both cases, we can write G = <a>, meaning G is a cyclic group generated by a.
Another group we can consider as an example is integers under addition. Every
integer can be written as n1. This means that integers under addition is a cyclic group
and 1 is the generator. Every integer can also be written as n(−1), therefore −1 is also a
generator. We can write this group as G = [Z, +] = <1> = <−1>.

WORKED Example 9
a Compute the cyclic subgroups <1>, <2>, <3>, <4>, <5> of the group addition modulo 6.
b Hence identify the generators of the group addition modulo 6.
THINK WRITE
a 1 We need to compute na for a = 1, 2, a n 1n 2n 3n 4n 5n
3, 4 and 5. For integers modulo 6,
0 0 0 0 0 0
there are 6 elements therefore n = 0,
1, 2, 3, 4, 5 will cover all elements. 1 1 2 3 4 5
2 2 4 0 2 4
3 3 0 3 0 3
4 4 2 0 4 2
5 5 4 3 2 1

2 Each column is the subgroup for that <1> = [{0, 1, 2, 3, 4, 5}, + mod 6]
element. Write the subgroups. <2> = [{0, 2, 4}, + mod 6]
<3> = [{0, 3}, + mod 6]
<4> = [{0, 2, 4}, + mod 6]
<5> = [{0, 1, 2, 3, 4, 5}, + mod 6]
b Identify the generators. A generator b The generators are 1 and 5 for the cyclic
produces all elements of the group, that group addition modulo 6.
is, {0, 1, 2, 3, 4, 5}. Look for a column
that has every element.
Chapter 4 An introduction to groups 191
2 Closed, IE = 0 0 ,
00 remember
remember
the inverse of For the group G = [S, ˚]:
1. A subgroup exists if there is a subset of S that is closed and contains the
a 0 is – a 0
identity element, and if inverses for all elements exist.
0b 0 –b
2. The group is cyclic if an element a can be found that generates the group. This
which is a member can be written as G = <a>.
of M, therefore
a subgroup. SLE 6: Construct a Cayley table and use it to identify subgroups (if any)
such as the rotations of a square about its centre.
SLE 7: Find the element(s) which generate(s) the group in a group table.

4D Cyclic groups and subgroups


WORKED 1 Does the set of numbers {0, 3, 6, 9} form a subgroup of integer addition modulo 12?
Example
Closed, IE+ = 0, inverses exist (3 + 9 ≡ 0 mod 12, 6 + 6 ≡ 0 mod 12), therefore subgroup. Generators are 3 and 9.
8
2 The set M(2, Z) is the set of all 2 × 2 matrices with integers as entries. These matrices
form a group under matrix addition. Prove that the subset of M containing only diag-
onal matrices (those with 0s in the upper right-hand and lower left-hand corners) form
a subgroup.
WORKED 3 a Compute the cyclic subgroups <3>, <6> and <9> of the group integer addition
Example
9
modulo 12. <3> = [{0, 3, 6, 12}, + mod 12]; <6> = [{0, 6}, + mod 12]; <9> = [{0,3,6,12}, + mod 12]
b Hence identify the generators of the subgroup defined in question 1. Generators are 3 and 9.
4 Which of the following groups are cyclic? For each cyclic group, name all generators
of the group.
eBook plus a [Z, +] Cyclic. Generators are 1 and −1.
Digital doc:
b [Q, +] (Q is the set of rational numbers) Not cyclic.
WorkSHEET 4.1 c [6n where n is an integer, +] Cyclic. Generators are 6 and −6.
d [3n where n is an integer, ×] Cyclic. Generators are 3 and --13- .

Application of groups —
permutations
° P1 P2 P3
A symmetry of a square (or any other shape) may be written as a permutation by
P1 P1 P2 P3 changing the positions of the vertices. For example, referring to the figure below
P2 P2
P3 P3 P1 right, we could write:
1
Not closed ∴ Not a group  
1 2 3 4 5 P2 =  1 2 3 4 5  , which means that vertex 1 goes
P2 ° P2 =   2 3 4 5 1
 3 4 5 1 2 5 2

1 2 3 4 5 to the position of vertex 2, and so on.


P2 ° P3 = 
 5 4 3 2 1
The only other two permutations allowed here are:
1 2 3 4 5
P3 ° P2 =     
4 3
 2 1 5 4 3
P1 =  1 2 3 4 5  and P3 =  1 2 3 4 5 
1 2 3 4 5  1 2 3 4 5  1 5 4 3 2
P3 ° P3 =  = P1
 1 2 3 4 5
Determine whether these permutations form a group under the operation ° meaning
‘on’. P1 ˚ P3 means perform P1 on P3.
192 M a t h s Q u e s t M a t h s C Ye a r 1 1 f o r Q u e e n s l a n d

Further examples of groups —


transformations
Consider all the transformations that a shape could
undergo. Rotations about its centre and reflections about
its axes of symmetry involve changes in the vertices
only. Carefully examine the diagram below. Make sure
you understand the symbols and the new positions of
the vertices.

Rotations anticlockwise: R90 → 90°


R180 → 180°
R270 → 270°

Reflections: RV → in the vertical axis of symmetry


RH → in the horizontal axis of symmetry
RR → in the top right diagonal
RL → in the top left diagonal
R0 → no change.

D A D A

D C A B
R0
RL
RL R 90
C B
B D
A C
C B
B A D RV B C
A
RL RR
RR RH R180

B
C D C A D

C B C D

A D
RH R 270

RV
D A B A

B C

Therefore the set of all transformations or symmetries is given by the set


{R90, R180, R270, RV, RH, RR, RL, R0} and the binary operation that combines any two of
these transformations is referred to as composition, where one operation follows another.
Chapter 4 An introduction to groups 193
All the computerised movements involved in screen animations are based on similar
compositions of transformations.
As with permutations, the operations are performed right to left.

WORKED Example10
Find the result of Rv ° R180 .

THINK WRITE/DRAW

1 Draw the initial square with labelled D A


vertices. Consider the order in which
the operations are to be performed.
Since the expression Rv ˚ R180 means
that Rv follows R180, then R180 is C B
performed first.
2 Transform the square using R180 — B C
180° rotation anticlockwise. Locate
vertex A and move it 180˚ anti-
clockwise. All other vertices follow in
order around the square. A D

3 For RV mark a vertical axis of B C


symmetry in this figure (from step 2)
and reflect or ‘flip’ the square about this
axis.
A D

4 Reposition the vertices one side at a time, C B


B ↔ C and A ↔ D.

D A

5 This matches with a single The result is RH.


transformation representing RH.

Functions
1 1
Consider functions f(x) = x, g(x) = −x, h(x) = --- and k(x) = − --- (where x ≠ 0).
x x
When these functions are involved in composition of functions such as g[h(x)], the
function h(x) is substituted as the inner function into the outer function which is g(x).
1 1
That is, g[h(x)] = − --- where --- (the inner function) is substituted into g(x) which is −(x).
x x
1
Similarly, k[g(x)] = − ---------- where g(x) = −x (the inner function) is substituted into
(–x)
1--- 1 1
k(x) = − (the outer function). That is, k[g(x)] = − ---------- = --- = h(x).
x (–x) x
194 M a t h s Q u e s t M a t h s C Ye a r 1 1 f o r Q u e e n s l a n d

WORKED Example 11
1 1
Show that functions f(x) = x, g(x) = −x, h(x) = --- and k(x) = − --- form a group under
composition. x x

THINK WRITE
1 Complete a Cayley Table
° f g h k
for these compositions.
f f g h k
g g f k h
h h k f g
k k h g f

2 Test the 4 group properties. Closure: yes — all results are elements of the original set.
Associative: yes — for example,
(f ° g) ° h = g ° h = k
f ° (g ° h) = f ° k = k
Identity element is f(x).
Inverse: yes — f(x) occurs in every row and column.
3 State your conclusion. Composition of these functions forms a group.

History of mathematics
A R T H U R C AY L E Y ( 1 8 2 1 – 1 8 9 5 )

During his life . . . mathematics at Cambridge University. One of


Thomas Edison his most famous non-mathematical
invents the accomplishments was his role in having
phonograph. women accepted at Cambridge.
Slavery is Like Niels Abel (see page 183), many of
officially his research topics are now used in abstract
abolished algebra and group algebra, as well as in work
throughout the with matrices and the theory of determinants.
western world. The Cayley Table is named after him.
Alfred Nobel He died on 26 January 1895 having
invents dynamite. received many academic distinctions. His total
works fill 13 volumes of about 600 pages each
Arthur Cayley, a famous English
— a testimony to his prodigious life and study
mathematician, was born on 16 August 1821.
in mathematics.
His published mathematical papers are
classics and include discussions on the
He worked towards having women
concept of n-dimensional geometry. At the Questions accepted at Cambridge University.
age of 25 he began practising law which he 1. What is one of Cayley’s most significant
continued to do until 1863. In his spare time non-mathematical accomplishments?
he wrote more than 300 mathematical papers. 2. List four fields of mathematics which
In 1863 he accepted a professorship in feature in Cayley’s work.
Abstract algebra, group algebra, n-dimensional
geometry, matrices and determinants
Chapter 4 An introduction to groups 195
remember
remember
The binary operation that combines any two transformations (for example,
rotation and reflection) is called composition, when one operation follows another.

SLE 5: Investigate groups formed by geometric transformations such as the


reflections of a rectangle in its axes of symmetry and rotations of an
equilateral triangle.

4E Further examples of groups —


transformations
1 a Draw a Cayley Table for the rotation of an equilateral triangle. Label each vertex.
b Does it form a group? Is it Abelian? IE = R0, Inverse exists for all elements. It is an Abelian group
because the table is symmetrical about the leading diagonal.
2 a Draw a Cayley Table for the reflections of an equilateral triangle through each of
the vertices R0, RV, RL, RR.
1 A B C
b Does it form a group? Does not form a group.

2 A a ° R0 RV RL RR
R0 R0 RV RL RR C R0 B A R120 C B R240 A
RR RL RR RL
RV RV R0 — — a
RL RL — R0 — ° R0 R120 R240
RR RR — — R0 R0 R0 R120 R240
R120 R120 R240 R0
C RV B
R240 R240 R0 R120
RV

4 c
Not Abelian.
3 Explain what the following diagrams represent about the group shown below.

3
WORKED
Example
10 1 1 3

4
2

4
4 a 4 3
F R240

3 R180

RH 1
3
3 4 RV 1 2
2 1 3 2 2 31 2 4

2
2 1 RH
1 2 2
RV 1
R0

3 4
1

3 R0 4 1 2 R240 F
3

R180
2

2
4

2 1 4 3 3 2 1 33 1
1

4 Describe the symmetries of the following figures, using fully annotated diagrams.
a a non-square rectangle
b a non-square rhombus
c an ellipse

5 Consider an infinitely long strip of Hs, printed on transparent paper, as shown below
…..H H H H H H …. … H H H H … RV … H H H H …
RH R180 … H H H H …
Describe the axes of symmetry of this group. … H H H H … R0 … H H H H …
6
196 M a t h s Q u e s t M a t h s C Ye a r 1 1 f o r Q u e e n s l a n d

6 Locate the axes of symmetry for the following figures.


7 a
° f1 f2 f3 f4
f1 f1 f2 f3 f4
f2 f2 f1 f4 f3
f3 f3 — — —
f4 f4 — — —

WORKED 7 a Complete a Cayley Table for the composition of the following functions.
Example
xample
11 1 1
f1(x) = x f2(x) = --- f3(x) = x − 1 f4(x) = -----------
x x–1
where f1 ° f2 = f1[f2(x)]
8 a Closed: addition of b Does this composition form a group? Not a group
2 × 2 matrices results
in a 2 × 2 matrix. 8 a Show that the set of all 2 × 2 matrices a b forms a group under matrix
Associative: matrix c d
addition.
addition is
associative.
Identity exists: 0 0 b The set of all 2 × 2 matrices a b does not form a group under matrix multi-
is the identity 0 0 c d
element. plication but it is possible to find a subset of 2 × 2 matrices that will form a multi-
Inverses exist: the plicative group.
inverse of A is −A.
i Apart from the identity matrix, give an example of a 2 × 2 matrix that would be
b i 12 ii 1 2 a member of this subset.
34 12 ii Give one example of a 2 × 2 matrix that would not be part of this subset.
iii The set of 2 × 2, iii Describe the largest subset of 2 × 2 matrices that would form a group under
non-singular matrix multiplication.
matrices.  
9 Show that the set of matrices  1 0 , 1 0 , – 1 0 , – 1 0  forms a group
Identity = I. Inverse is
present as I is present in under matrix multiplication.  0 1 0 – 1 0 1 0 –1 
each row and column.
Closed and associative z1 z2
10 Show that the set of matrices of the form , where z is a complex number,
forms a group –z2 z1
a IE+ = 0 0
0 0 a under matrix addition
1 z1 –z2
(Remember 0 is a b under matrix multiplication. IE× = I Inverse = --------------- where the determinant is real.
2 2
complex number.) Assume z12 + z22 ≠ 0. z1 + z2 z2 z1
–z1 –z2 The inverse exists if the determinant ≠ 0.
Inverse =
z2 –z1 11 S is the set of all 2 × 2 matrices such that 0 0 , where z is a non-zero complex
number. z z
b Yes.
Closed, associative, a Show that 0 0 is the identity element under matrix multiplication.
IE, Inverse of 1 1
b Does the set form a group under matrix multiplication?
0 0 is 0 0
1--- 1---
z z z z
12 C = i 0 , where i = – 1 . The set T consists of positive powers of C such that
0 i
T = C n where n is a positive integer. i 0 , –1 0 , –i 0 , 1 0
a Find all the elements of set T. 0 i 0 –1 0 –i 0 1
b Does the set T form a group under matrix multiplication? Yes.
Chapter 4 An introduction to groups 197

Some applications of group theory


1 Do the residues of {0, 1} mod 2 form a group under addition? Yes
2 A teacher of abstract algebra intended to give a typist a list of 9 integers that
form a group under multiplication modulo 91. Instead, one of the 9 integers
was omitted so that the list read: 1, 9, 16, 22, 53, 74, 79, 81. Which integer
was left out? 29
3 Show that {1, 2, 3} multiplication mod 4 is not a group but {1, 2, 3, 4}
multiplication mod 5 is a group.
4 Give an example of group elements a and b with the property that a−1ba ≠ b.
5 The integers 5 and 15 are two of 12 integers that form a group under
multiplication mod 56. List all 12 integers. 1, 3, 5, 9, 13, 15, 19, 23, 25, 27, 39, 45
6 If the following table is that of a group, fill in the blank entries.

e a b c d e a b c d
e e — — — — e e a b c d
a a b c d e
a — b — — e b b c d e a
b — c d e — c c d e a b
d d e a b c
c — d — a b
— — — — — —

7 Prove that if G is a group such that the square of every element is the identity,
then G is Abelian.
8 Examine whether
a rotations and Yes b reflections No
as stated earlier in this section, form Abelian groups.
9 Quaternions
The concept of a set of elements called quaternions was first developed by the
Irish mathematician William Hamilton (see page 118). Quaternions are
ordered sets of four ordinary numbers, satisfying special laws of equality,
addition and multiplication. Quaternions are useful for studying quantities
having magnitude and direction in three-dimensional space and this has
enabled great advances in quantum theory, relativity, number theory and group
theory.
The 4 numbers are 1, i, j and k and have the following properties:
12 = 1
i2 = j2 = k2 = ijk = −1
1i = i1
1j = j1
1k = k1
ij = −ji = k
i(jk) = (ij)k = ijk
All real and complex numbers do commute with i, j, and k but they are not
commutative with each other.
198 M a t h s Q u e s t M a t h s C Ye a r 1 1 f o r Q u e e n s l a n d

Follow this example that shows that jk = i


ijk = −1 from the definitions
i × ijk = i × −1 multiply both sides by i on the left (or pre-multiply by i)
i2jk = −i associativity
−1 × jk = −i from the definitions
−1 × −1 × jk = −1 × −i pre-multiply both sides by −1
jk = i
Because multiplication between these elements is not commutative it is
essential that all multiplication is done from a particular side of an expression
and to perform this multiplication on both sides of the equals sign. You must
respect the order of placement of terms in this system.
a Show that i jk = −kj ii ki = j
SLE 15 (Matrices and applications):
Investigate the group b Show that i–1 = −i
properties of matrices of the c If q = s + wi + vj + yk and p = m + ni + oj + jk, find the product of the two
z1 z2
form -------- ----- under both quaternions.
–z2 z1

addition and multiplication;


10 Pauli Matrices
find interesting subsets of The ideas introduced in the section on quaternions above can be extended to
this class of matrices
(known as quaternions); represent matrices. One 2 × 2 set is:
in particular, show that
the eight matrices
1= 1 0 i= –1 0 j= 0 1 k= 0 –1
1 0 , –1 0 , i 0 ,
01 0 –1 0 i
0 1 0 – –1 –1 0–1 0
–i 0 , 0 1 , 0 –1 , While the matrices for i and k might look a little daunting, they can be
0 i –1 0 1 0 simplified by replacing the – 1 elements with complex i.
0 i , 0 – i form a group The last three of these matrices are used in the study of quantum theories to
i0 –i 0 explain and predict the behaviour of electrons. They are called the Pauli Spin
under multiplication. Matrices and students of chemistry will appreciate the importance of the spin
of electrons in atomic bonding and the strength of different materials.
0 –i
–i 0

A variation of these matrices used in the study of nuclear physics is shown


W

Q
V

R
T

below:
i 0
0 –i

W
U

Q
V

R
P
T

P= 1 0 Q= 0 1 R = 0 –1 S = –1 0
0 1 –1 0 1 0 0 –1
–i 0
0 i

W
U

Q
V

P
R
T

T= 0 i V = –i 0 U= i 0 W = 0 –i
0 i
i0

W
U

Q
V

P
T
T

i 0 0 i 0 –i –i 0
–1 0
0 –1

W
Q

U
R

T
S

On examination of the first and second rows of the matrices above you will
notice that the second row is a reflection of each matrix in the first row,
0 –1
1 0

W
Q
U
R

R
P

V
T
S

multiplied by i.
a Construct a Cayley Table to display the results of matrix multiplication
01
–1 0

W
Q

U
P
R
V
T
S

using these 8 matrices. Arrange them in the order given, that is, from P to W.
b Determine whether the total set forms a group. Yes
10
01

W
Q

U
P

V
T

c Mark off the top left-hand 4 × 4 corner. Examine this section of the table
S

and show that this subset forms a group. This is an example of a subgroup,
W
Q

U
P

V
T
S

where a subset of a group forms a complete group of its own.


10 a

11 Internet search
The real life applications of groups are quite complex. Use the internet to
research this field of study. Include a list of distinct topics and a more detailed
report that highlights the use of group theory.
Chapter 4 An introduction to groups 199

History of mathematics
CRYPTOGRAPHY
Since World War II, mathematicians have played a large part in the development
and attack on ciphers. A cipher is a way of scrambling text so it can be read only
by the people who know how to unscramble it. Had the Government Code and
Cipher School (at Bletchley Park, England) not been able to read the German
Enigma ciphers, historians estimate that World War II may have lasted another
two to three years. Until 1974, the contribution made by mathematicians to the
war effort went largely unrecognised.
Why include talk of ciphers and code breaking in a chapter on group theory? It
is because, in the late 1970s, the two topics collided. In 1969, ARPAnet (the fore-
runner to the Internet) was born. Whitfield Diffie, a mathematician employed in a
series of jobs related to computer security, could see the potential of ARPAnet but
was concerned that people using email would be deprived of the right to privacy.
At that time, if two people (say, Alice and Bob) wanted to encrypt a message,
they needed to exchange a key.
For Alice to send a secure message to Bob, encryption can n 2n
be thought of as putting the message in a strong box and
locking it. The locked box can be sent securely. The problem 0 1
is that Bob needs a key to open the box. What is known as the 1 2
‘key distribution problem’ is basically ‘How do Alice and
2 4
Bob swap keys?’.
Whitfield Diffie and a cryptographer called Martin 3 8
Hellman worked on the key exchange problem from Sep- 4 16
tember 1974. In 1975, they came upon the ‘double padlock
solution’. Essentially, Alice locks the box and mails it to Bob. 5 13
Bob applies his own lock (leaving Alice’s in place) and 6 7
returns it. Alice removes her lock and sends the box (still 7 14
locked by Bob) back to Bob. Bob can now unlock the box.
What was needed was a mathematical function that was the 8 9
equivalent of a padlock (called a one-way function). Like a 9 18
padlock, anyone could lock (encrypt), but without a key, the
10 17
lock couldn’t be opened (decrypted).
Group theory provides the solution to the one-way 11 15
problem. Exponentiation (working out ab) is easy and, in the 12 11
real number system, it is almost as easy to reverse using log-
arithms. In the group formed by multiplication modulo [large- 13 3
prime-number], the logarithms are much harder to calculate. 14 6
In the table at right, you can see that 2 is a generator for
15 12
this group. Can you work out what modulo has been used? If
you were given a number from the group formed from 2n, say 16 5
10, but you knew the modulo that had been used and that 2 17 10
was the generator, could you work out the logarithm or power
(n) used? What if the modulo used was a much larger prime 18 1
number? (continued)
200 M a t h s Q u e s t M a t h s C Ye a r 1 1 f o r Q u e e n s l a n d

A solution to the key distribution problem is as follows. Alice and Bob agree to
use mod 19 (in practice, it would be a massive prime number) and the generator 2
(anyone can know this information). Alice chooses her key, a (let’s make it 13).
She evaluates 2a = 213 ≡ 3 mod 19 and sends this to Bob. Bob chooses his own
key, b (let’s make it 16). He evaluates 2b = 216 ≡ 5 mod 19 and sends this to Alice.

When Bob receives 2a from Alice, he


evaluates (2a)b ≡ 316 ≡ 17 mod 19.
Alice, receiving 2b from Bob, can
evaluate (2b)a ≡ 513 ≡ 17 mod 19.
They can then use the key generated
(2ab) to safely send messages. If an
eavesdropper, Evan, intercepts their
messages, he cannot calculate the key
without first being able to solve what
is called the ‘discrete log problem’.
Publicly, three researchers from
Massachusetts Institute of Technology
(MIT), computer scientists Ron Rivest
and Adi Shamir, and mathematician
Leonard Aldeman, used modular
arithmetic and properties of large
primes to create the RSA algorithm
for public key cryptography. (RSA
stands for Rivest, Shamir, Aldeman.)
If the primes used are large enough, this cipher is believed to be secure against
attack.
We now know that Diffie and Hellman were not the first people to develop the
idea of public key cryptography. That honour must go to James Ellis, an
employee of the British Government Communications Headquarters (formed
from the remnants of Bletchley Park after World War II). Like Diffie and
Hellman, Ellis was unable to find the one-way function. In 1969, he shared his
idea with his bosses but it wasn’t until 1973 that a mathematician called Clifford
Cocks found the answer using group theory. The discovery made by Ellis and
Cocks was not publicly acknowledged until 1997.
Chapter 4 An introduction to groups 201

summary
Modulo arithmetic
• Modulo arithmetic is like clock arithmetic where 5 + 9 = 4 in mod 10.
• The residues of modulo x are all the numbers less than x.
• Congruent numbers in mod x all differ by multiples of x.

The terminology of groups


For a set S = {u, a, b, c, …} and an operation ˚, we say that
• the operation is closed if a ˚ b is an element of S
• the operation is associative if (a ˚ b) ˚ c = a ˚ (b ˚ c)
• u is the identity element (IE) if a ˚ u = u ˚ a = a.
• a−1 is the inverse of a if a ˚ a−1 = a−1 ˚ a = u where u = IE.

Properties of groups
• A set forms a group under an operation if elements of the set are closed and
associative, and there is a unique identity element and every element has a unique
inverse.
• The group is an Abelian group if the operation is commutative (e.g. a ° b = b ° a).

Cyclic groups and subgroups


For the group G = [S, ˚]:
• a subgroup exists if there is a subset of S that is closed and contains the identity
element, and if inverses for all elements exist.
• the group is cyclic if an element a can be found that generates the group. This can
be written as G = <a>.

Transformations
• The set of all transformations (for example, rotations and reflections) and the binary
operation that combines any two of these transformations is referred to as a
composition.
202 M a t h s Q u e s t M a t h s C Ye a r 1 1 f o r Q u e e n s l a n d

1 a + 0 1 2 3 4 b × 0 1 2 3 4 5
0 0 1 2 3 4 0 0 0 0 0 0 0
1 1 2 3 4 0 1 0 1 2 3 4 5

CHAPTER 2
3
2
3
3
4
4
0
0
1
1
2
2
3
0
0
2
3
4
0
0
3
2
0
4
3

review 4 4 0 1 2 3 4
5
0
0
4
5
0
4
0
3
4
2
2
1

1 Draw up a Cayley Table that shows the residues for each of the following:
4A a addition mod 5
b multiplication mod 6.

2 Determine whether the following are groups:


4B a {1, −1} under multiplication Yes
b {0, 1} under addition. No, not closed

3 Determine whether the following are groups:


4C a the set {1, 2, 4, 5, 7, 8} under multiplication modulo 9 Yes
b the set {0, 1, 2, 3, 4} under multiplication modulo 5 No, 0 does not have an inverse
c the set {2, 4, 6, 8} under multiplication modulo 10 Yes
d the set {0, 1, 2} under addition modulo 3. Yes

4 Determine whether each of the following form groups:


a the set of integers where p ° q = p + 2q No, no identity
p
b the set of positive rational numbers where p ° q = --- . No, not associative
q
c Show that the set of all integers forms an Abelian group under the operation
a ° b = a + b − 3.

5 There are two lights in a room, one on the ceiling and one on the wall with 4 possible states
4C for the two lights — both on, both off, wall light on only, or ceiling light on only. There are
4 possible changes of state: no change, both change, ceiling light change and the wall light
change. These changes are denoted by N, CW, C and W respectively. Show that the set
{N, C, CW, W} forms a group with respect to the operation ‘followed by’. Check with your teacher.

6 What property of a group is displayed in a Cayley Table if:


4C a the elements are symmetrical about the leading diagonal Commutativity
There is only element x
such that p ° x = q and
x ° q = p.
b the same element does not appear more than once in any row or column
c the identity element occurs only once in each row or column. Each element has a unique inverse.
Chapter 4 An introduction to groups 203
7 Determine whether the following are groups:
a the set of integers, modulo n under addition Yes
4C
b the set of integers, modulo n under multiplication No, 0–1 doesn’t exist.
c the set of integers, modulo n, excluding 0, under multiplication No, inverses don’t always exist.
d the set of integers, modulo n, excluding 0, under multiplication, if n is prime. Yes

8 Determine whether the set of all moves that can be made by a knight on a chessboard forms
a group or not. (The diagram shows three of the possible moves a knight can make on a 4C
chessboard. The knight can move two squares horizontally and one square vertically, or two
squares vertically and one square horizontally — like a letter L.) Check with your teacher.

9 Consider an operation defined by the following Cayley Table.


4D
u a b c
˚
u u a b c
a a e c b
b b c e a
c c b a e

a Does the table define a cyclic group? No, there is no generator.


b Does the subset {u, a, b} form a subgroup? No, the operation is not closed on these elements.
c Identify a subgroup and the generator of the subgroup. <a> = {0, a}, <b> = {0, b}, <c> = {0, c}

10 a Verify that the set m 0 , where m ≠ 0 forms a group under matrix multiplication.
0 m 4E
Check with your teacher.

b Verify that all p × q matrices form a group under matrix addition.

11 Show that the following set of matrices forms a group under multiplication.
4E
1 0 –1 0 i 0 –i 0 0 1 0 –1 0 i 0 –i
0 1 0 –1 0 i 0 –i 1 0 –1 0 i 0 –i 0

12 Determine whether or not the following functions form a group under composition of
functions. Assume that they are associative. 4E
1 1 x x+1
f1(x) = x f2(x) = --- f3(x) = 1 + x f4(x) = ------------ f5(x) = ------------ f6(x) = ------------
x 1+x x+1 x
204 M a t h s Q u e s t M a t h s C Ye a r 1 1 f o r Q u e e n s l a n d

Modelling and problem solving


1 If a group consists of 4 elements, the identity element u and a, b, c, and an operation ˚, then
we can start to complete a Cayley Table as follows: Check with your teacher.
u a b c
˚
u u a b c
a a ✸
b b
c c
a Investigate the claim ‘The value indicated by ✸ can’t be filled with a’.
b If ✸ is filled with the identity element, u, the table can be completed in two possible ways
to describe a group. Find the two tables. (You need not check the associative law.)
c If ✸ is filled with b (or c), there is only one way to complete the table that results in a
group. Using b for ✸, find the table. (You need not check the associative law.)
d If you consider the three tables you have produced, two of them describe the same
structure. Determine which two tables these are and show how the elements in one table
eBook plus would have to be renamed to produce the other.
Digital doc: 2 Earlier in this chapter, we found that G = [Z6, + modulo 6] is a cyclic group with two
Test Yourself
Chapter 4 generators, 1 and 5. For two distinct primes p and q, investigate the number of generators for
the group [Zpq, + modulo pq]. (Distinct primes means that p ≠ q.) Check with your teacher.
Matrices
and their
applications
5
syllabus reference
Core topic:
Matrices and applications

In this chapter
5A Inverse matrices and
systems of linear equations
5B Gaussian elimination
5C Introducing determinants
5D Properties of determinants
5E Inverse of a 3 ¥ 3 matrix
5F Cramer’s Rule for solving
linear equations
206 M a t h s Q u e s t M a t h s C Ye a r 1 1 f o r Q u e e n s l a n d

Inverse matrices and systems of


linear equations
• inverse of a matrix The ideas you were introduced to in Chapter 3 on matrices provide a foundation for
• determinant of a matrix
• solution of systems of using matrices to solve simultaneous equations. The following section presents a sim-
homogeneous and non- plified account of complex aspects of economies and industries and will serve to intro-
homogeneous linear
equations using matrices duce the idea of matrix applications in a wide field of studies.
• applications of matrices in
both life-related and purely
mathematical situations
Input–output analysis SLE 7: Use input-output (Leontief) matrices in economics.
To enable government bodies and companies to plan for future needs, input–output
analysis models are employed. In these models the interaction between major com-
ponents of an economy is analysed so that the effect of an increase in one component
can be measured against the demand for one product in other industries.
Input–output models were first developed by Nobel Prize winner, Wassily
Leontief; they are used to describe economies where demand equals supply, or con-
sumption equals production. These models can be applied to whole economies or to
segments within economies.
The model, as presented here, is based on the idea that there is a finite number of
goods that are produced, consumed or used as input for the same finite number of
industries. Each industry produces only one type of product and can use some of its
own product. Each industry and its products are interdependent. The total demand of
product is the sum of the demands throughout the entire production process as well as
the demand for the product by consumers.
Consider an economy com-
prising only 2 industries — coal
and steel. One tonne of steel
requires an input of 0.5 tonne of
coal as well as 0.4 tonne of steel
(perhaps in the form of
machinery and plant). To pro-
duce 1 tonne of coal, 0.3 tonne of
steel and 0.4 tonne of coal are
required (perhaps to produce the
steel needed for machinery).
Suppose also that the final
demand for steel is 18 tonnes and
for coal 15 tonnes.
Let x1 and x2 denote the total demands for coal and steel respectively. This
information can be presented on an input–output table.

User
Final Total
Producer Coal Steel demand demand

Coal 0.4 0.3 15 x1

Steel 0.5 0.4 18 x2


Chapter 5 Matrices and their applications 207
This information can be represented by a system of simultaneous linear equations:
x1 = 0.4x1 + 0.3x2 + 15
x2 = 0.5x1 + 0.4x2 + 18
and put into matrix form:
x1 x1
= 0.4 0.3 + 15
x2 0.5 0.4 x2 18
which can be represented as
X = AX + D
x1
where X = , A = 0.4 0.3 and D = 15 .
x2 0.5 0.4 18
Rearrangement of the matrices (as shown below) isolates X so it can be solved.
X − AX = D
(I − A) X = D

where X post-multiplies other factors and I is the identity matrix 1 0 .


0 1
(I − A)–1(I − A)X = (I − A)–1D
X = (I − A)–1D
We call X the output matrix as it holds variables x1 and x2 that will state the total
demand for steel and coal. We call A the technology matrix and D the final demand
matrix. Because of its significance I − A is called the Leontief matrix.
To calculate X we first need to find I − A.

I − A = 1 0 − 0.4 0.3
0 1 0.5 0.4

I−A= 0.6 – 0.3


– 0.5 0.6
–1
x1
Therefore = 0.6 – 0.3 15
x2 – 0.5 0.6 18

1
= --------------------------- 0.6 0.3 15
0.36 – 0.15 0.5 0.6 18

1
= ---------- 14.4
0.21 18.3

= 68.57
87.14
Thus, in order to provide the final demand of 18 tonnes of steel and 15 tonnes of coal
in this economy, the steel industry must produce 87.14 tonnes of steel and the coal
industry must produce 68.57 tonnes of coal.
These values can also be regarded as equilibrium values — if these values are met
then the input will match the output — thus eliminating both over- and under-
production.
208 M a t h s Q u e s t M a t h s C Ye a r 1 1 f o r Q u e e n s l a n d

remember
remember
1. A pair of equations, ax + by = u and cx + dy = v, may be written in the form

AX = B where A = a b , X = x and B = u .
c d y v
2. The matrix equation AX = B can be rearranged to X = A−1B so that values for x
and y may be found. These values can also be called equilibrium values.
3. Matrices can be used for input–output analysis.
4. An input–output analysis matrix can be written as X = AX + D where:
matrix X contains the variables to be determined and is called the output matrix
matrix A contains information about the input details and is called the
technology matrix
matrix D contains information about the final demand and is called the final
demand matrix.
5. The matrix equation X = AX + D can be rearranged to make X the subject:
X = (I − A)–1D

Inverse matrices and systems


5A of linear equations
1 The equilibrium state for 2 commodities is given by
2a − 3b = 25
3a − 2b = 60.
26
Find the equilibrium values for these goods.
9
2 Find the equilibrium values of P and Q in
P = Q +Y
42
Q = X + yP where Y = $6 million, X = $15 million and y = 0.5.
36
3 Find the equilibrium values of
x = 45 − 2y 10.38
x = 0.6y. 17.31
400
370
4 Consider the simplified Keynesian system
Y = C + 30
C = 50 + 0.8Y
where Y denotes national income and C denotes consumption. Find the equilibrium
values for Y and C.
5 Find the equilibrium values for the supply and demand model
x = 21 − 4y (demand) 17
x = 14 + 3y (supply). 1
6 The technology matrix for an economy which produces only 2 commodities is given by
x 100.29
A = 0.2 0.6 and the final demand matrix D = 25 . Solve X = 1 .
0.1 0.5 36 x2 92.06
Chapter 5 Matrices and their applications 209
7 A simple economy is shown in the table.

User
Final
Producer p q demand Total
p 0.25 0.4 30 x1
q 0.30 0.5 25 x2
98.04
Find the total demand matrix.
108.82
8 A certain economy consists of 2 industries, mining ore and manufacturing. The pro-
duction of 1K dollars (K = 1000) of manufactured goods requires 0.6K dollars of
manufactured goods and 0.15K dollars of ore, while the production of 1K dollars of ore
requires 0.4K dollars of manufactured goods and 0.3K dollars of ore.
The final demand for manufacturing and mining ore is 120K dollars and 145K
dollars respectively.
a Prepare a matrix table for this information.
b Represent the information in matrix form, using X as the output matrix, A the
3.18 0.681
technology matrix and D the demand matrix.
c Find (I − A)–1.
1.81 1.81 480.68
d Find the total demand matrix (output matrix) X.
481.82
e Verify that D = (I − A) X.

Gaussian elimination
SLE 1: Solve linear The example based on the economy with only 2 industries used in the previous section
equations by using matrices
and Gaussian elimination; is obviously a little unrealistic. However, if this economy were to involve more than
solution of equations 2 industries we have no tools to solve this type of problem, at this stage. This is because
involving more than three
variables will involve the it would involve finding the inverse of a 3 × 3 matrix using the example of an economy
use of graphing calculators. with 3 industries, whereas we are limited to finding the inverse of a 2 × 2 matrix.
A variety of methods can be used to solve a system of simultaneous equations, such
as those generated in the previous section. We will concentrate at this stage on the
method known as Gaussian elimination. In future sections of this chapter we will
introduce 2 more methods, as well as graphics calculator techniques for solving
simultaneous equations.

Using the Gaussian elimination method to solve


simultaneous equations
The Gaussian elimination method is described below.
Consider a set of 2 simultaneous linear equations in 2 unknowns, x and y as given:
ax + by = u
cx + dy = v
As we saw earlier, this system can be represented as

A= a b B= u X= x
c d v y
where A is the coefficient matrix, B is the constant matrix and X is the variable or
unknown matrix.
210 M a t h s Q u e s t M a t h s C Ye a r 1 1 f o r Q u e e n s l a n d

We can combine matrices A and B as a b u where this is referred to as an


augmented matrix, denoted by [A | B]. c d v

The following example shows the familiar algebraic process involved in solving
simultaneous equations, only this time it is in matrix form. These steps will work
towards producing the identity matrix I on the left-hand side and then the solution will
be on the right-hand side.
[A | B] → [I | X]

WORKED Example 1
Solve this system of simultaneous linear equations:
x + 2y = 3
2x + 3y = 5
THINK WRITE

1 Set equations in matrix form. A= 1 2 B= 3 X= x


2 3 5 y

2 Convert to augmented matrix form. [A | B] = 1 2 3


2 3 5

3 Aim to produce the identity matrix on 1 2 3


the left-hand side of the augmented
R2 − 2R1: 0 – 1 – 1
matrix. Element a11 is already 1. We
need to eliminate the a21 element to
produce a 0.
Replace R2 with R2 − 2R1. That is,
replace row 2 with row 2 minus twice
row 1.
4 Next we want a22 to be 1. Multiply R2 1 2 3
by −1.
−R2: 0 1 1

5 Finally, we want to make a12 = 0. R1 − 2R2: 1 0 1


Replace R1 with R1 − 2R2. 0 1 1

6 [A | B] now resembles [I | X] and X = 1 . [I | X] = 1 0 1


1 0 1 1

X = 1 , that is x = 1 and y = 1
1
7 Verify your answers by substitution into Check:
the original equations. 1 + 2(1) = 3 (true)
2(1) + 3(1) = 5 (true)

All the communication given to the left of each matrix should be included in your own
work as justification for how you have proceeded from one step to the other, and also
for your own reference.
Chapter 5 Matrices and their applications 211
The following procedures are used in this method (they match operations you use
when solving equations by elimination):
1. multiply a row (equation) by a constant
2. add and subtract one row (equation) to or from another
3. swap rows (equations).
The object is to have ones in the leading diagonal and zeros under the leading diagonal.
This is known as row-echelon form. Once this is achieved, the solution can be found
either algebraically or by continuing with Gaussian elimination until all elements above
the leading diagonal are zero. This is known as reduced row-echelon form.
Let’s investigate these ideas by working with a 3 × 3 matrix.

WORKED Example 2
Solve this system of linear equations.
x + 2y + z = 8
x−y+z=7
x + y + 2z = 4
THINK WRITE
1 2 1 8 x
1 Set the equations in matrix form. A = 1 –1 1 B= 7 X= y
1 1 2 4 z

1 2 1 8
2 Convert to augmented matrix
[A | B] = 1 – 1 1 7
form.
1 1 2 4

3 Element a11 is already 1. 1 2 1 8


Eliminate a21 (make it become 0) R2 – R1: 0 – 3 0 – 1
by replacing R2 with R2 – R1.
1 1 2 4

1 2 1 8
4 Next eliminate a31 by replacing
R3 with R3 − R1. 0 –3 0 –1
R3 – R1: 0 – 1 1 – 4

5 To obtain a 1 in a22, we could 1 2 1 8


divide R2 by −3 (giving us 1--- in Swap R2 and R3: 0 – 1 1 – 4
3
b21) or swap R2 and R3. Let’s 0 –3 0 –1
swap the rows and then multiply 1 2 1 8
the new R2 by −1. – R2: 0 1 – 1 4
0 –3 0 –1

6 Eliminate a32 by replacing R3 1 2 1 8


with R3 + 3R2. 0 1 –1 4
R3 + 3R2: 0 0 – 3 – 11 Continued
over page
212 M a t h s Q u e s t M a t h s C Ye a r 1 1 f o r Q u e e n s l a n d

THINK WRITE

7 To obtain a 1 in a33, multiply R3 1 2 1 8


by − 1--- .
3 0 1 –1 4
− 1--- R3: 0 0 1 – 11
------
3
3

8 Row-echelon form has now been


achieved. We can solve for x, y
and z using equations or continue
to reduced row-echelon form.
Using equations
R3 gives z. z = − 11
------
3
Substituting the value for z in R2 Substituting z = − 11
------ in y – z = 4 gives y = 1
--- .
3 3
gives y.
Substituting the values for both y Substituting y = 1
--- and z = − 11
------ in x + 2y + z = 8 gives
3 3
and z in R1 gives x. x = 11.
Continuing to reduced Alternatively:
row-echelon form
1 2 1 8
We first need to eliminate above
a33. Eliminate a23 by replacing R2 R2 + R3: 0 1 0 1
---
3
with R2 + R3.
0 0 1 – 11
------
3

Eliminate a13 by replacing R1 R1 − R3: 1 2 0 35


------
3
with R1 – R3. 1
0 1 0 ---
3

0 0 1 – 11
------
3

Now eliminate above a22. R1 − 2R2: 1 0 0 11


Eliminate a12 by replacing R1 0 1 0 1
---
with R1 − 2R2. 3

0 0 1 – 11
------
3

11
1
9 State the solution. Therefore, X = ---
3
where x = 11, y = 1--- , z = − 11
------ .
3 3
– 11
------
3

10 Verify your results by substitution Check: substituting into x + y + 2z gives


into at least one of the original 11 + 1--- + 2(− 11
------ )
3 3
equations. In this case, substitute
= 11 + 1
--- − 22
------
values for x, y and z into the left 3 3
side of the third equation. =4
Solution is verified.
Chapter 5 Matrices and their applications 213
This method can be quite frustrating if you ‘get lost’. Work on only one row at a time
in any one step, unless the operation is quite straightforward and the line in question
isn’t involved in any other procedures in that step. (For example, Step 5 in Worked
example 2 would satisfy these criteria.) Use the following steps:
1. Start with a 1 in position a11, by swapping rows, dividing by a suitable number or both.
2. Eliminate the elements (make them become 0) below a11, starting with a21 and
working down.
3. Work to get a 1 in position a22 by either swapping rows, dividing by a suitable
number or both.
4. Eliminate the elements below a22, starting with a32 and working down.
5. Continue until there are ones in the leading diagonal and zeros below it (row-
echelon form).
6. Once you have row-echelon form, decide if you are going to work algebraically or
continue to reduced row-echelon form.
7. If you are working to reduced row-echelon form, start with the far right-hand
column and work up and to the left.

Using the Gaussian elimination method to find an inverse


In the previous section we used an augmented matrix [A | B] to find solutions for a
system of linear equations.
The Gaussian elimination method can also be used to find the inverse of matrix A.
This is achieved by augmenting A with I — as in [A | I] and performing row reduction
procedures to produce [I | A–1]. This is shown in the following example.

WORKED Example 3
Find the inverse of A = 1 1 .
1 2
THINK WRITE

1 Set up the augmented matrix [A | I]. [A | I] = 1 1 1 0


1 2 0 1
2 Perform Gaussian elimination to obtain the 1 1 1 0
identity matrix on the left side of the
R2 – R1: 0 1 – 1 1
augmented matrix. Element a11 is already 1.
Eliminate a21 by replacing R2 with R2 – R1.
3 Element a22 is already 1. Next eliminate a12 R1 – R2: 1 0 2 – 1
by replacing R1 with R1 – R2. 0 1 –1 1

This augmented matrix is now in the form 1 0 2 – 1 = [I | A−1]


4
[I | A−1]. 0 1 –1 1

A−1 = 2 –1
–1 1

5 Check this is so by verifying A−1A = I. Check:


A−1A = 2 –1 1 1 = 1 0 = I
–1 1 1 2 0 1
214 M a t h s Q u e s t M a t h s C Ye a r 1 1 f o r Q u e e n s l a n d

WORKED Example 4
1 2 1
Find A–1 (if it exists) for A = 1 0 1 .
0 1 3

THINK WRITE

1 Set up the augmented matrix. 1 2 1 1 0 0


[A | I] = 1 0 1 0 1 0
0 1 3 0 0 1

2 Perform Gaussian elimination to 1 2 1 1 0 0


obtain the identity matrix on the
R2 − R1: 0 – 2 0 – 1 1 0
left side of the augmented matrix.
Element a11 is already 1. 0 1 3 0 0 1
Eliminate a21 by replacing R2
with R2 − R1. Note that a31 is
already 0.
3 We need a 1 in a22. The easiest 1 2 1 1 0 0
way is to swap R2 and R3.
Swap R2 and R3: 0 1 3 0 0 1
0 –2 0 –1 1 0

4 Eliminate a32 by replacing R3 1 2 1 1 0 0


with R3 + 2R2. 0 1 3 0 0 1
R3 + 2R2: 0 0 6 – 1 1 2

5 Make a33 = 1 by dividing R3 by 6. 1 2 1 1 0 0


0 1 3 0 0 1
1 1
1
--- R3: 0 0 1 – 1--6- ---
6
---
3
6

6 Row-echelon form has now been


R1 − R3: 1 2 0 7---
– --61- – 1--3-
achieved. We now need to 6
eliminate above a33. Replace R1 R2 − 3R3: 0 1 0 1
--- – 1--2- 0
2
with R1 − R3 and replace R2 with
1 1
R2 − 3R3. 0 0 1 – 1--6- ---
6
---
3

7 Now eliminate above a22. R1 − 2R2: 1 0 0 1


---
6
5
---
6
– --13-
Replace R1 with R1 − 2R2. 1
0 1 0 ---
2
– --12- 0
1 1
0 0 1 – --16- ---
6
---
3
Chapter 5 Matrices and their applications 215
THINK WRITE
1 5
1 0 0 ---
6
---
6
– 1--3-
8 This is now in the form [I | A−1]. 0 1 0 1
--- – 1--2-
−1
0 = [I | A ]
State A−1.
2
1 1
0 0 1 – 1--6- ---
6
---
3

1 5
---
6
---
6
– 1--3-
A−1 = 1
--- – 1--2- 0
2
1 1
– 1--6- ---
6
---
3

9 Verify that A−1A = I. Check:


1 5
--- --- – 1--3-
6 6 1 2 1
A−1A = 1
--- – 1--2- 0 1 0 1
2
1 1 0 1 3
– 1--6- ---
6
---
3

1 0 0
= 0 1 0
0 0 1
=I

remember
remember
For a set of simultaneous equations represented by matrices A, X, B such that
AX = B, Gaussian elimination can be used to change an augmented matrix [A | X]
to [I | X] and [A | I] to [I | A–1] where I is the identity matrix.

5B Gaussian elimination
Use the Gaussian method of elimination to solve the following systems of linear equations.
WORKED 1 2x − y = 1 x = 1, y = 1 2 3a + 2b = −11 a = −3, b = −1 eBook plus
Example
1 3x + 2y = 5 a + 3b = −6
Digital doc:
WORKED 3 2y − z = 5 x = 2, y = 1, z = −3 4 3x + 4y − z = 11 x = 1, y = 2, z = 0 SkillSHEET 5.1
Using Gaussian
Example
2 x + z = −1 x − y + 2z = −1 elimination to solve
linear equations
2x − y − z = 6 5x + y − 3z = 7

5 3x + 4y + z = −10 x = −2, y = −1, z = 0 6 x + y − z = 6 x = −3, y = 5 1--4- , z = −3 3--4-


2x + y + 2z = −5 x + 2y + 2z = 0
x − 2y + 2z = 0 −2x − y + z = −3
216 M a t h s Q u e s t M a t h s C Ye a r 1 1 f o r Q u e e n s l a n d

7 Use Gaussian elimination to find the inverses of the following matrices, if they exist.
WORKED 1
Example
xample 1 2 –3 2 3 1 --- – --14-
3 a b 2
3---
2 3 2 –1 2 2 – 1--2- 4

1---
WORKED 1 2 2 2 1--3- – 2--3- – 1 1--3- 2 1 0 2
– 1--4- 0
Example
xample
c 0 1 3 –1 0 1 d 0 2 0 0 --1- 0
4 2

1 3 2 --1- --1- – --13- 2 2 1 – 1 – 1--2- 1


3 3

8 Write each of the systems of linear equations below in the matrix form AX = B and find
inverses to solve the equations.
x = 2.7, y = 0.6 a 2x + y = 6 b 3x + 2y = 9 c x + y − z = −6
x + 3y = 4 ---
1
x + 4y = 7 2x − y + 2z = 1
2
2x − z = 5 x + 2y = −7
x= 2 1--5- ,y= 1 1--5- , z = − --35- x = −2 1--7- , y = −2 3--7- , z = 1 3--7-

History of mathematics
CARL FRIEDRICH GAUSS (1777–1855)
During his lifetime . . . In 1801, he used the information from
Karl Marx and Friedrich Engels three sightings of an asteroid, Ceres,
jointly publish ‘The Communist to calculate its orbit. In the
Party Manifesto’. process of this work he showed
that the variation involved in
The Frenchman Nicéphore
experimental data followed a
Niepie produces the world’s
bell-shaped curve, now called
first photographic image.
the Gaussian or normal
Samuel Morse develops the distribution. In 1807 Gauss
‘Morse Code’. became professor of astronomy
and director of the new
You will come across the work of observatory in Göttingen. His
Carl Gauss in many fields of work involved branches of
mathematics. His work is so diverse astronomy, mechanics, optics, geodesy
that he is considered by many to be the and magnetism, and in collaboration with
greatest mathematician of all times. Gauss was Weber, the first practical telegraph. His
born in Brunswick, Germany, on 20 April 1777. extensive use of complex numbers advanced the
From the age of three he had shown his superior acceptance of them by fellow mathematicians,
skills by performing mental calculations and by although he was generally not supportive of
the age of ten had progressed to algebra and young, aspiring scholars.
analysis. While still a teenager he had He died in Göttingen in 1855. His memorial
developed the ‘least squares’ method used in bears the 17-point star of his early fame.
statistical data, and had devised a proof that a
regular 17-sided polygon could be constructed Questions
using a compass and ruler and his quadratic 1. Try to reproduce the 17-point star with
reciprocity theorem. At the age of 22 he all angles and sides the same.
received his PhD, proving the Fundamental 2. Research the Fundamental Theorem of
Theorem of Algebra. The next year he Algebra. Science of determining the
published his work on number theory, 3. What is geodesy? size and shape of the Earth.
organising previous work and introducing the 4. Find out about the quadratic reciprocity
notion of modular arithmetic. theorem.
Chapter 5 Matrices and their applications 217

Performing Gaussian elimination


using a graphics calculator
Matrix row operations can be performed on your graphics calculator. To demonstrate
this, we will repeat the steps of Worked example 4 to find the inverse of
1 2 1
A= 1 0 1 .
0 1 3

For the Casio fx-9860G AU


1. Enter Matrix A.
(a) Press F1 ( MAT) to enter the matrix

editing screen.
(b) Highlight Mat A and press EXE or F3
(DIM).
(c) Specify the number of rows, 3 in this case,
and press EXE .
(d) Specify the number of columns, 3 in this
case, and press EXE .
(e) Press EXE again to display the 3 × 3 array
for matrix A.
(f) Enter the values for the elements, pressing
EXE after each number.

2. Exit the Matrix input screen by pressing EXIT .


Press EXIT again to return to the MAT screen.

3. Set up an augmented matrix B = [A | I]. Enter the


augmented matrix [A | I] first and then save as
matrix B.
(a) Press OPTN then F2 (MAT) to bring up the
matrix menu. Press F5 (Aug), then F1
(Mat), then ALPHA [A] followed by ,
s

then press F6 ( ) and F1 (Iden). Enter 3


(for a 3 × 3 identity matrix) and press ) to
close the set of brackets.
(b) Press Æ then F6 ( ) then F1 (Mat) and
s

ALPHA [B] to save the augmented matrix as


matrix B.
(c) Press EXE . Matrix B will be displayed. You
can use the cursor keys to see more of the
augmented matrix.
218 M a t h s Q u e s t M a t h s C Ye a r 1 1 f o r Q u e e n s l a n d

4. To perform row operations, we need to be in the


matrix editing screen.

(a) Press EXIT three times to return to the


MAT screen as seen at right.

(b) Press F1 ( MAT). Highlight Mat B and


press EXE . Notice that this screen is
different from the one obtained in Step 3.

5. Replace R2 with R2 − R1 (or − R1 + R2).


(a) Press F1 (R-OP) for the Row Operation
menu.
(b) Press F3 (×Rw+). This allows the addition
of one row and the product of a specified
row with a scalar to be found.
(c) Input the scalar multiplier. Enter −1 for k
and press EXE .
(d) Specify the number of the row to be
multiplied by the scalar. Enter 1 for m and
press EXE .
(e) Specify the number of the row that the result
should be added to. Enter 2 for n and press
EXE .
(f) Press EXE . Compare this screen with the
matrix obtained in Step 2 of Worked
example 4.

6. Swap R2 and R3.


(a) Press F1 (SWAP).
(b) Specify the number of the rows to be
swapped. Enter 2 for m and press EXE .
Enter 3 for n and press EXE .
(c) Press EXE . Compare this screen with the
matrix obtained in Step 3 of Worked
example 4.
Chapter 5 Matrices and their applications 219
7. Replace R3 with R3 + 2R2.
(a) Press F3 (×Rw+). As before, this allows
addition of one row and the product of a
specified row with a scalar.
(b) Enter 2 for k (this is the scalar multiplier)
and press EXE . Enter 2 for m (this is the
row to be multiplied) and press EXE .
Enter 3 for n (this is the number of the row
where the result should be added) and
press EXE .
(c) Press EXE . Compare this screen with the
matrix obtained in Step 4 of Worked
example 4.

8. Multiply R3 by 1--- .
6
(a) Press F2 (×Rw). This allows the product of
a specified row and scalar to be found.
(b) Using the fraction key, enter 1--- for k (this is
6
the scalar multiplier) and press EXE . Enter
3 for m (this is the row to be multiplied) and
press EXE .
(c) Press EXE . Compare this screen with the
matrix obtained in Step 5 of Worked
example 4.

9. Now that we have row-echelon form, continue


performing operations until reduced row-
echelon form is achieved.

10. Use the cursor keys to scroll across to see A−1.

Now complete the questions at the end of this


investigation.
220 M a t h s Q u e s t M a t h s C Ye a r 1 1 f o r Q u e e n s l a n d

For the TI-Nspire CAS


1. Enter Matrix A.
(a) Open a new Calculator document. Press k to access the catalog. Select
Option 5 (by pressing 5) then highlight the m-by-n matrix symbol.
(b) Press ·. Create the matrix with Number of
rows: 3 and Number of columns: 3, pressing
e to move from one box to the next.
(c) Highlight OK and press ·. Fill in the values
in the matrix using the cursor keys to move
from one element to the next.
(d) Move the cursor to the right of the matrix.
Press / and h, then press A to store the
matrix as matrix A. Press · to display the
matrix.
2. Set up an augmented matrix B = [A | I]. Enter the
augmented matrix [A | I] first and then save as
matrix B.
(a) Press b and select 7: Matrix & Vector (by
pressing 7 or highlighting 7: Matrix &
Vector and pressing ·). Select 6: Create
and then select 7: Augment.
(b) Press A then press the comma key (,). To
enter the identity matrix, press b and select
7: Matrix & Vector then 6: Create and
2: Identity. Press 3 (for a 3 × 3 identity
matrix) and press ) twice to close the two
sets of brackets. Press / and h, then
press B to save the augmented matrix as
matrix B. Press · to display the matrix.
3. Replace R2 with R2 − R1 (or − R1 + R2).
(a) To proceed to the Row Operations menu,
press b then select 7: Matrix & Vector and
9: Row Operations. For − R1 + R2, select
4: Multiply Row & Add.
(b) Input the scalar multiplier, the matrix name,
the row number to be multiplied and then the
row number for the result to be added to, each
separated by a comma. In this case, enter −1,
B, 1, 2. Press ) to close the set of brackets,
then press /, h and C to save the matrix
as matrix C. (You can continue to save the
new matrix formed as matrix B if you wish or
use a new name after each row operation.)
(c) Press ·. Compare this screen with the matrix
obtained in Step 2 of Worked example 4.
Chapter 5 Matrices and their applications 221
4. Swap R2 and R3.
(a) Proceed to the Row Operations menu and
select 1: Swap Rows.
(b) Input the matrix name and the two row
numbers to be swapped, each separated by a
comma. In this case, enter c, 2, 3.
Press ) to close the set of brackets then
press /, h and D to save the matrix as
matrix D.
(c) Press ·. Compare this screen with the
matrix obtained in Step 3 of Worked
example 4.
5. Replace R3 with R3 + 2R2.
(a) As before, proceed to the Row Operations
menu and select 4: Multiply Row & Add.
(b) Enter 2, D, 2, 3 to represent
2 × R2 + R3 in matrix D. Press ) to close
the set of brackets, then press /, h and E
to save the matrix as matrix E.
(c) Press ·. Compare this screen with the
matrix obtained in Step 4 of Worked
example 4.
6. Multiply R3 by 1--- .
6
(a) Proceed to the Row Operations menu and
select 3: Multiply Row.
(b) Enter 1--- , E, 3 to represent 1--- × R3 in matrix E.
6 6
Press ) to close the set of brackets, then
press /, h and F to save the matrix as
matrix F.
(c) Press ·. Compare this screen with the
matrix obtained in Step 5 of Worked
example 4.
7. Now that we have row-echelon form, continue
performing operations until reduced row-echelon
form is achieved.
1 a x = 2.7, y = 0.6
b x = 2 1--5- , y = 1 1--5- , z = − 3--5-
c x = −2 --17- , y = −2 --37- , z = 1 --37-

QUESTIONS
1 Repeat the calculations for Exercise 5B Question 7 using a graphics calculator.
2 Solve the following system of equations by performing row operations using a
graphics calculator. x = 1, y = 0, z = 3
x − y − 4z = −11
6x + 2y + z = 9
−3x − 2y + 2z = 3
222 M a t h s Q u e s t M a t h s C Ye a r 1 1 f o r Q u e e n s l a n d

Introducing determinants
As mentioned in the previous section, Gaussian elimination is just one method used to
solve systems of linear equations. Other methods involving determinants of matrices
were used as early as 1100 BC by the Chinese and more recently by Gabriel Cramer
(1704–1752) and Augustine Cauchy (1789–1857).
You were introduced to determinants in Chapter 3 on matrices where a quick test to
determine whether a matrix was singular (that is, had no inverse) was to evaluate its
determinant.
1 1
For example, for A = 2 2 A–1 = ------------------------------ 1 – 2 = --- 1 – 2 .
1 1 2 × 1 – 2 × 1 –1 2 0 –1 2

Because the determinant of A = 0, no inverse exists such that A–1A = I.

By definition, if A = [a], then det A = | a | for a 1 × 1 matrix.

Determinant of a 2 ¥ 2 matrix
If A = a b then det A = a b = ad - bc.
c d c d
Therefore the determinant can only be found for a square matrix A and is denoted by
straight brackets, | |, not the square brackets [ ] used for a matrix. Its value is a single
numerical answer, not a table of values like a matrix.

WORKED Example 5
Evaluate the determinant of A = 2 1 .
3 2
THINK WRITE

For A = a b , det A = ad − bc. det A = 2 × 2 − 3 × 1


c d =4−3
=1

Determinant of a 3 ¥ 3 matrix
As with the inverse of matrices, we need to be able to find determinants of matrices
larger than 2 × 2.
The determinant of a 3 × 3 matrix involves evaluating three 2 × 2 determinants.
a b c e f d f d e
If A = d e f , det A = a -b +c
h i g i g h
g h i
where the 3 sub-determinants are referred to as minors.
Therefore det A = a(ei - fh) - b(di - fg) + c(dh - eg)
= aei - afh - bdi + bfg + cdh - ceg
Chapter 5 Matrices and their applications 223
Note that the coefficients of each sub-determinant are the elements of row 1 and the
minor of a11 is formed by using elements not in row 1 or column 1. That is, mentally
cross out the first row and first column.
a b c
d e f and similarly for the other minors.
g h i
The second coefficient is given a negative sign.

WORKED Example 6
2 1 3
Evaluate the determinant 1 –1 2 .
–1 2 0
THINK WRITE
1 Use elements of row 1 as
2 1 3
the coefficients of the
minors. 1 –1 2
–1 2 0

= 2 –1 2 − 1 1 2 + 3 1 –1
2 0 –1 0 –1 2
2 Evaluate minors. = 2(−1 × 0 − 2 × 2) − 1(1 × 0 − 2 × −1) + 3(1 × 2 − −1 × −1)
= 2(0 − 4) − 1(0 + 2) + 3(2 − 1)
= −8 − 2 + 3
= −7

If det A = 0 then the inverse of A does not exist. In this case A is said to be singular.
A special set of simultaneous equations such as
ax + by = 0
cx + dy = 0
is said to be homogeneous, where all the right-hand side constants are zero. The trivial
solution to this system yields x = y = 0.
If det A = 0 then an infinite number of non-trivial solutions exist. However, if
det A ≠ 0 only the trivial solutions exist. You will encounter questions later in this
chapter that deal with this situation.

remember
remember
1. By definition, if A = [a], then det A = | a | for a 1 × 1 matrix.

2. If A = a b then det A = a b = ad − bc.


c d c d
a b c
3. If A = e f −b d f +c d e
d e f , det A = a
h i g i g h
g h i
where the 3 sub-determinants are referred to as minors.
224 M a t h s Q u e s t M a t h s C Ye a r 1 1 f o r Q u e e n s l a n d

5C Introducing determinants
Evaluate the determinants of the following matrices:
1 eBook plus
2 3 5 1 0 1 − 1--2-
--- 1 --3-
WORKED 1 2 2 13 3 4 2
4
Example
xample 1 Digital docs:
0 1 2 3 ---
2
3 1
--- 2
5 4 SkillSHEET 5.2
Determinants of 2 × 2
and 3 × 3 matrices

WORKED 5 Evaluate the following determinants:


Example
xample
6
1 –3 2 –1 –1 0 0 2 –3
a –1 4 3 −5 b –3 4 2 −33 c –3 1 2 81

0 2 5 2 3 5 6 3 2

1 1 1 2 5 –1 0 0 0
d 2 2 3 −3 e 3 –2 4 61 f 2 3 –5 0

0 3 4 –1 –2 –3 6 8 1

1 1 1 1 2 3 2 1 3
g 2 2 2 0 h 0 4 5 24 i 4 0 5 −24

3 3 3 0 0 6 0 0 6

4 10 – 2 eBook plus
j 3 –2 4 122 Digital doc:
–1 –2 –3 WorkSHEET 5.1

Properties of determinants
In question 5 of the previous exercise, review parts a and b, f, g and h. Do you notice
any patterns that you think could occur in other situations? In fact there are 8 properties
of determinants that can be very useful. We have already used one of them (see prop-
erty 6 given on the next page) in using the Gaussian method to solve systems of linear
equations.
The 8 properties of determinants are given below.

Property 1 Determinant of a transpose


For every square matrix A, det A = det A′ where det A′ is the determinant that results
from the transpose of A as seen in questions 5 a and b above.

Property 2 Identical rows


If 2 or more rows (or columns) of a matrix are identical or in proportion, then det A = 0
(this can be seen in question 5 g).
Chapter 5 Matrices and their applications 225
Property 3 Zero rows/columns
If all the elements of a row (or column) are zero, then det A = 0 (see question 5 f).

Property 4 Interchanging rows or columns


If 2 rows (or columns) of A are interchanged to give B, then det A = −det B (see
questions 5 h and i).

Property 5 Multiples of rows/columns


If a row (or column) of matrix A is multiplied by a constant k (where k ≠ 0), to give
matrix B, then det B = k det A (see questions 5 e and j).

Property 6 Adding rows/columns


If a non-zero multiple of a row or column of A is added to another row or column, then
the determinant is unchanged.

Property 7 Zero lower-triangular matrix


If all the elements below the leading diagonal are zero, then det A equals the product of
the elements on the leading diagonal (see question 5 h).

Property 8 Det I
If I is the identity matrix then det I = 1 (this property follows from property 7).

Some of these properties are easier to identify than others. For instance, a matrix with
a zero row or column is readily identified. As well as these properties, there are other
short-cut methods that are very convenient for calculating determinants. The following
expansion is one of these.

Expansion of a 3 × 3 determinant using any row


or column
The initial example of expansion of a 3 × 3 determinant (Worked example 6 on page 223)
used the first row as coefficients of the minors. However, any row or column can be used
— with the following adaptations.
1. A minor can be obtained by blocking out all the elements of the row and column of
the coefficient.
2. Alternating signs are attached to the coefficients of each minor, as shown:

+ − +
− + −
+ − +

Note: The initial example (Worked example 6) used a (−) in front of the second
coefficient because the first row elements were used as the coefficients of expansion.
226 M a t h s Q u e s t M a t h s C Ye a r 1 1 f o r Q u e e n s l a n d

If the elements of the second row had been used as the coefficients of expansion, then
the signs on the minors would have been −, +, −. The same would have happened if
the second column had been used. The signed minors are called cofactors.

Follow the next worked example to see how this alternative row or column can be
used.

WORKED Example 7
1 2 1
Evaluate 1 3 6 .
1 4 9

THINK WRITE
1 Because we can expand by any row or
column, use column 1 as it will have
(+)
coefficients of 1. 1 2 1
(a) With a different colour pen (–)
1 3 6
write in the signs of the (+)
elements. Don’t get them 1 4 9
confused and think a21 = −1.
(b) Draw an arrow to indicate the
row/column of expansion.

2 Write out the expansion with the =1 3 6 −1 2 1 +1 2 1


signed minors (cofactors). 4 9 4 9 3 6

3 Complete the expansion. Take care = 1(27 − 24) − 1(18 − 4) + 1(12 − 3)


with the minus sign and the brackets. = 3 − 14 + 9
= −2

In the example above, any row or column could have been used, but you can see that if
column 1 is used, the coefficients will be 1 — a number that is easy to multiply by.
Also, any row or column with mostly zeros allows you to complete the expansion
faster, so it is useful to use that row or column to expand by.
For square determinants greater than 3 × 3, this simple alternating pattern of signs is
continued.
For example, a 4 × 4 determinant can be evaluated using alternating signs of the
coefficients. The signs to be used in front of the coefficients are shown below.

+ – + –
– + – +
+ – + –
– + – +
Chapter 5 Matrices and their applications 227
remember
remember
All determinants have the following properties:
1. For every square matrix A, det A = det A′ where det A′ is the determinant that
results from the transpose of A.
2. If 2 or more rows (or columns) of a matrix are identical or in proportion, then
det A = 0.
3. If all the elements of a row (or column) are zero, then det A = 0.
4. If 2 rows (or columns) of A are interchanged to give B, then det A = −det B.
5. If a row (or column) of matrix A is multiplied by a constant k (where k ≠ 0), to
give matrix B, then det B = k det A.
6. If a non-zero multiple of a row or column of A is added to another row or
column, then the determinant is unchanged.
7. If all the elements below the leading diagonal are zero, then det A equals the
product of the elements on the leading diagonal.
8. If I = identity matrix then det I = 1.

5D Properties of determinants
Evaluate the following determinants:

1 a 1 2 −3 b 1 4 −3 c State the property involved. Property 1


4 5 2 5

1 2 1
WORKED 2 a 3 6 0 0 b State the property involved. Property 2
Example
7 1 2 1

0 0 4
3 a 2 0 5 0 b State the property involved. Property 3

3 0 –2

2 3 6 3 2 6
4 a 1 –1 1 22 b –1 1 1 −22 c State the property involved. Property 4

2 1 0 1 2 0

5 a 2 5 −18 b 4 10 −36 c State the property involved. Property 5


4 1 4 1

6 a 2 4 2 b 2 4 2 c State the property involved. Property 6


0 1 2 5

2 –1 3
7 a 0 4 7 8 b State the property involved. Property 7

0 0 1
228 M a t h s Q u e s t M a t h s C Ye a r 1 1 f o r Q u e e n s l a n d

1 0 0
8 a 0 1 0 1 b State the property involved. Property 8

0 0 1

9 Use any of the properties investigated in earlier sections to evaluate the following
determinants:

2 3 4 4 3 1 1 1 –1
a –1 1 1 −6 b –1 6 1 −4 c 0 1 1 3

0 1 0 2 5 1 2 1 0

2 1 1 3 4 2 1 –0
1 0 0
d −8 e 1 0 0 1 3 f 0 2 –1 1 40
–1 0 4
–1 0 0 2 0 –0 1 3
3 2 –1
2 0 1 3 0 0 0 5

Inverse of a 3 × 3 matrix
In Chapter 3 on matrices you were introduced to the idea of an inverse matrix A–1
where AA–1 = A–1A = I.

a b 1
For A = the inverse matrix A–1 = ------------------ d – b , where ad - bc ≠ 0 and
c d ad – bc – c a
ad - bc = det A.

This rule is limited to 2 × 2 matrices. However, in its most general form it can be used
to find the inverse of any square matrix, if the inverse exists.
The steps below demonstrate how the above formula can be used to find the inverse
of a 3 × 3 matrix.

2 3 1
For matrix A = 4 6 5
9 7 8

1. Matrix C, the cofactor matrix of A, is obtained by replacing each element of A with


its corresponding cofactor or signed minor.

6 5 – 4 5 4 6
7 8 9 8 9 7
13 13 – 26
C= – 3 1 2 1 – 2 3 = – 17 7 13
7 8 9 8 9 7 9 –6 0
3 1 – 2 1 2 3
6 5 4 5 4 6
Chapter 5 Matrices and their applications 229
2. The adjoint of A is the transpose of the cofactor matrix and is written adj A.

13 – 17 9
adj A = 13 7 –6
– 26 13 0

The adjoint matrix has the property that A(adj A) = (det A)I and since det A is a
scalar we can divide by det A to produce
adj A
A -------------- = I
det A
therefore
adj A-
------------- = A–1.
det A

You can verify this result by showing A–1A = I.


In its simplest form, this is the formula that was used to find the inverse of 2 × 2
matrices.

WORKED Example 8
1 –1 2
Use the cofactor/adjoint matrices to find the inverse of A = 0 1 2 .
0 1 3

THINK WRITE

1 2 – 0 2 0 1
1 3 0 3 0 1

1 Set up the cofactor matrix using signed C= – –1 2 1 2 – 1 –1


minors and evaluate. 1 3 0 3 0 1

–1 2 – 1 2 1 –1
1 2 0 2 0 1

1 0 0
C= 5 3 –1
–4 –2 1

1 5 –4
2 Write the adjoint as the transpose of C. adj A = 0 3 – 2
0 –1 1

Continued over page


230 M a t h s Q u e s t M a t h s C Ye a r 1 1 f o r Q u e e n s l a n d

THINK WRITE

3 Calculate det A. det A = 1 1 2 − 0 + 0 = 1 (down column 1)


1 3
adj A
4 Set up A−1. A−1 = -------------
det A

1 5 –4
5 Calculate A−1. A−1 = 0 3 – 2
0 –1 1
6 Verify this matrix is A−1 by testing Check:
AA−1 = I.
1 –1 2 1 5 –4
AA−1 = 0 1 2 0 3 –2
0 1 3 0 –1 1

1 0 0
AA−1 = 0 1 0
0 0 1
AA−1 = I

This method can now be used to solve systems of linear equations involving a 3 × 3
matrix.

WORKED Example 9
Solve the system of linear equations given below.
x−y−z=0
2x + y = 4
x+y+z=2

THINK WRITE

1 –1 –1
1 Set up the matrices in the form AX = B where A = 2 1 0
AX = B.
1 1 1

x 0
X= y and B = 4
z 2

2 Rearrange to change X to be A−1AX = A−1B


the subject. X = A−1B
Chapter 5 Matrices and their applications 231
THINK WRITE

1 0 – 2 0 2 1
1 1 1 1 1 1

3 Use the cofactor matrix to find C= – –1 –1 1 –1 – 1 –1


A−1. 1 1 1 1 1 1

–1 –1 – 1 –1 1 –1
1 0 2 0 2 1

1 –2 1
C = 0 2 –2
1 –2 3

1 0 1
4 Find the adjoint of A. adj A = – 2 2 – 2
1 –2 3
5 Find the determinant of A.
det A = 1 – 1 – 1 –1 1 – 1 +1 1 – 1 (across row 3)
1 0 2 0 2 1
det A = 1 × 1 − 1 × 2 + 1 × 3
det A = 2

adj A
6 Find the inverse. A−1 = -------------
det A
1 0 1
1
A−1 = --- – 2 2 – 2
2
1 –2 3

1 1
---
2
0 ---
2
−1
A = –1 1 –1
1
---
2
–1 1 1--2-

1 1
---
2
0 ---
2 0
7 Solve for X and check the X = –1 1 –1 4
values provided. 1
---
2
–1 1 1--2- 2

1
X= 2
–1
So x = 1, y = 2, z = −1.
232 M a t h s Q u e s t M a t h s C Ye a r 1 1 f o r Q u e e n s l a n d

remember
remember
1 1. The determinant of a 3 × 3 matrix can be found using signed minors. The signs
a i 1 ii 3 –2
on the minors can be determined from the following:
–1 1
+ − +
iii 3 – 1 iv 3 – 1
–2 1 –2 1 − + −
+ − +
b i −12 ii 9 –6
–5 2 adj A
2. A–1 = ------------- where adj A = C′, the transpose of the cofactor matrix of signed
5 det A
9 –5 – 3--4- ------
12
iii iv minors.
--1-
1 – 1 1--2- 1
–6 2
2
– --16- 8 2 –4 8 – 12 8
1 c i 8 ii – 12 2 8 iii 2 2 –2 iv --1- --1- – --14-
4 4
8 –2 –4 –4 8 –4 – 1--2- 1 – 1--2-

5E Inverse of a 3 × 3 matrix
WORKED 1 Find the i determinant, ii cofactor matrix iii adjoint matrix and iv inverse of each of
Example
xample
8
the following.

1 2 1 –1 4 0
a 1 1 b 2 5 c d
eBook plus 2 0 4 2 3 –1
2 3 6 9
Digital doc:
3 –2 5 –1 4 3
SkillSHEET 5.3
Using the cofactor/adjoint
method
2 Find the inverse of the following matrices, if they exist.

2 1 1 4 1 3 3 5 –4 –1 2 2
a 3 –3 2 b 1 –1 1 c 2 –1 3 d –3 6 1
2 0 1 0 2 0 3 2 –2 2 0 5

1 2 –1
WORKED 3 Find the inverse of 0 3 1 and use it to solve the system of linear equations given
Example
xample
9 2 0 –2 1 d i −33
by x + 2y – z = 0 13 – 5 11
–3 –1 5 – 1.5 1 1.25 19
2 a 1 0 –1 3y + z = 9 0.5 0 – 0.25 X = –2
ii – 12 – 3 0
6 2 –9 2x − 2z = 8 – 1.5 1 0.75 – 4 – 1 – 11
15
13 – 12 – 4
1 –3 –2 iii
b 0 0 0.5 4 Solve the following systems of linear equations. –5 –3 –1
11 0 – 11
– 1 4 2.5 a x+y−z=9 b x − y − 4z = −11 1
2 12 4-
– 13
------ ------ -----
– 0.16 0.08 0.44 −2x − y + z = −11 3 6x + 2y + z = 9 0 33 33 33

c 0.52 0.24 – 0.68 3 iv 5-


----- 1-
----- 1-
-----
–4 33 11 33
x + 2y + 2z = 0 −3x − 2y + 2z = 3 1
0.28 0.36 – 0.52 – 1--3- 0 ---
3

– 1.5 0.5 0.5 2+x 1


d – 0.85 0.45 0.25 5 Find x if = 0. x=± 3
1 2–x
0.6 – 0.2 0
6 a (y − z)(v − u)
b
c
xy2 − xz − x2y + yz2 + x2 − y2z
(1 − x)(1 − y) + y + x − 1
Chapter 5 Matrices and their applications 233
d 2a3 − 3a2b + b3
6 Evaluate the following:

x y z 1 x yz 1 1 1 b a a
a u 1 1 b 1 y x c 1 1–x 1 d a b a
v 1 1 1 z xy 1 1 1–y a a b

7 Solve each of the following equations:


2
1 a a 1 a a
=0 a a
a a = 1, 2 b a 1 0 = 0 a= ± c =0 a = 0, 3
1---
1 1 1 2
2 a–1
1 2 4 a 0 1

a 0 0 x–3 0 0
a = 0, 1, 2 d =0 e x–1 –3 = 0 f =0 x = −1, 3
0 a–1 0 0 x 3
2 x–6
0 0 a–2 0 1 x–2
x = 3, 4
1--- 5---
3 0 5 3 6
– 1 2--3-
8 If A = 2 4 6 find A–1. – 1--6- 7-
----- – 2--3-
12
1 2 4 0 – 1--2- 1

2 1 –1
9 A = 7 –9 3
2 4 5
0.3 0.05 0.03
a Find det A. −189 0.15 – 0.06 0.07
b Find A–1. – 0.24 0.03 0.13
c Use the result from b to solve the system:
2x + y − z = 2 --1-
7x − 9y + 3z = −5 1--- 2
2 1
2x + 4y + 5z = 5
0
–1
1 a 0 1 –a 0
10 Show that 0 1 0 = 0 1 0 . Check with your teacher.
0 b 1 0 –b 1

1 2 1
11 If A–1 = 0 1 2

0.25 – 0.5 0.75 1 0 1


A= 0.5 0 – 0.5 a find A and A′ 0.25 0.5 – 0.25
– 0.25 0.5 0.25 b verify that (A′)–1 = (A–1)′. A′ = – 0.5 0 0.5
0.75 – 0.5 0.25
12 Evaluate for complex i:

1 i 0 1–i i 1+i
a 1 i 1+i b 0 c − 4 − 7i
–i i 1 1+i 1–i i
i–1 0
1 i 0 i 1+i 1–i
234 M a t h s Q u e s t M a t h s C Ye a r 1 1 f o r Q u e e n s l a n d

13 Verify that det (AB) = det A × det B for the following:

1 2 1 2 1 0
a A= 0 2 4 B = 3 2 –1
4 3 1 1 4 5
Check with your teacher.
5 1 –1 3 0 2
b A = 1 –3 2 B = 4 1 –2
2 0 1 0 –1 1

Cramer’s Rule for solving linear


equations
Determinants on their own can also be used to solve systems of linear equations. In
fact, determinants were first studied in this context. This method of solving linear
equations is known as Cramer’s Rule and will be used to solve systems of 2 linear
equations.

For the equations ax + by = u


and cx + dy = v

u b a u
v d c v
x = ----------------------- and y = ----------------------- provided ad - bc ≠ 0.
a b a b
c d c d

Note the pattern of elements in the determinant of the numerators. Mathematics isn’t
referred to as the study of patterns without good reason.
The proof of this statement follows.
Let the system of linear equations be ax + by = u
and cx + dy = v.

Written in matrix form they appear as a b x = u .


c d y v

In matrix equation form, this is AX = B


X = A–1B.

x 1
= ------------------ d – b u
y ad – bc – c a v

1 du – bv
= ------------------
ad – bc – cu + av
Chapter 5 Matrices and their applications 235
du – bv – cu + av
Therefore, x = ------------------ and y = ---------------------- where ad − bc ≠ 0.
ad – bc ad – bc
The numerators and denominators of both these expressions can be written as
determinants:

u b a u
v d c v
x = ----------------------- and y = ----------------------- .
a b a b
c d c d

WORKED Example 10
Use Cramer’s Rule to solve the following linear equations.
2x + 2y = 3
x − y = 1--2-

THINK WRITE

Write the general equations in matrix For a b x = u


1
form and apply Cramer’s Rule. c d y v

u b a u
v d c v
x = ----------------------- and y = -----------------------
a b a b
c d c d

2 2 x 3
2 Substitute the given values for a, b, c, d =
1
and u, v. 1 –1 y ---
2

3 2 2 3
1
---
2
–1 1 1--2-
x = -------------------------- and y = -------------------------
-
2 2 2 2
1 –1 1 –1

–4 –2
= ------ = ------
–4 –4
=1 = --1-
2

3 Verify these results by substitution into Check:


the original system of equations. 2(1) + 2( 1--- ) = 3 (Verified for the 1st equation)
2
1− 1
--- = 1
--- (Verified for the 2nd equation)
2 2
236 M a t h s Q u e s t M a t h s C Ye a r 1 1 f o r Q u e e n s l a n d

Cramer’s Rule can readily be extended to find the solution of 3 equations in


3 unknowns.

WORKED Example 11
Solve: 2x + y + z = 3
x + 2y − z = −6
5x − 2z = −1.

THINK WRITE

2 1 1 3
1 Write the system in matrix form. A = 1 2 –1 B= –6
5 0 –2 –1

2 Apply Cramer’s Rule.


3 1 1 2 3 1
–6 2 –1 1 –6 –1
–1 0 –2 5 –1 –2
Matrix B is used as column 1 for x, x = -------------------------------------- y = --------------------------------------
column 2 for y and column 3 for z. 2 1 1 2 1 1
1 2 –1 1 2 –1
5 0 –2 5 0 –2

2 1 3
1 2 –6
5 0 –1
z = -----------------------------------
2 1 1
1 2 –1
5 0 –2

– 21
3 Evaluate the determinants, don’t x = --------- = 1
forget the signed minors. – 21
42
y = --------- = −2
– 21
– 63
z = --------- = 3
– 21

4 Verify these results by substituting Check:


into the 3 equations. 2(1) + −2 + 3 = 3 (Verifying the 1st equation)
1 + 2(−2) – 3 = −6 (Verifying the 2nd equation)
5(1) − 2(3) = −1 (Verifying the 3rd equation)
Chapter 5 Matrices and their applications 237
remember
remember
Cramer’s Rule for solving linear equations: ax + by = u
cx + dy = v
u b a u
v d c v
states that x = ----------------------- and y = -----------------------
a b a b
c d c d

5F Cramer’s Rule for solving


linear equations
Use Cramer’s Rule to solve the following:
WORKED 1 2x + y = 1 x = 2, y = −3 2 2x + 4y = 14 x = −1, y = 4
Example
10
3x − 2y = 12 3x − y = −7

3 −x − y = 7 x = −2, y = −5 4 3x + 2y = 6 x = 0, y = 3
4x − y = −3 −2x + y = 3
WORKED 5 x + y + z = 4 x = 3, y = −1, z = 2 6 −x + 2y + 5z = −1 eBook plus
Example
11
2x + y − z = 3 −2x + 3y − z = 7
Digital doc:
3x + 3y + 2z = 10 x − 2y − 2z = 0 WorkSHEET 5.2
1
x = −11 1--3- , y = −5 --3- , z = − 1--3-

Graphics Calculator tip! Matrix operations


The graphics calculator can perform a number of matrix operations and can provide
quick and reliable answers to some of the problems that you have encountered in this
chapter. A number of operations are shown below, some of which have already been
covered in earlier graphics calculator tips.
2 3 2
Consider the matrix A = 0 1 4 .
2 1 –1

For the Casio fx-9860G AU


Entering a matrix
1. Select RUN-MAT from the Main Menu.
2. Press F1 ( MAT) to enter the matrix editing screen.

3. Highlight Mat A and press EXE or F3 (DIM).


4. Specify the number of rows, 3 in this case, and press EXE .
5. Specify the number of columns, 3 in this case, and press EXE .
238 M a t h s Q u e s t M a t h s C Ye a r 1 1 f o r Q u e e n s l a n d

6. Press EXE again to display the 3 × 3 array for


matrix A.
7. Enter the values for the elements, pressing EXE
after each number.
(To exit the Matrix input screen, press EXIT . Press
EXIT again to return to the MAT screen.)

Inverse of A
1. Press OPTN then F2 (MAT) to bring up the matrix
menu.

2. Press F1 (Mat) then ALPHA [A] and SHIFT [x -1] to


specify the matrix A−1.

3. Press EXE to display the answer screen. (Press


EXIT to leave or press EXE again to return to the
matrix menu screen.)

Powers of A
1. Press OPTN then F2 (MAT) to bring up the matrix
menu. (Ignore this step if you are already in the
matrix menu screen from a previous operation.)
2. Press F1 (Mat) then ALPHA [A] to specify
matrix A.
3. To find A2, press x 2 or press the power key Ÿ
and type in the required index of 2. For other powers
of A, press the power key and type in the required
index. (For example, press Ÿ and then 3 to
specify A3.)
4. Press EXE to display the answer screen.

Determinant of A
1. From the matrix menu screen, press F3 (Det) then
F1 (Mat) and ALPHA [A] to specify det A.
2. Press EXE to display the answer.

The identity matrix


s

1. From the matrix menu screen, press F6 ( ) then


F1 (Iden) to specify the identity matrix. For a 3 × 3
identity matrix, press 3 .
Chapter 5 Matrices and their applications 239
2. Press EXE to display the answer screen.

Calculate (I – A)-1 and store it as B


1. From the matrix menu screen, press ( then F6 ( )

s
and F1 (Iden) 3 to specify the 3 × 3 matrix I. Press
the subtraction key – and then press F6 ( )

s
followed by F1 (Mat) and ALPHA [A] to specify the
matrix A. Press ) to close the set of brackets and
then press SHIFT [x-1] to find the inverse matrix.
2. Press Æ then F1 (Mat) and ALPHA [B] to store the
resulting matrix as matrix B.
3. Press EXE to display the resulting matrix B.

1 2 1 2
Calculate the product A-1 3 4 by entering 3 4
as a list 5 6 5 6
1. Enter A−1 as before. (Press F1 (Mat) then ALPHA
[A] followed by SHIFT [x-1].)

2. For the second matrix, we enter each row of the


matrix as a set of elements enclosed in square
brackets. All of the row sets are then enclosed in
another set of square brackets. Use SHIFT [ [ ] to
open a set of square brackets and SHIFT [ ] ] to close
the set. The keys to be pressed can be seen in the
screen shown.

3. Press EXE to display the answer screen.

Fill cells of a matrix with a given value


1. Set the dimensions of matrix A. (Let’s use 3 × 3.)
s

2. From the matrix menu screen, press F6 ( ) then F3


(Fill). Enter the given value. (Let’s use 5.) Press the
comma key , and then press F6 ( ) followed by
s

F1 (Mat) and ALPHA [A] to enter matrix A. Press


) to close the set of brackets. Press EXE .

3. To display matrix A, press EXIT twice to return to


the MAT screen. Then press F1 ( MAT) to enter the

matrix editing screen and highlight Mat A. Press


EXE .
240 M a t h s Q u e s t M a t h s C Ye a r 1 1 f o r Q u e e n s l a n d

For the TI-Nspire CAS


Entering a matrix
1. Open a new Calculator document. Press k to access the catalog. Select Option 5
then highlight the m-by-n matrix symbol.
2. Press ·. Create the matrix with Number of rows: 3
and Number of columns: 3, pressing e to move
from one box to the next.
3. Highlight OK and press ·. Fill in the values in the
matrix using the cursor keys to move from one
element to the next.
4. Move the cursor to the right of the matrix. Press /,
h, then A to store the matrix as matrix A. Press ·
to display the matrix.

Inverse of A
1. With matrix A entered in the calculator, press A then
the power key l and type in the index of −1 to
specify the matrix A−1.
2. Press · to display the required matrix.

Powers of A
1. With matrix A entered in the calculator, press A to
recall matrix A to the screen.
2. To find A2, press q or press the power key l and
type in the required index (2). For other powers of
A, press the power key and type in the required index.
For example, press l and then 3 to specify A3.
3. Press · to display the required matrix.

Determinant of A
1. Press b and select 7: Matrix & Vector (press 8 or
use the cursor keys to highlight 7: Matrix & Vector
and press ·).

2. Select 2:Determinant.

3. Press A to specify matrix A and press ) to close the


set of brackets. Press · to display the answer.

The identity matrix


1. Press b and select 7: Matrix & Vector. Select
6: Create and then select 2: Identity.
2. For a 3 × 3 identity matrix, press 3. Press · to
display the required matrix.
Chapter 5 Matrices and their applications 241
Calculate (I – A)-1 and store it as B
1. Press ( to open a set of brackets. Press b and
select 7: Matrix & Vector then 6: Create and
2: Identity. Press 3 and then ) to specify the 3 × 3
matrix I.
2. Press the subtraction key (-) and then press A to
specify the matrix A. Press ) to close the set of
brackets and then press the power key (l) and type
in −1 as the index to specify the inverse matrix.
3. With the cursor placed after the full expression
(press the right arrow key once), press / and h,
then press B to store the matrix as matrix B.
4. Press · to display the resulting matrix B.
Alternatively, you can store the resulting matrix
after the calculation has been performed. Input
(I – A)−1 and press · to display the resulting
answer matrix. To store this as matrix B, press /,
h then B.

1 2 1 2
-1
Calculate the product A 3 4 by entering 3 4
as a list 5 6 5 6
1. Enter A−1 as before. (Press A then the power key
(l) and type in the index of −1.)
2. Press the right arrow to expand the cursor to the
baseline. Press the multiplication key (r).
3. Enter the second matrix. First press / and ( to
set up a set of square brackets. Enter each row of the
matrix. Each element in a row needs to be separated
by a comma (press the , key). Each row of the
matrix needs to be separated by a semi-colon. To
insert a semi-colon, press k to access the Catalog
and select Option 4 (by pressing 4). Highlight
the ; symbol and press ·. The keys to be pressed
can be seen in the screen shown.
4. Place the cursor at the end of the expression and
press · to display the resulting matrix.
242 M a t h s Q u e s t M a t h s C Ye a r 1 1 f o r Q u e e n s l a n d

Fill cells of a matrix with a given value


1. Set up a new matrix A with specified dimensions.
(Let’s use 3 × 3.) Press b and select 7: Matrix &
Vector. Select 6: Create and then 1: New Matrix.
Enter the number of rows followed by a comma and
then the number of columns. Press ) to close the
set of brackets. Press /, h and A to store the
new matrix as matrix A. Press · to display the
matrix.

2. Press b and select 7: Matrix & Vector. Select


6: Create and then 5: Fill. Enter the given value.
(Let’s use 5.) Press the comma key (,) and then
press A to enter matrix A. Press ·. You will see a
message on the screen that indicates that this
command has been done.

3. To display matrix A, press A and ·.

Solving simultaneous equations


Consider x+y+z=4
2x − y − 2z = 6
3x − 2y = 2
This set of linear equations can be set up as a matrix equation of the form
AX = B.
Use a graphics calculator to solve this set of equations by each of the methods
listed below.

1 Find A−1. Hence find X by calculating A−1B.

2 Set up the augmented matrix [A | B]. Perform Gaussian elimination (using row
operations) to obtain [I | X] and hence find X.

3 Set up the augmented matrix [A | B]. Perform Gaussian elimination but this
time use the Reduced Row-Echelon Form function of your calculator to
find X.
(Hint: For the TI-Nspire CAS calculator, look for the Reduced Row-Echelon
Form in the 7: Matrix & Vector menu.)

4 Use the determinant function of your calculator to apply Cramer’s Rule to solve
the set of equations.

Present your work in a report. Clearly communicate how you performed each
method and discuss your findings. x = 30 34 20
------ , y = ------ , z = − ------
11 11 11
Chapter 5 Matrices and their applications 243
Applications of determinants
Using determinants to find the equation of a line
One of the many applications of determinants is in B(x2, y2)
determining the equation of a straight line.
You are familiar with the equation of a straight line P(x, y)
through two points. Assume that the given line passes
through points A (x1, y1) and B (x2, y2). Let P (x, y) A(x1, y1)
be a point on this line.
As P is a point on the line then the slope of AP
equals the slope of AB.
y–y y2 – y1
-------------1- = ---------------
-
x – x1 x2 – x1
Therefore (y − y1)(x2 − x1) = (y2 − y1)(x − x1)
x2y − x1y − x2y1 + x1y1 = xy2 − x1y2 − xy1 + x1y1
(x1y2 − x2y1) − (xy2 − x2y) + (xy1 − x1y) = 0

x1 x2 x x2 x x1
1× −1× +1× =0
y1 y2 y y2 y y1

The multipliers of one have been included to display the determinant form more
clearly and can be written as:
1 1 1
x x1 x2 =0
y y1 y2

1 Find the equation of the straight line joining points (2, 4) and (4, −6) using
determinants. y = −5x + 14

Using determinants to find the area of a triangle


Similarly the area of a triangle can be
B(x3, y3)
represented in a determinant form:
where the area of LABC equals the total
area of the two trapeziums ADEB and C(x2, y2)
BEFC less the area of trapezium ADFC. A(x1, y1)

D E F

1 1 1
1
2 Demonstrate that the area of LABC = --- x 1 x 2 x 3 and use this
2
y1 y2 y3
determinant to find the area of a triangle of your design.
244 M a t h s Q u e s t M a t h s C Ye a r 1 1 f o r Q u e e n s l a n d

summary
Systems of linear equations
• A pair of equations, ax + by = u and cx + dy = v, may be written in the form AX = B

where A = a b ,X= x and B = u .


c d y v
• The matrix equation AX = B can be rearranged to X = A−1B so that values for x and
y may be found. These values can also be called equilibrium values.

Input–output (Leontief) matrices


• An input–output analysis matrix can be written as X = AX + D where:
matrix X contains the variables to be determined and is called the output matrix
matrix A contains information about the input details and is called the technology
matrix
matrix D contains information about the final demand and is called the final
demand matrix.
• The matrix equation X = AX + D can be rearranged to make X the subject:
X = (I − A)−1D.

Gaussian elimination
• An augmented matrix [A | B] can be used to find the solution of a set of
simultaneous equations when Gaussian row reduction changes [A | B] to [I | X].
• An augmented matrix [A | B] can be used to find the inverse of A when Gaussian
row reduction changes [A | B] to [A–1 | I].

Determinants
• The determinant of A is written det A; det A = ad − bc where A = a b .
c d
a b c
• If A = e f −b d f +c d e
d e f , det A = a
h i g i g h
g h i
where the 3 sub-determinants are referred to as minors.
• In general, the determinant of a 3 × 3 matrix is found by using the
+ − +
alternating signs of − + − attached to the coefficients of each minor.
+ − +
Chapter 5 Matrices and their applications 245
Adjoint matrix
adj A
• A–1 = ------------- , where adj A is the transpose of the cofactor matrix of A, made up of
det A
the minors of A.

Cramer’s Rule for solving linear equations


• Cramer’s Rule for solving equations: ax + by = u
cx + dy = v
u b a u
v d c v
states that x = ----------------------- and y = ----------------------- provided ad − bc ≠ 0.
a b a b
c d c d
246 M a t h s Q u e s t M a t h s C Ye a r 1 1 f o r Q u e e n s l a n d

CHAPTER
review
1 Find the equilibrium values of G and H in the following: G = 50, H = 10
5A G = H + 40
H = 2G − 90 X = 15.15 (15.15 tonnes of aluminium, 14.1 tonnes of gold)
14.1
2 A certain economy produces only two commodities, aluminium and gold. To produce
5A 1 tonne of aluminium, 0.01 tonne of aluminium is required. To produce 1 tonne of gold,
0.005 tonne of gold and 0.2 tonne of aluminium are required. The final demand is for
15 tonnes of aluminium and 11 tonnes of gold. Find the output matrix.
3 Use the Gaussian method of elimination to solve the following system of linear equations:
5B 2x − y + z = −1 x = −1, y = 8, z = 9
3x + 2y − z = 4
x − 2y + 2z = 1
4 Use Gaussian elimination to find the inverse of the following matrices:
5B
3 0 1 1 –1 0
a 3 2 0.2 – 0.4 b 2 0 1 5 –7 1
–1 1 0.2 0.6
–1 1 2 –2 3 0

5 Evaluate the following determinants:


5C
1 2 3 2 3 2 1 0 1
a −2 b 10 c −2
1 1 2 0 1 4 0 1 1
–2 0 0 2 1 –1 1 3 2

6 Evaluate the following determinants and list the property involved.


5D
1 2 3 3 1 4
a 0 0 0 0 Property 3 b 0 –2 2 −30 Property 7

2 3 5 −52 Property 4 0 0 5
13 Property 1
2 1 3 2 3 1 1 0 2 1 3 0
c –3 2 1 given that –3 1 2 = 52 d 3 1 0 given that 0 1 2 = 13
−24 1 3 –2 1 –2 3 0 2 1 2 0 1
Property 5
(twice) 1 1 1 1 1 1 1 –1 2
e 2 –2 2 given that 1 – 1 1 = −4 f 3 4 0 0 Property 2

–3 –3 3 – 1 –1 1 –2 2 –4

1 2 2 1 0 2
g –2 –3 4 given that –2 1 4 = −15 −15 Property 6

1 4 3 1 2 3
7 a i −2 ii 4 –3
–2 1
iii 4 –2
–3 1
iv –2 1
1.5 – 0.5
Chapter 5 Matrices and their applications 247
7 Find the i determinant, ii cofactor matrix, iii adjoint matrix and iv inverse of the following:
b i −12
5E
3 1 1
a 1 2 b 2 1 2 –7 4 5 –7 5 1 7 –5 –1
3 4 1
ii 5 –8 –7 iii 4 –8 –4 iv ------ – 4 8 4
–1 2 –3 12
1 –4 1 5 –7 1 –5 7 –1
8 Use the cofactor–adjoint method to solve the following system of linear equations:
2x − y + 3z = 4 5E
x = 17
------ , y = −2 -----
2
-, z = 2
−x + 2y − z = −5 27 27
---
9

4x + y − 2z = 0
9 a State Cramer’s Rule for solving two equations in two unknowns. a u
b Use this rule to solve c v
u b
v d
5F
y = ----------------------- and x = -----------------------
2x − 3y = 7 a b a b
3x + y = 5 c d c d
x = 2, y = −1
where ax + by = u and cx + dy = v
Modelling and problem solving
1 Let A be a 4 × 3 matrix. Consider matrix B which is a pre-multiplier of matrix A, that is, BA.
Find matrix B if it performs the following elementary row operations on A. 0 0 1 0
a Multiplies the second row of A by 4.
B= 0 4 0 0
b Adds twice row 3 to row 4. 1 0 0 0
c Interchanges rows 1 and 3. 0 0 2 1

2 Let A be a 3 × 4 matrix. Consider matrix C which is a post-multiplier of A, that is, AC. Find
matrix C if it performs the following elementary row operations on A. 0 3 0 1
a Adds 3 times the first column of A to the second column.
C= 0 1 0 0
b Interchanges the first and fourth columns of A. 0 0 –2 0
c Multiplies the third column of A by −2. 1 0 0 0

a+1 −10 a2
3 Find the value of a if 1 a−1 –0 = 0. a = ±3
2 2a −1

4 The table shown on the next page is part of Scotland’s ‘Aggregate Combined Use Matrix
2004 (Purchasers’ Prices), £millions’. The Industries’ intermediate consumption section
shows the inputs of commodities used by Scottish industries in the production of their output.
The Final demand section shows the purchases of each product by each category of final
3713 – 1160- – 17 - ---------------
– 93 - – 2 - ---------------
– 2 - ---------------
– 3 - ---------------
–6 -
demand (for example, consumers, government, exports). ------------ 0 --------------- 0 --------------- 0 ---------------
4532 81 609 14 100 10 501 48 238 13 756 19 116 10 168
a Find the final demand matrix D for this system. 0 3945 – 1716- –-----------
------------ --------------- 195- ---------------
– 285- ---------------
4273 81 609 8570 14 100 10 501 17 256 48 238 13 756
– 41 - ---------------
– 7 - ---------------
– 2 - ---------------
–3 - –4 -
0 ---------------
10 168
b Find the output matrix X. –-----------
821- –-----------
383- 69 438- –-----------
--------------- 107- –---------------
1 918- –---------------
1 982- ---------------
– 939- –---------------
1 511- –---------------
2 979- –---------------
1 601- ---------------
– 672-
4532 4273 81 609 8570 14 100 10 501 17 256 48 238 13 756 19 116 10 168
c Calculate the Leontief matrix (I – A) for this system. – 24- –-----------
----------- 106- ---------------
– 778- 4733 – 15 - ---------------
------------ --------------- – 83 - ---------------
– 36 - ---------------
– 78 - ---------------
– 141- ---------------
– 127- ---------------
– 33 -
4532 4273 81 609 8570 14 100 10 501 17 256 48 238 13 756 19 116 10 168
4 532
4 273
81 609
8 570
14 100
b X = 10 501
17 256
48 238
13 756
19 116
10 168

– 48- –-----------
136- – 136- –-----------
209- 16 713- – 78 - – 128- –---------------
1 415- – 628- – 286- – 41 -
----------- --------------- --------------- --------------- --------------- --------------- --------------- ---------------
4532 4273 81 609 8570 14 100 10 501 17 256 48 238 13 756 19 116 10 168
c (I − A) = –-----------
100- – 52-
----------- – 97 -
--------------- – 64-
----------- – 82 -
--------------- 10 194-
--------------- – 150-
--------------- – 373-
--------------- – 216-
--------------- – 152-
--------------- – 64 -
---------------
4532 4273 81 609 8570 14 100 10 501 17 256 48 238 13 756 19 116 10 168
– 130 – 416 – 981 – 103 – 106 – 1 587 14 203 – 2 140 – 649 – 384 – 274
------------ ------------ ---------------- ------------ ---------------- ---------------- ---------------- ---------------- ---------------- ---------------- ----------------
4532 4273 81 609 8570 14 100 10 501 17 256 48 238 13 756 19 116 10 168
– 209 – 915 – 2 120 – 522 – 1 370 – 2 606 – 1 226 39 454 – 2 302 – 848 – 1 348
2 413
1 692
56 524
3 313
8 382
a D = 8 844
7 433
25 989
13 159
16 487
7 195

------------ ------------ ---------------- ------------ ---------------- ---------------- ---------------- ---------------- ---------------- ---------------- ----------------
4532 4273 81 609 8570 14 100 10 501 17 256 48 238 13 756 19 116 10 168
–4 –9 – 34 –9 – 14 –7 – 25 – 452 13 719 –1 –5
------------ ------------ ---------------- ------------ ---------------- ---------------- ---------------- ---------------- ---------------- ---------------- ----------------
4532 4273 81 609 8570 14 100 10 501 17 256 48 238 13 756 19 116 10 168
– 29 – 18 – 75 – 38 – 11 – 39 – 61 – 349 – 365 17 543 – 71
------------ ------------ ---------------- ------------ ---------------- ---------------- ---------------- ---------------- ---------------- ---------------- ----------------
4532 4273 81 609 8570 14 100 10 501 17 256 48 238 13 756 19 116 10 168
– 23- – 42- – 192- – 44- – 11 - – 88 - – 113- – 423- – 438- – 129- 8 699-
----------- ----------- --------------- ----------- --------------- --------------- --------------- --------------- --------------- --------------- ---------------
4532 4273 81 609 8570 14 100 10 501 17 256 48 238 13 756 19 116 10 168
Digital doc:

Chapter 5
Test Yourself
eBook plus
248

Industries’ intermediate consumption Final demand

Total
demand
Product
for
products

Agriculture, forestry and


fishing
Mining
Manufacturing
Energy and water
Construction
Distribution and catering
Transport and
communication
Finance and business
Public admin.
Education , health and
social work
Other services
Total intermediate
demand
Consumers
Government
Gross capital formation
Exports RUK
Exports RoW
Total final demand

Agriculture, forestry 819 0 1 160 0 17 93 0 2 2 3 6 2 101 1 337 0 131 756 207 2 431 4 532
and fishing
Mining 0 328 1 716 195 285 41 7 2 3 0 4 2 581 27 0 91 1 126 448 1 692 4 273
Manufacturing 821 383 12 171 107 1 918 1 982 939 1 511 2 979 1 601 672 25 085 27 959 0 4 998 12 577 10 991 56 524 81 609
Energy and water 24 106 778 3 837 15 83 36 78 141 127 33 5 257 1 357 0 0 1 931 25 3 313 8 570
M a t h s Q u e s t M a t h s C Ye a r 1 1 f o r Q u e e n s l a n d

Construction 48 136 136 209 2 613 78 128 1 415 628 286 41 5 718 356 0 6 883 1 067 75 8 382 14 100
Distribution and 100 52 97 64 82 307 150 373 216 152 64 1 657 8 248 0 30 462 104 8 844 10 501
catering
Transport and 130 416 981 103 106 1 587 3 053 2 140 649 384 274 9 823 4 411 0 106 2 278 639 7 433 17 256
communication
Finance and 209 915 2 120 522 1 370 2 606 1 226 8 784 2 302 848 1 348 22 250 10 784 0 2 052 10 712 2 440 25 989 48 238
business
Public admin. 4 9 34 9 14 7 25 452 37 1 5 596 231 12 837 91 0 0 13 159 13 756
Education, health 29 18 75 38 11 39 61 349 365 1 573 71 2 629 3 172 12 499 1 663 151 16 487 19 116
and social work
Other services 23 42 192 44 11 88 113 423 438 129 1 469 2 973 5 117 1 057 344 654 23 7 195 10 168
Total intermediate 2 207 2 406 19 459 5 128 6 441 6 911 5 738 15 530 7 761 5 104 3 987 80 671 62 999 26 393 14 728 32 225 15 104 151 449 232 120
consumption
Transformations
using matrices

6
syllabus reference
Core topic:
Matrices and applications

In this chapter
6A Geometric transformations
and matrix algebra
6B Linear transformations
6C Linear transformations and
group theory
6D Rotations
6E Reflections
6F Dilations
6G Shears
250 M a t h s Q u e s t M a t h s C Ye a r 1 1 f o r Q u e e n s l a n d
• group properties of 2 × 2 matrices
• applications of matrices in both life-related and purely mathematical situations

Geometric transformations
and matrix algebra
In your junior mathematics studies you encountered the
idea of translation, reflection, rotation and dilation and how
these transformations changed the position, size and orientation
of the original figure. However, at that stage your investigations were
limited to identifying the type of transformation that had taken place, the position of
the mirror line or centre of rotation, and perhaps the size of the image figure.
However, now you have skills with matrices that will allow much greater detailed
explanation of the position of images or, conversely, the transformation necessary to
map point (x, y) onto point (z, w).
The matrix algebra used is very straightforward and because we are limiting our dis-
cussion at this stage to 2-dimensional space, most of our matrices will be of order
2 × 2. Throughout this section you will be reminded of the properties of groups and
how transformations involved in matrix algebra can be considered to be a group.

Transformations y
A transformation t is an operation which maps P (x, y)
each point of the Cartesian plane onto some
other point on the plane. t
Consider point P(x, y). Under a transformation
t this point is mapped onto P′(x′, y′). The point
P(x, y) is referred to as the original or pre-image P'(x', y')
point and P′(x′, y′) is the image.
This transformation can be written in its most
general form as (x′, y′) = t(x, y). x

WORKED Example 1
Find the coordinates of the image points of A(2, −1) and B(3, 0) under the transformation
defined by the equations:
x′ = 2x − 4xy + y2 − 4
y′ = 6 − 2xy + x − 2y2
THINK WRITE
1 Think of x′ and y′ as functions of x For A(2, −1)
and y. x′ = 2(2) − 4(2)(−1) + (−1)2 − 4
Substitute x = 2 and y = −1 into equations x′ = 4 + 8 + 1 − 4
for x′ and y′.
x′ = 9
y′ = 6 − 2(2)(−1) + (2) − 2(−1)2
x′ = 6 + 4 + 2 − 2
x′ = 10
2 Write the coordinates of the transformed A(2, −1) → A′(9, 10)
image. The symbol → is used to denote
‘maps onto’.
C h a p t e r 6 Tr a n s f o r m a t i o n s u s i n g m a t r i c e s 251
THINK WRITE
3 Substitute x = 3 and y = 0 for B. For B(3, 0)
x′ = 2(3) − 4(3)(0) + (0)2 − 4
x′ = 6 − 0 + 0 − 4
x′ = 2
y′ = 6 − 2(3)(0) + (3) − 2(0)2
x′ = 6 − 0 + 3 − 0
x′ = 9
4 Write the coordinates of the transformed image. B(3, 0) → B′(2, 9)
5 Sketch each original point and its image. y A'(9,10)
Notice that the transformation of A seems quite B'(2, 9)
unconnected with the transformation of B.
t
t

B(3, 0)
0 x
A(2, –1)

Translations
The equations used in the previous example define a general transformation or map-
ping. A translation is a specific transformation that involves a shift of each point in the
same direction. y
x′ = x + a P'(x', y')
y′ = y + b
Each x-coordinate is moved a units parallel to the t b
x-axis and each y-coordinate is moved b units
parallel to the y-axis. P(x, y) a

The image of P is written P′ and this x


translation can be expressed in matrix
equation form as

x¢ = x + a where
y¢ y b

1. x¢ is the vector holding the image coordinates (x′, y′) of point P′


2. x represents the original coordinates (x, y) of point P


y

3. a is the translation vector and represents information about the horizontal


b and vertical displacement.
252 M a t h s Q u e s t M a t h s C Ye a r 1 1 f o r Q u e e n s l a n d

Note t (lower case) denotes the translation itself and T (upper case) denotes the matrix
of the translation.
Therefore (x′, y′) = t(x, y) can be written in matrix form as

x′ = x +T
y′ y

= x + a
y b

= x+a
y+b

WORKED Example 2
Find the image of triangle PQR with vertices P(2, -3), Q(0, 1) and R(-1, -2) under the

translation vector 5 . Sketch the original and image figures.


–1

THINK WRITE

State the general translation matrix x′ = x + a


1
equation. y′ y b
2 Substitute x- and y-values for each For P(2, −3)
point in turn. Consider P(2, −3).
x′ = 2 + 5
y′ –3 –1

= 2+5
– 3 + –1

= 7
–4
3 Write the coordinates of the point P′, P(2, −3) → P′(7, −4)
the image of P. P(2, −3) maps to
P′(7, −4).
4 Consider Q(0, 1). For Q(0, 1)
x′ = 0 + 5
y′ 1 –1

= 0+5
1 + –1

= 5
0
C h a p t e r 6 Tr a n s f o r m a t i o n s u s i n g m a t r i c e s 253

THINK WRITE
5 Write the coordinates of the point Q′, the Q(0, 1) → Q′(5, 0)
image of Q. Q(0, 1) maps to Q′(5, 0).
6 Consider R(−1, −2). For R(−1, −2)
x′ = –1 + 5
y′ –2 –1

= –1+5
– 2 + −1

= 4
–3
7 Write the coordinates of the point R′, the R(−1, −2) → R′(4, −3)
image of R. R(−1, −2) maps to R′(4, −3).
Sketch both the original and the image y
8
points for the triangle PQR. Q(0, 1)
Q'(5, 0)
x

R(–1, –2) R'(4, –3)


P(2, –3)
P'(7, – 4)

Note that the image has been moved 5 units to the right and 1 unit down but remains
unchanged in shape, area, size and orientation. Such a transformation is said to be
congruent.

Successive translations
The translation above could have been achieved by a succession of translations that
have the final effect of 5 across and 1 down. Any number of successive translations

could achieve this: 5 and 0 or the reverse order, 3 and 2 , and so on.
0 –1 2 –3

WORKED Example 3
Show that the translation T1 = 3 followed by T2 = 2 maps the point P(2, -3) from the
2 –3
previous example to the same point P′(7, -4) as found in Worked example 2, and that the
order of the translation has no effect on the result.
THINK WRITE
1 Set up the general matrix equation. x′ x
= + T1
y′ y Continued over page
254 M a t h s Q u e s t M a t h s C Ye a r 1 1 f o r Q u e e n s l a n d

THINK WRITE
2 Use T1 followed by T2. For P(2, −3)
x′ = 2 + 3
y′ –3 2

=5
–1
P(2, −3) → P′(5, −1)
x″ is the image, P″ of image P′. x″ = x′ + T2
3
y″ y″ y′

= 5 + 2
–1 –3

=7
–4
P′(5, −1) → P″(7, −4)
Therefore P(2, −3) → P′(5, −1) → P″(7, −4)

Use T2 followed by T1. x′ = 2 + 2


4
y′ –3 –3

= 2+2
– 3 + –3

=4
–6
P(2, −3) → P′(4, −6)
x″ = 4 + 3
y″ –6 2

=7
–4
P′(4, −6) → P″(7, −4)
Therefore P(2, −3) → P′(4, −6) → P″(7, −4)
5 Sketch the translated image in 2 stages. y

P1' (5, – 1) x
t1
P(2, –3) t2
P'' (7, –4)
t2 t1
P2' (4, – 6)
C h a p t e r 6 Tr a n s f o r m a t i o n s u s i n g m a t r i c e s 255
This example shows, but does not prove, that a set of translations is commutative; that
is, the order of operation does not affect the final result.

Translation of a curve

WORKED Example 4
Find the equation of the curve y = x2 under the translation of T = 1 .
–4
Sketch the original and image curves on the same set of axes.

THINK WRITE

Set up the general matrix equation. x′ = x + 1


1
y′ y –4

2 To find the image of the curve we


must express x and y as found in the x = x′ − 1
original function in terms of x′ and y′. y y′ –4
x = x′ − 1
y = y′ + 4

3 Substitute for x and y in the original y = x2 becomes


function to obtain the function in y′ + 4 = (x′ − 1)2
terms of the image coordinates.
Rearrange and expand this function. y′ = x′2 − 2x′ + 1 − 4
y′ = x′2 − 2x′ − 3

4 State the equation of the image curve. The equation of the image is y′ = x′ 2 − 2x′ − 3.

5 To assist in graphing the image curve, x-axis intercepts occur when y = 0


first find the intercepts with the x-axis 0 = (x – 3)(x + 1)
and the y-axis.
x-axis intercepts occur when
x – 3 = 0 or x + 1 = 0
3−x=3 x = −1
y-axis intercepts occur when x = 0
y = 02 − 2(0) − 3
y = −3

6 Sketch the original and image y


y = x2
functions.
Note that the turning point (0, 0) maps y' = x' 2 – 2x' – 3
to (1, −4) which was the translation
vector. (3, 0)
(–1, 0) 0 x

–3
–4
256 M a t h s Q u e s t M a t h s C Ye a r 1 1 f o r Q u e e n s l a n d

Graphics Calculator tip! Graphing the original


and its image
We can use a graphics calculator to draw the original function and its image on the
same axes. Consider the function y = x2 and its image y ′ = x′ 2 − 2x′ − 3 found in
Worked example 4.
For the Casio fx-9860G AU
1. Press MENU to display the MAIN MENU. Use the
cursor keys to highlight GRAPH.

2. Press EXE . Enter the first equation y = x2 as Y1.


Press X,q,T to enter X and then press x 2 to show
the index of 2. Press EXE .

3. Enter the second equation y = x2 − 2x − 3 as Y2.


Remember to press X,q,T to enter X. Press EXE .

4. To distinguish between the two graphs on the screen,


we can change the appearance of Y2. Highlight the
equation and press F4 (STYL). Select F2 , F3 or
F4 for a different line style. In this example, F3
was chosen.

5. Press EXE to graph the curves.

6. To see the two curves more clearly, we need to set up


a view window. Press SHIFT F3 (V-WIN) and enter
values for Xmin and Xmax. From the screen at right
you can see that −6 and 6 have been chosen. Press
EXE after each entry. Scroll down to enter values
for Ymin and Ymax. The values −6 and 6 have been
chosen. Again, press EXE after each entry.

7. Press EXIT to return to the GRAPH function screen


and then press F6 (DRAW) to see the graphs.
C h a p t e r 6 Tr a n s f o r m a t i o n s u s i n g m a t r i c e s 257
For the TI-Nspire CAS
1. Open a new Graphs & Geometry document (press
/ N and select 2: Graphs & Geometry).

2. To draw the graph of y = x2, type in x2 (by pressing


X and then q) into the function entry line next to
f1(x) =.

3. Press · to obtain the curve labelled with its


equation.

4. To enter the second equation, type x2 − 2x − 3 into


the function entry line next to f2(x) = and press ·.

5. To see the two curves more clearly, we can alter the


scale settings of the axes. Press b and then press
4 to select 4: Window.

6. Press 1 to select 1: Window settings. The values


in the Window Settings box can now be changed.
Enter the values as shown. Press e to move to the
next setting. (To return to a previous setting, press
the shift button (g) followed by e.)

7. Press e until OK is highlighted and press ·. The


graphing window will appear with the new scale.
258 M a t h s Q u e s t M a t h s C Ye a r 1 1 f o r Q u e e n s l a n d

8. If necessary, the label for each graph can be moved


to a different position. Press d. A pointer (å) will
appear in the work area. Use the NavPad to move the
pointer so that it hovers over the equation. The
pointer will then appear as an open hand (÷). Press
/ and then x. The hand will close ({) and the
equation will flash. Use the NavPad to move the
equation to the desired location and then press d.
Note: When the function entry line is shaded, you cannot edit the function entry line
because you are in the graph work area. To move between the graph work area and the
function line, press e.
Also, if the function entry line is covering too much of the graph and you wish to hide
it, press / and then G. To bring the entry line back, press / and then G.

remember
remember
1. A translation T can be written as x ′ = x + T in matrix equation form.
y′ y
The matrix x is the vector representing the coordinates of the point (x, y)
y
and x ′ represents the coordinates of the point (x′, y′) — the image of (x, y)
y′
after translation.
2. A translation results in an image congruent to the original object.
3. A set of translations is commutative — the order of operation does not affect
the final result.

6A Geometric transformations
and matrix algebra
WORKED 1 Find the image of each of the following points under the transformation defined by
Example
xample
1 x′ = 2xy − 3y + x2
y′ = xy + 4y − x
a (0, 0) (0, 0) b (2, −4) (0, −26) c (1, 1) (0, 4) d (−5, −2) (51, 7)
Sketch the original point and its image.

WORKED 2 Find the image of each of the following points under the translation T = 2 .
Example
xample
a (0, 0) (2, −5) b (2, −4) (4, −9) –5
2
c (3, 5) (5, 0) d (−4, 1) (−2, −4)

3 The vertices of a triangle are given by A(0, 0), B(3, 5) and C(7, 2). Find the image of
the vertices under each of the following translations:

A′(4, 2), B′(7, 7), C′(11, 4) a 4 b 4 c 0 d 0


2 0 –2 0

A′(4, 0), B′(7, 5), C′(11, 2) A′(0, −2), B′(3, 3), C′(7, 0) A′(0, 0), B′(3, 5), C′(7, 2)
C h a p t e r 6 Tr a n s f o r m a t i o n s u s i n g m a t r i c e s 259
WORKED 4 The line y = −x + 4 undergoes a succession of translations defined by T1 = 4 and
Example 3
3 – 2
T2 = . Show that the order in which these translations take place has no effect on
–1
the result. Check with your teacher.

WORKED 5 The line y = 2x + 3 undergoes a translation defined by T = – 1 . Find the equation of


Example
the image and sketch the original line and its image. 2
4
6 Find the equation of the image of each of the following curves under the following
translations. Graph the original curve and its image using a graphics calculator.

y′ = −(x′ − 3)2 − 1 a y = −x2 3 b y = x2 − 4 5 c y = x2 − x − 6 – 2 y′ = x′2 + 3x′ − 4


–1 –1 0
y′ = (x′ − 5)2 − 5
d x2 + y2 = 4 2 e y2 + x2 + 6y = 0 – 4 x′2 + 8x′ + y′2 + 2y′ + 8 = 0
2 2
x′2 − 4x′ + y′2 − 4y′ + 4 = 0
7 Rearrange a matrix equation to find the translation vector that maps each of the pairs of
points:
–1 –4
a (2, 4) → (0, 1) – 2 b (4, −1) → (3, 5) c (6, 2) → (2, −5)
–3 6 –7

Linear transformations
5 Have you ever wondered how pro-
y′ = 2x′ + 7 grammers who develop computer
y games move and manoeuvre char-
7 y = 2x + 3
acters on a screen to get them to
y' = 2x' + 7 spin or shrink as they appear to
move further away from the
3
observer? The study of linear trans-
formation forms the foundation for
–3.5 –1.5
0 x these changes of form and size —
the warping of the plane on which
the characters are mapped.
There are many different ways in which the original, or pre-image, can be changed
or moved so that it looks different, or is in a different place.
A linear transformation l is a mapping of the pre-image P(x, y) onto the image
P′(x′, y′) where:
x′ = ax + by
y′ = cx + dy
for all real values of a, b, c, and d. In matrix form this system is written as:
x′ = a b x
y′ c d y

x′ =L x where L = a b and is called the transformation matrix.


y′ y c d
260 M a t h s Q u e s t M a t h s C Ye a r 1 1 f o r Q u e e n s l a n d

WORKED Example 5
y
Find the images of the vertices of a unit square ABCD under B (1, 1)
the transformation given by A(0, 1)

L= 1 2 .
–1 1
D(0, 0) C (1, 0) x
THINK WRITE
1 Set up the initial matrix equation where
the image of P is given as P′. x′ = 1 2 x
y′ –1 1 y

2 Investigate the transformation of each For point A(0, 1)


point in turn.
x′ = 1 2 0 = 2
(Recall that the symbol → is used to
denote ‘maps onto’.) y′ –1 1 1 1
That is, A(0, 1) → A′(2, 1)
For point B(1, 1)
x′ = 1 2 1 = 3
y′ –1 1 1 0
That is, B(1, 1) → B′(3, 0)
For point C(1, 0)
x′ = 1 2 1 = 1
y′ –1 1 0 –1
That is, C(1, 0) → C′(1, −1)
For point D(0, 0)
x′ = 1 2 0 = 0
y′ –1 1 0 0
That is, D(0, 0) → D′(0, 0)

3 Plot the image on the same axes as the y


original.
B(1, 1) A'(2, 1)
A(0, 1)

D(0, 0) C(1, 0) x
B' (3, 0)

C' (1, –1)

This type of transformation leaves the origin unchanged and therefore differs from a
translation. The transformation matrix can also be extracted from information about the
original and image points. An example of this is shown in the following worked
example.
C h a p t e r 6 Tr a n s f o r m a t i o n s u s i n g m a t r i c e s 261
WORKED Example 6
Find the matrix of the linear transformation that maps A(1, 1) onto A′(2, -1) and B(2, -1)
onto B′(1, -1).
THINK WRITE

Set up the initial matrix equation. x′ = a b x


1
y′ c d y
2 State matrix equations for points A, A′, For point A:
B and B′.
2 = a b 1
–1 c d 1
and for point B:
1 = a b 2
–1 c d –1
3 Multiply the matrices to arrive at 4 From the equation for point A:
simultaneous equations for 4 2=a+b
unknowns, a, b, c and d. −1 = c + d
From the equation for point B:
1 = 2a − b
−1 = 2c − d
4 This is really 2 sets of 2 equations in 2 a+b=2 [1]
unknowns that can be solved by c + d = −1 [2]
elimination.
2a − b = 1 [3]
If you wish to use your graphics
2c − d = −1 [4]
calculator, enter this as
[1] + [3]: 3a = 3
1 1 0 0 2
a=1
A= 0 0 1 1 , B = –1
[2] + [4]: 3c = −2
2 –1 0 0 1
c = − 2---
0 0 2 –1 –1 3

and find A−1B.


5 Find b by substituting a = 1 into Sub. a = 1 into [1]: 1+b =2
Equation [1] and find d by substituting b =1
c = − 2--- into Equation [2]. List the Sub. c = − 2--- into [2]: − 2--- + d = −1
3
values for a, b, c and d. 3 3
d = − 1---
3
a = 1, b = 1, c = − --- , d = − ---
2 1
3 3

1 1
6 Use these values to build L, the linear L=
transformation matrix. – 2--3- – 1--3-

As hinted at in the introduction to this section, there are two ways to conceptualise a
transformation. The more obvious way is to imagine that the points move to new
positions on the Cartesian plane. The other less obvious notion is that it is actually the
262 M a t h s Q u e s t M a t h s C Ye a r 1 1 f o r Q u e e n s l a n d

x
C'(3, –1)
B(2, 0)

Cartesian plane on which the original points are plotted that undergoes distortions to
A(1, –2) yield the transformed image. Perhaps the former is more straightforward, but the end
product will be the same.
0
y
A'(–1, 2)

B'(–2, 0)
C(–3, 1)

remember
remember
1. A linear transformation can be represented by x ′ = a b
ii

x or
y′
B'(4, 2)

c d y
B(2, 0)

x ′ = L x where L = a b is the transformation matrix that maps


y′
A(1, –2)

y c d
A'

point (x, y) onto the image (x′, y′).


0
y

–1
2. The transformed image can be found using x = a b x′ .
y c d y′
C'(–5, –2)
C(–3, 1)

3. The transformed image is not congruent to the object.


2 b i

2 a i
6B

iv
iii
ii
Linear transformations

A′(1, −2), B′(2, 0), C′(−3, 1)

A′(−1, 2), B′(−2, 0), C′(3, −1)


A′(2, − 4), B′(4, 0), C′(−6, 2)

A′(0, −1), B′(4, 2), C′(−5, −2)


2 –3
3 y 1 ii Which of the following transformations are linear? 3 2
B' 5
4
ii Write the transformation matrices for each of these.
3
2 C a x′ = x + y 1 1 b x′ = x − 1 c x′ = 2x – 3y
–3 1
–2 2 4 6 8 x y′ = 2x + y 2 1 y′ = y + 2 y′ = 3x + 2y
B –2
–3
–4
A
d x′ = x + y e x′ = x 2
–5
–6 A' 1
y′ = 1 + --- y′ = y2
–7 C' y
WORKED 2 a Find the images of the points A(1, −2), B(2, 0) and C(−3, 1) under the following
Example
xample
5
transformations:

i 2 1 ii – 1 0 iii 2 0 iv 1 0
B'(4, 0)
x

1 1 0 –1 0 2 0 1
A'(2, –4)

b Sketch the original triangle from a and its 4 different images. iv No change
B

3 Find the image of the points (given below) under the transformation defined by:
A

x′ = x − 2y
0
y

y′ = −2x + y
a A(2, −3) A′(8, −7) b B(−3, −1) B′(−1, 5) c C(4, 1) C′(2, −7)
Plot the original point and its image in each case.
C
C'(–6, 2)

4 Find the image of the pre-image points A(4, 1), B(−4, 1) and C(0, 5) under the trans-
formation defined by: 4 A′(7, − 6), B′(−1, 2), C′(15, −10)
2 b iii

x′ = x + 3y y
y′ = −x − 2y B'(–1, 2) 5 C(0, 5)
Plot the original and image points. B(–4, 1)
A(4, 1)
15 x
WORKED 5 Find the matrix of the linear transformation which maps: –4 –1 45 7 10
Example
xample
6
a (1, 2) → (−3, 1) and (3, 0) → (1, 4) –5
A'(7, –6)
b (−2, 3) → (0, 0) and (−2, 4) → (1, 1) –10
1 1 C'(15, –10)
c (2, −1) → (1, 1) and (2, 1) → (3, 6)
1.75 2.5 d (3, 4) → (5, 0) and (−3, −2) → (−2, 4)
– 1--3- 1.5 1 2 ---
3
– 1 --3- 1.5 1
5 a b
– 2 --23- 2
1 1--3- – 1--6- 1.5 1
C h a p t e r 6 Tr a n s f o r m a t i o n s u s i n g m a t r i c e s 263
Linear transformations and
group theory
Earlier in your Mathematics C course of study you were introduced to group theory
(Chapter 4). You found that a system formed a group if the properties of closure and
associativity applied and an identity element and inverse existed. These properties
apply to many areas of mathematics — including linear transformations. In Chapter 4
we investigated whether matrices, in general, formed a group; now we will study
groups that perform linear transformations.

Closure
If l1 is a linear transformation that maps (x, y) → (x′, y′) then
(x′, y′) = l1 (x, y)
If l2 is a linear transformation that maps (x′, y′) → (x″, y″) then
(x″, y″) = l2 (x′, y′)
Therefore it follows that
(x″, y″) = l2 [l1(x, y)]
where l1 is followed by l2 and maps (x, y) → (x″, y″). This double transformation can be
represented as a single, where l = l2 l1. This is known as composition of transform-
ations, where the order is significant.
From the Mathematics B course you would be familiar with the idea of composition
of functions, where g(x) = h(f(x)) indicates that f(x) is the ‘inner’ function within the
structure and general shape of h(x).
In matrix form
x″ = L2 x′
y″ y′

 
= L2  L 1 x 
 y 

= L2 L1 x
y

= L x where L is a 2 × 2 matrix and L = L2 L1


y
We can verify this result by considering the image
y
of the point P(1, 2) after a linear transformation
5
L1 = 1 2 followed by a linear transformation 4 l1
P'
2 1 3
2 P l2
3 – 3 . Show that following this double 1 l
L2 = P"
–3 4 0 1 2 3 4 5 6 x

transformation produces the point P″(3, 1). If we mapped


P(1, 2) directly to P″(3, 1) in a single transformation,
find the transformation matrix L. Is this transformation
matrix L equivalent to L1L2 or L2L1?
264 M a t h s Q u e s t M a t h s C Ye a r 1 1 f o r Q u e e n s l a n d

WORKED Example 7
If l1 and l2 are 2 linear transformations such that L1 = 1 1 and L2 = 0 0 :
2 –2 2 1
a find P′, the image of P (1, 3) under l1
b find P≤, the image of P′ under transformation l2
c find the single transformation of P such that l = l2 l1
d verify that P≤ (as found in part b) is equal to LP

THINK WRITE

a Use matrix operation to find P′, the x′


a = 1 1 1 = 4
image of P(1, 3) under l1.. y′ 2 –2 3 –4

b Find P≤, the image of P′ under b x″ = 0 0 4 = 0


transformation l2. y″ 2 1 –4 4

c Find the single transformation of P such c L= 0 0 1 1 = 0 0


that l = l2 l1. 2 1 2 –2 4 0

d Verify that P≤ (as found in part b) is d LP = 0 0 1 = 0 = P″


equal to LP. 4 0 3 4
Therefore P(1, 3) → P″(0, 4)

Associativity
As seen with matrix operations, matrix multiplication is associative; that is,
(L1 L2)L3 = L1(L2 L3). Therefore linear transformations are associative; that is,
(l1 l2)l3 = l1(l2 l3).

Identity
Remember the identity element (IE) is one which leaves the original number
unchanged. When dealing with linear transformation this means that matrix multi-
plication has been performed which leaves the original point unchanged. This is the
identity transformation and is denoted by li — and the matrix is I.

For a 2 × 2 matrix, I = 1 0 .
0 1

Inverse transformations
y
An inverse transformation is one that maps the image P'(x', y')
back to the original point — where l
(x, y) → (x′, y′) → (x, y).
This transformation is denoted by l–1. l–1

As with other inverses ll–1 = l–1l P(x, y)


x
As with other inverses ll–1 = li
C h a p t e r 6 Tr a n s f o r m a t i o n s u s i n g m a t r i c e s 265
If L is the linear transformation matrix and A is the transformation matrix which returns
the point to the original, then A = L–1.
As with general matrix terminology, the transformation l is non-singular; that is, l
has an inverse if it has a matrix l –1 that will map the image back to the original.
Therefore, only linear transformations that have an inverse l –1 can be considered to
form a group. If l is singular then the set of linear transformations does not form a
group.

Abelian groups
If the composition of linear transformations is commutative, then the set will form an
Abelian group. But in general, multiplication of linear transformations is not commu-
tative, that is l1 l2 ≠ l2 l1.

WORKED Example 8
a Find the image of the point P(2, 3) under l1 followed by l2 with

L1 = 2 3 L2 = 0.2 0.6
1 –1 0.2 – 0.4
b Verify that l2 = l1–1 in 2 ways.

THINK WRITE

a 1 Set matrices in x ′ = LP form. a x′ = 2 3 2 = 13


y′ y′ 1 –1 3 –1

P′ is the point (13, −1). Now find P″ x″ = 0.2 0.6 13 = 2


2
using P″ = L2P′. y″ 0.2 – 0.4 –1 3
3 State the image of P under l1 The image of the point P(2, 3) under l1
followed by l2. followed by l2 is (2, 3).
Since P″(x″, y″) = P(x, y), L2 has mapped
P′(x′, y′) back onto the original, therefore
L2 is the inverse linear transformation of L1.

b 1 Verify this by showing L2 L1 = I. b L2L1 = 0.2 0.6 2 3 = 1 0


0.2 – 0.4 1 –1 0 1
1
2 Verify by finding the inverse of L1. L1–1 = ------------------ d – b
ad – bc – c a

1
L1–1 = ---------------- – 1 – 3
– 2 – 3 –1 2

L1–1 = 0.2 0.6


0.2 – 0.4
L1–1 = L2
3 State the conclusion. Therefore L2 = L1–1
266 M a t h s Q u e s t M a t h s C Ye a r 1 1 f o r Q u e e n s l a n d

WORKED Example 9
Determine whether the following linear transformation, l1, is singular or non-singular.
x′ = 2x + y
y′ = −2x + 3y
THINK WRITE
1 State l1 in matrix form.
L1 = 2 1 for L = a b
–2 3 c d

2 Test to determine whether the | L1 | = ad − bc = 6 − −2 = 8


determinant = 0.
3 State your conclusion. Since det L1 ≠ 0, L1 is non-singular, that is, it
has an inverse.

Images of curves — non-singular transformations


So far we have mainly considered only the images of individual points under linear

transformation where x′ = L x . Now consider the image of a curve — essentially


y′ y
a set of points.

WORKED Example 10
Find the image of the line y = 2x − 3 under the linear transformation L = 2 1 .
Sketch the original line and its image. 0 3

THINK WRITE
1 We need to express the original
x′ =L x
function in terms of the image points so
y′ y
we need to find x and substitute
y L–1 x ′ = L–1L x
image points for the original points x y′ y
and y.
x = L–1 x ′
y y′

2 Evaluate the inverse.


x 1
= ------------ 3 – 1 x′
y 6–0 0 2 y′

1
--- – 1--6- x′
= 2

0 1
--- y′
3
C h a p t e r 6 Tr a n s f o r m a t i o n s u s i n g m a t r i c e s 267
THINK WRITE
1
x --- x′ – 1--6- y′
3 Express x′ and y′ in terms of the original = 2

points. y 0x ′ + 1--3- y ′
Therefore
x = 1--- x′ − 1--- y′
2 6

y= 1
--- y′
3
4 Substitute for x and y in terms of the y = 2x − 3 becomes
image points, into the original function. 1
--- y′ = 2( 1--- x′ − 1--- y′) − 3
Simplify and rearrange the image 3 2 6

equation. = x′ − 1--- y′ − 3
3
--- y′ = x′ − 3
2
3
5 State the equation of the image. Some y′ = 3--- x′ − 4 1---
2 2
texts drop the ‘primes’ on x′ and y′ at this The image of y = 2x − 3 has the equation
stage, but if they are left in it reminds us
that the graph of this function is the image y′ = 3--- x′ − 4 1--- .
2 2
of the original.
y y = 2x – 3
6 Sketch the original and image functions.

0 11–2 3 x

y' = 3–2 x' – 4 1–2


–3
– 4 1–2

Images of curves — singular transformations


If a linear transformation L is singular, then L does not have an inverse and the method
shown in Worked example 10 cannot be used. We need to use a different approach as
shown in the next worked example.

WORKED Example 11
Find the image of the circle x2 + y2 = 1 under the linear transformation L = 1 2 .
Sketch the original curve and its image. 2 4

THINK WRITE

State the initial transformation in x′ =L x


1
general matrix form. y′ y

= 1 2 x
2 4 y
Continued over page
268 M a t h s Q u e s t M a t h s C Ye a r 1 1 f o r Q u e e n s l a n d

THINK WRITE

Find values for x′ and y′. x′ = x + 2y


2
y′ 2x + 4y

3 Notice that the equation for y′ equals y′ = 2x′


twice the equation for x′. Therefore this
should be stated as the function of the
image.

4 State the equation of the image. The image of x2 + y2 = 1 has the equation
y′ = 2x′.

5 Sketch the original curve and its image. y


y' = 2x'
(0, 1)

x2 + y2 = 1
(–1, 0) (1, 0) x

(0, –1)

remember
remember
1. (a) Linear transformations are closed.
(b) If (x′, y′) = l1(x, y) where l1 is a linear transformation that maps
(x, y) → (x′, y′) and l2 is a linear transformation that maps
(x′, y′) → ( x″, y″) then (x″, y″) = l2(x′, y′)
= l2[l1(x, y)]
where l1 is followed by l2
2. Linear transformations are associative; that is, (l1l2)l3 = l1(l2l3).
3. The identity transformation is denoted by li and is represented by the identity
matrix I.
4. An inverse transformation is one that maps the image back to the original point
where (x, y) → (x′, y′) → (x, y) and is denoted by l –1.
As with other inverses ll –1 = l –1l
= li
Only linear transformations that have an inverse l –1 can be considered to form
a group.
5. If linear transformations are commutative, then they will form an Abelian
group. But in general, multiplication of transformations is not commutative,
that is l1l2 ≠ l2l1.
C h a p t e r 6 Tr a n s f o r m a t i o n s u s i n g m a t r i c e s 269
Linear transformations and
6C group theory
WORKED 1 If l1 and l2 are 2 linear transformations such that L1 = 2 1 and L2 = 0 1 :
Example
a find P′, the image of P(2, 5) under l1 P′(9, 5) 0 1 –1 2
7
b find P′′, the image of P′ under transformation l2 P′′(5, 1)
0 1 c find the single transformation of P such that l = l2l1
–2 1 d verify that P′′ is equal to LP. Check with your teacher.
WORKED 2 a Find the image of the point P(1, 4) under l1 followed by l2 with P′′(1, 4)
Example
8
L1 = 3 – 1 L2 = 2 – 1
5 –2 5 –3
-1
b Verify that l1 = l1 in 2 ways. Check with your teacher.
WORKED 3 Determine whether, in each of the following linear transformations, l1 is singular or
Example
9
non-singular:
a x′ = 3x − y Non-singular b x′ = 2x − y Singular
5
y
y′ = −x + 2y y′ = 4x − 2y
y = 3x + 2
4 A linear transformation l1 is defined as x′ = 2x + 5y d i y′ = --47- x′
2 y′ = x + 3y ii y′ = 10
------ x′ + -----
2-
y' = _12 x' – 2
a What will the image of P(3, 5) be? P′(31, 18) 17 17
– _23 0 x
iii 10x′2 − 34x′y′ + 29y′2 = 2
b Is this linear transformation singular? det A = 1 (non-singular)
4

c Show that l1–1(l1P ) = P.


–2

d Use this linear transformation to state the image of the following curves:
i y=x ii y = 3x + 2 iii x2 + y2 = 2
WORKED 5 Find the image of the line y = 3x + 2 under the linear transformation L = – 4 2 .
Example
Sketch the original line and its image. y′ = 1--- x′ − 2 1 0
10 2

6 Find the image of the circle x + y = 9 under each of the following transformations.
2 2

17x′2 − 26x′y′ + 10y′2 = 9 a 1 3 b 1 3 c –1 –1 13x′2 + 10x′y′ + 2y′2 = 9


1 4 1 0 3 2
10y′2 + x′2 − 2x′y′ = 81
WORKED 7 Find the image of the circle x2 + y2 = 9 under each of the following transformations.
Example
11 2 0 2 4 8 4
a y′ = x′ b y′ = --32- x′ c y′ = --12- x′
2 0 3 6 4 2
9 a, c ii 8 Show that under any linear transformation the image of a straight line is itself a
y
y = –x + 4
straight line. Check with your teacher.

y' = 4
9 a Sketch the following lines on separate axes.
4
i y = 2x − 1
ii y = −x + 4
b i y′ = -----
3-
x′ + 1---
b Find the image of each line under the linear transformation 2 4 10 5
0 4
x ii y′ = 4
c Sketch each image with the original line. 1 1
10 Find the image of each of the following functions under the linear transformation 5 3 .
a y = x2 x′2 − 6x′y′ + 9y′2 − 2x′ + 5y′ = 0 2 1
b y = 2x + 5 11y′ = 4x′ − 5
270 M a t h s Q u e s t M a t h s C Ye a r 1 1 f o r Q u e e n s l a n d

SLE 4: Demonstrate the use of the transformation matrices (rotation,


reflection, dilation) as an application of 2 × 2 matrices to geometric
Rotations transformations in the plane.

A rotation is a transformation in y
which the plane rotates about a
fixed point called the centre of
rotation. This point is usually taken B' C'
as the origin. The rotation in an C
anticlockwise direction is con- B A
sidered to be a positive rotation and θ A'
in a clockwise direction to be a 0 x
negative rotation.
Examine the diagram at right to
note that the centre of rotation is the
only point that doesn’t move.
In a rotation:
1. each original point rotates through the same angle of rotation.
2. the image is congruent to the original — the length, angle and area
remain unchanged in the image. This is referred to as a congruent
transformation.
3. rq denotes rotation in a positive direction through an angle of θ and Rθ
is the matrix of rotation.
With all the transformations that will be discussed we will generate matrices based
on where the points (1, 0) and (0, 1) are mapped to on the plane, as a result of the trans-
formation. These points are represented by columns 1 and 2 of the identity matrix:
↓ ↓
1 0
0 1
Special rotations
In this section we will discuss transformations involving rotations of 90°, 180°, 270°
and 360°, as well as general rotations.
Rotation of 90°
Consider the figure at right. y y
(0, 1) (0, 1)

90º (1, 0) (–1, 0) 90º


x x

As the plane rotates through θ = 90° about the origin, point (1, 0) will map to point
(0, 1) and point (0, 1) will map to point (−1, 0).
↓ ↓
Hence, the identity matrix, I, is altered to 0 – 1 to achieve a rotation of 90° about
the origin. 1 0
It is most important that you recognise the pattern that is displayed by the columns in
the matrix and the coordinates of the image points. This concept forms the basis of the
next section of work and totally eliminates ‘remembering’ formulas so that you will be
able to understand what is happening to the points.
C h a p t e r 6 Tr a n s f o r m a t i o n s u s i n g m a t r i c e s 271
y
Hence R90° = 0 – 1 and is the matrix of rotation.
1 0 P'(x', y')
In general terms P(x, y)
(x, y) → (−y, x)
x
x′ = 0 –1 x
y′ 1 0 y
x′ = −y
y′ = x
As mentioned earlier, these rotation matrices should not be learned. They are quite
similar and can be too readily confused. Sketch the original (1, 0) and (0, 1) points and
then use their images to build the rotation matrices.

Rotation of 180°
In the diagrams below, notice that point (1, 0) is mapped onto point (−1, 0) and point
(0,1) is mapped onto (0, −1).
y y y
(0, 1) P(x, y)

(1, 0)
180º

º
(–1, 0) 180º

180
x x 0 x

(0, –1) P'(x', y')

Therefore R180° = – 1 0 where (x, y) → (−x, −y).


0 –1

Rotation of 270°
In the diagrams below, notice that point (1, 0) is mapped onto point (0, −1) and point
(0, 1) is mapped onto point (1, 0).
y y
y
(0, 1)

P(x, y)
270º (1, 0) (1, 0)
270º

270º

0 x 0 x x

P'(x', y')
(0, –1)

Therefore R270° = 0 1 where (x, y) → (y, −x).


–1 0

Rotation of 360°
R360° = 0 1 because R360° essentially leaves the original unchanged (or mapped onto
itself). 1 0
272 M a t h s Q u e s t M a t h s C Ye a r 1 1 f o r Q u e e n s l a n d

y
General rotation of θ
B(0,1)
Consider the points (1, 0) and (0, 1)
that are rotated through angle θ about –sin θ A'
B' Q
the origin.
1
cos θ 1
θ sin θ
θ A(1, 0)
0 cos θ P x

Careful examination of the diagram shows that point (1, 0) is


mapped onto point (cos θ, sin θ) and point (0, 1) is mapped
onto point (−sin θ, cos θ) where sin θ

cos θ = x (horizontal)
θ
and sin θ = y (vertical) cos θ

Rθ = cos θ –sin θ
sin θ cos θ y
P(x, y)
R–θ , where θ is taken in a clockwise, negative rotation about
the origin, and is shown in the diagram to the right. –θ x

R−θ = cos ( – θ ) –sin ( – θ ) P'(x', y')


sin ( – θ ) cos ( – θ )

R−θ = cos θ sin θ since cos (−θ) = cos θ and sin (−θ) = −sin θ
–sin θ cos θ
Both Rθ and R−θ can be used to confirm the specific cases of R90°, R180° and R270°.

R90° = cos 90° –sin 90° = 0 – 1


sin 90° cos 90° 1 0
Remember that when you need to evaluate a trigonometric ratio:
1. sketch the angle concerned in the correct quadrant
2. write the coordinates or length of the sides on the right-angled triangle
3. in the unit circle, the cosine ratio involves only the x-coordinate and the sine ratio
involves only the y-coordinate.
Verification of the other angle measures is left as a future exercise.

WORKED Example 12
πc
Find the image of the point (2, -2) under a rotation of --- about the origin. Sketch the
original point and its image. 4

THINK WRITE

1 Write the general rotation matrix and Rθ = cos θ –sin θ


sketch the original point (shown on next sin θ cos θ
page).
C h a p t e r 6 Tr a n s f o r m a t i o n s u s i n g m a t r i c e s 273
THINK WRITE

π π
c cos --- –sin ---
π 4 4
2 Substitute ----- for θ. Rπ =
4 ---
4 π π
(Note: The small c is the symbol for sin --- cos ---
4 4
circular or radian measure.)
1-
------ 1-
– ------
Always use a sketch to develop the matrix. Rπ = 2 2
3
---
4 1-
------ 1-
------
–π
4 2 2
√2
1
–π
4
1

1-
------ 1-
– ------
Set up the general matrix form for x′ = 2 2 2
4
transformations. y′ 1- 1- –2
------ ------
2 2

2- + ------
------ 2-
= 2 2
2- – ------
------ 2-
2 2

4-
------
= 2
0

4- ------2-
Rationalise the denominator and simplify. =
------ ×
5 2 2
0

= 2 2
0
6 State the coordinates of the image point. The image of the point (2, −2) is (2 2 , 0).
7 Sketch the original and the image points. y

(2 √2, 0)
0 –π x
4

(2, –2)
274 M a t h s Q u e s t M a t h s C Ye a r 1 1 f o r Q u e e n s l a n d

WORKED Example 13
πc
Find the image of the line y = −x + 4 under the rotation of --- about the origin. Sketch the
original line and its image. 6

THINK WRITE

1 Write the general Rθ matrix. Rθ = cos θ –sin θ


sin θ cos θ
π π
cos --- –sin ---
πc 6 6
2 Substitute --- for θ and evaluate using R --π- =
6 6 π π
the relevant triangle of ratios. sin --- cos ---
6 6
–π
------3- – 1---
6

2
√3 = 2 2
1--- ------3-
–π 2 2
3
1

Set up the general transformation x′ = R --π- x


3
x y′ 6 y
matrix model, rearranged so that
y x –1 x′
is the subject. = R --π-
y 6 y′

------3- 1---
Evaluate the inverse of R.
1 -
= ------------ 2 2 x′
4 3 1
--- – – ---
3 y′
4 4
– 1--- -------
2 2

3 1
------- ---
= 2 2 x′
3 y′
– 1--- -------
2 2
3 1
5 Multiply out the matrices. x = ------- x′ + --- y′
2 2
1--- 3
y = − x′ + ------- y′
2 2
6 Substitute for x and y in the original y = −x + 4 becomes
function. 1 3 3 1
− --- x′ + ------- y′ = − ------- x′ − --- y′ + 4
2 2 2 2
(1 – 3 )x′ 8
7 After applying the Distributive Law and y′ = -------------------------- + ----------------
rationalising the denominator, this 3+1 3+1
expression can be simplified. y′ = ( 3 − 2)x′ + 4( 3 − 1)
C h a p t e r 6 Tr a n s f o r m a t i o n s u s i n g m a t r i c e s 275
THINK WRITE
8 Use your calculator only at the end to y
simplify surds for sketching purposes. y' = (√3 – 2) x' + 4 (√3 – 2)
4
2.9
4 10.9 x

y = –x + 4

remember
remember
1. For general rotation θ in an anticlockwise direction about the origin

Rθ = cos θ –sin θ .
sin θ cos θ
2. Use the special right-angled triangles to obtain the trigonometric ratios.
3. Rotation is a congruent transformation.

6D Rotations
1 Construct matrices for the following anticlockwise rotations about the origin (the
angles are given in radians).
π 0 –1 –1 0 3π
a --- b π c ------ 0 1 d 2π 1 0
2 1 0 0 –1 2 –1 0 0 1
WORKED 2 Find the image of the following points under the given anticlockwise rotations about
Example
12
the origin.
π c (1 − ------3- , 3 + 1--- )
3 b i a (2, 1) θ = --- 2 2
b (0, 4) θ = π c (0, −4)
y 3
πc ( 1--2- − , − ------ − 3--2- )
1 c (6, 3) θ = --- ( 3---------
2
2-
, 9---------2-
2
) d (1, −3) θ = −60° 3---------3-
2 2
3-

0.35 2 4

y = _x +√_2
4
πc
θ = 90° (−3, 2) θ = ---
x
e (2, 3) f − --12- , +
0 _1
–0.7 3 1
(1, 1) 2
3-
( ------ --1-
2
------3-
2
)
y = –3x + 1 6
WORKED 3 a Find the equation of the image of the line y = −3x + 1 as a result of the following
Example
13
rotations: x′ 2
a i y′ = ---- + -------
π c
π c
2 4
i θ = 45° ii θ = --- iii θ = − ---
2 2 ii y′ = --3x- + --13-
b Sketch each original line and its image. iii y′ = --3x- − 1--3-
eBook plus
πc
Digital doc:
4 Find the equation of the image of the circle x2 + y2 = 1 after a rotation of --- . What do
2
WorkSHEET 6.1
you notice? Can you explain why this is so? No change, rotation about the centre of the circle.
276 M a t h s Q u e s t M a t h s C Ye a r 1 1 f o r Q u e e n s l a n d

SLE 4: Demonstrate the use of the transformation matrices (rotation, reflection, dilation) as
Reflections an application of 2 × 2 matrices to geometric transformations in the plane.
y
A reflection is a linear transformation in
which every point of the original is reflected A' A

=
through a straight line called a mediator.
This line can be thought of as a mirror.
The diagram at right shows LABC reflected
through the mediator m, at x = 1.
In a reflection:

=
=
C' B' B C
1. corresponding points of the image and
original figures are equidistant from and x
x=1
perpendicular to the mediator m
2. length, angle and area of the image and
original are unchanged, hence it is a congruent transformation
3. any points of the original on the mediator are left unchanged.
We usually let m denote the reflection transformation and M the reflection matrix.

Reflection in the x-axis (where y ↓ ↓= 0)


Again, sketch the points (1, 0) and (0, 1) from the identity matrix I = 1 0 .
0 1
Under a reflection in the x-axis, point y
(0, 1) will map to (0, −1) and point (0, 1)
(1, 0) will map onto itself because it
is on the mediator.
(1, 0)
my = 0
0 x
Therefore My = 0 = 1 0 .
0 –1
(0, –1)
C h a p t e r 6 Tr a n s f o r m a t i o n s u s i n g m a t r i c e s 277
Reflection in the y-axis (where x = 0)
If you sketch the original points (1, 0) and (0, 1) mx = 0
you will notice that if these points are reflected in y
the y-axis then point (1, 0) will map to (−1, 0) and
point (0, 1), which is on the mediator, will map (0, 1)
onto itself.
(–1, 0) (1, 0)
0 x
Therefore, Mx = 0 = – 1 0 .
0 1

WORKED Example 14
Find the image of point (3, 1) under reflection My = 0. Sketch the original and its image.

THINK WRITE
1 Sketch the diagram to construct your y
reflection matrix.
(0, 1)

(1, 0)
my = 0
0 x

(0, –1)

My = 0 = 1 0
0 –1

Write the initial transformation matrix x′ = My = 0 x


2
statement. y′ y

3 Substitute the necessary values and = 1 0 3


evaluate. 0 –1 1

= 3
–1

4 Sketch the original and image points. The image is the point (3, −1).
y
P(3, 1)

my = 0
0 x

P'(3, –1)
278 M a t h s Q u e s t M a t h s C Ye a r 1 1 f o r Q u e e n s l a n d

WORKED Example 15
Find the image of y = x under reflection in the y-axis. Sketch the original and its image.

THINK WRITE
1 Sketch the diagram to construct your mx = 0
reflection matrix. y
(0, 1)

(–1, 0) (1, 0)
0 x

Mx = 0 = – 1 0
0 1

Write the initial transformation matrix x′ = Mx = 0 x


2
statement and rearrange it to have the y′ y
original points as the subject.
x = M x–1= 0 x′
y y′

Substitute for Mx = 0 and evaluate the x 1


= ------ 1 0 x′
3
inverse. y –1 0 –1 y′

= –1 0 x′
0 1 y′

4 Multiply to give expressions for x x = −x′


and y. y = y′

5 Substitute for x and y into the original y = x becomes


equation. y′ = −x′

6 Sketch the original and image graphs. mx = 0


Note the origin is left unchanged. y
y=x

0 x

y' = –x'
C h a p t e r 6 Tr a n s f o r m a t i o n s u s i n g m a t r i c e s 279
Reflection in line y = x
To find this reflection, sketch the situation as described.
y
Remember to note the main points from the introduc-
tion to this section: (0, 1) y=x

=
1. corresponding points of the image and original
figures are equidistant from and perpendicular to the (1, 0)

=
mediator 0 x
2. length, angle and area of the image and original are
unchanged, hence it is a congruent transformation
3. any points of the original on the mediator are left unchanged.
We find that (1, 0) and (0, 1) map to each other, therefore

My = x = 0 1 .
1 0

WORKED Example 16
Find the equation of the image y = x2 reflected in the line y = x.
THINK WRITE
1 Sketch the relevant diagram to establish y
the reflection matrix.
(0, 1) y=x
=

(1, 0)
=

0 x

My = x = 0 1
1 0

Set up the initial matrix equation x′ = M x


2 y=x
and rearrange to have x and y as the y′ y
subject.
x = My−1= x x ′
y y′

Find the inverse of My = x.


1
= ------ 0 – 1 x′
3
–1 –1 0 y′

= 0 1 x′
1 0 y′

Continued over page


280 M a t h s Q u e s t M a t h s C Ye a r 1 1 f o r Q u e e n s l a n d

THINK WRITE
4 Multiply matrices to determine x and y. x = y′
y = x′

5 Substitute for x and y in the original y = x 2 becomes x′ = y′2


expression. y′ = ± x′

6 Sketch the original and image curves. y


y = x2
Note that the points (1, 1) and (0, 0) are
y=x
unchanged as they are on the mediator.
(1, 1) y' = √x'

0 x

y' = –√x'
y=x

Reflection in the line y = x tan q y


This line might be more easily recognised as A' (cos 2θ , sin 2θ )
y = mx, where m is the gradient of the line
1 = y = x tanθ
which passes through the origin.
y2 – y1 θ
Remember that the gradient m = ---------------
- θ
= A(1, 0)
x2 – x1
0 x
y2 – y1
and tangent ratio = ---------------
-.
x2 – x1 y
B(0, 1)
Therefore the tangent and gradient ratios provide
my = x tanθ
rise
the same information: -------- . = 90° – θ
run
Carefully examine these diagrams that illustrate
θ
reflection of the points (1, 0) and (0, 1) in the line 0 90° – 2θ x
y = x tan θ.
=
Note the following from these diagrams.
B'(cos (90° – 2θ ), – sin (90° – 2θ ))
For the point A(1, 0):
1. point A is reflected to a point equidistant from and perpendicular to the line
2. the angle from the x-axis to A′ is 2θ
3. the x-coordinate of the right-angled triangle is cos 2θ
4. the y-coordinate of this triangle is sin 2θ.
5. Hence point (1, 0) → (cos 2θ, sin 2θ).
For the point B(0, 1):
1. point B is reflected to a point equidistant from and perpendicular to the line
2. ∠MOB = 90° − θ therefore ∠MOB′ = 90° − θ
C h a p t e r 6 Tr a n s f o r m a t i o n s u s i n g m a t r i c e s 281
3. therefore ∠XOB′ = (90° − θ) − θ = 90° − 2θ
4. the x-coordinate = cos (90° − 2θ)
5. the y-coordinate = −sin (90° − 2θ) because the angle is in the fourth quadrant.
6. Hence point (0, 1) → [cos (90° − 2θ), −sin(90° − 2θ)].
7. Using trigonometric ratios, this simplifies to yield (sin 2θ, −cos 2θ). (Remember that
sin 30° = cos 60°, etc.)
Using all this information from the reflection of points (1, 0) and (0, 1) in the line

y = x tan θ yields: My = x tan θ = cos 2 θ sin 2 θ .


sin 2 θ –cos 2 θ

WORKED Example 17
Find the matrix for the reflection in the line y = 3 x.

THINK WRITE
1 Note that the sign applies only to the 3.
Use a sketch to express 3 as the tangent
ratio of some angle. –π
6

2 √3

–π
3
1

π
tan --- = 3
3

2 State the general reflection matrix in the line My = x tan θ = cos 2 θ sin 2 θ
sin 2 θ –cos 2 θ
π
y = x tan θ, then substitute --- for θ.
3
2π 2π
cos ------ sin ------
My = = 3 3
3x
2 π 2 π
sin ------ –cos ------
3 3

1 3
− --- -------
Evaluate these ratios using the following = 2 2
3

------3-
1---
triangle.
2 2

2
√3

–π π
2—
3 3
–1
282 M a t h s Q u e s t M a t h s C Ye a r 1 1 f o r Q u e e n s l a n d

WORKED Example 18
Find the image of the line y = −x − 1 as reflected in the line y = 3 x.

THINK WRITE

1 3
– --- -------
2 2
1 Use the matrix from the previous My = 3x =
example as My = 3 x. 3 1
------- ---
2 2

Set up the initial matrix x′ = My = x


2 3x
transformation and inverse y′ y
statement.
x = My–1= x′
3x
y y′

1--- 3
− -------
Find the inverse and multiply the x 1
= ----------------- 2 2 x′
3
matrices. y – 1--- – 3--- 3 1 y′
4 4 − ------- − ---
2 2

1 3
− --- -------
= 2 2 x′
y′
------3- 1---
2 2

1 3
x = − --- x′ + ------- y′
2 2
3 1
y = ------- x′ + --- y′
2 2

4 Substitute for x and y into the original y = −x − 1 becomes


equation. Make sure you carry
through the minus sign from the ------3- x′ + 1--- y′ = 1--- x′ − ------3- y′ − 1
function. 2 2 2 2

1--- 3 1 3
y′ + ------- y′ = --- x′ − ------- x′ − 1
2 2 2 2

1---------------
+ 3- 1– 3
5 Simplify and rationalise the y′ = ---------------- x′ − 1
denominators to find the equation of 2 2
the image line. y′ = ( 3 − 2)x′ + 1 − 3
C h a p t e r 6 Tr a n s f o r m a t i o n s u s i n g m a t r i c e s 283

THINK WRITE

6 Sketch the original and its image. To y


assist in graphing the image, a
y = √3x
calculator can be used to obtain
y′ = −0.27x′ − 0.73.

0 x
y' = (√3 – 2) x' + 1 – √ 3 1

y = –x – 1 a –1 0
0 1

2 a b 1 0
i (−3, −1) ii (−4, 2) In the reflections covered so far, the mediator has always passed through the origin. If 0 –1
iii (1, −3) iv (2, 4)
v (−3, 0) vi (2, −1)
we return to the original reflection in the line x = 1, it needs to be broken into two trans-
0 1
y formations: a reflection and a translation. Reflecting in the line x = 1 can be thought of c
1 0
(iv) (iv)' as reflection in the y-axis (x = 0) followed by a shift 2 to the right (x = 1 is 1 unit to the
right of the origin therefore the image would be 2 × 1 = 2 units to the right). Thus the
(ii)' (ii) d 1 0
(v)' (v) 0 –1
(i)' (vi)
0
(vi)' (i)
x
transformations matrices would be – 1 0 followed by the addition of 2 . Verify this and
(iii) (iii)'
0 1 0
mx = 0 by checking that (0, 0) → (2, 0). then 0
4

remember
remember e
– 1--2-

------3-
3
-------
2

--1-
3 a i y′ = −x′ iii y′ = −x′ iii y′ = x′ iv y′ = --12- ( 3 − 1)2x′ or y′ ≈ 0.268x′

1. Reflection is a congruent transformation. 2 2

2. Reflection occurs through a mediator, m. 0 –1


f
3. Reflection in the line y = x tan θ is represented by M = cos 2 θ sin 2 θ . –1 0
3 ------3- 3--- sin 2 θ –cos 2 θ
2 d i ( --2- − 2 , 2
+ 2)
3---------3-
ii (2 +
1---
3 , 2 3 − 1) iii (
–-----1- 3 3- –---------3-
---------
2
− 2
, 2
+ 2)
2 b i (3, 1) ii (4, −2)
iv (−1 + 2 3 , − 3 − 2) v ( 3--2- , 3 3
----------
2
) vi (−1 − 3
-------
2
, − 3 + --12- ) iii (−1, 3) iv (−2, − 4)
v (3, 0) vi (−2, 1)

6E
y

Reflections (iv)
(iii)'
(ii)
(i)'
b i y′ = x′2 iii y′ = −x′2 iii y′ = ± – x′

my = 0 (vi)' (v)(v)'
1 Write the matrices for the following reflections: (vi) 0 x
(i) (ii)'
a mx = 0 b my = 0 c my = x (iii)
d my = 2 e my = 3 x f my = −x (iv)'

WORKED 2 Find the images of each of the following points under the reflection given below.
Example
14
Sketch each original and its image.
3
a y-axis b x-axis c y = −x d y = ------- x
3
i (3, −1) ii (4, 2) iii (−1, −3) iv (−2, 4) v (3, 0) vi (−2, −1)
WORKED 3 Find the image of the following curves under each of the reflections given below.
Example
15,16,17,18
a y=x b y = x2 c y = 2x2 + 1 d y = −x2 2 c i (1, −3)
ii (−2, − 4)
3
i y-axis ii x-axis iii y = −x iv y = ------- x (part a only) iii (3, 1)
3 iv (− 4, 2)
3 c i y′ = 2x′2 + 1 ii y = −2x′2 − 1 iii y′ = ± –x′ –1
----------------- v (0, −3)
2
d i y′ = −x′2 ii y′ = x′2 iii y′ = ± x′ vi (1, 2)
284 M a t h s Q u e s t M a t h s C Ye a r 1 1 f o r Q u e e n s l a n d

SLE 4: Demonstrate the use of the transformation matrices (rotation, reflection, dilation) as an
application of 2 × 2 matrices to geometric transformations in the plane.
Dilations
So far we have investigated 4 kinds of trans-
formations. The translation shifted the figure
on the plane; the general linear transform-
ation produced an image that, on occasions,
bore little resemblance to its original.
The rotation and reflection transform-
ations are congruent transformations with
the original basically repositioned on the
plane. A dilation is a transformation in
which point P and image P′ are collinear
from a fixed point, usually the origin O, as
shown in the figure below.

P'

The length OP′ = kOP where k is referred


to as the dilation factor.
If k > 0, a dilation may be an enlargement (for k > 1) or a reduction (for 0 < k < 1).
A' A

B' B
A
B A'
B'

O C C'
k>1
O C' C
0<k<1

If k < 0 then the image of the original has been mapped through the origin in a
reverse direction.
A
In this diagram, k = − 1--- , therefore the
2
image appears half the distance from the B'
fixed point O and on the opposite side of
O to the original points. C' O C
In a dilation:
1. length and area are not preserved; the
A'
shape will appear similar, but not con- k<0 B
gruent to the original
2. the dilation d is denoted by the matrix Dk, x with the dilation factor of k given parallel
to the x-axis and the anchor line being the y-axis.
C h a p t e r 6 Tr a n s f o r m a t i o n s u s i n g m a t r i c e s 285
Dilation parallel to the x-axis
The dilation matrix Dk, x of the points (1, 0) and (0, 1)

under the dilation dk, x is given by Dk, x = k 0 y


0 1
(0, 1)
where
1. (0, 1) is left unchanged since it is on the anchor line (1, 0) (k, 0)
0 x
2. the x-coordinate is mapped k × 1 units away from the
anchor line.
This is shown graphically in the figure at right.
Dilation parallel to the x-axis can be thought of as pulling the plane away from the
fixed point or anchor line — in this case, the y-axis.

Dilation parallel to the y-axis


This dilation pulls the plane away from the x-axis y
so all points on the x-axis are anchored. (0, k)
The figure at right shows that
(0, 1)
Dk, y = 1 0
0 k
(1, 0)
where 0 x
1. (1, 0) is left unchanged since it is on the anchor line
2. the y-coordinate is mapped k × 1 units away from
the anchor line (the x-axis).

WORKED Example 19
Find the coordinates of the image of point (4, 3) under the dilation factor of -2 parallel to
the x-axis. Sketch the original point and its image.
THINK WRITE
1 Sketch the dilation and construct the y
matrix from the sketch.
(0, 1)

(–2, 0) (1, 0)
0 x

D–2, x = – 2 0
0 1

Write the general transformation matrix x′ = D–2, x 4


2
equation. y′ 3
Continued over page
286 M a t h s Q u e s t M a t h s C Ye a r 1 1 f o r Q u e e n s l a n d

THINK WRITE

Multiply the matrices to produce the x′ = –2 0 4


3
image coordinates. y′ 0 1 3

= –8
3
Point (4, 3) maps to image point (−8, 3) under a
dilation of −2 parallel to the x-axis.
4 Sketch the original point and its image. y

P' (–8, 3) P(4, 3)

0 x

WORKED Example 20
Find the equation of the image of y = 2x + 1 under the dilation d2, x. Sketch the original line
and its image.
THINK WRITE
1 Sketch the dilation and construct the y
matrix from the sketch.
(0, 1)

(1, 0) (2, 0)
0 x

D2, x = 2 0
0 1

Set up the general transformation x′ = D2, x x


2
matrix equation and rearrange to y′ y

have x as the subject.


x = D2,–1x x ′
y y y′

Find the inverse of D2, x and substitute it


1
= --- 1 0 x′
3
into the equation. 2 0 2 y′
1
=
---
2
0 x′
0 1 y′

4 Multiply the matrix equations. x = 1--- x′


2
y = y′
C h a p t e r 6 Tr a n s f o r m a t i o n s u s i n g m a t r i c e s 287

THINK WRITE
5 Substitute x and y in the original y = 2x + 1 becomes
equation and simplify. y′ = 2( --1- x′) + 1
2
y′ = x′ + 1
6 Sketch the original and its image. y
Note that (0, 1) remains unchanged y = 2x + 1
since it is on the anchor line of the
y-axis.
y' = x' + 1

(0, 1)

0 x

WORKED Example 21
Find the image of the circle x2 + y2 = 9 with a dilation factor of 1
---
3
parallel to the y-axis.
Sketch the original circle and its image.
THINK WRITE
1 Sketch the situation and use this to y
construct the dilation matrix.

(0, 1)

(0, 1–3 ) (1, 0)


0 x

1 0
D --1-, y =
1
3 0 ---
3

Set up the general transformation x′ = D 1---, y x


2
matrix equation and rearrange to put y′ 3 y
x as the subject. x x′
= D 1--–1
-, y
y y 3 y′

1
Calculate the inverse of D and
1
= -----
-
---
3
0 x′
3 1
substitute it into the equation. ---
3 0 1 y′

= 1 0 x′
0 3 y′
Continued over page
288 M a t h s Q u e s t M a t h s C Ye a r 1 1 f o r Q u e e n s l a n d

THINK WRITE
4 Multiply the matrices and write x = x′
expressions for x and y. y = 3y′

5 Substitute x and y into the original x2 + y2 = 9 becomes


equation and rearrange to fit the general (x′)2 + (3y′)2 = 9
equation of an ellipse. x′2 + 9y′2 = 9
2 2
x y
This can be written as ----- + ----- = 1
2 2
9 1
x y
since ----- + ----- = 1 is the general equation of an
2 2
a b
ellipse about the origin.
Therefore
a = 3 So the length of the semi-major axis is 3.
b = 1 So the length of the semi-minor axis is 1.

6 Sketch the original and its image. y


x2 + y2 = 9

0 x

x' 2 y' 2

9
+ —
1
=1

If you think about the original shape and its image as shown in this example you will
understand that the dilation factor of 1--- , in effect, shrinks the original shape, parallel to
3
the y-axis so the figure falls back towards the anchor line (the x-axis) and leaves all
points on the x-axis unchanged.

Dilation about the origin, dk


The previous dilations have been mapped parallel y
to an axis, where that axis has provided the anchor P'(kx , ky)
line for the stretching of the plane. However, a dilation (0, k)
about the origin does not anchor to a line, but rather to
a point — the origin. The diagram at right shows
(0, 1)
this stretch that results in both x and y coordinates P (x, y)
being mapped a dilation factor of k from the origin. (1, 0) (k, 0)
Therefore, if the original point is on the origin it will 0 x
map onto itself. Only the dilation factor is given in
the dilation matrix:

Dk = k 0
0 k
C h a p t e r 6 Tr a n s f o r m a t i o n s u s i n g m a t r i c e s 289
WORKED Example 22
Find the image of y = x2 under a dilation factor of −2 about the origin. Sketch the original
and its image.
THINK WRITE
1 Sketch the situation to construct the y
matrix.
(0, 1)
(–2, 0) (1, 0)
0 x

(0, –2)

D–2 = – 2 0
0 –2

Set up the initial transformation matrix x′ = D–2 x


2
equation. y′ y
x = D –2
–1 x′
y y′

Evaluate the inverse and multiply.


1
= --- – 2 0 x′
3
4 0 –2 y′

– 1--2- 0 x′
=
0 – 1--2- y′

x = − 1--- x′
2
y = − 1--- y′
2

4 Substitute for x and y into the original y = x becomes


2
equation and simplify.
− 1--- y′ = (− 1--- x′)2
2 2

= 1 2
--- x′
4

y′ = − 1--- x′2
2
5 Sketch the original and its image. y
The minus sign results in the image y = x2
reversing its position with respect to the
origin, and the factor of 2 results in the
broader parabola.
0 x

y' = – –21 x' 2


290 M a t h s Q u e s t M a t h s C Ye a r 1 1 f o r Q u e e n s l a n d

History of mathematics
M AU R I T S C O R N E L I U S E S C H E R ( 1 8 9 8 – 1 9 7 2 )
Portrait of M. C. Escher Original Escher prints are highly prized
© 2000 Cordon Art, Baarn,
Holland. All rights reserved.
possessions now, but it was not until 1951
that he actually began to earn a reasonable
During his life . . . income from his prints. Widely regarded as a
World War I and graphic artist, his designs have appeared on
World War II take
postage stamps, bank notes, T-shirts, jigsaw
place.
puzzles, record album covers, and, as he
Flight technology
develops — from remarked, in many scientific and
the Wright mathematical publications.
brothers first flight
in 1903 to the
moon landing in
1969.
Israel is established.
Maurits Escher is quoted as having said ‘I
never got a pass mark in math. … And just
imagine — mathematicians now use my
prints to illustrate their books. … I guess they
are quite unaware that I am ignorant about the
whole thing.’ Escher was born on
17 June 1898 in the Netherlands.
His early work was mainly concerned with
the representation of visible reality, such as
landscapes and buildings. However, he
gradually became more interested in studying ‘Relativity’ by M. C. Escher
© 2000 Cordon Art, Baarn, Holland. All rights reserved.
the abstract space-filling patterns used by the
Moors in mosaics found in Spain. He also His work has been held in high regard by
studied a paper by Polya on 17-plane both artists and mathematicians. He died in
crystallographic groups; however, instead of 1972, in the Netherlands.
using geometrical motifs, Escher used Research
animals, plants or people to fill the space on 1. Research Mobius strips, stellations,
his intricate prints. deformations, reflections, rotations,
Even though he professed ignorance of all Platonic solids, spirals and the
things mathematical, Escher incorporated hyberbolic plane.
many mathematical ideas in his works — 2. Look through scientific and
infinity, Mobius strips, stellations, mathematical publications to see if any
deformations, reflections, rotations, Platonic use Escher’s prints as covers or
solids, spirals and the hyberbolic plane. illustrative pages.

remember
remember
1. Dilation occurs in relation to an anchor line or point.
2. Dilation is not a congruent transformation.
C h a p t e r 6 Tr a n s f o r m a t i o n s u s i n g m a t r i c e s 291
y
1 a i 4, 1) (v) (v)'
ii (−8, 3) (ii)' (ii) (iii) (iii)'
iii (0, 3)

6F
(i) (i)'
iv (6, 0)
Dilations v (4, 5) 0 (iv) (iv)' x
vi (1, −3)
(vi) (vi)'

Sketch the original and its image for all questions.


WORKED
1 Find the image of each of the following points under the dilations given:
Example
a 2, parallel to the x-axis see top right b −3, parallel to the y-axis see bottom left
19
i (2, 1) ii (−4, 3) iii (0, 3) iv (3, 0) v (2, 5) vi ( 1--- , −3)
2
2 y
WORKED 2 Find the image of the line y = 3x − 2 under the following dilations: y = 3x – 2
Example (a)
20 a dilation factor 2 parallel to the x-axis y′ = 3--2- x′ − 2
b dilation factor −1 parallel to the y-axis y′ = −3x′ + 2
0 x
2 2
x y
WORKED 3 Find the image of the ellipse ----- + ----- = 1 with a dilation factor of
Example 4 9 (b)
21 2
a 1--- , parallel to the y-axis ----
x
2
b 4, parallel to the x-axis
2 - + 4y
-------- = 1
4 a y y = 2 x2 4 9 2 2
x y-
------ + ---- =1
4 Find the image of y = 2x with a dilation factor of 4
2
64 9
y' = –18 x' 2
a parallel to the x-axis y′ = 1--8- x′2 b parallel to the y-axis y′ = 8x′2
x

WORKED 5 Find the image of the line y = 3x − 2 under a dilation factor of 1


--- about the origin. y′ = 3x′ − 1---
2
Example 4
22
6 Find the image of each of the points in question 1 under the following dilations:
6 a
a 1--- , about the origin b −4, about the origin 3 y
i (1, 1--2- ) ii (−2, 1 1--2- ) 2

iii (0, 1 --12- ) iv (1 --12- , 0) 7 Find the image of the ellipse in question 3 with a dilation factor of (a)

v (1, 2 1--2- ) vi ( 1--4- , −1 1--2- ) a −2, about the origin b --1- , about the origin x
4 (b)
x
2
y
2
16y
2 6 b i (−8, − 4) ii (16, −12) x2 y
+—
2

9 =1

7 a ------ + ------ = 1 b 4x2 + ----------- = 1 iii (0, −12) iv (−12, 0)
4
16 36 9
v (−8, − 20) vi (−2, 12)
Shears
1b The final transformation discussed in this y
i (2, −3) ii (− 4, −9)
iii (0, −9) iv (3, 0)
chapter is that of shears, which can be thought of
v (2, −15) vi ( 2 , 9)
1--- as a push from one side that results in a change
y in shape. An example of this is seen when Push
(vi)' changing a rectangle into a parallelogram.
Where a dilation ‘pulls’ the plane from a certain 0 x
(v) anchor point or line, a shear pushes from one
(ii) (iii)
side and any points on the anchor line again
(i)
remain unchanged. y
(iv)
0 (iv)' x
A shear parallel to the x-axis (see the figure shown at
(i)' right) moves every point in the plane parallel to the x-axis
(vi)
by a distance proportional to its distance from the x-axis.
That is, points on the x-axis remain anchored while
(ii)' (iii)' points further away are pushed further from their original
position. 0 x

(v)'
292 M a t h s Q u e s t M a t h s C Ye a r 1 1 f o r Q u e e n s l a n d

Similarly a shear parallel to the y-axis (see the figure shown y


at right) moves every point in the plane parallel to the
y-axis by a distance proportional to its distance from the
y-axis.
The shear transformation matrix uses similar notation to
other linear transformations, where Sk, x denotes the shear
with a shear factor of k parallel to the x-axis and Sk, y denotes
the shear with a factor of k parallel to the y-axis. 0 x

That is, Sk, x = 1 k , Sk, y = 1 0 .


0 1 k 1
y
Shears parallel to the x-axis
As can be seen from the figure at right, point (1, 0)
remains unchanged because it is anchored to the
(0, 1) (k, 1)
x-axis while point (0, 1) is mapped to (k, 1). This
means that point (2, 1) will map to (2k, 1).
(1, 0)
0 x

Shears parallel to the y-axis (1, k)


The figure at right shows the point (0, 1) unchanged by
the shear because it lies on the y-axis while point
(0, 1)
(1, 0) is mapped to (1, k).
(1, 0)
0 x

WORKED Example 23
The vertices of a triangle are O(0, 0), A(2, 0) and B(2, 3). Find the image of these
points O′A′B′ under a shear factor of 2 parallel to the y-axis. Sketch the original and
its image.
THINK WRITE
1 Sketch the initial unit diagram and use y
this to determine the shear matrix.
(1, 2)

(0, 1)

(1, 0)
0 x

S2, y = 1 0
2 1
C h a p t e r 6 Tr a n s f o r m a t i o n s u s i n g m a t r i c e s 293

THINK WRITE

Set up the initial general transformation x′ = S2, y x


2
matrix equation. y′ y

3 Substitute S2,y and solve for each point For O(0, 0), the equation is:
in turn.
x′ = 1 0 0
y′ 2 1 0

= 0
0
O(0, 0) is unchanged because it is on So O(0, 0) → O′(0, 0)
the anchor axis.

For A(2, 0), the equation is:


x′ = 1 0 2
y′ 2 1 0

= 2
4
So A(2, 0) → A′(2, 4)

For B(2, 3), the equation is:


x′ = 1 0 2
y′ 2 1 3

= 2
7
Note that the y-coordinate is not So B(2, 3) → B′(2, 7)
actually multiplied by 2.

4 Sketch the image points with the y


original. B'(2, 7)

A'(2, 4)
B(2, 3)

O(0,0) x
A(2, 0)
294 M a t h s Q u e s t M a t h s C Ye a r 1 1 f o r Q u e e n s l a n d

WORKED Example 24
Find the image of the parabola y = 2x2 under the shear factor of 3, parallel to the y-axis.
Sketch the original and its image.
THINK WRITE
1 Sketch the initial unit diagram and use y
this to determine the shear matrix. (1, 3)

(0, 1)

0 (1, 0) x

S3, y = 1 0
3 1

Set up the initial general transformation x′ = S3, y x


2
matrix equation. y′ y

Rearrange in terms of x and y. x –1


= S 3, y x′
3
y y′

Find the inverse of S3, y .


1
= --- 1 0 x′
4
1 –3 1 y′

= x′
– 3x′ + y′
5 Multiply to find expressions for x and y. x = x′
y = −3x′ + y′

6 Substitute for x and y into the original y = 2x2 becomes


equation. −3x′ + y′ = 2x′2
y′ = 2x′2 + 3x′
= x′(2x′ + 3)

7 Sketch the image with the original. y′ = 0 when x = 0 and x = −1 1---


2
Find the intercepts to aid in sketching y
the image. Notice that y= 2x2
ii(i) the origin is anchored
y' = x'(2x' + 3)
i(ii) the positive x-values are pulled up
from the x-axis
(iii) the negative x-region seems to have
slipped back from the x-axis.
0 x
1 i a

6
y
P'
C h a p t e r 6 Tr a n s f o r m a t i o n s u s i n g m a t r i c e s 295

0
P
x
remember
remember
3 1. A shear can be thought of as a push parallel to an anchor line that transforms
all points on the plane by a distance proportional to their distance from the
anchor line.
2. A shear is not a congruent transformation.

1 i c
1 ii a (3, 0) b (2 1--4- , 1) c (−3 3--4- , 1) d (2 1--2- , −2) e (−3 1--4- , −5) f (−1 1--4- , −5)

–2

P
P'
0

–4
6G Shears

–7
3
P

y
P'

0
x

x
WORKED 1 Find the image of each of the following points under a shear factor of
Example eBook plus
1
23 i 2 parallel to the y-axis ii --- parallel to the x-axis
4
Digital docs:
1 i b a (3, 0) b (2, 1) c (−4, 1) d (3, −2) SkillSHEET 6.1
y e (−2, −5) f (0, −5) 1 i a (3, 6) b (2, 5) c (−4, −7) Building a
transformation matrix
Sketch each pair of original and image points. d (3, 4) e (−2, −9) f (0, −5)
5 P' SkillSHEET 6.2
Finding the image of
2 A parallelogram has vertices A(0, 0), B(1, 3), C(6, 3) and D(5, 0). a curve after
transformation
1 P Sketch the original shape and its image under the shear factor of 2a y
B'(10, 3)
0 x
2 a 3 parallel to the x-axis b 3 parallel to the y-axis C(6, 3)
C'(15, 3)

c −1 parallel to the x-axis d −1 parallel to the y-axis B(1, 3)

A = A' D(5, 0) x

WORKED 3 Find the image of each of the following curves under a shear factor of
Example
i 3 parallel to the y-axis ii 3 parallel to the x-axis eBook plus
24
a y=x b y = −x 3 d ii y
Digital doc:
c y=x 2
d y = 2x + 5 y = 2x + 5
WorkSHEET 6.2
Sketch the original and image curves. y' = 2–7 x' + 5–7

Transformations
2 2
x y - = 1 under the dilation of d . Give the equation of
1 Transform the ellipse ----- + ---- –2
4 1
the new ellipse, fully supporting your response with matrix operations and a
fully labelled diagram.
2 Design a set of 4 or 5 transformations that map a shape of your own choosing
to another shape on the plane. Your response should include all working and
x

fully labelled diagrams.


1 =1

3 With your current knowledge of transformations using matrix applications,


+—
2
y

investigate whether the following transformations are possible. You may need

x2
4

to consider a series of transformations.


=1

a A square into a straight line Possible; perform a singular linear transformation


0
y

4 =1

b A triangle into a square Not possible


2
y′ -
------- + ------
+—

4
2y

c A circle into a straight line Possible; perform a singular linear transformation



16
x2

d A square into a circle Not possible


2

16
x′

e A kite into a square Possible; one way is to perform a translation then a linear transformation
1
296 M a t h s Q u e s t M a t h s C Ye a r 1 1 f o r Q u e e n s l a n d

summary
Geometric transformations and matrix algebra
• A general transformation maps each point of the Cartesian plane onto some other
point of the plane.
• A translation, t, moves each x-coordinate a units parallel to the x-axis and each

y-coordinate b units parallel to the y-axis, such that x ′ = x + T, where


y′ y
(x′, y′) is the image of point (x, y) and T is the transformation matrix. The
translation t results in a congruent transformation.

Linear transformations

• A linear transformation, l, warps the plane such that x′ = L x . Linear


y′ y
transformations are not congruent transformations.

Rotations
• A rotation, r, rotates the plane about a fixed point to result in a congruent

transformation. The matrix representing the rotation is Rθ = cos θ –sin θ .


sin θ cos θ

Reflections
• A reflection, m, reflects every point of the original through a straight line called a
mediator and results in a congruent transformation. The reflection matrix

My = x tan θ = cos 2 θ sin 2 θ .


sin 2 θ –cos 2 θ

Dilations
• A dilation, d, transforms each point P to P′ where P and P′ are collinear with a fixed
point O. The matrix Dk, x represents a dilation of k units parallel to the x-axis
anchored from the y-axis. The matrix Dk represents a dilation factor of k units
through the origin and D–k represents the same dilation in the reverse direction.

Shears
• A shear, s, is a transformation like a ‘push’ from one side. The matrix Sk, x moves
every point in the plane parallel to the x-axis by a distance proportional to its
distance from the x-axis. Points on the x-axis remain unchanged.
C h a p t e r 6 Tr a n s f o r m a t i o n s u s i n g m a t r i c e s 297
7 y′ = −2x′ + 3 9 y′ = x′-
---
2
+1
y y
y' = 1–2 x' + 1

CHAPTER y' = –2x' + 3

review y = –x + 1
x

y = –2x + 2
x

1 State all congruent transformations. Translations, rotations and reflections


6A
2 Find the image of the following points under the translation T = – 1
3 6A
a (0, 0) (−1, 3) b (3, 1) (2, 4) c (4, −2) (3, 1)

3 The line y = 2x − 1 undergoes a succession of translations defined by T1 = 1 and


–4 6A
T2 = – 2 . Show that the order in which these occur has no effect on the result.
3 Check with your teacher.

4 Find the equation of the image of y = 2x2 under the translation of 2 . y′ = 2x′2 − 8x′ + 7
–1 6A

5 Find the image of points A(0, 1) and C(3, 2) under the transformation 2 3 .
–1 0 6B
A′(3, 0) C′(12, −3)
– 2 --13- 2 --23-
6 Find the matrix of the linear transformation which maps (1, 2) to (3, 2) and (3, 3) to (1, −1).
– 2 2--3- 2 1--3- 6B
7 Find the image of y = −x + 1 under the linear transformation 1 2 . Sketch the original
1 –1 6C
and image curves. y′ = −2x′ + 3 See graph top of page.

8 Find the image of the following points under the given anticlockwise rotations about the
origin. 6D
π
a (2, −1) where θ = −π (−2, 1) b (−3, 0) where θ = --- ( – 3--2- , − 3---------
3-
)
3 2
π
c (4, 1) where θ = − --- ( 5---------
2
2-
, − 3---------
2
2-
) d (2, 5) where θ = −60° (1 + 5--2- 3 , − 3 + 2 1--2- )
4
π
9 Find the image of the line y = −2x + 2 through a rotation of --- . Sketch the original and the
2 6D
image. y′ = ---2- + 1 See graph top of page.
x′

10 Find the image of each of the following points under the reflection as given:
a (3, −1) in the y-axis (−3, −1) b (2, 2) in the line y = x (2, 2) 6E
c (1, 2) in the line y = 3 x (− 1-
--
2
+ 3 , 3-
------
2
+ 1) d (−2, 1) in the line y = 2 (−2, 3)
298 M a t h s Q u e s t M a t h s C Ye a r 1 1 f o r Q u e e n s l a n d

11 Find the image of the line y = −3x under reflection in the line y = −x. Sketch the original and
6E the image.

12 Find the image of each of the following points under the dilation factors given:
6F a (2, 1), 2 units parallel to the y-axis (2, 2) b (2, 4), --1- unit about the origin (1, 2)
2
c (4, −1), −4 units about the origin (−16, 4) d (0, 2), −3 units parallel to the x-axis (0, 2)

13 Find the image of y = −3x under the dilation factor of 3 parallel to the x-axis. y′ = −x′
6F
14 Find the image of each of the following points under the given shear factor:
6G 1
a (3, 1), --- unit parallel to the y-axis (3, 2 1--2- ) b (−1, 1), 3 units parallel to the x-axis (2, 1)
2

15 Find the image of the curve y = 2x2 under a shear factor of 2 parallel to the y-axis. y′ = 2x′2 + 2x′
6G
Modelling and problem solving
1 Under a certain transformation, the circle (x − 2)2 + (y − 2)2 = 4 becomes (x + 2)2 + (y − 2)2 = 4.
It is claimed that three different transformations could have achieved this outcome. Investigate
this claim giving details of the possible transformations and their matrices of transformation.

2 A square, ABCD, formed by the points A(0, 0), B(1, 0), C(1, 1) and D(0, 1) is mapped to
A′(0, 0), B′(−3, 0), C′(−3, 3) and D′(0, 3) after two successive transformations. Analyse this
eBook plus mapping to determine the transformations involved, and find a single transformation matrix
that could achieve this result.
Digital doc:
Test Yourself
Chapter 6
2 Dilation of 3 about the origin followed 11 y′ = − 1--3- x′
1 Rotation of 90°: 0 – 1 y
1 0 by reflection in the y-axis: – 3 0
0 3
or reflection in y-axis: – 1 0 x
0 1 y' = – 1–3 x'
y = –3x
or translation 4 left: – 4
0
7
Introduction
to vectors

syllabus reference
Core topic:
Vectors and applications

In this chapter
7A Vectors and scalars
7B Position vectors in two
and three dimensions
7C Multiplying two
vectors — the dot
product
7D Resolving vectors —
scalar and vector
resolutes
7E Time-varying vectors
300 M a t h s Q u e s t M a t h s C Ye a r 1 1 f o r Q u e e n s l a n d

Vectors and scalars


• definition of a vector, Introduction
including standard unit
vectors i, j and k In mathematics, one of the important distinctions that we make is between scalar quan-
• relationship between vectors
and matrices tities and vector quantities. Scalar quantities have magnitude only; vector quantities
• two- and three-dimensional have direction as well as magnitude. Most of the quantities that we use are scalar, and
vectors and their algebraic
and geometric representation include such measurements as time (for example 1.2 s; 15 min), mass (3.4 kg; 200 t)
• operations on vectors
including addition, and and area (3 cm2; 400 ha).
multiplication by a scalar However, consider the measurement of velocity. A velocity of 20 km/h has both
• unit vectors
• scalar product of two vectors magnitude and direction. One of the boats shown below may travel 20 km/h north from
• resolution of vectors into Townsville, while the other one may travel 20 km/h east from the same point. Although
components acting at right
angles to each other they both are travelling at the same speed (magnitude) they are travelling in different
• calculation of the angle directions; they do not end up in the same place!
between two vectors
• applications of vectors in
both life-related and purely
mathematical situations

Now consider the force involved in Daniel


and Anna fighting over who gets to use the
television remote control.
Daniel exerts a force of 40 N and Anna
exerts a force of 50 N and they apply these
forces as shown.
In what direction will the remote control
move and what is the force in that direction?
That is, what is the resultant force?
The resultant force depends not only on the size
of each force but the direction in which the forces 40 N 150°
are applied. In the following discussion we will develop 50 N
techniques to find the resultant force.
A vector is a quantity that has magnitude and direction.
Chapter 7 Introduction to vectors 301
Vector notation
A vector is shown graphically as a line, with a head (end) and B
tail (start). The length of the line indicates the magnitude and
the orientation of the line indicates its direction.
In the figure at right, the head of the vector is at point B A
(indicated with an arrow), while the tail is at point A.
When writing this vector we can use the points A and B to indicate the start and end
points with a special arrow to indicate that it is a vector: AB . Some textbooks use a
single letter, in bold, such as w, but this is difficult to write using pen and paper, so w
˜
can also be used. The symbol (~) is called a tilde.
Equality of vectors
Since vectors are defined by both magnitude and direction:
two vectors are equal if both their magnitude and direction are equal.
In the figure, the following statements can be made: w
~
u= v
˜ ˜
u≠w (directions are not equal) ~u
˜ ˜
u≠z (magnitudes are not equal).
˜ ˜ ~v
Addition of vectors ~z
Consider a vector u which measures the travel from A to B and another vector, v ,
which measures the˜ subsequent travel from B to C. The net result is as if the person˜
travelled directly from A to C (vector w ). Therefore we can say that w = u + v .
˜ ˜ ˜ ˜
To add two vectors, take the tail of one vector and join it to the head of another.
The result of this addition is the vector from the tail of the first vector to the head
of the second vector.
Returning to Daniel and Anna who are fighting over the television remote control
(see page 300), we see that the forces they apply to the remote control unit can be rep-
resented as a sum of two vectors.

From this figure we are able to get a rough idea of the magnitude and direction of the
resultant force. In the following sections, we will learn techniques for calculating the
resultant magnitude and direction accurately.
The negative of a vector
If u is the vector from A to B, then – u is the vector from –u
˜ A. ˜ A ~
B to
We can subtract vectors by adding the negative of the B
second vector to the first vector. ~u
302 M a t h s Q u e s t M a t h s C Ye a r 1 1 f o r Q u e e n s l a n d

WORKED Example 1
Using the vectors shown at right, draw the result of:
a u+v b –u c u–v d v – u. ~v
˜ ˜ ˜ ˜ ˜ ˜ ˜
THINK WRITE
~u
a 1 Move v so that its tail is at the head a
of u . ˜ ~v ~v
˜

~u
2 Join the tail of u to the head of v to
find u + v . ˜ ˜ ~u + ~v
˜ ˜ ~v

~u

b Reverse the arrow on u to obtain – u . b –u


˜ ˜ ~

~u

c 1 Reverse v to get – v . c
˜ ˜
–v
~

~u
2 Join the tail of – v to the head of u to
get – v + u which˜ is the same as u˜– v –v
~ –v
~+~
u
or u +˜ ( – v˜ ) . ˜ ˜
˜ ˜
~u

d 1 Reverse u to get – u . The vectors are d


˜ properly’
now ‘aligned ˜ with the head of
~v
– u joining the tail of v .
˜ ˜
–u
~
2 Join the tail of – u to the head of v to
get v – u . ˜ ˜ ~v – ~u
˜ this is the same as ( – u + v ) . ~v
Note˜ that
˜ ˜
–u
~

Multiplying a vector by a scalar N


Multiplication of a vector by a number (scalar) affects only
the magnitude of the vector, not the direction. For example, W E
if a vector u has a direction of north and a magnitude of 10,
˜
then the vector 3u has a direction of north and magnitude S
of 30. ˜
If the scalar is negative, then the direction is reversed. ~u 3u
~ –2u
~
Therefore, – 2u has a direction of south and a magnitude
of 20. ˜
Chapter 7 Introduction to vectors 303
WORKED Example 2
Use the vectors shown at right to draw the result of:
~s
a 2r + 3s b 2s – 4r .
˜ ˜ ˜ ˜ ~r
THINK WRITE
a 1 Increase the magnitude of r by a factor a
of 2 and s by a factor of 3.˜
˜ 3s
~

2r~
2 Move the tail of 3s to the head of 2r .
Then join the tail of ˜
˜ 2r to the head of 2r~ + 3s
~
3s
3s to get 2r + 3s . ˜ ~ 3s
~
˜ ˜
2r~

b 1 Increase the magnitude of s by a factor b


of 2 and r by a factor of 4.˜ 2s~
˜
4r~
2 Reverse the arrow on 4r to get – 4r .
˜ ˜
2s~

–4r~
3 Join the tail of – 4r to the head of 2s . 2s~ – 4r
~
˜ ˜
2s~

–4r~

WORKED Example 3
The parallelogram ABCD can be defined by the two vectors b D
˜ C
and c .
˜
In terms of these vectors, find:
a the vector from A to D ~c
b the vector from C to D
c the vector from D to B. A B
~b
THINK WRITE
a The vector from A to D is equal to the a AD = c
vector from B to C since ABCD is a ˜
parallelogram.
b The vector from C to D is the reverse of D b CD = – b
to C which is b . ˜
˜
c The vector from D to B is obtained by c DB = – c + b
adding the vector from D to A to the vector = b –˜ c ˜
from A to B. ˜ ˜
304 M a t h s Q u e s t M a t h s C Ye a r 1 1 f o r Q u e e n s l a n d

WORKED Example 4
A cube PQRSTUVW can be defined by the three vectors a , b and c T W
as shown at right. ˜ ˜ ˜
Express in terms of a , b and c : R
S
a the vector joining˜ P ˜to V ˜
b the vector joining P to W ~c
U V
c the vector joining U to Q
d the vector joining S to W ~b
P
e the vector joining Q to T. ~a Q

THINK WRITE
All of the opposite sides in a cube are equal in
length and parallel. Therefore all opposite
sides can be expressed as the same vector.
a The vector from P to V is obtained by a PV = a + b
adding the vector from P to Q to the vector ˜ ˜
from Q to V.
b The vector from P to W is obtained by b PW = a + b + c
adding the vectors P to V and V to W. ˜ ˜ ˜
c The vector from U to Q is obtained by c UQ = –b + a
adding the vectors U to P and P to Q. = a ˜– b ˜
˜ ˜
d The vector from S to W is obtained by d SW = a + b
adding the vectors S to R and R to W. ˜ ˜
e The vector from Q to T is obtained by e QT = –a + c + b
adding the vectors Q to P, P to S and S to T. = b +˜ c –˜ a ˜
˜ ˜ ˜

WORKED Example 5
A boat travels 30 km north and then 40 km west.
a Make a vector drawing of the path of the boat.
b Draw the vector that represents the net displacement of the boat.
c What is the magnitude of the net displacement?
d Calculate the bearing (from true north) of this net displacement vector.
THINK WRITE
a 1 Set up vectors (tail to head), one a N
W
~ (40 km)
pointing north, the other west.
2 Indicate the distances as 30 km and N (30 km) W
~ E
40 km respectively.
S

b Join the tail of the N vector with the head b W


of the W vector. ˜ ~ (40 km)
˜ N (30 km)
~N + W
~ ~
Chapter 7 Introduction to vectors 305

THINK WRITE
c 1 Let R km = length of N + W . c W
˜ ˜ ~ (40 km)
N (30 km)
~
R=~
~ N+W
~

2 The length (magnitude) of R can be


R = 30 2 + 40 2
calculated using Pythagoras’˜ theorem.
= 900 + 1600
= 50 km
d 1 Indicate the angle between N and d W
N + W as θ. ˜ ~ (40 km)
˜ ˜ N (30 km)
~N + W
~ ~
θ

2 Use trigonometry to find θ. 40


sin θ = ------
50
= 0.8
θ = 53.13°
3 The true bearing is 360° minus 53.13°. Therefore the true bearing is:
= 360° − 53.13°
= 306.87°

remember
remember
1. Definition: A vector is a quantity that has magnitude and direction.
2. Equality of vectors: Two vectors are equal if both magnitude and direction are equal.
3. Addition of vectors: To add two vectors, take the tail of one vector and join it
to the head of the other. The result of addition is the vector from the tail of the
a i first vector to the head of the second.
~s r +~s
~
~s 4. Subtraction of vectors: Subtract vectors by adding the negative of the second
~r
ii vector to the first vector.
~r
–s
~ ~s
5. Multiplication of a vector by a scalar: Multiply the magnitude of the vector by
~r – ~s the scalar; maintain the direction of the original vector.
iii –r
~
~s
~s – ~r

7A Vectors and scalars


WORKED 1 a Draw the result of:
Example
1 i r+s ii r – s iii s – r
˜ ˜ ˜ ˜ ˜ ˜ ~s
WORKED
Example
b Draw the result of: ~r
2 i 2r + 2s ii 2r – 2s iii 3s – 4r 3s~
–4r
~
˜ ˜ ˜ ˜ ˜ ˜
3s~ – 4r
~s ~
Same as 1 a i except scaled by a factor of 2. Same as 1 a ii except scaled by a factor of 2.
~r
306 M a t h s Q u e s t M a t h s C Ye a r 1 1 f o r Q u e e n s l a n d

WORKED 2 The pentagon ABCDE at right can be defined by the four C


Example
vectors, s , t , u and v . ~v ~u
3 ˜ ˜ of ˜ these˜ 4 vectors:
Find in terms B D
a the vector from A to D s + t
˜ ˜
b the vector from A to B s + t + u + v ~t
˜ ˜ ˜ ˜
c the vector from D to A –s – t A ~s E
˜ ˜
d the vector from B to E –v – u – t
˜ ˜ ˜
e the vector from C to A. –u – t – s
˜ ˜ ˜

3 multiple choice
A girl travels 4 km north and then 2 km south. What is the net displacement vector?
A 6 km north B 6 km south C 2 km north
D 2 km south E −2 km north

4 In the rectangle ABCD, the vector joining A to B is denoted B


~v C
by u and the vector joining B to C is v . Which pairs of points
are ˜joined by: ˜
~u
a u + v ? A to C b u – v ? D to B
˜ ˜ ˜ ˜
c v – u ? B to D d 3u + 2v – 2u – v ? A to C A D
˜ ˜ ˜ ˜ ˜ ˜

5 multiple choice
Consider the following relationships between vectors u , v and w .
˜ ˜ ˜
i u = 2v + w
˜ ˜ ˜
ii w = v – u
˜ ˜ ˜
Which of the following statements is true?
A u = w B u = v C u = 2--3- v D u = 3--2- v E u = 3v
˜ ˜ ˜ ˜ ˜ ˜ ˜ ˜ ˜ ˜
WORKED 6 A rectangular prism (box) CDEFGHIJ can be J I
Example
4
defined by three vectors r , s and t as shown F
at right. ˜ ˜ ˜ E
Express in terms of r , s and t : ~t H
G
a the vector joining˜ C˜to H ˜r + s ~s C
˜ ˜ D
b the vector joining C to J s + t ~r
˜ ˜
c the vector joining G to D r – s
˜ ˜
d the vector joining F to I r + s
˜ ˜
e the vector joining H to E t – s
˜ ˜
f the vector joining D to J s + t – r a, b
˜ ˜ ˜
g the vector joining C to I r + s + t Flight
˜ ˜ ˜
h the vector joining J to C. – s – t path
˜ ˜
WORKED 7 A pilot plans to fly 300 km north then 400 km east.
Example
a Make a vector drawing of her flight plan. eBook plus
5
b Show the resulting net displacement vector.
Digital doc:
c Calculate the length (magnitude) of this net displacement vector. 500 km EXCEL Spreadsheet
d Calculate the bearing (from true north) of this net displacement vector. Position vector
53.1° clockwise from N
Chapter 7 Introduction to vectors 307
SLE 3: Use addition and 8 Another pilot plans to travel 300 km
subtraction in
life-related east, then 300 km north-east. Show that
situations. the resultant bearing is 67.5 degrees.
How far east of its starting point has the
plane travelled? 512.1 km; find bearing
using trigonometry
9 An aeroplane travels 400 km west, then
600 km north. How far is the aeroplane
from its starting point? What is the
10
Each part of answer bearing of the resultant displacement?
has coordinate labelled 721.1 km, 326.3° (clockwise from N)
a, b, . . . j. The original 10 On a piece of graph paper draw a vector, a , that is 3 units east and 5 units north of the
vectors a and b are ˜
origin. Draw another vector, b , that is 5 units east and 3 units north of the origin.
also drawn. ˜ ˜ ˜
On the same graph paper, draw the following vectors.
h
15
a f
b a a+b b a + 3b c a–b d b–a
i a ˜ ˜ ˜ ˜ ˜ ˜ ˜ ˜
5 ~ e 3b – 4a f 0.5a + 2.5b g a – 2.5b h 4a
c ~b ˜ ˜ ˜ ˜ ˜ ˜ ˜
–15 g –5 d 5 15
–5 i 2.5a – 1.5b j b – 2.5a
j e ˜ ˜ ˜ ˜
–15
11 Find the direction and magnitude of a vector joining point A to point B, where B is
10 m east and 4 m north of A. Magnitude = 10.77, direction 68.2° True.
12 Consider a parallelogram defined by the vectors a and b ,
and its associated diagonals, as shown at right. Show˜ ˜
that ~b
the vector sum of the diagonal vectors is 2a .
˜ ~a
13 Show, by construction, that for any vectors u and v :
˜ ˜
3 ( u + v ) = 3u + 3v
˜ ˜ ˜ ˜
(This is called the Distributive Law.)
14 Show, by construction, that for any three vectors a , b and c :
˜ ˜ ˜
12–18 Check with (a + b) + c = a + (b + c)
your teacher. ˜ ˜ ˜ ˜ ˜ ˜
(This is called the Associative Law.)
15 Show, by construction, that for any two vectors r and s :
˜ ˜
3r – s = – ( s – 3r )
˜ ˜ ˜ ˜
16 As you will learn shortly, vectors can be represented by two values: the horizontal
(or x) component and the vertical (or y) component.
Consider the vector w , defined by joining the origin to the point (4, 5), and the
˜
vector v , defined by joining the origin to (2, 3). Find the horizontal and vertical com-
˜
ponents of each vector.
Demonstrate, graphically, that the sum w + v has an x-component of 6 (that is,
˜ ˜
4 + 2), and a y-component of 8 (that is, 5 + 3).

17 Using the same vectors, w and v , as in question 16, demonstrate graphically that the
˜ ˜
difference vector, w – v , has an x-component of 2 and a y-component of 2.
˜ ˜
18 Using the same vectors, w and v , as in question 16, demonstrate graphically that:
˜ ˜
a the vector 4w has an x-component of 16 and a y-component of 20
˜
b the vector – 2v has an x-component of −4 and a y-component of −6.
˜
308 M a t h s Q u e s t M a t h s C Ye a r 1 1 f o r Q u e e n s l a n d

19 Using the results from questions 16, 17 and 18, what can you deduce about an
algebraic method (as opposed to a graphical method) of addition, subtraction and
multiplication of vectors? One can deduce that x and y components can be added/subtracted/
multiplied separately.
20 multiple choice D

~a O b
~
In terms of vectors a and b in the figure above, the vector joining O to D is given by:
˜ ˜
A 3a + 3b B 2a + 4b C 3b – 2a D 2a – 3b E none of these
˜ ˜ ˜ ˜ ˜ ˜ ˜ ˜
21 multiple choice E

~a O b
~
In terms of vectors a and b , the vector joining E to O above is:
˜ ˜
A 3a + 4b B 4b – 3a C 3a – 4b D – 3 a – 4b E none of these
˜ ˜ ˜ ˜ ˜ ˜ ˜ ˜
22 A girl walks the following route: 400 m north — 300 m east — 200 m north —
500 m west — 600 m south — 200 m east
Make a vector drawing of these six paths. What is the net displacement vector? 0
˜
23 Which of the following are vector quantities?
speed velocity displacement force volume angle Displacement, velocity, force

24 Which of the following are scalar quantities?


speed time acceleration velocity length displacement Speed, time, length

25 A 2-dimensional vector can be determined by its length and its angle with respect to
(say) true north. What quantities could be used to represent a 3-dimensional vector?
1 magnitude and 2 angles

Position vectors in two and


three dimensions
Introduction
As a vector has both magnitude and direction, it can be represented in 2-dimensional
planes or 3-dimensional regions in space. (It is easier to discuss 2-dimensional vectors
as they fit the page nicely!)
y
Position vectors in two dimensions B
~u
In the figure at right, the vector u joins the point A to
point B. ˜ A
An identical vector can be considered to join the origin x
with the point C.
It is easy to see that u is made up of two components: y
C
one along the x-axis and ˜one parallel to the y-axis. Let i be ~u
a vector along the x-axis, with magnitude 1. Similarly,˜ let j
~
j be a vector along the y-axis, with magnitude 1. ~i D x
˜
Chapter 7 Introduction to vectors 309
We can say the vector u is the position vector of point C
relative to the origin. ˜
With vectors, it is equivalent to travel along u from the y
˜ to D and
origin directly to C, or to travel first along the x-axis C
3
then along the y-axis to C. In either case we started at the ~u 3j
origin and ended up at C. Clearly then, u is made up of some ~
multiple of i in the x-direction and some˜ multiple of j in the
O D x
y-direction. ˜ ˜ 6i~
For example, if the point C has coordinates (6, 3) then
u = 6i + 3 j .
˜ ˜ ˜

Position vectors in three dimensions z

In 3 dimensions, a point in space has 3 coordinates, so a third


component, along the z-axis, is needed. Let k be a vector 0
˜
along the z-axis, with magnitude 1. The orientation is now y
such that the x-axis is coming directly out from the page as
shown at right. x
For example, if the point C has coordinates z
(6, −2, 4), then its position vector would be C (6, –2, 4)
denoted by v = 6i – 2 j + 4k . ~v
˜ ˜ ˜ 0
˜ 4k 6i y
~ ~
–2j
Relationship between vectors ~
x
and matrices
From our earlier work on matrices, we know that a matrix is an array that can store
numbers. These numbers could be the components of a vector, so for the vector
a
v = ai + b j + ck , the components could be expressed as b .
˜ ˜ ˜ ˜
c
So as a matrix, the components of u = 6i + 3 j could be expressed as 6 and the
˜ ˜ ˜ 3
6
components of v = 6i – 2 j + 4k could be expressed as – 2 .
˜ ˜ ˜ ˜
4
We will look further at the relationship between vectors and matrices later in the
chapter.

The magnitude of a vector


By using Pythagoras’ theorem on a position vector, we can find its length, or magnitude.
Consider the vector u at right.
˜ y
The magnitude of u , denoted as u or u, is given by: C (6, 3)
˜ ˜
2 2 ~u 3j
u = 6 +3 ~
˜ 6i x
u = 45 ~
˜
=3 5
310 M a t h s Q u e s t M a t h s C Ye a r 1 1 f o r Q u e e n s l a n d

Consider, now, the general position vector relative to the origin, for the point with
coordinates (x, y):
u = xi + y j
˜ ˜ ˜
The magnitude of a vector, u = xi + y j , is given by u = x 2 + y 2 .
˜ ˜ ˜ ˜
The direction of a vector
From what we already know about trigonometry, we can work out the angle (θ ) that u
˜
makes with the positive x-axis (that is, anticlockwise from the positive x-axis). This
gives us the direction of u .
˜
This angle can be calculated as: y
C (6, 3)
θ = tan−1 ( 3--6- ) ~u 3j
= tan−1 0.5 θ ~
6i x
= 0.464 radians ~
= 26.6°
The result obtained by this method needs to be adjusted if the angle is in the 2nd,
3rd, or 4th quadrants.
y
The direction of a vector, u = xi + y j , is given by θ = tan−1 --- with appropriate
x
adjustment depending on ˜the quadrant
˜ ˜ involved.

WORKED Example 6
Using the vector shown at right, find: y
a the magnitude of u
b the direction of u ˜(express the angle with respect to the positive θ x
x-axis) ˜ ~u
c the true bearing of u . (3, –5)
˜
THINK WRITE
a 1 Use Pythagoras’ theorem or the rule a u = 3 2 + ( –5 )2
for magnitude of a vector with the ˜
x- and y-components 3 and −5
respectively.
2 Simplify the surd. u = 9 + 25
˜
= 34 (= 5.831 to 3 decimal places)
b 1 The angle is in the 4th quadrant b
since x = 3 and y = −5.
˙
2 Use trigonometry to find the angle θ, θ = tan−1  –-----
5
3
-
from the x- and y-component values.
3 Use a calculator to simplify. θ = −59°
c 1 The negative sign implies that the c
direction is 59° clockwise from the
x-axis.
2 The true bearing from north is the true bearing = 90° + 59°
angle measurement from the positive = 149°.
y-axis to the vector u .
˜
Chapter 7 Introduction to vectors 311
Graphics Calculator tip! Finding the magnitude and direction
of a vector in two dimensions
Consider the vector shown in worked example 6. The vector can be expressed in com-
ponent form (or rectangular form) as 3i – 5 j . One way of finding the magnitude and
˜ the
direction (the angle the vector makes with ˜ positive x-axis) is to convert the vector
from rectangular form to polar form using a graphics calculator.

For the Casio fx-9860G AU


1. Press MENU and select RUN-MAT. Decide whether
you want the angle displayed in degrees or radians.
In this example we want degrees. Press SHIFT [SET
UP] and alter the setting for Angle if necessary. Press
F1 (Deg) for degrees.

2. Press EXE to accept this setting. Next press OPTN


s

then F6 ( ) to show more options.


s

3. Press F5 (ANGL) and then F6 ( ) for more


options. There are three options shown. The one we
need is Pol(, which converts vectors in rectangular
form to polar form.

4. Press F1 (Pol() and enter the components 3 and −5,


separated by a comma (press , ). Press ) to close
the set of brackets.

5. Press EXE to perform the conversion. The first


number (which is highlighted) is the magnitude of
the vector and the second number is the angle the
vector makes with the positive x-axis.

For the TI-Nspire CAS


1. Decide whether you want the angle displayed in
degrees or radians. In this example we want degrees.
Press c to access the home screen and select
8: System Information followed by 2: System
Settings. Press e until you reach Angle and then
select Degree by using the arrow keys. Press · to
accept this setting.
2. Continue pressing e until you highlight OK. Press
· to select OK.
312 M a t h s Q u e s t M a t h s C Ye a r 1 1 f o r Q u e e n s l a n d

3. We will first define 3i – 5 j as the vector u and then


˜
convert this to polar form.˜Press / N and select
1: Add Calculator to open a new calculator
document. Press b and select 1: Actions.

4. Select 1: Define. Press U (to name the vector as u)


then = followed by / ( to set up square
brackets. Enter the components 3 and −5, separated
by a comma (press ,), within the square brackets.
Use the right arrow key to move the cursor to the
right of the second square bracket and press ·.

5. Press U to nominate vector u. Then press b and


select 7: Matrix & Vector followed by C: Vector.

6. Select 4: Convert to Polar and press · to perform


the conversion. The exact values for the magnitude
and the angle are shown.

7. To obtain the approximate values, press / ·.

Note that if no other operations are required, you can


enter the vector directly by using square brackets. You
then continue in the same way to convert to polar form.

Unit vectors
As we have seen, any vector u is composed of x and y (and z, in 3 dimensions)
components denoted by xi , y j˜ (and zk ). The vectors, i , j and k are called unit
vectors, as they each have ˜a magnitude
˜ of˜ 1. This allows us˜ to˜ resolve˜ a vector into its
components.
Chapter 7 Introduction to vectors 313
If a 2-dimensional vector u makes an angle of θ with the positive x-axis and it has
a magnitude of u then we ˜ can find its x- and y-components using the formulas:
˜
x = u cos q
˜
y = u sin q
˜

WORKED Example 7
Consider the vector u , whose magnitude is 30 and whose N
˜ y
bearing (from N) is 310°. Find its x- and y-components and
~u W E
write u in terms of i and j .
˜ ˜ ˜
x S
THINK WRITE 310°
1 Change the bearing into an angle with y
respect to the positive x-axis (θ).
The angle between u and the positive 50°
2
˜ ~u
y-axis is 360° − 310°. θ
x

3 Calculate θ. θ = 90° + 50°


= 140°
4 Find the x- and y-components using x = u cos θ
trigonometry. ˜
= 30 cos 140°
= –22.98
y = u sin θ
˜
= 30 sin 140°
= 19.28
5 Express u as a vector. u = – 22.98i + 19.28 j
˜ ˜ ˜ ˜

Graphics Calculator tip! Finding the x- and y-components


of a vector
Vectors can be expressed in different forms. In the graphics calculator tip on page 311,
we converted a vector in rectangular form to polar form so we could obtain the
magnitude and direction of the vector. The reverse process can also be performed.
Consider the vector in Worked example 7 where the magnitude is 30 and the angle to
the positive x-axis is 140°. To find the x- and y-components, we convert the vector to
rectangular form. Check that the calculator is set to degrees.

For the Casio fx-9860G AU


1. Press MENU and select RUN-MAT. Press OPTN
s

then F6 ( ) to show more options. Press F5


s

(ANGL) and then F6 ( ) for more options. There


are three options shown. The one we need is Rec(,
which converts vectors in polar form to rectangular
form.
314 M a t h s Q u e s t M a t h s C Ye a r 1 1 f o r Q u e e n s l a n d

2. Press F2 (Rec() and enter 30 for the magnitude


followed by a comma. Then enter 140 for the angle
and close the set of brackets.

3. Press EXE to perform the conversion. The first


number (which is highlighted) is the x-component
of the vector and the second number is the
y-component. Hence u = – 22.98 i + 19.28 j .
˜ ˜ ˜

For the TI-Nspire CAS


1. Open a new Calculator document (press / N and
select 1: Add Calculator). To enter the vector in
polar form, first press / ( to set up square
brackets. Then enter 30 for the magnitude followed
by a comma. Press / k and highlight the angle
symbol (∠).

2. Press · to insert the angle symbol on the calculator


page and enter 140 for the angle. Move the cursor to
the right of the second square bracket. The vector is
now entered in polar form.

3. Press b and select 7: Matrix & Vector followed by


C: Vector. Select 5: Convert to Rectangular and
press · to perform the conversion. The exact values
for the x- and y-components are shown. To obtain the
approximate values, press / ·. Hence
u = – 22.98 i + 19.28 j .
˜ ˜ ˜

WORKED Example 8
A bushwalker walks 16 km in a direction of bearing 050°, then walks 12 km in a direction
of bearing 210°. Find the resulting position of the hiker giving magnitude and direction
from the starting point.
THINK WRITE
1 Draw a clear diagram to represent 240°
210°
the situation. y
m 30°
6k
m

50° a~1
~b 12 k

x
Chapter 7 Introduction to vectors 315

THINK WRITE
2 Express position vectors as angles a = 16 cos 40° i + 16 sin 40° j
from the direction of the x-axis. b˜ = 12 cos 240°˜ i + 12 sin 240° ˜ j
Simplify position vectors. ˜a = 12.2567 i + ˜10.2846 j ˜
3
b˜ = −6 i − 10.3923
˜ j ˜
Use the Triangle Law ˜ a + b˜ = (12.2567 ˜− 6) i + (10.2846 − 10.3923) j
4 a
of addition of vectors. ~ ˜ ˜ = 6.2567 i − 0.1077
˜ j ˜
b
θ ~ ˜ ˜
a b
~+~
y
5 Find the angle θ. θ = tan–1 --
x
– 0.1077
= tan–1 -------------------
6.2567
= −0.986°
2 2
6 Find the magnitude. |a + b | = x +y
˜ ˜
2 2
= 6.26 + ( – 0.11 )
= 39.2
= 6.26
7 State the resultant vector in terms of Final position is 6.26 km from the starting point in
magnitude (distance) and direction a direction of bearing 091°.
(bearing).

Clearly, in 3 dimensions, this is much more difficult as you need two angles (for
instance, an angle with respect to the x-axis and another with respect to the z-axis).
Unit vectors can also be found in the direction of any vector. This is merely the
original vector divided by its magnitude.
The unit vector of any vector u , in the direction of u denoted by û , is:
˜ ˜ ˜
u
û = -----
˜ -
˜ u
˜

WORKED Example 9
Find the unit vector in the direction of u . C (6, 3)
˜ ~u 3j
~
6i~
THINK WRITE

1 Express the vector in component form. u = 6i + 3 j


˜ ˜ ˜
2 Compute the magnitude of the vector u . u = 6 + 32
2
˜ ˜
= 45
= 3 5
Continued over page
316 M a t h s Q u e s t M a t h s C Ye a r 1 1 f o r Q u e e n s l a n d

THINK WRITE
6 3
3 Divide each component of the original û = ---------- i + ---------- j
˜ 3 5˜ 3 5˜
vector by the magnitude to get û .
˜ 2 5 5
= ---------- i + ------- j
5 ˜ 5˜
2 2
4 Comfirm that û has a magnitude of 1. û = x + y
˜ ˜
20 5
= ------ + ------
25 25
25
------
=
25
=1

Graphics Calculator tip! Finding the unit vector in the


direction of the vector
As seen above, the unit vector is obtained by dividing each component by the
magnitude of the vector. We can also use a graphics calculator to achieve this. Consider
the vector u in Worked example 9 which can be expressed in component form as
6i + 3 j . ˜
˜ ˜
For the Casio fx-9860G AU
1. First find the magnitude of u . Repeat the steps
shown on page 311 to change˜ the vector into
polar form. The magnitude is 6.7082.
2 2
(Alternatively, calculate 6 + 3 .)

2. Consider the components of u as the matrix 6 .


Press EXIT until you return to ˜ the MAT 3

screen. Enter the matrix using square brackets (each


row is listed within a set of square brackets) as
shown in the screen at right.

3. Press Æ and then SHIFT [Mat] followed by


ALPHA [U] to store this matrix as matrix U.

4. Press EXE to display the matrix and then press


EXE again to return to your calculation screen.
Next we need to divide each component of the vector
by the magnitude of the vector to obtain the unit
vector. Since we are using matrices, we need to
change this to multiplying matrix U by the reciprocal
of the magnitude.
Chapter 7 Introduction to vectors 317
Enter 6.7082 and press SHIFT [x -1] to obtain the
reciprocal of the magnitude. Press ¥ and then enter
matrix U by pressing SHIFT [Mat] followed by
ALPHA [U].

5. Press EXE . The approximate values for the


components of the unit vector are shown. Hence
û = 0.8944i + 0.4472 j .
˜ ˜ ˜

For the TI-Nspire CAS


1. Begin with a new calculator page. To define the
vector as u, first press b and select 1: Actions
followed by 1: Define. Press U (to name the vector
as u) then = followed by / ( to set up square
brackets. Enter the components, separated by a
comma, within the square brackets. Move the cursor
to the right of the second square bracket and press
·.

2. Press b and select 7: Matrix & Vector followed by


C: Vector.

3. Select 1: Unit Vector and press U to enter the


required vector. Close the set of brackets by
pressing ) and then press ·. The exact values
for the components of the unit vector are shown.
2 5 5
Hence û = ---------- i + ------- j .
˜ 5 ˜ 5˜

These steps also apply to finding the unit vector for a


three-dimensional vector.

y
Locating vectors A
In the figure at right, a is the position vector of point A ( OA ) a
~
˜ B
and b is the position vector of point B ( OB ) relative to the origin. b
˜ ~
The vector describing the location of A relative to B ( BA ) is O x
easily found using vector addition as –b + a or a – b .
˜ ˜ ˜ ˜
Similarly, the vector describing the location of B relative to A ( AB ) is b – a . This
result also applies in 3 dimensions and can be formalised as follows. ˜ ˜

If A and B are points defined by position vectors a and b respectively, then


˜ ˜
AB = b – a
˜ ˜
318 M a t h s Q u e s t M a t h s C Ye a r 1 1 f o r Q u e e n s l a n d

WORKED Example 10
a Find the position vector locating point B (3, −3) from point A (2, 5).
b Find the length of this vector.
THINK WRITE

a 1 Express the point A as a position vector a . a Let OA = a = 2i + 5 j


˜ ˜ ˜ ˜
2 Express the point B as a position vector b . Let OB = b = 3i – 3 j
˜ ˜ ˜ ˜
3 The location of B relative to A ( AB ) is b – a . AB = b – a
˜ ˜ ˜ ˜
= 3i – 3 j – ( 2i + 5 j )
˜ ˜ ˜ ˜
= i – 8j
˜ ˜
b The length of AB is b – a . b AB = b – a
˜ ˜ ˜ ˜
= 1 2 + ( –8 )2
= 65 (or 8.06)

Magnitudes in three dimensions


Pythagoras’ theorem also applies in the case of a 3-dimensional line or vector. Let x, y
and z be the components of a vector, u , in 3-dimensional space, that is
u = xi + y j + zk . The magnitude of u is ˜
˜ ˜ ˜ ˜ ˜
u = x2 + y2 + z2
˜

WORKED Example 11
Consider the point in 3-dimensional space given by the coordinates z
(2, 4, 3). (2, 4, 3)
Find the magnitude of the position vector, u , joining this point to ~u
the origin. ˜
y

x
THINK WRITE

1 Express u as a position vector. u = 2i + 4 j + 3k


˜ ˜ ˜ ˜ ˜
2 Since the vector is in 3-dimensional u = x + y + z2
2 2
˜
space, use the 3-D version of
Pythagoras’ theorem to find the
magnitude.
3 Substitute the components for each u = 22 + 42 + 32
˜
direction and compute the magnitude.
= 29 (= 5.39 to 2 decimal places)
Chapter 7 Introduction to vectors 319
We are now in a position to resolve the problem of finding, accurately, the resultant
force acting on the television remote control when Daniel and Anna are pulling on it.

First redraw the diagram to show the addition of


Daniel’s
vectors. force
Taking the direction of Anna’s force as the ~i Resultant 40 N
force
direction, 30° 150°
Anna’s force, a = 50i~ + 0j Anna’s 50 N
˜ ~ force
Daniel’s force, b = 40 cos 150°i~ + 40 sin 150°j
˜ ~
= −34.6410i~ + 20j
~
Resultant force, a + b = (50 − 34.6410)i~ + 20j
˜ ˜ ~
= 15.3590i~ + 20j
~
Magnitude = | a + b |
˜ ˜
Magnitude = 25.2 N
20
Direction θ = tan–1 -------------------
15.3590
Direction θ = 52.48°

remember
remember
1. Magnitude of a vector: If u = xi + y j + zk , the magnitude is given by
˜ ˜ ˜ ˜
u = x2 + y2 + z2
˜
Speed is the magnitude of velocity which is a vector quantity.
2. Direction of a vector (2-D only): If u = xi + y j , the direction is given by
˜ ˜ ˜
θ = tan−1 --yx
3. The x- and y-components of a vector: Given magnitude and direction, the x-
and y-components are given by:
x = u cos θ
˜
y = u sin θ
˜
4. Unit vector: The unit vector of a vector u , in the direction of u , is denoted by
û and is: ˜ ˜
˜ u
û = -----
˜
˜ u
˜
5. Locating vectors: If A and B are points with position vectors a and b
˜ ˜
respectively then AB = b – a .
˜ ˜
320 M a t h s Q u e s t M a t h s C Ye a r 1 1 f o r Q u e e n s l a n d

Position vectors in two and


7B three dimensions
1 State the x, y and z components of the following vectors:
a 3i + 4 j – 2k 3, 4, −2 b 6i – 3k 6, 0, −3 c 3.4i + 2 j + 1--2- k 3.4, 2, --1-
˜ ˜ ˜ ˜ ˜ ˜ ˜ ˜ 2

WORKED 2 For each of the following find:


Example
6a, b i the magnitude of the vector eBook plus
ii the direction of each vector. (Express the direction with respect to
the positive x-axis.) Digital doc:
EXCEL Spreadsheet
a y (6, 6) b (–4, 7) y Position vector

~v w
~
x
x 2 a i 72 ii 45°
b i 65 ii 119.7°
c y d y
c i 4.88 ii 225.8°
d i 320.16 ii 358.2°
x x
~a ~b (320, –10)

(–3.4, –3.5)
3 a 045°
WORKED b 330.3°
Example 3 Find the true bearing of each vector in question 2. c 224.2° eBook plus
6c d 091.8°
Digital docs:
WORKED 4 Consider the vector w shown at right. Its y N SkillSHEET 7.1
Example ˜ bearing is 210° True.
magnitude is 100 and its Bearings
7 Find the x- and y-components of w , and W E
express them as exact values (surds). ˜ 210°
x
100 S
State the answer in the form w = xi + y j .
˜ ˜ ˜
–50i – 50 3 j w
5 multiple choice ˜ ˜ ~

A vector with a bearing of 60 degrees from N and a magnitude of 10 has:


A x-component = ------- , y-component = ---
3 1
2 2
B x-component = --- , y-component = -------
1 3
2 2
C x-component = 5 3 , y-component = 5
D x-component = 5, y-component = 5 3
E none of the above

6 An aeroplane travels on a bearing of 147 degrees


for 457 km. Express its position as a vector in terms
of i and j . 248.9i – 383.3 j
˜ ˜ ˜ ˜
7 A ship travels on a bearing of 331 degrees for
125 km. Express its position as a vector in terms
of i and j . –60.6i + 109.3 j
˜ ˜ ˜ ˜
Chapter 7 Introduction to vectors 321
WORKED 8 A pilot flies 420 km in a direction 45° south of east and then 200 km in a direction
Example
60° south of east. Calculate the resultant displacement from the starting position
8
giving both magnitude and direction. 615 km at 49.8° south of east
SLE 3: Use addition and
subtraction in
life-related 9 The instructions to ‘Black-eye the Pirate’s hidden treasure’ say: Take 20 steps in a
situations. north-easterly direction and then 30 steps in a south-easterly direction. However, a
rockfall blocks the first part of the route in the north-easterly direction. How could
you head directly to the treasure? 36 steps 11.3° south of east

10 Two scouts are in contact with home base. Scout A is 15 km from home base in a
direction 30° north of east. Scout B is 12 km from home base in a direction 40° west
of north. How far is scout B from scout A? 20.8 km
WORKED 11 Find unit vectors in the direction of the given vector for the following:
Example
9
a y b y
(3, 4)
3---
+ 4--5- j --3- i – --45- j 0 x

i ~a 5˜
˜
˜
~d (3, –4)
0 x
c b = 4i + 3 j 4---

i + 3--5- j d e = – 4i + 3 j – 4--5- i + 3--5- j
˜ ˜ ˜ ˜ ˜ ˜ ˜ ˜ ˜
e c = i + 2j ------- i + ------- j
1 2 f f = – 3.5i + 2.7 j –0.792i + 0.611 j
˜ ˜ 3˜ 3
˜ ˜ ˜ ˜ ˜
˜
12 multiple choice
A unit vector in the direction of 3i – 4 j is:
˜ ˜
A 3--5- i + 4--5- j B 3--5- i – 4--5- j C i– j D 3-
-----
25 ˜
i 4-
– -----
25
j E none of these
˜ ˜ ˜ ˜ ˜ ˜ ˜
13 Not all unit vectors are smaller than the original vectors. Consider the vector
v = 0.3i + 0.4 j . Show that the unit vector in the direction of v is twice as long as v .
˜ ˜ ˜ ˜ ˜ Check with
14 Find the unit vector in the direction of w = – 0.1i – 0.02 j . –0.98i – 0.20 j your teacher.
˜ ˜ ˜ ˜ ˜
15 Find a unit vector in the direction of w for the vector of question 4. – 1--2- i – ------23- j
˜ ˜ ˜
16 Consider the points A (0, 1) and B (4, 5) in the figure at y B
(4, 5)
right. A vector joining A to B can be drawn. Check with your teacher.
a Show that an equivalent position vector is given by:
(0, 1) A
4i + 4 j . x
˜ ˜ 17
b Similarly, show that an equivalent position vector joining
a i 4i – 7 j ii 65
B to A is given by: –4i – 4 j . ˜ ˜
˜ ˜ b i 3i + j ii 10
˜ ˜
WORKED 17 For each of the following pairs of points find: c i –4i + 7 j ii 65
Example ˜ ˜
10 i the position vectors locating the second point from the first point d i –3i – j ii 10
˜ ˜
18 a –4i + 7 j ii the length of this vector. e i 2i ii 2
˜ ˜ ˜
b –3i – j a (0, 2), (4, −5) b (2, 3), (5, 4) c (4, −5), (0, 2) f i – 4i ii 4
˜ ˜ ˜
c 4i – 7 j d (5, 4), (2, 3) e (3, 7), (5, 7) f (7, −3), (3, −3)
˜ ˜
d 3i + j 18 Find the position vectors from question 17, by going from the second point to the first.
˜ ˜
e – 2i
˜
f 4i
˜
322 M a t h s Q u e s t M a t h s C Ye a r 1 1 f o r Q u e e n s l a n d

19 a 4
---------- i
65 ˜
7
– ---------
65
-j 19 Find unit vectors in the direction of the position vectors for each of the vectors of
˜
question 17.
---------- i + ---------- j
3 1
b
10 ˜ 10
˜
c – ---------- i + ---------
4 7
-j
20 Let u = 5i – 2 j and e = –2i + 3 j .
65 ˜ 65 ˜ ˜ ˜ ˜ ˜ 5
˜ --------- 2
- i – ---------- j 2
- i + ---------- j
– --------- 3
3i + j
˜ a Find: 29 ˜ 29 13 ˜ 13
3 1
˜ ˜ ˜ ˜
d – --------- - i – ---------- j
10 ˜ 10
˜ i
u 29 ii e 13 iii û iv ê v u+e vi u + e 10
e i ˜ ˜ ˜ ˜ ˜ ˜ ˜ ˜
˜ b Confirm or reject the statement that u + e = u + e Reject, because magnitudes different.
f –i ˜ ˜ ˜ ˜
˜
21 Let u = –3i + 4 j and e = 5i – j .
˜ ˜ ˜ ˜ 26 ˜ ˜ – 3--- i + 4--- j 5 1
---------- i – ---------- j 2i + 3 j
a Find: 5˜ 5
˜ 26 ˜ 26
˜ ˜ ˜
iu 5 ii e iii û iv ê v u+e vi u + e 13
˜ ˜ ˜ ˜ ˜ ˜ ˜ ˜
b Confirm or reject the statement that u + e = u + e . Reject, because magnitudes different.
˜ ˜ ˜ ˜
22 To find the distance between two vectors, a and b , simply find a – b .
˜ ˜ ˜ ˜
Find the distance between these pairs of vectors:
a 3i + 2 j and 2i + 3 j 2 b 5i – 2 j and 2i + 5 j 58
˜ ˜ ˜ ˜ ˜ ˜ ˜ ˜
23 A river flows through the jungle from west to east at a speed of 3 km/h. An explorer
wishes to cross the river by boat, and attempts this by travelling at 5 km/h due north.
Find:
a the vector representing the velocity of the river 3i
˜
b the vector representing the velocity of the boat 5 j
c the resultant (net) vector of the boat’s journey 3i + 5 j˜
˜ ˜
d the bearing of the boat’s journey 031.0°
e the magnitude of the net vector. 34 km/h

24 Consider the data from question 23. At what bearing should the boat travel so that it
arrives at the opposite bank of the river due north of the starting position? 329.0°
Chapter 7 Introduction to vectors 323
WORKED 25 Find the magnitude of the following 3-dimensional vectors.
Example
a z b z
11 (–3, –4, 5)

5 2 0 5 2
y
0 y
(3, 4, –5)
x
x
c 0.5i – 2k + 3 j 3.64 d 2i – 2 2 j + k 11
˜ ˜ ˜ ˜ ˜ ˜
e –7i + 14 j – 21k 7 14 f i+ j+k 3
˜ ˜ ˜ ˜ ˜ ˜
26 By calculating the difference between two position vectors, a vector representing the
separation of the two vectors can be defined. Find the distance between the
following 3-dimensional vectors.
a 4i + 3 j – 2k and 5i – 2 j + k 35 b 2i + j – k and 5i + j + k 13
˜ ˜ ˜ ˜ ˜ ˜ ˜ ˜ ˜ ˜ ˜ ˜
c –i + 2 j + 3k and 3i + k 2 6 d i + 3 j – k and 8i + 5 j + 2k 62
˜ ˜ ˜ ˜ ˜ ˜ ˜ ˜ ˜ ˜ ˜
27 If four points C, D, E and F in 3-dimensional space are located as follows:
C = (2, 6, 0), D = (3, −1, –2), E = (−4, 8, 10), F = (−2, −6, 6), show that CD is parallel
to EF. CD = i – 7 j – 2k , EF = 2i – 14 j – 4k
˜ ˜ ˜ ˜ ˜ ˜
28 A boat travels east at 20 km/h, while another boat travels south at 15 km/h. Find:
a a vector representing each boat and the difference between the boats 20i , – 15 j , 20i + 15 j
˜ ˜ ˜ ˜
b the magnitude of the difference vector 25
c the bearing of the difference vector. 053.1°
29 Consider the vector u = 3i + 4 j and the vector v = 4i – 3 j . Find the angles of each
of these vectors with˜ respect
˜ ˜ to the x-axis. ˜Show˜ that˜ these two vectors are
perpendicular to each other. Also show that the products of each vector’s
corresponding x- and y-components add up to 0. Can you confirm that this is a pattern
for all perpendicular vectors? 53.1°, −36.9° Difference = 90°
SLE 3: Use addition and 30 A river has a current of 4 km/h westward. A boat which is capable of travelling at
subtraction in
life-related 12 km/h is attempting to cross the river by travelling due north. Find:
situations such a a vector representing the net velocity of the boat –4i + 12 j 0.0417 h or 2.5 minutes
as the effect of ˜ ˜
current flow on b the bearing of the actual motion of the boat 341.6°
a boat. c how long it takes to cross the river, if the river is 500 m wide (from north to south).
(Hint: The maximum ‘speed’ of the boat is still 12 km/h.)
324 M a t h s Q u e s t M a t h s C Ye a r 1 1 f o r Q u e e n s l a n d

Multiplying two vectors — the dot product


Introduction
In a previous section we studied the result of multiplying a vector by a scalar. What
happens if a vector is multiplied by another vector? There are two possibilities: either
the result is a scalar (called the scalar product or dot product) or the result is a vector
(called the cross, or vector product). In this course we will study only the former.

The scalar or dot product of two vectors, u and v , is denoted by u


˜ ˜ ˜
⋅ v˜ .
Calculating the dot product
There are two ways of calculating the dot product. The first method follows from its
definition. (The second method is shown later.) Consider the two vectors u and v below.
˜ ˜
By definition, the dot product u v is given by:
˜ ˜

~u
u • v = u v cos θ [1]
˜ ˜ ˜ ˜ θ
where θ is the angle between (the positive directions of) u and v . ~v
˜ ˜
Note: The vectors are not aligned as for addition or subtraction, but their two tails are
joined.

Properties of the dot product


1. The dot product is a scalar. It is the result of multiplying three scalar quantities: the
magnitudes of the two vectors and the cosine of the angle between them.
2. The order of multiplication is unimportant (commutative property), thus
u•v = v•u
˜ ˜ ˜ ˜
3. The dot product is distributive, thus
a • (u + v) = a • u + a • v
˜ ˜ ˜ ˜ ˜ ˜ ˜
4. Since the angle between u and itself is 0°
˜ u•u = u 2
˜ ˜ ˜

WORKED Example 12
Let u = 3i + 4 j and v = 6i . Find u • v .
˜ ˜ ˜ ˜ ˜ ˜ ˜
THINK WRITE
1 Find the magnitudes of u and v . u = 32 + 42
˜ ˜ ˜ = 5
v = 62
˜
= 6
2 Draw a right-angled triangle showing y
the angle that u makes with the
positive x-axis ˜since v is along the ~u
˜ 5
x-axis.
θ
3 x
Chapter 7 Introduction to vectors 325
THINK WRITE
3 Find cos θ, knowing that u = 5 and the cos θ = 3---
5
x-component of u is 3.
˜
4 Find u • v using equation 1. u • v = u × v × cos θ
˜ ˜ ˜ ˜ ˜ ˜
5 Simplify. = 5 × 6 × 3---
5
= 18

Note: An easier method for finding the dot product will now be shown.

Unit vectors and the dot product


Consider the dot product of the unit vectors i , j and k . Firstly, consider i • i in
detail. By definition, i = 1 and, since the angle ˜ ˜ between ˜ them is 0°, cos θ =˜ 1,˜ thus
˜
i • i = 1 . To summarise these results:
˜ ˜
i • i = 1 (since θ = 0°)
˜ ˜
j • j = 1 (since θ = 0°)
˜k • ˜k = 1 (since θ = 0°)
˜ ˜
i • j = 0 (since θ = 90°)
˜ ˜
i • k = 0 (since θ = 90°)
˜ ˜
j • k = 0 (since θ = 90°)
˜ ˜
Using this information, we can develop another way to calculate the dot product of
any vector. Let u = x 1 i + y 1 j + z 1 k and v = x 2 i + y 2 j + z 2 k , where x1, y1, z1, x2, y2, z2
are constants. Then ˜ we˜can write ˜ u˜ • v as:˜ ˜ ˜ ˜
˜ ˜
u • v = ( x1 i + y1 j + z1 k ) • ( x2 i + y2 j + z2 k )
˜ ˜ ˜ ˜ ˜ ˜ ˜ ˜
= x1 x2 ( i • i ) + x1 y2 ( i • j ) + x1 z2 ( i • k ) + y1 x2 ( j • i ) + y1 y2 ( j • j )
˜ ˜ ˜ ˜ ˜ ˜ ˜ ˜ ˜ ˜
+ y1 z2 ( j • k ) + z1 x2 ( k • i ) + z1 y2 ( k • j ) + z1 z2 ( k • k )
˜ ˜ ˜ ˜ ˜ ˜ ˜ ˜
Considering the various unit vector dot products (in brackets), the ‘like’ products
( i • i , j • j and k • k , shown underlined) are 1; the rest are 0. Therefore:
˜ ˜ ˜ ˜ ˜ ˜
u • v = x1 x2 + y1 y2 + z1 z2 [2]
˜ ˜
This is a very important result.
We only need to multiply the corresponding x, y and z components of two vectors
to find their dot product.

WORKED Example 13
Let u = 3i + 4 j + 2k and v = 6i – 4 j + k . Find u • v .
˜ ˜ ˜ ˜ ˜ ˜ ˜ ˜ ˜ ˜
THINK WRITE
1 Write down u • v using equation 2. u • v = ( 3i + 4 j + 2k ) • ( 6i – 4 j + k )
˜ ˜ ˜ ˜ ˜ ˜ ˜ ˜ ˜ ˜
2 Multiply the corresponding u•v = 3×6+4×–4+2×1
components. ˜ ˜

3 Simplify. = 18 – 16 + 2
= 4
326 M a t h s Q u e s t M a t h s C Ye a r 1 1 f o r Q u e e n s l a n d

Graphics Calculator tip! Finding the dot product


of two vectors
The following steps show how a graphics calculator can be used to find the dot product
of two vectors. Consider the vectors u = 3i + 4 j + 2k and v = 6i – 4 j + k in Worked
example 13. ˜ ˜ ˜ ˜ ˜ ˜ ˜ ˜

For the Casio fx-9860G AU


1. Press MENU and select RUN-MAT. Enter the
components of vector u as a matrix. Then press Æ
˜
and SHIFT [Mat] followed by ALPHA [U] to store
as matrix U. Press EXE to display the matrix and
EXE again to return to the calculation screen.
Repeat this procedure to enter the components of
vector v as a matrix and store as matrix V.
˜
2. To calculate the dot product using matrices, we first
need to transpose the elements of matrix U before we
multiply by matrix V. Press OPTN then F2 (MAT)
followed by F4 (Trn) to access the transpose
function.

3. Press SHIFT [Mat] then ALPHA [U] to enter matrix


U. Press ¥ followed by SHIFT [Mat] then ALPHA
[V] to enter matrix V.

4. Press EXE to display the value for the dot product.

For the TI-Nspire CAS


1. Open a new Calculator document. Define the
vector u and the vector v by entering each set of
˜
components ˜ brackets.
within square

2. Press b and select 7: Matrix & Vector then


C: Vector followed by 3: Dot Product. Press U,
then the comma key (,) followed by V and close
the set of brackets by pressing ). Press · to
obtain the value of the dot product.
Chapter 7 Introduction to vectors 327
Finding the angle between two vectors
Now that we have two formulas (equations 1 and 2) for calculating the dot product, we
can combine them to find the angle between the vectors:
u • v = x1 x2 + y1 y2 + z1 z2
˜ ˜
= u v cos θ
˜ ˜
Rearranging the final two equations, we obtain the result that:
x1 x2 + y1 y2 + z1 z2
cos q = ---------------------------------------------
- [3]
u v
˜ ˜
Note: The angle will always be between 0° and 180° as 180° is the maximum angle
between two vectors.

WORKED Example 14
Let u = 4i + 3 j + k and v = 2i – 3 j – 2k . Find the angle between them to the
˜ ˜ ˜ ˜ ˜ ˜
nearest degree. ˜ ˜

THINK WRITE
1 Find the dot product using equation 2.
˜ ˜ ˜ ˜ ˜ ˜ ˜

u • v = ( 4i + 3 j + k ) ( 2i – 3 j – 2k )
˜
2 Simplify. = 4 × 2 + 3 × −3 + 1 × −2
= −3
3 Find the magnitude of each vector. u = 42 + 32 + 12
˜
= 26
2 2 2
v = 2 + ( –3 ) + ( –2 )
˜
= 17
–3
4 Substitute results into equation 3. cos θ = --------------------
26 17
3
5 Simplify the result for cos θ. = – -------------
442
= −0.142 695
6 Take cos−1 of both sides to obtain θ and θ = cos−1 (−0.142 695)
round the answer to the nearest degree. = 98°

Special results of the dot product


Perpendicular vectors
If two vectors are perpendicular then the angle between them is 90° and equation 1
(page 324) becomes:
u • v = uv cos 90°
˜ ˜
= uv × 0 (since cos 90° = 0)
= 0
If u • v = 0 , then u and v are perpendicular.
˜ ˜ ˜ ˜
328 M a t h s Q u e s t M a t h s C Ye a r 1 1 f o r Q u e e n s l a n d

WORKED Example 15
Find the constant a if the vectors u = 4i + 3 j and v = – 3 i + a j are perpendicular.
˜ ˜ ˜ ˜ ˜ ˜
THINK WRITE
1 Find the dot product using equation 2. u • v = ( 4i + 3 j ) • ( – 3i + a j )
˜ ˜ ˜ ˜ ˜ ˜
2 Simplify. = – 12 + 3a
3 Set u • v equal to zero since u and v are perpendicular. u • v = –12 + 3a = 0
˜ ˜ ˜ ˜ ˜ ˜
4 Solve the equation for a. a=4

Parallel vectors
If vector u is parallel to vector v then u = k v where k ∈ R.
Note: When˜ applying the dot product
˜ to˜ parallel
˜ vectors, θ (the angle between them)
may be either 0° or 180° depending on whether the vectors are in the same or opposite
directions.

WORKED Example 16
Let u = 5i + 2 j . Find a vector parallel to u such that the dot product is 87.
˜ ˜ ˜ ˜
THINK WRITE
1 Let the required vector v = ku . Let v = k ( 5i + 2 j )
˜ ˜ ˜ ˜ ˜
= 5k i + 2k j
˜ ˜
2 Find the dot product of u • v . u • v = ( 5i + 2 j ) • ( 5k i + 2k j )
˜ ˜ ˜ ˜ ˜ ˜ ˜ ˜
3 Simplify. = 25k + 4k
= 29k
4 Equate the result to the given dot product 87. 29k = 87
5 Solve for k. k=3
6 Substitute k = 3 into vector v . v = 15i + 6 j
˜ ˜ ˜ ˜

remember
remember
1. Scalar (dot) product: The scalar or dot product of two vectors u and v is
denoted by u • v . ˜ ˜
˜ ˜
2. Calculation of dot product:
u • v = u v cos θ (where θ is the angle between the two vectors).
˜ ˜ ˜ ˜
3. Algebraic calculation of dot product:
Let u = x 1 i + y 1 j + z 1 k and v = x 2 i + y 2 j + z 2 k .
˜ ˜ ˜ ˜ ˜ ˜ ˜ ˜
Then u • v = x 1 x 2 + y 1 y 2 + z 1 z 2.
˜ ˜
4. Special results:
(a) If u • v = 0 , then u and v are perpendicular.
˜ ˜ ˜ ˜
(b) If u = kv , k ∈ R , then u and v are parallel.
˜ ˜ ˜ ˜
Chapter 7 Introduction to vectors 329
Multiplying two vectors —
7C the dot product
WORKED
Example 1 Find the dot product of the vectors 3i + 3 j and 6i + 2 j using equation 1. 23.99
12
˜ ˜ ˜ ˜
2 Compare the result from question 1 with that obtained by finding the dot product
using equation 2. Which is probably the most accurate? Dot product = 24; more accurate,
since no angle needed
WORKED 3 Find u • v in each of the following cases.
Example ˜ ˜
13 a u = 2i + 3 j + 5k , v = 3i + 3 j + 6k 45
˜ ˜ ˜ ˜ ˜ ˜ ˜ ˜
b u = 4i – 2 j + 3k , v = 5i + j – 2k 12
˜ ˜ ˜ ˜ ˜ ˜ ˜ ˜
c u = –i + 4 j – 5k , v = 3i – 7 j + k −36
˜ ˜ ˜ ˜ ˜ ˜ ˜ ˜
d u = 5i + 9 j , v = 2i – 4 j −26
˜ ˜ ˜ ˜ ˜ ˜
e u = –3i + j , v = j + 4k 1
˜ ˜ ˜ ˜ ˜ ˜
f u = 10i , v = – 2i −20
˜ ˜ ˜ ˜
g u = 3 j + 5k , v = i 0
˜ ˜ ˜ ˜ ˜
h u = 6i – 2 j + 2k , v = –i – 4 j – k 0
˜ ˜ ˜ ˜ ˜ ˜ ˜ ˜
4 multiple choice
The dot product of u = 3i – 3 j + 3k and v = i – 2 j + 6k is:
˜ ˜ ˜ ˜ ˜ ˜ ˜ ˜
A 0 B 3 C 12 D 21 E 27

5 multiple choice
Consider the two vectors shown at right. Their dot product is: 45° v
~u 5 ~
A 30 B 21.2 C −21.2 D 0
6
E There is insufficient data to determine the dot product.

6 Consider the vectors u and v at right. Their magnitudes are ~v


˜
7 and 8 respectively. Find u •˜ v . −36
˜ ˜ 50°

7 Let u = xi + y j . Show that u • u = x 2 + y 2 . ~u


˜ ˜ ˜ ˜ ˜
8 Let u = 2i – 5 j + k and let v = –i – 2 j + 4k . Find their dot product. 12
˜ ˜ ˜ ˜ ˜ ˜ ˜ ˜
9 Let u = 3i + 2 j , v = i – 2 j and w = 5i – 2 j . Demonstrate, using these vectors,
˜ ˜
the property: ˜ ˜ ˜ ˜ ˜ ˜ ˜
9 and 10 Check w • (u – v) = w • u – w • v
with your teacher. ˜ ˜ ˜ ˜ ˜ ˜ ˜
Formally, this means that vectors are distributive over subtraction.

10 Repeat question 9 for the property:


w • (u + v) = w • u + w • v
˜ ˜ ˜ ˜ ˜ ˜ ˜
Formally, this means that vectors are distributive over addition.
330 M a t h s Q u e s t M a t h s C Ye a r 1 1 f o r Q u e e n s l a n d

11 multiple choice
If u = 5i + 4 j + 3k , which of the following is perpendicular to u ?
˜ ˜ ˜ ˜ ˜
A –5i – 4 j – 3k B 3i + 4 j + 5k C – 5i
˜ ˜ ˜ ˜ ˜ ˜ ˜
D –3i + 5k E –5i + 3k
˜ ˜ ˜ ˜

12 multiple choice
If ( u – v ) • ( u + v ) = 0 then:
˜ ˜ ˜ ˜
A u is parallel to v B u and v have equal magnitudes
˜ ˜ ˜ ˜
C u is perpendicular to v D u is a multiple of v
˜ ˜ ˜ ˜
E None is true.

13 multiple choice
If ( u – v ) • ( u + v ) = v 2 then:
˜ ˜ ˜ ˜ ˜
A u = v B u must be equal to the zero vector, 0
˜ ˜ ˜ ˜
C u is perpendicular to v D u must be equal to 2 v
˜ ˜ ˜ ˜
E None is true.

14 Find the dot product of the following pairs of vectors.


a 4i – 3k and 7 j + 4k −12 b i + 2 j – 3k and –9i + 4 j – k 2
˜ ˜ ˜ ˜ ˜ ˜ ˜ ˜ ˜ ˜
c 8i + 3 j and 2i – 3 j + 4k 7 d 5i – 5 j + 5k and 5i + 5 j – 5k −25
˜ ˜ ˜ ˜ ˜ ˜ ˜ ˜ ˜ ˜ ˜
WORKED 15 Find the angle between each pair of vectors in question 14 to the nearest degree.
Example
14 15 a 107°
b 87°
16 multiple choice c 81°
d 109°
The angle between the vectors 2i + 3 j and 2i – 3 j is closest to:
˜ ˜ ˜ ˜
A 0° B 67° C 90°
D 113° E 180°

17 multiple choice
The angle between the vectors 2i – 3 j and – 4 i + 6 j is closest to:
˜ ˜ ˜ ˜
A 0° B 69° C 90°
D 111° E 180°

WORKED 18 Find the constant a, if the vectors v = ai + 3 j and u = 6i – 2 j are perpendicular. a = 1


Example ˜ ˜ ˜ ˜
˜ ˜
15
19 Find the constant a, such that v = ai – 2a j + 3k is perpendicular to
u = 4i – 3 j + 2k . a = − 3--5- ˜ ˜ ˜ ˜
˜ ˜ ˜ ˜
WORKED
Example 20 Let u = 2i + 4 j . Find a vector parallel to u such that their dot product is 40. 4i + 8 j
˜ ˜ ˜ ˜ ˜ ˜
16
21 Let u = 4i – 3 j . Find a vector parallel to u such that their dot product is 80. 64
------ i – 48
------ j
˜ ˜ ˜ ˜ 5˜ 5
˜
Chapter 7 Introduction to vectors 331

History of mathematics
CHARLES LUTWIDGE DODGSON (1832 – 1898)

Dodgson’s most famous achievements were


two books which have become classics of
children’s literature: Alice’s Adventures in
Wonderland and Through the Looking-Glass.
His main character was inspired by Alice
Liddell who was the second daughter of the
Dean of his college. Alice’s Adventures in
Wonderland was published in 1865 and became
a huge success. Through the Looking-Glass,
published in 1872, was equally successful.
Dodgson also wrote humorous poetry. His
poem Jabberwocky is a masterpiece, although it
is often thought of as being just a piece of
nonsense verse. The word ‘chortle’ which is
used for the first time in this poem was invented
by Dodgson and is derived from the two words
‘chuckle’ and ‘snort’.
Dodgson’s mathematical works include
Euclid and his Modern Rivals, A Syllabus of
Plane Algebraical Geometry, and An
During his lifetime ...
Elementary Treatise on Determinants. These
The Braille reading system is invented by were well written but have not survived the test
Louis Braille. of time.
Morse code is developed. Dodgson was interested in logic and loved
Famine devastates Ireland. puzzles. His books contain many mathematics
Edison develops the light bulb. and logic puzzles including this example from
Through the Looking-Glass: ‘Contrariwise’,
said Tweedledee, ‘if it was so, it might be; and
Charles Dodgson was an English writer and
if it were so, it would be; but as it isn’t, it
mathematician. He wrote several
ain’t. That’s logic.’ One of the puzzles he
mathematics books but is best known for
invented was ‘If 6 cats kill 6 rats in 6 minutes,
his fictional works produced under the pen
how many will be needed to kill 100 rats in
name Lewis Carroll. Dodgson was the son
50 minutes?’
of a clergyman and was the third of eleven
children. His education included a period at Questions
Rugby, a school where the game of Rugby
1. What was Dodgson’s job at Oxford
originated in 1823. Dodgson went on to Alice’s Adventures
University? Mathematics lecturer
study at Christ Church College at Oxford in Wonderland and
University. After completing his studies he 2. Name his two most famous works. Through the
3. Who was the inspiration for the Looking-Glass
lectured in mathematics at Oxford from
1855 until 1881. He also became a character of Alice? The daughter of the Dean of his college
clergyman in 1861 but did not take up a 4. If 6 cats kill 6 rats in 6 minutes, how
position in the church, possibly because he many will be needed to kill 100 rats in
had a stutter. 50 minutes? 12 cats
332 M a t h s Q u e s t M a t h s C Ye a r 1 1 f o r Q u e e n s l a n d

Resolving vectors — scalar and


vector resolutes
Introduction
Consider the two vectors, u and v , shown at right. The angle between
them, as for a dot product,˜ is given
˜ by θ. It can be shown that v is
~v
made up of a component acting in the direction of u and another ˜
˜ θ
component acting perpendicular to u .
˜ ~u
Firstly we wish to find the component in the direction of u .
˜
The scalar resolute
To obtain the component of v in the direction of u , we perform the following construction:
˜ ˜
1. Drop a perpendicular from the head of v to u (this is perpendicular to u ). This line
joins u at point A. ˜ ˜ ˜
˜
2. We wish to find the length of the line OA.
This construction is shown at right.
Let the length of v (its magnitude) be denoted by v . Then, ~v
from trigonometry:˜ ˜
OA = v cos θ θ
O
˜
But from the definition of the dot product: ~u A

u • v = u v cos θ
u˜ • v˜ = u˜ OA˜ (from the first equation)
˜
Therefore, solving for OA: ˜ ˜
u•v
OA = --------
˜ ˜-
u
˜
 u
= ----- v
 u˜  ˜
u ˜
˜ = û , the unit vector in the direction of u , and therefore
But we know that -----
u ˜ OA = û • v ˜
˜ ˜ ˜
This quantity, the length OA, is called the scalar resolute of v on u . It effectively
indicates ‘how much’ of v is in the direction of u . ˜ ˜
˜ ˜
The scalar resolute of v on u is given by û • v , where û is the unit vector in the
direction of u . ˜ ˜ ˜ ˜ ˜
˜

WORKED Example 17
Let u = 3i + 4 j and a = 6i – 2 j . Find:
˜ scalar
a the ˜ resolute
˜ ˜ u˜
˜of a on b the scalar resolute of u on a .
˜ ˜ ˜ ˜
THINK WRITE
a 1 Find the magnitude of u . a u = 32 + 42
˜ ˜ =5
u
2 Find û by dividing u by u . û = -----
˜
˜ ˜ ˜ u
˜
u
= --˜-
5
Chapter 7 Introduction to vectors 333
THINK WRITE
3 Simplify. = --- ( 3i +
1
4 j)
5 ˜ ˜
= --- i + 4
3

--- j
5
˜
4 Find the scalar resolute of a on u using û • a . û • a = ( 3--5- i + 4--5- j ) • ( 6i – 2 j )
˜ ˜ ˜ ˜ ˜ ˜ ˜ ˜ ˜ ˜
5 Simplify. = ------ − ---
18 8
5 5

= 10
------
5
=2
b 1 Find the magnitude of a . b a = 6 2 + ( –2 )2
˜ ˜
= 40
a
2 Find â by dividing a by a . â = -----
˜
˜ ˜ ˜ ˜ a
˜1
= ---------
- ( 6i – 2 j )
40 ˜ ˜
3 Find the scalar resolute of a on u using â • u . â • u = --------- 1-
( 6i – 2 j ) • ( 3i + 4 j )
˜ ˜ ˜ ˜ ˜ ˜ 40 ˜ ˜ ˜ ˜
4 Simplify. = ---------- ( 18 – 8 )
1
40
= 10-
---------
40

= 10-
---------
2

Notes:
1. The two scalar resolutes are not equal.
v.u
2. The scalar resolute of v on u can easily be evaluated as ----------
˜ ˜.
˜ ˜ u
˜
Vector resolutes
Consider, now, the vector joining O to A at right. Its magnitude
is just the scalar resolute ( û • v ), while its direction is the same ~v
˜ ˜is called the vector resolute of v ~v ⊥

as u , that is û . This quantity


˜
parallel to u ˜and is denoted by the symbol v || . ˜ θ
O
˜ ˜ ~v A ~u ||

The vector resolute of v parallel to u is given by:


˜ ˜
v || = ( û • v )û [4]
˜ ˜ ˜ ˜
Consider the geometry of the above figure. The original vector v can be seen to be
the sum of two other vectors, namely v || and v ⊥ . This second vector˜ is called the vector
resolute of v perpendicular to u and can ˜ be ˜computed simply as follows:
˜ ˜
v = v || + v ⊥ (by addition of vectors)
˜ ˜ ˜
v ⊥ = v – v || (by rearranging the vector equation)
˜ ˜ ˜
By substitution for v || from equation 4, the vector resolute of v perpendicular to u
is given by: ˜ ˜ ˜
v ^ = v – ( û • v )û [5]
˜ ˜ ˜ ˜ ˜
In practice, once v || has been calculated, simply subtract it from v to get v ⊥ .
˜ ˜ ˜
334 M a t h s Q u e s t M a t h s C Ye a r 1 1 f o r Q u e e n s l a n d

WORKED Example 18
Let u = – 2 i + 3 j + k and v = 3i + 2 j – k . Find:
˜ scalar
a the ˜ ˜ of v ˜on u ˜
˜ resolute ˜ ˜
˜ ˜
b the vector resolute of v parallel to u, namely v ||
c the vector resolute of v˜ perpendicular˜ to u , namely
˜ v^ .
˜ ˜ ˜
THINK WRITE

a 1 Find the magnitude of u . a u = ( –2 )2 + 3 2 + 1 2


˜ ˜
= 14
u
2 Find û . û = -----
˜
˜ u
˜
= ---------
1-
( –2i + 3 j + k )
14 ˜ ˜ ˜
3 Find the scalar resolute using û • v . û • v = ---------
1-
( –2i + 3 j + k ) • ( 3i + 2 j – k )
˜ ˜ ˜ ˜ 14 ˜ ˜ ˜ ˜ ˜ ˜
4 Simplify. = 1-
--------- ( –6 + 6 – 1)
14

= – ---------
1-
14

b 1 Find v || using equation 4. b v || = ( û • v )û


˜ ˜ ˜ ˜ ˜
= ( – --------- ) [ 1 - ( –2i + 3 j + k ) ]
1 - ---------
14 14 ˜ ˜ ˜
2 Simplify. = – -----
1-
14
( –2i + 3 j + k )
˜ ˜ ˜
= 7 i – 14 j – 14 k
1-
--
3-
-----
1-
-----
˜ ˜ ˜
c 1 Find v ⊥ by subtraction of v || from v c v ⊥ = v – v ||
˜
as in equation 5. ˜ ˜ ˜ ˜ ˜
= 3i + 2 j – k – ( 1--7- i – -----
3- 1-
j – ----- k)
˜ ˜ ˜ ˜ 14 ˜ 14 ˜
2 Simplify by subtracting i , j , and k v⊥ = 20
------ i

+ 31
------ j – 13
14
------ k
components. ˜ ˜ ˜ ˜ ˜ 14 ˜

Graphics Calculator tip! Finding scalar and vector


resolutes
The dot product and the unit vector can be found on a graphics calculator, and then
used to find a scalar resolute or a vector resolute. Consider the vectors
u = – 2 i + 3 j + k and v = 3i + 2 j – k in Worked example 18.
˜ ˜ ˜ ˜ ˜ ˜ ˜ ˜
For the Casio fx-9860G AU
1. Press MENU and select RUN-MAT. Enter the
components of both vectors as matrices and store as
matrix U and matrix V.
Chapter 7 Introduction to vectors 335
2. To find the scalar resolute of v on u , we need to
˜ vector of u )
calculate the dot product of û ˜ (the unit
˜ ˜
and the vector v . For a three-dimensional vector, we
˜
cannot calculate the magnitude of the vector using
polar form as we did previously for the two-
dimensional vector. So calculate the value of
2 2 2
( –2 ) + 3 + 1 .
3. Press Æ followed by ALPHA [M] and then EXE
to store this answer for future use.

4. As in the previous graphics calculator tip on


page 326, the dot product is performed by
multiplying the transpose of the first matrix by the
second matrix. The first matrix in this case is û ,
which is obtained by multiplying matrix U by ˜the
reciprocal of magnitude M. (To access the transpose
function, press OPTN then F2 (MAT) and F4
(Trn).)
5. Press EXE to display the scalar resolute.

6. To find the vector resolute of v parallel to u , we


need to calculate ( û • v )û . This˜ is the scalar ˜resolute
multiplied by the unit ˜
˜ ˜vector û .
˜

7. Press EXE to display the components for the


required vector resolute.

8. To find the vector resolute of v perpendicular to u ,


˜
we need to subtract the vector˜resolute of v parallel
˜
to u (calculated in Step 7) from v . Subtract the
˜
appropriate matrices. ˜

9. Press EXE to display the components of the


required vector resolute.
336 M a t h s Q u e s t M a t h s C Ye a r 1 1 f o r Q u e e n s l a n d

For the TI-Nspire CAS


1. Open a new Calculator document. Define the
vector u and the vector v by entering each set of
˜
components ˜ brackets.
within square
2. To find the scalar resolute of v on u , we need to
˜ vector of u )
calculate the dot product of û ˜ (the unit
and the vector v . Access the Dot Product function˜
˜
˜
(press b and select 7: Matrix & Vector then
C: Vector followed by 3: Dot Product). To enter û ,
first access the Unit Vector function (press b and ˜
select 7: Matrix & Vector then C: Vector followed by
1: Unit Vector). Press U and then ) to close the set
of inner brackets. Press the comma key (,) followed
by V and then close the set of outer brackets by
pressing ). Press · to display the value of the
scalar resolute.
3. To find the vector resolute of v parallel to u , we
˜ / v to ˜show the
need to calculate ( û • v )û . Press
˜ ˜ ˜
previous answer then press the multiplication key
r. Repeat the steps above to access the Unit Vector
function and press U. Close the set of brackets by
pressing ). Press · to display the components of
the required vector resolute.

4. The vector resolute can be found straight after


defining the vectors u and v (without finding the
˜ seen˜in the screen at right.
scalar resolute first) as

5. To find the vector resolute of v perpendicular to u ,


˜
we need to subtract the vector˜resolute of v parallel
to u (calculated in Step 4) from v . Press V˜ then -
˜
followed by / v (the answer˜ from the previous
line of working). Press · to display the components
of the required vector resolute.

remember
remember
1. Scalar resolute of v on u is given by:
˜ ˜ û • v
˜ ˜
2. Vector resolute of v , parallel to u is:
˜ v˜ || = ( û • v )û
˜ ˜ ˜ ˜
3. Vector resolute of v , perpendicular to u :
˜ v ⊥ = v –˜ ( û • v )û
or v˜ ⊥ = v˜ – v ˜|| ˜ ˜
˜ ˜ ˜
Chapter 7 Introduction to vectors 337
Resolving vectors — scalar and
7D vector resolutes 1 a i
23 13-
---------------

17
13
29-
ii
23 41-
---------------

17
41
10-
b i ---------------
29 ii ---------------
10
WORKED 1 For each of the following pairs of vectors, find: c i – 13 10-
--------------- ii – 26 17-
---------------
Example 10 17
17 i the scalar resolute of a on u .
d i – 6------------
13
ii – 6--5-
ii the scalar resolute of u˜ on a˜ .
-
13
˜ ˜ e i 23
– -----5- ii – 23 26-
---------------
a u = 2i + 3 j and a = 4i + 5 j b u = 5i – 2 j and a = 3i – j 13
˜ ˜ ˜ ˜ ˜ ˜ ˜ ˜ ˜ ˜ ˜ ˜
c u = – 2 i + 6 j and a = i – 4 j d u = 3i – 2 j and a = – 4 i – 3 j
˜ ˜ ˜ ˜ ˜ ˜ ˜ ˜ ˜1 ˜ ˜ ˜
e u = 8i – 6 j and a = – 5 i + j 2 a i --------- - ii v || = -----
3-
10 ˜
i 1-
– -----
10
j iii v ⊥ = 17------ i + 51
10 ˜
------ j
10
˜ ˜ ˜ ˜ ˜ ˜ 10 ˜ ˜ ˜ ˜
82
b i ---------
- ii v || = 8i + 10 j iii v ⊥ = 0
WORKED 2 For each pair of vectors u and v , find: 41 ˜ ˜ ˜ ˜ ˜
Example
18 i the scalar resolute of ˜v on ˜u . c i 0 ii v || = 0 iii v ⊥ = – 3 i + 4 j
˜
ii the vector resolute of ˜v , parallel to u , namely v || . ˜ ˜ ˜ ˜ ˜
------ ( 25i – 6 j – 8k )
˜

˜ ˜ ˜
------ k
11 ˜

iii the vector resolute of v , perpendicular to u , namely v ⊥ .


------ j – 28

˜ ˜ ˜
a u = 3i – j ; v = 2i + 5 j b u = 4i + 5 j ; v = 8i + 10 j
iii v ⊥ = --43- i + --13- j + --53- k

˜
˜

˜
------ i + 17
11

˜ ˜ ˜ ˜ ˜ ˜ ˜ ˜ ˜ ˜ ˜ ˜
˜

c u = 4i + 3 j ; v = –3i + 4 j d u = i + j + k ; v = 2i + j – k
˜ ˜

11 ˜
15

˜ ˜ ˜ ˜ ˜ ˜ ˜ ˜ ˜ ˜ ˜ ˜ ˜ ˜
29


4

e u = 2i + 3 j + 4k ; v = 2i – 3 j – 4k f u = 3i + j – k ; v = 2 j – 3k
=
=

˜ ˜ ˜ ˜ ˜ ˜ ˜ ˜ ˜ ˜ ˜ ˜ ˜ ˜ ˜
˜⊥
˜⊥
v
- ( 2i + 3 j + 4k ) iii v
˜

3 An injured bushwalker is located at a pos-


iii

ition relative to a camp given by the vector


˜

2i + 3 j . A searcher heads off from the camp


- i + ------ j – ------ k
11 ˜

in˜ a direction
˜ parallel to the vector 3i + 4 j .
5
ii v || = --23- i + --23- j + --23- k
˜

All measurements are in kilometres. ˜ ˜


˜
11
5
˜ ˜

a How far is the searcher from the camp


ii v || = – -----21
29

11 ˜
15
ii v || = -----

when closest to the bushwalker? 3.6 km


b What is the minimum distance between
˜

the searcher and the bushwalker?


0.2 km or 200 metres
-
29
e i – ---------
21

4 A distressed yacht is located at a position


----------
11
-------
3

5
2

given by the vector 5i – 2 j relative to a


d i

f i

˜ sent
2

cruiser. A rescue boat is ˜ off from the


eBook plus cruiser and travels in a direction parallel to
the vector 3i – j . If all measurements are in
Digital doc: ˜ ˜ to the nearest metre, how
kilometres find,
WorkSHEET 7.1
close the rescue boat gets to the yacht.
316 metres

Vectors and matrices


As we have seen earlier in the chapter, there is a relationship between vectors and
matrices. If you are using a Casio fx-9860G AU graphics calculator, you will have
used this relationship to perform certain functions or calculations.
If we consider the vector v = ai + b j + ck , the components could be expressed
˜ ˜ ˜ ˜
a
as the matrix b .
c
338 M a t h s Q u e s t M a t h s C Ye a r 1 1 f o r Q u e e n s l a n d

Similarly, for the vector w = pi + q j + rk , the components could be expressed as


˜ ˜ ˜ ˜
p
the matrix q .
r
The properties of vector addition and subtraction, scalar multiplication and the
dot product can be formulated in terms of the properties of matrices.
a p a+ p
Addition of v and w : b + q = b+q
˜ ˜
c r c+r

a 2a
Scalar multiplication of v : 2 × b = 2b
˜
c 2c
T
a p
Dot product of v and w : b q
˜ ˜
c r

p
= [a b c] q
r
= [ap + bq + cr]
1 Consider the vectors u = 2i + 3 j + 4k and v = 2i – 3 j – 4k .
˜ ˜ ˜ ˜ ˜ ˜ ˜ ˜
Use matrix operations to calculate:
a u • v −21
˜ ˜ 21
b the scalar resolute of v on u – ----------
˜ ˜ 29 21
c the vector resolute of v parallel to u – ------ ( 2i + 3 j + 4k )
˜ ˜ 29 ˜ ˜ ˜
d the vector resolute of v perpendicular to u . ----- 4
- ( 25i – 6 j – 8k )
˜ ˜ 29 ˜ ˜ ˜

In mathematics there is often more than one method to represent an idea. For
example, a fraction of a number can be written using a proper fraction 3--- or a
4
decimal fraction 0.75. Each method has its advantages and disadvantages.
A vector in two dimensions can be represented in three ways:
1 in component form using i and j
2 as a column matrix ˜ ˜
3 by giving magnitude and direction.

That is, 3i + 4 j , 3 or (5, 53.1˚) can all be used to represent the same vector.
˜ ˜ 4
2 Suppose that your class is going to use one of these forms of notation. By
considering the strengths and weaknesses of each, decide which one you will
use.
Chapter 7 Introduction to vectors 339
Time-varying vectors
Introduction
Consider a vector where the x- and y-components are not constants, but vary with time.
So, instead of writing u = xi + y j , we write u = x ( t )i + y ( t ) j .
˜ can be
In theory, x(t) and y(t) ˜ any ˜ often ˜there is ˜a limitation in that t ≥ 0.
˜ functions, but
For example, suppose x(t) = t and y(t) = t. Thus, u = t 2 i + t j .
2

We can tabulate values for u at various values of time˜ t. ˜ ˜


˜
t x(t) y(t) u
˜
0 0 0 0
˜
1 1 1 i+ j
˜ ˜
2 4 2 4i + 2 j
˜ ˜
3 9 3 9i + 3 j
˜ ˜
4 16 4 16i + 4 j
˜ ˜
So at each point of time we get a ‘new’ vector; thus y
we can talk about a ‘vector function of time’. 4
We can join the heads of each of these vectors, as 3
2
shown at right. The dashed line indicates the path that 1
the head of the vector takes. Note that the tail of all
these vectors is the origin. How can we find the 1 2 3 4 5 6 7 8 9 10 x
equation of this path?
Finding the equation of the path
Consider a vector function of time u = x ( t )i + y ( t ) j .
˜
The expressions x(t) and y(t) are referred ˜ as parametric
to ˜ equations.
If we can solve these simultaneous equations, by eliminating t, we can get an
expression in terms of x and y only. This is best seen by example.

WORKED Example 19
Let a particle’s position, as a function of time, be given by u = t 2 i + t j . Find the equation
of the path, assuming t ≥ 0. ˜ ˜ ˜
THINK WRITE
1 Express the i and j components of u x = t2
˜ time
in terms of their ˜ functions. ˜ y=t
2 Express each equation in terms of an x = t2
identical function of t. y2 = t2
3 Equate the two expressions. t2 = x = y2
So x = y 2, or
4 Make y the subject. y= x

The most difficult work is often in step 2, finding equivalent functions of t for both x
and y. Sometimes squaring (or taking the square root of) one or more of the x(t) and y(t)
functions will yield successful results. Otherwise the simultaneous equations can be
solved using substitution as demonstrated in the following example.
340 M a t h s Q u e s t M a t h s C Ye a r 1 1 f o r Q u e e n s l a n d

WORKED Example 20
Find the equation of the path of a particle whose position is given by:
v = 2ti + ( t 2 – t ) j , t ≥ 0
˜
Sketch ˜the graph˜ of its path.

THINK WRITE/DRAW
1 Express the i and j components in terms x = 2t
of their time˜functions.
˜ y = t2 − t
x
2 Express t as a function of x. t = ---
2
x 2 x
3 Substitute for t in the equation y = t2 − t. y =  --- – ---
 2 2
x x2
4 Simplify. = ----- – ---
4 2
x x
5 Since t ≥ 0, --- ≥ 0. Since t ≥ 0, --- ≥ 0
2 2
6 State the domain of the equation. x≥0
7 Use a graphics calculator to sketch the y
graph over the domain [0, ∞).

0 2 3 x
(1, – 1–)
4

WORKED Example 21
Let a particle’s position as a function of time be given by u = cos ti + sin t j .
a Find the equation of the path. ˜ ˜ ˜
b Sketch the graph of the motion of the particle.
c State the period of the motion.
THINK WRITE/DRAW
a 1 Express the i and j components of u in a x = cos t
terms of their˜ time ˜functions. ˜ y = sin t
2 Square both sides of each equation so that a x2 = cos2 t
trigonometric identity can be used to eliminate t. y2 = sin2t
3 Add the two equations. x2 + y2 = cos2t + sin2t
4 Use the trigonometric identity cos + sin = 1
2 2
x2 + y2 = 1 y
to simplify the equation.
b The equation represents a circle of radius 1 b 1
and centre (0, 0).

–1 0 1 x

–1
c The period of cos t and sin t is 2π (the path c Period = 2π.
makes one revolution every 2π).
Chapter 7 Introduction to vectors 341
WORKED Example 22
Let a particle’s position as a function of time be given by u = 2 cos ti + 3 sin t j . Find the
equation of the path and sketch its graph. ˜ ˜ ˜

THINK WRITE/DRAW
1 Express the i and j components of u in terms of x = 2 cos t
˜
their functions. ˜ ˜ y = 3 sin t
x
2 In this case, first eliminate the constants in front of --- = cos t
2
the trigonometric functions.
y
--- = sin t
3
x2
3 Square both sides of the equation. ----- = cos2 t
4
y2
----- = sin2 t
9
x2 y2
4 Add the 2 equations. - = cos2 t + sin2 t
----- + ----
4 9
x2 y2
5 Use the trigonometric identity cos2 + sin2 = 1. ----- + ----
- =1
4 9
6 This is the equation of an ellipse. y
3

–2 0 2 x

–3

Graphics Calculator tip! Vector


of time
functions

To draw the graph of a time-varying vector, we need to express the components in


terms of parametric equations. Consider drawing the graph of the position vector
u = 2cos t i + 3sin t j from Worked example 22 using a graphics calculator.
˜ ˜ ˜
For the Casio fx-9860G AU
1. Press MENU and select GRAPH. Ensure that the
Angle setting is shown as radians. Press SHIFT
[SET UP] and alter the setting for Angle if necessary.
Press F2 (Rad) for radians.
2. Press EXE to accept this setting. Press F3 (TYPE)
followed by F3 (Parm) to select the parametric
equations option.
342 M a t h s Q u e s t M a t h s C Ye a r 1 1 f o r Q u e e n s l a n d

3. Enter the x-component by completing the entry line


for Xt1= with 2 cos t. (Press X,q,T to enter t.)
Similarly, enter the y-component by completing the
entry line for Yt1= with 3 sin t. Press EXE after each
entry.
4. Press F6 (DRAW) to display the graph. Note that
with the standard window setting used, there are not
equal scale values on the two axes so it doesn’t give
a true representation of the shape of the graph.

s
5. Press SHIFT F2 (ZOOM) followed by F6 ( )
for more options. Press F2 (SQR) to correct the
V-Window settings so that the x-axis and y-axis
values are identical. (You can also experiment with
other Zoom options to enhance the display.)
6. To obtain a clearer view of the graph, you can adjust
the Window settings. Press SHIFT F3 (V-WIN) and
adjust the values for Xmin, Xmax, Ymin and Ymax.
Alternatively, press SHIFT F3 (V-WIN) followed by
F1 (INIT) to initialise the V-Window. Press EXE
and then press F6 (DRAW) to display the graph
with this new setting.

For the TI-Nspire CAS


1. First ensure that the Angle setting is shown as
radians. Press c to access the home screen and
select 8: System Information followed by 2:
System Settings. Press e until you reach Angle
and then select Radian by using the arrow keys.
Press · to accept this setting.

2. Continue pressing e until you highlight OK. Press


· to select OK. Open a new Graphs & Geometry
document (press / N and select 2: Add Graphs &
Geometry). Press b and select 3: Graph Type
followed by 2: Parametric.

3. Enter the x-component by completing the entry


line for x1(t) with 2 cos t. Similarly, enter the
y-component by completing the entry line for
y1(t) with 3 sin t. (Use the arrow keys to move
between entry lines.)
Chapter 7 Introduction to vectors 343
4. Press · to display the graph. If the entry line is
covering too much of the graph and you wish to hide
it, press / G. (To bring the entry line back, press
/ G again.)
5. To obtain a clearer view of the graph, you can adjust
the window settings. Press b and select 4: Window
followed by 1: Window Settings. Adjust the values
for XMin, XMax, YMin and YMax.
Alternatively, press b and select 4: Window
followed by 3: Zoom - In. Use the arrow keys to
define the centre point of the zoom and press ·.
This will increase the size of the graph by a factor of
approximately 2. (Continue to press · to increase
the size of the graph as required.)

These techniques work well for 2-dimensional vectors, but 3-dimensional vectors
usually are more difficult as the paths are much more complicated. Three-dimensional
vectors will not be covered in this part of the course.

WORKED Example 23
Let u = (2t + 1) i + (3t – 4) j be the position vector of Ship A.
˜ ˜ ˜
Let v = (3t – 2) i + (2t + 3) j be the position vector of Ship B.
˜ ˜
Find where the ships’ paths˜ cross.

THINK WRITE
1 Express the i and j components for the path Ship A: x = 2t + 1
of Ship A in˜terms˜of their time functions. y = 3t − 4
x–1
2 Make t the subject of each equation. ----------- =t
2
y+4
------------ = t
3
y + 4-
x – 1 -----------
3 Equate the two expressions. ----------- =
2 3
4 Simplify the equation and make y the 3x − 3 = 2y + 8
subject. Call this equation A. 3x – 11
y = ------------------ [A]
2
5 Express the i and j components of Ship B Ship B: x = 3t − 2
˜ time
in terms of their ˜ functions. y = 2t + 3
6 Make t the subject of each equation. x + 2-
----------- =t
3
y–3
----------- = t
2
Continued over page
344 M a t h s Q u e s t M a t h s C Ye a r 1 1 f o r Q u e e n s l a n d

THINK WRITE
x+2 y–3
7 Equate the two expressions. ------------ = -----------
3 2
8 Simplify the equation and make y the 2x + 4 = 3y − 9
subject. Call this equation B. 2x + 13
y = ------------------ [B]
3
9 The ships’ paths will cross when the two Equating equations A and B:
equations (equations A and B) are equal. 3x – 11 2x + 13
------------------ = ------------------
2 3
10 Solve for x. 9x − 33 = 4x + 26
5x = 59
x = 11.8
11 Substitute x = 11.8 into equation B to Substituting into equation B:
find y. 2 ( 11.8 ) + 13
y = -------------------------------
3
= 12.2
12 State the solution. The ships’ paths cross at the point with
Note: Although the paths cross, the ships coordinate (11.8, 12.2).
might not be there at the same time!

Note: Time-varying vectors will be considered in more detail in Chapter 8.

remember
remember
If u = x ( t )i + y ( t ) j , t ≥ 0, then the equation of the path of a particle can be found
by ˜solving the
˜ equations˜ x(t) and y(t) simultaneously.

7E Time-varying vectors
Use a graphics calculator to assist where appropriate in the following exercise.
1 For each of the following, find the equation of the path, assuming t ≥ 0.
x
WORKED a u = 2t i – t j y = – ---
Example ˜ ˜ ˜ 2
19 b u = ( t – 1 )i – 3t j y = –3x – 3
˜ ˜ ˜
c u = ( t + 3 )i + 4t 2 j y = 4 ( x – 3 ) 2
˜ ˜ ˜ 3
d u = 2t i + t 3 j y = ----
x
-
˜ ˜ ˜ 8
2 multiple choice
The value of u when t = 0, for the vector u = ( t + 3 )i + 4t 2 j is:
˜ ˜ ˜ ˜
A 0 B 3i C 3i + 4 j D 4i + 4 j E none of these
˜ ˜ ˜ ˜
Chapter 7 Introduction to vectors 345
b y
WORKED 3 a Find the equation of the path of a particle whose position is given by:
Example
20 v = --2t- i + ( t 2 + t ) j y = 4x2 + 2x
˜ ˜ ˜
b Sketch the graph of its path.
0 x
b y
4 a Find the equation of the path of a particle whose position is given by:
Note: x ≥ 1 v = ( t + 1 )i + ( t 2 – 2t ) j y = x2 − 4x + 3 b y
0 ˜ ˜ ˜ 1
–1 1 3 x b Sketch the graph of its path.
(2, –1)
WORKED 5 Let a particle’s position as a function of time be given by u = cos 2t i + sin 2t j . –1 0 1 x
Example ˜ ˜
a Find the equation of the path. x2 + y2 = 1 ˜
21 –1
b Sketch the graph of the motion of the particle.
c State the period of the motion of this particle. Period = π
6 A particle’s position as a function of time is given by u = 3cos 2t i + 3sin 2t j . b y
3
a Find the equation of the path. x2 + y2 = 9 ˜ ˜ ˜
b Sketch its graph. –3 0 3 x
c State the period of the motion of this particle. Period = π
–3
7 A particle’s position as a function of time is given by
b y u = ( 1 + cos t )i + ( –2 + sin t ) j .
˜ ˜ ˜
a Find the equation of the path. (x − 1)2 + (y + 2)2 = 1 x2
8 ----- + y 2 = 1
0 1 2 x
–1 b Sketch its graph. 9
y
–2
–3
c State the period of the motion of this particle. Period = 2π
1
WORKED 8 Let a particle’s position, as a function of time, be given by: –3 0 3 x
Example
22 u = 3cos 2t i + sin 2t j –1
˜ ˜ ˜
Find the equation of the path and sketch its graph.
x2 y2
9 Let a particle’s position, as a function of time, be given by: 9 ----- + ------ = 1
4 16
u = 2cos t i – 4sin t j y
˜ ˜ ˜ 4
Find the equation of the path and sketch its graph.
10 Find the equation of the paths described by each of the following vector functions: –2 0 2 x

t2 + 4 4t
a u = ------------------ i + ------------------ j (Hint: Add x -and y-components and factorise.) –4
˜ ( t + 2 ) ˜ ( t + 2 )2˜
2

What is the initial position of u ?


˜
When does x = y? What is the position at this time? y = 1 – x ; u ( 0 ) = i ; t = 2, u = 1--2- i + 1--2- j
˜ ˜ ˜ ˜ ˜
b u = ( t + 2 )i + ( t + 1 ) 2 j y = (x – 1)2, x ≥ 2
˜ ˜ ˜
c u = ( 2 cos t + 3 )i + ( 3 sin t – 1 ) j (------------------ x – 3 )2 ( y + 1 )2
- + ------------------- = 1
˜ ˜ ˜ 4 9
WORKED 11 Let u = ( 3t + 1 )i + ( 4t – 2 ) j be the position vector of Ship A.
Example ˜ ˜
23 Let v = ( 2t + 3 )i + ( 5t + 1 ) ˜j be the position vector of Ship B.
˜ ˜ ˜
Find where the ships’ paths cross. x = -----
19-
7
, y = --27-

eBook plus 12 Let u = t i + t 2 j . Find the equation of the path. Consider vector v = e t i + e 2t j .
Show˜ that the
˜ path ˜ both ˜vectors’
˜ of this vector is the same as the path of u . Assuming ˜
Digital doc:
˜
equations start when t = 0, do these vectors ever coincide? y = x ; No, since v is always ‘ahead’ of u .
2

WorkSHEET 7.2
˜ ˜
346 M a t h s Q u e s t M a t h s C Ye a r 1 1 f o r Q u e e n s l a n d

summary
Vectors and scalars
• Definition: A vector is a quantity that has magnitude and direction.
• Equality of vectors: Two vectors are equal if both magnitude and direction are
equal.
• Addition of vectors: To add two vectors, take the tail of one vector and join it to the
head of the other. The result of addition is the vector from the tail of the first vector
to the head of the second.
• Subtraction of vectors: Subtract vectors by adding the negative of the second vector
to the first vector.
• Multiplication of vectors (by a scalar): Multiply the magnitude of the vector by the
scalar, maintaining the direction of the original vector.

Position vectors in 2 and 3 dimensions


• Magnitude of a vector: If u = xi + y j + zk , magnitude = u = x2 + y2 + z2
˜ ˜ ˜ ˜ ˜
y
• Direction of a vector (2-D only): If u = xi + y j , direction = θ = tan−1 --
˜ ˜ ˜ x
• x- and y-components of a vector: Given magnitude and direction, the x- and
y- components are given by:
x = u cos θ
˜
y = u sin θ
˜
• Unit vector: The unit vector of a vector u , in the direction of u , denoted by û is:
˜ ˜ ˜
u
û = -----
˜
˜ u
˜
• Locating vectors: If A and B are points with position a and b respectively then
˜ ˜
AB = b – a
˜ ˜
Multiplying two vectors — the dot product
• Scalar (dot) product: The scalar or dot product of two vectors u and v is denoted
by u • v . ˜ ˜
˜ ˜
• Calculation of dot product: Where θ is the angle between two vectors:
u • v = u v cos θ
˜ ˜ ˜ ˜
• Algebraic calculation of dot product: Let u = x 1 i + y 1 j + z 1 k and
˜ ˜ ˜ ˜
v = x2 i + y2 j + z2 k .
˜ ˜ ˜
Then u • v = x 1 x 2 + y 1 y 2 + z 1 z 2 .
˜ ˜
If u • v = 0 , then u and v are perpendicular.
˜ ˜ ˜ ˜
If u = kv, k ∈ R , then u and v are parallel.
˜ ˜ ˜ ˜
Chapter 7 Introduction to vectors 347
Resolving vectors — scalar and vector resolutes
• Scalar resolute of v on u : Let û be a unit vector in the direction of u . The scalar
˜ ˜ ˜ ˜
resolute of v on u is given by û • v .
˜ ˜ ˜ ˜
• Vector resolute of v parallel to u : Let û be a unit vector in the direction
of u . ˜ ˜ ˜
˜
v || = ( û • v )û
˜ ˜ ˜ ˜
• Vector resolute of v perpendicular to u :
˜ ˜
v ⊥ = v – ( û • v )û or
˜ ˜ ˜ ˜ ˜
v ⊥ = v – v ||
˜ ˜ ˜

Time-varying vectors
• If u = x ( t )i + y ( t ) j , t ≥ 0, then the equation of the path of a particle can be found
by ˜solving the
˜ equations˜ x(t) and y(t) simultaneously.
348 M a t h s Q u e s t M a t h s C Ye a r 1 1 f o r Q u e e n s l a n d

CHAPTER
review
1 multiple choice
7A
If u = 4i – 3 j + 0.2k and v = 2i + 4 j – k , then 4u – 2.5v is equal to:
˜ ˜ ˜ ˜ ˜ ˜ ˜ ˜ ˜ ˜
A 11i – 22 j + 3.3k B 6i + j – 0.8k C 8i – 12 j – 0.2k
˜ ˜ ˜ ˜ ˜ ˜ ˜ ˜ ˜
D 21i – 13 j – 2.3k E 11i – 13 j
˜ ˜ ˜ ˜ ˜
2 A fire observation tower reaches 40 m above the ground. Susan is 400 m from the tower,
7A,B which is at a bearing of 60° (N 60° E) from her. State the position vector from Susan’s
current position to the top of the tower. 200 3i + 200 j + 40k
˜ ˜ ˜
40 m
N

400 m
60°

E
Susan

3 multiple choice
7B
The vector with a magnitude of 5 is:
A 3i + j + k B 2i + 5 j + 4k C 5i + 5 j + 5k
˜ ˜ ˜ ˜ ˜ ˜ ˜ ˜ ˜
D 6i + 3 j – 4k E 25i
˜ ˜ ˜ ˜

4 multiple choice
7B
Let the position vector for point P be 3i + 4 j – 5k and for point Q be – i + 3 j – 4k . Then
˜ ˜ ˜ ˜ ˜ ˜
the magnitude of the vector PQ is given by:

A 3 2 B 134 C 553 D 4 E 1

5 A boat sails 5 km due east from H, turns northward at a bearing of 45° (N 45° E) for a
7B distance of 10 km and then travels due north for a further 5 km to point X.
a Find the position vector from H to X. ( 5 + 5 2 )i + ( 5 + 5 2 ) j
˜
b Find the distance from H to X (correct to 2 decimal places).˜ 17.07 km

6 multiple choice
7C
If u = 3i + 2 j – 4k and v = –4i + 5 j + k , then u • v is equal to:
˜ ˜ ˜ ˜ ˜ ˜ ˜ ˜ ˜ ˜
A 10 B 0 C 480 D −6 E 33
Chapter 7 Introduction to vectors 349
7 multiple choice
7C
The angle between u = 3i – 2 j and v = 16i + 24 j is equal to:
˜ ˜ ˜ ˜ ˜ ˜
A 0° B 12.3° C 45° D 60° E 90°

8 multiple choice
7C
A unit vector perpendicular to 3i – 4 j is:
˜ ˜
A 4i + 3 j B 0i + 0 j C 0.8i + 0.6 j D 4i – 3 j E –3i + 4 j
˜ ˜ ˜ ˜ ˜ ˜ ˜ ˜ ˜ ˜
9 multiple choice
7C
If u = 3i + a j , v = 2ai – a j and it is known that u is perpendicular to v , then a is:
˜ ˜ ˜ ˜ ˜ ˜ ˜ ˜
A 0 B 1 C −1 D 6
E cannot be determined with the given information

10 multiple choice
7C
The angle between u = 4i – 2 j + 3k and v = 2i – 2 j is:
˜ ˜ ˜ ˜ ˜ ˜ ˜
−1 1- 3
A cos 0 B cos 58−1 --------- C cos−1 ---------
10
-

1 6
D cos−1 ---------
10
-
E cos−1 ---------
58
-

11 Let u = 4i + 3 j and v = –i + 2 j . Find the angle between the two vectors, in radians, to
˜ ˜ ˜ ˜
4 decimal places. 1.3909
˜ ˜ 7C
12 Let u = 3i – 5 j and v = –4i + j . Find:
˜ ˜ ˜ ˜ ˜ ˜ 7A–C
a u + v – i – 4j b u – v 7i – 6 j c u • v −17 d û ---------
3 5
- i – ---------- j
˜ ˜ ˜ ˜ ˜ ˜ ˜ ˜ ˜ ˜ ˜ 34 ˜ 34
˜
e the angle between u and v . 135°
˜
13 Find the angles that the vector v = 3i – 2 j + 4k makes with the x, y and z axes.
˜ ˜ ˜ ˜ 56.1°, 111.8°, 42.0° 7C
14 Find value(s) of p, such that pi + 2 ( 1 – 3 p ) j is perpendicular to 2 pi + 3 j .
˜ ˜ ˜ ˜
9 ± 69
7C
Using the vectors a = i – 2 j and b = 2i + 3 j answer questions 15 and 16. -------------------
2
˜ ˜ ˜ ˜ ˜ ˜
15 multiple choice
7D
The scalar resolute of a on b is given by:
˜ ˜
4 4 4
A – -----
- B – ---------
- C – 4--- D – ------- E −4
13 13 5 5

16 multiple choice
7D
The vector resolute of b parallel to a is:
˜ ˜
4-
A – ------ B – 4--5- ( i – 2 j ) C – ------ ( 2i
4
13 ˜
+ 3 j)
5 ˜ ˜ ˜
-( i – 2 j) --- ( 2i + 3 j )
4 4
D – -----
13 ˜
E – 5 ˜
˜ ˜
350 M a t h s Q u e s t M a t h s C Ye a r 1 1 f o r Q u e e n s l a n d

17 Let u = 2i + 3 j – k and v = i + j – 2k .
7D ˜ ˜ ˜ ˜ ˜ ˜ ˜ ˜
- i + ---------- j – ---------- k
a Find a unit vector parallel to u . ---------
2
14 ˜
3 1
14 ˜
˜ 14
˜
b Resolve v into components parallel and perpendicular to u .
˜ ˜
v || = 1--2- ( 2i + 3 j – k ) , v ⊥ = – 1--2- ( j + 3k )
18 multiple choice ˜ ˜ ˜ ˜ ˜ ˜ ˜
7E Let u ( t ) be a position vector of an object, whose position varies with time. If
˜
u = 3 sin t i + 2 cos t j then the path this object takes is: 2
˜ ˜ ˜ 19 y = ----2- – 2 , hyperbolic
A a straight line x
B a parabola y

C a circle
D an ellipse 1
x
0
E unable to be determined with the given information –2

1
19 Find the equation of the path of the time-varying position vector u = --- i + 2 ( t – 1 ) j . State
7E the type of path (linear, parabolic, etc.). Hence, sketch its graph. ˜ t˜ ˜

Modelling and problem solving


1 A radar station tracks a jet fighter flying with constant speed. If the radar station is considered
to be at the origin, the fighter’s starting position is 2i + 8 j + k and 1 minute later it is at
8i – 4 j + 13k . The units are in kilometres. ˜ ˜ ˜
˜ ˜
˜ the vector which indicates the path of the fighter. 6i – 12 j + 12k
a State
˜ ˜ ˜
b State a unit vector in the direction of this path. --13- ( ˜i – 2 j + 2k˜ )
˜
c Find a vector, in terms of a parameter m, which represents the position of the fighter at any
m
time along the path. 2i˜ + 8 j + k˜ + ---3- ( ˜i – 2 j + 2k˜ )
˜ ˜
d Find the point along the path where the fighter is closest to the station. 1--3- ( 10i + 16 j + 11k )
˜ ˜ ˜
e Find the distance from the station at this point. 7.28 km
f Find the speed of the plane in km/h. 1080 km/h
g How fast is the plane rising (or falling)? 720 km/h
Chapter 7 Introduction to vectors 351
2 Consider the box shown at right. z
The coordinates (in cm) of point D are (3, 0, 4), while the E
coordinates of E are (0, 5.5, 4).
a Find the coordinates of point C. (3, 5.5, 0) D
b Express the line joining C to E as a vector. – 3i + 4k
˜ ˜
c Show that the two diagonals in the same plane as CE y
intersect with an angle of 73.7°.
d Find the volume of the box in litres. 66 cm3 = 0.066 litres x C

e Express the longest diagonal, from the origin, as a vector. 3i + 5.5 j + 4k


˜ ˜ ˜
f Find the length of this diagonal. 7.43 cm
g Find the angle that this diagonal makes with the other long diagonal. 84.5°

3 The parallelogram OXYZ has O at the origin. The vector joining O to Z is given by 5i while a
the vector joining O to X is given by 2i + 7 j . ˜ y
˜ X Y(7, 7)
˜
a Sketch the parallelogram, labelling all vertices.
2i~ + 7j
b State the vectors joining Z to Y and Y to X. 2i + 7 j, – 5i ~
˜ ˜ ˜
c State the vectors which represent the diagonals of the parallelogram. 7i + 7 j, –3i + 7 j
˜ ˜ ˜ ˜ x
d Find the cosine of the angle between the diagonals. Express your answer in simplest surd O 5i~ Z

form. ------------
2 29-
29
e Find the angle that OX makes with the x-axis. 74.1°
f State the vector resolute of the vector joining O to X in the direction of OZ. 2i
˜
g Let P be a point on the extended line of XY, such that the vector joining P to Z is
perpendicular to OY. Find the coordinates of P. (−2, 7)
h Find the area of the parallelogram. 35 square units

4 A river flows west–east at 5 m/s. A swimmer, in still water, can swim 3 m/s and tries to swim
directly across the river from south to north. a N
5
a Draw a vector diagram to illustrate this situation.
3
b Find the resultant speed of the swimmer. 5.83 m/s
c Find the bearing of the swimmer. 059°
d If it took the swimmer 2 minutes to reach the opposite bank, how wide is the river? 360 m
e How far downstream would the swimmer be carried? 600 m
f Repeat parts b to e if the swimmer had started on the north bank.
Same result, except bearing = 180° − 059° = 121°
5 The top of an 8-m diving board lies over the
swimming pool as illustrated at right. Kate
Sally sits 25 metres away in the corner of the
swimming pool and takes a bearing of 35° (N 35° E)
to the feet of her friend, Kate, who is about to do a N 8m
belly flop.
X
a State the position vector from Sally’s current
Sally 35° 25 m
location to her friend’s feet. 14.34 i + 20.48 j + 8 k
˜ ˜ ˜ E
b Find the distance between Sally and her friend’s
feet. 26.25 m
c Determine the angle of elevation from Sally (at ground level) to the top of her friend’s
head if her friend is 1.75 metres tall. (Give your answer to the nearest tenth of a degree.)
21.3°
352 M a t h s Q u e s t M a t h s C Ye a r 1 1 f o r Q u e e n s l a n d

6 Because of air resistance, a skydiver is eventually falling at a constant speed. If his target
point on the ground is considered to be at the origin, the skydiver’s starting position is
6i + 4 j + 12k , while 1 minute later he is at 3i – 5 j + 4k . (The units are in kilometres.)
˜ ˜ ˜ ˜ ˜
a Indicate the skydiver’s path with a vector. −3 i ˜− 9 j − 8 k
˜ ˜ 1 -
eBook plus b State a unit vector in the path’s direction.
˜ ------------ (−3 i − 9 j − 8 k )
154 ˜ ˜ ˜
Digital doc:
c Find the distance from the target to the skydiver at this point (that is, after 1 minute). 5 2 km
Test Yourself d Find the speed of the skydiver in kilometres per hour. 744.6 km/h
Chapter 7
8
Vector
applications

syllabus reference
Core topic:
Vectors and applications

In this chapter
8A Force diagrams and the
triangle of forces
8B Newton’s First Law
of Motion
8C Momentum
8D Relative velocity
8E Using vectors in geometry
354 M a t h s Q u e s t M a t h s C Ye a r 1 1 f o r Q u e e n s l a n d

• two- and three-dimensional vectors and their algebraic and geometric representation
• resolution of vectors into components acting at right angles to each other
Introduction • applications of vectors in both life-related and purely mathematical situations

In the previous chapter, we discussed the theory of vectors. Vectors are the ideal math-
ematical tool for dealing with the motion of, and forces on, objects in two and three
dimensions. In this chapter we consider these applications of vectors — forces in
equilibrium, momentum and relative velocity.

Force diagrams and the triangle


of forces
Introduction
An understanding of the causes of motion is fundamental to our understanding of the
world. All around us objects are in motion. In the seventeenth century, Sir Isaac
Newton succeeded in producing three simple laws which accurately described the
motion of large objects. His mathematical model was based on the effect that forces
have on the acceleration of objects treated as point particles. These effects are
embodied in his First and Second Laws of Motion.

What is a force?
We all know from experience what a force is. It is a ‘push’ or a ‘pull’. The force due to
gravity acts on us all the time. A bar magnet repels a second bar magnet: a magnetic
force acts here. The force of friction slows down the wheel of a bike when the brakes
are applied. Air resistance retards the motion of athletes. In both these cases there is
relative motion between two objects. The strings of a tennis racquet when stretched
exert a force on a tennis ball while the strings and ball are in contact. In all these
examples, objects which have unbalanced forces acting on them tend to undergo a
change in their motion; objects which have a balanced set of forces acting on them
maintain their motion. Objects under balanced forces are said to be in equilibrium.
The forces that we will discuss in this chapter can be classified as one of three types:
field forces, applied forces, and resistive forces.

Field forces

Field forces act without physical


contact between objects. The
weight force which acts on an
object due to the presence
of an external
gravitational field or
the electric force
which influences
the motion of a
charged particle
are examples of
field forces.
C h a p t e r 8 Ve c t o r a p p l i c a t i o n s 355
Applied forces
Applied forces are the pushes or
pulls exerted on objects due to
contact. They are forces with
which we have daily experience.
The normal contact force acting
upwards on a book resting on a
table or on us as we stand on the
floor or sit on a chair are examples
of applied forces. Other examples
of applied forces include tensile
forces in taut strings and cables
(as in the cable used by a crane
or rescue helicopter), and com-
pressive forces acting on weight-
bearing rods.

Resistive forces
Air drag and friction are examples
of resistive forces. This type of
force occurs when two objects
move or attempt to move relative to
one another. Air drag has been put
to good use in the design of hang
glider; it is found also, in the resis-
tance between a moving body like a
car and the air. An example of fric-
tion is seen in a bicycle that is
slowing down on level ground, even
without the brakes being applied.

What is a particle?
We are all familiar with the notion of a particle, but in Newtonian dynamics a particle
is used to model an object and is taken to be a point. That is, the size of the object is not
relevant and any internal movements such as spin and change in shape are not included
in the model. A particle can, in principle, model the movement of a bullet, a car, a diver
or a planet, provided that we accept the model as an approximation to the motion of the
real objects they represent. Any force that acts on a particle is said to act through the
point that defines the position of the particle.
For example, if we were to describe the various forces acting on a car, we would
include the upwardly directed contact forces exerted on the car by the road at each of
the four tyres in addition to frictional forces exerted on the tyres by the road. If the car
was moving, we would add the drag forces due to the movement of the car through the
air. There would be the weight force acting on the car due to the gravitational field of
the earth. If we were to treat the car as a particle, we would describe all these forces as
acting through a single point. The word particle serves to define the position of an
object and sets it apart from the rest of the immediate environment.
356 M a t h s Q u e s t M a t h s C Ye a r 1 1 f o r Q u e e n s l a n d

What assumptions do we make in Newtonian dynamics?


In modelling motion we make the following assumptions so that problems can be
solved to give reasonably accurate predictions.

Term Meaning

Light (body or string) Object has no mass.

Smooth No frictional forces are exerted.

Inextensible Strings or ropes do not stretch.

Rigid Objects do not change shape when forces are applied


to them.

Perfectly elastic Applied forces do not permanently deform an object


(for example, a spring).

In many cases we ignore the presence of forces which would be insignificant, such as
air drag on slowly moving objects.

The resultant force R


Crucial to a good understanding˜of Newton’s laws of motion is the concept of a net or
resultant force acting on a particle. Force is a vector quantity because a force has not
only a magnitude but also a direction. The unit of force is the newton (N). (This is a
derived unit, which is simply one defined in terms of the standard units, namely dis-
tance, time and mass. This will be discussed later.)

Forces can be described in two ways:


1. Using i – j – k notation
˜ ˜ ˜
For example, F = 2i – 4 j N (Only coplanar forces will be considered.)
˜ ˜ ˜
2. Using size and direction notation
For example, F = 200 N, N45°E is a force of magnitude 200 N directed at an angle
˜
45° from north towards east.

The net or resultant force is simply the vector sum of all real physical forces acting
on the particle. It represents the sum or total force acting on a particle representing an
object. It is not in itself a real force, only the sum of real forces.

The net or resultant force acting on a particle is the vector sum of all real forces
acting on that particle.

Force diagrams
Individual forces are one of three types — field, contact or resistive — and are drawn
as vectors which indicate their direction and magnitude.
C h a p t e r 8 Ve c t o r a p p l i c a t i o n s 357
WORKED Example 1
Draw ‘vector diagrams’ to represent the forces involved in the situations shown below as a
set of vectors acting on a point particle. Indicate the relative size of the force by the length
of the vector arrows. Further indicate the nature of each of the forces acting by labelling
them W for weight, N for normal contact, F for friction, A for applied force and D for
air drag ˜
˜ or air resistance. ˜ ˜ ˜
a A stationary person b Constant velocity c Accelerating

d Velocity e f

Constant
velocity
Cricket ball
through the air Ball rolling down a slope

THINK WRITE
In the force diagrams for a to f, treat each
object as a particle.
a There are two equal, opposing forces: the a
~N
weight force down, which is a field force,
and the normal contact force up.

W
~

b 1 A cart moving at constant velocity has b


~N
balanced horizontal forces.
~D A
~
2 Any resistive forces (D ) must be balanced
by an applied force ( A˜) to keep the cart
˜
moving at constant velocity. W
~

3 The cart will have balanced vertical


forces arising from the weight force of
the cart and the normal contact force.
c An accelerating cart will have balanced c N
~
vertical forces as in a and b but
unbalanced horizontal forces giving rise to ~D A
~
an acceleration.
W
~
Continued over page
358 M a t h s Q u e s t M a t h s C Ye a r 1 1 f o r Q u e e n s l a n d

THINK WRITE

d 1 A ball moving through the air will have d


the vertical force of weight and the ~D
resistive force of air drag. W
~
2 The drag force will be in the opposite
direction to that of the ball.
e A ball rolling down a slope will have a e N
~
weight force directed vertically down, a ~F
normal contact force perpendicular to the
slope and a resistive or frictional force.
W
~
f The parachutist will have two balanced f
~D
vertical forces: one down due to the weight
force and the second upwards due to air
drag.
W
~

WORKED Example 2
Three forces — F 1 , F 2 and F 3 — act on a ball as shown in the force vector diagram at
˜ ˜ are described
right. The three forces ˜ by the vectors: ~F3 j
~
F1 = 4i – 5 j ~F2
˜ ˜ ˜ i~
F2 = 10i + 2 j
˜ ˜ ˜
F3 = –6i + 7 j . ~F1
˜ ˜ ˜
a Find the resultant force R , the sum of the three forces F 1 , F 2 and F 3 .
b Find the magnitude of the˜ resultant force R . ˜ ˜ ˜
˜
c Find the angle that the vector R makes with the i vector.
˜ ˜
d The force F 1 is changed so that R = 0; that is, the resultant force equals zero. Find the
˜ ˜
force F 1 .
˜
THINK WRITE
a 1 The resultant force R is the sum of the a j
forces F 1 , F 2 and F˜ 3 . ~ R
~
˜ ˜ ˜
~i ~F3

~F1
~F2
R = ΣF
˜ ˜
= F1 + F2 + F3
˜ ˜ ˜
2 Evaluate R . = 4i – 5 j + 10i + 2 j – 6i + 7 j
˜ ˜ ˜ ˜ ˜ ˜ ˜
= 8i + 4 j
˜ ˜
C h a p t e r 8 Ve c t o r a p p l i c a t i o n s 359
THINK WRITE

b 1 The magnitude of the vector R is its length. b


˜
2 The symbol R or R is used to represent
the magnitude˜ of a vector R . R = R•R
˜ ˜ ˜ ˜
3 Recall that i • i = j • j = 1 and = ( 8i + 4 j ) • ( 8i + 4 j )
i • j = j • i ˜ =˜ 0 , ˜and˜ evaluate R . ˜ ˜ ˜ ˜
˜ ˜ ˜ ˜ ˜ = 8 +4
2 2

= 80
= 4 5
c 1 Define θ. c Let the angle that R makes with i be θ.
˜ ˜
2 Then use the result for the dot product of Let R • i = R i cos θ
two vectors a • b = a b cos θ. ˜ ˜ ˜ ˜
˜ ˜ ˜ ˜ ( 8i + 4 j ) • i
cos θ = ---------------------------
˜ ˜ ˜-
4 5×1
8
3 Evaluate θ to the nearest tenth of a cos θ = ----------
degree. 4 5
cos θ ≈ 26.6°
d 1 The vector sum of all forces is now equal d R = F1 + F2 + F3
to zero. Set R = 0. ˜ ˜ ˜ ˜
˜ = 0
2 Make F 1 the subject of the equation. F 1 = –( F 2 + F 3 )
˜ ˜ ˜ ˜
3 Substitute F 2 and F 3 into the equation F 1 = – ( 10i + 2 j – 6i + 7 j )
˜ to find˜ F .
and simplify ˜ ˜ ˜ ˜ ˜
˜1 = –4i – 9 j
˜ ˜

The triangle of forces


If three non-parallel forces acting on a particle have a X
~
resultant force of zero then the three forces can be rep-
resented in a triangle since the vector sum of the forces is
zero.
From the figure at right, if R = X + Y + Z = 0, then
the three forces X , Y and Z ˜can be ˜ represented
˜ ˜ in a tri-
~Y Z
~
˜ ˜ ˜
angle representing the magnitude and direction of the three
forces. X
~
The advantage of representing three forces in a triangle Z
~
is that the sine rule and/or cosine rule can be used to solve
some problems involving three forces whose vector sum is
zero. That is: ~Y
B
a b c c a
Sine rule: ------------ = ------------ = ------------- .
sin A sin B sin C
A C
Cosine rule: a2 = b2 + c2 − 2bc cos A. b
360 M a t h s Q u e s t M a t h s C Ye a r 1 1 f o r Q u e e n s l a n d

WORKED Example 3
Three forces — A, B and C — act on an object such that the resultant force is zero. The
˜ angle
force A acts at an ˜ of ˜150° to the force B and they have the same magnitude of 20 N.
˜
a Determine the magnitude of C . ˜
˜
b Find the angle that the force C makes with B to the nearest degree.
˜ ˜
THINK WRITE

a 1 Draw a force vector diagram of three a A


~
forces acting through a point with an
angle of 150° between A and B and an
˜ C .˜
angle of θ between B and 150°
˜ ˜ θ
C
~
~B
2 Since the resultant of the three forces is A
~
zero, place them in a triangle of forces.
C
~
3 Mark the angle between A and B as (180 – θ )°
180° − 150° = 30° and the˜ angle ˜
between B and C as 180° − θ. 30°
˜ ˜
B
~
4 Substitute a = C, b = 20, c = 20 and C 2 = 20 2 + 20 2 – 2 × 20 × 20 cos 30°
A = 30° into the cosine rule. = 107.18
5 Solve for C to find the magnitude of C . C = 10.35
˜
6 State the solution. The magnitude of force C is
˜
approximately 10.35 newtons.
20 10.35
b 1 Use the sine rule with a = 20, b --------------------------------- = -----------------
A = 180° − θ, b = 10.35 and B = 30. sin ( 180 – θ )° sin 30°

sin ( 180 – θ )° sin 30°


2 Invert both sides of the equation. --------------------------------- = -----------------
20 10.35
3 Make sin(180 − θ)° the subject of the 20 sin 30°
equation. sin ( 180 – θ )° = -------------------------
10.35
= 0.9662
4 Find the value of 180° − θ. 180° − θ = sin−1 (0.9662)
= 75.06°
5 Solve for θ. θ = 104.94°
6 State the solution to the nearest degree. The angle between forces B and C is
˜ ˜
approximately 105°.

Note: The angle between two forces in the ‘real’ situation and the angle between them
in a triangle of three forces are supplementary; that is, they sum to 180°.
1a ~N
C h a p t e r 8 Ve c t o r a p p l i c a t i o n s 361
Book

b
W
~
D
~
remember
remember
1. Force is a vector quantity. Its derived unit is the newton.
Ball 1 N = 1 kg m/s2
2. Types of force:
W
~ (a) Field forces occur without physical contact. A common field force is the
gravitational force often referred to as ‘weight’.
c
~N (b) Normal contact forces occur between objects in contact or via strings,
where they are known as tensile forces.
~F ~A
Car (c) Friction forces occur when there is actual or attempted relative movement
W
between two objects in contact.
~
3. The resultant force R is the vector sum of all physical
d ˜ ~F1
~N forces acting on an object. A force vector diagram
illustrates this with two forces F 1 and F 2 : ~F1
F A
4. If the resultant of three coplanar, ˜
˜ non-parallel forces
~ Boat ~
acting on a particle is zero then the three forces can be F2
W
~ represented in a triangle of forces. R
~ = ~F1 + ~F2 ~
e N F
~2
~
a b c
5. Sine rule: ------------ = ------------ = ------------- .
Sliding object sin A sin B sin C
6. Cosine rule: a2 = b2 + c2 − 2bc cos A.
W
~

Force diagrams and the


8A triangle of forces
WORKED 1 Draw vector diagrams to represent forces which act on the following objects. Indicate
Example
1
the relative size of the force by the length of the vector arrows. Further indicate the
nature of each of the forces acting by labelling them: N for normal contact forces,
1 W for gravitational forces (that is, weight), A for applied˜ forces, D for air resistance
˜
(drag) forces, and F for friction forces. ˜ ˜
f N
~
˜
a A book sitting on a table.
~F ~A
Accelerating car b A ball falling vertically through the air at constant speed.
W
~ c A car driving on a horizontal road at a constant speed.
g d A boat drifting through the water at constant speed.
~N
~F e A body sliding across a smooth horizontal surface at constant velocity.
Body at rest
f A car accelerating on a horizontal road.
W~
g A body at rest on an inclined plane.
h N
~
h A body sliding down an inclined plane at constant speed.
~F i A ball travelling vertically up (include air resistance).
Sliding j A ball travelling vertically down (include air resistance).
body
W
~ 2 multiple choice y
i Ball moving 4 3
up Refer to the diagram at right to answer the following questions.
~D a Using i and j notation, the exact value of the resultant force is:
W ˜ ˜ + 7j
~ A ( 4 + 3 3 )i B 7 3i + ( 3 + 4 3 ) j 3
j ˜ ˜ ˜ ˜
~D C ( 4 + 3 3 )i + ( 3 + 4 3 ) j D ( 4 – 3 3 )i + 7 j
Ball moving ˜ ˜ ˜ ˜ x
down E ( 4 + 3 3 )i + ( 4 3 – 3 ) j 4 3 3
˜ ˜
W
~
362 M a t h s Q u e s t M a t h s C Ye a r 1 1 f o r Q u e e n s l a n d

b The magnitude and direction, anticlockwise from the i direction, respectively of a


˜
third force for the resultant force to be zero is nearest to:
A 182 N, 49° B 15.3 N, 41° C 13.5 N, 41° D 182 N, 229° E 13.5 N, 221°
3 multiple choice A (5 newtons)
~
The three coplanar forces A , B and C act in such a way that
the resultant force is zero. ˜Use˜ a triangle
˜ of forces to assist in
C
~
answering the following questions.
a The magnitude of C is:
A 15 N B 21 N˜ C 3 N D −21 N E 0 N B (4 newtons)
~
b The angle between the forces A and C , to the nearest degree, is:
A 37° B 53° ˜ C 143°˜ D 137° E 127°
WORKED 4 Two forces — F 1 and F 2 — act on an object. They are described by the vectors:
Example ˜ ˜
2 i F 1 = 13i – 5 j, F 2 = –4i + 9 j
˜ ˜ ˜ ˜ ˜ ˜ eBook plus
4 a i 9i + 4 j ii F 1 = 8i + 6 j , F 2 = –14i – 9 j
˜ ˜ ˜ ˜ ˜ ˜ ˜ ˜
ii –6i – 3 j
˜
iii F 1 = 2 2i – 3 j, F 2 = – 3 5i + 2 j . Digital doc:
˜ ˜ ˜ ˜ ˜ ˜ ˜ EXCEL
iii ( 2 2 – 3 5 ) i – j For each of the above find: Spreadsheet
˜ ˜ Vector addition
b i 97 a the resultant force, R , acting on the object
ii 3 5 b the magnitude of the˜ resultant force, R
c a third force, F 3 , applied to the body so ˜ that the resultant force is equal to zero.
iii 54 – 12 10 ˜
5 Three forces — F 1 , F 2 and F 3 — act on an object. They are described by the vectors:
c i –9 i – 4 j ˜ ˜ ˜
˜ ˜ i F 1 = 3i – 5 j, F 2 = 4i + 9 j and F 3 = –6i – 2 j
ii 6i + 3 j ˜ ˜ ˜ ˜ ˜ ˜ ˜ ˜ ˜
˜ ˜
iii ( 3 5 – 2 2 ) i + j ii F 1 = i – j, F 2 = – 2 i + 3 j and F 3 = 2i – 2 j . 5 a i i + 2 j ii ( 2 – 1 ) i
˜ ˜ ˜ ˜ ˜ ˜ ˜ ˜ ˜ ˜ ˜ ˜ ˜ ˜
For each of the above find: b i 5 ii 2 – 1
c i 63.4° ii 0°
a the resultant force, R , acting on the object d i – i – 2 j ii ( 1 – 2 ) i
b the magnitude of the˜ resultant force, R ˜ ˜ ˜
c the angle that the vector R makes to the ˜ i vector
d a fourth force, F 4 , applied ˜ to the body so˜ that the resultant force is equal to zero.
˜
6 In each of the following cases:
6a i 61.4 N at N9.4°W
i find the resultant force, R , for the specified forces acting on a body
ii 61.4 N at S9.4°E ˜
ii describe the additional force needed to attain equilibrium; that is, R = 0.
b i 50i – 5 j ˜
˜ a F = 25 N at N30°E and G = 45 N at N30°W
ii – 50 i + ˜5 j ˜ ˜
c i
˜
– 4 i + 12˜j b W = – 80 j and F = 50i + 75 j
˜ ˜ ˜ ˜ ˜ ˜
ii 4i – 12 j˜ c A = 26i – 80 j and B = –30i + 92 j
˜ ˜ ˜ ˜ ˜ ˜
d i 2.9 N at N26.9°W ˜ ˜
d F = 10 N at S50°W and G = 7.0 N at N45°W and H = 12 N at N70°E
ii 2.9 N at S26.9°E ˜ ˜ ˜
WORKED 7 Three forces — X , Y and Z — act on an object such that the resultant
force R = 0. The ˜force ˜ Y acts ˜ at an angle of 120° to the force X and has
Example eBook plus
3 ˜ ˜ ˜
the same magnitude as the force X which is 10 N.
a Determine the magnitude of Z˜ . 10 N
Digital docs:
SkillSHEET 8.1
b Find the angle that the force Z˜ makes to X . 120° Sine rule
˜ ˜ SkillSHEET 8.2
8 Three coplanar forces — A , B and C — have magnitudes of 1000 N, Cosine rule
1200 N and 1700 N respectively. ˜ ˜ ˜
They act on a body such that the
resultant force is zero. Find the angle between A and B . 79.2°
˜ ˜
C h a p t e r 8 Ve c t o r a p p l i c a t i o n s 363
9 Three forces — X , Y and Z — add to give a resultant force X
~
equal to zero. ˜ ˜ ˜
~Y
a Find β if α = 135°, Z = 200 N and X = 150 N. 77° α
˜ ˜ β
b Find Z if α = 100°, β = 135° and Y = 27 N. 37.6 N θ
˜ ˜
c If X = Y and α = 60°, show that Z = 3 X . Z
˜ ˜ ˜ ˜ ~
Z = 2 sin 60° × X
˜ ˜

History of mathematics
S I R I S A AC N E W T O N ( 1 6 4 2 – 1 7 2 7 )
During his lifetime ... the discovery first. It became one of the
The Great Fire destroys much of great controversies of the times.
London. In 1687 Newton published his
major work, the Philosophiae
The Great Plague of London
Naturalis Principia Mathematica.
causes the death of more than
The Principia is considered by
12000 people.
many to be the greatest scientific
Huygens develops the pendulum book ever written. In it Newton
clock. gives a mathematical description
Lloyds of London starts its of the laws of mechanics and
insurance society. gravitation and applies this theory
to explain planetary and lunar
Sir Isaac Newton was a great English motion. His three laws of motion can
mathematician and physicist. He be expressed in simple terms as: 1. A body
developed the theory of differential calculus, at rest or in constant motion will stay in that
discovered the law of gravitation and is state until a force is applied; 2. Force = mass
regarded as the founder of modern physics. multiplied by acceleration; 3. For any action
Newton was the son of a wealthy man but his there is an equal and opposite reaction.
father died just three months before his son was In 1696 Newton took up a new direction
born. He was not happy as a child and his when he was appointed Warden of the Royal
school reports describe him as idle and Mint and then Master in 1699. He was
inattentive. It is thought that he suffered from responsible for an urgently needed reform of the
bouts of depression throughout his life. coinage and also introduced measures to
In 1661 Newton was admitted to Trinity prevent counterfeiting. He was knighted in
College, Cambridge. When bubonic plague 1705. In 1703 he was elected President of the
broke out in London and closed the university Royal Society and was re-elected each year
in 1665 he went home to Lincolnshire. During until he died in 1727 at the age of 85.
the next two years he began the work which
Questions
would lead to his revolutionary advances in
mathematics, optics, physics and astronomy. He 1. Why did Newton leave Cambridge in
returned to Cambridge when it reopened after 1665? Bubonic plague broke out and the university closed.
the plague and was elected to a major 2. Who else claimed to have discovered
fellowship in 1668 at the age of only 26. He differential calculus? Leibniz
published his theory of differential calculus in 3. What did Newton describe and apply in
1671. However, the German philosopher the Principia? The laws of mechanics and gravitation
Leibniz had been working in the same area and 4. Name two of Newton’s achievements as
a bitter quarrel developed as to who had made Master of the Royal Mint.
He reformed the coinage and introduced measures
to prevent counterfeiting.
364 M a t h s Q u e s t M a t h s C Ye a r 1 1 f o r Q u e e n s l a n d

Newton’s First Law of Motion


Historically, Newton’s First Law of Motion had its origins in some of the work done by
Galileo. The law concerns itself with the motion of a particle when the resultant force R
acting on it is zero. It was a commonly held belief that in order to have motion, an ˜
unbalanced force was required. This belief seemed quite natural: clearly, carts need to
be pulled by horses in order to move and that balls rolling across a lawn come to rest.
However, experiments showed that the force of friction could be used in both cases to
explain the apparent necessity of an applied force to maintain constant motion. It was
Newton who stated that in the absence of an unbalanced force (that is, R = 0) acting on
a body, the body would continue in a state of uniform motion. The phrase˜ ‘uniform
motion’ is used to describe the motion of a body with zero acceleration; that is, the
velocity of the body is a constant.
Today we would write this as:
If R = 0 (the resultant force acting on the body is zero)
˜
then v = c (the velocity of the body is a constant)
and thus a = 0 (the acceleration of the body is zero).
If the resultant force acting on a body is zero, then its acceleration is zero.
It is important to recognise that the inference above is equally valid in reverse. That is,
If a = 0 (the acceleration of the body is zero)
then v = c (the velocity of the body is a constant)
and thus R = 0 (the resultant force acting on the body is zero).
˜
When the net or resultant force acting on an object is equal to zero, it is commonly
said that the body is ‘in equilibrium’. Such an object moves at a constant velocity.
These types of situation belong to a class of problems called statics.
To solve dynamics and related kinematics problems there is a clear strategy.
1. Read the question carefully.
2. Draw a clear diagram that contains all the information.
3. Superimpose arrows depicting vectors which act on the body in question. This
is called a force vector diagram.
4. Find the resultant force vector that is the sum of all forces acting on the body.

Mass and weight


We know from experience that some objects are harder to push than others; that is, it is
more difficult to modify their motion. We call this property of matter inertial mass. The
SI (Système International d’Unités) unit for mass is the kilogram. It is harder to stop or
accelerate a large truck than a small car, the truck having the greater mass.
Weight is a vector quantity. It is a force, equal to the product of the acceleration
due to gravity and the mass on which it is acting. We often refer to ‘the weight of the
object’ but the object does not possess weight as an intrinsic quantity; it does, how-
ever, possess mass. The SI unit for weight, since weight is a force, is the newton.
Suppose that an astronaut is somewhere in space where there is a zero gravitational
field (g = 0 N/kg); he may be 70 kg, but he is said to be ‘weightless’. The gravi-
tational field strength g is a measure of the magnitude of the force of gravity acting
on a unit mass. Consequently it has the dimensions of force per unit mass. In SI units
this is the newton per kilogram (N/kg). The same 70 kg man on the surface of the
Earth where the vector g has a magnitude of 9.8 N/kg down would have a weight of
˜
C h a p t e r 8 Ve c t o r a p p l i c a t i o n s 365
686 N or 70 g N downward. That is, the weight W of a mass m in a gravitational field
g is given by˜ the equation: ˜
˜ W = mg
Note: The value for g of 9.8 m/s down˜ is used
2 ˜ universally in examples and problems
˜
in this textbook. Sometimes the unit for g is quoted as N/kg. This is the acceleration
˜
due to gravity. For a mass of 1 kg, it becomes equal to the weight force when all other
forces are ignored.

Resolving a force into its components


In many cases the size of a force acting in a particular direction needs to be calculated.
For example, when a jet aircraft is taking off from a runway the engines provide a thrust.
Part of the thrust assists in lifting the plane up into the air (the vertical component) and
the other component exerts a horizontal push making the plane move forward. In the
diagram at right, the combined thrust of the jet’s engines is rep-
resented by a single vector F acting on a particle representing j
˜ ~
the aircraft. ~i
If F is the applied force acting on the plane due to the action
~F
of the˜ engines then we can write the force vector as the sum of ~Fy
θ
two vectors F x and F y which are parallel to the unit vectors i
˜ ˜ ˜ Aircraft ~Fx
and j respectively.
˜
Thus F = Fx + Fy
˜ ˜ ˜
F = Fxi + Fy j
˜ ˜ ˜ ˜ ˜
where F x = F cos θ
˜ ˜
and F y = F sin θ.
˜ ˜
The quantities Fx and Fy are referred to as the components of the vector force F in
the i and j directions respectively. ˜
˜ ˜
A force F can be resolved into perpendicular components:
˜
F = F cos θ i + F sin θ j
˜ ˜ ˜
where F = F and θ is the angle between F and i .
˜ ˜ ˜
For example, if the combined thrust of the engines is 2 × 106 N and the plane during
take-off had a 15° elevation angle then the vertical component would be:
Fy = F sin θ
˜
= 2 × 106 × sin 15°
≈ 5.18 × 105 N
while the horizontal component would be:
Fx = F cos θ
˜
= 2 × 106 × cos 15°
≈ 1.93 × 106 N
or F ≈ (1.93 × 106) i + (5.18 × 105) j
˜ ˜ ˜
Note: The i and j components
are usually ˜ ˜
horizontal and
vertical components respectively
but they can be rotated to suit a
particular problem.
366 M a t h s Q u e s t M a t h s C Ye a r 1 1 f o r Q u e e n s l a n d

WORKED Example 4
A water skier is being pulled along by rope attached to a speed boat across a horizontal lake.
The rope makes an angle of 5° to the horizontal and exerts a force of 6000 N on the skier.
a Calculate the horizontal component of the force exerted on the skier by the rope.
b Calculate the vertical component of the force exerted on the skier by the rope.
The skier is moving with a constant velocity.
c Calculate the size of the horizontal resistance forces on the skier.

THINK WRITE
1 The skier has four forces acting on him: the weight ~N
force, W , acting vertically downwards; the normal
˜ N , of the water on the skier, acting
reaction, ~T
˜
vertically upwards; the tension force, T , in the ~F 5°
rope, acting 5° to the horizontal; and the˜ resistance
forces, F , acting horizontally against the direction
˜
of motion. W
~
2 Draw a force vector diagram showing the forces
acting on the skier.
a Evaluate the magnitude of the horizontal a TH = 6000 cos 5°
component of T , TH. ≈ 5997 N
˜
b Evaluate the magnitude of the vertical b TV = 6000 sin 5°
component of T , TV. ≈ 523 N
˜
c 1 Constant velocity means the acceleration is zero c Since acceleration = 0,
and in turn the resultant of the horizontal forces
is zero.
2 The magnitude of the horizontal resistance F = TH = 5997 N
forces, F, is equal to the horizontal component of
the tension force, T .
˜
C h a p t e r 8 Ve c t o r a p p l i c a t i o n s 367
WORKED Example 5
In a science laboratory a 1.0 kg mass is suspended by two taut String 2
strings as shown at right. The tension forces in string 1 and String 1 j
string 2 are T 1 and T 2 respectively. ~
a Draw a force˜ ˜ diagram showing all three forces
vector 42°
~i
which act on the 1.0 kg mass.
1.0-kg mass
b By resolving vectors into i and j components find the
magnitudes of T 1 and T 2˜ respectively.
˜
˜ ˜
THINK WRITE
a 1 The mass has three forces acting on it: a T
~2
the weight force vertically downwards
and the two tension forces. One tension j 42° T1
~
force acts horizontally; the second acts ~
at an angle of 42° to the horizontal. ~i
W
~ = ~g
2 Draw the force vector diagram.

b 1 The weight vector can be written as −g j . b W = –g j


˜ ˜ ˜
2 The first tensile force can be written T 1 = T1 i
as T1 i . ˜ ˜
˜
3 The second tensile force can be resolved as T 2 = –T2 cos 42° i + T2 sin 42° j
T 2 = −T2 cos 42° i + T2 sin 42° j . ˜ ˜ ˜
˜
Express ˜ force, R , as˜ the
the resultant
4
˜ R = (T1 − T2 cos 42°) i +
sum of the three forces. ˜ ˜
(T2 sin 42° − g) j
˜
5 Set the sum of the three vectors to zero But R = 0 i + 0 j .
in accordance with Newton’s First Law ˜ ˜ ˜
of Motion.
6 Set the i component of R to zero and T1 − T2 cos 42° = 0 [1]
˜
call it equation [1]. ˜
7 Set the j component of R to zero and T2 sin 42° − g = 0 [2]
˜
call it equation [2]. ˜
g
8 Solve equation [2] for T2. T2 = -----------------
sin 42°
≈ 14.6 N
9 Solve for T1 by substituting T2 into T1 = T2 cos 42°
equation [1]. ≈ 10.9 N
Note: Part b could also be solved by j
~ W
drawing a triangle of forces and solving W = g = T1 tan 42° ~ = ~g ~T2
using trigonometry. g ~i 42°
T1 = -----------------
tan 42° ~T1
≈ 10.9 N
and W = g = T2 sin 42°
g
T2 = -----------------
sin 42°
≈ 14.6 N (as in part b above).
368 M a t h s Q u e s t M a t h s C Ye a r 1 1 f o r Q u e e n s l a n d

WORKED Example 6
A car of mass 800 kg is parked in a street which has an angle of j
~
elevation of 15°. The i direction is parallel down the street and the j
˜
direction is perpendicular to the street. ˜
The car is subject to three forces, namely its weight, W , the normal ~i 15°
˜
contact force, N , of the road acting on the car and the applied force
˜ is actually a static friction force) F .
of the brake (this
a Draw a vector diagram indicating the three forces, ˜ W , N and F , acting on the car,
taking the car as a particle. ˜ ˜ ˜
b What is the magnitude of the resultant force R ?
c Resolve the weight, W , into its components and˜ express it as a vector using i – j notation.
d Calculate the magnitude˜ of N , the normal contact force. ˜ ˜
˜
e Calculate the magnitude of the applied force of the brake F .
˜

THINK WRITE
a 1 A stationary car parked on a street will have a vertical a N
j ~
weight force, a normal contact force and a static frictional ~
force resisting its sliding or rolling down the street. ~F
2 Draw the force vector diagram. ~i 15°
W
~
b 1 The car is in equilibrium since it is stationary. b
2 Apply Newton’s First Law of Motion: the resultant R =0
force, R , must be zero. ˜
˜
Therefore, the magnitude of the resultant force, R, is zero. R=0N
3

c 1 Draw a diagram showing the resolution of the weight, c j


~
W , into components parallel to i and j .
˜ ˜ ˜ 15°
~i 15°
Wy
W
~ Wx
2 The component of W parallel to i , Wx , is W sin 15°.
˜ ˜
C h a p t e r 8 Ve c t o r a p p l i c a t i o n s 369
THINK WRITE
3 Substitute W = 800g and evaluate. Wx = 800g sin 15°
≈ 207g

4 The component of W parallel to j , W y , is W cos 15°. W y = W cos 15°


˜
5 Substitute W = 800g and evaluate. = −800g cos 15°
≈ –773g

6 Express W in vector notation. W = Wx i + Wy j


˜ ˜ ˜ ˜ j
W = 207g i − 773g
˜ ˜ ˜
d 1 The component of the net force parallel to the j d (N − 773g) j = 0 j
vector is zero. ˜ ˜ ˜

2 Solve for the magnitude of the normal N. N = 773g

e 1 The component of the net force parallel to the i e (207g − F) i = 0 i


vector is zero. ˜ ˜ ˜

2 Solve for the magnitude of the applied force of the F = 207g


brake, F.

Note: In general, when an object of mass m and hence weight mg is on an inclined


˜ components:
plane with incline angle θ, the weight vector can be resolved into two
1. one, of magnitude mg sin θ , acting down the plane
2. one, of magnitude mg cos θ, acting perpendicular to the plane. This is opposed by
the normal contact force, N .
˜
Friction N
A body resting on a table is acted on by ˜
a number of forces. As we have discussed Friction
the weight W acts downwards and is given G
˜ ˜
by the formula

W = mass × g . W
˜ ˜ ˜
The normal reaction, N , is the force exerted by the table on the body which opposes,
and just balances, the ˜weight. Suppose the body is further acted on by a horizontal
force G . If the table is smooth, there is no friction and the mass will move to the right.
˜
If the table is not smooth a frictional force will oppose the motion. The frictional force
depends on the roughness of the surface and the normal reaction N .
˜
Friction = µ × normal reaction, where µ is the coefficient of friction.
F =µ× N
˜ ˜
Note that this formula gives the maximum value for friction. If, in the diagram, G is
less than µ N , then the frictional force will just balance the force G . ˜
˜ ˜
370 M a t h s Q u e s t M a t h s C Ye a r 1 1 f o r Q u e e n s l a n d

WORKED Example 7 N
A body of mass 4 kg, at rest on a table, is acted on by a ˜
horizontal force, P , as shown in the figure at right.
˜
If the body just begins to move when P is 12 N, calculate
Friction
P
˜
the coefficient of friction between the body and the table. ˜

THINK WRITE W
˜
1 The weight of the body is m × g. W=m×g
= 4 × 9.8
= 39.2 N
2 The normal force, N, must balance the N=W
weight. = 39.2 N
3 The friction will oppose P and its F=µ×N
maximum value is µ × N. 12 = µ × 39.2
µ = 0.3

WORKED Example 8
A body of mass 4 kg rests on a plane inclined at an angle of Friction N
30° to the horizontal. It is just prevented from moving by ˜
friction. Calculate the:
a weight W
˜
b normal reaction N
c frictional force F ˜ W 30º
˜
d coefficient of friction µ. ˜

THINK WRITE
a W=m×g a W =m×g
m = 4 kg, g = 9.8 m/s2. ˜ = 4 × 9.8
The units are newtons. = 39.2 N
b First resolve each force into its b N = Nj.
components. W˜ = 39.2
˜ cos 60 i + −39.2 sin 60 j
Take the i direction to be down the plane ˜ j
˜ = 19.6 i + −33.9 ˜
and the j direction to be perpendicular to F = −µN i˜ ˜
the plane. ˜ ˜
If a body is at rest the vector sum of F+N+W=0
forces is 0. −F i + N j + 19.6 i + −33.9 j = 0
˜ ˜
If ai + b j + ci + d j = 0 then The i and˜ j components add˜to 0
a = ˜−c and
˜ b =˜ −d. ˜ N = 33.9 N
c c F = 19.6 N
d Coefficient of friction µ can be calculated d F = µN
using F = µN. 19.6 = µ × 33.9
µ = 0.58
C h a p t e r 8 Ve c t o r a p p l i c a t i o n s 371
remember
remember
~
j
1. Components of a force:
A force F can be resolved into perpendicular components: ~i
˜ ~F
F = Fx + Fy
˜ ˜ ˜ ~Fy
= F cos θ i + F sin θ j θ
˜ ˜ between F and i . Aircraft ~Fx
where F = | F | and θ is the angle
˜ ˜ ˜
2. Newton’s First Law of Motion is used for equilibrium or statics problems:
R = 0 (the resultant force is zero)
˜
a = 0 (the acceleration is zero)
v = c (the velocity is a constant).
3. The weight W (in N) of a mass m (in kg) in a gravitational field g is given by
W = m g where ˜ the value commonly used for g is 9.8 m/s2 (or 9.8˜ N/kg).
˜ ˜
4. Friction = µ × normal reaction, where µ is the coefficient of friction
F =µ× N
˜ ˜
SLE 4: Use resolution of vectors to consider the equilibrium of a body subject
to a number of coplanar forces acting at a point.

8B Newton’s First Law of Motion


Note: Use g = 9.8 m/s2 down for all problems involving weight.
˜
WORKED 1 A girl is pulling along her baby brother in a cart attached to a rope. The cart is on a
Example
4
horizontal path. The rope makes an angle of 20° to the horizontal and exerts a force on
the cart of magnitude 25 N.
a Calculate the horizontal component of the force exerted on the cart by the rope. 23.5 N
b Calculate the vertical component of the force exerted on the cart by the rope. 8.6 N
The cart moves along at a constant velocity; that is, the acceleration of the cart is zero.
c Calculate the size of the horizontal friction force acting on the cart. 23.5 N

2 multiple choice
A child of mass 40 kg is held on a ‘swinging rope’ at an angle of 25° to the vertical by
a horizontal force of 300 N. If T is the tension force of the rope acting on the child, then:
a the force vector diagram which˜ best represents this situation is:
A B C 25°
65° ~T 25° ~T ~T
300 N 300 N 300 N

40g N 40g N 40g N


D E
25° ~T 25° ~T
300 N 40g N

40g N 300 N
b the horizontal component of T is:
A 40 g N B 300 N ˜ C 127 N D 272 N E 200 N
372 M a t h s Q u e s t M a t h s C Ye a r 1 1 f o r Q u e e n s l a n d

c the vertical component of T is:


˜
A 300 N B 127 N C 40 g N D 200 N E 272 N
d the magnitude of T is nearest to:
˜
A 494 N B 440 N C 477 N D 92 N E 300 N

3 A ship of mass m is being pulled by two tugboats. It glides Tugboat


Ship 10°
through the water at constant velocity. The angle between the
two ropes connecting the two tugboats to the front of the ship 10°
is 20° and they each support a tension of magnitude 20 000 N. Tugboat
a Drag force
due to Tugboat a Draw a force vector diagram showing all three coplanar forces which act on the ship.
T
F
water ~ b What is the magnitude of the resultant force acting on the ship? (Be careful; use
~ Ship T
~ Newton’s First Law of Motion.) 0 N 39 392.3 N
Tugboat
c Calculate the magnitude of the force of friction due to the water acting on the ship.
WORKED 4 A swing chair of mass 8 kg is suspended
Example
5
by two taut ropes as shown at right. The
tension forces in rope 1 and rope 2 are
a
T 1 and T 2 respectively. 60° 30°
a˜ Draw˜ a force vector diagram showing
60° 30°
~T2 ~T1
j all three forces which act on the swing. Rope 1 Rope 2
~
b By resolving vectors into i – j com-
~i 8g N
~ ˜ ˜
ponents find the exact magnitudes of
T 1 and T 2 . T1 = 4g N; T2 = 4 3 g N
˜ ˜
5 A speaker in an auditorium has a mass of 50 kg and is suspended from the ceiling by
two 4.0-m ropes. The ropes are attached to the ceiling at points A and B whose separ-
a ation is 6.0 m as shown at right.
~Tleft ~Tright a Draw a force vector diagram showing all
Speaker forces which act on the speaker. 6.0 m
b Calculate the angle that each rope makes with A B
W
~ the ceiling. 41.4°
c Determine the vertical component of the ten- 4.0 m
d –278 i + 245 j N;
sion force in each rope. 245 N j
˜ ˜ d Give the vectors for the tensions in the left and ~
278 i + 245 j N
˜ ˜ right rope respectively using i – j notation. ~i
e Calculate the magnitude of the ˜ ˜ tension in
each rope. 370.6 N
f The speaker is to be raised by increasing the
separation between the points A and B, but the
ropes will break if the tension exceeds 4000 N.
Find the maximum possible separation
between A and B; that is, when the tensions in
the ropes are equal to 4000 N. 8.0 m
6 Sam earns some extra pocket money by mowing his neighbour’s
front lawn. When he pushes the lawnmower at a constant velocity he
applies a force of 120 N down the shaft of the mower which is
angled at 40° to the vertical. The lawnmower has a mass of 40 kg. 120 N
a ~N a Draw a force vector diagram illustrating all four forces
40°
acting on the lawnmower. (Treat the lawnmower as a
~A = 120 N at ~F
40° to particle.)
vertical
W
~ = 40g
~
c R = ( F friction – 77.1 )i + ( N – 483.9 ) j = 0
˜
with all forces in N.
˜ ˜
Av = 91.9 N down
C h a p t e r 8 Ve c t o r a p p l i c a t i o n s 373
AH = 77.1 N left
b Calculate both the vertical and horizontal components of the force that Sam
applies to the lawnmower.
c Using i – j notation, write a vector equation for the resultant force, R , acting on
˜ ˜
the lawnmower in terms of the four forces acting on the lawnmower. ˜
d Determine the magnitude of the force of friction acting on the lawnmower as it
moves across the lawn at constant velocity. 77.1 N
e Determine the magnitude of the normal contact force. 483.9 N
WORKED 7 A 1.5 kg mass is placed on a smooth inclined plane angled at j
Example ~
6
30° to the horizontal. To stop it from sliding down the plane ~i
a string is attached to the upper side as shown at right.
The unit vectors i and j are also shown. 30°
a ~N ˜ ˜
a Draw a force vector diagram showing the forces which act on the mass. Label the
~T
Mass forces N for the normal contact force, T for the tension force and W for the
˜
force arising from the effect of gravity. ˜ ˜
W
~
b What is the magnitude of the resultant force, R ? 0i˜ + 0 j
˜ ˜
c Determine the weight vector, W , using i – j notation. –7.4 i – 12.7 j N
˜ ˜ ˜
d Determine the tension force, T , using i – j ˜ notation. 7.4 N
˜
˜ ˜ ˜
e Determine the magnitude of the normal contact force, N. 12.7 N
8 A 1.5 kg mass is placed on a smooth inclined plane angled at j
30° to the horizontal. To stop it from sliding down the plane ~
Horizontal ~i
a horizontal force is applied to the mass as shown at right. force, ~
H
a N
The unit vectors i and j are also shown.
˜
a Draw a force vector ˜diagram showing the forces which act
~
30°
Mass H
~ on the mass. Label the forces N for the normal contact
force, H for the horizontal force ˜ and W for the force due to gravity.
˜ ˜
b Determine the weight vector, W , using i – j notation. – 7.4i˜ – 12.7 j Ν
W
~
˜ ˜ ˜
c Determine the horizontal force, H , using i˜– j notation. 7.4i – 4.3 j N
˜ ˜ ˜ ˜
d Determine the normal contact force, N , using˜ i – j notation. 17 j N
˜ ˜ ˜ ˜
W 3W
e Show that H = -------˜ - = ---------------
˜ .
˜ 3 3
WORKED 9 A box containing books has a mass of 40 kg. It requires a force of 300 N to move the
Example
7
box across the floor. Calculate the coefficient of friction between the box and the floor. 0.77
10 The maximum deceleration the BMV Tycoon can achieve under braking is 4 ms−2. If
this vehicle has a mass of 1900 kg, calculate:
a the frictional force on the car 7600 N
b the normal reaction 18 620 N
c the coefficient of friction between the tyres and the road. 0.41
11 A force of 200 N is needed to keep a log of mass 300 kg sliding along a horizontal
path. Calculate the coefficient of friction between the log and the path. 0.07
WORKED 12 A body of mass 6 kg rests on a plane inclined at an angle of 40° to the horizontal. It
Example
8
is just prevented from moving by friction. Calculate the:
a weight 58.8 N
b normal reaction 45.04 N
c frictional force 37.8 N
d coefficient of friction. 0.84
374 M a t h s Q u e s t M a t h s C Ye a r 1 1 f o r Q u e e n s l a n d

13 A book lies on a horizontal table. One end of the table is raised until, at an angle of
35°, the book starts to slide. Calculate the coefficient of friction between the book and
the table. 0.7
eBook plus
14 An object of mass 2 kg rests on a plane inclined at an angle of 40° to the horizontal.
Digital doc: If the coefficient of friction between the object and the plane is 0.2, calculate the
WorkSHEET 8.1 resultant force down the plane. 9.6 N

Momentum
Kirsten is an investigator with the Traffic Accident Squad and she is called to investi-
gate the collision between a large four-wheel-drive vehicle and a small hatchback.
Luckily no one was hurt in the accident but there was considerable damage to the
vehicles. An eyewitness report suggests that the small vehicle was speeding.
The diagram below indicates the direction of travel of the vehicles.
A indicates the direction of motion of the
small car before the accident.
B indicates the direction of motion of the
large car. A
C indicates the direction of motion of both 45º
vehicles after impact.
B 25º
What conclusion can Kirsten make con- C
cerning the speeds of the vehicles before
impact? By investigating the accident
scene Kirsten will be able to determine
whether any of the cars were speeding. In particular she will look for any skid marks
and look at the motion of the vehicles after collision. A key mathematical concept that
will help her to understand the events leading up to the accident is momentum.
Definition of momentum
One of the properties of a moving body is momentum. The momentum of a moving
body is given by:
Momentum = mass ¥ velocity
Because velocity is a vector quantity, momentum is also a vector quantity; that is, it has
both magnitude and direction.
The standard units of momentum are N s (newton seconds) where mass is in kg and
velocity is in m/s. (Alternative units for momentum are kg m/s.)

WORKED Example 9
Calculate the momentum of a car of mass 2.5 tonnes travelling north at 20 m/s.
THINK WRITE
1 Use standard units. Convert tonnes to Mass = 2.5 t = 2500 kg
kilograms.
2 Use the formula for calculating the Momentum = mass × velocity
magnitude of momentum. = 2500 × 20
= 50 000 N s
3 Momentum is a vector quantity so Momentum = 50 000 N s in a northerly
include direction as well as magnitude. direction.
C h a p t e r 8 Ve c t o r a p p l i c a t i o n s 375
WORKED Example 10
A tennis ball of mass 80 g is hit at a wall with a speed of 50 m/s. It rebounds with a speed
of 40 m/s. What is the change in momentum?

THINK WRITE
1 State the required quantities in Mass = 80 g = 0.08 kg
standard units. Mass should be Initial velocity = 50 m/s towards the wall
in kg and velocity in m/s. Final velocity = 40 m/s away from the wall
2 Calculate the initial Initial momentum = mass × initial velocity
momentum. = 0.08 × 50
= 4 N s towards the wall
3 Calculate the final momentum. Final momentum = mass × final velocity
= 0.08 × 40
= 3.2 N s away from the wall
4 Calculate the change in Change in momentum
momentum by subtracting the = final momentum − initial momentum
initial momentum from the = (3.2 N s away from the wall) − (4 N s towards
final momentum. the wall)
= (3.2 N s away from the wall) − (−4 N s away from
the wall)
= 7.2 N s away from the wall

Conservation of momentum
Momentum is a useful concept because it is conserved in collisions. That is, if two
bodies, A and B, collide their total momentum before collision is equal to their total
momentum after the collision.
Total momentum before collision = total momentum after collision

WORKED Example 11
A car weighing 1.9 tonnes is travelling east at 22 m/s. A second car travelling north and
weighing 2.4 tonnes passes through a stop sign at 14 m/s. It collides with the first car. If
after the collision the cars move off together, calculate their final speed and direction.
THINK WRITE
1 Momentum is conserved: the total
momentum before collision is equal to
the total momentum after collision.
2 First calculate the momentum of each Momentum of first car = 1900 × 22
vehicle before collision. Momentum of first car = 41 800 N s east
Momentum of second car = 2400 × 14
Momentum of second car = 33 600 N s north

Continued over page


376 M a t h s Q u e s t M a t h s C Ye a r 1 1 f o r Q u e e n s l a n d

THINK WRITE
3 Find the total momentum before the
collision by adding vectors.
x
33 600 N s
θ
41 800 N s
2 2
x = 33 600 + 41 800
= 53 630
33 600
tan θ = ----------------
41 800
θ = 38.8°
Total momentum before collision
= 53 630 N s at 38.8° north of east
4 Since momentum is conserved, state the Hence, total momentum after collision
total momentum after the collision. = 53 630 N s at 38.8° north of east
momentum
5 Calculate the final speed (magnitude of Final speed = ---------------------------
the final velocity) by rearranging the mass
formula for momentum. The mass of 53 630
Final speed = -----------------------------------
the cars as they move off together is the ( 1900 + 2400 )
sum of the individual masses. Final speed = 12.5 m/s
6 Write the final speed and direction of The final speed and direction of the vehicles as
the cars after the collision. they move off together is 12.5 m/s at 38.8°
north of east.

WORKED Example 12
Mass A, 2 kg, collides with mass B, 3 kg, moving as shown. 10 m/s
A
a Calculate the momentum of each body before the collision. 6 m/s
b Calculate the total momentum of the system before the collision.
B 30º
If, after the collision, mass A moves as shown, 8 m/s
c calculate the final speed and direction of mass B.
40º
A

B θ
x m/s
THINK WRITE
a Calculate the momentum of A and a Before the collision:
the momentum of B before the Momentum of A = 2 × 10
collision, using the formula = 20 N s, to the right
momentum = mass × velocity. Momentum of B = 3 × 6
= 18 N s, 30° left of the initial
= direction of A
C h a p t e r 8 Ve c t o r a p p l i c a t i o n s 377
THINK WRITE
b 1 Draw a vector diagram showing the b
situation before the collision. Add the T
˜ 18 N s
vectors for the momentum of A and α 30º
momentum of B (head to tail) to 20 N s
produce the vector for the total
momentum T .
˜
2 Use the cosine and sine rules to T 2 = 182 + 202 − 2 × 18 × 20 cos 150°
calculate T and α, the magnitude and = 1347.5
direction of the total momentum T = 36.7 N s
before the collision. sin α sin 150°
------------ = -------------------
18 36.7
sin α = 0.24523
α = 14.2°
Total momentum before the collision is
36.7 N s, 14.2° left of the initial direction of A.
c 1 Calculate the momentum of A after c After the collision:
the collision. Let B represent the Momentum of A = 2 × 8
momentum of B after the collision. = 16 N s, 40° left of the
initial direction of A
Let B represent the momentum of B after the
collision.
2 Since the momentum is conserved, state Total momentum after the collision =
the total momentum after the collision. 36.7 N s, 14.2° left of the initial direction of A
That is, momentum before the collision =
momentum after the collision.
3 Draw a vector diagram showing the θ
B Ns
situation after the collision. Add the β
vectors for the momentum of A and 16 N s
40º 36.7 N s
momentum of B (head to tail) to produce ω 14.2º
the vector for the total momentum.
4 Calculate B, the momentum of B2 = 162 + 36.72 − 2 × 16 × 36.7 cos ω
mass B, using the cosine rule. = 162 + 36.72 − 2 × 16 × 36.7 cos 25.8°
Note that the angle marked ω is B = 23.4 N s
40° − 14.2° = 25.8°.
5 The speed of B can be calculated Momentum = mass × velocity
from the momentum by using 23.4 = 3 × x
momentum = mass × velocity. x = 7.8 m/s
sin β sin 25.8°
6 To calculate θ, note that as marked in ------------ = ---------------------
the diagram β = θ +14.2°. First 16 23.4
sin β = 0.2976
calculate β using the sine rule.
β = 17.3°
Therefore θ = 3.1°.
7 State the final speed and direction of The final speed and direction of B is 7.8 m/s
B after the collision. in a direction 3° right of the initial path of A
as shown.
378 M a t h s Q u e s t M a t h s C Ye a r 1 1 f o r Q u e e n s l a n d

Conservation of momentum using


i and j notation
˜ ˜
Consider the problem posed in Worked example 12. Recalculate the answers by
expressing the momentum vectors in i and j notation.
˜ ˜

Using our understanding of momentum we return to the scene of the accident with
Kirsten (see page 374).
The driver of the four-wheel-drive B
vehicle, B, says she was travelling at 25º 45º
70 km/h (19.4 m/s). Further, the A j
~
masses of both vehicles are known: the C
mass of vehicle A is 1640 kg, the mass ~i
of vehicle B is 2060 kg.
Momentum is conserved. That is, in vector terms,
momentum of A + momentum of B = momentum of C.
Momentum of B = mass × velocity
= 2060 kg × (19.4 i + 0 j )
˜ ˜
= 39 964 i + 0 j .
˜ ˜
Let the momentum of A before impact be A and let the momentum of both vehicles
after collision be T.
Then since momentum before = momentum after
39 964 i + A cos 45° i − A sin 45° j = T cos 25° i − T sin 25° j
˜ ˜ ˜ ˜ ˜
Therefore
39 964 + A cos 45° = T cos 25° [1]
and
A sin 45° = T sin 25° [2]
Substituting [2] in [1] (noting that cos 45° = sin 45°)
39 964 + T sin 25° = T cos 25°
T cos 25° − T sin 25° = 39 964
T(cos 25° − sin 25°) = 39 964
39 964
T = ----------------------------------------------
( cos 25° – sin 25° )
= 82 623 N s
Substituting in [2] gives
A sin 45° = 82 623 sin 25°
A = 49 381.5 N s
Thus the speed of A before collision = momentum of A ÷ mass of A
= 49 381.5 ÷ 1640
= 30.1 m/s
= 108.4 km/h
Kirsten concludes that car A was speeding.
C h a p t e r 8 Ve c t o r a p p l i c a t i o n s 379
remember
remember
1. The momentum of a moving body is given by the equation
momentum = mass × velocity
2. Momentum is a vector quantity — it has both magnitude and direction. The
units of momentum are N s or kg m/s.
3. Momentum is conserved in collisions:
total momentum before collision = total momentum after collision.

8C Momentum
WORKED 1 Calculate the momentum of the following objects whose mass and velocity are given:
Example
9 a mass = 2.5 kg, velocity = 16 m/s east 40 N s east
b mass = 4 kg, velocity = 150 m/s south 600 N s south
c mass = 250 g, velocity = 30 m/s north 7.5 N s north
d mass = 2.5 tonnes, velocity = 13 m/s west 32 500 N s west
e mass = 88 kg, velocity = 40 km/h north 978 N s north
f mass = 3.4 tonnes, velocity = 120 km/h north 113 333 N s north
2 Calculate the speed of the following objects with momentum and mass given:
a momentum = 2000 N s, mass = 25 kg 80 m/s
b momentum = 3000 N s, mass = 80 g 37 500 m/s
c momentum = 42 000 N s, mass = 2.1 tonnes 20 m/s
3 Calculate the mass of the following objects with momentum and speed given:
a momentum = 2500 N s, speed = 50 m/s 50 kg
b momentum = 22 000 N s, speed = 40 km/h 1980 kg
4 Give the total momentum of the following systems.
a object A: mass = 3 kg, velocity = 10 m/s east
object B: mass = 2 kg, velocity = 4 m/s east 38 N s east
b object A: mass = 3 kg, velocity = 10 m/s east
object B: mass = 2 kg, velocity = 4 m/s west 22 N s east
c object A: mass = 3 kg, velocity = 10 m/s east
object B: mass = 2 kg, velocity = 4 m/s north 31 N s 14.9° N of E
d object A: mass = 3 kg, velocity = 10 m/s east
object B: mass = 2 kg, velocity = 4 m/s 30° north of east 37.1 N s 6.2° N of E
e object A: mass = 3 kg, velocity = 10 m/s east
object B: mass = 2 kg, velocity = 4 m/s north of west 25 N s 13° N of E
WORKED 5 At the carnival, a dodgem car with a mass of 300 kg with its driver travels at 6 m/s
Example
10
and crashes into a wall. If it rebounds at 3 m/s calculate the change of momentum. 2700 N s
6 A cyclist (plus bicycle, pack, clothes, etc.) has a combined mass of 100 kg. He is trav-
elling at 10 m/s.
a Find his momentum. 1000 kg m/s
b If the cyclist slows down to 5 m/s, what is the change in his momentum? −500 kg m/s
380 M a t h s Q u e s t M a t h s C Ye a r 1 1 f o r Q u e e n s l a n d

7 Find the change of momentum for:


a a mass of 10 kg which slows from 12 m/s to 8 m/s −40 kg m/s
b a mass of 8 kg which accelerates from 7 m/s to 13 m/s 48 kg m/s
c a car of mass 0.95 tonne which increases speed from 44 km/h to 80 km/h. 9500 kg m/s
d a horse of mass 400 kg which slows from a gallop of 50 km/h to a trotting speed
of 18 km/h. −3556 kg m/s
WORKED 8 Laurie is about to collide, at 90°, with Alan. After the collision the two players move
Example
xample
11
as one unit before coming to rest. Laurie has a mass of 85 kg and Alan has a mass of
100 kg.
Calculate the common speed and direction of the two players immediately after the
collision. 5.3 m/s at 44° to Alan’s current direction.

Laurie 8 m/s

Alan 7 m/s

WORKED 9 Two dodgem cars collide. The mass


Example
xample
12
of car A and its driver is 320 kg
while the mass of car B and its
driver is 280 kg.
Calculate the final speed and 1 8
direction of car B in each of the
following situations.

Before collision After collision


Car A Car B Car A Car B
9 a 6.6 m/s N
b 6 m/s N
a 8 m/s north 2 m/s north 4 m/s north ?
c 3.4 m/s S
b 8 m/s north 2 m/s south 1 m/s north ?
c 2 m/s north 8 m/s south 2 m/s south ?
C h a p t e r 8 Ve c t o r a p p l i c a t i o n s 381
10 Using the data from question 8, assume that Laurie does not tackle Alan at right
angles but rather at 60° as shown. Calculate the common velocity of the two players
after the collision. 3.7 m/s at 58° to Alan’s current direction.
Laurie
8 m/s

Alan 60º
7 m/s

11 Two billiard balls, A and B, each of mass 100 g, collide and then move off. Use the
principle of conservation of momentum to calculate the unknown values in the table
below.

(The direction of motion is specified by giving the angle with the positive x-direction.)

Before collision After collision

A B A B

v m/s θ v m/s θ v m/s θ v m/s θ

2.2 at 47.9° a 4 30° 0 — 2 10°

5.9 at 71.4° b 4 60° 2 0° 3 −45°

1.67 at 65.4° c 3 30° 2 150° 1 80°

2.4 at 234° d 3 30° 4 −120° 2 0°

12 The principle of momentum can be used to calculate the amount of ‘recoil’


experienced when a weapon is fired.
The momentum of a bullet and rifle before firing =
the momentum of the bullet and rifle after firing.
If a bullet has a mass of 25 g and is fired from a rifle at 180 m/s:
a calculate the momentum of the rifle immediately after firing 4.5 N s
b calculate the speed of the rifle immediately after firing if the rifle has a mass of
1.5 kg. 3 m/s

Collision momentum
See Solutions In the analysis of the accident scene, Kirsten concluded that the smaller vehicle
Manual. was speeding. Her analysis is based on the evidence of the person driving the
larger vehicle who claimed her speed was 70 km/h. What if this evidence was
inaccurate?
Imagine you are the accident investigator and have to write a report of the
accident. In your report consider alternative scenarios in which the driver of car B
was travelling at 70 km/h, at greater than 70 km/h and at less than 70 km/h.
382 M a t h s Q u e s t M a t h s C Ye a r 1 1 f o r Q u e e n s l a n d

Relative velocity SLE 3: Use addition and subtraction in life-related situations such as the
effect of current flow on a boat; consider the concept of relative velocity.

As you sit at your desk reading this paragraph it appears that you are not moving sig-
nificantly. While this may be apparent to someone in the room watching you, an
observer with a different point of reference may disagree entirely. As you sit in the
room you are moving around the Sun at about 100 000 km/h. Measurement of velocity
depends on the frame of reference.
Most people have experienced the sensation of movement while sitting in a
stationary train and observing a nearby train moving. The visual messages tell our brain
we are moving and, for a moment, we become disoriented.
Consider the situation of a boy rowing a boat across a swiftly flowing river. The boy
thinks he is rowing directly towards the bank opposite but to an observer on the shore
his velocity is different. The situation can be described using vectors.
Velocity of boy relative to river

Velocity of river
Velocity of boy

In general the relationship between relative velocities is


v a = v a rel b + v b
˜ ˜ ˜
where v a rel b is the velocity of a ( v a ) relative to the velocity of b ( v b ).
˜ ˜ ˜

WORKED Example 13
The pilot of a boat heads due north at a speed of 12 km/h with respect to the water. The
water moves at a velocity of 5 km/h in an easterly direction. Calculate the velocity of the
boat as seen by an observer on the shore.

THINK WRITE
1 The velocity of the boat relative to the v b rel w = 12 j
˜
water (vb rel w) is 12 m/s north. vw = 5 i ˜
The velocity of the water (vw) is 5 km/h ˜ ˜ vb 12
east.
We want to find the velocity of the boat, θ
vb. 5
2 Use the relative velocity formula. v b = v b rel w + v w
˜ ˜ ˜
= 12 j + 5 i
˜ ˜
2 2
3 Write the answer as a speed and | vb | = 12 + 5
direction. = 13
12
tan θ = ------
5
θ = 67.4°
An observer on the shore sees the boat moving
at 13 km/h at 67.4° north of east.
C h a p t e r 8 Ve c t o r a p p l i c a t i o n s 383
WORKED Example 14
A pilot is to fly a plane to a destination which is 450 km from
his present position in a direction N20°W. There is a wind from
the east at 22 km/h and the plane has an airspeed of 300 km/h.
In what direction should the plane head to reach the
destination?

THINK WRITE
1 Use the relative velocity formula and v a = v a rel b + v b
draw a diagram. ˜ ˜ ˜
22
The wind velocity is v b and the
˜ v
airspeed of the plane is a rel b .
˜
The speed of the plane, relative to the
300
ground is v a . va
˜
20º

2 Use the sine rule: 22


a b c 70º
------------ = ------------ = -------------
sin A sin B sin C

300
x

sin
----------x- = sin
70°
----------------
22 300
sin x = 0.0689
x = 3.95°
The plane should head 16.05° west of north or
N16.05°W.
384 M a t h s Q u e s t M a t h s C Ye a r 1 1 f o r Q u e e n s l a n d

remember
remember
The equation for determining the velocity of object a relative to object b is
v a = v a rel b + v b where v a rel b is the velocity of a ( v a ) relative to the velocity of
b˜ ( v b ).˜ ˜ ˜ ˜
˜

8D Relative velocity
WORKED 1 The pilot of a boat heads due east at a speed of 16 km/h with respect to the water. The
Example
xample
13 water moves south at a velocity of 4 km/h. Calculate the velocity of the boat as seen by
an observer on the shore. 16.5 km/h at 14° S of E

2 A plane with an airspeed of 300 km/h heads due north. A wind blows from the west at
30 km/h. What is the velocity of the plane relative to the ground? 301.5 km/h at 5.7° W of N

3 A rowing crew heads across river at 15 m/s. The crew heads in a direction 45° north of
west. At the same time the current flows at 3 m/s due north. What is the speed and
direction of the boat on the river? 17.3 m/s at 52.1° N of W

WORKED 4 The driver of a cross-river ferry wants to head to a pier directly across the stream, at
Example
xample
14 right angles to the bank. His boat can travel at 20 km/h and there is a current flowing
downstream at 6 km/h. At what angle to the bank should the driver head in order to
travel directly across the river? 72.5°

5 A passenger on a cruise ship walks at 6 km/h towards the stern (the back) of the ship.
The ship travels at 19 km/h in a north-westerly direction. What is the speed of the pas-
senger with respect to the water? 13 km/h

6 Copy and complete the following table.

va vb v a rel b
˜ ˜ ˜
a 40 m/s N 20 m/s E 44.7 m/s at 26.6° W of N

b 5 m/s N 15 m/s S 20 m/s N

c 25 m/s NE 20 m/s E 17.8 m/s at 7.6° W of N

d 4 m/s SE 32.95 m/s at 4.9° S of E 30 m/s W

e 3 m/s 30° S of E 5 m/s 25° W of N 7.7 m/s at 52° S of E

7 A cyclist rides north at 15 km/h and observes that the wind appears to come to him
from the north-east. On the return journey he rides at the same speed in the opposite
direction. Now the wind appears to be coming from the south-east. Calculate the true
speed and direction of the wind. 15 km/h W
C h a p t e r 8 Ve c t o r a p p l i c a t i o n s 385
8 A cyclist rides north at 15 km/h and observes the wind appears to come to him from the
north-east. On the return journey he rides at the same speed in the opposite direction.
Now the wind appears to be coming from 30° south of east. Calculate the true speed
and direction of the wind. 19.4 km/h 78° W of S
9 As a jogger runs along level ground at 9 km/h the rain appears to be heading
eBook plus
directly towards her at an angle of 10° with the vertical. When she turns around
Digital doc: and travels at the same speed in the opposite direction the rain still appears to be
WorkSHEET 8.2 coming directly towards her but now makes an angle of 5° with the vertical. Calcu-
late the speed and direction of the rain. 68.3 m/s at 2.5° with the vertical.

Using vectors in geometry SLE 6: Solve problems from geometry using vectors.

The rules for addition, subtraction and dot product of vectors can be used to prove
theorems and other general statements in geometry.

WORKED Example 15
In the drawing at right, ABC is a triangle. Point D is along the C
~r
line BC such that BD = 1--3- BC. The vectors q , r and t are as shown A
˜ ˜
in the diagram. Prove that: t = 1--3- (2 q + r ).˜ ~t
˜ ˜ ˜
q D
~
B

THINK WRITE
1 Express the line joining B to D as a vector v and the
line joining B to C as a vector s . ˜
˜
2 Redraw the diagram with v and s indicated. r C
˜ ˜ ~
A
t s
~ ~
q vD
~ ~
B
3 Express s as the vector sum of – q and r . s = –q+r
˜ ˜ ˜ ˜ ˜ ˜
4 Since D is one-third of the way from B to C, v = 1
--- s. v = 1--3- s
˜ 3 ˜ ˜ ˜
5 Express t as the vector sum of q and v . t = q+v
˜ ˜ ˜ ˜ ˜ ˜
6 Substitute v = 1--- s into the expression for t . t = q + 1--3- s
˜ 3˜ ˜ ˜ ˜ ˜
7 Substitute s = –q + r into t = q + 1--- s . t = q + 1--3- ( –q + r )
3 ˜
˜ ˜ ˜ ˜ ˜ ˜ ˜ ˜ ˜
8 Simplify by expanding the bracket. t = q – 1--3- q + 1--3- r
˜ ˜ ˜ ˜
9 Collect ‘like’ vectors. t = 2--3- q + 1--3- r
˜ ˜ ˜
10 ‘Factorise’ the vectors, by removing common scalar t = 1--3- ( 2q + r )
factors. The required expression for t is obtained. ˜ ˜ ˜
˜
386 M a t h s Q u e s t M a t h s C Ye a r 1 1 f o r Q u e e n s l a n d

WORKED Example 16
Consider the isosceles triangle at right, where AB = BC. Show, using B
the properties of vectors only, that line BD, drawn such that BD is
perpendicular to AC, divides AC in half, that is: AD = DC .

A C
D
THINK WRITE
1 Express AD as a vector, in terms of other
AD = AB + BD
vectors. (In this example, AD is the
vector joining A to D.)
2 Take the dot product of this expression
AD • AD = ( AB + BD ) • AD
with AD itself.
2 2
3 Simplify, and recall that AD • AD = AD . AD = AB • AD + BD • AD

4 Since BD is perpendicular to But BD • AD = 0 .


AD , BD • AD = 0 . so AD
2
= AB • AD

5 Express AB • AD using the definition of = AB AD cos θ


dot product.
6 Divide both sides by AD . AD = AB cos θ
where θ is the angle between AB and AD .
7 Express CD as a vector in terms of other
CD = CB + BD
vectors.
8 Take the dot product of this expression
CD • CD = ( CB + BD ) • CD
with CD itself.
2
9 Simplify. CD = CB • CD + BD • CD
10 Since BD and CD are perpendicular,
But BD • CD = 0 .
BC • CD = 0 .
2
11 Express CB • CD in dot product form. CD = CB • CD
= CB CD cos α
12 Divide both sides by CD . CD = CB cos α
where α is the angle between CB and CD .
13 Write down the respective expressions for AD = AB cos θ
AD and CD . CD = CB cos α
14 Since the triangle is isosceles, But AB = CB and θ = α.
AB = CB and θ = α.

15 It can be concluded that AD = CD . Therefore AD = CD .


C h a p t e r 8 Ve c t o r a p p l i c a t i o n s 387
WORKED Example 17
Use a vector method to show that the diagonals of a rectangle bisect each other.
THINK WRITE
1 Construct a rectangle ABCD. B C

~b E
A a D
~

2 Let the vector from A to D be a and Let AD = a


˜ ˜
the vector from A to B be b . Let AB = b
˜ ˜
3 Let E be the midpoint of diagonal AC. Let E be the midpoint of AC.

4 Find AC in terms of a and b . AC = a + b


˜ ˜ ˜ ˜
5 Find AE which is half of AC in terms AE = --12- ( a + b )
˜ ˜
of a and b .
˜ ˜
6 Determine BE in terms of a and b . BE = –b + 1--2- ( a + b )
˜ ˜ ˜ ˜ ˜
= --2- ( a – b )
1
˜ ˜
7 Determine ED in terms of a and b . ED = – 1--2- ( a + b ) + a
˜ ˜ ˜ ˜ ˜
= 1--2- ( a – b )
˜ ˜

8 Note that ED = BE . = BE

9 This means that BE and ED are So E is the midpoint of BD.


collinear and of equal length since they
are equal vectors.
10 State your conclusion. The diagonals of a rectangle bisect each other.

Three-dimensional non-zero vectors


See Solutions
If a , b and c are 3-dimensional non-zero vectors:
Manual. ˜ ˜ ˜
1 Show geometrically that:
a+b+c ≤ a + b + c
˜ ˜ ˜ ˜ ˜ ˜
2 Prove algebraically that:
a + b ≥ a+b
˜ ˜ ˜ ˜
3 Prove algebraically that:
a•b ≤ a b
˜ ˜ ˜ ˜
Under what circumstances are all of the above statements equal?
388 M a t h s Q u e s t M a t h s C Ye a r 1 1 f o r Q u e e n s l a n d

Vector geometry
See Solutions In any triangle ABC, the points D, E and F divide the lines BC, CA and AB
Manual. respectively in the ratio 1:2.

R F

E Q

P
C
D B
6
1 Show that the vector joining A to P is --- of the vector joining A to D.
7
3
2 Show that the vector joining R to P is --- of the vector joining A to D.
7

3 Can you calculate the ratio of the area of triangle QRP to the area of triangle
ABC?

remember
remember
In vector proofs, it is simpler to use pronumerals to represent vectors, but not
essential. Use as few vectors as necessary in completing a proof and apply the
rules for vector addition, subtraction and dot products as appropriate.

SLE 6: Solve problems from geometry using vectors: for example; prove that the
angle of a semicircle is a right angle; Pythagoras’ theorem; the concurrency of
(a) the medians and (b) the bisectors of the internal angles of a triangle.

8E Using vectors in geometry


C
WORKED 1 In the triangle ABC, point D is the midpoint of BC. ~r
Example
15
The vectors q , r and t are defined as indicated.
Show that: ˜ ˜ ˜ A ~t D
Check with
your teacher. t = 1
--- r

+ 1
--- q
2 q
˜ ˜ ~
B
2 ABC is a triangle with point D along the line BC such that:
1
BD = --- BC r C
n ~
A
The vectors q , r and t are as shown in the diagram. t
~
Prove that: ˜ ˜ ˜
D
1 (n – 1) q
t = --- r + ----------------q ~
˜ n˜ n ˜ B
C h a p t e r 8 Ve c t o r a p p l i c a t i o n s 389
3 Consider the triangle ABC. Point D is the midpoint of line B
AB and E is the midpoint of line BC.
Show, using vectors to represent various lines, that line D E
DE is parallel to AC and DE = 1--- AC. Check with your teacher.
2
A C
4 In the figure at right, D is the midpoint of CB and AD is per-
pendicular to CB. Let u be the vector joining D to A and v C
˜ ˜
the vector joining D to B.
Find, in terms of u and v only: D A
˜ ˜
a the vector, a , joining D to C − v˜
˜
b the vector, b , joining C to A u + v
˜ ˜ ˜
c the vector, c , joining B to A u˜ − v˜ B
˜
d the dot products, b • b and c • c . b.b = u2 + v2
˜ ˜ ˜ ˜ ˜ ˜ ˜ ˜
Hence, show that AC = AB. c .c = u2+ v2
˜ ˜ ˜ ˜ B
5–12 Check with 5 In the figure at right, a is the vector joining point A to point
your teacher. ˜
B, b joins B to C and c joins C to A.
˜ ˜ ~a ~b
Prove that a + b + c = 0 .
˜ ˜ ˜

A C
~c

WORKED 6 Consider the equilateral triangle at right. Show, using the B


Example
16
properties of vectors only, that line BD, drawn such that BD
is perpendicular to AC, divides AC in half, that is:
AD = DC .

A C
D

7 Show that the diagonals of a non-square rhombus intersect at right angles.


(Hint: Make a drawing; a rhombus is a parallelogram with all 4 sides of equal length.)

8 If the length of a vector, a , is given by a , show geometrically that for any two
vectors a and b : ˜ ˜
˜ ˜
a + b ≥ a+b
˜ ˜ ˜ ˜
This is known as the triangle inequality.
WORKED
Example 9 Use vectors to show that the diagonals of a parallelogram bisect each other.
17
10 Use vectors to show that the angle subtended by a diameter of a circle is a right angle.

11 Use a vector method to show that by joining the midpoints of a parallelogram, the
figure formed is a parallelogram.

12 Consider any two major diagonals of a cube. Use a vector method to show that:
a the diagonals bisect each other
b the acute angle between the diagonals is 70.53°.

13 Use vectors to demonstrate Pythagoras’ theorem.


390 M a t h s Q u e s t M a t h s C Ye a r 1 1 f o r Q u e e n s l a n d

summary
Force diagrams and the triangle of forces
• Force is a vector quantity. Its derived unit is the Newton. 1 N = 1 kg m/s2.
• Types of force:
Field forces occur without physical contact. A common field force is the
gravitational force often referred to as ‘weight’: W = mg down. The magnitude of
g is 9.8 m/s2. ˜ ˜
˜
Normal contact forces occur between objects in contact or via strings where they
are known as tensile forces.
Friction forces occur when there is actual or attempted ~F1
relative movement between two objects in contact.
~F1
• The resultant force R is the vector sum of all physical forces
acting on an object. ˜A force vector diagram illustrates this
with two forces F 1 and F 2 :
R ~F2
˜ ˜ F ~ = ~F1 + ~F2
~2
j
• A force F can be resolved into horizontal ~
˜ components.
and vertical ~i
F = Fx + Fy ~F
˜ ˜ ˜ ~Fy
= F cos θ i + F sin θ j θ
˜ ˜ ~Fx
Newton’s First Law of Motion
• Newton’s First Law of Motion is used for equilibrium or statics problems:
R =0 (the resultant force is zero)
a˜ = 0 (the magnitude of acceleration is zero)
v =c (the magnitude of velocity is a constant).
• Newton’s First Law of Motion: In the absence of an unbalanced force (that is,
R = 0) a moving body moves in a state of uniform motion (or constant velocity).
˜
• Friction = µ × normal reaction, where µ is the coefficient of friction
F =µ× N
˜ ˜
Momentum
• The momentum of an object is the product of its mass and velocity.
Momentum = mass × velocity
• In a collision the momentum of the system is conserved:
momentum of system before collision = momentum of system after collision
• Momentum is a vector quantity. Its units are N s or kg m/s.
Relative velocity
• The velocity of an object measured by an observer depends on the frame of
reference of the observer. In general:
v a = v a rel b + v b
˜ ˜ ˜
where v a rel b is the velocity of a relative to b.
˜
Using vectors in geometry
• In proofs using vectors it is simpler to use pronumerals to represent vectors.
• Use as few vectors as necessary in completing a proof and apply the rules for vector
addition, subtraction and dot products as appropriate.
C h a p t e r 8 Ve c t o r a p p l i c a t i o n s 391

CHAPTER
review
1 multiple choice
8A
A body is in equilibrium under the action of three forces f 1 , f 2 and f 3 .
˜ ˜ ˜
f 1 = −3 i + 7 j and f 2 = 5 i + 2 j
˜ ˜ ˜ ˜ ˜ ˜
The magnitude of the vector f 3 is
˜
A 2i − 9 j B 85 C 77 D − 85 E 17
˜ ˜
2 multiple choice
8A
If the three forces shown are in equilibrium, the magnitude of the 20 N
force F is
F
60°
A 34.6 N B 40 N C 20 2 N
D 54.3 N E 80 N 20 N

3 multiple choice
8A
Two forces, equal in magnitude, are sufficient to keep the mass
shown at right in equilibrium against the force of gravity.
The magnitude of each force is
A 8.9 N B 10.8 N C 17.8 N 50°
D 19.6 N E 22.8 N
2 kg

4 An object is in equilibrium under the action of three


forces f 1 , f 2 and f 3 . 8A
˜ ˜ ˜ f2 = 40 N
˜
Calculate the magnitude and direction of f 3 . 89.4 N at 153.4° to f 1 f1 = 80 N
˜ ˜ f3 ˜
˜
138.6° 5 Three coplanar forces have magnitudes of 400 N, 500 N and 600 N. They act on a body
such that the resultant force is zero. Find the angle between the 500 N and 600 N forces. 8A
6 A body of mass 6.0 kg is suspended from a ceiling by two ropes. The angle between the two
ropes is 90° and they are connected to the ceiling at points A and B respectively. The tension 8B
T2, in rope 2, is twice the tension T1, in rope 1.
a Draw a force vector diagram for the 6.0 kg mass.
b Calculate the magnitudes of the tensions T1 and T2. 26.3 N, 52.6 N
c Find the angles that rope 1 and rope 2 make with the horizontal. 26.6°, 63.4°
5x
d If rope 1 has a length x, show that the distance AB is ---------- .
2
392 M a t h s Q u e s t M a t h s C Ye a r 1 1 f o r Q u e e n s l a n d

7 A mass of 4 kg rests on a plane which has a coefficient of


8B friction of 0.25. Calculate the minimum force P , acting
P
˜
˜ slipping
parallel to the plane, needed to prevent the mass
down the plane. 17.7 N

40°
8 multiple choice
8C
The momentum of an 85 kg sprinter who runs 100 metres in 10 seconds is
A 0.85 N s B 10 N s C 85 N s D 850 N s E 8500 N s

9 multiple choice
8C
A tennis ball of mass 85 g hits a wall at 40 m/s and rebounds at 30 m/s. The change in
momentum of the tennis ball is
A 10 N s B 5.95 N s C 3.4 N s D 2.55 N s E 0.85 N s

10 multiple choice
8C
How fast would an object of mass 45 g have to move to have the same momentum as a car
of mass 1170 kg moving at 100 km /h?
A approximately 500 000 m/s B approximately 600 000 m/s
C approximately 700 000 m/s D approximately 800 000 m/s
E approximately 900 000 m/s

11 multiple choice
8C
A car of mass 1250 kg moves at 70 km/h in a northerly direction. It collides with a car of
mass 1300 kg moving at 80 km/h in an easterly direction. The magnitude of the momentum
of the system before the collision is closest to
A 23 457 N s B 24 543 N s C 29 992 N s D 37 747 N s E 43 450 N s

12 The diagram shows the view from above as a ball,


8C travelling horizontally, rebounds at a wall. The mass
30° 30°

of the ball is 150 g and its speed, before and after


the collision, is 20 m/s. Calculate the change in the
momentum of the ball. 3 N s away from the wall
13 A 100 kg rugby player, A, tackles an opposition player, B, at right
8C angles as shown. The opposition player has a mass of 85 kg.
A
v = 8 m/s
B

After the tackle they fall together. Calculate the direction and speed
of their joint motion after the tackle. 6.0 m/s at 43.7° with A’s original motion v = 9 m/s

14 multiple choice
8D
If v a = 2 i + 3 j and v b = −2 i + 2 j , the value of v a rel b is
˜ ˜ ˜ ˜ ˜ ˜ ˜
A 5j B 4i + j C −4 i − j D j E 5
˜ ˜ ˜ ˜ ˜ ˜
15 multiple choice
8D
A traveller in a car is moving in a northerly direction. He observes a second vehicle and to
him it appears to be travelling in a south-easterly direction. Which of the following could be
possible values for the true direction of the second vehicle?
A North B South C East D West E South-west
C h a p t e r 8 Ve c t o r a p p l i c a t i o n s 393
16 A river flows from north to south at 5 km/h. A boat heads directly across the river from the
river bank to the other side at 11 km/h. Find the true velocity of the boat. 12.1 km/h at 65.6° 8D
to the bank
17 A cyclist travels at 15 km/h on a road heading east. When she is travelling in this direction
the wind appears to be coming from a direction 60° E of N. When she turns around and 8D
travels west on the road at the same speed the wind appears to come from 60° W of N. Find
the true speed and direction of the wind. 8.7 km/h from the north

18 multiple choice
8E
Which of the statements regarding p is true? A
2 cm
A p =b −a B˜ p = 2 b − 4 a
˜ ˜ ˜ ˜ ˜ ˜ a
˜ p
C p = 1--- b + 1--- a D p = 1--- b + 2--- a ˜ 4 cm
2 ˜ 2 ˜ 3 ˜ 3 ˜
˜ ˜ B
E p = b + --- a
2--- 1 b
˜ 3 ˜ 3 ˜ ˜
19 Show that the diagonals of a square bisect each other at 90°.
8E
20 For any vectors u and v show that
(u + v ) • (u −
˜ ˜
v ) = | u | 2 − | v |2 8E
˜ ˜ ˜ ˜ ˜ ˜
Modelling and problem solving
1 Arnie is pushing against a trailer, preventing it from rolling down a hill. The trailer has a mass
of 200 kg and the hill is on an incline of 15° to the horizontal. At the moment there is no
problem because Arnie is capable of pushing with a force of 1000 N parallel to the plane.
However, it is raining and the trailer is filling with water at a rate of 25 litres per minute.
How long will Arnie be able to hold the trailer and stop it from running down the hill?
7 minutes 27 seconds
2 When the surf gets big at Kirra there is always a sweep running from south to north.

Take-off area
Sweep = 4 km/h
120 m
Water
Shore

Jodie is heading out at 8 km/h. If she heads directly out to sea and she wants to get to the
take-off area, how far up the beach should she walk before paddling out? 60 m
394 M a t h s Q u e s t M a t h s C Ye a r 1 1 f o r Q u e e n s l a n d

3 A ball of mass 0.20 kg is shot vertically in the air. It decelerates under the action of two
forces: the weight force and the force of air resistance. When the ball moves with a speed of
40 m/s, it has an air resistance of 1.0 N. When the ball is stationary, the air resistance force is
equal to zero.
a Determine the magnitude and direction of the resultant force acting on the ball when it is
moving upwards at a speed of 40 m/s. 2.96 N down
b Determine the magnitude and direction of the resultant force acting on the ball when it is
at its maximum height above the ground. 1.96 N down
Later, the ball is travelling toward the ground at 40 m/s.
c Determine the magnitude and direction of the resultant force acting on the ball when it is
moving downwards at a speed of 40 m/s. 0.96 N down

4 In a James Bond movie scene, a car of mass 1500 kg rolls in a straight line from rest down a
road inclined at 10° from the horizontal. It takes 10 seconds for the car to reach the bottom of
the incline at a speed of 43.2 km/h where the road becomes level. The handbrake of the car
was on, providing a constant retardation force of 20g newtons.
a Find the value of the coefficient of friction correct to 3 decimal places. 0.038
At the bottom of the incline the car jolts, and the handbrake is rendered inactive.
b How much further does the car travel? (Give your answer to the nearest tenth of a metre.) 193.5 m

5 A pyramid, OABCD, is shown in the figure at right. The height D (3, 4, − 1_3)
of the pyramid is the length of DE, where E is the point on the C B
base OABC such that DE is perpendicular to the base. (−2, 5, −2) E (2, 6, 2)
a Show that the base, OABC, is a rhombus. Check with your teacher.
O (0, 0, 0) A (4, 1, 4)
b Use a vector method to find ∠AOC correct to the nearest
tenth of a degree. 109.5°
c The unit vector p i + q j + r k , p > 0, is perpendicular to both OA and OC.
i Show that q =˜0 and˜ find ˜the exact values of p and r. p = ------
1
2
- ; r = − -------
1
2
ii Hence find the exact height of the pyramid. 5 2
---------- units
3
6 A parallelogram, OXYZ, has O at the origin. The vector joining O to Z is given by 6 i while
the vector joining O to X is given by 3 i + 5 j . ˜
˜ ˜ YX = −6 i ; ZY = 3 i + 5 j
a Sketch the parallelogram, labelling all vertices. ˜ ˜ ˜
b State the vectors joining Z to Y and Y to X.
c What vectors represent the diagonals of the parallelogram? OY = 9 i + 5 j ; ZX = −3 i + 5 j
˜ ˜ ˜ ˜
d Find the angle between the diagonals to the nearest tenth of a degree. 91.9°
eBook plus e Find the angle that OY makes with the x-axis. 29.1°
f State the vector resolute of the vector joining O to X, perpendicular to OZ. 5 ˜j
Digital doc: g Let P be a point on the line ZY, such that the vector joining P to X is perpendicular to ZX.
Test Yourself
Chapter 8 Find the exact coordinates of P. (12 3--8- , 10 5--8- )
h Find the exact area of the triangle XPZ. 31 7--8- square units
a
y
X (3, 5) Y (9, 5)

Z (6, 0)
O x
Sequences
and series

9
syllabus reference
Core topic:
Structures and patterns

In this chapter
9A Arithmetic sequences
9B Geometric sequences
9C Applications of geometric
sequences
9D Finding the sum of an infinite
geometric sequence
9E Contrasting arithmetic and
geometric sequences through
graphs
Fibonacci Sequence
The Mandelbrot Set
396 M a t h s Q u e s t M a t h s C Ye a r 1 1 f o r Q u e e n s l a n d
• sum to infinity of a geometric progression
• purely mathematical and life-related applications of arithmetic and geometric progressions
• sequences and series other than arithmetic and geometric
Introduction • recognition of patterns in well-known structures including Pascal’s Triangle and Fibonacci
sequence
• applications of patterns
Discovered in 1980 by Benoit Mandelbrot, the Mandelbrot Set (see the image on
page 446) is one of the most intricate and beautiful geometrical patterns in math-
ematics. The Mandelbrot Set is an image that captures many of the qualities people find
fascinating about mathematics. Although it is generated by repeating a simple formula,
its patterns are infinitely complex. If you select any portion of the Mandelbrot Set and
magnify it you will see that no detail is lost — the magnified shape is as intricate and
even contains parts that look like copies of the original. This notion of ‘worlds within
worlds’ appeals to the philosopher in all of us.
How is the Mandelbrot Set created? Using some of the concepts developed in the
following sections we will see how a computer or a graphics calculator can produce
this fascinating image.

Arithmetic sequences
A sequence in mathematics is an ordered set of numbers.
An arithmetic sequence is one in which:
1. the difference between any two successive terms is the same
2. the next term in the sequence is found by adding the same number.
Consider the arithmetic sequence:
4, 7, 10, 13, 16, 19, 22.
The difference between each successive term is +3, or similarly, the next term is found
by adding 3 to the previous term. We can see that a positive common difference gives a
sequence that is increasing. We say that the common difference is +3, stated as d = +3.

+3 +3 +3 +3 +3 +3
4 7 10 13 16 19 22

The first term of the sequence is 4. We refer to the first term of a sequence as ‘a’. So
in this example, a = 4.
In the arithmetic sequence above, the first term is 4, the second term is 7, the third
term is 10 and so on. Another way of writing this is:
t1 = 4, t2 = 7 and t3 = 10.
There are 7 terms in this sequence. Because there are a countable number of terms in
the sequence, it is referred to as a finite sequence.
The arithmetic sequence below

–7 –7 –7 –7
37, 30, 23, 16, 9...
is an infinite sequence since it continues endlessly as indicated by the dots. The first
term, a, is 37 and the common difference, d, is −7. We can see that a negative common
difference gives a sequence that is decreasing.

1. An arithmetic sequence is a sequence of numbers for which the difference


between successive terms is the same.
2. The first term of an arithmetic sequence is referred to as ‘a’.
3. The common difference between successive terms is referred to as ‘d’.
4. tn is the term number, for example, t6 refers to the 6th term in the sequence.
Chapter 9 Sequences and series 397
WORKED Example 1
Which of the following are arithmetic sequences?
a 7, 13, 19, 25, 31, . . .
b 1.3, 2.5, 3.7, 4.9, 6.3, . . .
c −1 1--2- , −1, − 1--2- , 0, 1--2- , . . .

THINK WRITE
a 1 Write the sequence. a 7, 13, 19, 25, 31, . . .
2 Calculate the difference between the first t2 − t1 = 13 − 7
term, t1, and the second term, t2. =6
3 Calculate the difference between the t3 − t2 = 19 − 13
second term, t2, and the third term, t3. =6
4 Calculate the difference between the third t4 − t3 = 25 − 19
term, t3, and the fourth term, t4. =6
5 Calculate the difference between t5 t5 − t4 = 31 − 25
and t4. =6
6 Check that the differences are the same There is a common difference of 6,
and write your answer. therefore d = 6.
This is an arithmetic sequence.

b 1 Write the sequence. b 1.3, 2.5, 3.7, 4.9, 6.3, . . .


2 Calculate the difference between t2 and t1. 2.5 − 1.3 = 1.2
3 Calculate the difference between t3 and t2. 3.7 − 2.5 = 1.2
4 Calculate the difference between t4 and t3. 4.9 − 3.7 = 1.2
5 Calculate the difference between t5 and t4. 6.3 − 4.9 = 1.4
6 Check that the differences are the same. There is no common difference.
This is not an arithmetic sequence.

c 1 Write the sequence. c −1 1--- , −1, − 1--- , 0, 1--- , . . .


2 2 2

2 Calculate the difference between t2 and t1. −1 − (−1 1--- ) = 1


---
2 2

3 Calculate the difference between t3 and t2. − 1--- − (−1) = 1


---
2 2

4 Calculate the difference between t4 and t3. 0 − (− 1--- ) = 1


---
2 2

5 Calculate the difference between t5 and t4. 1


--- −0= 1
---
2 2

6 Check that the differences are the same. There is a common difference of --1- .
2
This is an arithmetic sequence.
398 M a t h s Q u e s t M a t h s C Ye a r 1 1 f o r Q u e e n s l a n d

WORKED Example 2
Write down the value of a and d for each of the following arithmetic sequences.
a 1.2, 3.6, 6, 8.4, 10.8, . . .
b −1 2--5- , − 2--5- , 3--5- , 1 3--5- , 2 3--5- , . . .
THINK WRITE
a 1 Write the sequence. a 1.2, 3.6, 6, 8.4, 10.8, . . .
2 What is the first term? a = 1.2
3 What is the difference between t2 and t1? t2 − t1 = 3.6 − 1.2
You need check only once as the question = +2.4
states that this is an arithmetic sequence. d = +2.4
4 Write your answer. The arithmetic sequence has a first term,
a, of 1.2 and a common difference, d, of
+2.4.
b 1 Write the sequence. b −1 2--- , − 2--- , 3--- , 1 3--- , 2 3--- , . . .
5 5 5 5 5
2 What is the first term? a = −1 2---
5
3 What is the difference between t2 t2 − t1 = − 2--- − (−1 2--- )
5 5
and t1? = +1
d = +1
4 Write your answer. The arithmetic sequence has a first term,
a, of 1 2--- and a common difference, d,
5
of +1.

Finding the terms of an arithmetic sequence


Consider the arithmetic sequence for which a = 8 and d = 10.

+10 +10 +10 +10


8 18 28 38 48
Now, t1 = 8 t1 = a
t2 = 8 + 10 t2 = a + d t2 = a + 1d
t3 = 8 + 10 + 10 t3 = a + d + d t3 = a + 2d
t4 = 8 + 10 + 10 + 10 t4 = a + d + d + d t4 = a + 3d
t5 = 8 + 10 + 10 + 10 + 10 t5 = a + d + d + d + d t5 = a + 4d
We notice a pattern emerging. That pattern can be described by the equation:
tn = 8 + (n − 1) × 10
where n represents the number of the term.
For example, if n = 4, then the fourth term is:
t4 = 8 + (4 − 1) × 10
t4 = 8 + 3 × 10
t4 = 38
Therefore, the 4th term is 38.
We can generalise this rule for all arithmetic sequences.
tn = a + (n − 1)d
where tn is the nth term
a is the first term
d is the common difference.
Chapter 9 Sequences and series 399
This rule enables us to find any term of an arithmetic sequence provided we know the
value of a and d.

WORKED Example 3
Find the 20th term of the following arithmetic sequence.
5, 40, 75, 110, 145, . . .
THINK WRITE
1 Find the value of a. a=5
2 Find the value of d. You need to calculate only d = t2 − t1
one difference as the question states that it is an = 40 − 5
arithmetic sequence. = 35
3 Use the rule tn = a + (n − 1)d where n is 20 for t20 = 5 + (20 − 1) × 35
the 20th term. = 5 + 19 × 35
= 670
4 Write the answer. The 20th term is 670.

If we are given only two terms of an arithmetic sequence, we are able to use the rule
tn = a + (n − 1)d to set up two simultaneous equations to find the value of a and d and
hence write down the rule for the arithmetic sequence.

WORKED Example 4
The third term of an arithmetic sequence is −1 and the fifth term is 11.
a Write down the rule for the arithmetic sequence.
b Find the 50th term of the sequence.
THINK WRITE
a 1 We know that t3 = −1 and that a t3 = a + 2d = −1
tn = a + (n − 1)d.
2 We know that t5 = 11 and that
tn = a + (n − 1)d. t5 = a + 4d = 11
3 Solve the 2 equations simultaneously a + 2d = −1 [1]
using the elimination technique. a + 4d = 11 [2]
Eliminate a, by subtracting equation [1] 2d = 12 [2] − [1]
from equation [2]. d=6
4 Evaluate a by substituting d = 6 into Substituting d = 6 into [1]:
either of the two equations. a + 12 = −1
a = −13
5 To find the rule, substitute values for a tn = −13 + (n − 1) × 6
and d into tn = a + (n − 1) d. tn = −13 + 6n − 6
tn = −19 + 6n
b 1 To find the 50th term or t50, substitute b tn = −19 + 6n
n = 50 into the rule. t50 = −19 + 6 × 50
= −19 + 300
= 281
2 Write your answer. The 50th term is 281.
400 M a t h s Q u e s t M a t h s C Ye a r 1 1 f o r Q u e e n s l a n d

Graphics Calculator tip! Listing the terms of an


arithmetic sequence
If you know the rule for an arithmetic sequence, successive terms can be listed using a
graphics calculator. The steps for generating these terms are shown below. Consider an
example where the first term is 3 and the difference is 2 so the rule is tn = 3 + (n − 1) × 2.
For the Casio fx-9860G AU
1. Press MENU and select RUN-MAT. Press OPTN
then F1 (LIST) followed by F5 (Seq).

2. Enter the rule (3 + (n − 1) × 2) followed by the


variable name (n), the start value for n (1), the end
value of n (say, 10) and the increment (1), each
separated by a comma ( , ). Press ) to close the
set of brackets. (To enter the variable n, press
ALPHA [N].)
3. Press EXE to display the list of the first 10 terms.
Use the down arrow key to scroll down the list.

For the TI-Nspire CAS


1. Open a new Lists & Spreadsheet document. Press
b, then 3: Data.

2. Press 1: Generate Sequence and complete at least


the first two entry boxes, pressing e to move to the
next line. Enter 3 + (n − 1) × 2 for the Formula and
3 for the Initial Terms. Completing the boxes for
Max No. Terms (the default value is 255) and
Ceiling Value is optional. Press e until OK is
highlighted.

3. Press · to display the list of terms. Use the NavPad


to scroll down the list.

4. If you wish to widen Column A, use the up arrow on


the NavPad to highlight the column and then press
b followed by 1: Actions, 2: Resize and 1: Resize
Column Width. Press the right arrow until you reach
the required width and press ·.
Chapter 9 Sequences and series 401
The sum of a given number of terms of an
arithmetic sequence
When the terms of an arithmetic sequence are added together, an arithmetic series is
formed.
So, 5, 9, 13, 17, 21, . . . is an arithmetic sequence
whereas 5 + 9 + 13 + 17 + 21 + . . . is an arithmetic series.
The sum of n terms of an arithmetic sequence is given by Sn.
Consider the finite arithmetic sequence below.
5, 10, 15, 20, 25, 30, 35, 40, 45, 50
The sum of this arithmetic sequence is given by S10 since there are 10 terms in the sequence.
So, S10 = 5 + 10 + 15 + 20 + 25 + 30 + 35 + 40 + 45 + 50
= 275
Note that the sum of the first and last terms is 55. Also, the sum of the second and
second last terms is 55. Similarly, the sum of the third and third last term is 55. This
pattern continues with the fourth and fourth last terms as well as with the fifth and fifth
last terms. There are in fact five lots of 55.
We can formalise this pattern to obtain a rule which applies to all arithmetic sequences.
Let Sn = a + (a + d) + (a + 2d) + . . . + (l − 2d) + (l − d) + l
where l is the last term of the sequence.
By reversing the order of the series above, we obtain
Sn = l + (l − d) + (l − 2d) + . . . + (a + 2d) + (a + d) + a.
By adding these two equations, we obtain
2Sn = (a + l) + (a + d + l − d) + (a + 2d + l − 2d) + . . . (a + l) + (a + l) + (a + l)
2Sn = (a + l) + (a + l) + (a + l) + . . . (a + l) + (a + l) + (a + l)
2Sn = n(a + l) where n represents the number of terms in the sequence.
So, S n = 1--2- n ( a + l ) .
The sum of n terms of an arithmetic sequence with a as its first term and l as its
last term is given by
n
S n = --- ( a + l ) .
2
Now we know that the nth term of an arithmetic sequence is given by
t n = a + ( n – 1 )d .
So, for the sum of n terms, l is the last term; that is, tn = l.
So, the last term is l = a + (n − 1)d.
n n
Substituting this into Sn = --- ( a + l ) we obtain Sn = --- { a + [ a + ( n – 1 )d ] }
2 2
n
= --- [ 2a + ( n – 1 )d ] .
2
An alternative formula for the sum of n terms of an arithmetic sequence when the
value of a and d are known is given by
n
S n = --- [ 2a + ( n – 1 )d ] .
2
402 M a t h s Q u e s t M a t h s C Ye a r 1 1 f o r Q u e e n s l a n d

WORKED Example 5
Find the sum of the first ten given terms of the arithmetic sequence 4, 10, 16, 22, 28, 34, 40,
46, 52, 58.

THINK WRITE
Method 1
1 We know the values of the first and last a=4
term and that there are ten terms in the l = 58
series. n = 10
n n
2 Use the series formula Sn = --- ( a + l ) . Sn = --- ( a + l )
2 2
10
= ------ ( 4 + 58 )
2
= 5 × 62
= 310

Method 2 (alternative)
1 We know the value of a and d and n. a=4
d = 10 − 4 = 6
n = 10
n n
2 Use the formula Sn = --- [ 2a + ( n – 1 )d ] . Sn = --- [ 2a + ( n – 1 )d ]
2 2
10
S10 = ------ [ 2 × 4 + ( 10 – 1 )6 ]
2
S20 = 5[8 + 9 × 6]
= 5[8 + 54]
= 5 × 62
= 310
3 Write the answer. The sum of the first 10 terms is 310.

Graphics Calculator tip! Finding the sum of an


arithmetic sequence
For the arithmetic sequence 4, 10, 16, 22, …, 58 considered in Worked example 5, we
can use a graphics calculator to find the sum of the first 10 terms. The rule for this
sequence is 4 + (n − 1) × 6.

For the Casio fx-9860G AU


1. Press MENU and select RUN-MAT. Press OPTN
then F1 (LIST) followed by F5 (Seq). Enter the
rule (4 + (n − 1) × 6) followed by the variable name
(n), the start value (1), the end value (10) and the
increment (1), each separated by a comma ( , ).
Press ) to close the set of brackets.
Chapter 9 Sequences and series 403
2. To store this sequence of 10 terms as List 1, press
Æ then F1 (List) and 1 followed by EXE .

s
3. Press F6 ( ) twice to display the Sum option and
then press F1 (Sum).
s

4. Press F6 ( ) again for more options and then F1


(List) and 1 to indicate List 1. Press EXE to
display the sum of the first 10 terms. The sum of the
first 10 terms is 310.

For the TI-Nspire CAS


1. Open a new Lists & Spreadsheet document. Press
b, then 3: Data followed by 1: Generate
Sequence. Enter the rule 4 + (n − 1) × 6 with an
initial term of 4. Press e until OK is highlighted.

2. Press · to display the list of terms. Use the NavPad


to scroll across to cell B1.

3. To calculate the sum of the first 10 terms, we can add


the numbers in cells A1 to A10. Enter the formula
= sum(A1:A10) in cell B1.
(Press =SUM(A1:A10).)

4. Press · to display the sum in cell B1. The sum of


the first 10 terms is 310.
404 M a t h s Q u e s t M a t h s C Ye a r 1 1 f o r Q u e e n s l a n d

remember
remember
1. In an arithmetic sequence:
(a) the first term is a
(b) the common difference is d.
2. Given an unspecified sequence, establish whether it is arithmetic by testing all
terms for a common difference: d = t2 − t1 = t3 − t2 = t4 − t3 = . . .
3. To find the term of an arithmetic sequence use the following formula:
tn = a + (n − 1) d
where tn is the nth term
a is the first term
d is the common difference.
4. A series is the sum of terms in a sequence.
5. Sn is the sum of the first n terms in a series; for example, S25 represents the sum
of the first 25 terms.
6. Given the number of terms in a series, n, the first term, a, and the last term, l,
n
use Sn = --- ( a + l ) .
2
7. Given the number of terms in a series, n, the first term, a, and the common
n
difference, d, use Sn = --- [ 2a + ( n – 1 )d ] .
2

9A Arithmetic sequences
WORKED 1 State which of the following are arithmetic sequences. 1 a, c, d, g, h
Example
xample
1
a 2, 7, 12, 17, 22, . . . b 3, 7, 11, 15, 20, . . .
c 0, 100, 200, 300, 400, . . . d −123, −23, 77, 177, 277, . . .
e 1, 0, −1, −3, −5, . . . f 6.2, 9.3, 12.4, 15.5, 16.6, . . .
g 1--- , 1 1--- , 2 1--- , 3 1--- , 4 1--- , . . . h 1--- , 3--- , 1 1--- , 1 3--- , 2 1--- , . . .
2 2 2 2 2 4 4 4 4 4
WORKED
Example
xample 2 For those arithmetic sequences found in question 1, write down the values of a and d.
2
2 a a = 2, d = 5
3 For each of the arithmetic sequences given, find:
WORKED c a = 0, d = 100
Example
xample a the 25th term of the sequence 2, 7, 12, 17, 22, 122
... d a = −123, d = 100
3
b the 30th term of the sequence 0, 100, 200, 300, 400, 2900
...
g a = 1--2- , d = 1
c the 33rd term of the sequence 5, −2, −9, −16, −23,−.219
..
h a = --14- , d = --2-
4
WORKED 4 Evaluate the following.
Example
xample
4 a The 2nd term of an arithmetic sequence is 13 and the 5th term is 31. What is the
17th term of this sequence? 103
b The 2nd term of an arithmetic sequence is −23 and the 5th term is 277. What is the
20th term of this sequence? 1777
c The 2nd term of an arithmetic sequence is 0 and the 6th term is −8. What is the
32nd term of this sequence? −60
d The 3rd term of an arithmetic sequence is 5 and the 7th term is −19. What is the
40th term of this sequence? −217
e The 4th term of an arithmetic sequence is 2 and the 9th term is −33. What is the
26th term of this sequence? −152
Chapter 9 Sequences and series 405
WORKED 5 For each of the given series, find:
Example
a the sum of the first 20 terms of the sequence 3, 7, 11, 15, 19, 820
...
5 270
b the sum of the first 15 terms of the sequence 4, 6, 8, 10, 12, . . .
c the sum of the first 23 terms of the sequence −4, −1, 2, 5, 8, 667
...

6 multiple choice
The 41st term of the arithmetic sequence −4.3, −2.1, 0.1, 2.3, 4.5, . . . is:
A 83.7 B 85.9 C 92.3 D 172.4 E 178.5

7 multiple choice
The 2nd term of an arithmetic sequence is −2 and the 5th term is 2.5. The 27th term
of this sequence is:
A 32.5 B 35.5 C 42.5 D 89.5 E 96

8 multiple choice
The sum of the first 21 terms of the sequence, 0, −3 1--- , −7, −10 1--- , −14, . . . is:
2 2
A −1470 B −735 C −700 D 36.75 E 735

9 multiple choice
The first term of an arithmetic sequence is −5.2 and the 2nd is −6. The sum of the first
22 terms of the sequence is:
A −598.4 B −299.2 C −242 D −70.4 E 70.4
10 a, d, e 10 State which of the following situations are arithmetic sequences.
a A teacher hands out 2 lollies to the first student,
4 lollies to the second student, 6 lollies to the
third student and 8 lollies to the fourth student.
b The sequence of numbers after rolling a die 8
times.
c The number of layers of paper after each folding
in half of a large sheet of paper.
11 d The house numbers on the same side of a street on a newspaper delivery route.
a a = 2, d = 2 e The cumulative total of the number of seats in the first ten rows in a regular
d a = not specified, d = 2 cinema (for example, with 8 seats in each row, so there are 8 seats after the first
e a = 8, d = 8 row, 16 seats after the first 2 rows, and so on).
11 For those arithmetic sequences found in question 10, where
appropriate information is given, write down the value of a
and d.
12 For the following arithmetic sequences:
7th a 4, 13, 22, 31, . . . which term, tn, will be equal to 58?
b 9, 4.5, 0, . . . which term, tn, will be equal to −18? 7th
c −60, −49, −38, . . . which term, tn, will be the first to be
greater than 10? 8th
d 100, 87, 74, . . . which term, tn, will be the first to be less
than 58? 5th
13 A batsman made 23 runs in his first innings, 33 in his
second and 43 in his third. If he continued to add 10 runs
each innings, write down a rule for the number of runs he
would have made in his nth innings. tn = 13 + 10n
406 M a t h s Q u e s t M a t h s C Ye a r 1 1 f o r Q u e e n s l a n d

tn = 37 + 3n 14 In a vineyard, rows of wire fences are built to support the vines. The length of the
fence in row 1 is 40 m, the length of the fence in row 2 is 43 m, and the length of
the fence in row 3 is 46 m. If the lengths of the fences continue in this pattern, write
down a rule for the length of a fence in row number n.

SLE 12: Investigate the use 15 The first fence post in a fence is 12 m from the road, the next is 15.5 m from the road and
of an arithmetic progression
in real-life situations. the next is 19 m from the road. The rest of the fence posts are spaced in this pattern.
a Write down a rule for the distance of fence post n from the road. tn = 8.5 + 3.5n
b If 100 posts are to be erected, how far will the last post be from the road? 358.5 metres
16 The 1st term of an arithmetic sequence is 2 and the sum of the first 19 terms of the
sequence is 551. Find:
a the 19th term 56 b the first 3 terms of the sequence. 2, 5, 8
17 The 1st term of an arithmetic sequence is −4 and the sum of the first 30 terms of the
sequence is 2490. Find:
a the 30th term 170 b the first 3 terms of the sequence. −4, 2, 8
18 Sam makes $100 profit in his first week of business. If his profit increases by $75 each
week, what would his profit be in total by the end of week 15? $9375
19 George’s salary is to start at $36 000 a year and increase by $1200 each year after
that. How much will George have earned in total after 10 years? $414 000
20 A staircase is designed so that the height of each step increases by 0.8 cm for each step.
If the height of the first step is 15 cm, what is the total height of the first 17 steps? 363.8 cm
21 Paula collects stamps. She bought 250 in the first month to start her collection and
added 15 stamps to the collection each month thereafter. How many stamps will she
have collected after 5 years? 1135
22 Proceeds from the church fete were $3000 in 1990. In 1991 the proceeds were $3400
and in 1992 they were $3800. If they continued in this pattern:
a what were the proceeds from the year 2009 fete? $10 600
b how much in total would the proceeds from church fetes since 1990 have
amounted to in the year 2009? $136 000

Geometric sequences
A sequence in mathematics is an ordered set of numbers. A geometric sequence is one
in which the first term is multiplied by a number, known as the common ratio, to create
the second term which is multiplied by the common ratio to create the third term, and
so on. The first term in a geometric sequence is referred to as a and the common ratio
is referred to as r.
Consider the geometric sequence where a = 1 and r = 3. The terms in the sequence are
×3 ×3 ×3 ×3
1 3 9 27 81...
Chapter 9 Sequences and series 407
To discover the common ratio, r, of a geometric sequence you need to calculate the ratio
t t t
of successive terms, namely ---2- . You could alternatively calculate ---3- or ---4- and so on.
t1 t2 t3
A geometric sequence is a sequence of numbers for which the ratio of successive
terms is the same.
t t t
---2- = ---3- = ---4- = º = common ratio
t1 t2 t3
The first term of a geometric sequence is referred to as a.
The common ratio between a term and its preceding term is referred to as r.

WORKED Example 6
Which of the following are geometric sequences?
a 2, 10, 50, 250, 1250, . . .
b −2, −6, 18, 54, −162, . . .

THINK WRITE
a 1 Write the sequence. a 2, 10, 50, 250, 1250, . . .
t t
2 Calculate the ratio of ---2- . ---2- = 10
------
t1 t1 2
= 5
t t
3 Calculate the ratio of ---3- . ---3- = 50
------
t2 t2 10
= 5
t t
4 Calculate the ratio of ---4- . ---4- = 250
---------
t3 t3 50
= 5
t t
5 Calculate the ratio of ---5- . ---5- = 1250
------------
t4 t4 250
= 5
6 Check that all ratios are the same. There is a common ratio of 5.
This is a geometric sequence.

b 1 Write the sequence. b −2, −6, 18, 54, −162, . . .


t t
2 Calculate the ratio of ---2- . ---2- = –-----6-
t1 t1 –2
= +3
t t 18
3 Calculate the ratio of ---3- . ---3- = ----- -
t2 t2 –6
= –3
4 There is no need to check any further There is no common ratio.
as the two ratios are not the same. This is not a geometric sequence.
408 M a t h s Q u e s t M a t h s C Ye a r 1 1 f o r Q u e e n s l a n d

WORKED Example 7
Write down the value of a and r for each of the following geometric sequences.
a 1.2, −4.8, 19.2, −76.8, 307.2, . . .
b −1, 3--- , − -----
9-
, 27
------ , − --------
81-
, ...
4 16 64 256

THINK WRITE
a 1 Write the sequence. a 1.2, −4.8, 19.2, −76.8, 307.2, . . .
2 a is the first term. a = 1.2
t t 2 – 4.8
3 To find r use the ratio ---2- or any other ---- = ----------
t1 t1 1.2
ratio. = −4 or
t 3 19.2
---- = ----------
t 2 – 4.8
= −4
4 Write your answer. This is a geometric sequence with the first
term 1.2 and a common ratio of −4.
b 1 Write the sequence. b −1, 3--- , − -----
9-
, 27
------ , − --------
81-
, ...
4 16 64 256

2 a is the first term. a = −1


t 3
t2 ---
3 To find r use the ratio ---2- or any other ---- = -----
4
-
t1 t 1 –1
ratio.
= − 3--- or
4
9-
t 3 – -----
---- = -------
16
t2 3
---
4

= – -----
16
9-
× --4-
3

= − 3---
4

4 Write your answer. This is a geometric sequence with the first


term −1 and a common ratio of − 3--- .
4

Finding the terms of a geometric sequence


Consider the finite geometric sequence of seven terms for which a = 3 and r = 4.
×4 ×4 ×4 ×4
3 12 48 192 384

Now, t1 = 3 t1 = a
t2 = 3 × 4 t2 = a × r t2 = a × r1
t3 = 3 × 4 × 4 t3 = a × r × r t3 = a × r 2
t4 = 3 × 4 × 4 × 4 t4 = a × r × r × r t4 = a × r 3
t5 = 3 × 4 × 4 × 4 × 4 t5 = a × r × r × r × r t5 = a × r4 and so on . . .
Chapter 9 Sequences and series 409
We notice a pattern emerging. That pattern can be described by the equation:
tn = 3 × 4n − 1.
For example, if n = 5,
t5 = 3 × 44.
We can generalise this rule for all geometric sequences.
tn = ar n − 1
where tn is the nth term
a is the first term
r is the common ratio.

This rule enables us to find any term of a geometric sequence provided we know the
value of a and r.

WORKED Example 8
Find the 12th term of the geometric sequence:
2, 10, 50, 250, 1250, . . .

THINK WRITE
1 Find the value of a. a=2
10
2 It has been stated that it is a geometric r = ------
sequence, so find the value of r. 2
=5
3 Use the rule tn = a × r n − 1 to find the
12th term. t12 = 2 × 512 − 1
= 97 656 250
4 Write your answer. The value of the 12th term is 97 656 250.

WORKED Example 9
The 2nd term of a geometric sequence is 8 and the 5th is 512. Find the 10th term of
this sequence.

THINK WRITE
1 We know that t2 = 8 and that t2 = a × r1
tn = a × r n − 1. =8
2 We know that t5 = 512 and that t5 = a × r4
tn = a × r n − 1. = 512
3 Solve the 2 equations simultaneously a × r1 = 8 [1]
by eliminating a, to find r. a × r4 = 512 [2]
× r 4- 512
a-------------
Divide equation 2 by equation 1. = --------- [2] ÷ [1]
a×r 8
r = 64
3

r=4
Continued over page
410 M a t h s Q u e s t M a t h s C Ye a r 1 1 f o r Q u e e n s l a n d

THINK WRITE
4 To find a, substitute the value of r. Substituting r = 4 into equation [1]:
a×4=8
a=2
5 Write down the rule. tn = 2 × 4n − 1
6 To find the 10th term, let n = 10. t10 = 2 × 49
= 524 288
7 Write your answer. The 10th term in the sequence is 524 288.

WORKED Example 10
The first three terms of a geometric sequence are 2, 6, and 18.
Which numbered term would be the first to exceed 1 000 000 in this sequence?
THINK WRITE/DISPLAY
Method 1: Using logarithms
1 To find the rule for the sequence, find a a=2
and r and substitute them into t
tn = a × r n − 1. r = ---2
t1
= 6---
2
=3
tn = 2 × 3n − 1
2 Set up the equation to be solved. 2 × 3n − 1 = 1 000 000
3n − 1 = 500 000
3 Express 3n − 1 and 500 000 in terms of log 3n − 1 = log 500 000
logarithms to the base 10. (n − 1) log 3 = log 500 000
log 500 000
(n − 1) = ----------------------------
log 3
n − 1 = 11.9445
n = 12.9445
4 The next whole number term is the 13th. The 13th term would be the first to exceed
1 000 000.
Method 2: Trial and error
1 Find the rule for the sequence. a = 2 and r = 3
See Method 1. tn = 2 × 3n − 1
2 Set up the equation to be solved. 2 × 3n − 1 = 1 000 000
3n − 1 = 500 000
3 Try various values of n.
With n = 8, value is less than 1 000 000. Let n = 8, 377 = 2187 (too small)
With n = 15, value is greater than Let n = 15, 314 = 4 782 969 (too large)
1 000 000. Let n = 12, 311 = 177 147 (too small)
With n = 12, value is too small. Let n = 14, 313 = 1 594 323 (too large)
With n = 14, value is too large. Let n = 13, 312 = 531 441
With n = 13, value just exceeds 500 000. The 13th term is the required term.
Chapter 9 Sequences and series 411

THINK WRITE/DISPLAY
Method 3: Using a graphics calculator
1 Find the rule for the sequence. See Method 1. a = 2 and r = 3
tn = 2 × 3n−1
2 Locate the term that equals or exceeds 1 000 000. Two
methods are shown for this example.

Method A: Generating the terms of the sequence


For the Casio fx-9860G AU
(a) Press MENU and select RUN-MAT. Press OPTN
then F1 (LIST) followed by F5 (Seq). Enter the
rule (2 × 3n − 1) followed by the variable name (n),
the start value (1), the end value (say, 20) and the
increment (1), each separated by a comma. Press
) to close the set of brackets.

(b) Press EXE to display the list of terms.

(c) Scroll down until you reach the term that exceeds
1 000 000.

For the TI-Nspire CAS


(a) Open a new Lists & Spreadsheet document. Press
b, then 3: Data followed by 1: Generate
Sequence. Enter the rule 2 × 3n − 1 with an initial
term of 2. Press e until OK is highlighted.

(b) Press · to display the list of terms. Increase the


width of Column A to view the terms more easily.
(Press b then select 1: Actions, 2: Resize and
1: Resize Column Width.)

Continued over page


412 M a t h s Q u e s t M a t h s C Ye a r 1 1 f o r Q u e e n s l a n d

THINK WRITE/DISPLAY
(c) Scroll down until you reach the term that exceeds
1 000 000.

Method B: Solving an equation


Set up an equation to solve. 2 × 3n − 1 = 1 000 000

For the Casio fx-9860G AU


(a) Press MENU and select EQUA. Press F3 (SOLV) to
select F3: Solver. A previous equation and its
solution will be displayed.

(b) Enter the required equation in the first line and press
EXE . Notice that even though the variable in the
second line has changed to N, the value shown is not
the calculated solution.

(c) Press F6 (SOLV) to display the solution. (If you


wish to use the Solver again, press F1 (REPT).)

For the TI-Nspire CAS


Open a new Calculator document. Use the solve
function to find n when 2 × 3n − 1 = 1 00 000. Press b
and select 3: Algebra followed by 1: Solve (or type the
word solve and press (). Enter the equation to be
solved and then the variable, separated by a comma.
Press ) to close the set of brackets and press ·.

3 Write your answer. The first term to exceed


Note that the value of the 13th term may appear as 1.06E6 1 000 000 is the 13th.
or 1.06288E6 which is approximately 1.06 × 106 or
1 060 000. It is written in exponential form since it is too
large a number to fit on the screen.
Chapter 9 Sequences and series 413
The sum of a given number of terms of a
geometric sequence
SLE 1: Establish the formula When the terms of a geometric sequence are added, a geometric series is formed.
for the sum to n terms of a
geometric progression. So 3, 6, 12, 24, 48, . . . is a geometric sequence
whereas 3 + 6 + 12 + 24 + 48 + . . . is a geometric series.
The sum of n terms of a geometric sequence is given by Sn.
Consider the general geometric sequence a, ar, ar 2, ar 3, . . . ar n − 1.
Now, Sn = a + ar + ar 2 + ar 3 + . . . + ar n − 1.
Also, multiplying each term by r, rSn = ar + ar 2 + ar 3 + ar 4 + . . . + ar n.
So, rSn − Sn = −a + ar n since all the other terms cancel out.
So, Sn (r − 1) = a(r n − 1)
a(rn – 1)
Sn = ----------------------
r–1
This formula is useful if r < −1 or r > 1, for example, if r is 2, 10, 3.3, −4, −1.2.
By calculating Sn − rSn instead of rSn − Sn, as we did earlier, we obtain an alternative
form of the formula. That is,
S n – rS n = a – ar n
Sn( 1 – r ) = a( 1 – r n )
a(1 – rn)
S n = ----------------------
1–r
This formula is useful if r is in between −1 and 1 (shown as −1 < r < 1).
The sum of n terms, Sn, of a geometric sequence may be calculated using
a( rn – 1 )
Sn = ---------------------- if r < −1 or r > 1
r–1
or
a( 1 – rn )
Sn = ---------------------- if −1 < r < 1.
1–r

WORKED Example 11
Find the sum of the first 9 terms of the sequence 0.25, 0.5, 1, 2, 4, . . .
THINK WRITE/DISPLAY
1 Find the value of a. a = 0.25
t2 0.5- t 1-
2 Find the value of r by testing ratios of ---- = --------- ---3- = ------
the given terms. t 1 0.25 t 2 0.5
=2 =2
t t5 4
---4- = 2--- ---- = ---
t3 1 t4 2
=2 =2
a(r n – 1) 0.25 ( 2 – 1 )
9
3 Since r > 1, use S n = ---------------------- . S9 = ------------------------------
r–1 1
= 127.75
4 Write the answer. The sum of the first 9 terms is 127.75.
414 M a t h s Q u e s t M a t h s C Ye a r 1 1 f o r Q u e e n s l a n d

remember
remember
1. In a geometric sequence:
(a) the first term is a,
t
(b) the common ratio is r = ---2- .
t1
2. Given an unspecified sequence, establish whether it is geometric by testing all
t t t
terms for a common ratio, r = ---2- = ---3- = ---4- = …
t1 t2 t3
3. To find the terms of a geometric sequence use the following formula:
tn = ar n − 1
where tn is the nth term
a is the first term
r is the common ratio.
4. When the terms of a geometric sequence are added, a geometric series is
formed.
2, 4, 8, 16, 32, 64 is a finite geometric sequence.
2 + 4 + 8 + 16 + 32 + 64 is a finite geometric series.
5. The sum of n terms, Sn, of a geometric sequence may be calculated using
a(rn – 1)
S n = ---------------------- if r < –1 or r > 1, for example, r = –2, – 3--- , + 2 or + 1.5
r–1 2

or
a(1 – rn)
S n = ---------------------- if −1 < r < 1, for example, r = 0.2, 1
--- or −0.25.
1–r 8

a = 1, r = 4
9B
2 b
Geometric sequences c a = −1, r = 2
e a = 4, r = −3
f a = −6, r = −10
h a = 1.2, r = 2
WORKED 1 State which of the following are geometric sequences.
Example
xample i a = 7, r = 1--2-
6 a 2, 6, 12, 24, 48, . . . b 1, 4, 16, 64, 256, . . .
c −1, −2, −4, −8, −16, . . . d 1, 3, −9, 27, −81, . . . j a = --12- , r = --12-
e 4, −12, 36, −108, 324, . . . f −6, 60, −600, 6000, −60 000, . . .
1 b, c, e, f, h, i, j
g −5, 15, −25, 45, −85, . . . h 1.2, 2.4, 4.8, 9.6, 19.2, . . .
i 7, 3.5, 1.75, 0.875, 0.4375, . . . j 1--- , 1--- , 1--- , -----
1- -----
, 1- , . . .
2 4 8 16 32
WORKED
Example
xample 2 Look again at the geometric sequences found in question 1. Write down the values of
7 a and r.

WORKED 3 Find the value of the term specified for the given geometric sequences.
Example
xample
8 a Find the 10th term of the geometric sequence 2, 12, 72, 432, 2592, . . . 20 155 392
b Find the 18th term of the geometric sequence 8, 16, 32, 64, 128, . . . 1 048 576
c Find the 11th term of the geometric sequence 5, 15, 45, 135, 405, . . . 295 245
d Find the 10th term of the geometric sequence 2.3, 2.76, 3.312, 3.9744, . . . 11.867 494 81
e Find the 9th term of the geometric sequence −2, −8, −32, −128, −512, . . . −131 072
Chapter 9 Sequences and series 415
WORKED 4 Find the value of the term specified in each of the following geometric sequences.
Example
a The 2nd term of a geometric sequence is 6 and the 5th term is 162. Find the 10th term. 39 366
9
b The 2nd term of a geometric sequence is 6 and the 5th term is 48. Find the 12th term. 6144
c The 2nd term of a geometric sequence is 2 and the 5th term is 16. Find the 16th term. 32 768
d The 4th term of a geometric sequence is −32 and the 7th term is −256. Find the
14th term. −32 768
e The 4th term of a geometric sequence is −192 and the 7th term is −12 288. Find
the 12th term. −12 582 912
f The 3rd term of a geometric sequence is 36 and the 6th term is −972. Find the
12th term. −708 588
WORKED 5 Evaluate the following.
Example
10 a The first three terms of a geometric sequence are 5, 12.5 and 31.25. Which term
would be the first to exceed 50 000? 12th
b The first three terms of a geometric sequence are 3.2, 9.6 and 28.8. Which term
eBook plus
would be the first to exceed 1 000 000? 13th
Digital doc: c The first three terms of a geometric sequence are 5.1, 20.4 and 81.6. Which term
SkillSHEET 9.1 would be the first to exceed 100 000? 9th
Solving indicial
equations d The first three terms of a geometric sequence are 4.3, 9.46 and 20.812. Which
term would be the first to exceed 500 000? 16th
WORKED 6 a Find the sum of the first 12 terms of the geometric sequence 2, 6, 18, 54, 162, . . . 531 440
Example
11 b Find the sum of the first 7 terms of the geometric sequence 5, 35, 245, 1715,
12 005, . . . 686 285
c Find the sum of the first 15 terms of the geometric sequence 1.1, 2.2, 4.4, 8.8,
17.6, . . . 36 043.7
d Find the sum of the first 11 terms of the geometric sequence 3.1, 9.3, 27.9, 83.7,
251.1, . . . 274 576.3

7 multiple choice
There is a geometric sequence for which a is positive and r = −2. It is true to say that:
A only one term of the sequence is a positive number
B the 3rd term will be a negative number
C the 3rd term will be less than the 2nd term
D the 5th term will be greater than the 6th term
E the 4th term will be greater than the 3rd term.

8 multiple choice
The 12th term of the geometric sequence 21, 63, 189, 567, . . . is:
A 6804 B 413 343 C 1 240 029 D 3 720 087 E 5 931 980 229

9 multiple choice
The sum of the first 10 terms of the geometric sequence 2.25, 4.5, 9, 18, 36, . . . is
closest to:
A 1149.75 B 2301.75 C 5318.81 D 6648.51 E 8342.65

10 multiple choice
The 2nd term of a geometric sequence is −20 and the 5th is −1280. The sum of the
first 12 terms of the sequence is:
A −27 962 025 B −1 062 880 C −15 360 D 1 062 880 E 16 777 215
416 M a t h s Q u e s t M a t h s C Ye a r 1 1 f o r Q u e e n s l a n d

11 On the first day Jenny hears a rumour. On the second day, she
tells two friends. On the third day, each of these two friends tell
two of their own friends, and so on.
a Write the geometric sequence for the first five days of the
above real-life situation. 1, 2, 4, 8, 16
b Find the value of r. 2
c How many people are told of the rumour on the 12th day? 2048

20 million, 10 million,
5 million, 2 1--2- million, 12 Decay of radioactive material is modelled as a geometric sequence where r = 1
--- . If
1 1--4- million, 625 000, 2
there are 20 million radioactive atoms, write the first 7 terms of the sequence.
312 500
13 The number of cells of a micro-organism, after each process of cell division, can be
summarised as follows.
1, 2, 4, 8, 16
If the number of cells after each division continues to follow a geometric sequence, find:
a a rule for the number of cells after n divisions tn = 2n − 1
b the number of cells after 12 divisions. 2048

14 The takings at a new cinema each month are Month number Takings
recorded.
If the takings each month continue to 1 $10 000
follow a geometric sequence, find: 2 $8500
tn = 10 000 × 0.85n − 1 a a rule for the takings in month n
3 $7225
b the takings in week 9. $2724.91
Chapter 9 Sequences and series 417
15 A small town is renowned for spreading
rumours. All of its citizens are aware in Number of citizens
a short time of any new rumours. The Day number in the know
spread of the rumour can be summarised 1 1
in the table given at right.
If the number of citizens who have 2 6
been told the rumour each day con- 3 36
tinues to follow a geometric sequence,
find:
a a rule for the number of citizens in day n tn = 6n − 1
b the number of citizens told of the rumour by day 5 1296
c on which day all 4230 citizens will know of the rumour. 6

16 A ranger records the distance from a sand


Year Distance from
dune to the water’s edge at low tide over a
number water’s edge (m)
number of years.
1 60
2 64.8
3 69.984

If the distance from the water’s edge each year continues to follow a geometric
sequence, find:
a a rule for the distance from the water’s edge in year n tn = 60 × 1.08n − 1
b the distance from the water’s edge in year 6 (correct to 2 decimal places) 88.16 m
c in which year the distance from the water’s edge will exceed 100 m. 8th year
eBook plus
17 How many terms of the geometric sequence 600, 180, 54, 16.2, 4.86, . . . are required
Digital doc: for the sum to be greater than 855? Check with your teacher.
WorkSHEET 9.1
418 M a t h s Q u e s t M a t h s C Ye a r 1 1 f o r Q u e e n s l a n d

Applications of geometric sequences


Growth and decay of discrete variables is constantly found in real-life situations. Some
examples are increasing or decreasing populations and increase or decrease in financial
investments. Some of these geometric models are presented here.

WORKED Example 12
A city produced 100 tonnes of rubbish in the year 2008. Forecasts suggest that this may
increase by 2% each year. If these forecasts are true,
a what will be the city’s rubbish output in 2012?
b in which year will the amount of rubbish reach 120 tonnes?
c what was the total amount of rubbish produced by the city in the years 2008, 2009 and 2010?

THINK WRITE/DISPLAY
a 1 This is an example of a geometric a a = 100
sequence. Find the first term, a.
2 Determine the common ratio, r. Increase by 2%
The amount of rubbish increases by 2%, 1 + 2% = 1 + 0.02
that is, the original amount plus an extra r = 1.02
2%. Note that r ≠ 0.02.
3 Determine which term is represented by Year 2008 is the first term, so n = 1.
the amount of rubbish for the year 2012. Year 2009 is the second term, so n = 2.
Year 2012 is the fifth term, so n = 5.
4 Use tn = ar n − 1 to find the amount of t5 = 100 × 1.025 − 1
rubbish collected in the fifth year.
= 100 × 1.0824
= 108.24
5 Write your response. The amount of rubbish produced in the
fifth year, or 2012, will be 108.24
tonnes.

b Method 1: Using logarithms b


1 Use tn = ar n − 1 and tn = 120. 100(1.02)n − 1 = 120
(1.02)n − 1 = 1.2
2 Express 1.02 and 1.2 in terms of log 1.02n − 1 = log 1.2
logarithms with base 10. (n − 1) log 1.02 = log 1.2
log 1.2
n − 1 = -------------------
log 1.02
n − 1 = 9.207
n = 10.207
3 Write your answer. During the 11th year, that is, during
2018, the rubbish will have exceeded
120 tonnes.
Chapter 9 Sequences and series 419
THINK WRITE/DISPLAY
Method 2: Using trial and error
1 Use tn = ar n − 1 and tn = 120. 100(1.02)n − 1 = 120
(1.02)n − 1 = 1.2
2 Try various values of n. Let n = 10, (1.02)9 = 1.195
Let n = 11, (1.02)10 = 1.21
3 Write your answer. During the 11th year, that is, during
2018, the rubbish will have
exceeded 120 tonnes.
Method 3: Using a graphics calculator
1 Use tn = ar n − 1 to write the rule. tn = 100(1.02)n − 1
2 Locate the term that equals or exceeds 120.
Two methods are shown for this example.
Method A: Generating the terms of the
sequence
For the Casio fx-9860G AU
(a) Press MENU and select RUN-MAT. Press
OPTN then F1 (LIST) followed by F5
(Seq). Enter the rule (100 × 1.02n − 1)
followed by the variable name (n), the start
value (1), the end value (say, 20) and the
increment (1), each separated by a comma.
Press ) to close the set of brackets.
(b) Press EXE to display the list of the first 20
terms. Scroll down until you reach the term
that exceeds 120. (This screen is also
useful for part c.)

For the TI-Nspire CAS


(a) Open a new Lists & Spreadsheet
document. Press b, then 3: Data
followed by 1: Generate Sequence.
Enter the rule 100 × 1.02n − 1 with an
initial term of 100. Press e until OK
is highlighted.

(b) Press · to display the list of terms.


Scroll down until you reach the term that
exceeds 120.
(You may like to save this file for part c.)

Continued over page


420 M a t h s Q u e s t M a t h s C Ye a r 1 1 f o r Q u e e n s l a n d

THINK WRITE/DISPLAY
Method B: Solving an equation
Set up an equation to solve. 100(1.02)n − 1 = 120
For the Casio fx-9860G AU
Press MENU and select EQUA. Press
F3 (SOLV) to select F3: Solver.
(A previous equation and its solution
will be displayed.) Enter the required
equation in the first line and press
EXE . Press F6 (SOLV) to display the
solution.
For the TI-Nspire CAS
Open a new Calculator document. Press
b and select 3: Algebra followed by
1: Solve. Enter the equation to be solved
and then the variable, separated by a
comma. Press ) to close the set of
brackets and press ·.

3 Write your answer. The first term to exceed 120 tonnes is the
11th term or year 2018.

c 1 We need to find the sum of the first c


3 terms.

Method 1: Using the formula


a(rn – 1) 100 ( 1.02 3 – 1 )
Use S n = ---------------------- where n = 3. S3 = ------------------------------------
r–1 1.02 – 1
S3 = 306.04
Method 2: Using a graphics
calculator
Use the display generated by a graphics
calculator from Method A of part b
to add the first three terms.

(For the TI-Nspire CAS calculator, find


the sum of the numbers in cells A1 to
A3.)

100 + 102 + 104.04 = 306.04


2 Write your answer. The total output of rubbish for the years
2008, 2009 and 2010 will be 306.04
tonnes.
Chapter 9 Sequences and series 421
WORKED Example 13
A computer system decreases in value each year by 15% of the previous year’s value. Find
an expression for the value of the computer, which shall be referred to as Vn, after n years.
Its initial purchase price is given as V1 = $12 000.
THINK WRITE
1 This is a geometric sequence since there is a a = 12 000
15% decrease on the previous year’s value. r = 1 − 15%
Find a and r. = 1 − 0.15
Note: Since this is a decreasing value, r is a = 0.85
value less than 1.
2 We want an expression for the value after n Vn = 12 000 × (0.85)n − 1
years. Use tn = ar n − 1 which gives the value
of the nth term. Use Vn instead of tn.
3 Write your answer. The value of the computer is given by the
expression Vn = 12 000(0.85)n − 1.

Compound interest
Consider the case where a bank pays compound interest of 5% per annum on an
amount of $20 000. The amount is invested for 4 years and interest is calculated yearly.
Compound interest receives its name because the interest which is earned is paid
back into the account so that the next time interest is calculated, it is calculated on an
increased amount. There is a compounding effect on the money in the account.
If we calculated the amount in the account mentioned above each year, we would
have the following amounts.
Start $20 000
After 1 year $20 000 × 1.05 = $21 000
After 2 years $20 000 × 1.05 × 1.05 = $22 050
After 3 years $20 000 × 1.05 × 1.05 × 1.05 = $23 152.50
After 4 years $20 000 × 1.05 × 1.05 × 1.05 × 1.05 = $24 310.13
The amounts 20 000, 21 000, 22 050, 23 152.50, 24 310.13, . . . form a geometric
sequence where a = 20 000 and r = 1.05.
We need to be a little careful, however, in using the formula tn = ar n − 1 in calculating
compound interest. This is because the original amount in the account, that is, $20 000,
in terms of the geometric sequence would be referred to as t1 or a. In banking terms, t1
would represent the amount in the account after the first lot of interest has been calcu-
lated and added in.
To be clear and to be safe, it is best to use the following formula for compound interest.
A = PR n
r
where R = 1 + ---------
100
A = amount in the account, $
P = principal, $
r = interest rate per period (that is, per year or quarter etc.), %
n = the number of periods during the investment.
422 M a t h s Q u e s t M a t h s C Ye a r 1 1 f o r Q u e e n s l a n d

WORKED Example 14
Helen inherits $60 000 and invests it for 3 years in an account which pays compound
interest of 8% per annum compounding every 6 months.
a What will be the amount in Helen’s account at the end of 3 years?
b How much will Helen receive in interest over the 3-year period?
THINK WRITE
a 1 This is an example of compound interest. a P = 60 000
r n = 6 half years
Use A = PRn, where R = 1 + --------- . Interest r = 4% per half year
100
is calculated each 6 months so, over 3 years, 4
So, R = 1 + ---------
there are 6 periods: n = 6. Interest is 8% per 100
year or 4% per 6 months. = 1.04
So, r = 4%. A = PRn
= 60 000(1.04)6
= 75 919.14
2 Write your answer. At the end of 3 years, Helen will
have a total amount of $75 919.14.
b 1 Interest equals the amount in the account at b Interest = Total amount − Principal
the end of 3 years, less the amount in the = $75 919.14 − $60 000
account at the start of the investment. = $15 919.14
2 Write your answer. Amount of interest earned over
3 years is $15 919.14.

WORKED Example 15
Jim invests $16 000 in a bank account which earns compound interest at the rate of 12%
per annum compounding every quarter.
At the end of the investment, there is $25 616.52 in the account.
For how many years did Jim have his money invested?
THINK WRITE/DISPLAY
Method 1: Using logarithms
1 We know the value of A, P, r and R. A = 25 616.52
We need to find n using the compound P = 16 000
interest formula. r = 12
------
4
Note: There are 4 quarters per annum. r = 3% per quarter
and so R = 1 + --------
3-
= 1.03
100

2 Now substitute into A = PR . n


A = PR n

25 616.52 = 16 000(1.03)n
1.601 = 1.03n
3 Express 1.601 and 1.04 in terms of log 1.601 = log 1.03n
logarithms with base 10. So, log 1.601 = n × log 1.03
log 1.601
n = ----------------------
log 1.03
0.2044
n = ----------------
0.0128
Chapter 9 Sequences and series 423

THINK WRITE/DISPLAY
4 Round up the number of periods to 16 n = 15.92
to ensure the amount is reached.
5 Write your answer. It will take 16 periods where a period is
3 months. So, it will take 48 months or 4 years.
Method 2: Trial and error
1 We know the value of A, P, r and R. A = 25 616.52
We need to find n using the compound P = 16 000
interest formula. r = 12
------
4
r = 3% per quarter
and so R = 1 + --------
3-
100
= 1.03
Now, A = PRn
So, 25 616.52 = 16 000(1.03)n
1.601 = 1.03n
2 Try some different values of n. Let n = 5 1.035 = 1.159
Let n = 10 1.0310 = 1.344
Let n = 15 1.0315 = 1.558
Let n = 16 1.0316 = 1.605
3 Write your answer. It will take 16 periods where a period is
3 months. So, it will take 48 months or 4 years.
Method 3: Using a graphics calculator
1 Use tn = PR to write the rule for the P = 16 000
n

sequence. r = 12
------
4
= 3% per quarter
and so R = 1 + --------
3-
100
= 1.03
tn = 16 000(1.03)n
2 Locate the term that equals or exceeds For tn = 25 616.52,
25 616.52. 16 000(1.03)n = 25 616.52
You may wish to generate the sequence
and scroll down to the required term
(see page 400). Alternatively, an
equation can be set up to be solved.
This method is shown here.
For the Casio fx-9860G AU
Press MENU and select EQUA. Press
F3 (SOLV) to select F3: Solver. (A
previous equation and its solution will
be displayed.) Enter the required
equation in the first line and press
EXE . Press F6 (SOLV) to display the
solution. Continued over page
424 M a t h s Q u e s t M a t h s C Ye a r 1 1 f o r Q u e e n s l a n d

THINK WRITE/DISPLAY
For the TI-Nspire CAS
Open a new Calculator document. Press
b and select 3: Algebra followed by
1: Solve. Enter the equation to be solved
and then the variable, separated by a
comma. Press ) to close the set of
brackets and press ·.

3 Write your answer. Jim has money invested for 16 periods where a
period is 3 months.
So it will take 48 months or 4 years.

remember
remember
Geometric growth
1. Geometric growth or increase is expressed as a percentage increase.
2. Common ratio, r = 1 + % increase
3. r values are greater than 1, for example, an 8% increase gives r = 1.08.
Geometric decay
4. Geometric decay or decrease is expressed as a percentage decrease.
5. Common ratio, r = 1 − % decrease
6. r values are less than 1, for example, an 8% decrease gives r = 0.92.
Compound interest
7. The formula for compound interest is
A = PRn
r
where R = 1 + ---------
100
A = amount in the account, $
P = principal, $
r = interest rate per period (that is, per year or quarter etc.) as a percentage
n = the number of periods during the investment.

Applications of geometric
9C sequences
WORKED 1 A farmer harvests 4 tonnes of lucerne in his first year of production. In
Example
xample eBook plus
12
his business plan, he has estimated an annual increase of 6% on his
lucerne harvest.
Digital doc:
a According to this plan, how many tonnes of lucerne should he har- EXCEL
vest in his 7th year of production? 5.67 Spreadsheet
Sequences and series
b In which year will his harvest reach 10 tonnes? 17th year
c How much will he expect to harvest in the first 3 years? 12.7 tonnes
Chapter 9 Sequences and series 425
2 A taxi driver estimates that the cost of keeping her taxi on the road increases by 4.5%
each year. If the cost of keeping her taxi on the road in her first year of owning a taxi
was $1800:
a what was the cost in the 5th year? $2146.53
b during which year did costs exceed $2500? Year 9
c what were the total costs of keeping her taxi on the road in the first 3 years? $5646.65
WORKED 3 The population of a town is decreasing by 10% each year. Find an expression for the
Example
13
population of the town, which will be referred to as Pn. The population in the first
year, P1, was 10 000. Pn = 10 000 × (0.9)n − 1
WORKED 4 $13 000 is invested in an account which earns compound interest of 8%, com-
Example
14
pounding quarterly.
a After 5 years, how much is in the account? $19 317.32
b How much interest was earned in that period? $6317.32
5 The population of the newly established town of Alansford in its first year was 6000.
It is predicted that the town’s population will increase by 10% each year. If this were
to be the case, find:
a the population of the town in its 10th year 14 147
b in which year the population of Alansford would reach 25 000. 16th year
6 The promoters of ‘Fleago’ flea
powder assert that continued
application of the powder will
reduce the number of fleas on a
dog by 15% each week. At the end
of week 1, Fido the dog has 200
fleas left on him and his owner
continues to apply the powder.
a How many fleas would Fido be
expected to have on him at the
end of the 4th week? 123
b How many weeks would Fido
have to wait before the number
of fleas on him had dropped to
less than 50? 10 weeks
7 Young saplings should increase in
height by 9% each year under
optimum conditions. If a batch of
saplings which have been planted
out measure 2.2 metres in their
first year:
a how high should they be in
their 4th year? 2.85 m
Year 11 b in which year should they
exceed 5 metres in height?
8 A number of timber beams support a ramp. The first of the beams is 0.8 metres long
and each successive beam is 3% longer than the previous one.
a How long will the 7th beam in the line be? 0.96 m
b Which beam will be the first to exceed 2 metres in length? 32nd beam
426 M a t h s Q u e s t M a t h s C Ye a r 1 1 f o r Q u e e n s l a n d

9 multiple choice
A colony of ants is studied and the population of the colony in week 1 of the study is
800. If the population of the colony is expected to increase at the rate of 2% each week,
then the week in which the number of ants would exceed 1000 would be closest to:
A 6 B 10 C 13 D 26 E 32
10 A company exported $300 000 worth of manufactured goods in its first year of pro-
duction. According to the business plan of the company, this amount should increase
each year by 7.5%.
a How much would the company be expected to export in its 5th year? $400 640.74
b In which year would exports exceed $500 000? Year 9
2 636 196.56 c What is the total amount exported by the company in its first 7 years of operation?
11 Country football crowds have been decreasing by 3% each year since records of
crowd attendance were kept. If the number of people attending in the first year that
records were kept was 63 000 in a season:
a how many people attended in the 5th year? 55 773
b when did the number of people attending in a year drop below 50 000? 9th year
12 $10 000 is invested in an account which earns compound interest of 10% per annum.
Find the amount in the account after 5 years if the interest is compounded:
a yearly $16 105.10 b every 6 months $16 288.95
c quarterly $16 386.16 d monthly. $16 453.09
13 $20 000 is invested in an account earning compound interest of 10% per annum com-
pounding quarterly.
What is the amount in the account after:
a 1 year? $22 076.26 b 3 years? $26 897.78
c 5 years? $32 772.33 d 10 years? $53 701.28
14 $7000 is invested in an account which earns compound interest of 6% per annum
compounding monthly. After 3 years, how much is in the account? $8376.76
WORKED 15 In an account earning compound interest of 8% per annum compounding quarterly, an
Example
xample
15 amount of $6000 is invested. When the account is closed, there is $7609.45 in the
account. For how many years was the account open? 3 years
16 Sue earns 12% interest per annum compounding quarterly on her investment of
$40 000. For how many years would this investment need to operate for the amount to
rise to $50 670.80? 2 years
17 Helena receives $15 627.12 after closing an investment account which earned com-
pound interest of 9% per annum compounding every 6 months. If Helena originally
deposited $12 000 in the account, for how long was it in the account? 3 years
18 Todd receives $66 277.33 after having invested an inheritance of $60 000 in an
account earning compound interest of 12% per annum compounding monthly. For
how long did Todd have the money invested? 10 months
19 An amount of $14 500 is invested in an account attracting compound interest of
6% per annum compounding quarterly. After a certain time the interest earned in the
account is $1834.14. Find out for how long the amount had been invested. 2 years
Chapter 9 Sequences and series 427
Finding the sum of an infinite
geometric sequence
SLE 1: Establish the If you are 2 metres away from a wall and you move 1 metre
formula for the sum to n
terms of a geometric (or half-way) towards the wall and then move 1--- metre
2
progression, and hence the (or half-way again) towards the wall and continue to do
formula for the sum to
infinity of a geometric this, will you reach the wall? When will you reach the wall?
1– 1– 1–
progression; verify the Consider the following geometric sequence: 1m 2
m 4 8
formula by mathematical
induction. 1, 1--- , 1--- , 1--- , -----
1-
, . . . This is an infinite geometric sequence
2 4 8 16 1m
since it continues on with an infinite number of terms. 2m
Each term in the sequence is less than the previous term by
a factor of 1--- , that is, r = 0.5.
2
If we were to add n terms of this sequence together, we would have:
n
1 × ( 1 – 0.5 )
Sn = ---------------------------------
1 – 0.5
n
– 0.5
1------------------
=
0.5
n
1 0.5
= ------- − ---------
0.5 0.5
= 2 − 0.5n − 1
Consider 0.5n − 1 in the above equation. As n becomes very large, the term 0.5n − 1
becomes very small. Try this with your calculator.
Let n = 5, 0.5n − 1 = 0.54 = 0.0625; therefore, S5 = 2 − 0.0625 = 1.9375
Let n = 10, 0.5n − 1 = 0.59 = 0.001 95; therefore, S10 = 2 − 0.001 95 = 1.998 05
Let n = 20, 0.5n − 1 = 0.519 = 0.000 001 9; therefore, S20 = 2 − 0.000 001 9 = 1.999 998 1
We can see that as n becomes larger, 0.5n − 1 becomes smaller. If n were to approach
infinity (note that you can never reach infinity, you can only approach it), then the value
of 0.5n − 1 would approach zero. So, Sn = 2 − 0.5n − 1 would become S∞ = 2.
It is possible to generalise this in order to find the sum of an infinite geometric
sequence. We use the symbol S∞ which is referred to as the sum to infinity of a geo-
metric sequence.
The sum to infinity of a geometric sequence for which −1 < r < 1 is given by
a
S • = ----------- .
1–r

WORKED Example 16
Find the sum to infinity of the geometric sequence 2, 0.4, 0.08, 0.016, 0.0032, . . .
THINK WRITE
1 Find a and r. a=2
t
r = ---2-
t1
0.4
= -------
2
= 0.2
Continued over page
428 M a t h s Q u e s t M a t h s C Ye a r 1 1 f o r Q u e e n s l a n d

THINK WRITE
a
2 As r = 0.2 satisfies the condition S∞ = -----------
1–r
a
−1 < r < 1, use the formula S ∞ = ----------- . 2
= ----------------
1–r 1 – 0.2
2
= -------
0.8
= 2.5
3 Write your answer. The sum to infinity of the given sequence is 2.5.

WORKED Example 17
The sum to infinity of the geometric sequence is 6.25 and the value of r is 0.2. Write down
the first 4 terms of the sequence.
THINK WRITE
a a
1 Use the formula S ∞ = ----------- to find the S∞ = -----------
1–r 1–r
value of a. a
6.25 = ----------------
1 – 0.2
6.25 × 0.8 = a
a=5
2 a = 5 and r = 0.2. Use these to generate The first 4 terms of the sequence are
the terms. 5, 1, 0.2, 0.04.

WORKED Example 18
The sum to infinity of a geometric sequence is 15 and the value of a is 10. Write down the
first 4 terms of the sequence.

THINK WRITE
a a
1 Use the formula S ∞ = ----------- to find the S ∞ = -----------
1–r 1–r
value of r. Transpose the equation to 10
make r the subject. 15 = -----------
1–r
1–r = 10-
-----
15

r = 1 – 2--3-

r = 1
---
3

2 a = 10 and r = 1--- . Use these to generate The first 4 terms of the sequence are
3
the terms in the sequence. 10, 10
------ , 10
------ , 10
------ or 10, 3 1--- , 1 1--- , 10
------ .
3 9 27 3 9 27
Chapter 9 Sequences and series 429
Converting recurring decimals to fractions
We can use the sum to infinity formula to convert recurring decimals to fractions.

WORKED Example 19
.
Express 1.2 as a fraction.

THINK WRITE
. .
1 We need to express 1.2 as the sum of a 1.2 = 1.222 222 222 222 2 …
geometric sequence.
= 1 + 0.2 + 0.02 + 0.002 + 0.0002 + …
= 1 + (0.2 + 0.02 + 0.002 + 0.0002 + …)
2 The terms in the bracket form an a = 0.2
infinite geometric sequence where 0.02
r = ----------
a = 0.2 and r = 0.1. Use the formula 0.2
a = 0.1
S ∞ = ----------- .
1–r a
S∞ = -----------
1–r
. 0.2
0.2 = ----------------
1 – 0.1
. 0.2
0.2 = -------
0.9
Multiply both the numerator and . 2
0.2 0.2 = ---
denominator of ------- by 10 to eliminate 9
0.9
the decimal.
.
3 Express the final answer. So 1.2 = 1 + 2
---
9
= 1 2---
9

WORKED Example 20
Express 0.645 . . . as a fraction.
THINK WRITE
1 We need to express 0.645 as 0.645 = 0.645 454 5…
the sum of a geometric = 0.6 + 0.045 + 0.000 45 + 0.000 004 5 + …
sequence.
= 0.6 + ( 0.045 + 0.000 45 + 0.000 004 5 + … )
2 The terms in the bracket form a = 0.045
an infinite geometric sequence 0.00045
where a = 0.045 and r = 0.01. r = -------------------
0.045
= 0.01

Continued over page


430 M a t h s Q u e s t M a t h s C Ye a r 1 1 f o r Q u e e n s l a n d

THINK WRITE
a a
3 Use the formula S ∞ = ----------- . S ∞ = -----------
1–r 1–r
0.045
0.045 = -------------------
1 – 0.01
Multiply both the numerator and the 45
0.045 0.045 = ---------
denominator of ------------- by 1000 to eliminate 990
0.99
45
the decimal. 0.645 = 0.6 + ---------
990
6 -------- 45-
4 Express the final answer. = ------ +
10 990
594 45
= --------- + ---------
990 990
639
5 Write your answer. So 0.645 = ---------
990

WORKED Example 21
An injured rabbit attempts to crawl back to its hole. It moves 30 metres in the first hour,
21 metres in the second hour and 14.7 metres in the third hour and so on. If the rabbit
hole is 200 metres away, will the rabbit make it back to its hole?
THINK WRITE
1 Determine what sort of sequence we t1 = 30, t2 = 21 and t3 = 14.7
have. Now 21
------ = 0.7
30

and 14.7
---------- = 0.7
21
So, we have a geometric sequence where
a = 30, r = 0.7.
a 30
2 Find the value of S ∞ = ----------- . S∞ = ----------------
1–r 1 – 0.7
S∞ = 100
3 Write your answer. The rabbit will cover a total of 100 metres.
Since the rabbit hole is 200 metres away, the
rabbit won’t make it.

remember
remember
Finding the sum of an infinite geometric sequence:
1. For decreasing or decaying geometric series, the sum of an infinite number of
terms approaches a finite sum.
2. The sum to infinity of a geometric sequence for which −1 < r < 1 is given by
a
S ∞ = -----------
1–r
Chapter 9 Sequences and series 431
SLE 13: Use geometric progressions in situations involving the sum to infinity.

Finding the sum of an infinite


9D geometric sequence
WORKED 1 Find the sum to infinity of the following geometric sequences.
Example
16 a 50, 25, 12.5, 6.25, 3.125, . . 100
. b 20, 16, 12.8, 10.24, 9.192, . . .100
10 d 1, 1--- , 1--- , -----
1- -----
, 1- , . . . --32-
c 4, 2.4, 1.44, 0.864, 0.5184, . . . 3 9 27 81
e 1, 1--- , 1- --------
----- , 1 - , --------
1-
, ... 5--- f 2, 8- --------
----- , 32- , -----------
128- ---------------
, 512 - ,
. . . 3 1--3-
5 25 125 625 4 10 100 1000 10 000
g 3, −0.6, 0.12, −0.024, 0.0048, .2.5.. h −12, 7.2, −4.32, 2.592, −1.5552, −. .7.5 .
i 120, 48, 19.2, 7.68, 3.072, . . .200 j −50, 5, −0.5, 0.5, −0.05, . . . –45 -----
5
11
-

WORKED 2 Write down the first 3 terms of the geometric sequence for which:
Example
17 a r = 0.6 and S∞ = 25 10, 6, 3.6 b r = 0.2 and S∞ = 50 40, 8, 1.6
c r = 0.25 and S∞ = 8 6, 1.5, 0.375 d r = 0.9 and S∞ = 120 12, 10.8, 9.72
3 a 12.5, 6.25, 3.125 e r = −0.2 and S∞ = 3 1--- 4, −0.8, 0.16 f r = −0.5 and S∞ = 4 6, −3, 1.5
b 12.5, 9.375, 7.031 25 3
d 48, 28.8, 17.28
g r = −0.8 and S∞ = 5 9, −7.2, 5.76 h r = 0.2 and S∞ = −7.5 −6, −1.2, −0.24
i r = 0.6 and S∞ = −60 −24, −14.4, −8.64 j r = −0.3 and S∞ = −11 -----
7-
−15, 4.5, −1.35
13
WORKED 3 Write down the first 3 terms of the geometric sequence for which:
Example
18 a a = 12.5 and S∞ = 25 b a = 12.5 and S∞ = 50 c a = 6 and S∞ = 8 6, 1.5, 0.375
d a = 48 and S∞ = 120 e a = 2 4--- and S∞ = 3 2--- 2 --9- , -----
4 22- 22
27 81 , ------
9 3
WORKED 4 Express each of the following recurring decimals as fractions.
Example
5
. 8 . . 1 . 7
19, 20 a 0.5 --9- b 0.8. . --9- c 0.4. . 4--9- d 1.3 1 --3- e 3.7 3 --9-
14 55-
f 8.666 666 666 . . . 8 2--3- g 0.1 4 ----- -
99 h 0.5 7 ----- 99 i 0.529 j 1.321 1 321---------
999 262
---------
495
WORKED 5 A defiant child walks 10 metres towards his mother in the first minute, 4 metres in the
Example
second minute and 1.6 metres in the third minute. If the child continues to approach
21
in this same pattern, and if his mother is standing stationary, 20 metres from the
1
child’s initial position, will the child ever reach the mother? No — falls short by 3 --3- metres
6 A failing machine produces 35 metres of spouting in the first hour, 21 metres in the
second hour and 12.6 in the third hour. If this pattern continues and 280 metres of
spouting is required, how far short of the quota will the machine fall? 192.5 m
7 A nail penetrates 20 mm with the first hit of a hammer, 12 mm with the 2nd hit and
7.2 mm with the 3rd. If this pattern continues, will the 50 mm long nail ever be com-
pletely hammered in? Yes
8 A woman establishes a committee to raise money for a hospital. It raises $40 000 in
the 1st year, $36 800 in the 2nd year and $33 856 in the 3rd year. If the fundraising
continues in this pattern, how far short will they fall in raising $1 000 000? $500 000
9 An irrigation system sprays 25 mm of water over a crop in its 1st month, 20 mm in
the 2nd month and 16 mm in the 3rd month. If the crop requires 100 mm of water
during its lifetime, how far short or how far over is the irrigation system in supplying
the correct amount? 25 mm too much
10 A will of a recently deceased woman specifies how her money is to be donated to a
charity. Her total wealth of $12.5 million is to be donated for eternity with the first
donation of $1 million in the first year.
a What fraction of this first donation should be donated for the second year and sub-
sequent years? 23------
25
b Write the value of the donations for each of the first 5 years. $1 000 000, $920 000, $846 400,
c How much will be donated after 10 years? $7 070 144.32 $778 688, $716 392.96
432 M a t h s Q u e s t M a t h s C Ye a r 1 1 f o r Q u e e n s l a n d

Contrasting arithmetic and geometric


sequences through graphs
When discrete variables are presented graphically some distinct features may be evident.
This is especially so for discrete variables that have an arithmetic or geometric pattern.

Arithmetic patterns
Arithmetic patterns are distinguished by a straight line or a constant increase or decrease.

Value of term tn
Value of term tn

1 2 3 4 5 1 2 3 4 5
Term n Term n
d is positive d is negative
An increasing pattern or a positive A decreasing pattern or a negative
common difference gives an upward common difference gives a downward
straight line. straight line.

Geometric patterns
Geometric patterns are distinguished by a curved line or a saw form.
Value of term tn

Value of term tn

1 2 3 4 5 1 2 3 4 5
Term n Term n
An increasing pattern or a positive A decreasing pattern or a positive
common ratio greater than 1 (r > 1) fractional common ratio (0 < r < 1)
gives an upward curved line. gives a downward curved line.

+ +
Value of term tn

Value of term tn

1 2 3 4 5 1 2 3 4 5

– Term n – Term n
An increasing saw pattern occurs when A decreasing saw pattern occurs when
the common ratio is a negative value the common ratio is a negative fraction
less than −1 (r < −1). (–1 < r < 0).
Chapter 9 Sequences and series 433

WORKED Example 22
On the graph at right, the first 5 terms of a sequence 50

Value of term
are plotted. 40
State whether the sequence could be arithmetic or 30
geometric and give the value of a and the value of 20
either d or r. 10
0
0 1 2 3 4 5
Term number
THINK WRITE
1 Examine the difference between the There is a constant difference between each
value of each of the terms. In each case, successive term so the graph shows an
they are the same, that is, 10. arithmetic sequence.
2 Find a. Now, t1 = 40, so a = 40.
3 Find d. Now, t2 = 30 and t1 = 40.
So, d = t2 − t1 = −10.

WORKED Example 23
An amount of $10 000 is invested for 5 years and earns interest. Consider the following
two cases:
ii simple interest of 10% per annum
ii compound interest of 10% per annum compounding yearly.
a If the investment is earning simple interest, calculate the amount in the account at the
end of each of the 5 years.
b If the investment is earning compound interest, calculate the amount in the account at
the end of each of the 5 years.
c For each of the above cases, graph, on the same set of axes, the total assets over the
5 years. Use your graph or calculations to calculate the difference between the
accounts after 4 years.
THINK WRITE
a Calculate how much is in the account a After 1 year, amount in account
earning simple interest at the end of = 10 000 + 10% of 10 000
each of the 5 years. = 10 000 + 1000
= $11 000
After 2 years, amount in account
= 10 000 + 2 × 10% of 10 000
= 10 000 + 2000
= $12 000
After 3 years, amount in account
= $13 000
After 4 years, amount in account
= $14 000
After 5 years, amount in account
= $15 000
Continued over page
434 M a t h s Q u e s t M a t h s C Ye a r 1 1 f o r Q u e e n s l a n d

THINK WRITE
b Calculate the amount in the account b After 1 year, amount in account
earning compound interest at the end of = 10 000 × 1.11
each of the 5 years using A = PRn where = $11 000
r
R = 1 + --------- (that is, 1 + 10% = 1.1). After 2 years, amount in account
100 = 10 000 × 1.12
= $12 100
After 3 years, amount in account
= 10 000 × 1.13
= $13 310
After 4 years, amount in account
= 10 000 × 1.14
= $14 641
After 5 years, amount in account
= 10 000 × 1.15
= $16 105
c 1 Draw the graphs of the amount in the c
account earning simple interest and the 17 000
amount in the account earning Amount in account ($) 16 000 Compound interest
compound interest on the same set of 15 000
axes. 14 000
13 000
12 000 Simple interest
11 000
10 000
0 1 2 3 4 5
Number of years invested (n)

2 Use the values calculated for the end of Difference in amounts = 14 641 − 14 000
the fourth year. = $641
3 Write your answer. The compound interest account earned an
extra $641 in interest after 4 years.

Graphics Calculator tip! Comparison of simple and


compound interest
A graphics calculator can be used to compare the yearly amounts in an account earning
both simple interest and compound interest. Consider the scenario in Worked example
23 where the amounts are compared over the first 5 years. The steps are shown below.
Then investigate this scenario further by drawing the graphs for both cases over the first
10 years.

For the Casio fx-9860G AU


1. Press MENU and select STAT. Enter 1, 2, 3, 4 and 5
into List 1 to indicate the 5 years to be considered.
Enter a title for List 1 if you wish.
Chapter 9 Sequences and series 435
2. Press MENU and select RUN-MAT. Use the Seq
function to enter the yearly amounts for the simple
interest case. The rule is tn = 10 000 + n × 1000.
Press Æ then F1 (List) and 2 followed by
EXE to store these 5 terms as List 2.

3. Use the Seq function again to enter the yearly


amounts for the compound interest case. The rule is
tn = 10 000 × 1.1n. Press Æ then F1 (List) and
3 followed by EXE to store these 5 terms as
List 3.

4. Press MENU and select STAT. The amounts at the


end of each of the first 5 years are displayed. Enter
a title for each list if you wish.

5. Press F1 (GRPH) then F6 (SET). Press F1


(GPH1) to display the properties of Statgraph1.
This graph is to display the yearly amounts for the
simple interest case, so set Graph Type as a
scatterplot (press F1 (Scat)), XList as List 1 and
YList as List 2. Keep Frequency as 1 and adjust
the Mark Type if you wish.

6. Press EXE to accept the changes you have made to


StatGraph1. Press F6 (SET) again and then press
F2 (GPH2) to display the properties of
Statgraph2. This graph is to display the yearly
amounts for the compound interest case, so set
Graph Type as a scatterplot, XList as List 1 and
YList as List 3. Keep Frequency as 1 and adjust
the Mark Type so it is different from the first
graph.

7. Press EXE to accept the changes you have made to


StatGraph2. Press F4 (SEL) and ensure that both
StatGraph1 and StatGraph2 show DrawOn (and
StatGraph3 shows DrawOff).

8. Press F6 (DRAW) to display both graphs on the


same set of axes. Use the Trace function to
investigate the graphs. Press SHIFT F1 (TRCE)
and use the arrow keys to move to different points
on the graphs. The coordinates of the point and the
title of the graph are displayed.
436 M a t h s Q u e s t M a t h s C Ye a r 1 1 f o r Q u e e n s l a n d

For the TI-Nspire CAS


1. Open a new Lists & Spreadsheet document. Enter
1, 2, 3, 4 and 5 into Column A to indicate the 5 years
to be considered. Enter a title or variable name
(say, n) for this column.

2. We wish to generate terms of the sequence for the


simple interest case in Column B and the compound
interest case in Column C. Increase the width of
Columns B and C, and enter a title for each. Then
return to Column B.

3. Press b, then 3: Data followed by 1: Generate


Sequence. Enter the rule 10 000 + n × 1000 with a
first term of 11 000 and a maximum number of terms
of 5 to generate the yearly amounts for the simple
interest case. Press e until OK is highlighted and
then press ·.
Move to Column C and again press b, then 3: Data
and 1: Generate Sequence. Enter the rule
10 000 × 1.1n with a first term of 11 000 and a
maximum number of terms of 5 to generate the
yearly amounts for the compound interest case. Press
e until OK is highlighted and then press ·.

4. Press / I to insert a new document and select


2: Add Graphs & Geometry. Press / b and
select 4: Scatter Plot.

5. Click the a button to bring up an option screen and


highlight the variable n for the x entry.

6. Press · and then press e to move to the y entry.


Again click the a button, highlight the variable
simple and press ·. This has entered the
information for Scatterplot 1. Press d. Now press
b and select 4: Window followed by A: Zoom - Fit
to best display the graph for the simple interest case.
Chapter 9 Sequences and series 437
7. Now press e to highlight the x entry for Scatterplot
2. Enter the information for the compound interest case
in the same way. That is, select the variable n for x and
the variable compound for y. Press / G to hide the
function entry line. (Press / G again to bring it
back.) Use the Trace function to investigate the graphs.
Press b and select 5: Trace followed by 1: Graph
Trace and use the NavPad to move to different points
on the graphs to display the coordinates.

remember
remember
1. Arithmetic patterns

Value of term tn
Value of term tn

1 2 3 4 5 1 2 3 4 5
Term n Term n
d is positive d is negative
An increasing pattern or a positive A decreasing pattern or a negative common
common difference gives an upward difference gives a downward straight line.
straight line.
2. Geometric patterns
Value of term tn

Value of term tn

1 2 3 4 5 1 2 3 4 5
Term n Term n
An increasing pattern or a positive A decreasing pattern or a positive fractional
common ratio greater than 1 (r > 1) common ratio (0 < r < 1) gives a downward
gives an upward curved line. curved line.
+ +
Value of term tn
Value of term tn

1 2 3 4 5 1 2 3 4 5

– Term n – Term n
An increasing saw pattern occurs An decreasing saw pattern occurs when the
when the common ratio is a negative common ratio is a negative fraction
value less than −1 (r < −1). (–1 < r < 0).
438 M a t h s Q u e s t M a t h s C Ye a r 1 1 f o r Q u e e n s l a n d

Contrasting arithmetic and


9E geometric sequences through
graphs
WORKED 1 On the graph at right, the first five terms of a
Example
xample 5
sequence are plotted.

Value of term
22 4
State whether the sequence could be arithmetic or 3
geometric and give the value of a and the value of 2
eBook plus either d or r. Arithmetic sequence with a = 0 and d = 1 1
0
Digital doc: 0 1 2 3 4 5
EXCEL Spreadsheet Term number
Sequences and series

2 On the graph at right, the first five terms of a


12
sequence are plotted. 10

Value of term
State whether the sequence could be arithmetic or 8
geometric and give the value of a and the value of 6
either d or r. Arithmetic sequence with a = 10 and d = −2 4
2
0
0 1 2 3 4 5
Term number

3 On the graph at right, the first five terms of a


60
sequence are plotted. 50
Value of term

State whether the sequence could be arithmetic or 40


geometric and give the value of a and the value of 30
either d or r. Geometric sequence with a = 10 and r = 1.5 20
10
0
0 1 2 3 4 5
Term number

4 On the graph at right, the first five terms of a


Value of term

20
sequence are plotted. 15
State whether the sequence could be arithmetic or 10
geometric and give the value of a and the value of 5
either d or r. Geometric sequence with a = 20 and r = 0.5 0
0 1 2 3 4 5
Term number

5 On the graph at right, the first five terms of a


sequence are plotted. 5
Value of term

4
State whether the sequence could be arithmetic or
3
geometric and give the value of a and the value of
2
either d or r. Arithmetic sequence with a = 4 and d = −0.5 1
0
0 1 2 3 4 5
Term number
Chapter 9 Sequences and series 439
6 On the graph at right, the first five terms of a
sequence are plotted. 100

Value of term
State whether the sequence could be arithmetic 80
60
or geometric and give the value of a and the
n

40
8

value of either d or r.
20
7

Geometric sequence with a = 100 and r = 0.8


6

0
0 1 2 3 4 5
5

7 Draw a graph showing the first 8 terms of each Term number


4 3

of the following sequences:


2

a arithmetic, a = 7, d = 2 b geometric, a = 5, r = 1---


1

2
c arithmetic, a = –14, d = 3.5 d arithmetic, a = 32, d = –5
tn
15
10
5
0
–5
–10
–15

e geometric, a = 12, r = ------


10
f geometric, a = 0.02, r = 6
7c

8 multiple choice
8 n

On the graph at right, the first five terms of a 180


sequence are plotted.
7

150

Value of term
The sequence could be described by which one of
6

120
5

the following? 90
4

A Arithmetic sequence with a = 10 and d = 10 60


3

B Arithmetic sequence with a = 10 and d = 0.5 30


2

C Geometric sequence with a = 10 and r = 0.5 0


1

0 1 2 3 4 5
D Geometric sequence with a = 10 and r = 2
0
tn
6
5
4
3
2
1

Term number
E Geometric sequence with a = 10 and r = 1.5
7b

9 multiple choice 10
Value of term
8 n

On the graph at right, the first five 5


terms of a sequence are plotted. 0
7

–5 1 2 3 4 5 Term number
6

The sequence could be described


–10
5

by which one of the following?


4

A Arithmetic sequence with a = 10 and d = 20


3

B Arithmetic sequence with a = 10 and d = −20


2

C Geometric sequence with a = 10 and r = 10


1

D Geometric sequence with a = 10 and r = −20


0
tn
30
25
20
15
10
5

E Geometric sequence with a = 10 and r = −1


7a

WORKED 10 An amount of $5000 is invested for 3 years and earns interest. Consider the following
Example
23
two cases:
i simple interest of 10% per annum
Legend

ii compound interest of 10% per annum compounding yearly.


Un
Vn

a If the investment is earning simple interest, calculate the amount in the account at
3

the end of each of the 3 years. $5500, $6000, $6500


Year

b If the investment is earning compound interest, calculate the amount in the


2

account at the end of each of the 3 years. $5500, $6050, $6655


1

c On the same set of axes, plot points showing the amount in each account at the
end of each of the 3 years.
6500
6000
5500
5000

Amount ($) 11 An amount of $100 000 is invested for 3 years and earns:
a simple interest of 15% per annum
10 c

b compound interest of 15% per annum compounding yearly.


On the same set of axes, plot points showing the amount in each account at the end of
each of the 3 years.
440 M a t h s Q u e s t M a t h s C Ye a r 1 1 f o r Q u e e n s l a n d

12 An amount of $10 000 is invested for 3 years and earns:


a simple interest of 20% per annum
b compound interest of 20% per annum compounding yearly.
On the same set of axes, plot points showing the amount in each account at the end of
each of the 3 years.

13 On the same set of axes, sketch the graphs of the sequences with the rule un = 10n and
vn = 10 × 1.5n − 1. Use your graph to decide for how many of the first five terms un is
greater than vn. 3 terms

14 On the same set of axes, sketch the graphs of the sequences with the rule
un = 120 − 20n and vn = 100 × 0.8n − 1. Use your graph to decide for how many of the
first five terms un is greater than vn. 0 terms
15 On the same set of axes, sketch the graphs of the sequences with the rule un = 120n
and vn = 120 × 1.5n − 1. Use your graph to decide for how many of the first five terms
un is greater than vn. 3 terms
eBook plus
16 On the same set of axes, sketch the graphs of the sequences with the rule
Digital doc: un = 150 − 30n and vn = 100 × 0.5n − 1. Use your graph to decide for how many of the
WorkSHEET 9.2 first five terms un is greater than vn. 4 terms

Reward time
A teacher decides to give students lollies at an end-of-year party. He wishes to give
different amounts to each student based on worst to best attendance record.
12 1 He decides to give 1 lolly to the student with the highest absence, 2 lollies to the
18 000 student with the next highest absence, 4 to the next, then 8 and so on.
17 000 a If there are 30 students in the class, show how much each of the first
16 000 6 students will receive. 1, 2, 4, 8, 16, 32
Amount ($)

15 000 b How many lollies will the 30th student (with the best attendance record)
14 000 Legend expect to receive? 229
13 000 Un
Vn 2 The teacher realises this is not practical. He investigates a distribution
12 000
summarised as follows: 1, 3, 5, 7, . . .
11 000
a What type of pattern is this? Explain why. Arithmetic sequence
1 2 3 b How much will the 30th student expect to receive? 59
Year c How many lollies will the teacher need to give to the whole class? 900
d The value in c was still unacceptable to the teacher. What is the least total
possible number of lollies required if each student is to receive a whole lolly
using an arithmetic pattern. Show the pattern clearly. 465
3 The teacher decides against giving lollies. He now prefers to share one single
cake. He decides to give out the pieces using the rule: 1--- cake to first student,
1 2
--- cake to second student, 1--- to third student and so on.
4 8
a How many cakes will he need if he continues the pattern for all 30 students?
b How much of the cake will go to: 1
1-
i the 4th student? -----
16
1-
-----
64 ii the 6th student? 1
iii the last student? ------
2
30
-
There is some cake left. c Using this method the teacher realises there is something in it for him. Why?
Chapter 9 Sequences and series 441
F

Changing shape
In this investigation you will study how the area and perimeter of a closed shape
change as you systematically modify the shape.
E

Consider a square of side length 1 unit as shown in diagram (a) below. Consider now
making one change to each of the four sides as shown in the diagram where the side
length, AB (b), is transformed into the articulated side length, AB (c). The new side
D

length can be thought of as being composed of five sections all of equal length.

A B A B A B
C

1 unit 1 unit

(a) (b) (c)


B

1 Draw all the different shapes which can be made if all side lengths of the
original square undergo one transformation. (Hint: The new articulated edge
length can be oriented two ways on each of the four sides. One way is to have
the shape point out.)
A
1

2 For each of the five different shapes, calculate the area and perimeter of the
shape. Construct a table which details the area and perimeter of each of the
2
five shapes, arranging the values in ascending order. What pattern do you
Shape Area Perimeter
notice, if any?
5 20
F ---
9
------
3

E 7
--- 20
------ 3 Take the shape with the greatest area and now to each of the 20 sides apply
9 3
9 20
the transformation a second time to make a new shape of maximum area.
C, D --- ------
9 3 Draw a diagram depicting the new shape. Calculate the perimeter and the
11 20
B ------
9
------
3 area of this new shape.

Perimeter = 4 × ( 3--5- )x
13 20
A ------
9
------
3
4 We are now going to look for a pattern in the value of the perimeter and area

Area = 1 +
The area increases in an
arithmetic progression. The for successive applications of the transformation when applied to all sides.
perimeter stays constant. Let n be the number of times the transformation is applied to the original
square. For example, when n = 0 the area is 1 unit2 and the perimeter is 4
9
--4-

3 units. Investigate the value of the area and perimeter for n = 1 (question 2),
… + … 9--4- ( 9--5- )x – 1

2 (question 3), 3, 4, 5, . . .

5 Can you find a method for calculating the area and perimeter for n = x?

6 What happens to the value of the perimeter as n gets very large? Perimeter tends to infinity.

Perimeter = 20
------
3
× 5---
3 7 What happens to the value of the area as n gets very large? Area tends to 2.
= 100-
--------
9

Area = 1 + 4---
+5×4× 1
-----
8 If time permits you could extend this investigation to a new shape, for
9 2
9 example, an equilateral triangle or a cube. In the case of the cube you would
= 137
---------
81
investigate the surface area and volume.
442 M a t h s Q u e s t M a t h s C Ye a r 1 1 f o r Q u e e n s l a n d

Fibonacci Sequence
SLE 8: Investigate the way in Consider the two sequences:
which a sequence can be
defined recursively. 2, 4, 8, 16, 32, . . . and 2, 5, 11, 23, . . .
Both are sequences whose terms can be predicted using a simple formula. However, the
first sequence is considered fundamental and worthy of detailed study, whereas the
second is not.
The first sequence has numerous applications ranging from population growth to the
study of investments. The second sequence has no such applications.
Another sequence considered worthy of detailed examination is the Fibonacci Sequence:
1, 1, 2, 3, 5, 8, . . .
This sequence takes the name of a mathematician, Fibonacci (better known as
Leonardo of Pisa) who lived in Italy around AD 1200. The sequence arose as a result of
the problem Fibonacci posed:
If you had a pair of rabbits and it took a month for them to mature, and then pro-
duce a new pair after that, how many pairs would you have in twelve months?

At the start of the first month No. of pairs = 1

At the start of the second month No. of pairs = 1

At the start of the third month No. of pairs = 2

At the start of the fourth month No. of pairs = 3

At the start of the fifth month No. of pairs = 5


Chapter 9 Sequences and series 443
This process can be continued and will reveal that the number of pairs at any stage
can be obtained by adding the number of pairs at the two preceding stages. The
resulting sequence is:
1, 1, 2, 3, 5, 8, 13, . . .
In general then, if Fn stands for the number of pairs at the nth stage then
Fn + 1 = Fn + Fn − 1 with F0 = 1 and F1 = 1.
A sequence that specifies the general term by referring to the preceding terms is called
recursive.
An explict formula for the nth term is
n n
Fn = -------  ---------------- − -------  ----------------
1 1+ 5 1 1– 5
5  2  5 2 
The Fibonacci Sequence has been used to describe patterns in nature from shells to
pine cones.

Graphics Calculator tip! Generating terms in the


Fibonacci Sequence
A graphics calculator can be used to generate Fibonacci-type sequences in a number of
ways.

For the Casio fx-9860G AU


A spreadsheet approach is shown in the steps below.
1. Press MENU and select S-SHT to open a
spreadsheet. Consider the first column, A, as Fn − 1,
the second column, B, as Fn and the third column, C,
as Fn + 1. Enter the two starting values in cells A1 and
B1. That is, enter 1 in cell A1 and 1 in cell B1.

2. In cell C1, type the formula = A1 + B1. This will


generate the third term.

3. Press EXE to display the 3rd term in cell C1. Next


we need to repeat the last 2 terms (2nd and 3rd
terms) in row 2 so they can be added to produce the
next term. In cell A2, type the formula = B1 and
press EXE . In cell B2, type the formula = C1 and
press EXE .
4. Highlight cell C1 and press F2 (EDIT) followed by
F2 (COPY) to copy the formula. Move to cell C2
and press F1 (PASTE) to paste the formula into this
cell. This generates the 4th term of the sequence.

5. We can use this copy and paste technique to complete each column down to, say, the
10th row. For Column A, highlight cell A2, press EXIT to display the Copy option
and then F2 (COPY) to copy the formula. Move to cell A3 and press F1 (PASTE)
to paste the formula into this cell.
444 M a t h s Q u e s t M a t h s C Ye a r 1 1 f o r Q u e e n s l a n d

Now move to cell A4 and press F1 (PASTE) to paste the formula into this cell.
Continue repeating this paste step as you move down the cells of this column to row 10.
Repeat the same process for Column B and then for
Column C. Note that some cells may show a value of
0 until all the pasting has been completed. The first 10
terms of the Fibonacci sequence can be seen in
Column A.

For the TI-Nspire CAS


On this calculator, we can use the Generate Sequence function to display the terms of
any Fibonacci-style sequence. The rule for a Fibonacci Sequence is Fn + 1 = Fn + Fn − 1
with F0 = 1 and F1 = 1 (or any two initial values). An equivalent rule to this is
Fn = Fn − 1 + Fn − 2 with F1 = 1 and F2 = 1. The second version of the rule suits this
calculator as the rule needs to be expressed as u(n) =.
So the rule becomes u(n) = u(n−1) + u(n−2) with
u(1) = 1 and u(2) = 1.
1. Open a new Lists & Spreadsheet document. Press
b, then 3: Data and 1: Generate Sequence. Enter
the rule as u(n−1) + u(n−2) with initial terms of 1
and 1. Press e until OK is highlighted.

2. Press · to display the list of terms.

Note that you can also use a spreadsheet approach to


generate a Fibonacci Sequence. Follow the steps given
for the Casio fx-9860G AU calculator above and use
/C to copy a cell and /V to paste the formula
into another cell.

The golden ratio


An interesting, if mysterious, product of the Fibonacci Sequence is the ‘golden ratio’
(or golden mean).
For centuries, artists have recognised in painting and architecture that some shapes
are, of themselves, more appealing than others.
Chapter 9 Sequences and series 445
The ratio of height to width (or width to height) of rectangles that appeal to the eye is
called the ‘golden ratio’ and its value is 1.6 : 1. More exactly the ‘golden ratio’ is taken
to be
1+ 5
---------------- : 1
2
How is this number connected with the Fibonacci Sequence?
We can write the Fibonacci Sequence as follows:
F1, F2, F3, . . . Fn − 1, Fn, Fn + 1 where Fn is the nth term in the Fibonacci Sequence.
Fn + 1
Consider the ratio of successive terms, ------------ . As n increases, this fraction tends to a
Fn
fixed value, the ‘golden ratio’. For proof of this:
Fn + 1
Let ------------ = r.
Fn
By definition of the Fibonacci Sequence this means
Fn + Fn – 1
------------------------- = r
Fn
Fn Fn – 1
------ + ------------ = r
Fn Fn
1
1 + --- = r
r
r + 1 = r2
r2 − r − 1 = 0
1+ 5
r = ---------------- (We reject the negative solution.)
2
Thus the ratio of terms in the Fibonacci Sequence tends to the ‘golden ratio’.

Fibonacci numbers
1 Use your graphics calculator to develop a table of the Fibonacci Sequence.
10 946 a Find the ratio of the 20th and 21st terms.
---------------- = 1.618 034
6765 1+ 5
b How close is this ratio to the ‘golden ratio’? ---------------- = 1.618 034
2
2 Verify that the formula for the nth Fibonacci number is:
n n
Fn = -------  ---------------- − -------  ----------------
1 1+ 5 1 1– 5
5  2  5 2 
for the first 5 terms of the sequence. Only use your calculator as a last resort
and use techniques such as difference of squares to do the calculations. See Solutions Manual

3 Consider a population of rabbits where each pair produces another pair each
SLE 8: Investigate the way in See Solutions
which a sequence can be month. However, after a pair of rabbits produces a sixth pair, both rabbits die.
defined recursively. Manual
a Produce a spreadsheet to model the number of rabbits over time.
b Produce a formula (either explicit or recursive) to model the number of
rabbits in the population over time.
446 M a t h s Q u e s t M a t h s C Ye a r 1 1 f o r Q u e e n s l a n d

The Mandelbrot Set


Now that we have developed some of the key concepts of sequences let us return to the
development of the Mandelbrot Set.
SLE 8: Investigate the way in which
a sequence can be defined
recursively.
SLE 12: (Real and complex number
systems): Investigate fractals;
for example, the Mandelbrot Set,
using a CAS calculator.

First recall that complex numbers can be represented as points on the complex plane.
For each point c in the complex plane, consider the sequence formed by zn + 1 ← z2n + c
where zn and c are complex numbers and z0 = 0.
If the sequence converges to a point for a particular c then c belongs to the Mandel-
brot Set. If the sequence diverges, then c does not belong to the Mandelbrot Set.
If the sequence, for a particular c, does not diverge, this point in the complex plane
is coloured black. Otherwise this point is assigned a colour.
A two-colour (black and red) version of the Mandelbrot Set is arranged thus:

No —
colour the point black.
Does the sequence diverge?
Yes —
colour the point red.

Colour versions of the Mandelbrot Set are arranged thus:

No —
colour the point black.
Does the sequence diverge?
Yes — assign a colour
depending on how quickly
the divergence occurs.
Consider, for example, the complex number:
c = 0.2 + 0.3i
Chapter 9 Sequences and series 447
Using your graphics calculator or a spreadsheet it is possible to determine
eBook plus whether or not this value of c belongs to the Mandelbrot Set. (That is, whether the
sequence formed by zn + 1 ← z2n + c converges with a given value of c.) The
Digital doc:
EXCEL Spreadsheet following spreadsheet shows that 0.2 + 0.3i converges and hence belongs to the
Mandelbrot Set Mandelbrot Set.

For the complex number c = 0.8 + 0.1i we obtain the following spreadsheet screen.
The number diverges and hence does not belong to the Mandelbrot Set.

This spreadsheet approach can also be used with a graphics calculator. Use the for-
mulas shown in the spreadsheet above.
Alternatively, a CAS graphics calculator can be used to generate a sequence of the
type zn + 1 = (zn)2 + c and enable you to check for the convergence of the sequence for
given values of c.
448 M a t h s Q u e s t M a t h s C Ye a r 1 1 f o r Q u e e n s l a n d

For the TI-Nspire CAS


1. Open a new Calculator document (press /N and
select 1: Add Calculator). So that we can use
different values for c, we shall first define c. Press
b and select 1: Actions and 1: Define. Enter
c = 0.2 + 0.3i and press ·.

2. Insert a new Lists & Spreadsheet document (press


/I and select 3: Add Lists & Spreadsheet). To
generate a sequence, press b and select 3: Data
and 1: Generate Sequence. Notice that the formula
needs to be of the form u(n) =.

3. The formula zn + 1 = (zn)2 + c can also be written as


zn = (zn − 1)2 + c. So, in terms of u rather than z, the
formula to be entered into the calculator needs to be
u(n) = [u(n – 1)]2 + c.
Enter the formula in the first line and then press e
to move to the next line. Enter 0 and c as the initial
terms. Continue to press e until OK is highlighted.

4. Press · to generate the sequence. Resize column A


to clearly view the terms.

5. Use the NavPad to scroll down the column. You will


see that the terms converge to a value of
approximately 0.0792 + 0.3565i.

6. To try other values of c, return to the Calculator


document (press / and the left arrow on the
NavPad). Again, press b and select 1: Actions and
1: Define. This time define c as 0.8 + 0.1i.
Chapter 9 Sequences and series 449
7. Return to the Lists & Spreadsheet document (press
/ and the right arrow on the NavPad). The new
terms of the sequence will be automatically
displayed. Use the NavPad to scroll down the column
to check whether the terms converge or diverge.

Draw the Mandelbrot Set


In groups of four, use four sheets of 1-centimetre grid paper and proceed as
follows.
Assign each member of the group a portion of the Mandelbrot Set for which he or
she is responsible. Ensure each member uses the same scale so that the four
individual sheets can be collated to form a coherent picture.
For each square on the grid paper:
1 Calculate the coordinates of the midpoint.
2 Use a graphics calculator or a spreadsheet to determine if the sequence
zn + 1 ← z2n + c diverges with this value of c.
3 If it diverges, colour the square white, otherwise colour the square black.
4 Repeat until all squares are coloured.
450 M a t h s Q u e s t M a t h s C Ye a r 1 1 f o r Q u e e n s l a n d

summary
Arithmetic sequences
Recognising arithmetic sequences
• An arithmetic sequence is a sequence of numbers for which the difference between
successive terms is the same.
• Given an arithmetic sequence, identify:
the first term, a
and the common difference, d = t2 − t1 .
• Given an unspecified sequence, establish whether it is arithmetic by testing all
terms for a common difference: d = t2 − t1 = t3 − t2 = t4 − t3 = . . .
Finding the terms of an arithmetic sequence
tn = a + (n − 1) d where tn is the nth term.
The sum of a given number of terms of an arithmetic sequence
• A series is the sum of terms in a sequence.
• Sn is the sum of the first n terms in series, for example, S25 represents the sum of the
first 25 terms.
• Given a number of terms in a series, n, the first term, a, and the last term, l, use
n
S n = --- ( a + l ) .
2
• Given a number of terms in a series, n, the first term, a, and the common difference,
n
d, use S n = --- [ 2a + ( n – 1 )d ] .
2

Geometric sequences
Recognising geometric sequences
• A geometric sequence is a sequence of numbers for which the ratio of successive
terms is the same.
• Given a geometric sequence, identify:
the first term, a
t
and the common ratio, r = ---2- .
t1
• Given an unspecified sequence, establish whether it is geometric by testing all
t t t
terms for a common ratio, r = ---2- = ---3- = ---4- = …
t1 t2 t3
Finding the terms of a geometric sequence
• tn = ar n − 1
where tn is the nth term
a is the first term
r is the common ratio.
The sum of a given number of terms of a geometric sequence
• 2, 4, 8, 6, 32, 64 is a finite geometric sequence.
• 2 + 4 + 8 + 16 + 32 + 64 is a finite geometric series.
Chapter 9 Sequences and series 451
• The sum of n terms, Sn, of a geometric sequence may be calculated using
a(rn – 1)
S n = ---------------------- if r > 1 or r < –1 for example, r = –2, – 3--- , +2 or +4.5
r–1 2

or
a(1 – rn)
S n = ---------------------- if −1 < r < 1, for example, r = 0.2, 1--- or −0.25.
1–r 8

Geometric growth
• Growth or increase is expressed as a percentage increase.
• Common ratio, r = 1 + percentage increase
• r values are greater than 1, for example, an 8% increase gives r = 1.08.

Geometric decay
• Decay or decrease is expressed as a percentage decrease.
• Common ratio, r = 1 − percentage decrease
• r values are less than 1, for example, an 8% decrease gives r = 0.92.

Compound interest
• A = PRn
r
where R = 1 + ---------
100
A = amount in the account, $
P = principal, $
r = interest rate per period (that is, per year or quarter etc.) as a percentage
n = the number of periods during the investment.

Finding the sum of an infinite geometric sequence


• For decreasing or decaying geometric series, the sum of an infinite number of terms
approaches a finite sum.
• The sum to infinity of a geometric sequence for which −1 < r < 1 is given by
a
S ∞ = ----------- .
1–r

Contrasting arithmetic and geometric sequences through graphs


• Arithmetic patterns are distinguished by a straight line.
Value of term tn
Value of term tn

1 2 3 4 5 1 2 3 4 5
Term n Term n
d is positive d is negative
An increasing pattern or a positive A decreasing pattern or a negative
common difference gives an upward common difference gives a downward
straight line. straight line.
452 M a t h s Q u e s t M a t h s C Ye a r 1 1 f o r Q u e e n s l a n d

• Geometric patterns are distinguished by a curved line or a saw form.

Value of term tn

Value of term tn
1 2 3 4 5 1 2 3 4 5
Term n Term n
An increasing pattern or a positive A decreasing pattern or a positive
common ratio greater than 1 (r > 1) fractional common ratio (0 < r < 1) gives
gives an upward curved line. a downward curved line.

+ +
Value of term tn

Value of term tn
1 2 3 4 5 1 2 3 4 5

– Term n – Term n

An increasing saw pattern occurs A decreasing saw pattern occurs when


when the common ratio is a negative the common ratio is a negative fraction
value less than −1 (r < −1). (–1 < r < −0).
Chapter 9 Sequences and series 453

CHAPTER
review
1 multiple choice
9A
Which of the following could be the first 5 terms of an arithmetic sequence?
A 1, 2, 4, 8, 12, . . . B 3, 3, 6, 6, 9, . . . C 56, 57, 58, 59, 60, . . .
D −5, 5, 10, 15, 20, . . . E 1, 4, 9, 16, 25, . . .

2 multiple choice
9A
For the following sequence, −3.6, −2.1, −0.6, 0.9, 2.4, . . ., it is true to say that it is:
A an infinite sequence with a = −3.6 and d = −0.15
B an infinite sequence with a = −3.6 and d = 1.5
C an infinite sequence with a = −0.15 and d = −3.6
D a finite sequence with a = −0.15 and d = −3.6
E a finite sequence with a = −3.6 and d = 0.15.
3 For the sequences below, state whether or not they are an arithmetic sequence. If they are,
give the value of a and d. 9A
a −123, −23, 77, 177, 277, . . . Yes, a = −123, d = 100
b −5 1--- , −2 1--- , 3--- , 3 3--- , 6 3--- , . . . Yes, a = −5 --14- , d = 3
4 4 4 4 4

4 multiple choice
9A
For the arithmetic sequence, −1, 1, 3, 5, 7, . . . the value of a, the value of d and the rule for
the sequence are given respectively by:
A a = −1, d = 2, tn = −3 + 2n B a = −1, d = 2, tn = −3 − n
C a = 1, d = −1, tn = 2 − n D a = 2, d = −1, tn = 3 − n
E a = 2, d = −1, tn = 3 − n

5 multiple choice
9A
The 43rd term of the arithmetic sequence −7, 2, 11, 20, 29, . . . is:
A −327 B −243 C 371 D 380 E 387

6 multiple choice
9A
The 3rd term of an arithmetic sequence is 3.1 and the 7th term is −1.3. The value of the
31st term is:
A −153.7 B −27.7 C 28.9 D 38.3 E 157.9
7 If the second term of an arithmetic sequence is −5 and the 5th term is 16, which term in the
sequence is equal to 226? Term number 35 9A
8 Blood donations at a suburban location increased by 40 each year. If there were
520 donations in the first year: 9A
a how many donations were made in the 15th year? 1080
b what was the total number of donations made over those 15 years? 12 000
454 M a t h s Q u e s t M a t h s C Ye a r 1 1 f o r Q u e e n s l a n d

9 multiple choice
9A
The sum of the first 24 terms of the sequence −16, −12, −8, –4, 0, . . . is:
A 720 B 912 C 1344 D 1440 E 1488

10 multiple choice
9A
The first term of an arithmetic sequence is 14 and the 3rd is 8. The sum of the first 30 terms
of the sequence is:
A −1770 B −1095 C −885 D 1725 E 2190

11 multiple choice
9B
There is a geometric sequence for which a = 3 and r is a negative number. We can be certain
that:
A r is a fraction less than 1
B the 3rd term will be a positive number
C the 3rd term will be greater than the 1st term
D only one number in the sequence is positive
E the 4th term will be greater than the 3rd term.

12 multiple choice
9B
Which of the following is a geometric sequence?
A 2, −2, 2, −2, 2, . . . B 2, 4, 6, 8, 10, . . . C 1, 1--- , − 1--- , 1-
----- , − -----
1-
, ...
3 9 27 81
D 4, −4, 2, −2, 1, . . . E 100, 10, 0.1, 0.01, 0.001, . . .

13 For each of the sequences below, state whether or not they are a geometric sequence. If they
9B are, state the value of a and r.
a 5, 5--- , 5--- , 5--- , 5-
Yes, a = 5, r =
----- , ... 1---
2 4 8 16 2

b −700, −70, −7, 7, 70, .No


..

14 multiple choice
9B
The 19th term of the geometric sequence 3.25, 6.5, 13, 26, 52, . . . is:
A 425 984 B 851 968 C 1 703 936 D 41 978 243 E 3 272 883 098

15 multiple choice
9B
The 3rd term of a geometric sequence is 19.35 and the 6th is 522.45. The 12th term of the
sequence is:
A 16 539.15 B 417 629.75 C 126 955.35 D 380 866.05 E 1 142 598.15

16 multiple choice
9B
The first 3 terms of a geometric sequence are 2.25, 4.5, 9. The first term which would
exceed 1000 is:
A t9 B t10 C t11 D t12 E t13
Chapter 9 Sequences and series 455
17 The amount of garbage (in tonnes) collected in a particular area by the local council each
year is recorded over 3 successive years. 9B
Amount of
garbage
Year number (tonnes)
1 7.2
2 8.28
3 9.522

If the amount collected each year were to continue to follow a geometric sequence:
a Write down a rule for the amount of garbage, tn, which would be collected in the area in
year n. tn = 7.2 × 1.15n − 1
b How much garbage would be collected in the 8th year? (Answer correct to 2 decimal
places.) 19.2 tonnes
c In which year would the amount of garbage collected exceed 30 tonnes? Year 12

18 multiple choice
The sum of the first 10 terms of the geometric sequence 8, 4, 2, 1, 1--- , . . . is closest to:
9B
2
A 15 B 16 C 17 D 18 E 20

19 multiple choice
The 3rd term of a geometric sequence is 0.9 and the 6th is 7.2. The sum of the first 12 terms
9B
of the sequence is closest to:
A −2 B 122 C 921 D 4122 E 8190
20 How many terms of the geometric sequence 164, 131.2, 104.96, 83.968, 67.1744, . . . are
required for the sum to exceed 800? 17 9B
21 multiple choice
A tree increases in height each year by
9C
5%. If it was 1.2 m high in its 1st year,
in its 6th year its height would be closest
to:
A 1.53 m
B 1.61 m
C 5.5 m
D 9.11 m
E 3750 m

22 multiple choice
Profits in a company are projected to increase by 8% each year. If the profit in the first year
9C
was $60 000, in which year could a profit in excess of $100 000 be expected?
A year 6 B year 7 C year 8 D year 9 E year 10
456 M a t h s Q u e s t M a t h s C Ye a r 1 1 f o r Q u e e n s l a n d

23 Anya invests $25 000 in an account earning


9C compound interest of 10% per annum
compounding quarterly. $33 622.22
a Find the amount in the account after 3 years.
b Find how long it would take to have
$40 965.41 in her account. 5 years

24 multiple choice
9D The sum to infinity of the geometric sequence
1, 4--- , 16 64- 256
------ , -------- , --------- , . . . is:
5 25 125625
1
A --- B 5--- C 1 4--- D 4 E 5
5 4 5

25 multiple choice
9D The first term of the geometric sequence for which r = −0.5 and S∞ = 5 1--- is:
3
A −1 B −2 1--- C 2 2--- D 8 E 10 2---
3 3 3

9D 26 Express 3.7 as a fraction. 3 7--9-

27 The batteries in a toy soldier are running down. The toy soldier marches 50 cm in the first
9D minute, 30 cm in the second minute, 18 cm in the next and so on. By how much does the toy
soldier fall short of marching 1.5 m? 25 cm

28 multiple choice
9E The first five terms of a sequence are plotted
300
on the graph at right.
250
Value of term

The sequence could be described by which of the


200
following?
150
A Arithmetic sequence with a = 50 and d = 25 100
B Arithmetic sequence with a = 50 and d = 0.5 50
C Geometric sequence with a = 50 and r = 0.5
0
D Geometric sequence with a = 50 and r = 1.5 0 1 2 3 4 5
E Geometric sequence with a = 50 and d = 2 Term number

29 multiple choice
9E The first five terms of a sequence are plotted
on the graph at right. 15
The sequence could be described by which of 10
Value of term

the following? 5
A Arithmetic sequence with a = 10 and d = −5 0
1 2 3 4 5 Term number
B Arithmetic sequence with a = 10 and d = −0.5 –5
C Geometric sequence with a = 10 and r = 5 –10
D Geometric sequence with a = 10 and r = −5 –15
30
E Geometric sequence with a = 10 and d = 5 80
70
60
30 On the same set of axes, sketch the graph of the sequence with the rule:
9E
Amount ($)

50

a un = 10n 40

b vn = 10 × 2n − 1
30
Legend
20 Vn = 10 × 2n – 1
10 Un = 10n

1 2 3 4
Term number (n)
Chapter 9 Sequences and series 457
Modelling and problem solving
1 A newly established quarry produces crushed rock for the building of roads and freeways.
The amount of crushed rock, in tonnes, it produces increases by 3 1--- tonnes each month and its
2
production for the first 3 months of operation is shown below.

Month Crushed rock produced (tonnes)


1 8
2 11.5
3 15

a Write down the amount of crushed rock produced in the 4th month. 18.5 tonnes
b Write down a rule for tn, the amount of crushed rock produced in month n, expressed in
terms of n, the nth month. tn = 4.5 + 3.5n
c Write down the amount of crushed rock produced in the 60th month. 214.5 tonnes
d During which month will the amount of crushed rock coming from the quarry exceed
100 tonnes? 28th month
e The local council has ordered that after a total of 3050 tonnes of crushed rock has been
extracted from the quarry, an environmental impact survey must be completed. After how
many months will that happen? 40 months
458 M a t h s Q u e s t M a t h s C Ye a r 1 1 f o r Q u e e n s l a n d

2 The amount of crushed rock produced each month at a second quarry is shown below.

Month Crushed rock produced (tonnes)


1 10
2 11
3 12.1

Given that production at this quarry increases geometrically, find:


a the common ratio, r 1.1
b a rule for the amount of crushed rock produced, tn, in tonnes, expressed in terms of the
number of months, n tn = 10 × 1.1n − 1
c the amount of crushed rock produced in the 5th month 14.641 tonnes
d in which month the amount of crushed rock produced exceeds 30 tonnes 13th month
e the total amount of crushed rock produced by the quarry in its first year of operation. 213.84 tonnes

3 During its first month of production, the second quarry produced more crushed rock than the
first quarry. In the months after that, however, the first quarry produced more crushed rock
than the second quarry. 24th month
After how many months does the second quarry produce more than the first quarry again?

4 One bank offers a simple interest rate of 5% per annum on an investment of $100. For the
same investment, another bank offers 5% interest compounded annually. When will the value
of the investment earning compound interest be twice as much as the value of the investment
eBook plus earning simple interest? 35 years
Digital doc:
Test Yourself
Chapter 9
10
Permutations
and
combinations

syllabus reference
Core topic:
Structures and patterns

In this chapter
10A The addition and
multiplication principles
10B Factorials and
permutations
10C Arrangements involving
restrictions and like
objects
10D Combinations
10E Applications of
permutations and
combinations
10F Pascal’s Triangle, the
binomial theorem and the
pigeonhole principle
460 M a t h s Q u e s t M a t h s C Ye a r 1 1 f o r Q u e e n s l a n d

• permutations and combinations and their use in purely mathematical and life-related situations

Introduction
Combinatorics deals with determining the number of ways in which activities or events
may occur.
The study of combinatorics provides ways of
answering questions such as:
1. How many doubles teams can be selected from
a group of 6 volleyball players?
2. From a group of 4 candidates, in how many
ways can a class captain and deputy class cap-
tain be selected?
3. How many different outfits can be chosen from
3 skirts and 5 tops?
4. If a Lotto ticket consists of a choice of 6 numbers
from 45, how many different tickets are there?
5. How many different car number plates of 3 digits
and 3 letters can be made using the digits 0 to 9
and the letters A, B and C?

The addition and multiplication


principles
To count the number of ways in which an activity can occur, first make a list. Let each
outcome be represented by a letter and then systematically list all the possibilities.
Consider the following question:
In driving from Melbourne to Sydney I can take any one of 4 different roads and in
driving from Sydney to Brisbane there are 3 different roads I can take. How many
different routes can I take in driving from Melbourne to Brisbane?
To answer this, let M1, M2, M3, M4 stand for the 4 roads from Melbourne to Sydney
and B1, B2, B3 stand for the 3 roads from Sydney to Brisbane.
Use the figure to systematically list the roads:
M1B1, M1B2, M1B3 M1
M2 B1
M2B1, M2B2, M2B3 M B
M3 S B2
M3B1, M3B2, M3B3 M4 B3
M4B1, M4B2, M4B3
Hence, there are 12 different ways I can drive from Melbourne to Brisbane.
In the above example it can be argued logically that if there are 4 ways of getting
from Melbourne to Sydney and 3 ways of getting from Sydney to Brisbane then there
are 4 × 3 ways of getting from Melbourne to Brisbane.
This idea is formalised in the multiplication principle.
The multiplication principle should be used when there are two operations or events
(say, A and B), where one event is followed by the other.
It states:
If there are n ways of performing operation A and m ways of performing
operation B, then there are n × m ways of performing A and B.
Note: In this case ‘and’ means to multiply.
Chapter 10 Permutations and combinations 461
A useful technique for solving problems based on the multiplication principle is to
use boxes. In the example above we would write

1st 2nd
4 3

The value in the ‘1st’ column represents the number of ways the first operation —
the trip from Melbourne to Sydney — can be performed.
The value in the ‘2nd’ column stands for the number of ways the second operation
— the trip from Sydney to Brisbane — can be performed.
To apply the multiplication principle you multiply the numbers in the lower row of boxes.
Now consider a different situation, one in which the two operations do not occur one
after the other.
I am going to travel from Melbourne to either Sydney or Adelaide. There are 4 ways
of travelling from Melbourne to Sydney and 3 ways of travelling from Melbourne to
Adelaide.
How many different ways can I travel to either Sydney or Adelaide?
It can be seen from the figure that there 1
are 4 + 3 = 7 ways of completing the 1
2
A 2 S
journey. This idea is summarised in the M
3
3 4
addition principle.
The addition principle should be used when two distinct operations or events occur
in which one event is not followed by another.
It states:

If there are n ways of performing operation A and m ways of performing


operation B then there are n + m ways of performing A or B.
Note: In this case ‘or’ means to add.

WORKED Example 1
Two letters are to be chosen from A, B, C, D and E, where order is important.
a List all the different ways that this may be done.
b State the number of ways that this may be done.
THINK WRITE
a 1 Begin with A in first place and make a list of a AB AC AD AE
each of the possible pairs.
2 Make a list of each of the possible pairs with B BA BC BD BE
in the first position.
3 Make a list of each of the possible pairs with C CA CB CD CE
in the first position.
4 Make a list of each of the possible pairs with D DA DB DC DE
in the first position.
5 Make a list of each of the possible pairs with E EA EB EC ED
in the first position.
Note: AB and BA need to be listed separately as
order is important. Continued over page
462 M a t h s Q u e s t M a t h s C Ye a r 1 1 f o r Q u e e n s l a n d

THINK WRITE
b Method 1 b There are 20 ordered pairs.
Count the number of ordered pairs and answer
the question.
Alternatively, the multiplication principle could have
been used to determine the number of ordered pairs.
b Method 2 b
5 4
1 Rule up two boxes which represent the pair.
2 Write down the number of letters which may
be selected for the first box. That is, in first
place any of the 5 letters may be used.
3 Write down the number of letters which may be
selected for the second box. That is, in second place,
any of the 4 letters may be used.
Note: One less letter is used to avoid repetition.
4 Evaluate. 5 × 4 = 20 ways
5 Answer the question. There are 20 ways in which 2 letters
may be selected from a group of 5
where order is important.

A selection where order is important is called an arrangement.

WORKED Example 2
How many ways could an arrangement of 5 letters be chosen from A, B, C, D, E and F?
THINK WRITE
1 Instead of listing all possibilities, draw 5 boxes to
represent the 5 letters chosen.
Label each box on the top row as 1st, 2nd, 3rd, 4th
and 5th.
Note: The word arrangement implies order is
important.
2 Fill in each of the boxes showing the number of
ways a letter may be chosen.
(a) In the 1st box there are 6 choices for the first
letter.
(b) In the 2nd box there are 5 choices for the
second letter as 1 letter has already been used.
(c) In the 3rd box there are 4 choices for the third 1st 2nd 3rd 4th 5th
letter as 2 letters have already been used. 6 5 4 3 2
(d) Continue this process until each of the 5 boxes
is filled.
3 Use the multiplication principle as this is an No. of ways = 6 × 5 × 4 × 3 × 2
‘and’ situation. = 720
4 Answer the question. An arrangement of 5 letters may be
chosen 720 ways.
Chapter 10 Permutations and combinations 463
WORKED Example 3
One or two letters are to be chosen from 6 letters A, B, C, D, E and F. In how many ways
can this be done?
THINK WRITE
1 Determine the number of ways of choosing 1 letter. No. of ways of choosing 1 letter = 6.
2 Rule up two boxes for the first and second letters.
3 Determine the number of ways of choosing 2 letters
from 6. 1st 2nd
(a) In the 1st box there are 6 choices for the first letter. 6 5
(b) In the 2nd box there are 5 choices for the second
letter as 1 letter has already been used.
4 Use the multiplication principle (as this is an No. of ways of choosing 2 letters
‘and’ situation) to evaluate the number of ways of =6×5
choosing 2 letters from 6. = 30
5 Determine the number of ways of choosing 1 or 2 The number of ways of choosing
letters from 6 letters. Use the addition principle as 1 or 2 letters is 6 + 30 = 36.
this is an ‘or’ situation.
6 Answer the question. There are 36 ways of choosing 1 or 2
letters from 6.

WORKED Example 4
Jeannine’s restaurant offers its patrons a choice
of 3 entrees, 9 main courses and 4 desserts.
a How many choices of 3-course meals
(entree, main, dessert) are available?
b How many choices of entree and main
course are offered?
c How many choices of main course and
dessert are offered?
d How many choices of 2- or 3-course meals
are available (assuming that a main course
is always ordered)?
THINK WRITE
a 1 Rule up 3 boxes to represent each course — a
E M D
entree, main, dessert. Label each box on the
top row as E, M and D. 3 9 4
2 Determine the number of ways of choosing
each meal: entree = 3, main = 9, dessert = 4.
3 Use the multiplication principle (as this is an No. of choices = 3 × 9 × 4
‘and’ situation) to evaluate the number of = 108
choices of 3-course meals.
4 Answer the question. There are 108 choices of 3-course
meals.
Continued over page
464 M a t h s Q u e s t M a t h s C Ye a r 1 1 f o r Q u e e n s l a n d

THINK WRITE
b 1 Rule up 2 boxes to represent each course — b
entree, main. Label each box on the top row as E M
E and M. 3 9

2 Determine the number of ways of choosing


each meal: entree = 3, main = 9.
3 Use the multiplication principle (as this is No. of choices = 3 × 9
an ‘and’ situation) to evaluate the number = 27
of choices of entree and main courses.
4 Answer the question. There are 27 choices of entree and
main course.

c 1 Rule up 2 boxes to represent each course — c


main and dessert. Label each box on the top M D
row as M and D.
9 4

2 Determine the number of ways of choosing


each meal: main = 9, dessert = 4.
3 Use the multiplication principle (as this is No. of choices = 9 × 4
an ‘and’ situation) to evaluate the number = 36
of choices of main course and dessert.
4 Answer the question. There are 36 choices of main
course and dessert.

d 1 Determine the number of ways of choosing d The number of ways of choosing


2- or 3-course meals, assuming that a main 2- or 3-course meals, assuming
course is always ordered. that a main course is always
Use the addition principle as this is an ordered, is:
‘or’ situation. 108 + 27 + 36 = 171
2 Answer the question. There are 171 ways of choosing
2- or 3-course meals, assuming
that a main course is always
ordered.

remember
remember
1. The multiplication principle should be used when there are two operations or
events (say, A and B) where one event is followed by the other. It states:
If there are n ways of performing operation A and m ways of performing
operation B, then there are n × m ways of performing A and B.
2. The addition principle should be used when two distinct operations or events
occur in which one event is not followed by another. It states:
If there are n ways of performing operation A and m ways of performing
operation B, then there are n + m ways of performing A or B.
3. A selection where order is important is called an arrangement.
Chapter 10 Permutations and combinations 465
The addition and
10A multiplication principles
WORKED 1 Two letters are to be chosen from A, B and C, where order is important.
Example
1 a List all the different ways that this may be done. a AB BA CA AC BC CB
b State the number of ways that this may be done. 6
2 BG GB YB RB
BY GY YG RG 2 List all the different arrangements possible for a group of 2 colours to be chosen from
BR GR YR RY B (blue), G (green), Y (yellow) and R (red).
3 ACB BAC CAB 3 List all the different arrangements possible for a group of 3 letters to be chosen from
ABC BCA CBA A, B and C.
WORKED 4 a In how many ways can an arrangement of 2 letters be chosen from A, B, C, D, E,
Example
2
F and G? 42
b In how many ways can an arrangement of 3 letters be chosen from 7 different
letters? 210
c In how many ways can an arrangement of 4 letters be chosen from 7 different
letters? 840
d How many different arrangements of 5 letters can be made from 7 letters? 2520
5 a A teddy bear’s wardrobe consists of 3 different hats, 4 different shirts and 2 dif-
ferent trousers. How many different outfits can the teddy bear wear? 24
b A surfboard is to have 1 colour on its top and a different colour on its bottom. The
3 possible colours are red, blue and green. In how many different ways can the
surfboard be coloured? 6
c A new computer system comes with a choice of 3 keyboards, 2 different monitors
and 2 different mouse attachments. With these choices, how many different
arrangements are possible? 12
d Messages can be sent by placing 3 different coloured flags in order on a pole. If
the flags come in 4 colours, how many different messages can be sent? 24
e A yacht race has a field of 12 competitors. In how many different ways can first,
second and third place be filled by these 12 yachts? 1320
WORKED 6 a One or 2 letters are to be chosen from the letters A, B, C, D, E, F and G. In how
Example
3
many ways can this be done? 49
b Two or 3 letters are to be chosen from the letters A, B, C, D, E, F and G. In how
many ways can this be done? 252
c How many 1- or 2-digit numbers can be made using the digits 1, 3, 5 and 7 if no
digit can be used more than once? 16
7 Nadia is in a race with 10 other girls.
a If we are only concerned with the first, second and third placings, in how many
ways can:
i Nadia finish first? 90
ii Nadia finish second? 90
b In how many ways can Nadia finish first or second? 180
8 White Wolf is a horse in a race with 7 other runners. If we are concerned only with
the first, second and third placings, in how many ways can White Wolf finish first or
second or third? 126
466 M a t h s Q u e s t M a t h s C Ye a r 1 1 f o r Q u e e n s l a n d

9 multiple choice
There are 12 people on the committee at the local softball club. In how many ways
can a president and a secretary be chosen from this committee?
A 2 B 23 C 132 D 144 E 66
10 multiple choice
Phone numbers consist of 8 digits. The first must be a 9. The second digit can be a 3,
4, 5 or 8. There are no restrictions on the remaining digits.

How many different telephone numbers are possible?


A 4320 B 499 999 C 4 000 000 D 4 999 999 E 10 000 000

11 multiple choice
A TV station runs a cricket competition called Classic Catches. Six catches, A to F,
are chosen and viewers are asked to rank them in the same order as the judges. The
number of ways in which the six catches can be ranked:
A 1 B 6 C 30 D 720 E 128
12 The local soccer team sells ‘doubles’ at each of their games to raise money. A
‘double’ is a card with 2 digits on it representing the score at full time. The card with
the actual full time score on it wins a prize. If the digits on the cards run from 00 to
99, how many different tickets are there? 100
13 Marcus has a briefcase that has a 4-digit security code. He remembers that the first
number in the code was 9 and that the others were 3, 4 and 7 but forgets the order of
the last 3 digits. How many different trials must he make to be sure of unlocking the
briefcase? 6
14 Julia has a briefcase that has two 4-digit locks. She
remembers that she used the digits 1, 3, 5 and 7 on the
left lock and 2, 4, 6 and 8 on the right lock, but can not
remember the order. What is the maximum number of
trials she would need to make before she has opened
both the left lock and the right lock? 48
Chapter 10 Permutations and combinations 467
15 How many different 4-digit numbers can be made from the numbers 1, 3, 5 and 7 if
the numbers can be repeated (that is 3355 and 7777 are valid)? 256
16 How many 4-digit numbers can be made from the numbers 1, 3, 5, 7, 9 and 2 if the
numbers can be repeated? 1296
17 How many 4-digit numbers can be made from the numbers 1, 3, 5, 7, 9 and 0 if the
numbers can be repeated? (Remember — a number cannot start with 0.) 1080
18 How many numbers less than 5000 can be made using the digits 2, 3, 5, 7 and 9 if
repetition is not permitted? 133
19 A combination lock has 3 digits each from 0 to 9.
a How many combinations are possible? 1000 8
9
0
c 271 371 471 2 3
The lock mechanism becomes loose and will open if the 1
272 372 472 4 5 6

1
digits are within one either side of the correct digit. For

4
0
273 373 473

7 8
2 3
281 381 481 example if the true combination is 382 then the lock will

5
9
9 0 1

2
282 382 482 open on 271, 272, 371, 493 and so on. 8 6
7
283 383 483 b How many combinations would unlock the safe? 27 5 3
291 391 491 4
292 392 492 c List the possible combinations that would open the
293 393 493 lock if the true combination is 382.
WORKED 20 Hani and Mary’s restaurant offers its patrons a choice of 4 entrees, 10 main courses
Example
4
and 5 desserts.
a How many choices of 3-course meals (entree, main, dessert) are available? 200
b How many choices of entree and main course are offered? 40
c How many choices of main course and dessert are offered? 50
d How many choices of 2- or 3-course meals are available (assuming that a main
course is always ordered)? 290
21 Jake is able to choose his work outfits from the following items of clothing: 3 jackets,
7 shirts, 6 ties, 5 pairs of trousers, 7 pairs of socks and 3 pairs of shoes.
a How many different outfits are possible if he wears one of each of the above
items? (He wears matching socks and shoes.) 13 230
b If Jake has the option of wearing a jacket and each of the above items, how many
different outfits are possible? Explain your answer. 17 640 Jack may wear 13 230 outfits with a
jacket or 4410 outfits without a jacket.
Therefore he has a total of 17 640
outfits to choose from. The assumption
made with this problem is that no item
Factorials and permutations of clothing is exactly the same; that is,
none of the 7 shirts is exactly the same.
Factorials
The Physical Education department is to display 5 new trophies along a shelf in the
school foyer and wishes to know how many ways this can be done.
Using the multiplication principle from the previous section, the display may be
done in the following way:

Position 1 Position 2 Position 3 Position 4 Position 5


5 4 3 2 1

That is, there are 5 × 4 × 3 × 2 × 1 = 120 ways.


468 M a t h s Q u e s t M a t h s C Ye a r 1 1 f o r Q u e e n s l a n d

Depending on the number of items we have, this method could become quite time
consuming.
In general when we need to multiply each of the integers from a particular number,
n, down to 1, we write n!, which is read as n factorial.
Hence: 6! = 6 × 5 × 4 × 3 × 2 × 1
= 720
8! = 8 × 7 × 6 × 5 × 4 × 3 × 2 × 1
= 40 320
n! = n × (n − 1) × (n − 2) × (n − 3) × . . . × 3 × 2 × 1

1. The number of ways n distinct objects may be arranged is n! (n factorial) where:


n! = n × (n − 1) × (n − 2) × (n − 3) × . . . × 3 × 2 × 1
That is, n! is the product of each of the integers from n down to 1.
2. A special case of the factorial function is: 0! = 1.

Graphics Calculator tip! Calculating


factorials
The following steps show how to calculate 12! using a graphics calculator.

For the Casio fx-9860G AU


1. From the MENU select RUN-MAT. Press OPTN
s

and then F6 ( ) for more options.

2. Press F3 (PROB) and you will be able to see the


function x!. Enter 12 and press F1 (x!). Press
EXE to display the value of 12!.

For the TI-Nspire CAS


1. Open a new Calculator document. Enter 12 and
then press / and k to access the symbol palette.
Use the arrow keys to highlight the factorial symbol
(exclamation mark).

2. Press · to insert the factorial symbol into the


entry line. Press · to display the value.
Chapter 10 Permutations and combinations 469
WORKED Example 5
Evaluate the following factorials.

18! 9! ( n – 1 )!
a 7! b 13! c -------- d ----- e -------------------
5! 3! ( n – 3 )!

THINK WRITE
a 1 Write 7! in its expanded form and a 7! = 7 × 6 × 5 × 4 × 3 × 2 × 1
evaluate. = 5040
2 Verify the answer obtained using
the factorial function on the
calculator.

b 1 Write 13! in its expanded form b 13! = 13 × 12 × 11 × 10 × 9 × 8 × 7 × 6 × 5 × 4


and evaluate. ×3×2×1
= 6 227 020 800
2 Verify the answer obtained using
the factorial function on the
calculator.

c 1 Write each factorial term in its c 8! × 7 × 6 × 5 × 4 × 3 × 2 × 1-


expanded form. ----- = 8-------------------------------------------------------------------
5! 5×4×3×2×1
2 Cancel down like terms. =8×7×6
3 Evaluate. = 336
4 Verify the answer obtained using
the factorial function on the
calculator.

d 1 Write each factorial term in its d 9! × 8 × 7 × 6 × 5 × 4 × 3 × 2 × 1-


----- = 9----------------------------------------------------------------------------
expanded form. 3! 3×2×1
2 Cancel down like terms. =9×8×7×6×5×4
3 Evaluate. = 60 480
4 Verify the answer obtained using
the factorial function on the
calculator.

e 1 Write each factorial term in its e ( n – 1 )!


expanded form. ------------------
( n – 3 )!
( n – 1 ) ( n – 2 ) ( n – 3 ) ( n – 4 ) ×… × 3 × 2 × 1
= -----------------------------------------------------------------------------------------------------------
(n – 3)(n – 4) × … × 3 × 2 × 1
2 Cancel down like terms. = (n − 1)(n − 2)
470 M a t h s Q u e s t M a t h s C Ye a r 1 1 f o r Q u e e n s l a n d

In parts c, d and e of Worked example 5, there was no need to fully expand each fac-
torial term.
8! 8 × 7 × 6 × 5!
The factorial ----- could have first been simplified to -------------------------------- and then the
5! 5!
5! terms cancelled.
9! 9 × 8 × 7 × 6 × 5 × 4 × 3!
The factorial ----- could have first been simplified to ------------------------------------------------------------ and
3! 3!
then the 3! terms cancelled.
( n – 1 )! ( n – 1 ) ( n – 2 ) ( n – 3 )!
The factorial ------------------ could have first been simplified to ----------------------------------------------------- and
( n – 3 )! ( n – 3 )!
then the (n − 3)! terms cancelled.

Permutations
The term permutation is often used instead of the term arrangement and in this section
we begin by giving a formal definition of permutation.
Previously, we learned that if you select 3 letters from 7 where order is important,
the number of possible arrangements is:

1st 2nd 3rd


7 6 5

The number of arrangements = 7 × 6 × 5


= 210
7 × 6 × 5 × 4! 7!
This value may also be expressed in factorial form: 7 × 6 × 5 = -------------------------------- = -----
4! 4!
Using more formal terminology we say that in choosing 3 things from 7 things
where order is important the number of permutations is 7P3 = 7 × 6 × 5. The letter P is
used to remind us that we are finding permutations.

The number of ways of choosing r things from n distinct things is given by the rule:
n
Pr = n × (n − 1) × . . . × (n − r + 1)
n ¥ ( n – 1 ) ¥ º ¥ ( n – r + 1 ) ( n – r )!
= ------------------------------------------------------------------------------------------
( n – r )!
n!
= ------------------
( n – r )!

The definition of nPr may be extended to the cases of nPn and nP0.
n
Pn represents the number of ways of choosing n objects from n distinct things.
n
Pn = n × (n − 1) × (n − 2) × . . . × (n − n + 1)
= n × (n − 1) × (n − 2) × . . . × 1
= n!
From the definition:
n!
n
Pn = ------------------
( n – n )!
n!
= -----
0!
n!
Therefore, equating both sides, we obtain: n! = ----- .
0!
This can occur only if 0! = 1.
Chapter 10 Permutations and combinations 471
n!
P0 = ------------------
n
( n – 0 )!
n!
= -----
n!
=1
In summary, the two special cases are:
1. nPn = n!
2. nP0 = 1

WORKED Example 6
a Calculate the number of permutations for 6P4 by expressing it in expanded form.
b Write 8P3 as a quotient of factorials and hence evaluate.
THINK WRITE
a 1 Write down the first 4 terms beginning with 6. a 6P4 = 6 × 5 × 4 × 3
2 Evaluate. = 360
b n!
1 Write down the rule for permutations. b nPr = ------------------
( n – r )!
2 Substitute the given values of n and r into the 8!
permutation formula.
8
P3 = ------------------
( 8 – 3 )!
8!
= -----
5!
40 320
3 Use a calculator to evaluate 8! and 5! = ----------------
120
4 Evaluate. = 336

WORKED Example 7
The netball club needs to appoint a president, secretary and treasurer. From the
committee 7 people have volunteered for these positions. Each of the 7 nominees is happy
to fill any one of the 3 positions. In how many different ways can these positions be filled?
THINK WRITE
n!
1 Write down the rule for permutations. Pr = ------------------
n
( n – r )!
Note: Order is important, so use permutations.
7!
2 Substitute the given values of n and r P3 = ------------------
7
( 7 – 3 )!
into the permutation formula.
7!
= -----
4!
5040
3 Use a calculator to evaluate 7! and 4! = ------------
24
4 Evaluate. = 210
5 Answer the question. There are 210 different ways of filling
the positions of president, secretary
and treasurer.
472 M a t h s Q u e s t M a t h s C Ye a r 1 1 f o r Q u e e n s l a n d

Graphics Calculator tip! Calculating


permutations
To find the number of permutations of n objects taken r at a time, we need to calculate
n
Pr .
The following steps show how to calculate 5P3 using a graphics calculator.

For the Casio fx-9860G AU


1. From the MENU select RUN-MAT. Press OPTN and
s
then F6 ( ) for more options.

2. Press F3 (PROB) and you will be able to see the


function nPr. Enter 5 (for n), then press F2 (nPr)
and enter 3 (for r). Press EXE to display the value.

For the TI-Nspire CAS


1. Open a new Calculator document. Press k to
access the catalog and press 1 for the list of
functions. Scroll down to select nPr(. You can do this
more quickly by first pressing N.

2. Press · and then complete the entry line to obtain


nPr(5,3). Press · to display the value.
(Alternatively, you can use the letter keys to type npr
followed by ( directly into the calculator screen
instead of accessing it from the catalog. Press
NPR followed by (, then complete the entry line
and press ·.)

Arrangements in a circle
Consider this problem: In how many different ways can 7 people be seated, 4 at a time,
on a bench?
By now you should quickly see the answer: 7P4 = 840.
Let us change the problem slightly: In how many different ways can 7 people be
seated, 4 at a time, at a circular table?
The solution must recognise that when people are seated A D
on a bench, each of the following represents a different B D A C
arrangement: C B
ABCD BCDA CDAB DABC C B
However, when sitting in a circle, each represents the
D B C A
same arrangement.
In each case B has A on the left and C on the right. A D
Chapter 10 Permutations and combinations 473
We conclude that the number 7P4 gives 4 times the number of arrangements of 7 people
7P
in a circle 4 at a time. Therefore, the number of arrangements is --------4- = 210 .
4
In general, the number of different ways n people can be seated, r at a time, in a
circle is:
nP
---------r
r

WORKED Example 8
How many different arrangements are possible if, from a group of 8 people, 5 are to be
seated at a round table?

THINK WRITE

1 Write down the rule for the number of nP


arrangements in a circle. --------r
r
2 Substitute the given values of n and r 8P
into the formula. = --------5-
5
6720
3 Use a calculator to evaluate 8P5. = ------------
5
4 Evaluate. = 1344
5 Answer the question. The number of ways of seating 5 from a group
of 8 people at a round table is 1344.

remember
remember
1. (a) The number of ways n distinct objects may be arranged is n! (n factorial)
where:
n! = n × (n − 1) × (n − 2) × (n − 3) × . . . × 3 × 2 × 1
(b) 0! = 1
(c) 1! = 1
2. (a) The number of different arrangements (permutations) when r things are
chosen from n things and order is important is given by the rule nPr ,
where:
n n!
Pr = ------------------
(b) P = n!
n ( n – r )!
n
(c) nP0 = 1
3. The number of different ways n people can be seated, r at a time, in a
circle is:
nP
--------r
r
474 M a t h s Q u e s t M a t h s C Ye a r 1 1 f o r Q u e e n s l a n d

10B Factorials and permutations


1 Write each of the following in expanded form.
a 4! 4 × 3 × 2 × 1 b 5! c 6! d 7! 7 × 6 × 5 × 4 × 3 × 2 × 1
5×4×3×2×1 6×5×4×3×2×1
WORKED 2 Evaluate the following factorials.
Example
5a, b a 4! 24 b 5! 120 c 6! 720 d 10! 3 628 800
e 14! f 9! 362 880 g 7! 5040 h 3! 6
8.717 829 12 × 1010
WORKED 3 Evaluate the following factorials.
Example
5c, d 9! 10! 7! 6!
a ----- 3024 b -------- 151 200 c ----- 840 d ----- 720
5! 4! 3! 0!
1 1
------------------------------------- -------------------------------------------------------
WORKED 4 Evaluate the following factorials. n(n – 1)(n – 2) ( n + 2 ) ( n + 1 )n ( n – 1 )
Example
5e n! ( n + 3 )! (------------------
n – 3 )! ( n – 2 )!
a ------------------ b ------------------- c d -------------------
( n – 5 )! ( n + 1 )! n! ( n + 2 )!
n(n − 1)(n − 2)(n − 3)(n − 4)
(n + 3)(n + 2)
WORKED 5 Calculate each of the following by expressing it in expanded form.
Example eBook plus
6a a 8P2 8 × 7 = 56 b 7P5 c 8P7
7 × 6 × 5 × 4 × 3 = 2520 8 × 7 × 6 × 5 × 4 × 3 × 2 = 40 320 Digital docs:
WORKED 6 Write each of the following as a quotient of factorials and hence SkillSHEET 10.1
Example Calculating nPr
6b evaluate.
9! 5! 18! EXCEL Spreadsheet
a 9P6 ---- - = 60 480 b 5P2 ----- = 20 - = 1 028 160
c 18P5 ------- Permutations
3! 3! 13!

7 Use your calculator to find the value of:


a 20P6 27 907 200 b 800P2 639 200 c 18
P5 1 028 160
WORKED 8 A soccer club will appoint a president and a vice-president. Eight people have
Example
7 volunteered for either of the two positions. In how many different ways can these
positions be filled? 56
9 The school musical needs a producer, director, musical director and script coach. Nine
people have volunteered for any of these positions. In how many different ways can
the positions be filled? (Note: One person cannot take on more than 1 position.) 3024

Producer Director Coach Musical


Director

10 There are 14 horses in a race. In how many different ways can the 1st, 2nd and 3rd
positions be filled? 2184
11 There are 26 horses in a race. How many different results for 1st, 2nd, 3rd and 4th can
occur? 358 800
12 A rowing crew consists of 4 rowers who sit in a definite order. How many different
crews are possible if 5 people try out for selection? 120
Chapter 10 Permutations and combinations 475
WORKED 13 How many different arrangements are possible if, from a group of 15 people, 4 are to
Example
be seated in a circle? 8190
8
14 A round table seats 6 people. From a group of 8 people, in how many ways can 6 people
be seated at the table? 3360
15 At a dinner party for 10 people all the guests were seated at a circular table. How
many different arrangements were possible? 362 880
16 At one stage in the court of Camelot, King Arthur and 12 knights would sit at the
round table. If each person could sit anywhere how many different arrangements were
possible? 479 001 600

17 multiple choice
Which one of the following permutations cannot be calculated?
A 1000P100 B 1P0 C 8P8 D 4P8 E 5
P4

18 multiple choice
The result of 100! is greater than 94!.
Which of the following gives the best comparison between these two numbers?
A 100! is 6 more than 94!
B 100! is 6 times bigger than 94!
C 100! is about 6! times bigger than 94!
D 100! is about 10 000 more than 94!
E 100! is 100P6 times bigger than 94!

For questions 19 to 21 show your answers in the form nPr and then evaluate.
19 In how many ways can the letters of the word TODAY be arranged if they are used
once only and taken: 5
P4 = 120
a 3 at a time? 5P3 = 60 b 4 at a time? c 5 at a time? 5P5 = 120
20 In how many ways can the letters of the word TUESDAY be arranged if they are used
once only and taken: 7
P4 = 840
7
P3 = 210 a 3 at a time? b 4 at a time? c 7 at a time? 7P7 = 5040
21 In how many ways can the letters of the word NEWTON be arranged if they are used
once only and taken 6 at a time, assuming:
a the first N is distinct from the second N? 6P6 = 720
b there is no distinction between the two Ns?
6P
--------6- = 360
2
476 M a t h s Q u e s t M a t h s C Ye a r 1 1 f o r Q u e e n s l a n d

Arrangements involving restrictions


and like objects
A 5-letter word is to be made from 3 As and 2 Bs. How many different arrangements
can be made?
If the 5 letters were all different, it would be easy to calculate the number of arrange-
ments. It would be 5! = 120. Perhaps you can see that when letters are repeated, the
number of different arrangements will be less than 120. To analyse the situation let us
imagine that we can distinguish one A from another. We will write A1, A2, A3, B1 and
B2 to represent the 5 letters.
As we list some of the possible arrangements we notice that some are actually the
same, as shown in the table.

A1A2B1A3B2 A1A2B2A3B1 Each of these 12


arrangements is the
A1A3B1A2B2 A1A3B2A2B1 same — AABAB.
A2A1B1A3B2 A2A1B2A3B1

A2A3B1A1B2 A2A3B2A1B1

A3A1B1A2B2 A3A1B2A2B1
A3A2B1A1B2 A3A2B2A1B1

B2A1A2B1A3 B1A1A2B2A3 Each of these 12


arrangements is the
B2A1A3B1A2 B1A1A3B2A2 same — BAABA.
B2A2A1B1A3 B1A2A1B2A3

B2A2A3B1A1 B1A2A3B2A1

B2A3A1B1A2 B1A3A1B2A2

B2A3A2B1A1 B1A3A2B2A1

The number of repetitions is 3! for the As and 2! for the Bs. Thus, the number of
5!
different arrangements is ---------------- .
3! × 2!
The number of different ways of arranging n things made up of groups of
indistinguishable things, n1 in the first group, n2 in the second group and so on is:
n!
-------------------------------------- .
n 1!n 2!n 3!ºn r!

Note: If there are elements of the group which are not duplicated, then they can be
considered as a group of 1. It is not usual to divide by 1!; it is more common to show
only those groups which have duplications.
Chapter 10 Permutations and combinations 477
WORKED Example 9
How many different arrangements of 8 letters can be made from the word PARALLEL?
THINK WRITE
1 Write down the number of letters in the The word PARALLEL contains 8 letters;
given word. therefore n = 8.
2 Write down the number of times any of The letter A is repeated twice; therefore n1 = 2.
the letters are repeated. The letter L is repeated 3 times; therefore n2 = 3.
3 Write down the rule for arranging n!
------------------------------------
groups of like things. n 1!n 2!n 3!…n r!
4 Substitute the values of n, n1 and n2 into 8!
= ----------------
the rule. 2! × 3!
5 Evaluate each of the factorials. 40 320
= ----------------
2×6
6 Simplify the fraction. 40 320
= ----------------
12
7 Evaluate. = 3360
8 Answer the question. 3360 arrangements of 8 letters can be made
from the word PARALLEL.

WORKED Example 10
How many different arrangements of 7 counters can be made from 4 black and
3 white counters?

THINK WRITE
1 Write down the total number of There are 7 counters in all; therefore n = 7.
counters.
2 Write down the number of times any of There are 3 white counters; therefore n1 = 3.
the coloured counters are repeated. There are 4 black counters; therefore n2 = 4.
3 Write down the rule for arranging n!
------------------------------------
groups of like things. n 1!n 2!n 3!…n r!
4 Substitute the values of n, n1 and n2 into 7!
= ----------------
the rule. 3! × 4!
5 Evaluate each of the factorials. 5040
= ---------------
6 × 24
6 Simplify the fraction. 5040
= ------------
144
7 Evaluate. = 35
8 Answer the question. Thirty-five different arrangements can be made
from 7 counters, of which 3 are white and 4 are
black.
478 M a t h s Q u e s t M a t h s C Ye a r 1 1 f o r Q u e e n s l a n d

WORKED Example 11
A rowing crew of 4 rowers is to be selected, in order from the first seat to the fourth seat,
from 8 candidates. How many different arrangements are possible if:
a there are no restrictions?
b Jason or Kris must row in the first seat?
c Jason must be in the crew, but he can row anywhere in the boat?
THINK WRITE
n!
a 1 Write down the permutation formula. a n P r = ------------------
Note: 4 rowers are to be selected from ( n – r )!
8 and the order is important.
8!
2 Substitute the given values of n and r
8
P4 = ------------------
into the permutation formula. ( 8 – 4 )!
8!
= -----
4!
40 320
3 Use a calculator to evaluate 8! and 4!. = ----------------
24
4 Evaluate. = 1680
5 Answer the question. There are 1680 ways of arranging 4 rowers
from a group of 8.
b 1 Apply the multiplication principle b No. of arrangements
since two events will follow each = no. of ways of filling the first seat × no. of
other; that is, Jason will fill the first ways of filling the remaining 3 seats.
seat and the remaining 3 seats will be = 2 × nPr
filled in 7 × 6 × 5 ways or Kris will
fill the first seat and the remaining
3 seats will be filled in 7 × 6 × 5 ways.
J 7 6 5 or K 7 6 5
2 Substitute the values of n and r into = 2 × 7P3
the formula and evaluate. = 2 × 210
= 420
3 Answer the question. There are 420 ways of arranging the 4 rowers
if Jason or Kris must row in the first seat.
c 1 Apply the addition principle, since c No. of arrangements =
Jason must be in either the first, No. of arrangements with Jason in seat 1
second, third or fourth seat. The + No. of arrangements with Jason in seat 2
remaining 3 seats will be filled in + No. of arrangements with Jason in seat 3
7 × 6 × 5 ways each time. + No. of arrangements with Jason in seat 4.
J 7 6 5 + 7 J 6 5 +
7 6 J 5 + 7 6 5 J
2 Substitute the values of n and r into No. of arrangements
the formula. = 1 × 7P3 + 1 × 7P3 + 1 × 7P3 + 1 × 7P3
= 4 × 7P3
= 4 × 210
3 Evaluate. = 840
4 Answer the question. There are 840 ways of arranging the 4
rowers if Jason must be in the crew of 4.
Chapter 10 Permutations and combinations 479
WORKED Example 12
a How many permutations of the letters in the word COUNTER are there?
b In how many of these do the letters C and N appear side by side?
c In how many permutations do the letters C and N appear apart?
THINK WRITE
a 1 Count the number of letters in the a There are 7 letters in the word COUNTER.
given word.
2 Determine the number of ways the The 7 letters may be arranged 7! = 5040
7 letters may be arranged. ways.
3 Answer the question. There are 5040 permutations of letters in the
word COUNTER.
b 1 Imagine the C and N are ‘tied’ together b Let C and N represent 1 unit.
and are therefore considered as 1 unit. They may be arranged 2! = 2 ways.
Determine the number of ways C and
N may be arranged: CN and NC.
2 Determine the number of ways 6 things Six things may be arranged 6! = 720 ways.
can be arranged.
Note: There are now 6 letters: the ‘CN’
unit along with O, U, T, E and R.
3 Determine the number of permutations in The number of permutations = 2 × 6!
which the letters C and N appear together.
4 Evaluate. = 2 × 720
= 1440
5 Answer the question. There are 1440 permutations in which the
letters C and N appear together.
c 1 Determine the total number of c Total number of arrangements = 7!
arrangements of the 7 letters. = 5040
2 Write down the number of Arrangements with C and N together = 1440
arrangements in which the letters C and
N appear together, as obtained in a.
3 Determine the difference between the The number of arrangements = 5040 − 1440
values obtained in steps 1 and 2. = 3600
Note: The number of arrangements in
which C and N are apart is the total
number of arrangements less the number
of times they are together.
4 Answer the question. The letters C and N appear apart 3600 times.

remember
remember
1. The number of different ways of arranging n things made up of groups of
indistinguishable things, n1 in the first group, n2 in the second group and so
n!
on is: ------------------------------------ .
n 1!n 2!n 3!…n r!
2. When restrictions apply to arrangements, use the multiplication and addition
principles as well as nPr .
480 M a t h s Q u e s t M a t h s C Ye a r 1 1 f o r Q u e e n s l a n d

Arrangements involving
10C restrictions and like objects
WORKED 1 How many different arrangements can be made using the 6 letters of the
Example eBook plus
9
word NEWTON? 360
2 How many different arrangements can be made using the 11 letters of Digital doc:
EXCEL
the word ABRACADABRA? 83 160 Spreadsheet
Permutations
WORKED 3 How many different arrangements of 5 counters can be made using 3
Example
10
red and 2 blue counters? 10
4 How many different arrangements of 9 counters can be made using 4 black, 3 red and
2 blue counters? 1260
5 A collection of 12 books is to be arranged on a shelf. The books consist of 3 copies of
Great Expectations, 5 copies of Catcher in the Rye and 4 copies of Huntin’, Fishin’
and Shootin’. How many different arrangements of these books are possible? 27 720

6 A shelf holding 24 cans of dog food is to be stacked using 9 cans of Yummy and 15 cans
of Ruff for Dogs. In how many different ways can the shelf be stocked? 1 307 504
WORKED 7 A cricket team of 11 players is to be selected, in batting order, from 15. How many
Example
11
different arrangements are possible if:
a there are no restrictions? 5.4 × 1010
b Mark must be in the team at number 1? 3.6 × 109
c Mark must be in the team but he can be anywhere from 1 to 11? 4.0 × 1010
8 The Student Council needs to fill the positions of president, secretary and treasurer
from 6 candidates. Each candidate can fill only one of the positions. In how many
ways can this be done if:
a there are no restrictions? 120
b Jocelyn must be secretary? 20
c Jocelyn must have one of the 3 positions? 60
9 The starting 5 in a basketball team is to be picked, in order, from the 10 players in the
squad. In how many ways can this be done if:
a there are no restrictions? 30 240
b Jamahl needs to play at number 5? 3024
c Jamahl and Anfernee must be in the starting 5? 6720
WORKED 10 a How many permutations of the letters in the word MATHS are there? 120
Example
12 b In how many of these do the letters M and A appear together? 48
c In how many permutations do the letters M and A appear apart? 72
Chapter 10 Permutations and combinations 481
11 A rowing team of 4 rowers is to be selected in order from 8 rowers.

a In how many different ways can this be done? 1680


b In how many of these ways do 2 rowers, Jane and Lee, sit together in the boat? 180
c In how many ways can the crew be formed without using Jane or Lee? 360
12 A horse race has 12 runners.
a In how many ways can 1st, 2nd and 3rd be filled? 1320
b In how many ways can 1st, 2nd and 3rd be filled if Najim finishes first? 110

13 multiple choice
If the answer is 10, which of the following statements best matches this answer?
A The number of ways 1st and 2nd can occur in a race with 5 entrants.
B The number of distinct arrangements of the letters in NANNA.
C The number of permutations of the letters in POCKET where P and O are
together.
D The number of permutations of the letters in POCKET where P and O are apart.
E 10P2 ÷ 4P2

14 multiple choice
If the answer is 480, which of the following statements best matches this answer?
A The number of ways 1st and 2nd can occur in a race with 5 entrants.
B The number of distinct arrangements of the letters in NANNA.
C The number of permutations of the letters in POCKET where P and O are
together.
D The number of permutations of the letters in POCKET where P and O are apart.
E 10P2 ÷ 4P2
15 The clue in a crossword puzzle says that a particular answer is an anagram of
STOREY. An anagram is another word that can be obtained by rearranging the letters
of the given word.
a How many possible arrangements of the letters of STOREY are there? 720
b The other words in the crossword puzzle indicate that the correct answer is O--T--
How many arrangements are now possible? 24
c Can you see the answer? OYSTER
16 There are 30 students in a class. The students are arranged in order and asked to give
eBook plus
the month and date of their birthday.
Digital doc: a How many different arrangements of these dates are possible? 36530
WorkSHEET 10.1 b How many arrangements of these dates are possible if no 2 students have the same
birthday? 365P30
482 M a t h s Q u e s t M a t h s C Ye a r 1 1 f o r Q u e e n s l a n d

Combinations
A group of things chosen from a larger group where order is not important is called a
combination. In previous sections we performed calculations of the number of ways a
task could be done where order is important — permutations or arrangements. We now
examine situations where order does not matter.
Suppose 5 people have nominated for a committee consisting of 3 members. It does
not matter in what order the candidates are placed on the committee, it matters only
whether they are there or not. If order was important we know there would be 5P3, or
60, ways in which this could be done. Here are the possibilities:
ABC ABD ABE ACD ACE ADE
BDE BCD BCE CDE CAB DAB
EAB DAC EAC EAD EBD DBC
EBC ECD BCA BDA BEA CDA
CEA DEA DEB CDB CEB DEC
CBA DBA EBA DCA ECA EDA
EDB DCB ECB EDC BAC BAD
BAE CAD CAE DAE DBE CBD
CBE DCE ACB ADB AEB ADC
AEC AED BED CDB BEC CED
The 60 arrangements are different only if we take order into account; that is, ABC is
different from CAB and so on. You will notice in this table that there are 10 distinct
committees corresponding to the 10 distinct rows. Each column merely repeats, in a
different order, the committee in the first row. This result (10 distinct committees) can
be arrived at logically:
1. There are 5P3 ways of choosing or selecting 3 from 5 in order.
2. Each choice of 3 is repeated 3! times.
3. The number of distinct selections or combinations is 5P3 ÷ 3! = 10.
This leads to the general rule of selecting r things from n things:
1. The number of ways of choosing or selecting r things from n distinct things
where order is not important is given by the rule nCr , where:
nP
r
n
Cr = ---------
r!
2. The letter C is used to represent combinations.

WORKED Example 13
Write these combinations as statements involving permutations, then calculate them.
a 7C2 b 20C3
THINK WRITE
nP
a 1 Write down the rule for nCr . a n
Cr = --------r
r!
7P
2 Substitute the given values of n and r 7
C2 = --------2-
into the combination formula. 2!
Chapter 10 Permutations and combinations 483

THINK WRITE

 7!
-----
 5!
3 Simplify the fraction. = ----------
2!
7!
= ----- ÷ 2!
5!
7! 1
= ----- × -----
5! 2!
7!
= ----------
5!2!
7 × 6 × 5!
= -----------------------
5! × 2 × 1
7×6
4 Evaluate. = ------------
2×1
42
= ------
2
= 21

nP
b 1 Write down the rule for nCr . b n
Cr = --------r
r!
20 P
2 Substitute the values of n and r into 20
C3 = ----------3-
3!
the formula.

 20!
--------
Simplify the fraction.  17!
3 = -------------
3!
20!
= -------- ÷ 3!
17!
20! 1
= -------- × -----
17! 3!
20!
= -------------
17!3!
20 × 19 × 18 × 17!
= --------------------------------------------
17! × 3 × 2 × 1
20 × 19 × 18
4 Evaluate. = ------------------------------
3×2×1
6840
= ------------
6
= 1140
484 M a t h s Q u e s t M a t h s C Ye a r 1 1 f o r Q u e e n s l a n d

WORKED Example 14
In how many ways can a basketball team of 5 players be selected from a squad of 9 if the
order in which they are selected does not matter?

THINK WRITE
nP
1 Write down the rule for nCr . n
Cr = --------r
Note: Since order does not matter use the r!
n
Cr rule.
Substitute the values of n and r into the 9P
2
C5 = --------5-
9
formula. 5!
3 Simplify the fraction.
 9!
-----
 4!
= ----------
5!
9!
= ----- ÷ 5!
4!
9! 1
= ----- × -----
4! 5!
9!
= ----------
4!5!
9 × 8 × 7 × 6 × 5!
= ------------------------------------------
4 × 3 × 2 × 1 × 5!
4 Evaluate. 9×8×7×6
= ------------------------------
4×3×2×1
3024
= ------------
24
= 126

The formula we use to determine the number of ways of selecting r things from n
distinct things, where order is not important, is useful but needs to be simplified.
nP
n
Cr = --------r
r!
n! -
-----------------
( n – r )!
= -----------------------
r!
n!
= -----------------------
( n – r )!r!
n!
n
Cr = -----------------------
( n – r )!r!

Note: nCr may also be written as  n .


 r
Chapter 10 Permutations and combinations 485
WORKED Example 15
Determine the value of the following. a 12
C5 b  10
 2
THINK WRITE
n!
a 1 Write down the rule for nCr . a Cr = -----------------------
n
( n – r )!r!
12!
2 Substitute the given values of n and r 12
C5 = ---------------------------
( 12 – 5 )!5!
into the combination formula.
12!
= ----------
7!5!
12 × 11 × 10 × 9 × 8 × 7!
3 Simplify the fraction. = ------------------------------------------------------------
7! × 5 × 4 × 3 × 2 × 1
12 × 11 × 10 × 9 × 8
= -------------------------------------------------
5×4×3×2×1
95 040
4 Evaluate. = ----------------
120
= 792
b 1 Write down the rule for  n . b  n = nCr
 r  r
n!
= -----------------------
( n – r )!r!

2 Substitute the given values of n and r   = --------------------------


10 10! -
 2  ( 10 – 2 )!2!
into the combination formula.
10!
= ----------
8!2!
10 × 9 × 8!
3 Simplify the fraction. = --------------------------
8! × 2 × 1
10 × 9
= ---------------
2×1
4 Evaluate. 90
= ------
2
= 45

WORKED Example 16
A committee consisting of 3 men and 4 women is to be chosen from 7 men and 9 women.
In how many ways can this be done?
THINK WRITE
n!
1 Write down the rule for nCr . n
Cr = -----------------------
( n – r )!r!
Note: Since order does not matter, use
the nCr rule.

2 Write down the number of ways of Number of ways of choosing 3 men = 7C3.
choosing 3 men from 7.
Continued over page
486 M a t h s Q u e s t M a t h s C Ye a r 1 1 f o r Q u e e n s l a n d

THINK WRITE
3 Write down the number of ways of Number of ways of choosing 4 women = 9C4.
choosing 4 women from 7.
7! 9!
4 Evaluate each of the combinations 7
C3 = ------------------------ 9
C4 = ------------------------
( 7 – 3 )!3! ( 9 – 4 )!4!
obtained in steps 2 and 3.
7! 9!
= ---------- = ----------
4!3! 5!4!
7 × 6 × 5 × 4! 9 × 8 × 7 × 6 × 5!
= -------------------------------- = ------------------------------------------
4!3 × 2 × 1 5! × 4 × 3 × 2 × 1
7×6×5 9×8×7×6
= --------------------- = ------------------------------
3×2×1 4×3×2×1
210 3024
= --------- = ------------
6 24
= 35 = 126
5 Use the multiplication principle to find The number of ways of choosing 3 men and
the number of ways of choosing men 4 women = 7C3 × 9C4
and women. = 35 × 126
= 4410
6 Answer the question. There are 4410 ways of choosing 3 men and
4 women.

Graphics Calculator tip! Calculating


combinations
To find the number of combinations of n objects taken r at a time, we need to calculate
n
Cr . The following steps show how to calculate 5C3 using a graphics calculator.

For the Casio fx-9860G AU


1. From the MENU select RUN-MAT. Press OPTN and
s

then F6 ( ) for more options.

2. Press F3 (PROB) and you will be able to see the


function nCr. Enter 5 (for n), then press F3 (nCr)
and enter 3 (for r). Press EXE to display the value.

For the TI-Nspire CAS


1. Open a new Calculator document. Press k to
access the catalog and press 1 for the list of
functions. Scroll down to select nCr(. You can do this
more quickly by first pressing N.
Chapter 10 Permutations and combinations 487
2. Press · and then complete the entry line to obtain
nCr(5,3). Press · to display the value.
(Alternatively, you can use the letter keys to type ncr
followed by ( directly into the calculator screen
instead of accessing it from the catalog. Press
NCR followed by (, then complete the entry line
and press ·.)

WORKED Example 17
Evaluate the following using your calculator and comment on your results.
a 9C3 b 9C6 c 15C5 d 15C10 e 12C7 f 12C5

THINK WRITE
a-f Use a graphics calculator (see above for steps) a 9C3 = 84
to evaluate the listed combinations. b 9C6 = 84
c 15C5 = 3003
d 15C10 = 3003
e 12C7 = 792
f 12C5 = 792
Comment on your results. So 9C3 = 9C6, 15C5 = 15C10 and
12
C7 = 12C5.

For each of the preceding examples, it can be seen that nCr = nCn − r . This may be
derived algebraically:
nP
n
Cn − r = -----------------
n–r
-
( n – r )!

 -------------------------------
n! -
 [ n – ( n – r ) ]!
= -------------------------------------
( n – r )!

 n!
-----
 r! 
= ------------------
( n – r )!
n!
= ----- ÷ (n − r)!
r!
n! 1
= ----- × ------------------
r! ( n – r )!
n!
= -----------------------
r! ( n – r )!
n!
= -----------------------
( n – r )!r!
= nCr
488 M a t h s Q u e s t M a t h s C Ye a r 1 1 f o r Q u e e n s l a n d

remember
remember
1. The number of ways of selecting r things from n things when order is
important is nPr .
2. The number of ways of selecting r things from n things when order is not
important is nCr .
nP
3. nCr = --------r
r!
n!
= -----------------------
( n – r! )r!
4. nCr may also be written as  n .
 r
5. nCr = nCn − r
SLE 9: Investigate permutations and
combinations which arise in games of chance
such as, in poker, the number of hands

10D
containing two aces, the number of hands
Combinations that are a full house.

8P WORKED 1 Write each of the following as statements in terms of permutations.


Example
1 a --------3- a 8C3 b 19C2 c 1C1 d 5C0
3! 13
19 P 2 Write each of the following using the notation nCr . eBook plus
b ----------2-
2! 8P 9P 8P 10P
a --------2- 8C2 b --------3- 9C3 c --------0- 8
- 10C4 Digital doc:
C0 4
1P
1 2! 3! 0! d ---------
4!
c --------- EXCEL
1! Spreadsheet
5P
WORKED 3 A committee of 3 is to be chosen from a group of people. In how many Combinations
0
Example
d --------- 14
ways can this be done if the group contains:
0!
a 3 people? 1 b 6 people? 20 c 10 people? 120 d 12 people? 220
4 A cricket team of 11 players is to be chosen from a squad of 15 players. In how many
ways can this be done? 1365
5 A basketball team of 5 players is to be chosen from a squad of 10 players. In how
many ways can this be done? 252
WORKED 6 Determine the value of the following:
Example
15
a 12C4 495 b 11C1 11 c 15
C0 1 d 12
C12 1
e  21 54 264 f  10 120 g  100 100 h  17 680
 15  7  1   14
7 From a pack of 52 cards, a hand of 5 cards is dealt.
eBook plus
a How many different hands are there? 2 598 960
b How many of these hands contain only red cards? 65 780 Digital doc:
c How many of these hands contain only black cards? 65 780 SkillSHEET 10.2
Listing possibilities
d How many of these hands contain at least one red and one black card?
2 467 400
WORKED 8 A committee of 3 men and 3 women is to be chosen from 5 men and 8 women. In
Example
16
how many ways can this be done? 560
9 A mixed netball team must have 3 women and 4 men in the side. If the squad has
6 women and 5 men wanting to play, how many different teams are possible? 100
10 A rugby union squad has 12 forwards and 10 backs in training. A team consists of
8 forwards and 7 backs. How many different teams can be chosen from the squad? 59 400
Chapter 10 Permutations and combinations 489
11 A quinella is a bet made on a horse race which pays a win if the punter selects the first
2 horses in any order. How many different quinellas are possible in a race that has:
a 8 horses? 28 b 16 horses? 120
12 A CD collection contains 32 CDs. In how many ways can 5 CDs be chosen from the
collection? 201 376
13 multiple choice
At a party there are 40 guests and they decide to have a toast. Each guest ‘clinks’
glasses with every other guest. How many clinks are there in all?
A 39 B 40 C 40! D 780 E 1560
14 multiple choice
On a bookshelf there are 15 books — 7 geography books and 8 law books. Jane
selects 5 books from the shelf — 2 geography books and 3 law books. How many dif-
ferent ways can she make this selection?
A 3003 B 360 360 C 1176 D 366 E 15
Questions 15, 16 and 17 refer to the following information. The Maryborough Tennis
Championships involve 16 players. The organisers plan to use 3 courts and assume that
each match will last on average 2 hours and that no more than 4 matches will be played on
any court per day.
15 In a ‘round robin’ each player plays every other player once. If the organisers use a
round robin format:
a How many games will be played in all? 120
b For how many days does the tournament last? 10 days
16 The organisers split the 16 players into two pools of 8 players each. After a ‘round
robin’ within each pool, a final is played between the winners of each pool.
a How many matches are played in the tournament? 57
b How long does the tournament last? 4 days 6 hours
17 A ‘knock out’ format is one in which the loser of every match drops out and the win-
ners proceed to the next round until there is only one winner left.
a If the game starts with 16 players how many matches are needed before a winner
is obtained? 15
b How long would the tournament last? 1 day 4 hours
18 Lotto is a gambling game played by choosing 6 numbers from 45. Gamblers try to
match their choice with those numbers chosen at the official draw. No number can be
drawn more than once and the order in which the numbers are selected does not matter.
a How many different selections of 6 numbers can be made from 45? 8 145060
b Suppose the first numbers drawn at the official draw are 42, 3 and 18. How many
selections of 6 numbers will contain these 3 numbers? 11 480
c Suppose the first numbers drawn at the official draw are 42, 3, 18 and 41. How
many selections of 6 numbers will contain these 4 numbers? 820
Note: This question ignores supplementary numbers. Lotto is discussed further in the
next section.
WORKED 19 a Calculate the value of: 6435, 6435
Example 12 12 15 15
17
i C3 and C9 220, 220 ii C8 and C7 iii 10C1 and 10C9 10, 10
8 8 10 10
iv C3 and C5 56, 56 v C0 and C10 1, 1
SLE 6: Search for patterns in
Pascal’s Triangle and verify b What do you notice? Give your answer as a general statement such as ‘The value
any claims algebraically or n n n
of Cr is . . .’. The value of C r is the same as Cn − r .
otherwise.
490 M a t h s Q u e s t M a t h s C Ye a r 1 1 f o r Q u e e n s l a n d

Applications of permutations and


combinations SLE 9: Investigate permutations and combinations which arise in games of chance.

Counting techniques, particularly those involving permutations and combinations, can


be applied in gambling, logistics and various forms of market research. In this section
we investigate when to use permutations and when to use combinations as well as
examining problems associated with these techniques.
Permutations are used to count when order is important. Some examples are:
1. the number of ways the positions of president, secretary and treasurer can be filled
2. the number of ways a team can be chosen from a squad in distinctly different
positions
3. the number of ways the first three positions of a horse race can be filled.
Combinations are used to count when order is not important. Some examples are:
1. the number of ways a committee can be chosen
2. the number of ways a team can be chosen from a squad
3. the number of ways a hand of 5 cards can be dealt from a deck.
An interesting application of combinations as a technique of counting is a game that
Australians spend many millions of dollars on each week — Lotto. To play, a player
selects 6 numbers from 45 numbers. The official draw chooses 6 numbers and 2 sup-
plementary numbers. Depending on how the player’s choice of 6 numbers matches the
official draw, prizes are awarded in different divisions.
Division 1: 6 winning numbers
Division 2: 5 winning numbers and one of the supplementary numbers 3
Division 3: 5 winning numbers
Division 4: 4 winning numbers
Division 5: 3 winning numbers and one of the supplementary numbers
If the official draw was:
8
Winning numbers Supplementaries
13 42 6 8 20 12 2 34 11
A player who chose:
8 34 13 12 20 45
would win a Division 4 prize and a player who chose:
40
8 34 13 12 22 45
would win a Division 5 prize.
25
Lotto systems
A player may have 7 lucky numbers 4, 7, 12, 21, 30, 38 and 45, and may wish
to include all possible combinations of these 7 numbers in a Lotto entry.
This can be done as follows:
4 7 12 21 30 38
4 7 12 21 30 45
4 7 12 21 38 45
4 7 12 30 38 45
4 7 21 30 38 45
4 12 21 30 38 45
7 12 21 30 38 45
Chapter 10 Permutations and combinations 491
The player does not have to fill out 7 separate entries to enter all combinations of these
7 numbers 6 at a time but rather can complete a ‘System 7’ entry by marking 7 numbers
on the entry form.
A System 9 consists of all entries of 6 numbers from the chosen 9 numbers.

WORKED Example 18
a Ten points are marked on a page and no three of these points are in a straight line.
How many triangles can be drawn joining these points?
b How many different 3-digit numbers can be made using the digits 1, 3, 5, 7 and 9
without repetition?
THINK WRITE
n!
a 1 Write down the rule for nCr . a n
Cr = -----------------------
( n – r )!r!
Note: A triangle is made by choosing 3
points. It does not matter in what order
the points are chosen, so nCr is used.
10!
2 Substitute the given values of n and r 10
C3 = ---------------------------
( 10 – 3 )!3!
into the combination formula.
10!
= ----------
7!3!
3 Simplify the fraction. 10 × 9 × 8 × 7!
= -----------------------------------
7! × 3 × 2 × 1
10 × 9 × 8
= ------------------------
3×2×1
4 Evaluate. 720
= ---------
6
= 120
5 Answer the question. 120 triangles may be drawn by joining
3 points.
6 Verify the answer obtained by using the
combination function on the calculator.
n!
b 1 Write down the rule for nPr . b n
Pr = ------------------
( n – r )!
Note: Order is important here.
5!
2 Substitute the given values of n and r P3 = ------------------
5
( 5 – 3 )!
into the permutation formula.
5!
= -----
2!
3 Evaluate. 5 × 4 × 3 × 2!
= --------------------------------
2!
=5×4×3
= 60
4 Answer the question. Sixty 3-digit numbers can be made without
repetition from a group of 5 numbers.
5 Verify the answer obtained by using the
permutation function on the calculator.
492 M a t h s Q u e s t M a t h s C Ye a r 1 1 f o r Q u e e n s l a n d

WORKED Example 19
Jade and Kelly are 2 of the 10 members of a basketball squad. In how many ways can a
team of 5 be chosen if:
a both Jade and Kelly are in the 5?
b neither Jade nor Kelly is in the 5?
c Jade is in the 5 but Kelly is not?
THINK WRITE
n!
a 1 Write down the rule for nCr . a n
Cr = -----------------------
( n – r )!r!
Note: Order is not important, so nCr is
used.
8!
2 Substitute the given values of n and r 8
C3 = ------------------------
( 8 – 3 )!3!
into the combination formula.
8!
Note: If Jade and Kelly are included = ----------
then there are 3 positions to be filled 5!3!
from the remaining 8 players.
3 Simplify the fraction. 8 × 7 × 6 × 5!
= --------------------------------
5! × 3 × 2 × 1
8×7×6
= ---------------------
3×2×1
4 Evaluate. 336
= ---------
6
= 56
5 Answer the question. If Jade and Kelly are included, then there are
56 ways to fill the remaining 3 positions.

n!
b 1 Write down the rule for nCr . b n
Cr = -----------------------
( n – r )!r!
Note: Order is not important, so nCr is
used.
8!
2 Substitute the given values of n and r 8
C5 = ------------------------
( 8 – 5 )!5!
into the combination formula.
Note: If Jade and Kelly are not 8!
= ----------
included then there are 5 positions to 3!5!
be filled from 8 players.
3 Simplify the fraction. 8 × 7 × 6 × 5!
= --------------------------------
3 × 2 × 1 × 5!
8×7×6
= ---------------------
3×2×1
4 Evaluate. 336
= ---------
6
= 56
5 Answer the question. If Jade and Kelly are not included, then there
are 56 ways to fill the 5 positions.
Chapter 10 Permutations and combinations 493
THINK WRITE
n!
c 1 Write down the rule for nCr . c n
Cr = -----------------------
( n – r )!r!
Note: Order is not important, so nCr is used.
8!
2 Substitute the given values of n and r 8
C4 = ------------------------
( 8 – 4 )!4!
into the combination formula.
Note: If Jade is included and Kelly is 8!
= ----------
not then there are 4 positions to be filled 4!4!
from 8 players.
3 Simplify the fraction. 8 × 7 × 6 × 5 × 4!
= ------------------------------------------
4 × 3 × 2 × 1 × 4!
8×7×6×5
= ------------------------------
4×3×2×1
4 Evaluate. 1680
= ------------
24
= 70
5 Answer the question. If Jade is included and Kelly is not, then
there are 70 ways to fill the 4 positions.
6 Verify each of the answers obtained by
using the combination function on the
calculator.

WORKED Example 20
Use the information on Lotto systems given on page 490.
A player uses a System 8 entry with the numbers 4, 7, 9, 12, 22, 29, 32 and 36.
The official draw for this game was 4, 8, 12, 15, 22, 36 with supplementaries 20 and 29.
a To how many single entries is a System 8 equivalent?
b List 3 of the player’s entries that would have won Division 4.
c How many of the player’s entries would have won Division 4?

THINK WRITE
n!
a 1 Write down the rule for nCr . a n
Cr = -----------------------
( n – r )!r!
Note: Order is not important, so nCr
is used.
8!
2 Substitute the given values of n and r into C6 = ------------------------
8
( 8 – 6 )!6!
the combination formula.
Note: A System 8 consists of all entries 8!
= ----------
consisting of 6 numbers chosen from 8. 2!6!
3 Simplify the fraction. 8 × 7 × 6!
= -----------------------
2 × 1 × 6!
8×7
= ------------
2×1
Continued over page
494 M a t h s Q u e s t M a t h s C Ye a r 1 1 f o r Q u e e n s l a n d

THINK WRITE
4 Evaluate. 56
= ------
2
= 28
5 Answer the question. A System 8 is equivalent to 28 single
entries.
6 Verify each of the answers obtained by
using the combination function on the
calculator.
b List 3 of the player’s entries that would have b Some of the possibilities are:
won Division 4. 4 12 22 36 7 9
Note: Division 4 requires 4 winning 4 12 22 36 7 29
numbers. The player’s winning numbers are 4 12 22 36 7 32
4, 12, 22 and 36. Any of the other 4 numbers
can fill the remaining 2 places.

n!
c 1 Write down the rule for nCr . c n
Cr = -----------------------
( n – r )!r!
Note: Order is not important, so nCr is
used.
4!
2 Substitute the given values of n and r into 4
C2 = ------------------------
( 4 – 2 )!2!
the combination formula.
Note: To win Division 4 the numbers 4, 4!
= ----------
12, 22 and 36 must be included in the 2!2!
entry. The other 2 spaces can be filled
with any of the other 4 numbers in any
order.
3 Simplify the fraction. 4 × 3 × 2!
= -----------------------
2 × 1 × 2!
4×3
= ------------
2×1
4 Evaluate. 12
= ------
2
=6
5 Answer the question. Six of the player’s entries would have
won Division 4.
6 Verify each of the answers obtained by
using the combination function on the
calculator.

remember
remember
1. Permutations are used to count when order is important.
2. Combinations are used to count when order is not important.
Chapter 10 Permutations and combinations 495
Applications of permutations
10E and combinations
WORKED 1 How many ways are there:
Example
18 a to draw a line segment between 2 points on a page with 10 points on it? 45
b to make a 4-digit number using the digits 2, 4, 6, 8 and 1 without repetition? 120
c to allocate 5 numbered singlets to 5 players? 120
d to choose a committee of 4 people from 10 people? 210
e for a party of 15 people to shake hands with one another? 105
2 How many ways are there:
a for 10 horses to fill 1st, 2nd and 3rd positions? 720
b to give only 5 players in a group of 10 an unnumbered singlet? 252
c to choose a team of 3 cyclists from a squad of 5? 10
d to choose 1st, 2nd and 3rd speakers for a debating team from 6 candidates? 120
e for 20 students to seat themselves at 20 desks arranged in rows? 2.4 × 1018
3 The French flag is known as a tricolour
flag because it is composed of the 3 bands
of colour. How many different tricolour
flags can be made from the colours red,
white, blue and green, if each colour can
be used only once in one of the 3 bands? 24
4 In a taste test a market research company has asked people to taste 4 samples of coffee
and try to identify each as one of four brands. Subjects are told that no 2 samples are
the same brand. How many different ways can the samples be matched to the brands? 24
376 992 5 In the gambling game roulette, if a
gambler puts $1 on the winning number
he will win $35. Suppose a gambler
wishes to place five $1 bets on 5 different
numbers in one spin of the roulette wheel.
If there are 36 numbers in all, in how
many ways can the five bets be placed?
WORKED 6 A volleyball team of 6 players is to be
Example
19
chosen from a squad of 10 players. In
how many ways can this be done if:
a there are no restrictions? 210
b Stephanie is to be in the team? 126
c Stephanie is not in the team? 84
d two players, Stephanie and Alison, are not both in the team together? 140
7 A cross-country team of 4 runners is to be chosen from a squad of 9 runners. In how
many ways can this be done if:
a there are no restrictions? 126
b Tony is to be one of the 4? 56
c Tony and Michael are in the team? 21
d either Tony or Michael but not both are in the team? 70
496 M a t h s Q u e s t M a t h s C Ye a r 1 1 f o r Q u e e n s l a n d

8 A soccer team of 11 players is to be chosen from a squad of 17. If one of the squad is
selected as goalkeeper and any of the remainder can be selected in any of the pos-
itions, in how many ways can this be done if:
a there are no restrictions? 8008
b Karl is to be chosen? 5005
c Karl and Andrew refuse to play in the same team? 5005
d Karl and Andrew are either both in or both out? 4004

9 multiple choice
A netball team consists of 7 different positions: goal defence, goal keeper, wing
defence, centre, wing attack, goal attack and goal shooter. The number of ways a
squad of 10 players can be allocated to these positions is:
A 10! B 7! 10! D 10P7 E 10C7
C --------
7!
10 multiple choice
A secret chemical formula requires the mixing of 3 chemicals. A researcher does not
remember the 3 chemicals but has a shortlist of 10 from which to choose. Each time
she mixes 3 chemicals and tests the result, she takes 15 minutes.

How long does the researcher need, to be absolutely sure of getting the right combi-
nation?
A 1 hour B 7.5 hours C 15 hours D 30 hours E 120 hours
WORKED 11 Use the information on Lotto given on page 490.
Example
20 A player uses a System 8 entry with the numbers 9, 12, 14, 17, 27, 34, 37 and 41. The
official draw for this game was 9, 13, 17, 20, 27, 41 with supplementaries 25 and 34.
a To how many single entries is a System 8 equivalent? 28 b 9 17 27 41 12 14,
b List 3 of the player’s entries that would have won Division 4. 9 17 27 41 12 37,
9 17 27 41 12 34
c How many of the player’s entries would have won Division 4? 6
Chapter 10 Permutations and combinations 497
12 Use the information on Lotto given on page 490.
A player uses a System 9 entry with the numbers 7, 10, 12, 15, 25, 32, 35, 37 and
41. The official draw for this game was 7, 11, 15, 18, 25, 39 with supplementaries
23 and 32.
a To how many single entries is a System 9 equivalent? 84 b 7 15 25 32 10 12,
7 15 25 32 10 35,
b List 3 of the player’s entries that would have won Division 5. 7 15 25 32 10 37
c How many of the player’s entries would have won Division 5? 10

Questions 13 and 14 refer to the following information: Keno is a popular game in clubs
and pubs around Australia. In each round a machine randomly generates 20 numbers from
1 to 80. In one entry a player can select up to 15 numbers.
13 Suppose a player selects an entry of 6 numbers.
The payout for a $1 bet on 6 numbers is:
Match 6 $1800 Match 5 $80
Match 4 $5 Match 3 $1
a In how many ways can an entry of 6 numbers contain 6 winning numbers? 38 760
Suppose an entry of 6 numbers has exactly 3 winning numbers in it.
b In how many ways can the 3 winning numbers be chosen? 1140
c In how many ways can the 3 losing numbers be chosen? 34 220
d How many entries of 6 numbers contain 3 winning numbers and 3 losing numbers? 39 010 800

14 Suppose a player selects an entry of 20 numbers.


The payout for a $1 bet on 20 numbers is:
Match 20 $250 000
Match 16 $50 000
Match 14 $15 000
Match 12 $450
Match 10 $20
Match 8 $2
Match 2 $2
Match 0 $100
a In how many ways can an entry of 20 numbers contain 20 winning numbers? 1
b Suppose an entry of 20 numbers has exactly 14 winning numbers in it.
eBook plus
i In how many ways can the 14 winning numbers be chosen? 38 760
Digital doc: ii In how many ways can the 6 losing numbers be chosen? 50 063 860
WorkSHEET 10.2 iii How many entries of 20 numbers contain 14 winning numbers and 6 losing
numbers? 1 940 475 213 600
iv How many entries of 20 numbers contain no winning numbers? 4 191 844 505 805 495

Pascal’s triangle, the binomial theorem


and the pigeonhole principle
Combinations are useful in other areas of mathematics, such as probability and
binomial expansions. If we analyse the nCr values closely, we notice that they produce
the elements of any row in Pascal’s triangle or each of the coefficients of a particular
binomial expansion.
498 M a t h s Q u e s t M a t h s C Ye a r 1 1 f o r Q u e e n s l a n d

Counting paths
Dorothy and Toto enter a maze, and they have a compass. To prevent themselves
from going round in circles they decide that they will only travel south or east and
never north or west. The maze is shown below left and each intersection is labelled.

A B C D E F G
1
2
3
4
5
6
7

Dorothy and Toto enter the maze at A1.


There are 2 ways to get to B2:
A1 → A2 → B2 or A1 → B1 → B2
Are you able to list the 6 ways of going from A1 to C3? (Remember that they
can only travel south or east.)
If you answered: A1 → A2 → A3 → B3 → C3
A1 → A2 → B2 → B3 → C3
A1 → A2 → B2 → C2 → C3
A1 → B1 → B2 → B3 → C3
A1 → B1 → B2 → C2 → C3
A1 → B1 → C1 → C2 → C3
then you are correct.
How many different ways can they A B C D E F G
travel to get to D6, E5 and E6?
1
If you answered 56, 70 and 126
respectively, then you are correct.
2 2
Can you devise a method of
counting the number of ways of 3 6
getting to each intersection and so
show that there are 924 ways of
4
getting to G7? Use the grid at right
to help record your results.
5 70
This activity provides a link
between two apparently separate
ideas — combinations and Pascal’s 6 56 126
Triangle. Let us consider Pascal’s
Triangle and then look at its 7
connection with combinations.
Chapter 10 Permutations and combinations 499
Pascal’s Triangle
The triangle below was named after the French mathematician Blaise Pascal. He was
honoured for his application of the triangle to his studies in the area of probability.

0th row
1st row
1
0th position 2nd row
1 1

1 2 1
This 5 is in the
1st position in 1 3 3 1
the 5th row.
1 4 6 4 1

1 5 10 10 5 1

1 6 15 20 15 6 1

The next entry here is 21.


It is the sum of the 2 numbers
above it — 6 and 15.

Each element in Pascal’s Triangle can be calculated using combinations. For


example, 10 is the 2nd element in the 5th row of Pascal’s Triangle; that is, 5C2 = 10
(this assumes 1 is the zeroth (0th) element).
Compare the values from Pascal’s Triangle with the number of ways of getting to
each intersection in the investigation ‘Counting paths’. What do you notice?
Pascal’s Triangle shows that the rth element of the nth row of Pascal’s Triangle is
given by nCr . It is assumed that the 1 at the beginning of each row is the 0th element.
Another application of combinations is the binomial theorem. This theorem gives a
rule for expanding an expression such as (a + b)n. Expanding expressions such as this
may become quite difficult and time consuming using the usual methods of algebra.
(a + b)1 = a + b
(a + b)2 = a2 + 2ab + b2
(a + b)3 = a3 + 3a2b + 3ab2 + b3
(a + b)4 = a4 + 4a3b + 6a2b2 + 4ab3 + b4
To use the binomial theorem you will need to recall the following conventions and
terminology of algebra:
1. In the term 3a2b, ‘3’ is called the coefficient of the term.
2. In the term a2b, the coefficient of the term is 1 even though it is not written.
3. In the term 3a2b, the power of ‘a’ is 2 and the power of ‘b’ is 1.
The binomial theorem is defined by the rule:
(a + b)n = nC0 an + nC1an − 1b + nC2 an − 2b2 + . . . + nCr an − rbr + . . . + nCnbn
= an + nC1an − 1b + nC2 an − 2b2 + . . . nCr an − rbr + . . . + bn
500 M a t h s Q u e s t M a t h s C Ye a r 1 1 f o r Q u e e n s l a n d

When expanding brackets which are in the form (a + b)n using the binomial theorem, recall:
1. The power of a in the first term of the expansion corresponds to the power of n and
in each successive term decreases by 1 until it corresponds to the power of 0.
2. The power of b starts at 0 and in each successive term increases by 1 until it
corresponds to the power of n.
3. The coefficient of the rth term is nCr .
4. The rth term is obtained by using nCr an − r br.
Again, this assumes that the initial term of the expansion is the 0th element.
The binomial theorem is particularly useful in probability calculations.

WORKED Example 21
Refer to Pascal’s Triangle on page 499 and answer the following questions.
a What number is in the 4th position in the 6th row?
b Complete the 7th row in Pascal’s Triangle.
c The numbers 7 and 21 occur side by side in the 7th row. What element in the 8th row
occurs below and in between these numbers?
THINK WRITE
a 1 Locate the 6th row and the 4th position. a 6th row ⇒ 1 6 15 20 15 6 1
Note: Remember the 0th row is 1 and the
first row is 1 1. In the 6th row the 1 on
the left is in the 0th position.
2 Answer the question. The number in the 4th position in the 6th
row is 15.
b 1 Write down the elements of the 6th row. b 6th row ⇒ 1 6 15 20 15 6 1
2 Obtain the 7th row.
(a) Place the number 1 at the beginning of 7th row ⇒ 1 7 21 35 35 21 7 1
the row.
(b) Add the first 2 adjacent numbers from
the 6th row (1 and 6).
(c) Place this value next to the 1 on the
new row and align the value so that it
is in the middle of the 2 numbers
(directly above) which created it.
(d) Repeat this process with the next 2
adjacent numbers from the 6th row
(6 and 15).
(e) Once the sums of all adjacent pairs
from the sixth row have been added,
place a 1 at the end of the row.
3 Answer the question. The 7th row is
1 7 21 35 35 21 7 1.
c 1 Add the numbers 7 and 21 in order to c 7 21
obtain the element in the 8th row which
occurs below and in between these 28
numbers.
2 Answer the question. The element in the 8th row which occurs
below and in between 7 and 21 is 28.
Chapter 10 Permutations and combinations 501
WORKED Example 22
Use combinations to calculate the number in the 5th position in the 9th row of Pascal’s
Triangle.

THINK WRITE
1 Write down the combination rule. n
Cr
2 Substitute the values for n and r into C5 = 126
9
the rule.
Note: The row is represented by n = 9.
The position is represented by r = 5.
3 Evaluate using a calculator.
4 Answer the question. The value of the number in the 5th position in
the 9th row is 126.

WORKED Example 23
Use the binomial theorem to expand (a + 2)4.

THINK WRITE
1 Write down the rule for the binomial (a + b)n = an + nC1an − 1b1 + . . . nCr an − rbr + . . . bn
theorem.
2 Substitute the values for a, b and n into (a + 2)4 = a4 + 4C1a321 + 4C2a222 + 4C3a123 + 24
the rule: a = a, b = 2 and n = 4.
3 Simplify. = a4 + 4 × a3 × 2 + 6 × a2 × 4 + 4 × a × 8 + 16
= a4 + 8a3 + 24a2 + 32a + 16

WORKED Example 24
What is the 4th term in the expansion of (x + y)7?

THINK WRITE
1 Write down the rule for the rth term. rth term = nCr an − rbr
Note: The rule for the 4th term is obtained
from the binomial theorem:
(a + b)n = an + nC1an − 1b1 + . . . nCr an − rbr
+ . . . bn
2 Substitute the values for a, b, n and r into n
Cr an − rbr = 7C4x7 − 4y4
the rule: a = x, b = y, n = 7 and r = 4.
3 Simplify. = 35x3y4
Note: The 0th term corresponds to the
first element of the expansion.
4 Answer the question. The 4th term is equal to 35x3y4.
502 M a t h s Q u e s t M a t h s C Ye a r 1 1 f o r Q u e e n s l a n d

SLE 17: Apply the pigeonhole principle to solve problems in situations such
as showing there are at least 2 in a group of 8 people whose birthdays fall on
Pigeonhole principle the same day of the week in any given year.
Henri Poincaré, a famous mathematician, once described mathematics as ‘the art of
giving the same name to different things’. Consider three phenomena, which on the
surface appear different — population growth, the value of investments and radioactive
decay. Each can be described by one mathematical concept: exponential change. The
mathematician gives three seemingly different things the same name.
The pigeonhole principle is a good example of how mathematics gives the same
name to different things.
The pigeonhole principle states that:
If there are (n + 1) pigeons to be placed in n pigeonholes then there is at least one
pigeonhole with at least two pigeons in it.
In this statement:
1. Note the precise use of language; in particular the importance of the phrase ‘at least’.
2. Some may view the pigeonhole principle as an obvious statement, but used cleverly
it is a powerful problem-solving tool.

WORKED Example 25
In a group of 13 people show that there are at least 2 whose birthday falls in the same
month.
THINK WRITE
1 Think of each person as a pigeon There are 12 months and 13 people.
and each month as a pigeonhole.
2 If there are 13 pigeons to be Using the pigeonhole principle:
placed in 12 holes at least one 13 people to be assigned to 12 months.
hole must contain at least two At least one month must contain at least two people.
pigeons. That is, at least two people have birthdays falling in
the same month.

Generalised pigeonhole principle:


If there are (nk + 1) pigeons to be placed in n pigeonholes then there is at least one
pigeonhole with at least (k + 1) pigeons in it.

WORKED Example 26
In a group of 37 people show that there are at least 4 whose birthdays lie in the same month.
THINK WRITE
1 Think of each person as a pigeon and There are 12 months and 37 people.
each month as a pigeonhole.
2 Use the generalised pigeonhole Using the generalised pigeonhole principle:
principle. 37 people to be assigned to 12 months.
3 (nk + 1) pigeons to be allocated to n The value of n is 12 and k is 3. So at least one
holes; month has at least (k + 1) or 4 people in it.
n = 12 → k = 3 That is, at least 4 people have birthdays falling
in the same month.
Chapter 10 Permutations and combinations 503
WORKED Example 27
On resuming school after the
Christmas vacation, many of the
22 teachers of Eastern High
School exchanged handshakes.
Mr Yisit, the social science
teacher said ‘Isn’t that unusual
— with all the handshaking, no
two people shook hands the same
number of times’.
Not wanting to spoil the fun,
the mathematics teacher, Mrs
Pigeon said respectfully, ‘I am
afraid you must have counted
incorrectly. What you say is not
possible.’
How can Mrs Pigeon make this
statement?

THINK WRITE
1 Think of the possible number of For each person there are 22 possible
handshakes by a person as a pigeonhole. numbers of handshakes; that is 0 to 21.

2 If two or more people have 0 handshakes, 1 person with 0 handshakes:


the problem is solved. Consider the cases If there is 1 person with 0 handshakes
where there is 1 person with 0 handshakes there can be no person with 21
or 0 persons with 0 handshakes. handshakes. Thus there are 21 people to
be assigned to 20 pigeonholes. Therefore
there must be at least one pigeonhole with
at least two people in it.
0 people with 0 handshakes:
If there is no person with 0 handshakes
there are 22 people to be assigned to
21 pigeonholes. Therefore there must be
at least one pigeonhole with at least two
people in it (at least two people have
made the same number of handshakes).

3 Conclude using a sentence. Thus, there are at least two people who
have made the same number of
handshakes.
504 M a t h s Q u e s t M a t h s C Ye a r 1 1 f o r Q u e e n s l a n d

remember
remember
1. Pascal’s Triangle shows that the rth element of the nth row of Pascal’s Triangle
is given by nCr .
2. Each new row in Pascal’s Triangle is obtained by first placing a 1 at the
beginning and end of the row and then adding adjacent entries from the
previous row.
3. Row 1 is the row containing the elements ‘1 and 1’.
4. The ‘1’ on the left-hand side of each row is in the 0th position of that row.
5. The binomial theorem is defined by the rule:
(a + b)n = an + nC1an − 1b + nC2an − 2b2 + . . . + nCr an − rbr + . . . + bn
1
1

0th term
1

rth term
1

6. When expanding brackets which are in the form (a + b)n using the binomial
28
1

6
21

theorem, recall:
1

5
15

56
1

(a) The power of a in the first term of the expansion corresponds to the power
10

35
1

of n and in each successive term decreases by 1 until it corresponds to the


20

70
1

power of 0.
10

35
1

(b) The power of b starts at 0 and in each successive term increases by 1 until
15

56
1

it corresponds to the power of n.


21
1

(c) The coefficient of the rth term is nCr .


28
1

(d) The rth term is obtained by using nCr an − rbr.


1

7
1

Again, this assumes that the initial term of the expansion is the 0th element.
7. The pigeonhole principle: If there are (n + 1) pigeons to be placed in n
1
1

pigeonholes then there is at least one pigeonhole with at least two pigeons in it.
1 Row
0
1
2
3
4
5
6
7
8

Pascal’s Triangle, the


10F binomial theorem and the
pigeonhole principle
1 Write the first 8 rows in Pascal’s Triangle.
WORKED 2 Refer to Pascal’s Triangle on page 499 and answer the following eBook plus
Example
21
questions.
a What number is in the 4th position in the 8th row? 70 Digital doc:
b Complete the 9th row in Pascal’s Triangle. 1 9 36 84 126 126 84 36 9 1 EXCEL
Spreadsheet
c If 9 and 36 occur side by side in the 9th row, what element in the Pascal’s Triangle
10th row occurs below and in between these numbers? 45
WORKED 3 Use combinations to:
Example
22
a calculate the number in the 7th position of the 8th row of Pascal’s Triangle 8
b calculate the number in the 9th position of the 12th row of Pascal’s Triangle 220
c generate the 10th row of Pascal’s Triangle.
10
C0 10C1 10C2 10C3 10C4 10C5 10C6 10C7 10C8 10C9 10C10
WORKED 4 Use the binomial theorem to expand: 1 10 45 120 210 252 210 120 45 10 1
Example
23
a (x + y)2 x2 + 2xy + y2
b (n + m)3 n3 + 3n2m + 3nm2 + m3
c (a + 3)4 a4 + 12a3 + 54a2 + 108a + 81
Chapter 10 Permutations and combinations 505
WORKED 5 a What is the 4th term in the expansion of (x + 2)5? 80x
Example
b What is the 3rd term in the expansion of (p + q)8? 56p5q3
24
c What is the 7th term in the expansion of (x + 2)9? 4608x
2

6 multiple choice
x
1 9 36 84 126 126 84 36 9 1
A row of Pascal’s Triangle is given above. What number is located at position x?
A 8 B 28 C 45 D 120 E 136

SLE 6: Search for patterns in Pascal’s Triangle and verify any claims algebraically or otherwise.
7 multiple choice
16x3 is definitely a term in the binomial expansion of:
A (x + 2)3 B (x + 4)3 C (x + 2)4 D (x + 4)4 E (x + 2)5
b i
The sum of the 8 a In Pascal’s Triangle, calculate the sum of all elements in the:
elements in each row i 0th row 1 ii 1st row 2 iii 2nd row 4
of Pascal’s triangle
is a power of 2:
iv 3rd row 8 v 4th row 16 vi 5th row 32
Row Sum b i What do you notice?
0 20 = 1 ii Complete the statement: ‘The sum of the elements in the nth row of Pascal’s
1 21 = 2 n
2 22 = 4 Triangle is . . .’ ‘The sum of the elements in the nth row of Pascal’s triangle is 2 .’
3 23 = 8
4 24 = 16 9 Use the result from question 8 to deduce a simple way of calculating: 26 = 64
5 2 = 32
5
C 0 + C 1 + C 2 + C3 + C 4 + C 5 + C 6
6 6 6 6 6 6 6

WORKED 10 In a cricket team consisting of 11 players, show that there are at least 2 whose phone
Example
25
numbers have the same last digit.

11 Two whole numbers add to give 21. Show that at least one of the numbers is greater
than 10.
WORKED 12 A squad of 10 netballers is asked to nominate when they can attend training. They can
Example
26
choose Tuesday only, Thursday only or Tuesday and Thursday. Show that there is at
least one group of at least 3 players who agree with one of these options.

13 S&M lollies come in five great colours — green, red, brown, yellow and blue. How
many S&Ms do I need to select to be sure I have 6 of the same colour? 26

14 The new model WBM roadster comes in burgundy, blue or yellow with white or black
trim. That is, the vehicle can be burgundy with white or burgundy with black and so
on. How many vehicles need to be chosen to ensure at least 3 have the same colour
combination? 13

15 Is it possible to show that in a group of 13 people, there are at least 2 whose birthdays
fall in February?
WORKED 16 Nineteen netball teams entered the annual state championships. However, it rained
Example
27
frequently and not all games were completed. No team played the same team more
than once. Mrs Organisit complained that the carnival was ruined and that no two
teams had played the same number of games. Show that she is incorrect in at least
part of her statement and that at least two teams played the same number of games.
506 M a t h s Q u e s t M a t h s C Ye a r 1 1 f o r Q u e e n s l a n d

History of mathematics
B L A I S E PA S C A L — ( 1 6 2 3 – 1 6 6 2 )
He demonstrated that air pressure
During his life . . .
decreases with height by taking accurate
Construction of
measurements at various levels on the side of
the Taj Mahal is
the Puy de Dôme mountain. He persuaded his
started.
brother to climb the mountain and take
Rembrandt measurements using a heavy barometer.
completes many Like many mathematicians, Blaise Pascal
of his famous had arguments with other mathematicians,
paintings. including René Descartes, who came to visit
Oliver Cromwell him. Descartes did not believe that Pascal
governs England. was capable of such difficult mathematics and
claimed that Pascal had stolen some of his
Blaise Pascal was a French mathematician ideas from Descartes himself. Blaise Pascal
and physicist who studied combinatorics and developed the pattern of numbers now known
developed the theory of probability. as Pascal’s Triangle that is used in
He was born in the town of Clermont in probability, permutations and combinations.
France. His father was a taxation officer. His When Blaise Pascal’s father died, his
mother died when he was only 4. Pascal was unmarried sister went into a convent and he
a sickly child and so was not sent to school was left to live free of family and spiritual
initially but was educated at home by his conflicts. His health improved and he took up
father. Because he was not healthy his father an active social life including gambling and
forbad him from studying mathematics. It driving a fast, horse-drawn carriage!
took about 5 years before Blaise could In late 1654 he was involved in an
convince his father to let him try. accident. His horses went over the edge of a
When Blaise was 16, his father was in bridge and were killed, but he survived.
trouble with the courts because he would not Pascal was shaken up by this and again saw
set any more taxes. He had to leave Paris, and the event as a message from God. In 1655 he
the family moved to Rouen. moved in with his married sister. Later that
Blaise Pascal discovered and proved a year, Pascal became ill and eventually died
major theorem of geometry when he was only from the effects of a brain tumour and
16 years old. This theorem was about the stomach ulcer in 1662.
intersections of points on a conic plane. The computer language ‘Pascal’ is named
When he was 18 he became very ill. He after him.
eventually recovered, after being temporarily
paralysed and close to death. After this scare
Questions
he became very religious and started to study
1. How old was Pascal when he proved
philosophy and religion. His research into
his theorem on conics? 16
mathematics and science often conflicted
2. What did he develop at age 19 that
with his religious beliefs. earned him a lot of money? Calculating machine
At age 19, Pascal invented a calculating 3. Upon which mountain was his work on
machine that could do simple addition and air pressure done and who did the real
subtraction. He sold many of these machines work? Puy de Dôme; his brother
and they were so well made that some still 4. What is ‘Pascal’s Triangle’ used for? Probability
exist today. 5. What did he die from? Brain tumour
Chapter 10 Permutations and combinations 507

summary
The addition and multiplication principles
• Combinatorics is often called ‘counting’ and deals with counting the number of
ways in which activities or events can happen.
• The multiplication principle should be used when there are two operations or events
(say, A and B) where one event is followed by the other. It states that: If there are n
ways of performing operation A and m ways of performing operation B, then there
are n × m ways of performing A and B.
• The addition principle should be used when two distinct operations or events occur
in which one event is not followed by another. It states that: If there are n ways of
performing operation A and m ways of performing operation B then there are n + m
ways of performing A or B.
• A selection where order is important is called an arrangement.

Factorials and permutations


• The number of ways in which n distinct objects may be arranged is n! (n factorial)
where:
n! = n × (n − 1) × (n − 2) × (n − 3) × . . . × 3 × 2 × 1
• 0! = 1
• 1! = 1
• The number of different arrangements or permutations when r things are chosen
from n things and order is important is given by the rule nPr , where:
n!
• nPr = ------------------
( n – r )!
• nPn = n!
• nP0 = 1
• The number of different ways in which n people can be seated, r at a time, in a
circle is:
nP
--------r .
r

Arrangements involving restrictions and like objects


• The number of different ways of arranging n things made up of groups of
indistinguishable things, n1 in the first group, n2 in the second group and so
on is:
n!
-----------------------------------
-.
n 1!n 2!n 3!…n r!
• When restrictions apply to arrangements use the multiplication and addition
principles as well as nPr .
508 M a t h s Q u e s t M a t h s C Ye a r 1 1 f o r Q u e e n s l a n d

Combinations
• The number of ways of selecting r things from n things when order is not important
is nCr .
nP
• nCr = ---------r
r!
n!
= -----------------------
( n – r )!r!
• nCr may also be written as  n .
 r
• nCr = nCn − r

Applications of permutations and combinations


• Permutations are used to count when order is important.
• Combinations are used to count when order is not important.

Pascal’s Triangle and the binomial theorem


• Pascal’s Triangle shows that the rth element of the nth row of Pascal’s Triangle is
given by nCr .
• Each new row in Pascal’s Triangle is obtained by first placing a 1 at the beginning
and end of the row and then adding adjacent entries from the previous row.
• The top row is row 0.
• Row 1 is the row containing the elements ‘1 1’.
• The ‘1’ on the left-hand side of each row is in the 0th position of that row.
• The binomial theorem is defined by the rule:
• (a + b)n = an + nC1an − 1b + nC2 an − 2b2 + . . . + nCr an − rbr + . . . + bn

0th term rth term

• When expanding brackets which are in the form (a + b)n using the binomial
theorem, recall:
1. The power of a in the first term of the expansion corresponds to the power of n
and in each successive term decreases by 1 until it corresponds to the power of
0.
2. The power of b starts at 0 and in each successive term increases by 1 until it
corresponds to the power of n.
3. The coefficient of the rth term is nCr.
4. The rth term is obtained by using nCr an − rbr.
• Points 3 and 4 both assume that the initial term of the expansion is the 0th element.

The pigeonhole principle


• If there are (n + 1) pigeons to be placed in n pigeonholes then there is at least one
pigeonhole with at least two pigeons in it.
• The generalised pigeonhole principle:
If there are (nk + 1) pigeons to be placed in n pigeonholes then there is at least one
pigeonhole with at least (k + 1) pigeons in it.
Chapter 10 Permutations and combinations 509

CHAPTER
review
1 multiple choice
Barbie’s wardrobe consists of 5 different tops, 4 different skirts and 3 different pairs of
10A
shoes. The number of different outfits Barbie can wear is:
A 5 B 12 C 60 D 80 E 120

2 multiple choice
How many different 3-digit numbers can be made from the numbers 1, 3, 5, 7 and 9 if the
10A
numbers can be repeated?
A 60 B 125 C 243 D 729 E 999

3 multiple choice
There are 7 candidates seeking election to the positions of either president or secretary of
10A
the Soccer Club Committee. If one of these candidates, George, is to be either president or
secretary, in how many ways can positions be filled?
A 12 B 21 C 42 D 49 E 56
4 How many 4-digit numbers less than 4000 can be made using the digits 1, 2, 3, 5, 7 and 9 if:
a repetition is not permitted? 180 10A
b repetition is permitted? 648

5 multiple choice
The permutation 9P6 is equal to:
10B
A 9×8×7 B 9×8×7×6×5×4×3 9! 9! 9!
C ----- D ----- E -----
6! 3! 4!
6 multiple choice
There are 12 horses in a race. In how many different ways can the 1st, 2nd and 3rd positions
10B
be filled?
A 12P3
B 123
C 312
D 12C3
E 12C12

7 multiple choice
A round table seats 5 people. From a group of 8 people, in how many ways can 5 people be
10B
seated at the table:
8P 8P
9! B 8P5 E 8C5
A ----- C --------5- D --------5-
6! 5 5!
8 Use your calculator to place these in ascending order: 19P6, 12P9, 2000P2. 2000
P2, 19P6, 12P9 10B
510 M a t h s Q u e s t M a t h s C Ye a r 1 1 f o r Q u e e n s l a n d

9 multiple choice
10C
How many different arrangements can be made using the 8 letters of the word NONSENSE?
A 1680 B 2520 C 3360 D 5040 E 40 320

10 How many different arrangements of 4 letters can be made from the letters of the word
10C PILL? 12

11 multiple choice
10D
Which of the following is equivalent to 8C2?
6P 8P 8P 8P 8P
A --------2- B --------6- C ---------2- D --------2- E --------2-
2! 2! 6!2! 6! 2!

12 multiple choice
10D
A committee of 4 men and 3 women is to be formed from 5 men and 8 women. In how
many ways can this be done?
A 61 B 280 C 1320 D 20 160 E 40 320

13 Use your calculator to place these in ascending order: 19C6, 22C15, 2000C2. 19
C6, 22C15, 2000C2
10D
14 A committee of 3 men and 4 women is to be formed from 7 men and 5 women. In how
10D many ways can this be done? 175

15 Two cards are dealt from a pack of 52. What is the number of ways that:
10D a both are black? 325
b both are aces? 6
c the cards are of different colours? 676

16 multiple choice
10E
A cycling team of 3 riders is to be chosen from a squad of 8 riders. In how many ways can
this be done if one particular rider, Jorge, must be in the team?
A 56 B 336 C 21 D 210 E 420

17 A ward in a city
10E hospital has 15 nurses
due to work on
Friday. There are
3 shifts that need to
be staffed by 5 nurses
on each shift. How
many different
arrangements for
staffing these 3 shifts
are possible, assuming
that each nurse only
works 1 shift? 756 756
Chapter 10 Permutations and combinations 511
18 multiple choice
What is the 4th term in the expansion of (p + 1)7?
10F
A p4 B 35p3 C 35p4 D 21p3 E 21p4

19 multiple choice
A row of Pascal’s Triangle is given below. What number is located at position x?
10F
1 9 36 84 126 126 84 36 9 1
x
A 48 B 120 C 56 D 210 E 252

20 a In the 10th row of Pascal’s Triangle, what is the 6th entry? 210
b Write the 10th row of Pascal’s Triangle using combinations. 10C 10C 10C 10C 10C . . . 10C 10F
0 1 2 3 4 10
c What is the sum of the elements of the 10th row? 1024

21 In the expansion of (x + 3)10, what is the:


a 2nd term? 405x8 b 3rd term? 3240x7 c 9th term? 196 830x
10F

Modelling and problem solving


1 Assume that car number plates are sequenced as follows: DLV334 → DLV335 → ... DLV339
→ DLV340 → ... DLV999 → DLW000 and so on. Using this sequence, how many number
plates are there between DLV334 and DNU211 inclusive? 50 878

2 How many paths are there from A to B if you are only allowed to move either down or to the
right on the lines of the grid? 2 944 656

B
512 M a t h s Q u e s t M a t h s C Ye a r 1 1 f o r Q u e e n s l a n d

3 Poker is a card game in which initially each


person is dealt 5 cards.
a How many different hands are possible?
(Order is not important.) 2 598 960
b How many hands contain only diamonds? 1287
c How many hands contain only red cards? 65 780
d When Wild Bill Hickock died at Deadwood,
Dakota, he was holding in his hand 2 pairs —
aces and eights. This is called the dead man’s
hand. In how many ways can you be dealt the
dead man’s hand? 1584

4 From a group of 20 female students, 2 female staff, 18 male students and 3 male staff, a
committee of 6 is to be formed. Find the number of different committees if:
a there are no restrictions 6 096 454
b all committee members must be students 2 760 681
c one female staff member, one male staff member and 4 students must be on the committee 442 890
d there is an equal number of males and females on the committee 2 048 200
e one particular student must be on the committee 850 668
f one particular student must not be on the committee 5 245 786
g the committee must comprise 2 male staff members, 2 male students, 1 female staff
member and 1 female student. 18 360

5 Two women and three men approach an ATM at the same time.
a How many different queues are possible if the position of each person in the queue is
taken into account? 120
b How many queues of at least two people are possible if the position of each person in the
queue is not taken into account? 26

6 A school is using identification cards (ID cards) that consist of 2 letters selected from A to D
inclusive followed by 3 digits chosen from 0 to 9 inclusive.
a How many different ID cards can be issued to students if a digit may be used more than
once but all 2 letters of each ID are different? 12 000
b New ID cards are issued to all students each year and the old cards discarded. However,
eBook plus the old ID numbers are not used again. If, on average, the school’s population increases by
10% each year and was 2000 during the year when the ID cards were first used, how many
Digital doc: years will elapse before cards with numbers already used will have to be issued? 5 years
Test Yourself
Chapter 10
11
Dynamics

syllabus reference
Option topic:
Dynamics

In this chapter
11A Displacement, velocity
and acceleration
11B Projectile motion
11C Motion under constant
acceleration
• derivatives and integrals of vectors
514 M a t h s Q u e s t M a t h s C Ye a r 1 1 f o r Q u e e n s l a n d • Newton's laws of motion in vector form
applied to objects of constant mass
• application to:
– straight line motion in a horizontal plane
with variable force
Displacement, velocity and – vertical motion under gravity without air
resistance
– projectile motion without air resistance
acceleration
To explore the motion of a particle in a plane, we must consider both its magnitude and
direction. As this motion may not be in a straight line, it is convenient to use a vector
approach to calculate the displacement, velocity and acceleration at time t.
To illustrate this vector approach, consider the point P in the following diagram.
y Relative to the origin, O, the position vector of the displacement of P from
P (x, y)
the origin, at time t, may be written as OP = x i + y j where x and y denote
˜
magnitude of the displacement along the Ox and Oy˜ axes respectively. The
position of P varies with time so it is appropriate to think of OP as a vector
O x which is time dependent. Displacement at time t is the change in position of
the point P, and as the reference point is the origin, the displacement of P can
be represented by the position vector.
If we write r (t) to represent OP at time t, then we may say that r (t) = x(t) i + y(t) j
˜ ˜ ˜ ˜
where x(t) and y(t) are the respective horizontal and vertical components of this
displacement at time t. As a general rule, when we write the magnitude of the displace-
ment or position vector r (t), we will write r where it is understood that r = r ( t ) .
˜ ˜

Vector expressions for velocity and acceleration


If the position vector r (t) = x(t) i + y(t) j represents the displacement of P at time t,
˜ ˜ ˜ 2
dr d r
then -----˜ represents the instantaneous velocity and -------2˜- represents the acceleration at
dt dt
time t. (It is often convenient to make use of the time-derivative notation of r˙ (t) for
˜
velocity and r˙˙(t) for the acceleration at time t.) We calculate the speed of a particle at
˜
time t as ṙ = r˙( t ) and the magnitude of the acceleration at time t as ṙ˙ = r˙˙( t ) . In our
˜ ˜
work, the distance or magnitude of the displacement will be measured in metres (m),
the speed or magnitude of the velocity in metres per second (m/s) and the magnitude of
the acceleration in metres per second per second (m/s2).

WORKED Example 1
Relative to the origin, O, the displacement (in metres) of a particle at time t seconds is
given by r (t) = 4t i + (10t - 5t2) j .
˜ ˜ ˜
a Find expressions for the velocity and acceleration vectors at time t seconds.
b Determine the initial position and speed of the particle.
c When t = 4 seconds, calculate
i the distance of the particle from the origin
ii the velocity (both magnitude and direction) of the particle
iii the measure of the angle between the velocity and displacement vectors.
d Is the acceleration constant for this motion?
Chapter 11 Dynamics 515
THINK WRITE
a 1 Write the position vector. a r (t) = 4t i + (10t − 5t2) j
˜ ˜ ˜
. dr . dr
2 Find r (t) = -----˜ for the velocity r (t) = -----˜
˜ dt ˜ dt
vector. = 4 i + (10 − 10t) j
˜ ˜
So v (t) = 4 i + (10 − 10t) j m/s
.
˜ . ˜ ˜
.. dr .. d r
2 .. dr
3 Find r (t) = -----˜ or r (t) = -------˜- for the r (t) = -----˜
˜ dt ˜ 2 ˜ dt
dt
acceleration vector. = −10 j
˜
So a (t) = −10 j m/s2
˜ ˜
b 1 The initial position occurs when b When t = 0,
t = 0. Substitute t = 0 into the r (0) = (4 × 0) i + (10 × 0 − 5 × 02) j
displacement vector expression. ˜ ˜ ˜
= 0i + 0 j
˜
This means ˜ the initial position of the
that
particle is at the origin.
2 The initial speed occurs when t = 0. When t = 0,
Substitute t = 0 into the velocity v (0) = 4 i + (10 − 10 × 0) j
vector expression. Remember that ˜ ˜ ˜
speed is the magnitude of velocity. = 4 i + 10 j
˜ ˜
The initial speed of the particle is the
magnitude of the initial velocity.
2 2
v ( 0 ) = 4 + 10
˜
= 2 29
The initial speed is 2 29 m/s.
c i 1 Distance is the magnitude of c i r (t) = 4t i + (10t − 5t2) j
˜ ˜ ˜
displacement, so first calculate r (4) r (4) = 16 i − 40 j
˜
to find the displacement at t = 4. ˜ ˜ ˜
2 Find the magnitude of the The required distance is the magnitude of
resulting vector expression to the displacement.
calculate the distance. 2 2
r ( 4 ) = 16 + ( – 40 )
˜
= 8 29
3 State the answer. The distance of the particle from the
origin at t = 4 s is 8 29 m.
ii 1 Calculate v (4) to find the velocity ii v (t) = 4 i + (10 − 10t) j
vector at t ˜= 4. ˜ ˜ ˜
v (4) = 4 i − 30 j
˜ ˜ ˜
2 Find the magnitude of the velocity. 2 2
v ( 4 ) = 4 + ( – 30 )
˜
= 2 229
≈ 30.27 m/s
Continued over page
516 M a t h s Q u e s t M a t h s C Ye a r 1 1 f o r Q u e e n s l a n d

THINK WRITE
3 To find the direction, draw a Let θ be the angle the velocity vector
diagram to represent the velocity makes with the horizontal.
vector v (4) = 4 i − 30 j . y
˜ ˜ ˜
4
O x
2 229 30

~v (4) = 4i~ – 30j


~
4 Use trigonometry to obtain an 30
equation to solve for θ. tan θ = ------
4
θ ≈ 82°24′
5 Write the answer. When t = 4 s, the velocity of the particle
is 2 229 m/s (or approx. 30.27 m/s)
downwards at an angle of 82˚24′ to the
horizontal.
iii 1 Write the two vectors for velocity iii When t = 4,
and displacement at t = 4 s. r (4) = 16 i − 40 j and v (4) = 4 i − 30 j
˜ ˜ ˜ ˜ ˜ ˜
2 To calculate the angle between the Let θ be the angle between the two
velocity and displacement vectors vectors.
we can use the dot product of the Using the dot product:
two vectors. r (4) • v (4) = r ( 4 ) v ( 4 ) cos θ
The dot product of two ˜ ˜ ˜ ˜
vectors a and b is given as
a • b = a˜ b cos˜ θ where θ is the
˜ ˜ included
angle ˜ ˜ by a and b .
˜ ˜
3 Calculate the product on the left of (16)(4) + (−40)(−30) =
the equation by multiplying the (8 29 )(2 229) cos θ
corresponding components of i
and j , and substitute the ˜
˜
magnitudes of r (4) and v (4) on
the right of the ˜equation. ˜
1264
4 Solve the equation to find θ. cos θ = ----------------------
16 6641
θ ≈ 14°12′
5 Write the answer. The angle between the velocity and
displacement vectors is 14˚12′.
d 1 Consider the vector expression for d a (t) = −10 j
acceleration found in part a. ˜ ˜
2 Since −10 j does not contain t, it is a Since −10 j is a constant, the acceleration is
˜
constant. State the answer. ˜
constant for this motion.
Chapter 11 Dynamics 517
WORKED Example 2
At time t seconds, a particle has a position vector given by the expression
r (t) = 2t i + (25 − t2) j metres.
˜ ˜ ˜
a Use a graphics calculator to plot the trajectory of this particle across the interval
0 ≤ t ≤ 5 seconds.
b Repeat part a above using an Excel spreadsheet.
c Determine the equation of this trajectory in the form y = f (x).

THINK WRITE/DISPLAY
a 1 Consider the components of the position a r (t) = 2t i + (25 − t2) j
vector. Assign x to the horizontal ˜ ˜ ˜
component and y to the vertical. This Let x = 2t and y = 25 − t2
produces two parametric equations that
we can use to graph the trajectory.

2 Use a graphics calculator to generate a SLE 9: Use the parametric facility of a graphing
graph of the particle’s motion. calculator to model the flight of a projectile.

For the Casio fx-9860G AU


(a) Press MENU and select GRAPH.
Press F3 (TYPE) followed by F3
(Parm) to select the parametric
equations option.

(b) Enter the x-component by completing


the entry line for Xt1 with 2t. (Press
X,q,T to enter t.) Similarly, enter the
y-component by completing the entry
line for Yt1 with 25 − t2. Press EXE
after each entry.
(c) Press F6 (DRAW) to display the
graph. To obtain a clearer view of the
graph, you can adjust the View
Window settings. Press SHIFT F3
(V-WIN) and adjust the values for
Xmin, Xmax, Ymin and Ymax.
(d) Press EXE until you return to the
equations screen and then press F6
(DRAW) to display the graph with this
new setting. To display the t, x and y
values at different points on the
graph, press SHIFT F1 (TRCE) and
use the arrow keys to move along the
line.
Continued over page
518 M a t h s Q u e s t M a t h s C Ye a r 1 1 f o r Q u e e n s l a n d

THINK WRITE/DISPLAY
For the TI-Nspire CAS
(a) Open a new Graphs & Geometry
document. Press b and select
3: Graph Type followed by
2: Parametric. In the entry panel, enter
x = 2t and y = 25 − t2. Change the
domain for t to 0 ≤ t ≤ 5.

(b) Press · to display the graph of the


particle’s trajectory. To adjust the
viewing window, press b, then select
4: Window and 1: Window Settings.
Enter the settings as shown.

(c) Highlight OK and press ·. To see


more of the screen, press /G to hide
the entry panel. (Press /G again to
bring the entry panel back.)
To investigate the coordinates of the
points along the graph, press b,
select 5: Trace and 1: Graph Trace.

b 1 Set up a spreadsheet with 3 columns titled b


t, x and y. In the first column, enter values t x y
from 0 to 5 for t. Enter the appropriate
formula for both x and y and generate 0 0 25
corresponding values in columns 2 and 3. 1 2 24
eBook plus
2 4 21
SLE 16: Use spreadsheets to investigate problems.
Digital doc: 3 6 16
EXCEL Spreadsheet
Plotting the trajectory
of a particle
4 8 9
5 10 0

2 Generate a graph of the particle’s trajectory. y


Trajectory
30
Metres

25
20
15
10
5
0 x
0 2 4 6 8 10 12

c 1 Write the parametric equations for the c x = 2t [1]


position vector. y = 25 − t2 [2]
Chapter 11 Dynamics 519
THINK WRITE/DISPLAY
2 Eliminate the shared parameter t between x
Rearranging equation [1]: t = ---
x = 2t and y = 25 − t2. Simplify to obtain an 2
equation in the form y = f (x). Substituting for t in equation [2]:
2
y = 25 −  ---
x
 2
2
x
= 25 − -----
4
= 1--- (100 − x2)
4

3 Write the answer. The equation of the trajectory is


y = 1--- (100 − x2).
4

WORKED Example 3
At time t seconds, the displacement (in metres) of a particle A is given by
r A(t) = (8 − t) i + (8 − 4t + t2) j and the displacement (in metres) of a particle B is given by
˜ ˜ ˜
r B(t) = (t + 2) i + (2 − 2t + t2) j .
˜ ˜ ˜
a If these particles collide, determine when they collide.
b What are the coordinates of the impact point?
c Find the Cartesian equations of the trajectories of the particles.
d Use a graphics calculator to verify the coordinates of the impact point.

THINK WRITE/DISPLAY
a 1 If the particles collide, they meet at a a At impact point, horizontal components are
common point. So the horizontal equal:
components of the two vectors will 8−t=t+2 [1]
be equal and the vertical components At impact point, vertical components are
will be equal. Produce two equations equal:
to reflect this. 8 − 4t + t2 = 2 − 2t + t2 [2]

2 Solve equation [1] to find t. Rearranging equation [1]:


2t = 6
2t = 3

3 Check the solution by substituting Check by substituting t = 3 into equation [2]:


t = 3 into the left and right sides of LHS = 8 − 4t + t2 = 8 − 12 + 9 = 5
equation [2]. RHS = 2 − 2t + t2 = 2 − 6 + 9 = 5
LHS = RHS so solution is correct.

4 Write the answer. Particles A and B collide after three


seconds.
Continued over page
520 M a t h s Q u e s t M a t h s C Ye a r 1 1 f o r Q u e e n s l a n d

THINK WRITE/DISPLAY
b 1 Substitute for t in the expression b r A(t) = (8 − t) i + (8 − 4t + t2) j
for the displacement of either ˜ ˜ ˜
At the point of impact, t = 3.
particle A or B. (In this case, select
particle A.) So r A(3) = (8 − 3) i + (8 − 12 + 9) j
˜ ˜ ˜
So r A(3) = 5 i + 5 j
˜ ˜ ˜
2 Write the answer. The coordinates of the impact point are (5, 5).

c In both expressions, assign the c Let x represent the horizontal component and
variables x and y respectively to the y the vertical component of the displacement.
horizontal and vertical components. For particle A:
Eliminate the shared parameter t to x = 8 − t and y = 8 − 4t + t2
obtain each Cartesian equation.
Hence, t = 8 − x and so
y = 8 − 4(8 − x) + (8 − x)2
= x2 − 12x + 40
yA = (x − 6)2 + 4
For particle B:
x = t + 2 and y = 2 − 2t + t2
Hence, t = x − 2 and so
y = 2 − 2(x − 2) + (x − 2)2
= x2 − 6x + 10
yB = (x − 3)2 + 1

d Verify that the coordinates of the d


impact point are (5, 5) by drawing the
graphs of the displacement for each
particle and locating the point of
intersection.

For the Casio fx-9860G AU


(a) Press MENU and select GRAPH.
Press F3 (TYPE) followed by
F1 (Y=) to select the function
option. Complete the entry line for
Y1 with the equation for particle A.
Similarly, complete the entry line
for Y2 with the equation for
particle B.
(b) Press F6 (DRAW) to display the
graph. (Adjust the View Window
settings if necessary.)
Press SHIFT F5 (G-SLV)
followed by F5 (ISCT) to display
the coordinates of the intersection Point of intersection is (5, 5) as required.
point.
Chapter 11 Dynamics 521
THINK WRITE/DISPLAY
For the TI-Nspire CAS
(a) Open a new Graphs & Geometry
document. Complete the entry line for
f1(x) with the equation for particle A.
Press ·. Similarly, complete the
entry line for f2(x) with the equation
for particle B and press ·. Adjust the
Window Settings if necessary.

(b) To display the point of intersection,


press b, select 6: Points & Lines
and 3: Intersection Point(s). Navigate
the pointer to each line and press ·.
(Press d to fix any errors.) The
coordinates of the point of intersection
will be displayed.
Point of intersection is (5, 5) as required.

remember
remember
1. If the position vector r (t) = x(t) i + y(t) j represents the displacement of P at
˜ ˜ ˜
SLE 1: Given the position 2
vector of a point as a dr d r
function of time such as time t, then -----˜ represents the instantaneous velocity and -------2˜- represents the
r (t ) = t i + t 2 j + sin t k
dt dt
~ ~ ~ ~
` ` ` ` instantaneous acceleration at time t.
determine the velocity and
acceleration vectors. 2 2
2. For the magnitude of a vector x i + y j : xi + y j = x + y
SLE 9: Use the parametric ˜ ˜ ˜ ˜
facility of a graphing 3. The dot product of two vectors a and b is given as a • b = a b cos θ where
calculator to model the ˜ ˜ ˜ ˜ ˜ ˜
flight of a projectile. θ is the angle included by a and b.
˜ ˜
SLE 16: Use spreadsheets to
investigate problems.

Displacement, velocity and


11A acceleration
WORKED 1 Relative to the origin, O, the displacement (in metres) of a particle at time t seconds is
Example
given by r (t) = 6t i + (3t − 7t2) j . v (t) = 6 i + (3 − 14 t) j , a (t) = −14 j
1 ˜ ˜ ˜ ˜ ˜
˜ ˜ ˜
a Find expressions for the velocity and acceleration vectors at time t seconds.
b Determine the initial displacement and speed of the particle. r (0) = 0 m, v(0) = 3 5 m/s
˜ ˜
c When t = 2 seconds, calculate:
i the distance of the particle from the origin r˜ (2) = 12 ˜i − 22 j m 25.7 m/s downwards at an
˜
ii the velocity (both magnitude and direction) of the particle angle of 13°30′ to the vertical
iii the measure of the angle between the velocity and displacement vectors. 15°7′
d Is the acceleration constant for this motion? Yes, 14 m/s2 downwards
522 M a t h s Q u e s t M a t h s C Ye a r 1 1 f o r Q u e e n s l a n d

WORKED 2 At time t seconds, a particle has a position vector given by the expression
Example
2 r (t) = 5t i + (49 − t2) j metres.
˜ ˜ ˜
a Use a graphics calculator to plot the trajectory of this particle across the interval
ii r (0) = 0 m, v(0) is undefined

0 ≤ t ≤ 7 seconds. Check with your teacher.


3 e i v (t) = i + 6t j , a (t) = 6 j g i v (t) = 2t i − 4 j , a (t) = 2 i
˜

- i − 6j, t ≠ 0
h i v (t) = 1 - i − 6t j , t ≠ 0;
ii r (0) = 0 m, v(0) = 4 m/s

b Repeat part a above using an Excel spreadsheet. Check with your teacher.
c Determine the equation of this trajectory in the form y = f(x). y = -----
1-
(1225 − x2)
˜
˜ ˜

25
˜

3 For each of the following position vectors (a to h) for a particle, where the displace-
4t t ˜
a (t) = − ----------
2 t ˜
1

ment is measured in metres and time in seconds,


˜

--------

˜
˜
iii y2 = 16x

iii y = −3x4

i calculate the corresponding vector expressions for the velocity and acceleration
ii find the initial position and speed
˜
˜

˜
˜

iii determine the Cartesian equation of the trajectory of the particle.


a r (t) = 2t i + (5 − t) j 3 a i v (t) = 2 i − j , a (t) = 0 c i v (t) = − i − 6 t j , a (t) = −6 j
˜ ˜ ˜ ˜ ˜ ˜ ˜ ˜ ˜ ˜ ˜ ˜

ii r (0) = 4 i m, v(0) = 1 m/s

f i v (t) = 2t i − j , a (t) = 2 i
˜
˜

b r (t) = t i + (6t − t ) j ii r (0) = 5 j m, v(0) = 5 m/s ii r (0) = 10 j m, v(0) = 1 m/s


ii r (0) = 0 m, v(0) = 1 m/s

˜ ˜ ˜ ˜ ˜ ˜
˜ iii y = 10 − 3x2
c r (t) = −t i + (10 − 3t2) j iii y = 5 − 1--2- x
˜ ˜ d i v (t) = 3 i + (5 − 4t) j ,
˜ ˜

˜ ˜ ˜

˜ b i v (t) = i + (6 − 2 t) j ,
d r (t) = 3t i + (5t − 2t2) j ˜ ˜ ˜ ˜ ˜
˜ ˜ ˜ a (t) = − 4j
2 ˜ a (t) = −2 j
iii y = 3(x − 4)2

e r (t) = (4 + t) i + 3t j ˜ ˜
˜ ˜
˜

˜ ˜ ˜ ii r (0) = 0 m, v(0) = 34 m/s


˜
˜

f r (t) = t i − t j
2 ii r (0) = 0 m, v(0) = 37 m/s ˜ ˜
iii y2 = x

˜ ˜ ˜ ˜ 2
˜ = − iii y = --9x- (15 − 2x)
g r (t) = t i − 4t j
2 iii y 6 x x
˜

˜
˜

˜ ˜ ˜
h r (t) = t i − 3t2 j
˜ ˜ ˜
WORKED 4 At time t seconds, the displacement (in metres) of a particle A is given by
Example
3 r A(t) = (2 − t) i + (3 − 2t + t2) j and the displacement (in metres) of a particle B
˜ ˜ ˜
is given by r B(t) = (t − 8) i + (t2 + t − 12) j . yA = (x − 1)2 + 2,
˜ ˜ ˜ yB = (x + 8.5)2 − 12.25
a If these particles collide, determine when they collide. t = 5 seconds
b What are the coordinates of the impact point? (−3, 18)
c Find the Cartesian equations of the trajectories of the particles.
d Use a graphics calculator to verify the coordinates of the impact point.
5 The displacement (in metres) of a particle at time t seconds is given by
r (t) = 2t i + (4 + 2t − t2) j . v (t) = 2 i + 2(1 − t) j , a (t) = −2 j
˜ ˜ ˜ ˜ ˜ ˜ ˜ ˜
a Find expressions for the velocity and acceleration vectors at time t seconds.
b Determine the initial velocity of the particle. 2 2 m/s upwards at an angle of 45° to the vertical
c When t = 3 seconds, calculate
2 5 m/s downwards at
i the distance of the particle from the origin 37 m an angle of 26°34′ to the
ii the velocity (both magnitude and direction) of the particle vertical
iii the measure of the angle between the velocity and acceleration vectors. 26°34′
.. .
d Show that r (t) + r (t) = 2 i − 2t j .
˜ ˜ ˜ ˜
6 At time t seconds, a particle has a position vector given by the expression
r (t) = 2t i + (5 − t)2 j metres.
˜ ˜ ˜
a Use a graphics calculator to plot the trajectory of this particle across the interval
0 ≤ t ≤ 10 seconds. Check with your teacher.
b Repeat part a using an Excel spreadsheet. Check with your teacher.
c For what values of t are the velocity vectors and acceleration vectors perpendicular?
t = 5 seconds
Chapter 11 Dynamics 523
7 P and Q are particles with displacements (in metres) at time t seconds given by
12
r P(t) = (t + 2) i + (5 − t)3 j and r Q(t) = ---------- i + (t − 3) j .
˜ ˜ ˜ ˜ t –2˜ ˜
a If these particles collide, determine when and where they collide. P and Q do not collide.
b Verify your response to part a by using suitable technology.
c Express the displacement of Q as an equation written in Cartesian form. y = 12------ − 1
x

8 The displacement (in metres) of a particle at time t seconds is given by


2
r (t) = --- i + 3t j .
˜ t˜ ˜
a Use a graphics calculator to plot the path of this particle across the interval
1 ≤ t ≤ 10 seconds. Check with your teacher.
b Calculate the average velocity across the interval 1 ≤ t ≤ 10 seconds. − --15- i + 3 j m/s
˜ ˜
c Calculate the instantaneous velocity and acceleration of the particle when
t = 5 seconds. v (5) = − -----
2
25 ˜
- i + 3 j , a (5) = --------- i
4
125 ˜
˜ ˜ ˜
d What can be said about the acceleration as t increases? Approaches zero
9 The displacement (in metres) of an object at time t seconds is given by
r (t) = (2 + t) i + (3 + 12t − 5t2) j .
˜ ˜ ˜
a From what position is this object projected? 2 i + 3 j
˜ ˜
b Calculate the speed of projection. 145 m/s
c At what angle to the horizontal is this object projected? 85°14′
d When t = 2 seconds, 65 m/s downwards
i calculate the velocity (both magnitude and direction) of the object at an angle of 7°08′
to the vertical
ii determine the position of the object relative to the origin. 4 i + 7 j
˜ ˜
10 Two particles are projected from the origin, O, such that at time t seconds the dis-
placement (in metres) of the first particle is given by r 1(t) = 20t i + (15t − 5t2) j and
˜ ˜ ˜ i +
the displacement (in metres) of the second particle is given by r 2(t) = (80 − 12t)
˜ ˜
(a + 9t − 5t ) j where a is a real number.
2
˜
a What are the initial positions of the particles? r 1(0) = 0 , r 2(0) = 80 i + a j
˜ ˜ ˜ ˜ ˜
b If these two particles meet after 2 1--2- seconds of flight, what is the value of a in the
expression for r 2(t)? a = 15
c Locate the point˜ of collision. (50, 6.25)
d Calculate the angle between the velocity vectors at this time. 100°18′
11 A particle is projected from the origin so that at time t seconds its displacement
(in metres) is given by r (t) = 20t i + (c + bt − 5t2) j where both b and c are real
˜ ˜ ˜
numbers. y
The point Q is a point on an inclined plane OQ where Q
has coordinates (80, 15). When the particle reaches Q, its
.
velocity vector is given by r (t) = 20 i − 15 j m/s. Q
˜ ˜
a What are the values of b and c? b = 25, c˜ = −5
y = − -----
1-
(x − 50)2 + 26 --14- b Find the equation of the trajectory of the particle.
80 O x
c How far up the incline does the projectile land? 81.4 m
y
12 A particle is projected from the origin up a plane that is
inclined at an angle of θ to the horizontal such that at
time t seconds, its displacement (in metres) is given by Q
r (t) = 35 cos α t i + (35 sin α t − 5t 2) j where α is the α
˜ ˜
angle of projection of the particle. ˜ O x
524 M a t h s Q u e s t M a t h s C Ye a r 1 1 f o r Q u e e n s l a n d

a If the point Q is located a distance R metres along the incline, show that
R cos θ = 35 cos α t and that R sin θ = 35 sin α t − 5t2.
4R
b Deduce that if tan θ = 3--- , then t = ------------------------ .
4 175 cos α
16R
c Now show that (4 sin α − 3 cos α) cos α = ------------ .
Rmax = 76 -----
9-
m 1225
16
when α ≈ 63°26′
d By making suitable use of a graphics calculator, find the greatest value that R may
have and the value of α when this occurs if 0 < α < 90°. Give your answer correct
to the nearest minute.
e Use an Excel spreadsheet to validate your solution obtained in part d above.

Projectile motion
A projectile is a body that is projected at or near the Earth’s surface, and which has no
eBook plus
mechanical means of propulsion after leaving a firing mechanism. The only external
Digital doc: force acting on such a body is its weight force. For speeds that are relatively low, we
Introduction to will assume that air-resistance is negligible and that the projectile behaves like a point
Integral Calculus
mass. (In reality, shape, size, mass and speed are indeed significant, particularly so for
high-speed motion.) It should be understood that the motion of a projectile occurs in
the vertical plane containing an initial velocity vector, and that the motion is parabolic.
When the projectile is set in motion, the only external force is the particle’s weight
force (by Newton’s Second Law of Motion, force = mass × acceleration). If we write g
to represent gravitational acceleration, then weight = mass × g; that is, W = mg.
In the ideal situation where there are no resisting forces
acting on the projectile, the net force of the system is zero. F
Hence, if F represents the upwards force of motion, then the
equation F + W = 0 describes the sum of the forces acting ver- mg
tically on the projectile.
Since we can write F = ma, then F + W = 0 becomes ma + mg = 0 so a = −g m/s2.
Note: Various approximations for g exist, and in our work we shall use g = 9.8 m/s2
unless stated otherwise. The unit of force is the newton (N).
SLE 8: Model the path of a In the vertical plane, we may express the sum of the forces acting using vector notation.
projectile without air resistance,
using the vector form of the F + mg j = 0
˜ ˜
equations of motion starting
with a = −g j where upwards
m a + mg j = 0
~ ~ ˜ ˜
`
is positive. `
m a = −mg j
˜ ˜
a = −g j .
˜ ˜
Since a = ax i + ay j , the horizontal component of acceleration (ax) is zero and hence
˜ ˜ ˜
the horizontal component of the velocity is always a constant.
.
Now if r (t) represents the displacement of a particle at time t, then r (t) represents the
˜ .. ˜
associated velocity vector and r (t) represents the acceleration vector.
..˜
Since a = −g j and a = r (t)
.. ˜ ˜ ˜ ˜
r (t) = −g j
˜. ˜
˜ ∫
r (t) = (−g j ) dt = −gt j + c 1
˜ ˜ ˜
so v (t) = −gt j + c 1
˜ ˜ ˜
This is the velocity vector at time t where c 1 is a vector constant of integration.
˜
Chapter 11 Dynamics 525
If we integrate this expression for velocity, we obtain an expression for the displace-
ment of the particle.
r (t) = − 1--- gt2 j + c 1t + c 2
˜ 2 ˜ ˜
˜
The constant vectors c 1 and c 2 can be determined by any initial data.
˜ ˜

WORKED Example 4
A particle moves so that its acceleration at time t is given by a = 2 i + (3 + 4t) j . Find
vector expressions for the velocity and displacement of the particle ˜ ˜given that the
˜ initial
velocity is 3 i + 4 j and the initial position is 0 i + 0 j .
˜ ˜ ˜ ˜
THINK WRITE
1 Integrate the expression for a to
find v . ˜
v = a dt
˜ ∫˜
˜ ∫
= (2 i + (3 + 4t) j ) dt
˜ ˜
= 2t i + (3t + 2t2) j + c 1
˜ ˜ ˜
2 Apply the initial condition (initial When t = 0, v = 3 i + 4 j and so
˜ ˜ ˜
velocity) to determine the constant 3i + 4 j = 0i + 0 j + c 1
vector of integration. ˜ ˜ ˜ ˜ ˜
c 1 = 3i + 4 j
˜ ˜ ˜
3 Substitute the expression for c 1 into v . v = 2t i + (3t + 2t2) j + 3 i + 4 j
˜ ˜ ˜ ˜ ˜ ˜ ˜
4 Simplify to write the vector expression Hence the velocity vector is
for the velocity of the particle. v = (2t + 3) i + (4 + 3t + 2t2) j
˜ ˜ ˜
5 Integrate the expression for v to
find r . ˜
r = v dt
˜ ∫
˜
˜

= [(2t + 3) i + (4 + 3t + 2t2) j ] dt
˜
3 2 ˜
= (t2 + 3t) i + (4t + --- t2 + --- t3) j + c 2
˜ 2 3 ˜ ˜
6 Apply the initial condition (initial When t = 0, r = 0 i + 0 j and so
˜ ˜ ˜
position) to determine the constant 0i + 0 j = 0i + 0 j + c 2
vector of integration. ˜ ˜ ˜ ˜ ˜
c 2 = 0i + 0 j
˜ ˜ ˜
7 Substitute the expression for c 2 into r Hence the displacement vector is
˜ ˜ 3 2
and write the vector expression for the r = (t2 + 3t) i + (4t + --- t2 + --- t3) j
displacement of the particle. ˜ ˜ 2 3 ˜

Components of velocity and displacement


For projectile motion where resistance to motion is ignored, we have established that
a = −g j and hence v = −gt j + c . The initial velocity data will determine the vector
˜ ˜ c. ˜ ˜ ˜
constant,
˜
y
v
~v vy

O x vx
526 M a t h s Q u e s t M a t h s C Ye a r 1 1 f o r Q u e e n s l a n d

For an object projected at an angle of θ to the horizontal, the velocity vector can be
expressed as v = vx i + vy j .
˜ ˜ ˜
Using trigonometric ratios, the initial velocity (at t = 0) can be written as
v = v cos θ i + v sin θ j .
˜ ˜ ˜
To find c , we substitute t = 0 and the initial velocity into v = −gt j + c .
˜ ˜ ˜ ˜
So v cos θ i + v sin θ j = 0 j + c
˜ ˜ ˜ ˜
c = v cos θ i + v sin θ j
˜ ˜ ˜
Hence v = −gt j + c becomes
˜ ˜ ˜
v = −gt j + v cos θ i + v sin θ j
˜ ˜ ˜ ˜
v = v cos θ i + (v sin θ − gt) j
˜ ˜ ˜
The horizontal component of the velocity, vx, is given by vx = v cos θ and the vertical
component of the velocity, vy , is given by vy = v sin θ − gt.
In a similar way, the components of the displacement can be established by inte-
grating v = v cos θ i + (v sin θ − gt) j with respect to t.
˜ ˜ ˜
That is,

˜ ∫
r = [v cos θ i + (v sin θ − gt) j ] dt
˜ ˜
= vt cos θ i + (vt sin θ − 1--- gt2) j + c
˜ 2 ˜
˜
If the origin is the launch point of the projectile, then
r (0) = 0 i + 0 j and so c = 0 i + 0 j .
˜ ˜ ˜ ˜ ˜ ˜
Hence r = vt cos θ i + (vt sin θ − 1--- gt2) j .
˜ ˜ 2
˜
The horizontal component of the displacement, rx, is given by rx = vt cos θ and the
vertical component of the displacement, ry , is given by ry = vt sin θ − 1--- gt2.
2

What do we do if the launch point is not at the origin?

Now if the launch point is at (a, b), then r (0) = a i + b j y


˜ ˜ ˜ v
and so c = a i + b j . P
˜ ˜ ˜
r = vt cos θ i + (vt sin θ − 1--- gt2) j + c becomes b
˜ ˜ 2 ˜
˜
r = (a + vt cos θ) i + (b + vt sin θ − gt ) j
1--- 2
O a x
˜ ˜ 2
˜

Characteristics of flight
The launch velocity and angle of projection determine such things as the time of flight,
greatest height reached above the point of projection and the range.
The period of time for which the projectile is moving under the influence of gravity
alone is called the time of flight of the projectile. The range is the horizontal displace-
ment of the projectile from its launch point. To determine the greatest height reached,
we must find when the vertical component of velocity is zero; that is, solve v y = 0
for t. The vertical component of the displacement using this value for t determines the
greatest height reached.
Chapter 11 Dynamics 527
WORKED Example 5
A projectile is fired from the foot of a cliff and its displacement (in metres) at time
t seconds is given by r (t) = 10t i + (24t - 1--- gt2) j .
˜ ˜ 2
Use g = 9.8 m/s2 in your calculations. ˜
a Determine an expression for the velocity vector of the projectile at time t seconds.
b Calculate the launch speed and angle of projection of the projectile.
c Find the greatest height reached and the time of flight of the projectile.
d Calculate the impact velocity of the projectile when it returns to the ground (as a
horizontal plane through the origin).

THINK WRITE

a Find the derivative of the displacement a r (t) = 10t i + (24t − 1--- gt2) j
˜ . ˜ 2
˜
vector to obtain an expression for the v (t) = r (t)
velocity vector. ˜ ˜
v (t) = 10 i + (24 − gt) j
˜ ˜ ˜
b 1 To find the launch speed, substitute b At launch, t = 0. Therefore,
t = 0 into the velocity vector and v (0) = 10 i + 24 j
then calculate the magnitude. ˜ ˜ ˜
2 2
v (0) = 10 + 24
˜
= 26

2 Draw a diagram of the velocity Let θ be the angle of projection.


vector at t = 0 to assist in
y ~v (0) = 10i~ + 24j
~
calculating the angle of projection
or launch angle.
26
24

O 10 x

3 Use trigonometry to obtain an tan θ = 24


------
10
equation to solve for θ.
θ ≈ 67°23′
4 Write the answer. The projectile is launched with a speed of
26 m/s upwards at an angle of 67°23′ to the
horizontal.
c 1 The greatest height occurs when c If the vertical component of v is vy then
˜
the vertical component of the vy = 24 − gt
velocity is 0. At the greatest height, vy = 0.
0 = 24 − gt
t= 24
------
g
≈ 2.45 s

Continued over page


528 M a t h s Q u e s t M a t h s C Ye a r 1 1 f o r Q u e e n s l a n d

THINK WRITE
2 To calculate the greatest height, When t = 24-
----- , the vertical component of
g
substitute for t in the expression for the displacement, ry, becomes:
vertical component of the displacement
vector. ry  24
------ = 24 ×

24
------ − 1---
×g×  24
------
 g
2
g g 2

= 288
---------
g

= 288
---------
9.8
≈ 29.4 m
The greatest height reached is 29.4 m.
3 For the time of flight, calculate when Projectile returns to the ground when
the vertical component of the ry(t) = 0.
displacement is 0; that is, the time when 24t − 1--- gt2 = 0
the projectile returns to the ground. 2
t(24 − 1--- gt) = 0
Note that there are two solutions for t 2
when the displacement is 0. t = 0 or t = 48 ------
g
The solution t = 0 gives the time when the
projectile is launched from the ground so
the projectile returns to the ground at
t = 48 ------ s (or ≈ 4.90 s).
g
Hence, the time of flight of the projectile
is 4.90 s.
d 1 Since we know that the projectile returns d v (t) = 10 i + (24 − gt) j
˜ ˜ ˜
to the ground at t = 48
------ s, substitute for t in v  48
------ = 10 i + (24 − g × 48 ------ ) j
g
˜ g  ˜ g
the velocity vector. ˜
= 10 i − 24 j
˜ ˜ velocity
2 Calculate the magnitude of the velocity The magnitude of the
at this time. (This is the speed.) 2 2
= 10 + ( – 24 )
= 26 m/s
3 Find the direction of the projectile at Let θ be the angle the velocity vector at
impact. Use a diagram showing the t= 48
------ s makes with the horizontal.
g
velocity vector at t = 48
------ s and
g y
trigonometry to find the required angle. 10 x
O
24
26

~v —g = 10i~ – 24j
( 48 )
~
tan θ = 24
------
10
θ ≈ 67°23′
4 Write the answer. At impact, the velocity of the projectile is
26 m/s downwards at an angle of 67°23′
to the horizontal.
Chapter 11 Dynamics 529
Consider the calculations in Worked example 5. What conclusions can you draw when
you compare the launch velocity and the impact velocity of the projectile? What can
you conclude about the time taken for the projectile to reach its greatest height com-
pared with the total time of the flight? What condition must you have for these observ-
ations to be true in other situations?

WORKED Example 6
A particle is fired from the top of a 125 m high cliff with a velocity of 50 m/s inclined at an
angle of 60∞ to the horizontal. Use g = 9.8 m/s2 in your calculations.
y
50 m/s
A
60º

125 m 150 m

B
O 200 m x

a Show that the velocity vector of the projectile at time t seconds is given by
v (t) = 25 i + (25 3 − gt) j m/s.
˜ ˜ ˜
b Develop a vector expression for the displacement, r (t), of the particle at time t seconds.
˜
c Determine if this projectile will clear a 150 m high building situated 200 m from the
base of the cliff and in the same horizontal plane.

THINK WRITE
a 1 Write an expression for the acceleration a For projectile motion,
vector. In projectile motion, the a = −g j
horizontal component of a is 0 and the ˜ ˜
˜
vertical component is −g. (The negative
signifies a downward direction.)
2 Integrate the expression for a to find v .
˜ ˜
v =
˜ ∫ a˜ dt
=
∫ (−g ˜j ) dt
= −gt j + c 1
˜ ˜
3 Determine the initial velocity. Draw a
diagram to represent the launch
velocity and its horizontal and vertical
50 vy
components.

60º
vx

4 Write expressions for the horizontal vx = 50 cos 60°


component, vx, and the vertical vy = 50 sin 60°
component, vy. Continued over page
530 M a t h s Q u e s t M a t h s C Ye a r 1 1 f o r Q u e e n s l a n d

THINK WRITE
5 Use the components to write the Since v = vx i + vy j
expression for the velocity vector at ˜ ˜ ˜
v (0) = 50 cos 60° i + 50 sin 60° j
t = 0. This is the initial or launch ˜ ˜ ˜
velocity of the particle. = 25 i + 25 3 j
˜ ˜
6 Apply the initial condition (launch When t = 0, v = 25 i + 25 3 j and so
velocity) to determine the constant ˜ ˜ ˜
vector of integration. 25 i + 25 3 j = 0 j + c 1
˜ ˜ ˜ ˜
c 1 = 25 i + 25 3 j
˜ ˜ ˜
7 Substitute the expression for c 1 into v . v = −gt j + 25 i + 25 3 j
˜ ˜ ˜ ˜ ˜ ˜
8 Simplify to write the vector expression Hence the velocity vector is
for the velocity of the particle. v = 25 i + (25 3 − gt) j
˜ ˜ ˜
b 1 Integrate the expression for v to find r .
˜ ˜
b r =
˜ ∫ v˜ dt
=
∫ [25 ˜i + (25
3 − gt) j ] dt
˜
= 25t i + (25 3 t − 1--- gt2) j + c 2
˜ 2 ˜
˜
2 Apply the initial condition to determine When t = 0, r = 0 i + 125 j and so
the constant vector of integration. The ˜ ˜ ˜
0 i + 125 j = 0 i + 0 j + c 2
initial position of the particle is at the ˜ ˜
c 2 = 0 i +˜ 125 j ˜ ˜
point (0, 125). ˜ ˜ ˜

3 Substitute the expression for c 2 into r Hence the displacement vector is


˜ ˜ r = 25t i + (125 + 25 3 t − 1--- gt2) j
and write the vector expression for the ˜ ˜ 2
˜
displacement of the particle.
c 1 Write an expression for the range of the c Range = rx
projectile. The horizontal component of = 25t
r gives the range.
˜
2 Calculate when the range is 200 m. When the range is 200 m,
25t = 200
t=8s
3 Write an expression for the height of Height of projectile = rx
the projectile. The vertical component = 125 + 25 3 t − 1--- gt2
of r gives the height. 2
˜
4 Substitute t = 8 to calculate the height When t = 8,
of the projectile at this time. ry = 125 + 25 3 × 8 − 1
--- × g × 82
2
≈ 157.8 m
5 Write the answer. Since the height of the projectile is
about 157.8 m when it reaches a range
of 200 m, the projectile easily clears the
150 m high building.
Chapter 11 Dynamics 531
In the next worked example, we examine how to find the angle of projection of a par-
ticle, given prescribed data. Even though the principles and techniques are the same as
those used in solving purely numeric problems, we need to employ some trigonometry
in order to reach the solution.
Note: The angle of projection that gives a projected particle its greatest range is often
intuitively accepted as an angle of elevation of 45° … but this is true only when the
point of projection and point of impact are on the same horizontal plane (or more accu-
rately, at the same elevation).

WORKED Example 7
A missile is fired from a point on level ground with a y
Target
velocity V m/s and an angle of elevation of q to the
horizontal. The target is positioned on top of an 80 m high V
tower which is located 100 m away. The base of the tower 80 m
is in the same horizontal plane as the point of projection
O 100 m x
of the missile.
a If g is the gravitational acceleration of the missile, then show that the motion of this
missile satisfies V2 cos q (5 sin q − 4 cos q) = 250g.
b Use a graphics calculator to show those values of q in the domain 0 < q < 90° for which
the expression cos q (5 sin q − 4 cos q) is positive. State the greatest possible value this
expression has and the value of q which produces it. Provide supporting argument for
your solutions.
c Extend from your answer to part b above to find the least value of V for which the
missile can reach the target. (Use g = 9.8 m/s2.)
d Generate a spreadsheet to validate the solution obtained using a graphics calculator.

THINK WRITE
a 1 Use a first principles approach to a a = −g j
develop an expression for the ˜ ˜
velocity of the missile at time t
seconds after launch.
˜ ∫
v = a dt
˜

∫ ˜
= (−g j ) dt
= −gt j + c 1
˜ ˜
2 Determine the initial velocity. Draw
a diagram to represent the launch
velocity and write expressions for V vy
its horizontal and vertical
components.
vx

vx = V cos θ and vy = V sin θ


Since v = vx i + vy j
˜ ˜ ˜
v (0) = V cos θ i + V sin θ j
˜ ˜ ˜
Continued over page
532 M a t h s Q u e s t M a t h s C Ye a r 1 1 f o r Q u e e n s l a n d

THINK WRITE
3 Apply the initial condition When t = 0, v = V cos θ i + V sin θ j and
(launch velocity) to determine the ˜ ˜ ˜
constant vector of integration in v = −(g × 0) j + c 1 so
˜ ˜ ˜
v = −gt j + c 1. V cos θ i + V sin θ j = 0 j + c 1
˜ ˜ ˜ ˜ ˜ ˜ ˜
c 1 = V cos θ i + V sin θ j
˜ ˜ ˜
4 Substitute the expression for c 1 v = −gt j + V cos θ i + V sin θ j
into v = −gt j + c 1. ˜ ˜ ˜ ˜ ˜
˜ ˜ ˜
5 Simplify to write the vector Hence the velocity vector is
expression for the velocity of the v (t) = V cos θ i + (V sin θ − gt) j
missile at time t seconds. ˜ ˜ ˜
Integrate the expression for v to
∫ v˜ dt
6 r =
find r . ˜ ˜
˜

= [V cos θ i + (V sin θ − gt) j ] dt
˜ ˜
= Vt cos θ i + (Vt sin θ − 1--- gt2) j + c 2
˜ 2 ˜
˜
7 Apply the initial condition to When t = 0, r = 0 i + 0 j and so
determine the constant vector of ˜ ˜ ˜
integration. The initial position 0i + 0 j = 0i + 0 j + c 2
˜ ˜ ˜ ˜ ˜
of the missile is at the c 2 = 0i + 0 j
point (0, 0). ˜ ˜ ˜

8 Substitute for c 2 and write the Hence the displacement vector is


˜ for the
vector expression r (t) = Vt cos θ i + (Vt sin θ − 1--- gt2) j
displacement of the missile at time ˜ ˜ 2
˜
t seconds.
9 Write an expression for the range of Range of missile is given by
the missile at time t. Use the given rx = Vt cos θ
information that the range to the When rx = 100,
target is 100 m to obtain an Vt cos θ = 100
expression for t. 100
t = ----------------
V cos θ

10 Write an expression for the height Height of missile is given by


of the missile at time t. Use the ry = Vt sin θ − 1--- gt2
2
information that the height of the
target at the given range is 80 m 100
When ry = 80, t = ---------------- and so
100 V cos θ
and that this occurs at t = ----------------
V cos θ
Vt sin θ − 1--- gt2 = 80 becomes
seconds. 2

100 100 2
 ---------------
V × ---------------- × sin θ − 1--- g × - = 80
V cos θ 2  V cos θ
Chapter 11 Dynamics 533
THINK WRITE
11 Continue simplifying. Multiply both 100 sin θ 5000g -
-------------------- − ------------------- = 80
sides of the equation by V2 cos2 θ. cos θ 2 2
V cos θ
2 2 2 2
100 sin θ × V cos θ 5000g × V cos θ
----------------------------------------------- − ------------------------------------------
cos θ 2
V cos θ
2

= 80 V 2 cos2 θ
100V 2 sin θ cos θ − 5000g = 80 V 2 cos2 θ
20V 2 cos θ (5 sin θ − 4 cos θ) = 5000g
V 2 cos θ (5 sin θ − 4 cos θ) = 250g
(as required)
b 1 Use a graphics calculator to graph the b
function f(θ) = cos θ (5 sin θ − 4 cos θ).
You will need to use x in place of the
variable θ.
For the Casio fx-9860G AU
(a) Press MENU and select GRAPH .
Ensure that your calculator is set to
degrees. (Press SHIFT [SET UP] to
change the Angle setting.) Press F3
(TYPE) followed by F1 (Y=) to select
the function option. Complete the entry
line for Y1 with the expression
(cos x) (5 sin x − 4 cos x) and press EXE .
(b) Press F6 (DRAW) to display the
graph. To obtain a clearer view of the
graph, you can adjust the View
Window settings. Press SHIFT F3
(V-WIN) and adjust the values for
Xmin, Xmax, Ymin and Ymax.
(c) Press EXE until you return to the
function screen. Press F6 (DRAW) to
display the graph with the new setting.
The graph shows where the expression
is positive for 0 < x < 90°.
Use the Trace function (press
SHIFT F1 (TRCE)) to investigate
points along the line. (You can also
find the maximum value for the
expression and the value of x which
produces it in this way.)
(d) To find the maximum value (other
than using the Trace function), press
SHIFT F5 (G-SLV) followed by F2
(MAX). The coordinates of the
required point are displayed.
Continued over page
534 M a t h s Q u e s t M a t h s C Ye a r 1 1 f o r Q u e e n s l a n d

THINK WRITE
For the TI-Nspire CAS
(a) Ensure that your calculator is set to
degrees. (Access the home screen
and select 8: System Info to
change the Angle setting.)
Open a new Graphs & Geometry
document. Complete the entry
line for f1(x) with
cos (x)(5 sin(x) − 4 cos(x)).
(b) Press · to display the graph of the
function. To adjust the viewing
window, press b, select
4: Window and 1: Window
Settings. Enter the settings as
shown.

(c) Highlight OK and press ·. The


graph shows where the expression
is positive for 0 < x < 90˚.

(d) From the home screen (press c),


select 1: Calculator. Press b and
select 4: Calculus followed by
7: Function Maximum. Press
F1(X),X), then press
·. This gives the value of x (or θ)
for the maximum value of f(x)
(or f(θ)).
(e) To find the function value for this x
value, press F1( followed by
/v to copy the answer value
from above and then ) to close
the set of brackets. Press · to
display the maximum value.

2 Write a conclusion from your The graph shows where the expression
observations and calculations. cos θ (5 sin θ − 4 cos θ) is positive in the
domain 0 < θ < 90°. There is one local
maximum in this domain. The greatest
possible value of this expression is approx.
1.202 when the angle is approx. 64.3°.
Chapter 11 Dynamics 535
THINK WRITE
c 1 Rearrange the equation from part a to c V 2 cos θ (5 sin θ − 4 cos θ) = 250g
make V 2 the subject of the equation. 250g
V 2 = -------------------------------------------------------
cos θ ( 5 sin θ – 4 cos θ )

2 The least value of V 2 occurs when The least value of V 2 occurs when
the denominator is greatest. Use cos θ (5 sin θ − 4 cos θ) is greatest.
the maximum value for The maximum value of
cos θ (5 sin θ − 4 cos θ) cos θ (5 sin θ − 4 cos θ) is 1.202
found in part b. (from part b).
250 × 9.8
3 Substitute the maximum value for the V 2 = ----------------------
denominator and the value for g to 1.202
calculate the least value for V for Since V > 0, then V ≈ 45.1.
which the missile can reach the target.
4 Write the answer. The required least velocity of the missile to
reach the target is approx. 45.1 m/s at an
angle of elevation of approx. 64.3°.
d 1 Set up a spreadsheet with 3 columns d Theta 64.3299
titled t, x and y. In the first column,
enter values for t. Enter the Velocity 45.1000
appropriate parametric equations of Delta t 0.2500
the displacement as x = Vt cos θ and
y = Vt sin θ − 1--- gt2 using the calculated t x y
2
value for V and θ and substituting
9.8 for g. Generate corresponding 0 0 0
values in columns 2 and 3. 0.25 4.884 9.856
0.50 9.768 19.099
eBook plus 0.75 14.653 27.730
.......

.......

.......

.......

Digital doc:
EXCEL Spreadsheet
Missile flight 4.75 92.800 82.525
5.00 97.684 80.744
5.25 102.568 78.350

2 Generate the graph to provide a visual y


Missile flight
validation of the results. 90
80
70
60
Metres

50
40
30
20
10
0 x
0 20 40 60 80 100 120
Metres
536 M a t h s Q u e s t M a t h s C Ye a r 1 1 f o r Q u e e n s l a n d

remember
remember dv
1. Finding the velocity given the acceleration: a = -----˜- → v = a dt.
˜ dt ˜ ˜ ∫
dr
2. Finding the displacement given the velocity: v = -----˜ → r = v dt.
˜ dt ˜ ˜ ∫
Remember that the constant of integration is always a vector constant.
3. The period of time for which the projectile is moving under the influence of
gravity alone is called the time of flight of the projectile.
4. The range is the horizontal displacement of the projectile from its launch point.

11B Projectile motion


SLE 3: Given the force on an Unless otherwise stated, use g = 9.8 m/s2.
object as a function of time
and suitable prescribed 1 Given the following information about the motion of a particle at time t, determine a
conditions, such as velocity
and displacement at certain vector expression for the particle’s displacement.
times, use integration to find
the position vector of the a v (t) = i and r (0) = 3 i + 2 j r (t) = (t + 3) i + 2 j
object. ˜ ˜ ˜ ˜ ˜ ˜ ˜ ˜
b v (t) = 2t i − 3 j and r (0) = i + j r (t) = (t2 + 1) i + (1 − 3t) j
˜ ˜ ˜ ˜ ˜ ˜ ˜ ˜ ˜
dr
c -----˜ = 4 i + 7t j and r (1) = 2 i − j r (t) = 2(2t − 1) i + --12- (7t2 − 9) j
dt ˜ ˜ ˜ ˜ ˜ ˜ ˜ ˜
dr
d -----˜ = (3 + t) i − 2 j and r (2) = i + 5 j r (t) = (3t + 2 t − 7) i + (−2t + 9) j1---
dt ˜ ˜ ˜ ˜ ˜ ˜ ˜ ˜
.
e r (t) = 3t i + (4 + t) j and r (0) = − i + j r (t) = ( 2 t − 1) i + (4t + 2 t + 1) j
3--- 2 1--- 2
˜ ˜ ˜ ˜ ˜ ˜ ˜ ˜ ˜
. = − − − t + 18) j
f r (t) = (2 − t) i + (6 + t ) j and r (0) = 0 i + 0 j ˜
2 r ( t ) 1---
2
t(t 4) i 1--- 2
t(
˜ ˜ ˜ ˜ ˜ 3 ˜
˜ ˜
g v (t) = 15 i + (10 − 3t) j and r (0) = 20 j r (t) = (15t) i + (10t − 3--2- t2 + 20) j
˜ ˜ ˜ ˜ ˜ ˜ ˜ ˜
dr  1 
h -----˜ = −2t i + 5 + ---- j and r (1) = 2 i − 7 j r (t) = (−t + 3) i + (5t − t − 11) j 2 1---
dt ˜  2
t ˜ ˜ ˜ ˜ ˜ ˜ ˜
WORKED 2 Determine vector expressions at time t for the velocity, v , and displacement, r ,
Example
xample
of a particle which moves according to the following ˜ relationships involving ˜
4
acceleration, a .
˜
a a (t) = 2 i and when t = 0, v = i − 3 j and r = − i + 5 j 2
r (t) = (− --12- t2 + 3t + 2) i + ( --16- t3 − 4t − 3) j
˜

e
˜ ˜ ˜ ˜ ˜ ˜ ˜ ˜
r (t) = ( 3 t − t + 1) i + (− 2 t − t + 1) j
˜

dv
r (t) = ( --1- t3 + --1- t2 − -----

r (t) = ( 2--1- t2 + 2--7- t − 5) i + (− 6--1- t3 + 3t˜ 2 − -----

r (t) = (− --5- t2 + -----


v (t) = (t2 + t − -----

v (t) = (t + 2--7- ) i + (− 2--1- t2 + 6t − -----

v (t) = (−5t + -----


v (t) = (3t − 6) i + (− 2--5- t2 + 12t − -----
˜

˜
r (t) = ( 2--3- t2 − 6t − -----

˜
˜

b -----˜- = 3 i − 5 j and when t = 0, v = 2 i − j and r = 0 i + 0 j


˜

dt ˜ ˜ ˜ ˜ ˜ ˜ ˜ ˜
r (t) = (t2 + t − 1) i +˜ (−3t + 5) j
˜

r (t) = ( 3--2- t2 + 2t) i + (− 5--2- t2 − ˜t) j


˜

..
b v (t) = (3t + 2) i + (−5t − 1) j ,

c v (t) = (−t + 3) i + ( 1--2- t2 − 4) j ,

d v (t) = (t2 − 1) i + (−3t − 1) j ,

c r = − i + t j , v (0) = 3 i − 4 j and r (0) = 2 i − 3 j


˜

3--- 2˜

˜ ˜ ˜ ˜ ˜ ˜ ˜ ˜ ˜
.. .
˜
2

d r = 2t i − 3 j , r (1) = − 4 j and r (0) = i + j


˜
a v (t) = (2t + 1) i − 3 j ,

15-
˜

2 ˜
15-
2
23-

) i + ( 2--1- t2 + t − -----
6 ˜

˜ ˜ ˜ ˜ ˜ ˜ ˜
˜

t + 3) i + ( --1- t3 + 2--1-˜t2 − -----

˜
) i + ( 2--5- t2 − 3t + 3--5- ) j ,
11
˜

2 ˜
- ) i + (− --5- t + 6t − -----

dv
˜

23-
6

e -----˜- = −5 i + (t + 1) j , r (0) = 3 i + 4 j and r (2) = 8 i + 3 j


˜
˜
˜

t) i + ( --5- t3 − --3- t2˜+ 3--5- t + 8) j


˜

dt
˜

˜ ˜ ˜ ˜ ˜ ˜ ˜ ˜
1--- 3

f a (t) = i + (6 − t) j , r (0) = −5 i + 12 j and r (1) = − i + 4 j


6

6
6
3

˜ ˜ ˜ ˜ ˜ ˜ ˜ ˜ ˜
65-
6

13-

dv
6
35-

g -----˜- = (2t + 1) i − (3 − 5t) j , r (0) = 8 j and r (1) = −3 i + 9 j


)j,
2

)j,
2
)j,

dt ˜ ˜ ˜ ˜ ˜
˜

˜
˜

˜ ˜
65-

..
35

6
2

13-
6

h r = 3 i + (12 − 5t) j , v (3) = 3 i − 4 j and r (5) = 2 i − 3 j


- t + --------

t + 12) j

t + 4) j

˜ ˜ ˜ ˜ ˜ ˜ ˜
2

˜ ˜
116
3
-) j

˜
˜

˜
˜
Chapter 11 Dynamics 537
WORKED 3 A projectile is fired from the foot of a cliff and its displacement (in metres) at time
Example
t seconds is given by r (t) = 12t i + (15t − 1--- gt2) j . v (t) = 12 i + (15 − gt) j
5 ˜ ˜ 2
˜ ˜ ˜ ˜
a Determine an expression for the velocity vector of the projectile at time t seconds.
19.2 m/s at an
angle of 51°20′ b Calculate the launch speed and angle of projection of the projectile.
above the c Find the greatest height reached and the time of flight of the projectile. 11.5 m 3.06 seconds
horizontal
d Calculate the impact velocity of the projectile when it returns to the ground in a
horizontal plane through the origin. 19.2 m/s downwards at an angle of 51°20′ to the horizontal
4 A projectile is fired from the origin, O, and its displacement (in metres) at time t sec-
onds is given by r (t) = 15t i + (30t − 1--- gt2) j .
˜ ˜ 2
v (t) = 15 i + (30 − gt) j
˜ ˜
a Determine an expression for the velocity˜ vector of the projectile at time t seconds.
˜ 33.5 m/s at an angle of
b Calculate the launch speed and angle of projection of the projectile.
63°26′ above the
c Determine the greatest height reached by the projectile. 45.9 m horizontal
d Will this projectile clear a 4 m high wall located 90 m from the launch point in the
same horizontal plane? No, as it falls short by about 40 cm.
WORKED 5 A particle is fired from the top of a 50 m high y
Example
6
cliff with a velocity of 36 m/s inclined at an
angle of 30° to the horizontal.
30º

50 m

O x

a Show that the velocity vector of the projectile at time t seconds is given by
v (t) = 18 3 i + (18 − gt) j m/s.
˜
b Develop ˜ expression
a vector ˜ for the displacement, r (t), of the particle at time
t seconds. r (t) = 18 3 t i + (50 + 18t − --12- gt2) j ˜
˜ ˜ Maximum height is 66.5 m which is
47.7 m/s downwards ˜
c Calculate the greatest height reached by the projectile. 16.5 m above the launch platform.
at an angle of 49°12′
to the horizontal d What is the impact velocity of the projectile when it reaches a point in the hori-
zontal plane through the origin, O?
Particles don’t e At this impact point, what is the range of the projectile? 172.1 m
collide but they do
pass through two 6 At time t, the velocities of two particles A and B are given by
common points at dr B
different times. At v A(t) = 4 i + (5 − 2t) j and --------
˜ = −3 i + (4t − 7) j . r A(t) = 4(t − 1) i + (3 + 5t − t2) j ,
˜ ˜ ˜ dt ˜ ˜ ˜ ˜
(13.70, 5.55), one r B(t) = 3(8 − t) i + (6 − 7t + 2t2)˜ j
particle arrives when When t = 0, r A = −4 i + 3 j and r B = 24 i + 6 j . ˜ ˜ ˜
t = 4.43 s and the ˜ ˜ ˜ ˜ ˜ ˜
a Determine vector expressions for the displacement of both particles at time t.
other when t = 3.43 s.
At the point b Do these particles ever collide? If so, calculate when and where. If not, explain
(18.20, −0.06), one what happens. yA = 3 + -----
1-
(x + 4)(16 − x),
particle arrives when c Find the Cartesian equations of the trajectories of the particles. y = 6 +
16
1-
(24 − x)(27 − 2x)
t = 5.55 s and the B --
d Use a graphics calculator to verify your response to part b above. 9
other when t = 1.93 s.
7 The acceleration of a particle λ, at time t, is given by a λ(t) = −8 j . For this particle,
˜
r λ(2) = −4 i + 3 j and r λ(5) = 11 i + 15 j . The displacement of a˜ second particle, ξ,
˜ ˜ ˜ ˜
at time t, is given˜ by r ξ (t) = (6 + t) i + (15t
˜ − 10 − 2t2) j . r λ(t) = (5t − 14) i + (32t − 45 − 4t2) j
˜ ˜ ˜ ˜ ˜
a Determine the vector expression for the displacement˜ of the particle, λ, at time, t.
b Do these particles ever collide? If so, calculate when and where.
c Validate your response to part b above by using suitable technology.
When t = 5 s, the particles do collide at (11, 15). Particles appear to collide at (9.15, 17.4) but actually pass
through this point at different times; so they don’t collide. Particle λ at t = 4.63 s and particle ξ at time t = 3.15 s.
538 M a t h s Q u e s t M a t h s C Ye a r 1 1 f o r Q u e e n s l a n d

WORKED 8 A missile is fired from a point on level ground with a velocity V m/s and an angle of
Example
xample
elevation of θ to the horizontal. The target is positioned on top of a 40 m high tower
7
which is located 80 m away. The base of the tower is in the same horizontal plane as
the point of projection of the missile.
y

40 m

O 80 m x

a If g is the gravitational acceleration of the missile, then show that the motion of
this missile satisfies V 2 cos θ (2 sin θ − cos θ) = 80g.
b Use a graphics calculator to show those values of θ in the domain 0 < θ < 90° for
which the expression cos θ (2 sin θ − cos θ) is positive. State the greatest possible
value this expression has, and the value of θ which produces it. Provide supporting
argument for your solutions. 0.618 when θ ≈ 58°17′
c Extend from your answer to part b above to find the least value of V for which the
missile can reach the target. (Use g = 9.8 m/s2.) 35.6 m/s
SLE 16: Use spreadsheets to d Generate a spreadsheet to validate the solutions obtained using a graphics
investigate problems.
calculator.

9 A particle is projected with an initial speed y


of 35 m/s in such a direction that it just
passes over the top of a 5 m high wall. The 35 m/s
foot of the wall is situated 50 m from the
point of projection of the particle and, as 5m
shown at right, is in the same horizontal
plane through the point of projection.
O 50 m x
Determine all possible angles of projec-
tion, θ. θ ≈ 17°46′ and θ ≈ 77°56′

10 A particle is projected horizontally from y


the top of a tower 10 m high with an initial
speed of 15 m/s. 21.429 m 15 m/s

If the surrounding ground is horizontal,


how far from the base of the tower does the 10 m
particle land?

O x

7
11 A projectile is fired at an angle θ below the horizontal, where tan θ = -----
24
- , from the top
of a cliff, 75.6 m high, which overlooks the sea. If the projectile reaches the sea at the
instant 2 s after projection, find its initial speed and the distance from the base of the
cliff to the point of impact of the projectile with the sea. 100 m/s, 192 m
Chapter 11 Dynamics 539
12 Two particles, P and Q, are fired simultaneously from points A and B respectively
which are located 60 m apart on horizontal ground, as shown below. The particle P has
an initial speed of 24 m/s and an angle of projection θ, where θ = tan−1  3--- . The particle
4
Q has an initial speed of 18 m/s with an angle of projection φ where φ = tan−1  4--- .
3
y

Do these particles collide? If 24 m/s 18 m/s


so, when and where do they
collide? Yes, they meet at (38.4, 9.2)
when t = 2 s.

ø
A 60 m B x

13 Two walls are situated 16 m apart on y


level ground. One wall is 7.1 m high
and the other is 4.4 m high. A projec- V
tile is to be fired from the point on the
ground which is 16 m from the 7.1 m
high wall and on the side remote from 7.1 m 4.4 m
the second wall as seen in the figure at
right. O 16 m 16 m x
Determine the initial speed and angle of projection required if this projectile will just
clear both walls. 20 m/s at an angle of approx. 36°52′
14 A particle is projected with an initial speed of 35 m/s from the base of a plane
inclined at an angle φ to the horizontal where φ = tan−1  3--4- . If the projectile is initially
angled at θ to the horizontal, where tan θ = 5--- , find how far along the plane the projec-
2
tile lands, and the time of flight. 75.431 m, approx. 4.64 s
y
15 A particle is projected up a plane
inclined at an angle θ to the horizontal,
the angle of projection being φ above
V
the horizontal.
If the initial speed of projection is V,
a show that the range, R, on this inclined
plane is given by
2 ø
sin ( φ – θ ) cos φ
R = 2V-----------------------------------------------
g cos θ
2
O x
b determine the maximum range (Rmax) on the plane and the value of φ for which
2
this occurs. Rmax = ------------------------- - when φ = 45° + --θ-
V
(1+sin θ )g 2

16 A sprinkler sprays water symmetrically about its vertical axis at a constant speed of V.
The initial direction of the spray varies continuously between angles of 15° and 60° to
the horizontal.
a Prove that from a fixed position, O, on level ground, the sprinkler will wet the sur-
eBook plus 2 2
V V
face of an annular region with respective internal and external radii ------ and ------ .
2g g
Digital doc:
b Now show that if the sprinkler is located appropriately to a rectangular garden bed
WorkSHEET 11.1
2
V
of size 6 m by 3 m, the entire garden will be watered provided ------ ≥ 1 + 7 .
2g
540 M a t h s Q u e s t M a t h s C Ye a r 1 1 f o r Q u e e n s l a n d

Motion under constant acceleration


Even though our study in this course is restricted
to motion involving constant acceleration, this
study does include many situations of practical
significance.
The common example of bodies in free fall
towards the Earth’s surface, regardless of their
size, mass or composition, involves motion under
constant acceleration — that of acceleration due
to gravity.
Galileo illustrated this property by dropping
various sized cannon balls from the top of the
Leaning Tower of Pisa. Provided the effects of air
resistance are minimised (that is, the object
should not be too large in area or fall too far) all
bodies will fall with constant acceleration. Other
variations will be considered later in your studies.
Since the acceleration due to gravity depends
on the distance from the centre of the Earth, it
varies slightly at different places on the Earth’s
surface — being greater at the poles than at the
equator and less at high altitudes. It is usual to
state the constant of acceleration due to gravity as
9.8 m/s2 unless you are told otherwise.
Since acceleration is constant, the displacement
of a falling object can be modelled by the general
quadratic expression
lt 2 + mt + n
This can be used to model the motion of a body in free fall or one tossed upwards.
The important difference is that the velocity and acceleration are in different directions.
When vector notation is used the vertical displacement vector is written as
r = (lt 2 + mt + n) j
˜ ˜
Because j does not depend on t the derivative of this function is as follows:
˜ v = (2lt + m) j
˜ ˜
and a = 2l j
˜ ˜
The following analysis examines the motion of a ball as it rests at the top of a cliff.
As soil falls away the ball begins to move.
(a) At t = 0, find v.
At t = 0 the ball is at rest, therefore v = 0.
(b) If the bottom of the cliff is the origin and the cliff is h metres high, find r when
t = 0.
At t = 0, r = h j
˜ ˜
(c) Given the constant acceleration towards the origin equals −9.8 m/s2 (a negative
value is used to indicate that the direction of the acceleration is downwards), find
m and l.
Chapter 11 Dynamics 541
Use the original equation for displacement and differentiate:
r = (lt2 + mt + n) j [1]
˜ ˜
v = (2lt + m) j [2]
˜ ˜
a = 2l j [3]
˜ ˜ when t = 0
From part (a) above
v = [2l(0) + m] j in [2]
˜ ˜
but v = 0 (initially at rest)
˜
Therefore, m = 0
From part (b) above, if at t = 0, r = h j
˜ ˜ [1]
r = (lt 2 + mt + n) j
˜ ˜
r = [l(0) + m(0) + n] j
2
˜ ˜ at t = 0)
=hj (as given
Therefore, n=h ˜
From (c) g = −9.8 m/s2 so a = −9.8 j (as a = g j )
˜ ˜ ˜ ˜
Also a = 2l j
˜ ˜
So 2l j = −9.8 j
Therefore, ˜l = −4.9 ˜
Therefore the general equation for this motion can be written:
r = (−4.9t 2 + h) j
˜
Do not attempt to memorise ˜these equations. Equations [1], [2] and [3] are the basic
equations and all others are specific rearrangements of these.
In general,
1. l can be readily found if the acceleration is given: a = 2l.
2. m can be readily found if the initial velocity is given: v = 2l(0) + m.
3. n can be readily found if the initial displacement is given: r = l(0)2 + m(0) + n

WORKED Example 8
Examine the motion of a stone thrown upwards with a velocity of 6 m/s from the top of a
cliff 60 m high. Find the time taken for it to reach the ground and its velocity on impact.

THINK WRITE
1 As the acceleration is constant, the r = (lt2 + mt + n) j
vertical displacement is quadratic. v˜ = (2lt + m) j ˜
Write the three equations of motion. a˜ = 2l j ˜
˜ ˜
2 The position of the origin needs to be Let the origin be the top of the cliff.
stated. Generally where the motion
begins or finishes can be thought of as
the origin.
3 Use the given information to find each At t = 0, r = 0 j and
value of n, m and l. First use the initial r˜ = [l(0)
˜ 2 + m(0) + n] j
position to find n. So ˜
0=0+0+n ˜
therefore n = 0
4 Rewrite the equation for r with the r = (lt 2 + mt) j
new information. ˜ ˜ ˜
Continued over page
542 M a t h s Q u e s t M a t h s C Ye a r 1 1 f o r Q u e e n s l a n d

THINK WRITE
5 Use the equations for a to find l. a = 2l j and
˜
Constant downwards acceleration away a˜ = −9.8
˜ j
from the origin so the acceleration is So ˜ = −9.8 ˜
2l
negative. l = −4.9
r = (−4.9t 2 + mt) j
˜ ˜
6 Use the initial velocity to find m. Initial At t = 0, v = 6 or v = 6 j
velocity at t = 0 is 6 m/s up (positive ˜ j ˜
6 j = [2l(0) + m]
sign). m˜ =6 ˜

7 Rewrite r with all current information. r = (−4.9t 2 + 6t) j


˜ ˜ ˜
8 Find the time when the stone reaches At r = −60 j
the ground at r = −60 j . ˜ −60 j ˜ = (−4.9t 2 + 6t) j
Drop the vector˜ notation.
˜ ˜0 = −4.9t 2 + 6t +˜ 60
Solve for t using the quadratic formula 2
where a = −4.9, b = 6 and c = 60. –b ± b – 4ac
t = -------------------------------------
2a
–6 ± 36 – 4 ( – 4.9 ) ( 60 )
= -----------------------------------------------------------
2 ( – 4.9 )
t = 4.16 s or –2.94 s
Reject the negative solution.
Therefore the stone reaches the ground 4.16 s
after it was thrown upwards.
9 Find the velocity on impact by finding v v = (−9.8t + 6) j
when t = 4.16. At ˜t = 4.16 s ˜
Note that the negative sign indicates a v = [−9.8(4.16) + 6] j
vector moving downwards, in the same ˜ = −34.81 j ˜
direction as the negative acceleration. ˜

10 State the solution. Therefore, the stone reaches the ground


with a velocity of 34.81 m/s downwards 4.16 s
after launching.

WORKED Example 9
A stone (A) is thrown upwards from a cliff with a velocity of 30 m/s. After stone A has
been in motion for 4 s another stone (B) is dropped from the same point. Find when and
where the two stones will meet.
THINK WRITE/DRAW
1 Draw a diagram and state the origin 30 m/s
position and direction of positive motion.
A r = (lt2 + mt + n) j
˜
Write the 3 equations of motion. O v = (2lt + m) j ˜
B ˜ ˜
a = 2l j
˜ ˜
The origin is at the top of the cliff and there is
positive motion up.
Chapter 11 Dynamics 543

THINK WRITE/DRAW
2 Stone A: Stone A:
Substitute the given information to At t = 0, r = 0 j and
˜ ˜ 2 + m(0) + n] j
find r . r = [l(0)
˜ ˜ ˜
So 0=0+0+n
therefore n = 0
r = (lt 2 + mt) j
˜ ˜
3 Use information about v to find m. At t = 0, v = 30 j and
˜ ˜ ˜ + m] j
v = [2l(0)
˜ ˜
So 30 = 0 + m
m = 30
4 Rewrite r . r = (lt 2 + 30t) j
˜ ˜ ˜
5 Use information about a to find l. At t = 0, a = −9.8 j and
˜
a = 2l j ˜
˜
So −9.8 = 2l ˜
l = −4.9
r = (−4.9t 2 + 30t) j
˜ ˜
6 For stone B: Repeat the method used Stone B:
for stone A. At t = 0, a = −9.8 and l = −4.9
For many of your problems l = –4.9. At t = 0, v = 0 j and
˜ ˜
v = [2l(0) + m] j
˜ ˜
therefore m = 0
At t = 0, r = 0 j and
˜ ˜ 2 + m(0) + n] j
r = [l(0)
˜
therefore n = 0 ˜
7 Rewrite the equation for r using all r = −4.9t 2 j
current information. ˜ ˜ ˜
8 Find where the two stones meet: The two stones meet where r A = r B
when r A = r B; but tB = tA – 4. ˜ ˜
(−4.9tA2 + 30tA) j = −4.9tB2 j
˜ all information
˜
Express in terms of A. −4.9tA2 + 30t˜A = −4.9(tA ˜– 4)2
= −4.9(tA2 − 8tA + 16)
0 = 39.2tA − 30tA – 78.4
= 9.2tA − 78.4
tA = 8.52 s
9 Find the displacement of B at rB = −4.9 (4.52)2
tA = 8.52 s and tB = tA – 4 = 4.52 s. = −100.1 m
The information for stone B is easier to The stones meet about 100 m below the top of
use to find the displacement because A the cliff 8.52 seconds after stone A is thrown.
has had upwards and downwards
motion (both unknown) whereas B has
had downwards motion only.
544 M a t h s Q u e s t M a t h s C Ye a r 1 1 f o r Q u e e n s l a n d

In general this procedure can be shortened. If an object starts at the origin, n = 0 and m
has the value of the initial velocity (upwards is positive, downwards is negative). If the
body is influenced by gravity then l = −4.9.
If an object is initially at rest then m = 0, but if an object is part of a system that is itself
moving, and the object is released from that moving system, then the initial velocity (m)
of the object will be the velocity of the whole system at the instant prior to release.

remember
remember
1. If the origin is the starting point, then n = 0.
2. If the body is initially at rest, then m = 0.
3. In vertical motion, displacement up is positive; displacement down is negative.
4. Velocity up is positive; velocity down is negative.
5. Acceleration up is positive; acceleration down is negative.

SLE 5: Model vertical motion under gravity alone.

Motion under constant


11C acceleration
1 A particle moving from rest with constant acceleration reaches a speed of 16 m/s in
4 seconds. Find:
a the acceleration 4 m/s2
b the distance travelled. 32 m
2 An object travelling at 8 m/s accelerates uniformly over a distance of 20 metres until
it reaches a speed of 18 m/s. Find:
a the acceleration 6.5 m/s2
b the time taken. Approx. 1.54 s
3 a A racing car accelerates
constantly from rest
and covers a distance
of 400 metres in
10 seconds. Find its
velocity at the end of
the 400 metres. 80 m/s
Approx. 8.94 s b Another car travels the
400 metres with a
constant acceleration of
10 m/s2. Find its time
for the 400 metres.
4 A train travelling at a
constant speed decelerates
uniformly for 30 seconds
over a distance of
270 metres, coming to
a stop. Find:
a the initial speed 18 m/s
b the acceleration.
−0.6 m/s2
Chapter 11 Dynamics 545
5 A parachutist free-falls from an aircraft for 6 seconds. If the acceleration due to
gravity is 9.8 m/s2, find:
a the speed of the parachutist after 6 seconds 58.8 m/s
b the distance travelled after 6 seconds. 176.4 m
6 A ball is thrown up from the ground with an initial velocity of 19.6 m/s. The acceleration
due to gravity is 9.8 m/s2 downwards, that is, −9.8 m/s2. Find:
a the maximum height attained by the ball 19.6 m
b the total time taken for the ball to return to the ground. 4 s
WORKED 7 A stone is dropped from a bridge which is 39.2 metres above a river.
Example eBook plus
8 a How long does it take the stone to reach the water? 2.83 s
b What is its speed on impact? 27.72 m/s Digital docs:
SkillSHEET 11.1
8 A ball is dropped from a tower and reaches the ground in 4 seconds. Find: Vertical motion
a the height of the tower 78.4 m
b the velocity of the ball when it hits the ground. 39.2 m/s
9 A particle is projected vertically up from the top of a building that is 50 metres above
the ground. If the initial speed of the particle is 28 m/s, find:
a the maximum height, above the ground, that it reaches 90 m
b total time taken to reach the ground 7.14 s
c the speed of the particle when it reaches the ground. 42 m/s
10 A train travels a distance of 1800 metres in 90 seconds while accelerating uniformly
from rest. What is its velocity at the end of 500 metres? 21.08 m/s or 20 10-
---------------
3

11 A car accelerates uniformly from rest, increasing its speed from 5 m/s to 25 m/s in
10 seconds. Find:
a the acceleration 2 m/s2
b the distance travelled, from rest, in 12 seconds 144 m
c the time taken to increase its speed from 15 m/s to 30 m/s. 7.5 s
12 A sprinter accelerates uniformly to his top speed after running 30 metres of a 100-metre
race. He maintains this speed for the remainder of the race and takes 10.4 seconds to
complete it. Find:
a the top speed of the athlete 12.5 m/s
b the time taken to reach the top speed. 4.8 s
13 A bus is travelling at 16 m/s when the brakes are applied, reducing the speed to 6 m/
s in 2 seconds. Assuming the retardation is constant, find:
a the acceleration −5 m/s2
b the distance travelled 2 seconds after the brakes are applied 22 m
c how long after applying the brakes the tram comes to a stop 3.2 s
d the braking distance of the bus. 25.6 m
14 A car moving from rest with uniform acceleration takes 12 seconds to travel 144 metres.
What is its speed after 6 seconds? 12 m/s
15 A bus travels 60 metres in 10 seconds and the next 60 metres in 15 seconds. If the
acceleration is constant, find:
a how much further it will travel before coming to rest 24.5 m
b how many more seconds it takes before coming to rest. 17.5 s
546 M a t h s Q u e s t M a t h s C Ye a r 1 1 f o r Q u e e n s l a n d

16 A juggler throws balls vertically


into the air so that they rise to a
height of 4.4 metres above the
ground. He fails to catch one and it hits
the ground with a speed of 1.155 times
that of its initial speed. Find:
a the speed of projection of the ball 8.04 m/s
b the height from which the ball is thrown 1.1 m
c the total time the ball is in the air. 1.77 s
17 An object is projected vertically up from a
14 metre tower and reaches the ground
4 seconds later.
a What is the projection speed of the
object? 16.1 m/s
b What is the maximum height
above the ground that is
attained by the object? 27.23 m
18 a An object is dropped from
the top of a building 39.2 m high. Calculate:
i its velocity when it is halfway to the ground −19.6 m/s
ii its velocity on striking the ground −27.7 m/s
b i −20.2 m/s iii the time taken to each the ground. t = 8 s
ii −28.15 m/s b Repeat the above questions for the case when the object
iii t = 2.4 s
is thrown to the ground with a velocity of 4.9 m/s.
19 a A stone rolls off the top of a cliff and is found to take
4 seconds to reach the sea below. What is the height of
the cliff? 78.4 m
b What is the difference in time to reach the bottom
between part a and if the stone were launched vertically upwards from this cliff
with a velocity of 20 m/s? 2.53 s longer
20 A vertical slit 1.5 m long is positioned in a stone wall 9.8 m below the top of the wall.
A small object is dropped from the top of the wall so that it falls in line with the slit.
For what length of time is the falling object visible through the slit? 0.1044 s
WORKED 21 A body is dropped from the top of a 100 m high tower at the same time as a body is
Example
xample
9
launched vertically upwards from the bottom of the tower with a velocity of 25 m/s.
Find when and where the two bodies are at the same height above the ground. 4 s, 78.4 m
22 A body is rising with a velocity of 20 m/s and it releases a small particle when it is
50 m above the ground. How long will it take for the small particle to reach the ground? 5.83 s
23 A cage is descending into a well at a constant velocity of 2 m/s when an object falls
through the wire of the cage. If the object reaches the water at the bottom of the well
10 seconds before the cage find the height above the water level at which the object
fell out of the cage. 195 m
24 A fireworks rocket is fired vertically upwards with uniform acceleration of 19.6 m/s2.
eBook plus
After 2 seconds a small particle is released from the rocket. How long after release
Digital doc: will the particle fall to the ground? 8.9 s
WorkSHEET 11.2
Chapter 11 Dynamics 547

summary
Displacement, velocity and acceleration
• Displacement gives the position of a particle relative to some reference point
(usually the origin).
• Instantaneous velocity is the rate of change of displacement with respect to time
and is represented by the derivative of the displacement variable.
• Average velocity during a time interval is the change in velocity across that
interval.
• Speed is the magnitude of velocity just as distance is the magnitude of
displacement.
• Instantaneous acceleration is the rate of change of velocity with respect to time and
is represented by the derivative of the velocity variable.

Vector relationships
• If the position vector r (t) = x(t) i + y(t) j represents the displacement of P at time t,
˜ ˜ ˜ 2
dr . d r .. dv
then -----˜ (or r (t)) represents the instantaneous velocity and -------2˜- (or r (t) or -----˜- )
dt ˜ dt ˜ dt
represents the instantaneous acceleration at time t.
dv
• Finding the velocity given the acceleration: a = -----˜- → v = a dt.
˜ dt ˜ ˜ ∫
dr
• Finding the displacement given the velocity: v = -----˜ → r = v dt.
˜ dt ˜ ˜ ∫
Remember that the constant of integration is always a vector constant.
2 2
• For the magnitude of a vector x i + y j : xi + y j = x + y .
˜ ˜ ˜ ˜
• The dot product of two vectors a and b is given as a • b = a b cos θ where θ is
˜ ˜ ˜ ˜ ˜ ˜
the angle included by a and b .
˜ ˜
Projectile motion
• The period of time for which the projectile is moving under the influence of gravity
alone is called the time of flight of the projectile.
• The range is the horizontal displacement of the projectile from its launch point.

Motion under constant acceleration


• r = (lt2 + mt + n) j , v = (2lt + m) j , a = 2l j
˜ ˜ ˜ ˜ ˜ ˜
• If the origin is the starting point, then n = 0.
• If the body is initially at rest, then m = 0.
• In vertical motion, displacement up is positive; displacement down is negative.
• Velocity up is positive; velocity down is negative.
• Acceleration up is positive; acceleration down is negative.
548 M a t h s Q u e s t M a t h s C Ye a r 1 1 f o r Q u e e n s l a n d

CHAPTER
review
Questions 1 to 5 refer to a particle that has a displacement r (t) at time t seconds, where
r (t) = 4t i + (8t − t2) j metres. ˜
˜ ˜ ˜
1 multiple choice
11A The initial speed of the particle in m/s is:
A 4 B 4 5 C 0 D 2 13 E 2 10
2 multiple choice
11A The vector which represents the average displacement (in m) in the first two seconds is:
A 4i + 6 j B 2 i + 3.5 j C 4i + 7 j D 8 i + 12 j E 4i + 4 j
˜ ˜ ˜ ˜ ˜ ˜ ˜ ˜ ˜ ˜
3 multiple choice
11A The particle reaches its greatest height after how many seconds?
A 0 B 2 C 3 D 4 E 5
4 multiple choice
11A The acceleration of the particle in m/s2 is given by:
A 2i B −2 j C 2i − 2 j D 0i + 0 j E 2j
˜ ˜ ˜ ˜ ˜ ˜ ˜
5 multiple choice
11A The Cartesian equation of the trajectory of the particle is:
x x
A y = 4x B y = 8x – x2 C y = ------ (32 – x) D y = --- (2 – x) E y = x2 – 16x

a (t) = −6 j m/s2
v (t) = 6 i + (12 − 6t) j m/s,
˜
˜
16 8
6 Relative to the origin, O, the displacement of a particle at time t seconds is given by
11A

˜
r (t) = 6t i + 3t(4 − t) j metres.

˜
a˜ Find expressions
˜ ˜ the velocity and acceleration vectors at time t seconds.
for
b Determine the initial displacement and speed of the particle. Origin, 6 5 m/s
c When t = 2 seconds, calculate:

˜
i the distance of the particle from the origin 12 2 m
ii the velocity (both magnitude and direction) of the particle 6 m/s to the right
iii the measure of the angle between the velocity and displacement vectors. 45°
7 At time t, a particle A has a displacement given by r A(t) = t i + (4t − t2) j , while a second
11A particle B has a displacement given by r B(t) = t i + ˜(t − 10) ˜j . ˜
˜
a If these particles collide, determine when they˜ collide. 5˜ seconds
b What are the coordinates of the impact point? (5, −5)
c Find the Cartesian equations of the trajectories of the particles. yA = 4x − x , yB = x − 10
2

d Use a graphics calculator to verify the coordinates of the impact point.


8 At time t seconds, a particle has a position vector given by the expression
11A r (t) = (t + 1) i + (25 − t2) j metres.
a˜ Use a graphics˜ ˜ to plot the trajectory of this particle across the interval
calculator
0 ≤ t ≤ 5 seconds.
b Repeat part a using an Excel spreadsheet.
c Determine the equation of this trajectory in the form y = f (x). y = 25 − (x − 1)2
Chapter 11 Dynamics 549
Questions 9 to 11 refer to a projectile that has velocity v (t) at time t seconds, where
v (t) = 6 i + (10 − 2t) j m/s. The initial displacement of ˜this projectile is given by
r˜ (0) = 4˜ j m. ˜
˜ ˜
9 multiple choice
The angle of projection of this particle, correct to the nearest degree, is:
11B
A 39° B 49° C 51° D 59° E 61°
10 multiple choice
The range (in m) of the projectile through the foot of the launch platform is nearest in value to:
11B
A 45 B 50 C 60 D 70 E 75
r (t) = 1--2- t2 i + ---t6- (9 −˜ 2t) j m

11 multiple choice
11B
v (t) = t i + (3t − t2) j m/s,

The greatest height (in m) reached by the particle is closest in value to:
A 29 B 32 C 35 D 39 E 95
2

12 A particle moves so that its acceleration at time t is given by a = i + (3 − 2t) j . Find vector
˜ ˜ that v = 0˜ and r = 0 11B
˜

expressions for the velocity and displacement of the particle given


˜

when t = 0. ˜ ˜
˜

13 A projectile is fired from the foot of a cliff and its displacement (in metres) at time t seconds
˜

is given by r (t) = 6t i + (45t − 1--- gt2) j . 11B


˜ 2 in your 2
˜ calculations. ˜ v (t) = 6 i + (45 − gt) j m/s
Use g = 10 m/s ˜ ˜ ˜
b 3 229 m/s a Determine an expression for the velocity vector of the projectile at time t seconds.
at an angle b Calculate the launch speed and angle of projection of the projectile.
of elevation c Find the greatest height reached and the time of flight of the projectile. 101 1--- m, 9 s
4
of 82°24′
d Calculate the impact velocity of the projectile when it returns to the ground in a
horizontal plane through the origin. 3 229 m/s downwards at an angle of 7˚36′ to the vertical
14 A particle is fired from the top of a 60 m high building with a velocity of 24 m/s inclined at
an angle of 30° to the horizontal. 11B
a Show that the velocity vector of the projectile at time t seconds is given by
v (t) = 12 3 i + (12 − gt) j m/s.
˜
b Develop ˜ expression
a vector ˜ for the displacement, r (t) metres, of the particle at time
t seconds. r (t) = 12t 3 i + (60 + 12t − 1--2- gt2) j ˜
˜ ˜ ˜
c Determine if this projectile will clear a 14 m high tower located 94 m from the launch
point and in the same horizontal plane. Clears by about 5 cm
15 A missile is fired from a point on level ground with a velocity V m/s and with an angle of
elevation of θ to the horizontal. The target is positioned on top of a cliff 30 m above the 11B
ground in the same horizontal plane as the point of projection of the missile, and with a
range of 60 m from it.
a If g is the gravitational acceleration of the missile, show that the motion of this missile
satisfies V 2 cos θ (2 sin θ – cos θ) = 60g.
b Use a graphics calculator to find those values of θ in the domain 0 < θ < 90° for which
the expression cos θ (2 sin θ – cos θ) is positive. State the greatest possible value this
expression has, and the value of θ that produces it. Provide supporting argument for your
solutions. Expression is positive for 26°34′ ≤ θ < 90°. Maximum is 0.618 when θ ≈ 58°17′.
c Extend from your answer to part b above to find the least value of V for which the
missile can reach the target. (Use g = 9.8 m/s2.) 30.845 m/s
d Generate a spreadsheet to validate the solution obtained using a graphics calculator.
550 M a t h s Q u e s t M a t h s C Ye a r 1 1 f o r Q u e e n s l a n d

16 A stone is projected vertically up from the ground with an initial velocity of 24.5 m/s.
11C Taking the acceleration due to gravity to be −9.8 m/s2, find:
a the maximum height reached by the stone 30.625 m
b the times at which its height is 20 metres above the ground. t ≈ 1 s and 4 s

17 multiple choice
11C An object is dropped from the top of a 200 metre high building. If the acceleration due to
gravity is 9.8 m/s2, what will be the height of the object after 5 seconds?
A 151 m B 49 m C 122.5 m D 20 m E 77.5 m

18 multiple choice
11C A particle initially moving at 6 m/s is subject to a constant retardation of 2 m/s2.
The distance, in metres, travelled before coming to rest is:
A 27 B 8 C 9 D 10 E 12

19 multiple choice
11C A bus travels 500 metres in 25 seconds when accelerating uniformly from rest. The
acceleration, in m/s2, is:
A 0.4 B 1.6 C 1.25 D −1.2 E 0.625

20 A parachutist drops from an aeroplane so that


11C the constant acceleration during free fall due
to gravity and air resistance is 8 m/s2. The
parachute is released after 6 seconds,
uniformly retarding the parachutist in
28 seconds to a constant speed of 2.5 m/s.
This speed is maintained until the parachutist
reaches the ground which is 1101 metres
below the point of release.
a How long is the parachutist in the air?
b After how long has the parachutist fallen
2 min 14 s half the distance (answer to the nearest
tenth of a second)? 16.2 s

21 Jogger A is running in a straight line at a


11C constant speed of 4 m/s when passing jogger
B who has stopped to tie a lace. Jogger B
heads off in the same direction as jogger A
6 seconds later, accelerating uniformly at
2 m/s2 until reaching a speed of 5 m/s.
21 a
a Sketch a velocity–time graph showing the
motion of both joggers.
v (m/s)
5 B
b How long is it after jogger A first passes
4 A jogger B until B catches up to A? 36.25 s
c How far has jogger B travelled to catch
0 _6 8.5 t (s) jogger A? 145 m
d How far ahead will jogger B be after
jogger B has travelled 225 metres? 16 m
Chapter 11 Dynamics 551
Modelling and problem solving
1 Car A is 600 metres from the centre of the intersection when it starts from rest and accelerates
uniformly at 4 m/s2, reaching a speed of 24 m/s which it maintains. At the instant car A takes
off, car B is 780 metres from the centre of the intersection and travelling at a constant speed
of 28 m/s. When car B is 52 metres from the centre of the intersection it decelerates uniformly
at 5 m/s2.
a Which car gets to the centre of the intersection first? Car A
b How far past the centre of the intersection is the first car
when the second car reaches it? 8.4 m Car B
c If all other conditions remain the same, what constant
acceleration would:

780 m
ii the second car need to have for a collision to occur? −2 m/s2
ii the first car need to have for a collision to occur? 3.58 m/s2 Car A
d If all other conditions remain the same, at what constant
speed would:
Centre of
ii the first car need to travel for a collision to occur? intersection
(Use a graphics calculator or a numerical method to 600 m
assist.) 23.62 m/s
ii the second car need to travel for a collision to occur? 28.33 m/s
(Give answers correct to 2 decimal places where appropriate.)

2 Martin is trying out his new spaceship by challenging a local alien to a race. He places his
hand on the throttle and his spaceship starts to accelerate at a constant rate a for a time ∆t.
During that time ∆t, he travels a distance s through space. Martin’s spaceship has an initial
speed u at the beginning of the time interval and a final speed v after a time ∆t.
The local space police are on to Martin and want to photograph him just as he breaks the
speed limit. However, they need to set their camera up at the precise point, P, where Martin is
travelling at his average speed. The police know when the average speed will occur; it will
occur at a time T = ∆t ----- . They want to find out where the average speed will occur. Let x be
2
the distance travelled when the spaceship reaches its average velocity for the time interval ∆t.
All distances and times are in metres (m) and seconds (s). It is clear that x is a fraction of s,
the total distance covered during the time interval ∆t.

Positive

Average speed
occurs here

P
x

a The initial velocity (u) is 0, the acceleration (a) is 1.0 and the time interval (∆t) is 10. Find
x
the value of x and hence state the value of the fraction -- . --xs = 1--4-
s
b Find equations for x and s in terms of the acceleration, a; the initial speed, u; and the time
interval ∆t. x = --------
u∆t-
2
+ 1--8- a(∆ t)2 s = 1--2- a(∆ t)2 + u∆ t
s
c Under what conditions will x = --- ? When a = 0
2
552 M a t h s Q u e s t M a t h s C Ye a r 1 1 f o r Q u e e n s l a n d

3 When a projectile is fired from a body out into space, it decelerates due to the gravitational
k
pull of the body. The general equation for this deceleration is a = − ----2 where r is the distance
from the centre of the body and k is a positive constant. r
a On the surface of the moon (r = 1760 km), the deceleration is a = 1.6 m/s2. Find the value
of k. k = 4.96 × 1012
b If a body is launched from the surface of the moon with a velocity of 500 m/s, find the
velocity of the body at a distance r from the centre of the moon; that is, find v(r).
c Find the distance from the centre of the moon when the body is momentarily stationary.
1.84 × 106 m

12 6
4.96 × 10 -
b v(r) = 2  ---------------------------
r
– 2.69 × 10 
Appendix 553

Instructions for the TI-89 Titanium graphics calculator


Chapter 1 — Number systems: the Real Number System
Graphics Calculator tip: Square, cube and nth roots (page 7) ................................... 555
Graphics Calculator tip: Approximation mode ........................................................... 556

Chapter 2 — Number systems: complex numbers


Graphics Calculator tip: Simple algebra of complex numbers (page 93) .................. 557
Graphics Calculator tip: Modulus and Argument (page 101) .................................... 559
Graphics Calculator tip: Expressing complex numbers in polar form (page 105) .... 561
Graphics Calculator tip: Expressing complex numbers in Cartesian form
(page 106) ............................................................................................................... 561
Graphics Calculator tip: Pascal’s Triangle coefficients (page 115) ............................ 562

Chapter 3 — Matrices
Graphics Calculator tip: Solving matrix equations (page 156) .................................. 563
Investigation: Matrix multiplication using a graphics calculator (page 162) ............ 564
Graphics Calculator tip: Alternative method for adding the elements in each row
in a dominance matrix (page 167) ......................................................................... 565

Chapter 5 — Matrices and their applications


Investigation: Performing Gaussian elimination using a graphics calculator
(page 217) ............................................................................................................... 566
Graphics Calculator tip: Matrix operations (page 237) .............................................. 568

Chapter 6 — Transformations using matrices


Graphics Calculator tip: Graphing the original and its image (page 256) ................. 571

Chapter 7 — Introduction to vectors


Graphics Calculator tip: Finding the magnitude and direction of a vector in two
dimensions (page 311) ........................................................................................... 572
Graphics Calculator tip: Finding the x- and y-components of a vector
(page 313) ............................................................................................................... 573
Graphics Calculator tip: Finding the unit vector in the direction of the vector
(page 316) ............................................................................................................... 574
Graphics Calculator tip: Finding the dot product of two vectors (page 326) ............ 574
Graphics Calculator tip: Finding scalar and vector resolutes (page 334) .................. 575
Graphics Calculator tip: Vector functions of time (page 341) ................................... 576
554 Appendix

Chapter 9 — Sequences and series


Graphics Calculator tip: Listing the terms of an arithmetic sequence (page 400) .... 577
Graphics Calculator tip: Finding the sum of an arithmetic sequence (page 402) ..... 578
Worked example 10 (page 410) .................................................................................. 579
Worked example 12 (page 418) .................................................................................. 580
Worked example 15 (page 422) .................................................................................. 581
Graphics Calculator tip: Comparison of simple and compound interest
(page 434) ............................................................................................................... 582
Graphics calculator tip: Generating terms in the Fibonacci Sequence
(page 443) ............................................................................................................... 584

Chapter 10 — Permutations and combinations


Graphics Calculator tip: Calculating factorials (page 468) ........................................ 585
Graphics Calculator tip: Calculating permutations (page 472) .................................. 585
Graphics Calculator tip: Calculating combinations (page 486) ................................. 586

Chapter 11 — Dynamics
Worked example 2 (page 517) .................................................................................... 587
Worked example 3 (page 519) .................................................................................... 588
Worked example 7 (page 531) .................................................................................... 589
Appendix 555
Chapter 1 page 7
Graphics Calculator tip! Square, cube and nth roots

A graphics calculator can be used to find the square root, cube root or higher root of a
number.
1. From the MENU (press APPS or 2ND [QUIT]),
highlight Home.

2. Press ENTER . To calculate the square root of a


number (for example, 8 ), press 2ND [ ]
followed by the number (8 in this case), close the
brackets and press ENTER . Your calculator may give
this answer as a simplified surd, in this case, 2 2 . If
so, you will need to change the set up of your
calculator to approximation mode. (See the graphics
calculator tip that follows.)
3. To calculate higher order roots you will need to use a
fractional index. That is, for the cube root use 1--- ,
3
for the fourth root use 1--- and so on. To calculate
4
3 8 , enter 8 then press Ÿ and enter (1 ÷ 3) as the

index.
556 Appendix

Graphics Calculator tip! Approximation mode


The following steps show how to set your calculator in approximation mode.
1. From the MENU, highlight Home. (Press APPS to
display the MENU.)

2. Press ENTER and then press MODE to display your


set up options. Press F2 (Page 2) to display the
second page of options.

3. Press the down arrow key until the entry next to


Exact/Approx is highlighted and then press the right
arrow to display your options.

4. Select 3: APPROXIMATE for approximation mode


(press 3 or highlight the required mode and press
ENTER ). Your selection will be flashing.

5. Press ENTER to save this setting. Notice that the


bottom of the display shows APPROX.
Appendix 557
Chapter 2 page 93
Graphics Calculator tip! Simple algebra of
complex numbers
Operations with complex numbers, finding the real and imaginary parts of a complex
number and finding the complex conjugate can be achieved with a graphics calculator.
You may not need to use a graphics calculator with simple complex numbers but it can
be useful in more complicated questions.

Operations with complex numbers


1. From the MENU, highlight Home. (Press APPS to
display the MENU.)

2. Press ENTER . To perform simple algebra on


complex numbers, use the standard keys to enter the
expression. To enter i, press 2ND [i].
For example:
(a) input (2 − 2i)(1 + 3i) and then press ENTER .
(b) input (2 − 2i) ÷ (1 + 3i) and then press ENTER .
(c) input (2 − 2i)^3 and then press ENTER .
Notice that a multiplication symbol (a dot) appears
on the screen for any multiplication operations in
these examples. The calculator has assumed that 2i
means 2 × i.
3. A complex number can be stored and then retrieved
if a number of operations need to be performed on it.
Input 2 − 2i and then press STO followed by Z

to store this expression as the variable z. Press


ENTER .
Input 1 + 3i and then press STO followed by

ALPHA [W] to store this expression as the variable


w. Press ENTER .
We can now calculate expressions involving z and/or w,
for example, z × w, z ÷ w and z3. Press ENTER to
obtain each answer.
558 Appendix

Features of a complex number


1. To find the complex conjugate, the real part or the
imaginary part of a complex number or expression,
press CATALOG . You may then access conj( for the
conjugate, real( for the real part and imag( for the
imaginary part of a complex number as required. The
screen at right shows conj( highlighted.
2. Press ENTER to display the selection in the entry
line of the Home screen. Enter the complex number,
close the brackets and press ENTER .

3. Alternatively, you can enter an expression, for


example, (2 − 2i)(1 + 3i), and then press STO


followed by ALPHA [A] to store the output as the
variable a. Press ENTER .
(a) To find ( 2 – 2i ) ( 1 + 3i ) , press CATALOG and
select conj(. (To select conj(, highlight conj( and
then press ENTER .) Key in the variable assigned
to the stored data (a in this case, so press ALPHA
[A]) and close the brackets. Press ENTER .
(b) To find Re ((2 − 2i)(1 + 3i)), press CATALOG
and select real(. Key in the variable assigned to
the stored data and close the brackets. Press
ENTER .
(c) To find Im ((2 − 2i)(1 + 3i)), press CATALOG
and select imag(. Key in the variable assigned to
the stored data and close the brackets. Press
ENTER .
Appendix 559
Chapter 2 page 101
Graphics Calculator tip! Modulus and Argument

Your graphics calculator is also able to calculate the modulus and Argument of a com-
plex number. Consider 3 + i and −1 − 2 i from Worked example 23.

Modulus (magnitude, or absolute value) of a complex number


For this activity your calculator will need to be changed into AUTO or EXACT mode
and must not be in APPROX mode.
1. From the Home screen, press CATALOG and access
abs(.

2. (a) Press ENTER to display abs( in the entry line of


the Home screen, then enter 3 + i and close the
brackets. Press ENTER to obtain the answer.
(b) The modulus of −1 − 2 i is also shown in the
screen at right.
(c) The modulus of a stored complex number can
also be calculated. For example, using the
expression from the previous graphics
calculator tip where we assigned the variable a to
(2 − 2i)(1 + 3i), we can obtain the magnitude as
shown in the screen at right.

Argument of a complex number


1. Decide whether you want the angle shown in radians
or degrees. See the instructions below for changing
the system settings for Angle.
2. Press CATALOG and access angle(.

3. Press ENTER to display angle( in the entry line of


the Home screen, then enter the required complex
number 3 + i and close the brackets. Press ENTER
to obtain the answer. If the calculator is set to
radians, the answer will be shown as π--- . If the
6
calculator is set to degrees, the answer will be shown
as 30 for 30°.
4. (a) The Argument of −1 − 2 i is shown as an exact
answer in radians in the screen at right.
(b) For an approximate answer, switch the calculator
back to approximation mode as shown earlier on
page 556.
560 Appendix

(c) To convert this angle in radians to degrees,


minutes and seconds, first press 2ND [ANS].
Then press CATALOG and select DMS. Now


that you are back in the Home screen, press
ENTER to obtain the answer.
Note: You can convert degrees to radians by selecting
Rad in the CATALOG.

5. (a) The Argument of a stored complex number can


also be calculated. See the screen at right for the
Argument of the variable a where a is assigned to
(2 − 2i)(1 + 3i). The exact answer is shown.
(b) For the approximate answer, set the calculator
back to approximation mode. The calculator is set
to degrees for this example.

Changing the system settings for Angle


The following steps show how to set your calculator in degrees mode.
1. From the MENU, highlight Home. (Press APPS to
display the MENU.)

2. Press ENTER and then press MODE to display your


set up options.

3. Press the down arrow until the entry next to Angle is


highlighted and then press the right arrow to display
your angle options.

4. Select 2: DEGREE for degree mode (press 2 or


highlight the required mode and press ENTER ). Your
selection will be flashing.

5. Press ENTER to save this setting. Notice that the


bottom of the screen displays DEG.

The same method is used to change back to radians.


Appendix 561
Chapter 2 page 105
Graphics Calculator tip! Expressing complex numbers
in polar form
Complex numbers in Cartesian form (also known as rectangular form) can be written in
polar form if we know the modulus and the Argument. Consider 1 + i and 1 − 3 i from
Worked example 24.
To begin this activity, have your calculator set in DEGREES and EXACT form.
1. From the MENU, select Home. Enter the complex
number in Cartesian form. Press CATALOG and
select Polar. Press ENTER to show the modulus

and Argument.
For 1 + i, we can see that the modulus is 2 and the
Argument is 45° (as the calculator is set to degrees).
For 1 − 3 i, the modulus is 2 and the Argument is
−60°.
2. If the calculator is set to radians, the answer is
shown in a different format but we can still see the
modulus and Argument. Another way of iπ
-----
π 4
expressing 2 cis --- (the polar form of 1 + i) is 2 e .
4
(This is beyond the scope of this course.)

Chapter 2 page 106


Graphics Calculator tip! Expressing complex numbers
in Cartesian form
Complex numbers in polar form can be written in Cartesian form by entering both the
modulus and the Argument into the calculator. Consider 3 cis --π- from Worked example
4
25. The modulus is 3 and the Argument is --π- or 45°.
4
1. From the MENU, select Home. To enter the complex
number 3 cis π--- , first open a set of brackets and press
4
3 to enter the modulus of 3. Then press 2ND [– ]
to enter the ∠ function. Complete the entry line by
entering the angle in radians ( π--- in this case) or
4
degrees (45°) as appropriate to the calculator setting.
The screen at right shows the angle in radians. Close
the set of brackets and press ENTER .
2. If the calculator is set to degrees with the Argument
entered as 45, the same result is obtained.
562 Appendix

Chapter 2 page 115


Graphics Calculator tip! Pascal’s Triangle coefficients

The coefficients of each term of the expansion of (a + bi)n can be found using your
graphics calculator. For example, the coefficients of the expansion of (a + bi)5 can also
be written as:
5
C0 a5 + 5C1 a4(bi)1 + 5C2 a3(bi)2 . . . 5C5(bi)5
where 5C0, 5C1, . . ., 5C5 represent the coefficients.
The following steps show how to calculate 5C3 using a graphics calculator.
1. From the Home screen, press CATALOG and access
nCr(. You can find nCr( more quickly by first
pressing ALPHA [N].

2. Press ENTER and then complete the entry line to


obtain nCr(5, 3). Press ENTER to display the value.

Note that with this calculator, we can obtain the actual


expansion of (a + bi)5.

1. From the MENU, select Home. Press 2ND [F6] and


press 1 to select 1: Clear a-z.... This sets the
variables a-z to their default values and makes them
ready for use.

2. Press ENTER to indicate YES. Enter (a + bi)^5


and press ENTER . You will see a small arrow at the
end of the answer line indicating that there are more
terms. Use the arrow keys to see more of the
expansion. The full expansion is shown as
a(a4 − 10a2b2 + 5b4) + (5a4 − 10a2b2 + b4)bi.
Appendix 563
Chapter 3 page 156
Graphics Calculator tip! Solving matrix equations

Most graphics calculators provide a facility for calcu-


lating inverses of matrices. To solve the equations in
Worked example 12, follow these steps.
1. From the MENU, highlight Data/Matrix Editor.

2. Press ENTER and select 3: New... to open a new


matrix.

3. Ensure that Type is shown as Matrix (to change,


press the right arrow to display the options and select
2: Matrix), then enter a for the variable (press
ALPHA [A]), 2 for the Row dimension and 2 for
Col dimension to define the matrix as a 2 × 2 matrix
with variable name a.

4. Press ENTER to accept the settings. Fill in the 2 × 2


matrix. Press ENTER after each number to move the
cursor from cell to cell.

5. Press F1 (Tools) and select 3: New... to define a


second matrix.

6. Enter the settings shown at right to define a 2 × 1


matrix with variable name b.

7. Enter the values 16 and 5 for matrix b as shown.

8. Press APPS to return to the MENU and select


Home. Enter the calculation a-1 ¥ b as shown to
obtain the result.
564 Appendix

Chapter 3 page 162


Matrix multiplication using a
graphics calculator
Worked example 13 may be solved using a graphics
calculator as follows.

1. From the MENU, highlight Data/Matrix Editor.

2. Press ENTER and select 3: New... to open a


new matrix.

3. Enter the settings shown at right to define the


matrix as a 3 × 3 matrix with variable name a.

4. Press ENTER and complete the matrix.

5. Press F1 (Tools) and select 3: New... to define


a second matrix.

6. Enter the settings shown at right to define a 3 × 1


matrix with variable name b.

7. Enter the values for matrix b as shown.

8. Press APPS to return to the MENU and select


Home. Enter the calculation a ¥ b as shown to
obtain the result.
Appendix 565
Chapter 3 page 167
Alternative method for adding the
Graphics Calculator tip! elements in each row in a
dominance matrix
Multiplying a square matrix by a column vector with the same number of rows and all
entries shown as 1 has the effect of adding the elements in each row of the matrix. In
the example on page 165, the dominance vector V1 could have been obtained using the
following steps. (The main advantage is if the dominance matrix is 5 × 5 or larger. You
don’t need to arrow across the screen to see the elements when you are adding them.)
1. From the MENU, select Data/Matrix Editor and
establish a 4 × 4 dominance matrix with a variable
name m. (Refer to the graphics calculator tip on page
563 if you are unsure how to do this.) Note that you
will need to use the arrow keys to display the fourth
column of the matrix.

2. Press F1 (Tools) and select 3: New... to create a


new 4 × 1 vector with variable name n. Enter 1 as
shown for every element.

3. Press APPS to return to the MENU and select


Home. Perform the multiplication m ¥ n as shown.
566 Appendix

Chapter 5 page 217


Performing Gaussian elimination
using a graphics calculator
Matrix row operations can be performed on your graphics calculator. To demonstrate
this, we will repeat the steps of Worked example 4 to find the inverse of

121
A = 101 .
013
1. Enter Matrix A.
(a) From the MENU, select Data/Matrix Editor.
Select 3: New... and enter the settings shown
to create a 3 × 3 matrix with variable a (to
represent matrix A).

(b) Press ENTER and then enter the values for


matrix A as shown.

2. Set up an augmented matrix B = [A|I]. Enter the


augmented matrix [A|I] first and then save as
matrix B.
(a) Press APPS to return to the MENU and
select Home.
(b) Press CATALOG and select augment(. Press
ALPHA [A] for matrix A and then the
comma key , . To enter the identity matrix,
press CATALOG and select identity(. Press
3 (for a 3 × 3 identity matrix) and press
) twice to close the two sets of brackets.
Press STO and then press ALPHA [B] to

save the augmented matrix as matrix B. Press


ENTER to display the matrix.

3. Replace R2 with R2 − R1 (or − R1 + R2).


(a) To perform the required row operation, press
CATALOG and access mRowAdd(.
Appendix 567
(b) Press ENTER to display mRowAdd( in the
entry line of the Home screen. Input the
scalar multiplier, the matrix name, the row
number to be multiplied and then the row
number for the result to be added to, each
separated by a comma. In this case, enter −1,
B, 1, 2. Press ) to close the set of brackets.
Then press STO and ALPHA [C] to save


the matrix as matrix C. Press ENTER to
display the matrix.
Compare this screen with the matrix obtained in
Step 2 of Worked example 4.
4. Swap R2 and R3.
(a) To perform the required row operation, press
CATALOG and access rowSwap(.

(b) Press ENTER to display rowSwap( in the


entry line of the Home screen. Input the
matrix name and the two row numbers to be
swapped, each separated by a comma. In this
case, enter C, 2, 3. Press ) to close the set
of brackets. Then press STO and ALPHA

[D] to save the matrix as matrix D. Press


ENTER to display the matrix.
Compare this screen with the matrix obtained in
Step 3 of Worked example 4.
5. Replace R3 with R3 + 2R2.
(a) As before, press CATALOG and select
mRowAdd(.
(b) Enter 2, D, 2, 3 to represent 2 × R2 + R3 in
matrix D. Press ) to close the set of
brackets. Then press STO and ALPHA [E]

to save the matrix as matrix E. Press ENTER


to display the matrix.
Compare this screen with the matrix obtained in
Step 4 of Worked example 4.
6. Multiply R3 by 1--- .
6
(a) To perform the required row operation, press
CATALOG and access mRow(.
568 Appendix

(b) Press ENTER to display mRow( in the entry


line of the Home screen. Enter 1--- , E, 3 to
6
represent 1--- × R3 in matrix E. Press ) to
6
close the set of brackets. Then press STO


and ALPHA [F] to save the matrix as matrix
F. Press ENTER to display the matrix.
Compare this screen with the matrix obtained in
Step 5 of Worked example 4.
7. Now that we have row-echelon form, continue
performing operations until reduced row-echelon
form is achieved.

Chapter 5 page 237


Graphics Calculator tip! Matrix operations

The graphics calculator can perform a number of matrix operations and can provide
quick and reliable answers to some of the problems that you have encountered in this
chapter. A number of operations are shown below, some of which have already been
covered in earlier graphics calculator tips.
2 3 2
Consider the matrix A = 0 1 4 .
2 1 –1
Entering a matrix
(a) From the MENU, select Data/Matrix Editor.
Select 3: New... and enter the settings shown to
create a 3 × 3 matrix with variable a (to represent
matrix A).

(b) Press ENTER and then enter the values for matrix A
as shown.
Appendix 569
Inverse of A
Press APPS to return to the MENU and select Home.
With matrix A entered in the calculator, press ALPHA
[A] then the power key, Ÿ , and type in the index of −1
to specify the matrix A−1. Press ENTER to display the
required matrix.

Powers of A
(a) From the Home screen, and with matrix A entered in
the calculator, press ALPHA [A] to recall matrix A
to the entry line. To find A2, press the power key,
Ÿ , and type in the required index of 2. Press
ENTER to display the required matrix.

(b) Similarly, press ALPHA [A] followed by Ÿ and


then 3 to specify A3. Press ENTER to display the
required matrix.

Determinant of A
(a) From the Home screen, press CATALOG and access
det(.

(b) Press ENTER to display det( in the entry line and


then press ALPHA [A] to specify matrix A. Close
the set of brackets and press ENTER to display the
answer.

The identity matrix


(a) From the Home screen, press CATALOG and access
identity(.

(b) Press ENTER to display identity( in the entry line


of the Home screen. For a 3 × 3 identity matrix,
press 3 . Close the set of brackets and press
ENTER to display the required matrix.
570 Appendix

Calculate (I – A)−1 and store it as B


(a) Press ( to open a set of brackets and specify the
3 × 3 matrix I as shown above. Press the subtraction
key (–) and then press ALPHA [A] to specify the
matrix A. Press ) to close the set of brackets and
then press the power key ( Ÿ ) and type in −1 as the
index to specify the inverse matrix.
(b) Press STO and then ALPHA [B] to store the


matrix as matrix B. Press ENTER to display the
resulting matrix B.

Alternatively, you can store the resulting matrix after


the calculation has been performed. Input (I – A)−1 and
press ENTER to display the resulting answer matrix. To
store this as matrix B, press STO and then ALPHA

[B]. Press ENTER .

12 12
Calculate the product A−1 3 4 by entering 3 4 as a list
56 56
From the MENU, select Data/Matrix Editor and create a
new matrix of dimension 3 × 2 and store it as matrix B.
Return to the Home screen and enter A−1 as previously
shown, then multiply by B.

Fill cells of a matrix with a given value


(a) From the MENU, select Data/Matrix Editor and set
up a new matrix A with specified dimensions. (Let’s
use 3 × 3.) Press APPS to return to the MENU and
select Home. Press CATALOG and access Fill.

(b) Press ENTER to display Fill in the entry line of the


Home screen. Enter the given value. (Let’s use 5.)
Press the comma key ( , ) and then press
ALPHA [A] to enter matrix A. Press ENTER .
You will see a message on the screen indicating
that this command has been done. To display
matrix A, press ALPHA [A] and then ENTER .
Appendix 571
Chapter 6 page 256
Graphics Calculator tip! Graphing the original and
its image
We can use a graphics calculator to draw the original
function and its image on the same axes. Consider the
function y = x2 and its image y′ = x′ 2 − 2x′ − 3 found in
Worked example 4.
1. From the MENU, highlight Graph.

2. Press ENTER . To draw the graph of y = x2, press


♦ [Y=] and enter x2 in the function entry line next
to y1(x).

3. Press ♦ [GRAPH] to view the graph of this curve.

4. For the second equation, press ♦ [Y=] and enter


x2 − 2x − 3 in the function entry line next to y2(x)
and press ♦ [GRAPH] to view the graphs.

5. To see the two curves more clearly, alter the scale


settings of the axes. Press ♦ [WINDOW] and enter
the settings shown.

6. Press ♦ [GRAPH] to redraw the graph with the new


scale.

7. The TI-89 Titanium does not show the function


names on the screen as other calculator models may
do. However, you can trace each function by pressing
F3 (Trace). The coordinates of the cursor are shown
and the number of the function (1 or 2) is displayed
in the top right of the screen.
572 Appendix

Chapter 7 page 311


Graphics Calculator tip! Finding the magnitude and direction
of a vector in two dimensions
Consider the vector shown in Worked example 6. The vector can be expressed in com-
ponent form (or rectangular form) as 3 i − 5 j . One way of finding the magnitude and
˜ the
direction (the angle the vector makes with ˜ positive x-axis) is to convert the vector
from rectangular form to polar form using a graphics calculator.
1. From the MENU, select Home. Decide whether you
want the angle displayed in degrees or radians. In
this example we want degrees. Press MODE and
select 2: DEGREE at the Angle line.

2. Press ENTER until you return to a blank Home


screen. We will first define 3 i − 5 j as the vector u
and then convert this to polar ˜form.˜ Press CATALOG
to access Define.

3. Press ENTER to display Define in the entry line and


then press ALPHA [U] to name the vector as u. Press
= and then open a set of square brackets (press
2ND [[].) Enter the components 3 and −5, separated
by a comma (press , ), then close the set of square
brackets (press 2ND []]) and press ENTER .
4. Press ALPHA [U] for vector u. Then press
CATALOG to access Polar.

5. Press ENTER to display Polar in the entry line


and then press ENTER again to display the exact


value of the vector in polar form.

6. To obtain the approximate values, change to


approximation mode (press MODE and select
3: APPROXIMATE at the Exact/Approx line). Press
ENTER to recalculate in APPROX mode.

Note that if no other operations are required, you can


enter the vector directly by using square brackets. You
then continue in the same way to convert to polar form.
Appendix 573
Chapter 7 page 313
Graphics Calculator tip! Finding the x- and y-components
of a vector
Vectors can be expressed in different forms. In the previous graphics calculator tip on
page 572, we converted a vector in rectangular form to polar form so we could obtain
the magnitude and direction of the vector. The reverse process can also be performed.
Consider the vector in Worked example 7 where the magnitude is 30 and the angle to
the positive x-axis is 140°. To find the x- and y-components, we convert the vector to
rectangular form.
Before beginning this activity, check that the calculator is set to degrees and is in AUTO
mode.
1. From the MENU, select Home. To enter the vector in
polar form, open a set of square brackets, enter 30
for the magnitude followed by a comma. Press 2ND
[– ] and enter 140 for the angle. Close the set of
square brackets and press ENTER to display the
vector in polar form.
2. Move the cursor to the right of the expression in the
entry line. Press CATALOG to access Rect and

press ENTER .

3. Press ENTER to perform the conversion. The exact


values for the x- and y-components are shown.

4. To obtain the approximate values, switch the


calculator to APPROX mode and press ENTER
to perform the calculation again.
Hence u = −22.98 i + 19.28 j .
˜ ˜ ˜
574 Appendix

Chapter 7 page 316


Graphics Calculator tip! Finding the unit vector in the
direction of the vector
As seen previously, the unit vector is obtained by dividing each component by the mag-
nitude of the vector. We can also use a graphics calculator to achieve this. Consider the
vector u in Worked example 9 which can be expressed in component form as 6 i + 3 j .
˜ ˜ ˜
To begin this exercise, set your calculator to EXACT
mode.
1. From the MENU, select Home. To define the vector
[6, 3] as u, first press CATALOG and select Define.
Press ALPHA [U] (to name the vector as u) followed by
= and then 2ND [[] to open a set of square brackets.
Enter the digits 6 and 3 separated by a comma. Close
the set of square brackets and press ENTER .
2. Press CATALOG to access unitV(.

3. Press ENTER to display unitV( in the entry line of


the Home screen and then press ALPHA [U] to
enter the required vector. Close the set of brackets
and press ENTER . The exact values for the
components of the unit vector are shown.
2 5 5
Hence u = ---------- i + ------- j .
˜ 5 ˜ 5˜
These steps also apply to finding the unit vector for a three-dimensional vector.

Chapter 7 page 326


Graphics Calculator tip! Finding the dot product of
two vectors
The following steps show how a graphics calculator can
be used to find the dot product of two vectors. Consider
the vectors u = 3 i + 4 j + 2 k and v = 6 i − 4 j + k in
˜
Worked example ˜13. ˜ ˜ ˜ ˜ ˜ ˜

1. From the MENU, select Home. Define the vector u


˜
and the vector v by entering each set of components
˜
within square brackets.
2. Press CATALOG and select dotP(. Press ALPHA [U]
then the comma key ( , ) followed by ALPHA [V].
Close the set of brackets and press ENTER to obtain
the value of the dot product.
Appendix 575
Chapter 7 page 334
Graphics Calculator tip! Finding scalar and vector resolutes

Finding the dot product and the unit vector on a graphics calculator can be used to
find a scalar resolute or a vector resolute. Consider the vectors u = −2 i + 3 j + k and
v = 3 i + 4 j − k in Worked example 18. ˜ ˜ ˜ ˜
˜ ˜ ˜ ˜
1. From the MENU, select Home. Define the vector u
and the vector v by entering each set of components˜
˜
within square brackets.

2. To find the scalar resolute of v on u , we need to


calculate the dot product of û˜ (the ˜unit vector of u )
and the vector v . Access the ˜dot product function ˜
(press CATALOG ˜ and select dotP( ). To enter û , first
access the unit vector function (press CATALOG ˜ and
select unitV( ) and then press ALPHA [U]. Press )
to close the set of inner brackets and then press the
comma key ( , ) followed by ALPHA [V]. Close the
set of outer brackets and press ENTER to display the
value of the scalar resolute.
3. To find the vector resolute of v parallel to u , we need
˜ [ANS] to˜ show the
to calculate ( û • v )û . Press 2ND
˜ ˜ ˜
previous answer then press the multiplication key.
Repeat the steps above to access the unit vector
function and press ALPHA [U]. Close the set of
brackets and press ENTER to display the
components of the required vector resolute.
4. The vector resolute can be found straight after
defining the vectors u and v (without finding the
˜ seen˜ in the screen at right.
scalar resolute first) as

5. To find the vector resolute of v perpendicular to u ,


˜
we need to subtract the vector˜resolute of v parallel
˜
to u (calculated in step 4) from v . Press ALPHA [V]
˜ – followed by 2ND [ANS]
then ˜ to show the
previous answer. Press ENTER to display the
components of the required vector resolute.
576 Appendix

Chapter 7 page 341


Graphics Calculator tip! Vector functions of time

To draw the graph of a time-varying vector, we need to express the components in


terms of parametric equations. Consider drawing the graph of the position vector
u = 2cos t i + 3sin t j from Worked example 22 using a graphics calculator.
˜ ˜ ˜
1. From the MENU, highlight Graph.

2. Press ENTER . First ensure that the angle setting is


shown as radians. Press MODE and select
1: RADIAN at the Angle line.

3. Before leaving this screen, select 2: PARAMETRIC at


the Graph line.

4. Press ENTER to save the settings. Press ♦ [Y=]


and enter the x-component by completing the entry
line for xt1(t) with 2cos t and then press ENTER .
Similarly, enter the y-component by completing the
entry line for yt1(t) with 3sin t and press ENTER .

5. Press ♦ [GRAPH] to display the graph.

To obtain a clearer view of the graph, you can adjust the window settings. Press
♦ [WINDOW]. Adjust the values for xmin, xmax, ymin and ymax.
Alternately, press F2 (Zoom) and select 2: ZoomIn. Use the arrow keys to define the
centre point of the zoom and press ENTER . This will increase the size of the graph.
Appendix 577
Chapter 9 page 400
Graphics Calculator tip! Listing the terms of an
arithmetic sequence
If you know the rule for an arithmetic sequence, successive terms can be listed using a
graphics calculator. The steps for generating these terms are shown below. Consider an
example where the first term is 3 and the difference is 2 so the rule is tn = 3 + (n − 1) 2.

1. From the MENU, select Data/Matrix Editor and then


3: New....

2. Adjust the settings with Type shown as Data and


Variable as n.

3. Press ENTER to accept the settings. With the cursor


in the cell containing c1, press CATALOG to access
seq(.

4. Press ENTER to display seq( in the entry line.


The sequence function has four parts each separated
by a comma (,). The first part is the formula
(enter 3 + (n − 1) × 2), the second part is to define the
variable (enter n), while the third and fourth parts
define the value of n for the initial term (enter 1) and
the ceiling value of n (enter 255). Enter this
information as shown in the entry line and press
ENTER .
5. Use the arrow keys to scroll down the list.

6. If you wish to widen the column, press F1 (Tools)


and select 9: Format. Press ENTER and select a cell
width of 10.

7. Press ENTER to display the table with wider


columns.
578 Appendix

Chapter 9 page 402


Graphics Calculator tip! Finding the sum of an
arithmetic sequence
For the arithmetic sequence 4, 10, 16, 22, . . ., 58 considered in Worked example 5, we
can use a graphics calculator to find the sum of the first 10 terms. The rule for this
sequence is 4 + (n − 1) × 6.
1. From the MENU, select Data/Matrix Editor and then
3: New.... Enter the settings shown with variable n.

2. Press ENTER to accept the settings. With the cursor


in the cell containing c1, press CATALOG and select
seq(. Enter the sequence 4 + (n − 1) × 6 as shown in
the entry line.

3. To calculate the sum of the first 10 terms, we can add


the terms in the sequence. Move the cursor to an
empty cell in column 2 (say, r1 c2). Press CATALOG
and select sum(. Again press the CATALOG function
and select seq(.

4. Re-enter the sequence, the variable, the starting value


of n (enter 1) and the last value of n (enter 10) as
shown in the entry line.

5. Press ENTER to display the sum of the term.

Note that the sum of the sequence could have been calculated without actually dis-
playing the sequence first.
Appendix 579
Chapter 9 page 410
WORKED Example 10
The first three terms of a geometric sequence are 2, 6 and 18. Which numbered term
would be the first to exceed 1 000 000 in this sequence?
THINK WRITE/DISPLAY
Method 3: Using a graphics calculator
1 Find the rule for the sequence. See Method 1 a = 2 and r = 3
on page 410. tn = 2 × 3n−1
2 Locate the term that equals or exceeds
1 000 000. Two methods are shown for this
example.
Method A: Generating the terms of the
sequence
(a) From the MENU, select Data/Matrix Editor
and then 3: New... with variable n as
shown.

(b) Press ENTER to accept the settings. Press


CATALOG and select seq( to enter the
sequence 2 × 3n−1 into column 1 (c1) as
shown.

(c) Press ENTER and scroll down until you


reach the term that exceeds 1 000 000. You
will need to widen the column to see this
number. (Refer to page 577 for instructions
on widening a column.)

Method B: Solving an equation


Set up an equation to solve. 2 × 3n−1 = 1 000 000
From the Home screen, press CATALOG
and select solve(. Enter the equation
2 × 3n−1 = 1 000 000 to be solved and then the
variable, separated by a comma. Close the set
of brackets and press ENTER .

3 Write your answer. The first term to exceed 1 000 000 is the
13th.
580 Appendix

Chapter 9 page 418


WORKED Example 12
A city produced 100 tonnes of rubbish in the year 2008. Forecasts suggest that this may
increase by 2% each year. If these forecasts are true,
a what will be the city’s rubbish output in 2012?
b in which year will the rubbish reach 120 tonnes?
c what was the total amount of rubbish produced by the city in the years 2008, 2009,
2010?
For the solution to part a, refer to page 418.
THINK WRITE/DISPLAY
b Method 3: Using a graphics calculator b
1 Use tn = ar
n−1
to write the rule. tn = 100(1.02)n−1
2 Locate the term that equals or exceeds 120.
Two methods are shown for this example.
Method A: Generating the terms of the
sequence
(a) From the MENU, select Data/Matrix
Editor and 3: New... with variable n as
shown.

(b) Press ENTER to accept the settings.


Press CATALOG and select seq( to
enter the sequence 100 × 1.02n−1 into
column 1 (c1) as shown. Press ENTER
to display the terms.

(c) Scroll down until you reach the term


that exceeds 120. You may need to
widen the column to see this number.
(Refer to page 577 for instructions on
widening a column.)

Method B: Solving an equation


From the Home screen, press CATALOG
and select solve(. Enter the equation
100 × 1.02n−1 = 120 to be solved and then
the variable, separated by a comma. Close
the set of brackets and press ENTER .
3 Write your answer. The first term to exceed 120 tonnes is
the 11th term or year 2018.
Appendix 581
THINK WRITE/DISPLAY
c 1 We need to find the sum of the first 3 terms. c
Method 2: Using a graphics calculator
Return to the Data/Matrix Editor used
earlier. With the cursor in row 1 of column
2 (r1c2), press CATALOG and select sum(
and then press CATALOG again and select
seq( to find the sum the first three terms of
the sequence. The entry line should display
sum(seq(100 ¥ 1.02^(n–1), n, 1, 3)).
2 Write your answer. The total output of rubbish for the
years 2008, 2009 and 2010 will be
306.04 tonnes.

Chapter 9 page 422


WORKED Example 15
Jim invests $16 000 in a bank account which earns compound interest at the rate of 12%
per annum compounding every quarter.
At the end of the investment, there is $25 616.52 in the account.
For how many years did Jim have his money invested?
THINK WRITE/DISPLAY
Method 3: Using a graphics calculator
n n
1 Use t = PR to write the rule for the sequence. P = 16 000
12
r = ------ = 3% per quarter
4
3
and so R = 1 + ---------
100
= 1.03
tn = 16 000(1.03)n
2 Locate the term that equals or exceeds 25 616.52.
You may wish to generate the sequence and scroll
down to the required term (see page 577).
Alternatively, an equation can be set up to be solved.
This method is shown here. From the Home screen,
press CATALOG and select solve(. Enter the
equation 16 000 × 1.03n−1 = 25 616.52 to be solved
and then the variable, separated by a comma. Close
the set of brackets and press ENTER .
3 Write your answer. Jim has invested money for
16 periods where a period is 3
months. So it will take 48 months or
4 years.
582 Appendix

Chapter 9 page 434


Graphics Calculator tip! Comparison of simple and
compound interest
A graphics calculator can be used to compare the yearly amounts in an account from
earning both simple interest and compound interest. Consider the scenario in Worked
example 23 where the amounts are compared over the first 5 years. The steps are shown
below. Then investigate this scenario further by drawing the graphs for both cases over
the first 10 years.
1. From the MENU, select Data/Matrix Editor and then
3: New... with variable n. Each row of the table can
be considered as a year of the investment.

2. We wish to generate terms of the sequence for the


simple interest case in column 1 (c1) and the
compound interest case in column 2 (c2). Increase
the width of the columns to 10 (see page 577 for
instructions) and then return to column 1.

3. With the cursor in the cell containing c1, press


CATALOG and select seq(. Enter the rule
10 000 + n × 1000 with variable n, initial value for n
of 1 and a maximum number of terms of 5 to
generate the yearly amounts for the simple interest
case.
4. Move to column 2 and repeat step 3 using the rule
10 000 × 1.1n to generate the yearly amounts for the
compound interest case.

5. To see the graph of the two functions, first press


APPS and select Graph from the MENU. Press
♦ [Y=] to display the function editor.

6. Enter the functions y1 = 10000 + 1000x and


y2 = 10000 ¥ 1.1x. Press ENTER after each function
is entered.
Appendix 583
7. Press ♦ [WINDOW] and enter the following
settings.

8. Press ♦ [GRAPH] to display both graphs.

9. Press F3 (Trace) and use the arrow keys to compare


the value of each investment at various points in
time. The number displayed in the top right of the
screen indicates which graph is being traced.
584 Appendix

Chapter 9 page 443


Graphics Calculator tip! Generating terms in the
Fibonacci Sequence
On this calculator, we can use the Sequence form of entering a function to display
the terms of any Fibonacci style sequence. The rule for a Fibonacci Sequence is
Fn + 1 = Fn + Fn − 1 with F0 = 1 and F1 = 1 (or any two initial values). An equivalent
rule to this is Fn = Fn − 1 + Fn − 2 with F1 = 1 and F2 = 1. The second version of the rule
suits this calculator as the rule needs to be expressed as u(n) =. So the rule becomes
u(n) = u(n−1) + u(n−2) with u(1) = 1 and u(2) = 1.
1. From the MENU, highlight Table.

2. Press ENTER and then press MODE . Select


4: SEQUENCE at the Graph line.

3. Press ENTER to return to the Table screen and then


press ♦ [Y=]. Enter u1(n–1) + u1(n–2) in the entry
line for u1(n) and press ENTER . Also set the initial
values of 1 and 1 by entering {1, 1} for ui1 and press
ENTER .

4. Press ♦ [TBLSET] and define the table beginning


at 1 and with increments of 1. Press ENTER to save
the settings.

5. Press ♦ [TABLE] to display the table. The required


sequence is shown in the second column.
Appendix 585
Chapter 10 page 468
Graphics Calculator tip! Calculating factorials

The following steps show how to calculate 12! using a graphics calculator.
1. From the MENU, select Home. Enter 12 and then
press CATALOG to access ! (factorial symbol).

2. Press ENTER to insert the factorial symbol into the


entry line of the Home screen and then press
ENTER again to perform the calculation.

Chapter 10 page 472


Graphics Calculator tip! Calculating permutations

To find the number of permutations of n objects taken r at a time, we need to calculate


n
Pr. The following steps show how to calculate 5P3 using a graphics calculator.
1. From the MENU, select Home. Press CATALOG to
access nPr(.

2. Press ENTER and then complete the entry line to


obtain nPr(5, 3). Press ENTER to display the value.

(Alternatively, you can use the letter keys to enter npr followed by ( directly into the
entry line of the Home screen instead of accessing it from the CATALOG . Press
ALPHA [N] then ALPHA [P] and ALPHA [R] followed by ( , then complete the
entry line and press ENTER .)
586 Appendix

Chapter 10 page 486


Graphics Calculator tip! Calculating combinations

To find the number of combinations of n objects taken r at a time, we need to calculate


n
Cr. The following steps show how to calculate 5C3 using a graphics calculator.
1. From the MENU, select Home. Press CATALOG to
access nCr(.

2. Press ENTER and then complete the entry line to


obtain nCr(5, 3). Press ENTER to display the value.

(Alternatively, you can use the letter keys to enter ncr followed by ( directly into the
entry line of the Home screen instead of accessing it from the CATALOG . Press
ALPHA [N] then ALPHA [C] and ALPHA [R] followed by ( , then complete the
entry line and press ENTER .)
Appendix 587
Chapter 11 page 517
WORKED Example 2
At time t seconds, a particle has a position vector given by the expression
r (t) = 2t i + (25 − t2) j metres.
˜ Use a ˜graphics calculator
a ˜ to plot the trajectory of this particle across the interval
0 £ t £ 5 seconds.
b Repeat part a above using an Excel spreadsheet.
c Determine the equation of this trajectory in the form y = f (x).
THINK WRITE/DISPLAY
a 1 Consider the components of the position a r (t) = 2t i + (25 − t 2) j
vector. Assign x to the horizontal component ˜ ˜ ˜
Let x = 2t and y = 25 − t 2
and y to the vertical. This produces two
parametric equations that we can use to
graph the trajectory.
2 Use a graphics calculator to generate a
graph of the particle’s motion.
(a) From the MENU, select Graph. Press
MODE and select 2: PARAMETRIC at
the Graph line.

(b) Press ENTER to return to the graphing


screen and then press ♦ [Y=]. In the
entry line, enter 2t for xt1(t) and press
ENTER . Similarly, enter 25 − t2 for
yt1(t) and press ENTER .

(c) Press ♦ [WINDOW] to adjust the


viewing window. Enter the settings as
shown.

(d) Press ♦ [GRAPH] to draw the graph


of the function. Press F3 (Trace) to
investigate points on the curve.

For the solution to parts b and c, refer to pages 518 and 519.
588 Appendix

Chapter 11 page 519


WORKED Example 3
At time t seconds, the displacement (in metres) of a particle A is given by
r A(t) = (8 − t) i + (8 − 4t + t2) j and the displacement (in metres) of a particle B is given by
r˜ B(t) = (t + 2)˜i + (2 − 2t + t2)˜ j .
a˜ If these particles
˜ ˜
collide, determine when they collide.
b What are the coordinates of the impact point?
c Find the Cartesian equations of the trajectories of the particles.
d Use a graphics calculator to verify the coordinates of the impact point.
For the solution to parts a, b and c, refer to pages 519 and 520.
THINK WRITE/DISPLAY
d Verify that the coordinates of the impact point are (5, 5) d
by drawing the graphs of the displacement for each
particle and locating the point of intersection.
(a) From the MENU, select Graph. Ensure that the
Graph mode is set to FUNCTION (press MODE and
select 1: FUNCTION at the Graph line).
Complete the entry line for y1(x) with the equation
for particle A. Press ENTER . Similarly, complete
the entry line for y2(x) with the equation for particle
B and press ENTER .
(b) Adjust the WINDOW settings as shown at right.

(c) To display the graphs, press ♦ [GRAPH].

(d) To display the point of intersection, first press F5


(Math) and select 5: Intersection.

(e) Navigate the pointer to each line and press ENTER .


Then navigate the pointer to set a lower and upper
bound. This is done by pressing ENTER at any point
either side of the intersection point. The coordinates
of the point of intersection will be displayed.
Appendix 589
Chapter 11 page 531
WORKED Example 7 y
A missile is fired from a point on level ground with a
Target
velocity V m/s and an angle of elevation of q to the
horizontal. The target is positioned on top of an 80 m high V
tower which is located 100 m away. The base of the tower 80 m
is in the same horizontal plane as the point of projection
O 100 m x
of the missile.
a If g is the gravitational acceleration of the missile, then show that the motion of this
missile satisfies V2 cos q (5 sin q − 4 cos q) = 250g.
b Use a graphics calculator to show those values of q in the domain 0 < q < 90° for which
the expression cos q (5 sin q − 4 cos q) is positive. State the greatest possible value this
expression has and the value of q which produces it. Provide supporting argument for
your solutions.
c Extend from your answer to part b above to find the least value of V for which the
missile can reach the target. (Use g = 9.8 m/s2.)
d Generate a spreadsheet to validate the solution obtained using a graphics calculator.
For the solution to parts a, c and d, refer to pages 531 to 535.
THINK WRITE/DISPLAY
b Use a graphics calculator to graph the function b
f(θ) = cosθ (5sinθ − 4 cosθ). You will need to
use x in place of the variable θ.
(a) Ensure that your calculator is set to
degrees.
From the MENU, select Graph.
Complete the entry line for y1(x) with
cos(x)(5sin(x) − 4cos(x)).

(b) Press ♦ [WINDOW] to enter the settings as


shown.

(c) Press ♦ [GRAPH]. The graph shows


where the expression is positive for
0 < x < 90°.

Continued over page


590 Appendix

THINK WRITE/DISPLAY
(d) To display the maximum point, first press
F5 (Math) and select 4: Maximum.

(e) Navigate the pointer to set a lower and


upper bound. This is done by pressing
ENTER at any point either side of the
maximum. The coordinates of the maximum
point will be displayed.
591

answers
Answers

CHAPTER 1 Number systems: the Exercise 1C — Surds


Real Number System 1 b d f g h i l m o q s t w
2 A 4 E 5 B 6 C
Exercise 1A — Classification of 7 2 (when a = 64) 8 m=4
numbers
Exercise 1D — Simplifying surds
1 a Q b Q c Q d I
e I f Q g Q h I 1 a 2 3 b 3 2 c 2 6 d 2 14
i Q j Q k Q l Q e 3 3 f 5 3 g 5 5 h 3 11
m I n Q o I p Q
i 3 6 j 2 15 k 4 7 l 7 2
q Q r I s I t I
u Q v I w I x Q m 2 17 n 5 6 o 6 5 p 13 2
y I
q 2 22 r 3 15 s 9 2 t 10 2
2 a Q b Q c Q d Q
e Q f I g I h Q u 7 5 v 8 5 w 8 7 x 7 15
i I j Undefined k I l I y 9 5
m I n Q o Q p Q 2 a 4 2 b 15 2 c 24 10 d 24 7
q I r I s Q t Q
u I v Q w Q x Q e 36 5 f 10 17 g 21 6 h 40 2
y I i – 30 3 j 18 7 k – 28 5 l 18 30
3 B 4 E 5 C 6 D
m 64 3 n 10 o 2 2 p 5
7 a Z+ b Q c Q d Q
e Z+ f Q g Z+ h Z+ q 2 r 2 6 s 2 3 t 2 2
i Z+ j Z− k Z− l Q 1 3 7
u --- 15 v 20 5 w --- 7 x --- 11
m Z− n Z+ o Q p Q 3 2 2

q Z+ r Z− s Z+ t Q y −8 3
u Z− v Q w Z+ x Z+
3 a 4a b 9ab c 6a 2
y Z−
8 a Z+ b I c I d Q d 3ab 6 e 3a 10b f 4a 3ab
e Z− f I g Z− h I g 13a 2 2 h 5a 2 b 6 i 13ab 2ab
i Q j Q k Z+ l Z−
m Q n Q o Z+ p I j 2a 2 b 3 3ab k 2ab 2 17ab l 4x 3 5y
q Z− r I s I t Z− m 5x 3 y 2 5 n 24x y o 20xy 5x
u I v Z+ w Q x I
y Z+ p 14xy 7xy q 54c 3 d 2 2cd r 9c 2 d 2 14d
9 C 10 C 11 B 12 D s 18c 3 d 4 5cd t 28c 5 d 5 6 u 22ef
2 2 3 6
Exercise 1B — Recurring decimals v ---
3
e f 3 30 w 7e 5 f 5 2ef x e f2 7f
---
4

1 c def g j k l n opq r t uvw x y y 1


--- xy 4 6xy z 1 5 6
x y
--- 3
9 3
2 7 8 5
2 a ---
9
b ---
9
c ---
9
d ---
9 4 E 5 C 6 D 7 C
4 1 17 19
e --- f --- g ------ h ------
9 6 45 45 Exercise 1E — Addition and subtraction
i 31
------
45
j 32
------
45
28
k 2 -----
45
- l 53
------
99
of surds
m 4
------ n 1 34
------ o 3 367
--------- p 361
--------- 1 a 7 5 b 17 2 c 8 3
33 99 495 999

q 427
--------- r 868
------------ s 323
--------- t 3 152
---------
d 19 7 e 15 5 + 5 3 f 15 2 + 7 6
999 1665 999 333

u 13
------ v 157
--------- w 1237
------------ x 5611
------------
g 4 11 h 5 13 i 13 2
18 300 1980 9000
2 j 10 7 – 11 5 k – 3 6 l –7 2+5 6
y ------
13
m 17 3 – 18 7 n 5 xy o 8 x+3 y
3 E 4 D 5 C 6 E
7 Irene. It can also be written as 0.02 . p x – 5 y + 7 xy 1A

1E
answers 592 Answers

2 a 10 ( 2 – 3 ) b 2 2 Exercise 1G — The Distributive Law


c 5( 5 + 6) d –6 6+2 3 1 a 21 + 6 3 b 3 10 – 7 5
e 7 3 f 10 2 c 2 5 – 10 d 6 + 10
g 4 5 h 5 5 e 126 2 – 14 3 f 10 21 – 4 6
i 14 3 + 3 2 j 11 – 4 11
g 72 + 14 30 h – 30 15 + 80 6
k 3 6+6 3 l 17 2
i – 24 2 + 12
m 15 10 − 10 15 + 10 n 0
2 a 3 10 + 9 2 – 5 5 – 15
o – 8 11 + 22 p 39 3
b – 35 – 11
q 12 30 – 16 15
c – 4 – 40 3
r – 2 5 – 5 2 – 2 30 + 2 15
d 24 3 – 18 30 – 8 10 + 60
s 12 ab + 7 3ab t 0
7
e 112 – 140 3 + 24 6 – 90 2
u ---
2
2+2 3 v –3 2
f 2 55 – 2 22 – 4 15 + 4 6
5
w 15 2 x ---
8
3
g 10 35 + 14 14 – 15 10 – 42
3 a 34 a – 6 2a b 52 a – 29 3a h 180 – 30 3 – 18 6 + 9 2
c 6 6ab d 32a + 2 6a + 8a 2
i 15x + 26 xy + 8y
e a 2a f a + 2 2a
j 4x + 2 5xy – 10y
g 3a a + a 2 3a h ( a 2 + a ) ab
3 a 27 + 10 2 b 16 + 4 15
i 4ab ab + 3a 2 b b j 3 ab ( 2a + 1 )
c 18 + 6 5 d 53 + 10 6
k – 6ab 2a + 4a 2 b 3 3a l – 2a b
4 D 5 E 6 A 7 E 8 B e 35 + 12 6 f 53 + 12 10
9 a 12 2 cm b ( 6 6 + 8 3 ) cm g 104 + 60 3 h 14 – 6 5
c ( 18 – 2 3 + 2 5 ) cm d 3π 5 m i 10 – 2 21 j 37 – 8 10
e ( 18 2 + 2 5 ) m f 21 11 m 4 a −46 b 18 c 11
d 50 e 6 f −2
Exercise 1F — Multiplication of surds g 10 h −5 i 7
1 a 14 b 55 c 42 d 2 6 j 51 k 26 l −1
m 7 n 17 o 63
e 4 3 f 6 2 g 10 h 5 15
p 44 q 53 r 343
i 3 7 j 4 10 k 27 l 30 3 s 76 t 17 u x−y
m 10 33 n 96 6 o 180 5 p 126 v 2x − 3y w 9x − 16y x 4x3 − 25y
q 120 r 144 y xy(49x − 9y) z xy(81x − 25y)
s 120 3 t 360 3
5 A 6 C 7 E 8 D
u 2 6 v 6 w 2 2--3- x 4
--- 5
3
9 a 57 – 12 15 b 59 – 12 15 + 9 5 – 6 3
2
y ---
5
6 z 3 3
Exercise 1H — Division of surds
2 a x2 y y b x2 y3 x
1 a 5 b 7 c 2 d 2 3
c 3a 4 b 2 2ab d 5abc 2 2abc
3
e 6 f 15 g 4 h ---
e 6a 5 b 2 2b f 6a 3 b 4 5

g 3x 2 y 2 10xy h 15x 6 y 2 2 3 4 2 5
i ------- j ---------- k ------- l 5 6
4 3 2
9 1
i ---
2
a 2 b 4 5ab j ---
2
a 3 b 2 2ab
m 2 3 n 1 --15- o 1 p 2 6
3 a 98 cm2 b 75π cm2
d 6 6 m2 q 1 4--5- r 3 3 s 2 17 t 2 2--5-
c 20 11 m2
e ( 45 π + 96 10 ) m2 f 72 15 m2 x 1 2
u -- v ---------
- w ---------
- x 6x xy
4 E 5 C 6 D 7 A 8 15 360 2 y x3 y2 x3 y4
593

answers
Answers

2x 2 4 a 3b 2 2b
2 a 2xy 3y b -----------
- c ---------- d ------------------- 2 21 – 35
3y 3 2a a f -----------------------------
14
2 2 15
e ---------------------- f --------------
- 15 15 – 20 6
3m 3 n m 2m 2 n 2 g ------------------------------------
13
3 B 4 E 5 A 6 C
9 11 + 9
7 a 4 13 m b 4 6 cm c 7 11 m h ----------------------
20
15
d 3 7 m e 5 13 cm f ------
2
5 cm 5 14 + 2 10 – 25 7 – 10 5
i ------------------------------------------------------------------------
155
8 a 4 5 5
b ------- c 2 2
3 j 12 2 – 17
9 a 35 2 cm b 126 L 19 – 4 21
k -------------------------
5
Exercise 1I — Rationalising 9 2 + 154
denominators l ------------------------------
4
1 a 5---------2- 7 3
b ----------
4 11
c -------------
4 6
d ---------- – 20 2 + 9 10 + 4 30 – 9 6
m --------------------------------------------------------------------------
2 3 11 3 2
2 21 10 2 15 3 35 3
e ------------- f ---------- g ------------- h ------------- n -------
7 2 5 5 12
5 6 4 15 5 7 8 15 3 3+2 6
i ---------- j ------------- k ---------- l ------------- o ---------------------------
6 15 14 15 18
8 21 8 105 10 – ( 10 3 + 15 6 + 9 2 + 27 )
m ------------- n ---------------- o ---------- p ----------------------------------------------------------------------
49 7 3 42
3 10 – 2 33 12 3 – 4 + 3 6 – 2
2 a 2+2 b --------------------------------- q ----------------------------------------------------
6 52

12 5 – 5 6 9 10 60 2 + 10 30 – 6 10 – 5 6
c ------------------------------ d ------------- r ------------------------------------------------------------------------
10 5 35

3 10 + 6 14 5 6 115 + 31 21
e --------------------------------- f ---------- s -------------------------------
4 3 148
71 – 12 33
3 22 – 4 10 21 – 15 t ----------------------------
g --------------------------------- h -------------------------- 17
6 3
u 18 2 + 10 6 – 9 3 – 15
14 – 5 2 12 – 10
i ---------------------- j ----------------------
6 16 102 + 48 6
v ----------------------------
95
6 15 – 25
k ------------------------- – 9 154 + 132 + 42 2 – 8 77
70 w ---------------------------------------------------------------------------
3 B 4 D 5 C 6 A 50

21 15 2 6 7 3+9
7 a ± ---------- b ± ---------- c ± ---------- x -------------------
7 3 3 3
21 5 – 6 14 – 5 70 – 20
Exercise 1J — Rationalising y ------------------------------------------------------------------
27
denominators using conjugate surds
– 6 + 6 2 + 10 – 2 5
1 a 5–2 z ---------------------------------------------------------
2
3+ 6
b ---------------- 9 2+8
3 2 a -------------------
14
2 2+ 5
c ------------------------ 9 7 – 13 3
3 b ------------------------------
120
2 6+ 7
d ------------------------ 16 210 – 12 14
17 c ------------------------------------------
77
8 11 + 4 13
e ---------------------------------
31 d 6–7 2 1E

1J
answers 594 Answers

6 a
– ( 45 + 15 14 + 9 10 + 6 35 ) x –2 –1 –0 –1 2 3 4 5 6
e ----------------------------------------------------------------------------
5
y –8 –6 –4 –2 0 2 4 6 8
66 + 24 6
f ------------------------- |y| –8 –6 –4 –2 0 2 4 6 8
5
g 5 – 4 14 b y = |2x – 4|
y y = 2x – 4
c R and y ≥ 0
4
959 + 281 77 + 182 7 + 6 11
h -----------------------------------------------------------------------------
629 x
2
3 + 7 65 – 16 11 –4
i ---------------------------------------------
28
– ( 41 + 6 30 ) Exercise 1L — Solving equations using
j ---------------------------------
12 absolute values
1 a x = ±5 b x = 4 or x = −6
230 + 257 3 – 137 5 – 80 15
3 a ----------------------------------------------------------------------------- c x = 1 --12- or x = − --12- d x = --23- or x = −2
431
–6 + 2 15 + 3 10 – 5 6 e x = − 1--2- f x = ±9
b --------------------------------------------------------------
6 2 a x=2 b No solutions
4 A c No solutions d x=5
14
5 a −2 ----- 6 35 312 35 3 a x = 6 or x = 1 1--3- b x = 10 or x = 2
---
19
- b ------------- c – ------------------- 5
19 361
c x = 3 or x = − 1--5- d x = 1 1--2- or x = 3 1--4-
210 2 – 120
6 a -------------------------------
41
Exercise 1M — Solving inequations
200 2 – 126
b ------------------------------- 1 a 1<x<2
41 0 1 2 3 4

99 238 – 50 400 2 b x > 2 or x < −2


c --------------------------------------------- –3 –2 –1 0 1 2 3
1681
c 1 1--2- <x<2 1
99 120 – 50 460 2 –1 0 1 1–2 2 3 4
d ---------------------------------------------
1681
d 2<x<3
e 44 0 1 2 3 4

f 103 – 90 2 2 a x < 0 or x > 2


–1 0 1 2 3 4
g 7 2+4
b x < 1 or x > 1 1--3- 1
0 1 1–3 2 3
295 2 – 382
h -------------------------------
49 c x < 3 or x > 4 1--2- 1
1 2 3 4 4–2 5
7 Yes
d x > 2 1--2- or x < 2 1
11 + 5 0 1 2 2–2 3 4
8 a -----------------------
6
3 a −4 < x < 4
–5 –4 –3 –2 –1 0 1 2 3 4 5
42 5 + 28 7
b ---------------------------------
17 b −5 < x < −3
–6 –5 –4 –3 –2 –1 0

Exercise 1K — Further properties of c 2<x<3


0 1 2 3 4 5
real numbers — modulus
1 d −9 < x < 11
1 a 19 b ---
4
c 0.75 d 15 –10 –5 0 5 10
1
e 8 f 2a g 12 h ---
2 4 a x < −1 --13- or x > − --23-
–3 –2 –1 0 1 2
–11–3 – 2–3
i 3.21 j 0 k − 2--3- l 4
b x < 1 1--4- or x > 1 3--4-
m 10 n 10 o a2b2 p −16 0 1 11– 13–4 2 3
4
q 27 r 15 s −72 t −54
u 1 c x < 3 or x > 9
---
2
v −8 w −11 x 30 2 3 4 5 6 7 8 9 10

y −3a z −6cd d x < −3 or x > − --13-


–4 –3 –2 –1 _ 1– 0 1 2
2 C 3 E 4 D 5 B 3
595

answers
Answers

e 1 2--3- < x < 7, x ≠ 3 34 a −11 b −3


0 112– 2 3 4 5 6 7 8 35 C 36 C 37 C 38 A
3
39 D 40 B
f 3 1--5- < x < 5 1--3- , x ≠ 4
3 31–5 4 5 51–3 6 7
Modelling and problem solving
Chapter review 1 a 5------------
10
- amps
5 102
b ---------------- amps
1 A 3 6
2 a Irrational, since equal to non-recurring and non- 135 38 5 34
c ------------------- amps d ------------- amps
terminating decimal 19 2
b Rational, since can be expressed as a whole b 18 π 37 cm3
2 a 2 37 cm
number
c Rational, since given in a rational form c 54 π 3 cm3 d 18 π ( 37 + 3 3 ) cm3
d Rational, since it is a recurring decimal 3 10 π 2 15 π
e Irrational, since equal to non-recurring and non- 3 a 360 10 cm3 b ----------------- cm c ----------------- cm
π π
terminating decimal
3 A
4 B CHAPTER 2 Number systems:
5 a Z− b Z+ c Q d I
6 D
complex numbers
7 E
Exercise 2A — Introduction to complex
62
8 a ------
99
b 337
--------- c 157
--------- numbers
900 165
9 E 1 a 3i b 5i c 7i d 3i
20 3 3 2 6
10 a 2m , ------ , m, 8m e 11 i f 7i g --- i h --- i
m 3 5

m 20 2 a 9, 5 b 5, −4 c −3, −8
b 25m , ------ , ------ d −6, 11 e 27, 0 f 0, 2
16 m
11 B 12 C g –5, 1 h 0, –17
13 a 72x 3 y 4 2xy b – --14- x 2 y 5 xy 3 a −1 + i b 1+i c 1−i d 0 + 0i
14 A e −1 + 2i f −1 + i g 1 + 0i h 1 – 2i
15 a 25 3 b 3ab ab 4 z = −2 − 3i, w = 7 + 3i
16 a 5 m b ( 17 – 4 6 ) cm 5 a −5 b 15 c 0 d −6
e 2 f 0 g −9 h 2
c ( 26 – 4 2 ) m d 22 cm 6 4−i
17 D 7 a E b C c C d E
18 a 27 b 720 2 8 Check with your teacher.
19 A 20 D 21 B 22 A
23 E 24 ( 23 6 – 48) cm Exercise 2B — Basic operations using
25 C complex numbers
5 7 1 a Im (z) b Im (z)
26 ----------
4
–1 1 2 3 4 Re (z)
x 5y b x2 y 2 c 3a 1 3+i –2
27 a ------------- –3
2
28 E 0 1 2 3 Re (z) –4
–5 4 – 5i
29 C
c Im (z) d Im (z)
2323 – 594 14
30 a -------------------------------------
50 –2 –1 –10 Re (z) 7 + 3i
–2 3
2
2277 – 606 14 –3
b ------------------------------------- –4 1
50 –5 0 1 2 3 4 5 6 7 Re (z)
–2 – 6i –6
c 51 – 12 14 – 18 7 + 27 2
e Im (z) f Im (z)
3 7– 3
31 -----------------------
40 –8 + i 3 2
0 1 2 3 Re (z)
3 1
32 ------- m2 –1
2
33 A
–2 5 – 2i –8 0 Re (z)
1J

2B
answers 596 Answers

2 a 4−i b 1 − 14i c −4 − 2i b Im (z)


z = –1 + 3i
d 9 − 13i e −12 + 4i f −9 − 5i
3 a −12 + 3i b −19 − 8i c 12 + 23i
d −25 + 3i e −50 − 48i f −41 − 28i
Re (z)
4 a 7 − 23i b 4 + 45i c −50 − 13i
d 63 − 37i e −85 − 132i f 176 − 61i
z = –1 – 3i
5 a 111 + 33i b 31 − 8i c 22 − 48i
d 61 e −53 f 32 − 126i c z = – 4 + 5i Im (z)
6 14 + 52i 7 −3
8 a −8 b −5 c −9 d 35
Re (z)
e −30 f −115
21 16
9 a x = 5, y = −2 b x = ------ , y = – ------ z = – 4 –5i
41 41
3 Check with your teacher.
c x = 1, y = 5 d x = −2, y = −3
1
4 --- + 1--2- i
10 a E b B c C 2

11 a Im (z) b 7
c − -----
- +
26
i
Im (z) 5 a 0−i b 0−i 25
------
25
4 3 + 4i
14 23 43 18
d ------
29
− ------
29
i e ------
53
+ ------
53
i
0 11 Re (z)
–2 2 5 – 6 2--------------------------
2 + 15
-i
0 3 Re (z) 11 – 2i f ----------------------- +
7 7
c Im (z) d Im (z) 2+i 3–i 4 + 3i
6 a ----------- b ---------- c --------------
5 10 25
0 2 Re (z) 5 – 4i – 3 – 2i
d -------------- e ------------------ 3 + 2i
0 6 Re (z)
f ----------------------
41 13 5
–2
6 – 2i –4 2 – 4i 7 10 + 24i
e Im (z) f 23 9 17
Im (z) 8 a ------
10
b ------
10
c ------
5
d − 16
------
5
e − 14
------
5
0 Re (z) 9 17
------ + --92- i 10 –29 11 −33 + 58i
2
0 10 Re (z)
12 a D b C c B
–10 –10i 13, 14 and 15 Check with your teacher. 16 −16
g Im (z) h Im (z)
17 a −12 + 11i b −30 − 19i c 0
0 32 Re (z) –88 + 16i
16 18 Check with your teacher. 19 a –4, 16, –64

–24 –88 0 Re (z) 20 x = −1, y = ± 2 21 a = − --12- , b = 1


---
2
32 – 24i
22 x = 2, y = 1; x = −2, y = −1
12 Im (z) 23 a i 13 ii 5
2 3
iz, i5z
3 z, i4z, –i2z c i -----
13
- − ------ i
13
ii --15- − --25- i
2
1 d −8 + 16i
–3 –2 –1–10 1 2 3 Re (z) e −2 + 10i
–2 – 4 + 7i
i2z i3z, –iz f ------------------
–3 65

Exercise 2C — Conjugates and division Exercise 2D — Radians and coterminal


of complex numbers angles
1 Im (z)
1 a 7 − 10i b 5 + 9i c 3 − 12i π
π
3— π –
4 – 4
d 7 + 3i e 5 − 2i f – 6 + 11 i 2
π
5— π

6
2 a Im (z) π 6 0
2π Re (z)
7—π
z=3+i 6
π
5— π
7—
Re (z) 4 π
3— 4
z= 3 – i 2
597

answers
Answers

π π 3π 3π 5π e i Im (z) ii 130
2 a --- b --- c ------ d ------ e ------
4 3 4 2 6
0 9 Re (z)
3 a 210° b 225° c 240° d 300°
4 Im (z)
–7 z+w–u
(b)
(c)
(a) (d) f i Im (z) ii 10
z2
6
Re (z)
(e)
(f) 0 8 Re (z)

4 a Im (z)
History of mathematics
1 Probability 6 z2 z3
2 He was a foreigner. 4
3 Tutoring students and writing books z1 2 z4
4 Newton –4 –2 2 4 6 8 10 Re (z)
5 De Moivre predicted it.
b 42.5 square units
Exercise 2E — Complex numbers in 5 a Im (z)
polar form 12 w
10
1 a Im (z) b z = 4 5 8
z = 4 + 8i 6
8
4
z
2
u
–4 –2 0 2 4 6 8 Re (z)

b 24 square units
0 4 Re (z)
π π
6 a 0.588 b --- c – --- d 2.034
6 4
2 a 13 b 3 c 65 2π π
e – ------ f −1.030 g --- h π
d 3 5 e 5 f 5 3 2
π
i – --- j 0
3 a i Im (z) ii 17
z–w 4 2
π π π 3π
7 a – --- b --- c – --- d ------
2 6 8 4
–1 0 Re (z) 5π 6π 2π 11 π
e – ------ f ------ g ------ h ---------
b i Im (z) ii 37 6 7 5 12
6 u+z
π 3π
8 a 3 2, – --- b 5 2, ------
4 4
2π π
0 1 Re (z)
c 2, – ------ d 8, ---
3 6
c i Im (z) ii 10 e 149, – 2.182 f 2 10, 1.893
0 Re (z) π
6 g 4, ---
3
3π π
–8 w–u 9 a 2 cis ------ b 2 2 cis ---
4 6
d i Im (z) ii 53
3π π
c 10 cis  – ------ d 2 5 cis  – ---
 4  3
0 7 Re (z)
–2 2π 3π
w+z e cis  – ------ 2
 3 f ------- cis ------
4 4 2B

2E
answers 598 Answers

3 2
10 a – 1 + 3i b ---------- (1 + i) Chapter review
2
1 B 2 B 3 −1 − 2i
5 4 C 5 E 6 C
c ------- ( − 3 + i ) d 2–2 3 i
2 7 a −48 b a = −549, b = 296
14
e ---------- (1 + i) 8 a 11 − i b 29 c 6 5
f 8i g – 3
2 9 C 10 A 11 A
1
11 C 12 B 13 D 14 E 15 D 12 ------ ( 12 – 14i )
17
Exercise 2F — Basic operations on π
13 a --- b π
complex numbers in polar form 6
14 a 135° b 210°
3π π π
1 a 6 cis -----
- b 20 cis --- c 6 5 cis  – --- 15 B 16 D 17 B
4 3  4

18 7 2 cis  – ------ 19 E 20 A
2π π  4
d 6 cis ------ e 2 7 cis  – ---
3  6 21 B
22 a –527–336i 117 44
2 a –3 2+3 2 i b 10 + 10 3 i b ---------------- + ---------------- i
15 625 15 625
c ±(1 − 2i)
6 3 2
c 3 10 – 3 10 i d – ------- + ---------- i
2 2 Modelling and problem solving
π π
e 21 – 7 i 1 a cos ------ + sin ------ i
12 12
5π π b z = 1 + 3 i, w = 2 + 2 i
3 a 4 2 cis ------ b 8 3 cis  – ---
12  2
2 + 6 + ( 6 – 2 )i
c ------------------------------------------------------
π 4
c 8 2 cis  – ------
 12 6+ 2 6– 2
d i -------------------- ii -------------------- iii 2 – 3
π 11 π 4 4
4 a 3 cis --- b 4 cis ---------
e Check with your teacher.
2 12
2 a i 2 ii 0 b x=3
3π 3π
c 2 cis  – ------ d 2 2 cis  – ------ 3 a iz = −2 + 3i, i2z = −3 − 2i, i3z = 2 − 3i, i4z = 3 + 2i
 10   14 
b i4z = z
3 2 7π c Im z d Im z
e ---------- cis ------
4 12 4 4
iz 3 iz 3
π 3 6 3 6 2 4
i z 2 i 4z
5 a i 3 3 cis --- ii ---------- + ---------- i 1 1
4 2 2
0 –4 –3 –2 –1 0 Re z
b i 16 cis π ii −16 –1 1 2 3 4 Re z
–4 –3 –2 –1 –1 1 2 3 4
–2 2 –2
c i 9 cis π ii −9 i 2z –3 i 3z i z –3 i 3z
–4 –4
3π ii – 16 2 + 16 2i
d i 32 cis ------
4 e 13
6 a − 1--4- b − 1--8- i c − 1--4- + 1--4- i f i, ii and iii One-quarter turn (rotation by 90°) in an
anticlockwise direction.
3 1
d ------- – ------ i e 0.171 – 0.046i f 16 g One-quarter turn in an anticlockwise direction
64 64
h Circle with centre at the origin and radius
7 – 64 3 – 64i 2 2
8 1 r = x +y
9 a B b C c E Im z
8 π z = x + yi
10 ---, – --- 11 16 − 16i
9 6 zi
r y
2 π
12 −64 + 64i 13 ---, – --------- x
5 120 Re z
3
14 a ±(3 + 2i) b ±(3 − 2i) zi
zi 2
c ±( 1 + 2 + – 1 + 2 i ) d ±(2 − i)
599

answers
Answers

CHAPTER 3 Matrices Exercise 3B — Multiplying matrices


1 a A (2 × 2), B (2 × 2), C (3 × 2), D (1 × 2),
Exercise 3A — Operations with
E (2 × 3), I (2 × 2)
matrices
b CA, DB, AE, AI, IA, IB, A2, EC
1 Matrix Order 2, 1 element 1, 3 element c (3 × 2), (1 × 2), (2 × 3), (2 × 2), (2 × 2), (2 × 2),
(2 × 2), (2 × 2)
A 2×2 8 —
20 14
B 3×1 5 — d – 4 18 – 8 2 – 3
44 22 2 – 2
–8 –8 6 4 5
C 1×4 — 10 4 5

D 2×3 4 4 2 – 3 1 1 – 8 – 21 14 15
4 5 1 0 28 13 – 24 – 30
E 3×3 1 2

10 20 8 26 b No
33 –7 3 6 –3 3 –6 2 a
2 a b c d – 5 10 – 4 12
09 85 –2 8 –2 –1
3 a 4 3 b 2 –7 c 00 d 32
3 a –4 6 b –9 6 c 11 6 0 –9 24 9 00 –8 5
8 14 12 12 – 4 20
e 2 0 f 10 – 4 g 31 0 h 10 – 11
d 9 9 e 7 18
0 –3 – 24 – 9 0 31 16 1
0 27 – 12 – 4
4 a C b D c E d A e B i 00 j 32
00 –8 5
2 0 14 0 8 0 2 8 14
5 a b c 4 a i 10 ii 1 0 iii 10
40 0 0 10 16 4 10 16
01 01 01
6 0 18 0 12 0 6 12 18
b All are I
3 4 21 4 0 28 –1 4 –7 c Multiplicative inverses
d 65 8 e 80 0 f –2 5 8 5 a A b C c D d B
9 6 27 12 0 36 –3 6 –9 3
6 a 1
2 –6
2 0 b –2 0 c – 1 0 1 d – 1 – 12 0
6 a
3 –1 –6 2 2 3 –1
–2 –1 32
b Sharks have a total of 32 points. Dolphins
7 Different orders 31 have a total of 31 points.
18 12
0 1 3 1 0 0 1 1
1 0 2 2 0 0 1 2 7 a 14 15 b 6
8 a b 10 14 1
3 2 0 1 1 1 0 3
9 16
1 2 1 0 1 2 3 0
c Southport 120, Broadbeach 99, Lions 74,
Eagles 70
82 54
9 75 68 10
91 82 8 a 25
12
10 a 15 14 104 b 13 7 5 1 31 18 26 b Shop A = $820, Shop B = $345
7 10 52 12 4 4 4 17 15 16
History of mathematics
14 8 5 1 35 19 29 1 Matrix theory and number theory
11 2 Computer development
13 4 4 5 18 19 16
3 Cross of Honour
12 a True b True c False d True 4 Caltech 2E

3B
answers 600 Answers

Exercise 3C — Powers of a matrix


1 1 1
g ------ 1 – 5 h ------ – 132 – 114 or --- – 44 – 38
4 –2 8 –4 16 – 8 15 2 5 15 186 162 5 62 54
1 a b c
0 0 0 0 0 0
13 a 2 b 2
1 0 0 1 0 0 1 0 0 –1 3
2 a 0 1 0 b 0 1 0 c 0 1 0 14 a x = −2, y = 1 b x = 1, y = 2
0 0 1 0 0 1 0 0 1 c x = −2, y = 3 d x = 7, y = 4

1 0 0 1 00 Exercise 3E — The transpose of a


3 a 8 9 0 b 26 27 0 matrix
2 4 1 10 13 1 Check with your teacher.

–1 –1 –1 Exercise 3F — Applications of matrices


4 a 1 1 2 1 a (5, −1) b (3, 0) c (10, 2)
–1 –1 –1 d (0, 0) e (4, 4) f (−2, −3)
2 a and b Answers will vary. c det = 0
d i y ii y Both lines
Exercise 3D — Multiplicative inverse
and solving matrix equations 0 2 x
0 4– x
3
2 3
1 1 –1
1 AB = 6 1 0 a A –1 = --- B b B –1 = --- A –3
01 6 6 –2
–3
1 1
2 MN = – 2 1 0 , M –1 = – --- N , N –1 = – --- M e In i there are parallel lines; in ii there is only one
01 2 2
line.
3 a 5 b 12 c −2 d −8 e 7 f 14 3 a E b B
4 a C b D
1 5 16, 4 6 15, 10 7 Anh 8 $51 070
4 a --- 10 – 3 1
b ------ 0 3
1
c --- 1 6
5 –5 2 12 – 4 – 2 2 0 –2
Exercise 3G — Dominance matrices
1 1 1
d --- – 1 3 e --- – 5 1 f ------ 4 1 1 a C
8 4 –4 7 3 –2 14 – 6 2
T K
5 a C b E c D d A B
6 Answers will vary. b Cameron, Breanna, Kayley, Teagan
2 Mair, Ann, Janine
3 a Hamilton, Leslie, Cunningham, Barnes
1 1
7 a --- 1 6 b --- 1 2 c 12 2 b 20 points to Hamilton, 15 to Leslie, 10 to
2 0 –2 4 –2 0 –2 –1 Cunningham, 5 to Barnes

1 1 1 1 2 0 0 1 1 0
d --- 1 2 e --- – 11 2 f ---
8 – 2 – 12 8 40 8 – 2 – 12 1 0 1 1 0
4 a M= 0 0 0 0 1
8 a D − det = 0 b E − det = 0
c F − Not a square matrix 0 0 1 0 1
1 1 0 0 0
0 8 1
9 a b --- – 2 – 8 b 5 points to Warwick, 4 points to Ipswich, 3 points
–1 –2 8 1 0
to Stanthorpe, 2 points to Clifton, 1 point to
10, 11 Check with your teacher. Goondiwindi

1
12 a --- – 31 – 22
1
b --- – 5 5
1
c --- – 6 2 Chapter review
2 24 18 2 14 – 8 6 –6 4 1 D 2 B 3 C 4 B 5 a, f, g, i

1 1
d --- 18 23 e ------ 78 103 f 10 6 a –3 2 b –7 –6 c 5 – 14
2 – 12 – 16 30 – 24 – 34 01 –2 –4 6 –4 14 12
601

answers
Answers

100 100 1 00
CHAPTER 4 An introduction to
7 a 142 b 384 c 7 16 8 groups
100 100 1 00 Exercise 4A — Modulo arithmetic
8 a A b B 1 a 4, 12, 20, 28, 36 … b 4, 10, 16, 22
2 a 0, 1, 2 b 0, 1, 2, … 8
9 D 10 C 11 C c 0, 1, … 10
12 A 13 93 14 B 3 a + 0 1 2 3 4 5
–6 0 0 0 1 2 3 4 5
1 1 2 3 4 5 0
1
15 a ------ 1 2 b (5, −1) 2 2 3 4 5 0 1
10 – 3 4 3 3 4 5 0 1 2
16 a [0.25 0.40 0.20 0.15] [800] = [a b c d] 4 4 5 0 1 2 3
5 5 0 1 2 3 4
b A = 800B
c [200 320 160 120] b × 0 1 2 3
0 0 0 0 0
˙ 0
1.12 278.88 27.55 1 0 1 2 3
17 a b
0 0.95 761.60 46.55 2 0 2 0 2
3 0 3 2 1
18 James, Cameron, Glen, William
c × 0 1 2 3 4
Modelling and problem solving 0 0 0 0 0 0
1 a 1: Despatch for Deluxe model takes 1 hour. 1 0 1 2 3 4
b 14: Packaging at Plant 1 has a wage rate of $14 per 2 0 2 4 1 3
hour. 3 0 3 1 4 2
4 0 4 3 2 1
c 3 × 3, 3 × 2, 3 × 2

433.25 420.50 History of mathematics


d 529.50 514.00 1 He tutored students.
605.25 587.50 2 Abelian groups are those that have the property of
commutativity.
e The total costs for the Standard model at Plants 1
and 2 Exercise 4B — The terminology of
f The assembly costs for each model at Plant 1 groups
g i $529.50 ii $514.00 3+2
1 ------------ = 2 1--2- and 2 1--2- is not an element of the set of
2 a 145x + 103y + 121z = 20 260 2
whole numbers.
130x + 110y + 90z = 18 400
142x + 115y + 80z = 19 200 2 1 ° 3 = 1 + 9 = 10
Not a whole number, ∴ not closed.
145 103 121 x 20 260 3 a IE = 1 b b = 2 − a
b 130 110 90 y = 18 400 4 IE = 0 a ° 0 = a + 0 − a × 0
142 115 80 z 19 200 5 No identity. 4a × ( 1--2- )2 = a but 1--2- ° a ≠ a
6 Assuming this operation has an identity then let
0.025 544 –0.093 523 0.066 579
a+b
c A−1 = – 0.039 222 0.091 991 – 0.044 166 ------------ = a
ab
0.011 042 0.033 767 – 0.042 189 a + b = a2b
d The cost of an adult’s ticket is $75, a child’s ticket a = a2b − b
is $50 and a pensioner’s ticket is $35. But a ≠ a2b − b therefore the operation has no
identity.
a b 7 Let (0, 1) = (a, b) = IE. Therefore, (0, 1) ° (c, d) =
3 A= 2 (0 × d + 1 × c, 0 × c + 1 × d) = (c, d) and (a, b) °
a
– ----- – – a (0, 1) = (a × 1 + b × 0, a × 0 + b × 1) = (a, b).
b
8 Let (a + b)2 = a where b = IE
a b Take the square root of both sides: a + b = ± a
4 A= 2
If a is negative then a ∉ R. Since an identity must
a–a be applicable to all elements of the set, there is no IE
-------------- 1 – a
b for a ° b. 3C

4B
answers 602 Answers

Exercise 4C — Properties of groups Exercise 4D — Cyclic groups and


1 a [R, +] It is closed, associative, IE+ = 0, subgroups
inverse = −a, therefore it is a group. 1 Closed, IE+ = 0, inverses exist (3 + 9 ≡ 0 mod 12,
b It is Abelian. 6 + 6 ≡ 0 mod 12), therefore subgroup. Generators are
2 a Closed, associative, no IE since 3 and 9.
0 ∉ {even numbers}, there is an inverse; therefore
not a group. 2 Closed, IE = 0 0 , the inverse of a 0 is – a 0
b Closed, associative, no IE since 00 0b 0 –b
1 ∉ {even numbers}, no inverse; therefore not a
group. which is a member of M, therefore a subgroup.
3 a 12 + 13 is not closed; not a group. 3 a <3> = [{0, 3, 6, 12}, + mod 12];
b 12 × 13 is closed, and associative, IE× = 1, there is <6> = [{0, 6}, + mod 12];
an inverse; so it is a group. <9> = [{0,3,6,12}, + mod 12]
4 Check with your teacher. b Generators are 3 and 9.
5 × 1 2 3 4 4 a Cyclic. Generators are 1 and −1.
1 1 2 3 4 b Not cyclic.
2 2 4 1 3 c Cyclic. Generators are 6 and −6.
3 3 1 4 2 d Cyclic. Generators are 3 and 1--3- .
4 4 3 2 1
Closed, associative, IE× = 1 and there is an inverse;
therefore it is a group.
History of mathematics
6 a 1 He worked towards having women accepted at
+ 4 16 64 256 … Cambridge University.
4 8 20 68 260 … 2 Abstract algebra, group algebra, n-dimensional
16 20 32 80 272 … geometry, matrices and determinants
64 68 80 128 320 …
256 260 272 320 512 …
. . . . . Exercise 4E — Further examples of
. . . . .
. . . . . groups — transformations
b Under addition: not closed, associative, no IE+ 1 A B C
since 0 ∉ 22n, no inverse (always +ve); not a
group.
c Under multiplication: closed, associative, IE× = 1
is not present as no 20 (0 ∉ {even numbers}),
inverse is 2–2n; not a group. C R0 B A R120 C B R240 A
7 × 0 1 2 a ° R0 R120 R240
0 0 0 0 R0 R0 R120 R240
1 0 1 2
R120 R120 R240 R0
2 0 2 1
R240 R240 R0 R120
It is closed and associative, IE× = 1; inverse does not
exist since there are no 1s in the first row or column. b IE = R0, Inverse exists for all elements. It is an
This is not a group; therefore, it is not Abelian, even Abelian group because the table is symmetrical
though the commutative law does apply. about the leading diagonal.
8 a Yes b No, not closed 2 A
c No, no inverse for b d Yes
9 a 5 10 20
° RR RL
5 5 10 20
10 10 10 20
20 20 20 20
C RV B
b It is closed, associative, IE° = 5, no inverse; so
not a group. a R0 RV RL RR
°
10 N L R A
° R0 R0 RV RL RR
N N L R A RV RV R0 — —
L L A N R RL RL — R0 —
R R N A L RR RR — — R0
A A R L N
b Does not form a group.
Closed, associative, IE° = N, there is an inverse,
N appears in every row and column. 3 Not Abelian.
603

answers
Answers

4 a 4 3 8 a Closed: addition of 2 × 2 matrices results in a


2 × 2 matrix.
Associative: matrix addition is associative.
3 4 RV 1 2 Identity exists: 0 0 is the identity element.
2 1 00
2 1 RH Inverses exist: the inverse of A is −A.
3 4 b i 12 ii 1 2 iii The set of 2 × 2, non-
3 R0 4 1 2
R180 34 12 singular matrices.
9 Identity = I. Inverse is present as I is present in each
row and column. Closed and associative
2 1 4 3
b 3 2 3 2 10 a IE+ = 0 0 (Remember 0 is a complex number.)
0 0
–z1 –z2
Inverse =
RL RR z2 –z1
4 1 4 1
3 4 1 2 1 2
1 z –z
b IE× = I Inverse = --------------
2
- 1 2 where the
2
z1 + z2 z2 z1
R180 R0
determinant is real. The inverse exists if the
2 1 4 3 4 3 determinant ≠ 0.
c 11 b Yes. 00
4 2 Closed, associative, IE, Inverse of 0 0 is
1 1
3 1 R RV 1 3 z z --- ---
z z
H 2 i 0 – 1 0 – i 0 1 0
2 4 12 a , , ,
3 1 0 i 0 –1 0 –i 0 1
4 2
4 R0 b Yes.
1 3 R180 3 1
2 4 Chapter review
5 …H H H H… RV …H H H H… 1 a + 0 1 2 3 4
RH R180 …H H H H… 0 0 1 2 3 4
…H H H H… R0 …H H H H… 1 1 2 3 4 0
2 2 3 4 0 1
6 3 3 4 0 1 2
4 4 0 1 2 3
b × 0 1 2 3 4 5
0 0 0 0 0 0 0
1 0 1 2 3 4 5
2 0 2 4 0 2 4
3 0 3 0 3 0 3
4 0 4 0 0 4 2
5 0 5 4 3 2 1
2 a Yes b No, not closed
3 a Yes
b No, 0 does not have an inverse
c Yes d Yes
4 a No, no identity b No, not associative
5 Check with your teacher.
6 a Commutativity
b There is only element x such that p ° x = q and
7 a ° f1 f2 f3 f4 x ° q = p.
f1 f1 f2 f3 f4 c Each element has a unique inverse.
f2 f2 f1 f4 f3 7 a Yes b No, 0–1 doesn’t exist
f3 f3 — — — c No, inverses don’t always exist
f4 f4 — — — d Yes
8 Check with your teacher.
b Not a group 9 a No, there is no generator. 4C

4E
answers 604 Answers

b No, the operation is not closed on these elements. Exercise 5E — Inverse of a 3 ¥ 3 matrix
c <a> = {0, a}, <b> = {0, b}, <c> = {0, c}
10, 11 and 12 Check with your teacher. 1 a iii 1 iii 3 –2
–1 1
Modelling and problem solving
1 and 2 Check with your teacher.
iii 3 –1 iv 3 –1
–2 1 –2 1
CHAPTER 5 Matrices and their
applications b iii −12 iii 9 –6
–5 2
Exercise 5A — Inverse matrices and
systems of linear equations – --34- 5
------
iii 9 –5 iv 12
26 42 10.38 400 1
1 2 3 4 –6 2 ---
2
– 1--6-
9 36 17.31 370
17 100.29 98.04 8 2 –4
5 6 7
1 92.06 108.82 c iii 8 iii – 12 2 8
8 –2 –4
8 c 3.18 0.681 d 480.68
1.81 1.81 481.82 1 – 1 1--2- 1
8 – 12 8
Exercise 5B — Gaussian elimination iii 2 2 –2 iv 1
---
4
1
---
4
– --14-
1 x = 1, y = 1 2 a = −3, b = −1 –4 8 –4 – 1--2- 1 – 1--2-
3 x = 2, y = 1, z = −3 4 x = 1, y = 2, z = 0
5 x = −2, y = −1, z = 0 6 x = −3, y = 5 1--4- , z = −3 3--4-
13 – 5 11
1
–3 2 ---
2
– --14- d iii −33 iii – 12 – 3 0
7 a b
2 –1 – 1--2- 3
--- – 4 – 1 – 11
4

13 12 4
1
– 1--4- 0 – -----
- ------ ------
2 1--3- – 2--3- – 1 1--3- ---
2 13 – 12 – 4 33 33 33

c d 1 iii –5 –3 –1 iv 5
------
1
------
1
------
–1 0 1 0 ---
2
0 33 11 33
1 1
– 1--3- 11 0 – 11 – 1--3- 0 1
---
---
3
---
3 – 1 – 1--2- 1 3

8 a x = 2.7, y = 0.6
–3 –1 5 1 –3 –2
b x = 2 1--5- , y = 1 1--5- , z = − 3--5- 2 a b
1 0 –1 0 0 0.5
c x = −2 --17- , y = −2 --37- , z = 1 --37- 6 2 –9 – 1 4 2.5
History of mathematics
– 0.16 0.08 0.44 – 1.5 0.5 0.5
3 Science of determining the size and shape of the Earth. c 0.52 0.24 – 0.68 d – 0.85 0.45 0.25
Exercise 5C — Introducing determinants 0.28 0.36 – 0.52 0.6 – 0.2 0
1 2 2 13 3 − --12- 4 3
---
4 – 1.5 1 1.25 19
5 a −5 b −33 c 81 d −3 3 0.5 0 – 0.25 X = –2
e 61 f 0 g 0 h 24
i −24 j 122 – 1.5 1 0.75 15

Exercise 5D — Properties of 2 1
determinants 4 a 3 b 0
1 a −3 b −3 c Property 1 –4 3
2 a 0 b Property 2
3 a 0 b Property 3 5 x=± 3
4 a 22 b −22 c Property 4 6 a (y − z)(v − u)
5 a −18 b −36 c Property 5 b xy2 − xz − x2y + yz2 + x2 − y2z
6 a 2 b 2 c Property 6 c (1 − x)(1 − y) + y + x − 1
7 a 8 b Property 7 d 2a3 − 3a2b + b3
8 a 1 b Property 8 7 a a = 1, 2 b a = ± 1--2- c a = 0, 3
9 a −6 b −4 c 3 d −8 e 3 f 40 d a = 0, 1, 2 e x = 3, 4 f x = −1, 3
605

answers
Answers

1 5
---
3
---
6
– 1 2--3- –7 5 1
1
7 –5 –1
iii 4 –8 –4 iv ------ – 4 8 4
8 – --16- 7
------ – --23- 12
12
5 –7 1 –5 7 –1
0 – 1--2- 1 17 2 2
8 x= ------
27
, y = −2 -----
27
-, z = ---
9
1
0.3 0.05 0.03 ---
2 a u u b
9 a −189 b 0.15 – 0.06 0.07 c c v v d
1
9 a y = ----------------------- and x = -----------------------
– 0.24 0.03 0.13 0
a b a b
10 Check with your teacher. c d c d
0.25 – 0.5 0.75 where ax + by = u and cx + dy = v
11 a ′A = 0.5 0 – 0.5 b x = 2, y = −1
– 0.25 0.5 0.25 Modelling and problem solving

0.25 0.5 – 0.25 0 0 1 0


a A′ = – 0.5 0 0.5 1 B= 0 4 0 0
0.75 – 0.5 0.25 1 0 0 0
0 0 2 1
12 a 1 + i b 0 c −4 − 7i
13 Check with your teacher.
0 3 0 1
Exercise 5F — Cramer’s Rule for 2 C= 0 1 0 0
solving linear equations 0 0 –2 0
1 x = 2, y = −3 2 x = −1, y = 4 1 0 0 0
3 x = −2, y = −5 4 x = 0, y = 3 3 a = ±3
5 x = 3, y = −1, z = 2
6 x = −11 --13- , y = −5 --13- , z = − --13- 2 413 4 532
1 692 4 273
Chapter review 56 524 81 609
1 G = 50, H = 10 3 313 8 570
8 382 14 100
2 X = 15.15 (15.15 tonnes of aluminium, 14.1 tonnes
14.1 of gold) 4 a D = 8 844 b X = 10 501

3 x = −1, y = 8, z = 9 7 433 17 256


25 989 48 238
1 –1 0
0.2 – 0.4 13 159 13 756
4 a b 5 –7 1
0.2 0.6 16 487 19 116
–2 3 0
7 195 10 168
5 a −2 b 10 c −2 c
6 a 0 Property 3
b −30 Property 7 3713
------------ 0 – 1160
---------------- 0 – 17
----------------
– 93
---------------- 0 –2
----------------
–2
----------------
–3 –6
---------------- ----------------
4532 81 609 14 100 10 501 48 238 13 756 19 116 10 168
c −52 Property 4 3945 – 1716 – 195 – 285 – 41 –7 –2 –3 –4
d 13 Property 1 0 ------------
4273
----------------
81 609
------------
8570
----------------
14 100
----------------
10 501
----------------
17 256
----------------
48 238
----------------
13 756
0 ---------------
10 168
-

e −24 Property 5 (twice) – 821


------------
– 383
------------
69 438
----------------
– 107
------------
– 1 918
----------------
– 1 982
----------------
– 939
----------------
– 1 511
----------------
– 2 979
----------------
– 1 601 – 672
---------------- ----------------
4532 4273 81 609 8570 14 100 10 501 17 256 48 238 13 756 19 116 10 168
f 0 Property 2 – 24 – 106 – 778 4733 – 15 – 83 – 36 – 78 – 141 – 127 – 33
------------ ------------ ---------------- ------------ ---------------- ---------------- ---------------- ---------------- ---------------- ---------------- ----------------
g −15 Property 6 4532 4273 81 609 8570 14 100 10 501 17 256 48 238 13 756 19 116 10 168
– 48 – 136 – 136 – 209 16 713 – 78 – 128 – 1 415 – 628 – 286 – 41
------------ ------------ ---------------- ------------ ---------------- ---------------- ---------------- ---------------- ---------------- ---------------- ----------------
7 a iii −2 iii 4 –3 4532 4273 81 609 8570 14 100 10 501 17 256 48 238 13 756 19 116 10 168
(I − A) = – 100 – 52 – 97 – 64 – 82 10 194 – 150 – 373 – 216 – 152 – 64
–2 1 ------------ ------------ ---------------- ------------ ---------------- ---------------- ---------------- ---------------- ---------------- ---------------- ----------------
4532 4273 81 609 8570 14 100 10 501 17 256 48 238 13 756 19 116 10 168
– 130 – 416 – 981 – 103 – 106 – 1 587 14 203 – 2 140 – 649 – 384 – 274
------------ ------------ ---------------- ------------ ---------------- ---------------- ---------------- ---------------- ---------------- ---------------- ----------------
4532 4273 81 609 8570 14 100 10 501 17 256 48 238 13 756 19 116 10 168
iii 4 –2 iv –2 1
– 209 – 915 – 2 120 – 522 – 1 370 – 2 606 – 1 226 39 454 – 2 302 – 848 – 1 348
------------ ------------ ---------------- ------------ ---------------- ---------------- ---------------- ---------------- ---------------- ---------------- ----------------
–3 1 1.5 – 0.5 4532 4273 81 609 8570 14 100 10 501 17 256 48 238 13 756 19 116 10 168
–4 –9 – 34 –9 – 14 –7 – 25 – 452 13 719 –1 –5
------------ ------------ ---------------- ------------ ---------------- ---------------- ---------------- ---------------- ---------------- ---------------- ----------------
4532 4273 81 609 8570 14 100 10 501 17 256 48 238 13 756 19 116 10 168
–7 4 5 – 29 – 18 – 75 – 38 – 11 – 39 – 61 – 349 – 365 17 543 – 71
------------ ------------ ---------------- ------------ ---------------- ---------------- ---------------- ---------------- ---------------- ---------------- ----------------
b iii −12 iii 5 –8 –7 4532 4273 81 609 8570 14 100 10 501 17 256 48 238 13 756 19 116 10 168

1 –4 1
– 23
------------
4532
– 42
------------
4273
– 192
----------------
81 609
– 44
------------
8570
– 11
----------------
14 100
– 88
----------------
10 501
– 113
----------------
17 256
– 423
----------------
48 238
– 438
----------------
13 756
– 129 8 699
---------------- ----------------
19 116 10 168 5A

5F
answers 606 Answers

CHAPTER 6 Transformations 3 a A′(8, −7) b B′(−1, 5) c C′(2, −7)


using matrices y
B' 5
4
Exercise 6A — Geometric 3
2 C
transformations and matrix algebra –3 1
1 a (0, 0) b (0, −26) c (0, 4) d (51, 7) –2 2 4 6 8 x
B –2
A
2 a (2, −5) b (4, −9) c (5, 0) d (−2, −4) –3
–4
3 a A′(4, 2), B′(7, 7), C′(11, 4) –5
–6 A'
b A′(4, 0), B′(7, 5), C′(11, 2) –7 C'
c A′(0, −2), B′(3, 3), C′(7, 0) 4 A′(7, −6), B′(−1, 2), C′(15, −10)
d A′(0, 0), B′(3, 5), C′(7, 2) y
4 Check with your teacher. B'(–1, 2) 5 C(0, 5)
5 y′ = 2x′ + 7 y B(–4, 1)
y = 2x + 3 A(4, 1)
7
–4 –1 45 7 10 15 x
y' = 2x' + 7
–5
A'(7, –6)
3 –10
C'(15, –10)

–3.5 –1.5 1
0 x ---
3
– 1 2--3- 1.5 1
5 a b
6 a y′ = −(x′ − 3)2 − 1 b y′ = (x′ − 5)2 − 5 1 --13- – --16- 1.5 1
c y′ = x′2 + 3x′ − 4 d x′2 − 4x′ + y′2 − 4y′ + 4 = 0
e x′2 + 8x′ + y′2 + 2y′ + 8 = 0
1 1 – 1--3- 1.5
–2 –1 –4 c d
7 a b c 1.75 2.5 – 2 2--3- 2
–3 6 –7
Exercise 6B — Linear transformations Exercise 6C — Linear transformations
1 1 c 2 –3 and group theory
1 a 0 1
2 1 3 2 1 a P′(9, 5) b P′′(5, 1) c
d Check with your teacher. –2 1
2 a iii A′(0, −1), B′(4, 2), C′(−5, −2) 2 a P′′(1, 4) b Check with your teacher.
iii A′(−1, 2), B′(−2, 0), C′(3, −1) 3 a Non-singular b Singular
iii A′(2, −4), B′(4, 0), C′(−6, 2) 4 a P′(31, 18)
iv A′(1, −2), B′(2, 0), C′(−3, 1)
b det A = 1 (non-singular)
b iii y
d iii y′ = 4--7- x′ ii y′ = 10
17 17
2
------ x′ + ------

B'(4, 2)
iii 10x′2 − 34x′y′ + 29y′2 = 2
5 y′ = 1--2- x′ − 2
C(–3, 1)
B(2, 0) y
0 x
A' y = 3x + 2
C'(–5, –2) A(1, –2)
iii y
A'(–1, 2)
2
C(–3, 1)

y' = _12 x' – 2


B(2, 0)
0 x
C'(3, –1) x
B'(–2, 0)
– _23 0 4
A(1, –2)
iii y
–2
C'(–6, 2)
C B'(4, 0)
B
0 x

A
6 a 17x′2 − 26x′y′ + 10y′2 = 9
A'(2, –4) b 10y′2 + x′2 − 2x′y′ = 81
iv No change c 13x′2 + 10x′y′ + 2y′2 = 9
607

answers
Answers

7 a y′ = x′ b y′ = 3--2- x′ c y′ = 1--2- x′ b iii, iii y


8 Check with your teacher.
9 a, c i
y' = _3x' + _13
y 1
()
y = 2x – 1
_1
3
y' = _3x' – _13
_1
3 ()
y' = 0.3x' + 0.2 _1 0 x
–1 –3 1

0.2 y = –3x + 1
–_
2 0 _1 x
3 2

4 No change, rotation about the centre of the circle.


–1

Exercise 6E — Reflections

1 a –1 0 b 1 0 c 0 1
a, c ii 0 1 0 –1 1 0
y
y = –x + 4
d 1 0 and then 0
y' = 4 0 –1 4
4

3
– 1--2- -------
2 0 –1
e f
0 x 3 1 –1 0
4 ------- ---
2 2

2 a iii (−3, −1) ii (−4, 2) iii (1, −3)


3 1
b i y′ = ------
10
x′ + ---
5
ii y′ = 4 iv (2, 4) v (−3, 0) vi (2, −1)
y
10 a x′2 − 6x′y′ + 9y′2 − 2x′ + 5y′ = 0
(iv) (iv)'
b 11y′ = 4x′ − 5
(ii)' (ii)
(v)' (v)
Exercise 6D — Rotations 0 x
(i)' (vi) (vi)' (i)
1 a 0 –1 b –1 0 c 0 1 d 1 0 (iii) (iii)'
1 0 0 –1 –1 0 0 1 mx = 0

2 a (1 − 3
------- , 3 + ) 1
--- b (0, −4) b iii (3, 1) ii (4, −2) iii (−1, 3)
2 2
iv (−2, −4) v (3, 0) vi (−2, 1)
c ( 3---------
2
2
-,
9 2
----------
2
) d ( 1--2- − 3 3
----------
2 2
3
, − ------ 3
- − --- )
2
y
(iv)
3 1 1 3
e (−3, 2) f - − --- ,
( ------ --- + ------- ) (iii)'
2 2 2 2 (ii)
(i)'
my = 0 (vi)' (v)(v)'
x′ 2
3 a iii y′ = ---- + ------- (vi) 0 x
2 4 (i) (ii)'
a iii y′ = x
--- + 1
--- (iii)
3 3 (iv)'
x 1
a iii y′ = ---
3
− ---
3
c iii (1, −3) ii (−2, −4) iii (3, 1)
b iii y iv (−4, 2) v (0, −3) vi (1, 2)
y
(iv)
(iv)' (vi)' (ii)
1 (iii)'

y = _x +√_2
2 4
(v)
x
0.35 (vi) 0 (i)

0 x (iii)
–0.7 _1
3 1 (ii)' (v)' (i)'
my = – x
y = –3x + 1
d iii ( 3--2- − 3
-------
2
, 3 3
----------
2
+ 1--2- ) iii (2 + 3 , 2 3 − 1) 6A

6E
answers 608 Answers

2 2 y
iii ( –-----21- − 3 3
---------- , – 3
---------- + 3--2- ) x 4y
2 2 3 a ----- + -------- = 1
4 9
iv (−1 + 2 3 , − 3 − 2) iv ( 3--2- , 3 3
----------
2
) 2 2
x y (a)
3
b ------ + ----- = 1
vi (−1 − -------
2
, − 3 + --12- ) 64 9 x
(b)
y my =√–3 x2 y 2

3

4 +—
9 =1
(iv)
(i)'
(v)' (ii)'
(ii)
(iii)'
(v) 4 a y′ = --18- x′2 b y′ = 8x′2
0 x
(vi) (i) y = 2 x2 y
(vi)' y y' = 8x' 2
(iii) (iv)' y = 2x2

y' = –18 x' 2


3 a iii y′ = −x′ iii y′ = −x′ iii y′ = x′
iv y′ = --12- ( 3 − 1)2x′ or y′ ≈ 0.268x′ x
b iii y′ = x′2 iii y′ = −x′2 iii y′ = ± – x′
c iii y′ = 2x′ + 1 iii y = −2x′2 − 1
2
x
1
–x′ – 1 5 y′ = 3x′ − ---
iii y′ = ± ----------------- 2
2 y
y' = 3x' – 1–2
2 2
d iii y′ = −x′ iii y′ = x′ iii y′ = ± x′
y = 3x – 2
(c)
Exercise 6F — Dilations
y 0 x
1 a i (4, 1)
ii (−8, 3) (v) (v)'
iii (0, 3) (ii)' (ii) (iii) (iii)'
iv (6, 0) (i) (i)'
v (4, 5) 0 (iv) (iv)' x 6 a iii (1, 1--2- ) ii (−2, 1 1--2- ) iii (0, 1 1--2- )
vi (1, −3)
(vi) (vi)' iv (1 --12- , 0) v (1, 2 --12- ) vi ( --14- , −1 --12- )
b i (2, −3) y y
ii (−4, −9) (vi)' (v)
iii (0, −9) (ii) (iii)
iv (3, 0) (v)'
(ii)' (iii)' (i)
v (2, −15) (v) (i)'
vi ( 1--2- , 9) (ii) (iii)
0 (iv)' (iv) x
(vi)'
(i) (vi)
(iv)
0 (iv)' x
b i (−8, −4) y
(vi)'
(i)'
(vi) ii (16, −12)
iii (0, −12) (ii)
(v)'
(iii)
iv (−12, 0) (iv)' (i)
v (−8, −20) 0 (iv) x
(ii)' (iii)' vi (−2, 12) (i)' (vi)

(iii)' (ii)'

(v)' (v)'

3
2 a y′ = x′ − 2--- y y = 3x – 2 2 2 y
2
b y′ = −3x′ + 2 x y
(a)
7 a ------ + ------ = 1
16 36
2
16y
b 4x2 + ----------- = 1 (b)
(a)
0 x 9 x

x2 y 2

4 +—
9 =1
(b)
609

answers
Answers

Exercise 6G — Shears 2 a y b y
B'(10, 3) C'(6, 21)
1 ii a (3, 6) b (2, 5) C(6, 3)
C'(15, 3)
a y y B(1, 3)
D'(5, 15)

6 P' B'(1, 6)
5 P' A = A' D(5, 0) x
B C
A = A' D(5, 0) x
1 P
P
0 x 0 x c y B(1, 3) C(6, 3) d y
3 2
B'(–2, 3) C'(3, 3) B(1, 3) C(6, 3)
B'
D(5, 0)
c (−4, −7) d (3, 4) A = A' D(5, 0) x A = A' x
C'(6, –3)
a y y
P 4 P' D'(5, –5)
0 x 3 ii a y y' = 4x'
b y y' = 2x'
–4
y=x
0 3 x
–2 P
x x
P' –7
y = –x

e (−2, −9) f (0, −5) c y d y


y = x2
a y y y = 2x + 5

0 x 0 x
–2
x

y' = x' 2 + 3x' x


P –5 –5 P = P'
y' = 5x' + 5

P' –9 ii a y y =x
b y
y' = 1–2 x'
1–
1 ii a (3, 0) b (2 1--4- , 1) y' = 4
x'
x
a y y x
y = –x
P P'
P = P' 1
c y d y
0 (3, 0) x 0 1 2 x
y = x2
y = 2x + 5

y' = (x' –3y')2 y' = 2–7 x' + 5–7


c (−3 --34- , 1) d (2 --12- , −2)
x x
a y y

P P' Chapter review


1
x 0 x
1 Translations, rotations and reflections
–4–3–2–1 0 3 2 a (−1, 3) b (2, 4) c (3, 1)
–2
P' P 3 Check with your teacher.
4 y′ = 2x′2 − 8x′ + 7
5 A′(3, 0) C′(12, −3)

– 2 1--3- 2 2--3-
e (−3 1--4- , −5) f (−1 1--4- , −5) 6
a – 2 2--3- 2 1--3-
y y
7 y′ = −2x′ + 3 y
y' = –2x' + 3
0 x x
–2 –1 0
x

P' P
–5
P' P
–5 y = –x + 1 6E

6G
answers 610 Answers

8 a (−2, 1) b ( – 3--2- , − 3---------


2
3
-) 2 a s+t b s + t + u + v c –s – t
˜ ˜ ˜ ˜ ˜ ˜ ˜ ˜
c ( 5---------
2
- , − 3---------
2
- ) d –v – u – t e –u – t – s
2 2 ˜ ˜ ˜ ˜ ˜ ˜
3 C
5
d (1 + ---
2
3 , − 3 + 2 --12- ) 4 a A to C b D to B c B to D d A to C
x′
5 D
9 y′ = ----
2
+1 y 6 a r+s b s+t c r–s
y' = 1–2 x' + 1
˜ ˜ ˜ ˜ ˜ ˜
d r+s e t–s f s+t–r
˜ ˜ ˜ ˜ ˜ ˜ ˜
g r+s+t h –s–t
x 7 a, b˜ ˜ ˜ ˜ ˜
y = –2x + 2 c 500 km
d 53.1° clockwise from N Flight
10 a (−3, −1) b (2, 2) path
8 512.1 km; find bearing using
c (− 1--2- + 3, 3
-------
2
+ 1) d (−2, 3) trigonometry
9 721.1 km, 326.3° (clockwise from N)
11 y′ = − 1--3- x′ y 10 Each part of answer has h
15
coordinate labelled a, b, . . . j. a f
b
The original vectors a and b 5
i a
~
x are also drawn. ˜ ˜ c ~b
y' = – 1–3 x' 11 Magnitude = 10.77, direction –15 g –5–5 d 5 15
y = –3x 68.2° True. j e
12–18 Check with your teacher. –15
12 a (2, 2) b (1, 2)
c (−16, 4) d (0, 2) 19 One can deduce that x and y
components can be added/subtracted/multiplied
13 y′ = −x′ separately.
14 a (3, 2 1--2- ) b (2,1)
20 B 21 D 22 0
15 y′ = 2x′ + 2x′ 2 23 Displacement, velocity, force ˜
24 Speed, time, length
Modelling and problem solving 25 1 magnitude and 2 angles

1 Rotation of 90°: 0 – 1 Exercise 7B — Position vectors in two


1 0 and three dimensions
1
1 a 3, 4, −2 b 6, 0, −3 c 3.4, 2, ---
or reflection in y-axis: – 1 0 2
0 1 2 a i 72 ii 45°
b i 65 ii 119.7°
or translation 4 left: – 4 c i 4.88 ii 225.8°
0
d i 320.16 ii 358.2°
2 Dilation of 3 about the origin followed by reflection
3 a 045° b 330.3° c 224.2° d 091.8°
4 –50i – 50 3 j 5 C
in the y-axis: – 3 0 ˜ ˜
0 3 6 248.9i – 383.3 j 7 –60.6i + 109.3 j
˜ ˜
8 615 km at 49.8°˜ south of east ˜
9 36 steps 11.3° south of east
CHAPTER 7 Introduction to vectors 10 20.8 km
Exercise 7A — Vectors and scalars 11 a 3
--- i

+ 4--5- j b 3--5- i – 4--5- j c 4--5- i + 3--5- j
˜ ˜ ˜ ˜ ˜
1 a i ~s r +~s ii
~ ~r 4 3 1
d – --5- i + --5- j e ------- i + ------- j2
f –0.792i + 0.611 j
~s
–s
~ ~s
˜ ˜ 3˜ 3
˜ ˜ ˜
~r
~r – ~s 12 B 13 Check with your teacher.
iii –r
~ 14 –0.98i – 0.20 j 15 – 1--2- i – ------
3
-j
~s ˜ ˜ ˜ 2˜
~s – ~r
16 Check with your teacher.
17 a i 4i – 7 j ii 65
b i Same as 1 a i except scaled by a factor of 2. ˜ ˜
ii Same as 1 a ii except scaled by a factor of 2. b i 3i + j ii 10
˜ ˜
iii –4r
c i –4i + 7 j ii 65
3s~
~ ˜ ˜
d i –3i – j ii 10
˜ ˜
~s
3s~ – 4r
~ e i 2i ii 2
˜
f i – 4i ii 4
~r ˜
611

answers
Answers

18 a –4i + 7 j b –3i – j c 4i – 7 j History of mathematics


˜ ˜ ˜ ˜ ˜ ˜
d 3i + j e – 2i f 4i 1 Mathematics lecturer
˜ ˜ ˜ ˜ 2 Alice’s Adventures in Wonderland and Through the
4 7 3 1
- i – ---------- j
19 a --------- b ---------- i + ---------- j Looking-Glass
65 ˜ 65 10 ˜ 10
˜ ˜
4 7 3 1
3 The daughter of the Dean of his college
c – ---------- i + ----------j d – ---------- i – --------- -j
65 ˜ 65
˜ 10 ˜ 10
˜ 4 12 cats
e i f –i
˜ ˜ Exercise 7D — Resolving vectors —
20 a i 29 ii 13
scalar and vector resolutes
2 5 2 3
iii ---------- i
– ---------
-j - i + ---------- j
iv – --------- 23 13 23 41
29 29 ˜
˜ 13 ˜ 13
˜ 1 a i ----------------
13
ii ----------------
41
v 3i + j vi 10 17 29 17 10
˜ ˜ b i ----------------
29
ii ----------------
10
b Reject, because magnitudes different.
21 a c i – 13 10
---------------- ii – 26 17
----------------
i 5 ii 26 10 17

iii – --35- i + --45- j 5 1


- i – ---------- j
iv --------- d i – 6------------
13
- ii – 6--5-
26 ˜ 26 13
˜ ˜ ˜
v 2i + 3 j e i – 23
------ ii – 23 26
----------------
˜ vi 13 5 13
˜
b Reject, because magnitudes different. 2 a i 1
---------- ii v || = ----- 3 1 17 51
- i – ------ j iii v ⊥ = ------ i + ------ j
10 ˜ 10 ˜ 10 ˜ 10 ˜ 10
22 a 2 b 58 ˜ ˜
82
b i ---------- ii v || = 8i + 10 j iii v ⊥ = 0
23 a 3i b 5j c 3i + 5 j 41 ˜ ˜ ˜ ˜ ˜
˜ ˜ ˜ ˜ c i 0 ii v || = 0 iii v ⊥ = – 3 i + 4 j
d 031.0° e 34 km /h ˜ ˜ ˜ ˜ ˜
2 2 2 2
24 329.0° d i ------- ii v || = 3 i + 3 j + 3 k
--
- --
- --
-
3 ˜ ˜ ˜ ˜
25 a 5 2 b 5 2 c 3.64 iii v ⊥ = 4 1
--- i + --- j – --- k
5
˜ 3˜ 3 3˜
˜
d 11 e 7 14 f 3 21
e i – ---------
- ii v || = – 21 ------ ( 2i + 3 j + 4k )
29
29 ˜ ˜ ˜
26 a 35 b 13 c 2 6 d 62 ˜
4
iii v ⊥ = ------ ( 25i – 6 j – 8k )
29
˜ ˜ ˜ ˜
27 CD = i – 7 j – 2k , EF = 2i – 14 j – 4k 5 15 5 5
˜ ˜ ˜ ˜ ˜ ˜ f i ---------
- ii v || = ----- - i + ------ j – ------ k
11 ˜ 11 ˜ 11 11 ˜
28 a 20i , –15 j , 20i + 15 j b 25 c 053.1° ˜
˜ ˜ ˜ ˜ iii v ⊥ = – 15 17 28
------ i + ------ j – ------ k
11 ˜ 11 11 ˜
29 53.1°, −36.9° ˜ ˜
3 a 3.6 km b 0.2 km or 200 metres
Difference = 90°
4 316 metres
30 a –4i + 12 j
˜ ˜
b 341.6° Exercise 7E — Time-varying vectors
c 0.0417 h or 2.5 minutes
x
1 a y = – --- b y = –3x – 3
2
Exercise 7C — Multiplying two vectors
x3
— the dot product c y = 4( x – 3 )2 d y = -----
8
1 23.99
2 B
2 Dot product = 24; more accurate, since no angle
3 a y = 4x2 + 2x b y
needed
3 a 45 b 12 c −36 d −26
e 1 f −20 g 0 h 0
4 E 5 C 6 −36 8 12
0 x
9 and 10 Check with your teacher.
11 D 12 B 13 D
4 a y = x2 − 4x + 3 b y
14 a −12 b 2 c 7 d −25
15 a 107° b 87° c 81° d 109° Note: x ≥ 1
16 D 17 E 18 a = 1 0
19 a = − 3--5- 20 4i + 8 j
˜ ˜
21 645˜
48
------ i – ------ j
5
–1 1 3 x
(2, –1) 7A
˜

7E
answers 612 Answers

5 a x2 + y2 = 1 b y c Period = π 18 D
1 2 y
19 y = ----2- – 2 , hyperbolic
x
–1 0 1 x
1
0 x
–1 –2
y
6 a x2 + y2 = 9 b 3 c Period = π Modelling and problem solving
1 a 6i – 12 j + 12k b 1--3- ( i – 2 j + 2k )
˜ ˜ ˜ ˜ ˜ ˜
–3 0 3 x
m
c 2i + 8 j + k + ---- ( i – 2 j + 2k )
–3 ˜ ˜ ˜ 3 ˜ ˜ ˜
1
2 2 d --- ( 10i
+ 16 j + 11k )
7 a (x − 1) + (y + 2) = 1 b y 3 ˜ ˜
˜
e 7.28 km f 1080 km/h g 720 km/h
0 1 2 x
–1 2 a (3, 5.5, 0) b – 3i + 4k
˜ ˜
–2 d 66 cm3 = 0.066 litres
–3
e 3i + 5.5 j + 4k f 7.43 cm g 84.5°
c Period = 2π ˜ ˜ ˜
x2 y 3 a y
8 ----- + y 2 = 1 X Y(7, 7)
9 1

–3 0 3 x 2i~ + 7j
~
–1

x2 y2 y
9 ----- + ------ = 1 4 O 5i~ Z x
4 16
b 2i + 7 j, – 5i c 7i + 7 j, –3i + 7 j
˜ ˜ ˜ ˜ ˜ ˜ ˜
–2 0 2 x
d 2------------
29
29
- e 74.1° f 2i
˜
–4 g (−2, 7) h 35 square units
10 a y = 1 – x ; u ( 0 ) = i ; t = 2, u = 1--2- i + 1--2- j 4 a
5
N
˜ ˜ ˜ ˜ ˜
b y = (x – 1)2, x ≥ 2
3
( x – 3 )2 ( y + 1 )2
c ------------------- + ------------------- = 1
4 9 b 5.83 m/s c 059° d 360 m e 600 m
11 x = 19
------ , y = ---
2 f Same result, except bearing = 180° − 059° = 121°
7 7
5 a 14.34 i + 20.48 j + 8 k b 26.25 m c 21.3°
12 y = x2; No, since v is always ‘ahead’ of u . ˜ ˜ ˜
˜ ˜ 1
6 a −3 i − 9 j − 8 k b ------------- (−3 i − 9 j − 8 k )
Chapter review ˜ ˜ ˜ 154 ˜ ˜ ˜
1 A 2 200 3i + 200 j + 40k c 5 2 km d 744.6 km/h
˜ ˜ ˜
3 B 4 A
5 a ( 5 + 5 2 )i + ( 5 + 5 2 ) j b 17.07 km
6 D
˜
7 E ˜8 C 9 D
CHAPTER 8 Vector applications
10 E Exercise 8A — Force diagrams and the
11 1.3909 triangle of forces
12 a – i – 4j b 7i – 6 j
˜ ˜ 1 a b
˜ ˜ ~N D
~
3 5
c −17 - i – ---------- j
d ---------
34 ˜ 34
˜ Book Ball
e 135°
13 56.1°, 111.8°, 42.0° W W
~ ~
9 ± 69
14 ------------------- c d
2 ~N ~N
15 B 16 B
2
17 a --------- 3 1
- i + ---------- j – ---------- k ~F Car ~A F
~
A
Boat ~
14 ˜ 14 14 ˜
˜
b v || = --2- ( 2i + 3 j – k ) , v ⊥ = – 1--2- ( j + 3k )
1
W
~
W
~
˜ ˜ ˜ ˜ ˜ ˜ ˜
613

answers
Answers

e N f N 5 a b 41.4° c 245 N
~
~ ~Tleft ~Tright d –278 i + 245 j N;
~F ~A Speaker 278 i ˜+ 245 j ˜ N
Sliding object Accelerating car ˜ N f˜ 8.0 m
e 370.6
W
~ W
~
W
~ 6 a ~N
g ~N
h ~N
~F
~F ~A = 120 N at ~F
Body at rest 40° to
Sliding vertical
body W
W
~ W ~ = 40g
~
~
i j b Av = 91.9 N down
Ball moving ~D
up AH = 77.1 N left
Ball moving
~D down c R = ( F friction – 77.1 )i + ( N – 483.9 ) j = 0
W
˜ ˜ ˜
~ with all forces in N.
W
~
d 77.1 N e 483.9 N
2 a C b E 7 a ~N b 0i + 0 j
3 a C b E T ˜ ˜
~ c – 7.4 i – 12.7 j N
4 a i 9i + 4 j ii –6i – 3 j iii ( 2 2 – 3 5 ) i – j Mass ˜ ˜
˜ ˜ ˜ ˜ ˜ ˜ d 7.4 N
W
b i 97 ii 3 5 iii 54 – 12 10 ~ e 12.7 N
c i – 9 i – 4 j ii 6i + 3 j iii ( 3 5 – 2 2 ) i + j 8 a N b – 7.4i – 12.7 j Ν
˜ ˜ ˜ ˜ ˜ ˜ ~ ˜ ˜
5 a i i + 2j ii ( 2 – 1 ) i Mass H c 7.4i – 4.3 j N
˜ ˜ ˜ ~ ˜ ˜
b i 5 ii 2 – 1 d 17 j N
c i 63.4° ii 0° W ˜
~
d i – i – 2j ii ( 1 – 2 ) i
˜ ˜ ˜ 9 0.77
6 a i 61.4 N at N9.4°W ii 61.4 N at S9.4°E 10 a 7600 N b 18 620 N c 0.41
b i 50i – 5 j ii – 50 i + 5 j 11 0.07
˜ ˜ ˜ ˜
c i – 4 i + 12 j ii 4i – 12 j 12 a 58.8 N b 45.04 N c 37.8 N d 0.84
˜ ˜ ˜ 13 0.7
d i 2.9 N at N26.9°W ii 2.9 N at˜ S26.9°E
14 9.6 N
7 a 10 N b 120°
8 79.2° Exercise 8C — Momentum
9 a 77° b 37.6 N c Z = 2 sin 60° × X 1 a 40 N s east b 600 N s south
˜ ˜
c 7.5 N s north d 32 500 N s west
History of mathematics e 978 N s north f 113 333 N s north
1 Bubonic plague broke out and the university closed. 2 a 80 m/s b 37 500 m/s c 20 m/s
2 Leibniz 3 a 50 kg b 1980 kg
3 The laws of mechanics and gravitation 4 a 38 N s east b 22 N s east
4 He reformed the coinage and introduced measures to c 31 N s 14.9° N of E
prevent counterfeiting. d 37.1 N s 6.2° N of E
e 25 N s 13° N of E
Exercise 8B — Newton’s First Law 5 2700 N s
of Motion 6 a 1000 kg m/s b −500 kg m/s
1 a 23.5 N b 8.6 N c 23.5 N 7 a −40 kg m/s b 48 kg m/s
c 9500 kg m/s d −3556 kg m/s
2 a D b B c C d A 8 5.3 m/s at 44° to Alan’s current direction.
3 a Drag force b 0N 9 a 6.6 m/s N b 6 m/s N
due to Tugboat
T c 39 392.3 N c 3.4 m/s S
water ~
F 10 3.7 m/s at 58.6° to Alan’s current direction.
~ Ship T
~ 11 a 2.2 at 47.9° b 5.9 at 71.4°
Tugboat c 1.67 at 65.4° d 2.4 at 234°
4 a 60° 30° b T1 = 4g N; 12 a 4.5 N s b 3 m/s
~T2 ~T1
j
~
T2 = 4 3 g N Exercise 8D — Relative velocity
1 16.5 km/h at 14° S of E
~i 8g N
~ 2 301.5 km/h at 5.7° E of N 7E

8D
answers 614 Answers

3 17.3 m/s at 52.1° N of W CHAPTER 9 Sequences and


4 72.5°
5 13 km/h
series
6 a 44.7 m/s at 26.6° W of N Exercise 9A — Arithmetic sequences
b 5 m/s N 1 a, c, d, g, h
c 17.8 m/s at 7.5° W of N 2 a a = 2, d = 5 c a = 0, d = 100
d 32.95 m/s at 4.9° S of E
d a = −123, d = 100 g a = 1--2- , d = 1
e 7.7 m/s at 52° S of E
7 15 km/h W h a = 1--4- , d = 2--4-
8 19.4 km/h 78° W of S 3 a 122 b 2900 c −219
9 68.3 km/s at 2.5° with the vertical. 4 a 103 b 1777 c −60
d −217 e −152
Exercise 8E — Using vectors in geometry 5 a 820 b 270 c 667
1–3 Check with your teacher. 6 A 7 B 8 B 9 B
4 a −v 10 a, d, e
˜
b u +v 11 a a = 2, d = 2 d a = not specified, d = 2
˜ ˜
c u −v e a = 8, d = 8
˜ ˜ 12 a 7th b 7th c 8th d 5th
d b .b = u2 + v2
˜ ˜ ˜ ˜ 13 tn = 13 + 10n
c .c = u2 + v2 14 tn = 37 + 3n
˜ ˜ ˜ ˜
5–12 Check with your teacher. 15 a tn = 8.5 + 3.5n b 358.5 metres
16 a 56 b 2, 5, 8
Chapter review 17 a 170 b −4, 2, 8
1 B 2 A 3 B 18 $9375 19 $414 000 20 363.8 cm
4 89.4 N at 153.4° to f 1 21 1135
5 138.6° ˜ 22 a $10 600 b $136 000
6 b 26.3 N, 52.6 N c 26.6°, 63.4°
7 17.7 N Exercise 9B — Geometric sequences
8 D 9B 10 C 11 D
1 b, c, e, f, h, i, j
12 3 N s away from the wall
2 b a = 1, r = 4 c a = −1, r = 2
13 6.0 m/s at 43.7° with A’s original motion
e a = 4, r = −3 f a = −6, r = −10
14 B 15 C
h a = 1.2, r = 2 i a = 7, r = 1--2-
16 12.1 km/h at 65.6° to the bank 1 1
j a = --2- , r = --2-
17 8.7 km/h from the north
18 D 3 a 20 155 392 b 1 048 576 c 295 245
d 11.867 494 81 e −131 072
Modelling and problem solving
4 a 39 366 b 6144 c 32 768
1 7 minutes 46 seconds
2 60 m d −32 768 e −12 582 912 f −708 588
3 a 2.96 N down b 1.96 N down c 0.96 N down 5 a 12th b 13th c 9th d 16th
4 a 0.038 b 193.5 m 6 a 531 440 b 686 285 c 36 043.7
5 a Check with your teacher. b 109.5° d 274 576.3
1 1 5 2 7 D 8 D 9 B 10 A
c i p= ------- ; r = − ------
- ii ---------- units 11 a 1, 2, 4, 8, 16 b 2 c 2048
2 2 3
12 20 million, 10 million, 5 million, 2 1--2- million,
6 a
y
1 --14- million, 625 000, 312 500
X (3, 5) Y (9, 5) 13 a tn = 2n − 1 b 2048
14 a tn = 10 000 × 0.85n − 1 b $2724.91
15 a tn = 6n − 1 b 1296 c 6
n−1
Z (6, 0) 16 a tn = 60 × 1.08 b 88.16 m c 8th year
O x 17 Check with your teacher.
b YX = −6 i ; ZY = 3 i + 5 j
˜ ˜ ˜ Exercise 9C — Applications of
c OY = 9 i + 5 j ; ZX = −3 i + 5 j geometric sequences
˜ ˜ ˜ ˜
d 91.9° e 29.1° f 5j 1 a 5.67 b 17th year c 12.7 tonnes
˜ 2 a $2146.53 b Year 9 c $5646.65
g (12 3--8- , 10 5--8- ) h 31 7--8- square units
3 Pn = 10 000 × (0.9)n − 1
615

answers
Answers

4 a $19 317.32 b $6317.32 7 a tn


5 30
a 14 147 b 16th year
25
6 a 123 b 10 weeks
20
7 a 2.85 m b Year 11
15
8 a 0.96 m b 32nd beam 10
9 C 5
10 a $400 640.74 b Year 9 c 2 636 196.56
11 a 55 773 b 9th year 0 1 2 3 4 5 6 7 8 n
12 a $16 105.10 b $16 288.95
c $16 386.16 d $16 453.09 b tn
13 a $22 076.26 b $26 897.78 6

c $32 772.33 d $53 701.28 5

14 $8376.76 15 3 years 16 2 years 4

17 3 years 18 10 months 19 2 years 3


2
Exercise 9D — Finding the sum of an 1
infinite geometric sequence 0 1 2 3 4 5 6 7 8 n
3
1 a 100 b 100 c 10 d ---
2
e 5
--- f 3 1--3- g 2.5 h −7.5 c tn
4
15
i 200 5
j –45 -----
-
11 10
2 a 10, 6, 3.6 b 40, 8, 1.6 5

c 6, 1.5, 0.375 d 12, 10.8, 9.72 0 n


1 2 3 4 5 6 7 8
e 4, −0.8, 0.16 f 6, −3, 1.5 –5

g 9, −7.2, 5.76 h −6, −1.2, −0.24 –10


i −24, −14.4, −8.64 j −15, 4.5, −1.35 –15
3 a 12.5, 6.25, 3.125 b 12.5, 9.375, 7.031 25
c 6, 1.5, 0.375 d 48, 28.8, 17.28 d tn
40
e 2 4--9- , 22
------
27
, 22
------
81
35
30
5 8 4
4 a ---
9
b ---
9
c ---
9
d 1 1--3- 25
20
e 3 7--9- f 8 2--3- g 14
------
99
h 57
------
99
15
10
i 262
--------- j 1 321
--------- 5
495 999 0 n
–5 1 2 3 4 5 6 7 8
5 No — falls short by 3 1--3- metres 6 192.5 m
7 Yes
8 $500 000 e tn
9 25 mm too much 8 × 104
10 a 23
------
25
6 × 104
4 × 104
b $1 000 000, $920 000, $846 400, $778 688,
2 × 104
$716 392.96
c $7 070 144.32 0 1 2 3 4 5 6 7 8 n

Exercise 9E — Contrasting arithmetic f tn


6000
and geometric sequences through
5000
graphs
4000
1 Arithmetic sequence with a = 0 and d = 1
2 Arithmetic sequence with a = 10 and d = −2 3000
3 Geometric sequence with a = 10 and r = 1.5 2000
4 Geometric sequence with a = 20 and r = 0.5 1000
5 Arithmetic sequence with a = 4 and d = −0.5
6 Geometric sequence with a = 100 and r = 0.8
0 1 2 3 4 5 6 7 8 n 8D

9E
answers 616 Answers

8 D 30
9 E 80
10 a $5500, $6000, $6500 70
b $5500, $6050, $6655 60

Amount ($)
c 50
6500
Amount ($)

40
6000
Legend 30
5500 Legend
Un 20
Vn Vn = 10 × 2n – 1
5000
10 Un = 10n
1 2 3
1 2 3 4
Year
Term number (n)
11
160 000 Modelling and problem solving
150 000 1 a 18.5 tonnes
Amount ($)

140 000 b tn = 4.5 + 3.5n


130 000 Legend
c 214.5 tonnes
120 000 Un d 28th month
110 000
Vn e 40 months
2 a 1.1
1 2 3 b tn = 10 × 1.1n − 1
Year c 14.641 tonnes
12 d 13th month
18 000 e 213.84 tonnes
17 000 3 24th month
16 000 4 35 years
Amount ($)

15 000
14 000 Legend
Un
CHAPTER 10 Permutations and
13 000
12 000
Vn combinations
11 000 Exercise 10A — The addition and
multiplication principles
1 2 3
Year 1 a AB BA CA
13 3 terms AC BC CB
14 0 terms b 6
15 3 terms 2 BG GB YB RB
16 4 terms BY GY YG RG
BR GR YR RY
Chapter review 3 ACB BAC CAB
1 C 2 B ABC BCA CBA
3 a Yes, a = −123, d = 100 b Yes, a = −5 1--4- , d = 3 4 a 42 b 210 c 840 d 2520
4 A 5 C 6 B 5 a 24 b 6 c 12 d 24 e 1320
7 Term number 35
8 a 1080 b 12 000 6 a 49 b 252 c 16
9 A 10 C 11 B 12 A 7 a i 72 ii 72
13 a Yes, a = 5, r = 1--2- b No b 144
14 B 15 D 16 B 8 126
17 a tn = 7.2 × 1.15n − 1 b 19.2 tonnes 9 C
c Year 12 10 C
18 B 19 C 20 17 11 D
21 A 22 C 12 100
23 a $33 622.22 b 5 years 13 6
24 E 14 48
25 D 15 256
26 3 7--9- 16 1296
27 25 cm 17 1080
28 D 18 48
29 A 19 a 1000 b 27
617

answers
Answers

c 271 371 471 Exercise 10C — Arrangements


272 372 472 involving restrictions and like objects
273 373 473 1 360
281 381 481 2 83 160
282 382 482 3 10
283 383 483 4 1260
291 391 491 5 27 720
292 392 492 6 1 307 504
293 393 493
7 a 5.4 × 1010 b 3.6 × 109 c 4.0 × 1010
20 a 200 b 40 c 50 d 290 8 a 120 b 20 c 60
21 a 13 230
9 a 30 240 b 3024 c 6720
b 17 640
Jack may wear 13 230 outfits with a jacket or 4410 10 a 120 b 48 c 72
outfits without a jacket. Therefore he has a total of 11 a 1680 b 180 c 360
17 640 outfits to choose from. The assumption made 12 a 1320 b 110
with this problem is that no item of clothing is 13 B 14 D
exactly the same; that is, none of the 7 shirts is 15 a 720 b 24 c OYSTER
exactly the same.
16 a 36530 b 365P30

Exercise 10D — Combinations


Exercise 10B — Factorials and
8P 19 P 1P 5P
permutations 3
1 a --------- b ----------2- c --------1- d --------0-
1 a 4×3×2×1 3! 2! 1! 0!
b 5×4×3×2×1 2 a 8C2 b 9C3 c 8C0 d 10C4
c 6×5×4×3×2×1
3 a 1 b 20 c 120 d 220
d 7×6×5×4×3×2×1
4 1365
2 a 24 b 120 c 720 5 252
d 3 628 800 e 8.717 829 12 × 1010 f 362 880 6 a 495 b 11 c 1 d 1
g 5040 h 6
e 54 264 f 120 g 100 h 680
3 a 3024 b 151 200 c 840 d 720
7 a 2 598 960 b 65 780 c 65 780 d 2 467 400
4 a n(n − 1)(n − 2)(n − 3)(n − 4)
b (n + 3)(n + 2) 8 560
1 9 100
c ------------------------------------- 10 59 400
n(n – 1)(n – 2)
11 a 28 b 120
1
d ------------------------------------------------------- 12 201 376
( n + 2 ) ( n + 1 )n ( n – 1 )
13 D
5 a 8 × 7 = 56
b 7 × 6 × 5 × 4 × 3 = 2520 14 C
c 8 × 7 × 6 × 5 × 4 × 3 × 2 = 40 320 15 a 120 b 10 days
9! 5! 18! 16 a 57 b 4 days 6 hours
6 a ----- = 60 480 b ----- = 20 c -------- = 1 028 160 17 a 15 b 1 day 4 hours
3! 3! 13!
7 a 27 907 200 b 639 200 c 1 028 160 18 a 8 145060 b 11 480 c 820
8 56 19 a i 220, 220 ii 6435, 6435 iii 10, 10
9 3024 iv 56, 56 v 1, 1
10 2184 b The value of nCr is the same as nCn − r .
11 358 800
12 120 Exercise 10E — Applications of
13 8190 permutations and combinations
14 3360 1 a 45 b 120 c 120 d 210 e 105
15 362 880 2 a 720 b 252 c 10 d 120 e 2.4 × 1018
16 479 001 600 3 24
17 D 4 24
18 E 5 376 992
5 5 6
19 a 5P3 = 60 b P4 = 120 c P5 = 120 a 210 b 126 c 84 d 140
7 7 7 7 a 126 b 56 c 21 d 70
20 a P3 = 210 b P4 = 840 c P7 = 5040
6P
8 a 8008 b 5005 c 5005 d 4004
6 6 9 D
21 a P6 = 720 b --------- = 360
2 10 D 9E

10E
answers 618 Answers

11 a 28 Chapter review
b 9 17 27 41 12 14, 9 17 27 41 12 37, 1 C 2 B 3 A
9 17 27 41 12 34 4 a 180 b 648
c 6 5 D 6 A 7 C
12 a 84 2000
b 7 15 25 32 10 12, 7 15 25 32 10 35, 8 P2, 19P6, 12P9 9 A 10 12 11 E
7 15 25 32 10 37 12 B 13 19C6, 22C15, 2000C2 14 175
c 10 15 a 325 b 6 c 676
13 a 38 760 b 1140 c 34 220 d 39 010 800 16 C 17 756 756 18 B 19 D
14 a 1 20 a 210 b 10C0 10C1 10C2 10C3 10C4 . . . 10C10 c 1024
b i 38 760 ii 50 063 860
21 a 405x8 b 3240x7 c 196 830x
iii 1 940 475 213 600 iv 4 191 844 505 805 495
Modelling and problem solving
Exercise 10F — Pascal’s triangle, the
1 50 878 2 2 944 656
binomial theorem and the pigeonhole 3 a 2 598 960 b 1287
principle c 65 780 d 1584
1 4 a 6 096 454 b 2 760 681
Row c 442 890 d 2 048 200
0 1 e 850 668 f 5 245 786
1 1 1 g 18 360
2 1 2 1 5 a 120 b 26
3 1 3 3 1 6 a 12 000 b 5 years
4 1 4 6 4 1
5 1 5 10 10 5 1
6 1 6 15 20 15 6 1 CHAPTER 11 Dynamics
7 1 7 21 35 35 21 7 1
8 1 8 28 56 70 56 28 8 1 Exercise 11A — Displacement, velocity
2 a 70 and acceleration
b 1 9 36 84 126 126 84 36 9 1 1 a v (t) = 6 i + (3 − 14t) j , a (t) = −14 j
c 45 ˜ ˜ ˜ ˜ ˜
3 a 8 b r (0) = 0 m, v(0) = 3 5 m/s
b 220 ˜ ˜
10 c i r (2) = 12 i − 22 j m
c C0 10C1 10C2 10C3 10C4 10C5 10C6 10C7 10C8 10C9 ˜ ˜
ii 25.7 m/s downwards˜ at an angle of 13°30′ to
10
C10 the vertical
1 10 45 120 210 252 210 120 45 10 1
iii 15°7′
4 a x2 + 2xy + y2
d Yes, 14 m/s2 downwards
b n3 + 3n2m + 3nm2 + m3
2 a, b Check with your teacher.
c a4 + 12a3 + 54a2 + 108a + 81 2
1
c y = ------ (1225 − x )
5 a 80x b 56p5q3 c 4608x2 25
6 B 7 D 3 a i v (t) = 2 i − j , a (t) = 0
8 a i 1 ii 2 iii 4 ˜ ˜ ˜ ˜ ˜
iv 8 v 16 vi 32 ii r (0) = 5 j m, v(0) = 5 m/s
˜ ˜
b i The sum of the elements in each row of Pascal’s iii y = 5 − 1--2- x
triangle is a power of 2:
Row Sum b i v (t) = i + (6 − 2t) j , a (t) = −2 j
˜ ˜ ˜ ˜ ˜
0 20 = 1
ii r (0) = 0 m, v(0) = 37 m/s
1 21 = 2 ˜ ˜
2 22 = 4 iii y = 6x − x2
3 23 = 8 c i v (t) = − i − 6t j , a (t) = −6 j
4 24 = 16 ˜ ˜ ˜ ˜ ˜
5 25 = 32 ii r (0) = 10 j m, v(0) = 1 m/s
˜ ˜
ii ‘The sum of the elements in the nth row of iii y = 10 − 3x2
Pascal’s triangle is 2n.’
d i v (t) = 3 i + (5 − 4t) j , a (t) = −4 j
9 26 = 64 13 26 14 13 ˜ ˜ ˜ ˜ ˜
ii r (0) = 0 m, v(0) = 34 m/s
History of mathematics ˜ ˜
1 16 iii y = --9x- (15 − 2x)
2 Calculating machine e i v (t) = i + 6t j , a (t) = 6 j
3 Puy de Dôme; his brother ˜ ˜ ˜ ˜ ˜
4 Probability ii r (0) = 4 i m, v(0) = 1 m/s
˜ ˜ 2
5 Brain tumour iii y = 3(x − 4)
619

answers
Answers

f i v (t) = 2t i − j , a (t) = 2 i d v (t) = (t2 − 1) i + (−3t − 1) j ,


˜ ˜ ˜ ˜ ˜ ˜ ˜ ˜
r (t) = ( --13- t3 − t + 1) i + (− --32- t2 − t + 1) j
ii r (0) = 0 m, v(0) = 1 m/s ˜ ˜ ˜
˜ ˜ e v (t) = (−5t + 15 1 2 13
------ ) i + ( --- t + t − ------ ) j ,
iii y2 = x ˜ 2 ˜ 2 6
1˜ 2 13
g i v (t) = 2t i − 4 j , a (t) = 2 i r (t) = (− --52- t2 + ----- 15
2
1 3
- t + 3) i + ( --- t + --- t − ------ t + 4) j
6 2 6
˜ ˜ ˜ ˜ ˜ ˜ ˜ ˜
ii r (0) = 0 m, v(0) = 4 m/s f v (t) = (t + 7--2- ) i + (− 1--2- t2 + 6t − 65 ------ ) j ,
6
˜ ˜ ˜ ˜ ˜ 65-
iii y2 = 16x r (t) = ( --12- t2 + --72- t − 5) i + (− --16- t3 + 3t2 − ----- 6
t + 12) j
˜ ˜ ˜
2 23 5 2 5
h i v (t) = --------1
- i − 6t j , t ≠ 0; g v (t) = (t + t − -----6- ) i + ( --2- t − 3t + --3- ) j ,
˜ 2 t ˜ ˜ ˜ ˜23 3 2˜ 5
r (t) = ( --13- t3 + --12- t2 − ----- 6 ˜
5 3
- t) i + ( --- t − --- t + --- t + 8) j
6 2 3
1
a (t) = − ----------
- i − 6j, t ≠ 0 ˜ ˜
˜ 4t t ˜ ˜ h v (t) = (3t − 6) i + (− 5--2- t2 + 12t − 35 ------ ) j ,
2
˜ ˜ 11 2˜ 35
ii r (0) = 0 m, v(0) is undefined r (t) = ( --32- t2 − 6t − ----- 5 3 116
- ) i + (− --- t + 6t − ------ t + --------- ) j
˜ ˜ ˜ 2 ˜ 6 2 3
˜
iii y = −3x4 3 a v (t) = 12 i + (15 − gt) j
4 a t = 5 seconds b (−3, 18) ˜ ˜
b 19.2 m/s at an angle of 51°20′ above the ˜
c yA = (x − 1)2 + 2, yB = (x + 8.5)2 − 12.25 horizontal.
5 a v (t) = 2 i + 2(1 − t) j , a (t) = −2 j c 11.5 m 3.06 seconds
˜ ˜ ˜ ˜ ˜
b 2 2 m/s upwards at an angle of 45° to the d 19.2 m/s downwards at an angle of 51°20′ to the
vertical horizontal
4 a v (t) = 15 i + (30 − gt) j
c i 37 m ˜ ˜ an angle of˜63°26′ above the
b 33.5 m/s at
ii 2 5 m/s downwards at an angle of 26°34′ to horizontal
the vertical
c 45.9 m
iii 26°34′
d No, as it falls short by about 40 cm.
6 a, b Check with your teacher. c t = 5 seconds
7 a P and Q do not collide. c y = -----12
- − 1 5 b r (t) = 18 3 t i + (50 + 18t − --12- gt2) j
x ˜ ˜
8 a Check with your teacher. 1
b − --5- i + 3 j m/s ˜
˜ c Maximum height is 66.5 m which is 16.5 m
c v (5) = − -----2 4
- i + 3 j , a (5) = --------- i
˜
25 ˜ 125 ˜
above the launch platform.
˜ ˜ ˜
d Approaches zero d 47.7 m/s downwards at an angle of 49°12′ to the
9 a 2i + 3 j b 145 m/s c 85°14′ horizontal
˜ ˜ e 172.1 m
d i 65 m/s downwards at an angle of 7°08′ to
6 a r A(t) = 4(t − 1) i + (3 + 5t − t2) j ,
the vertical ˜ ˜ ˜
ii 4 i + 7 j r B(t) = 3(8 − t) i + (6 − 7t + 2t2) j
˜ ˜ ˜ ˜
10 a r 1(0) = 0 ,˜ r 2(0) = 80 i + a j b a = 15 b Particles don’t collide but they do pass through
˜ ˜ ˜ ˜ ˜
c (50, 6.25) d 100°18′ two common points at different times. At
11 a b = 25, c = −5 1
b y = − ----- 2 1
- (x − 50) + 26 --- (13.70, 5.55), one particle arrives when t = 4.43 s
80 4
c 81.4 m and the other when t = 3.43 s. At the point
12 d Rmax = 76 ----- 9
- m when α ≈ 63°26′ (18.20, −0.06), one particle arrives when
16
t = 5.55 s and the other when t = 1.93 s.
Exercise 11B — Projectile motion c yA = 3 + 1
------ (x + 4)(16 − x),
16
1 a r (t) = (t + 3) i + 2 j 1
˜ ˜ ˜ − 3t) j yB = 6 + --- (24 − x)(27 − 2x)
b r (t) = (t2 + 1) i + (1 9
˜ ˜ ˜ 7 a r λ(t) = (5t − 14) i + (32t − 45 − 4t2) j
c r (t) = 2(2t − 1) i + --12- (7t2 − 9) j ˜
b When t = 5 s, the˜ particles do collide˜at (11, 15).
˜ ˜ ˜
d r (t) = (3t + 1--2- t − 7) i + (−2t+ 9) j Particles appear to collide at (9.15, 17.4) but
˜ ˜ ˜ actually pass through this point at different times;
e r (t) = ( 3--2- t2 − 1) i + (4t + 1--2- t2 + 1) j
˜ ˜ ˜ so they don’t collide. Particle λ at t = 4.63 s and
f r (t) = − --12- t(t − 4) i − --13- t(t2 + 18) j particle ξ at time t = 3.15 s.
˜ ˜ ˜
g r (t) = (15t) i + (10t − 3--2- t2 + 20) j 8 b 0.618 when θ ≈ 58°17′
˜ ˜ ˜ c 35.6 m/s
h r (t) = (−t2 + 3) i + (5t − --1t- − 11) j
˜ ˜ ˜ 9 θ ≈ 17°46′ and θ ≈ 77°56′
2 a v (t) = (2t + 1) i − 3 j , 10 21.429 m
˜ ˜
r (t) = (t2 + t − 1) i +˜ (−3t + 5) j 11 100 m/s, 192 m
˜ ˜ ˜
b v (t) = (3t + 2) i + (−5t − 1) j , 12 Yes, they meet at (38.4, 9.2) when t = 2 s.
˜ ˜
r (t) = ( 3--2- t2 + 2t) i + (− 5--2- t2 − ˜t) j 13 20 m/s at an angle of approx. 36°52′
˜ ˜ ˜ 14 75.431 m, approx. 4.64 s
c v (t) = (−t + 3) i + ( 1--2- t2 − 4) j ,
10E
2
˜ ˜ 15 b Rmax = V
when φ = 45° + θ
r (t) = (− 1--2- t2 + 3t + 2) i + ( 1--6-˜t3 − 4t − 3) j
-------------------------- ---
(1+sin θ )g 2

˜ ˜ ˜
11B
answers 620 Answers

Exercise 11C — Motion under constant 8 c y = 25 − (x − 1)2


acceleration 9 D 10 C 11 A
1 a 4 m/s2 b 32 m 12 v (t) = t i + (3t − t2) j m/s,
˜ ˜
r (t) = --12- t2 i + t---6- (9 −˜2t) j m
2
2 a 6.5 m/s2 b Approx. 1.54 s ˜ ˜ ˜
3 a 80 m/s b Approx. 8.94 s 13 a v (t) = 6 i + (45 − gt) j m/s
˜ ˜
4 a 18 m/s b −0.6 m/s2 b 3 229 m/s at an angle˜ of elevation of 82°24′
5 a 58.8 m/s b 176.4 m c 101.25, 4.5 s
6 a 19.6 m b 4s d 3 229 m/s downwards at an angle of 7˚36′ to the
7 a 2.83 s b 27.72 m/s vertical
8 a 78.4 m b 39.2 m/s 14 b r (t) = 12t 3 i + (60 + 12t − 1--2- gt2) j
9 a 90 m b 7.14 s c 42 m/s ˜
c Clears ˜ 5 cm
by about ˜
20 10 15 b Expression is positive for 26°34′ ≤ θ < 90°.
10 21.08 m/s or ----------------
3 Maximum is 0.618 when θ ≈ 58°17′.
11 a 2 m/s2 b 144 m c 7.5 s c 30.845 m/s
12 a 12.5 m/s b 4.8 s 16 a 30.625 m b t ≈ 1 s and 4 s
13 a −5 m/s2 b 22 m c 3.2 s d 25.6 m 17 E
14 12 m/s 18 C
15 a 24.5 m b 17.5 s 19 B
16 a 8.04 m/s b 1.1 m c 1.77 s 20 a 2 min 14 s b 16.2 s
17 a 16.1 m/s b 27.23 m 21 a v (m/s) b 36.25 s
18 a i −19.6 m/s ii −27.7 m/s iii t = 8 s 5 B c 145 m
b i −20.2 m/s ii −28.15 m/s iii t = 2.4 s 4 A d 16 m
19 a 78.4 m b 2.53 s longer
20 0.1044 s
0 _6 8.5 t (s)
21 4 s, 21.6 m above the ground
22 5.83 s
23 24.04 m Modelling and problem solving
24 8.9 s 1 a Car A b 8.4 m
Chapter review c i −2 m/s2 ii 3.58 m/s2
d i 23.62 m/s ii 28.33 m/s
1 B 2 A 3 D 4 B 2
5 C 2 a --xs = --14- u∆t
b x = --------
2
1
- + --- a(∆t)
8
s = --12- a(∆t)2 + u∆t
6 a v (t) = 6 i + (12 − 6t) j m/s, a (t) = −6 j m/s2 c When a = 0
˜ ˜ ˜ ˜ ˜
b Origin, 6 5 m/s 3 a k = 4.96 × 1012
c i 12 2 m ii 6 m/s to the right iii 45° × 10
12 6
b v(r) = 2  4.96
--------------------------- – 2.69 × 10
7 a 5 seconds b (5, −5) r 

c yA = 4x − x2, yB = x − 10 c 1.84 × 106 m


Index 621

Abel, Niels Henrik 183 in polar form 111


Abelian groups 185, 265 of two numbers 83–4
absolute value of a number 49–50 complex numbers
solving equations using 51–4 addition 81–2
acceleration applications 120–1
motion under 540–4 Argand diagrams 80–1
vector expressions for 514–21 argument 95–9, 100
addition principle 461–4 in Cartesian form 106–7
adjoint matrix 229 conjugates 87–8
algebraic structures 178 definition 76–9
applied forces 355 division 88–9
Argand diagrams 80–1, 97 equality of two numbers 84–5
argument of complex number 95, 97–103 geometrical representation 95–6
arithmetic sequences imaginary part 77
definition 396–8 modulus 96–7, 100
finding terms rule 398–9 multiplicative inverses 89–90
graphs of 432 Pascal’s Triangle 114–15
listing terms of 400 plotting 84
simple interest 433–7 polar form 103–6
sum of terms 401–3 powers 114–17
arrangements 462 in quadratic equations 80
in a circle 472–3 real part 77
involving restrictions and like objects 476–9 simple algebra 93–5
see also permutations subtraction 81–2
associativity of operations 181, 264 composition of transformation 263
augmented matrices 210 composition and transformations 192–3
axioms related to operations and whole numbers 188 compound interest 421–3
compared with simple interest 434–7
binary number systems 17 conformable matrix 137
binary operations 180 congruent transformations 270, 276
binomial theorem 499–501 conjugate pairs of surds
product of pair 34
Cartesian form of complex numbers 106 rationalising denominators using 44–7
Cayley, Arthur 194 conjugates of complex numbers 87–8
Cayley table 179–80, 186–8 conservation of momentum 375–8
circle, number of arrangements in 472–3 constant acceleration, motion under 540–4
cis θ 103 coterminal angles 96
closure of operations 181, 263 counting paths 498
coefficient of friction µ 369 Cramer’s rule 234–7
coefficient matrix 155 cryptography 199–200
cofactor matrix 228 cube roots 7
collision momentum 381 curves, images of 266–8
combinations 482–7 translation of 255
applications of 490–4 cyclic groups and subgroups 189–91
combinatorics 460
common differences 396 De Moivre, Abraham 108
common ratio 407 decimals, recurring 11–13, 429–30
commutativity of operations 182 determinants of matrices 148
complex conjugate numbers 87–8 2 × 2 and 3 × 3 matrices 222–3
complex number multiplication Cramer’s rule 234–7
by constant (scalar) 82 expansion of 225–6
by i 85 finding area of triangle 243
622 Index

determinants of matrices (continued) finding terms of 408–12


finding equation of line 243 graphs of 432
properties 224–5 in growth and decay 418–21
dilations 284–9 sum of terms 413
about origin 288–9 see also infinite geometric sequences
dilation factor 284 geometric transformations, and matrix algebra 250–6
parallel to x- and y-axes 285–8 geometry, vectors in 385–8
displacement, velocity and acceleration 514–21 golden ratio (golden mean) 444–5
Distributive Law 32–4 graphics calculator use
division addition of elements in dominance matrices 167–8
of complex numbers 88–9 combinations 486–7
of surds 36–8 comparison of simple and compound
Dodgson, Charles Lutwidge 331 interest 434–7
dominance matrices 164–8 complex numbers 93–5
dot product of two vectors complex numbers in Cartesian form 106–7
angle between vectors 327 complex numbers in polar form 105–6
calculation 324 compound interest 434–7
perpendicular and parallel vectors 327–8 dot product of two vectors 326
properties 324–5 factorials 468
and unit vectors 325 Gaussian elimination 217–21
using calculator 326 graphing original and translated image 256–8
DOTS (difference of two squares) identity 34 listing terms of arithmetic sequence 400
Mandelbrot Sets 447–9
equation solving matrix equations 156–8
using absolute values 51–4 matrix multiplication 162–4
using matrices 133–4 matrix operations 237–42
equilibrium state 364 modulus and argument of complex numbers 101–3
equilibrium values in input–output analysis 206–8 Pascal’s Triangle coefficients 115
Escher, Maurits Cornelius 290 permutations 472
plotting missile flight 533–5
factorials 467–70 plotting particle trajectory 517–19, 520–1
Fibonacci numbers 445 roots 7
Fibonacci Sequence 442–3 scalar and vector resolutes 334–6
generating terms in 443–4 simultaneous equations 242
golden ratio 444–5 sum of arithmetic sequence 402–3
field forces 354 terms in Fibonacci Sequence 443–4
FOIL method 32 terms of geometric sequence 411–12, 419–20
force diagrams 356–9 unit vector in direction of vector 316–17
force vector diagrams 364 vector functions of time 341–3
forces vector magnitude and direction in two
resolving into components 365–9 dimensions 311–12
resultant of 356 x- and y-components of vector 313–14
triangle of 359–60 gravity, acceleration under 540–4
types 354–5 group theory
friction force 369–70 applications 197–8
functions in groups 193–4 and linear transformations 263–8
groups
Gauss, Carl Friedrich 216 Abelian 185
Gaussian elimination cyclic 189–91
to find inverse of matrix 213–15 inner and outerfunctions 193–4
to solve simultaneous equations 209–13 permutations 191
using calculator 217–21 properties 184–8
geometric sequences terminology 180–3
common ratio 407 transformations of shapes 192–3
in compound interest 421–4, 433–7
definition 406–8 Hamilton, William Rowan 118
Index 623
idempotent matrices 149–50 information summarising using 158–9
identity element (IE) 181–3, 264 introduction 128–9
identity matrix, multiplicative 139–40 inverse of 147–9, 228–32
images, graphing using calculator 256–8 linear transformation 261
images of curves multiplication by scalar 131–3
non-singular transformations 266–7 multiplication of 137–40, 162–4
singular transformations 267–8 multiplicative inverse 146–7, 150–1
images as result of translation 251 powers of 143–4
imaginary number i 76 reflection 276
multiplication by 85 simple equation solving using 133–4
imaginary part of complex numbers 77 simultaneous equation solving using 155–8
inequations, solving 55–62 singular 149–50
inertial mass 364 subtraction of 131
infinite geometric sequences transition 159
converting recurring decimals to fractions 429–30 transpose of 154
sum of 427–9 and vectors 337–8
see also geometric sequences matrix algebra
input–output analysis 206–8 transformations 250–1
integers 4 translations 251–5
interest, simple and compound 421–4, 433–7 matrix applications
inverse input–output analysis 206–8
of 3 × 3 matrix 228–32 modelling and problem solving 247–8
of set elements 181–2 matrix equations 133–4, 150–2
inverse of matrix 147–9 matrix operations, using calculator 237–42
by Gaussian elimination 213–15 maze, paths through 498
inverse transformations 264–5 mediator 276
irrational numbers 4–5 missile flight 531–5
proof by contradiction 18–19 modular arithmetic 17
surds 17–18 modulo arithmetic 179–80
modulus
Leontief matrix 207 of complex number 96–7, 100–3
linear equation solving of a number 49–50
using Cramer’s rule 234–7 momentum
using determinants 222–37 conservation of 375–8
using inverse matrices 206–21 definition 374–5
linear transformation matrix 261 motion
linear transformations 259–62 Newton’s First Law of 364–70
and group theory 263–8 Newton’s Second Law of 524
Lotto systems 490–1, 493–4 of projectiles 514–35
under constant accleration 540–4
Mandelbrot Set 446–9 multiplication
mass and weight 364–5 of complex numbers 82–4, 85, 111–14
matrices of matrices 137–40, 150–1, 162–4
addition 130 of matrix by scalar 131–3
adjoint 229 of surds 27–30
applications 145–6 multiplication principle 460–4
augmented 210 multiplicative identity matrix 139–40
coefficient 155 multiplicative inverse 146–7
cofactor 228 of complex number 89–90
conformable 137
determinants of 148, 222–7 negative numbers, square root of 76
dilation 285 Newton, Isaac (Sir) 363
dominance 164–8 First Law of Motion 364–70
equality of 133 Second Law of Motion 524
Gaussian elimination in 209–15 Newtonian dynamics, assumptions made in 356
identity 139–40 nilpotent matrices 149–50
624 Index

non-singular transformations 266–7 reflection transformation 276


nth roots 7 reflections
number systems, other 16–17 in line y + x tan θ 280–1
real 3–5 in line y = x 279–80
with mediator not through origin 282–3
operations on numbers 180 in x-axis where y = 0 276, 277
operations and whole numbers, axioms relating to 188 in y-axis where x = 0 277, 278
relative velocity 382–4
particles, in Newtonian dynamics 355 resistive forces 355
Pascal, Blaize 506 resolution of forces 365–9
Pascal’s triangle 114, 499–501 resolution of vectors
Pauli matrices 198 scalar resolute 332–3, 334
perfect square identities 33 using calculator 334–6
permutations 191, 470–2 vector resolutes 333, 334
applications of 490–4 resultant force 356, 364
see also arrangements rewards allotment 440
pigeonhole principle 502–3 roots, square, cube and nth 7
π (pi) 5 rotations
approximations for 63 general 272–5
place value number system 16 special 270–1
polar form, of complex numbers 96–106
position vectors scalar quantities 300
direction 310 scalar resolutes 332–3
locating 317–18 sequences
magnitude 309–10 arithmetic 396–403
magnitude and direction using calculator 311–12 geometric 406–13
magnitude in three dimensions 318–19 Mandelbrot Set 446–9
relationship with matrices 309 set notation 7–8
resultant 314–15 shapes, changes in area and perimeter 441
in two and three dimensions 308–9 shears, definition 291–2
unit vectors 312–13, 315–17 parallel to x- and y-axes 292–5
x- and y-components using calculator 313–14 simultaneous equations
powers in input–output analysis 206–8
of complex numbers 114–17 simultaneous equations solving
of a matrix 143–4 using calculator 156–8, 242
projectile motion 524–36
using Gaussian elimination 209–13
proof by contradiction 18
using inverse matrices 206
using matrices 155–6
quadratic equations, complex numbers in 80
singular matrices 149–50
quaternions 197–8
singular transformation 267–8
radian measure 95–6 square roots 7
rational numbers 3–4 of negative numbers 76
recurring decimals 11–13 statics 364
rationalising denominators of surds 40–2 subgroups 189–90
using conjugate surds 44–7 successive translations 253–5
Real Number System surds, addition and subtraction 24–5
classification 3–5 conjugate pairs 34
relationships between subsets 5 definition 17–18
real numbers Distributive Law applied to multiplication of 32–4
application and modelling 64–5 division 36–8
investigations 15 multiplication 27–30
modulus 49–50 proof of irrationality 18–19
real part of complex number 77 rationalising when denominator of fraction 40–2,
recurring decimals 11–13 44–7
conversion to fractions 429–30 simplifying 21–2
reflection matrix 276 squaring 29–30
Index 625
Taussky-Todd, Olga 142 definition 300
ternary operations 180 dot product of 324–8
time-varying vectors equality of 301
equation of path 339–41, 343–4 expressions for velocity and acceleration 514–21
using calculator 341–3 finding angle between 327
transformations, congruent 270 in geometry 385–8
definition 250–1 and matrices 309, 337–8
dilations 284–9 multiplication of 324–8
linear 259–68 multiplying by a scalar 302
reflections 276–83 negative of 301
rotations 192–3, 270–5
notation 301
shears 291–5
properties 301
transition matrix 159
relationship with matrices 337–8
translations, of a curve 255
scalar and vector resolutes 332–4
definition 251–3
successive 253–4 three-dimensional non-zero 387
transpose of a matrix 154 time-varying 339–44
triangle of forces 359–60 see also position vectors
velocity
unary operations 180 relative 382–4
unit vectors 312–13, 315–17 vector expressions for 514–21
and dot product 325
weight and mass 364–5
vector (force) diagrams 356–9 whole numbers and operations, axioms relating
vector quantities 300 to 188
vectors
addition of 301–5 zero matrix 133

Anda mungkin juga menyukai